Sei sulla pagina 1di 1092

1.

A 65yo man presents with painless hematuria, IVU is normal, prostate is mildly
enlarged with mild frequency. What is the most appropriate next step?
a. US Abdomen
b. Flexible cystoscopy
c. MRI
d. Nuclear imaging
e. PSA
Q. 1. What is the key?
Q. 2. Points in favour of the key.
Ans. 1. The key is B. Flexible cystoscopy.
Ans. 2. Painless hematuria in an elderly (here 65 years old man) indicates carcinoma
bladder for which flexible cystoscopy is done.
An elderly gentleman complaining of painless hematuria : always exclude bladder cancer
The most important and definite Investigation for bladder cancer is a cystoscopy+ Biopsy.
Initially : Urine microscopy but it does not rule out CA.
Other causes of painless hematuria are rhabdomyolysis , coagulation disorder , prostate cancer , hemolytic
anemia , renal tumor , and polycystic kidney disease
you can exclude those by absence of :
1- History of crush injury for rhabdomyolysis
2- No bleeding from other orifices for coagulation disorder
3- No symptoms of prostatism for Prostate Cancer
4- No signs of anemia
5- No tenderness in loin or masses (renal tumor)
6- No hypertension (in polycystic kidney)
although other investigations like Mid urine sample , IVU , may show UTI , other findings like filling defects ,
etc.. they dont help with diagnosis and prognosis

Diagnosis : Bladder CA. (1 in 10,000)


Most common : Transistional cell CA. 3x in MEN of 50+ age.
Inc factors :
Smoking, schistosomiasis, rubber dye industries, White ppl, recurrent infections.
Symptoms :
Painless hematuria (on and off)
Pain in lower abdomen

Treatment :
TUR with 1 chemotherapy within 24 hours. If needed, BCG is used for next chemo
cycles.
2. A 74yo smoker presented to his GP with cough and SOB. Exam revealed
pigmentation of the oral mucosa and also over the palms and soles. Tests show that he
is diabetic and hypokalemic. What is the most probable dx?
a. Pseudocushing syndrome
b. Conns disease
c. Ectopic ACTH
d. Cushings disease
e. Hypothyroidism
Q. 1. What is the likely key?
Q. 2. Please explain the key.

Ans. 1. The key is C. Ectopic ACTH.


Ans. 2. The patient is smoker and probably developed small cell lung cancer which is
working as a tumour producing ectopic ACTH resulting in pigmentation. Resultant raised
cortisol is leading to diabetes and hypokalemia.
the features can be explained by increased levels of ACTH and adrenocortical hormones. So the question is
: ectopic or pituitary ACTH excess? It seems to be from an ectopic source since the patient is smoker and
has SOB and cough ( Lung tumor whether small cell CA bronchus or carcinoid tumor- both may

secrete ectopic ACTH ) . Furthermore , Cushing's disease is often the result of pituitary
ACTH-secreting adenoma that also causes pressure symptoms like headache and
visual disturbances which are absent in this case.
Ectopic ACTH increase the secretion of aldosterone from adrenal gland and aldosteronism causes
hypernatraemia and hypokalaemia.
Cortisol is a form of stress hormone. So it induces glycogenolysis causing increase in blood glucose.
No pigmentation in conn
Conns must have hypertension n not necessarily hypokalemia but it presents with signs of hypokalemia like
weakness quadriparsis cramps.
Why not Cushings?
SCLC is a direct cause of ectopic ACTH (statement is clear cut - Smoker). Further ectopic acth can
lead to cushings at later stages but its major cause is use of steroids and pituitary adenoma while
ectopic acth is down the list. And if cushings happen, the major indicative symptoms are stria, moon
face, easily fractured bones, plethora.

Diagnosis : Small Cell Lung CA causing ectopic ACTH.


Main reason : SMOKING for years. At Least 20 so age goes up to 50 to 60. Male.
Symptoms :
Persistent cough, hemoptysis, chest n shoulder pains, SOB, clubbing. +/- pleural effusion,
pneumonia, pins and needles in arm n shoulder sensation.
Invs :
Initial : CXR. Shows shadowing
Confirmatory : CT chest THEN Biopsy thru bronchoscope or transthoracic needle biopsy. Depends
on the location.
Pleural Tap can be done if pleural effusion.
Treatment :
Surgery, radio and chemo.
Prognosis : Good if early diagnosed. Bad if late.

3. A 44yo woman has lost weight over 12 months. She has also noticed episodes where
her heart beats rapidly and strongly. She has a regular pulse rate of 90bpm. Her ECG
shows sinus rhythm. What is the most appropriate inv to be done?
a. Thyroid antibodies
b. TFT
c. ECG
d. Echocardiogram
e. Plasma glucose
Q. 1. What is the key?
Q. 2. What is the diagnosis?
Q. 3. What is the significance of episodes of rapid strong heart beat?
Ans. 1. The key is B. TFT.
Ans. 2. Thyrotoxicosis [weight loss over 12 months, episodes of rapid strong heart beet
(thyrotoxicosis induced paroxysmal atrial fibrillation) points towards the diagnosis of
thyrotoxicosis].

Ans. 3. Episodes of rapid strong heart beat indicates thyrotoxicosis induced paroxysmal
atrial fibrillation.

Diagnosis : Hyperthyroidism/thyrotoxicosis.
Gender : more in females. 20 to 50 age.
Most common : Graves'
Symptoms :
Irritable, always on the go, losing weight despite increase appetite, palpitations, heat
intolerance, sweating, Diarrhea, SOB, itch, very light periods, increase risk of AF
and osteoporosis.
Meds that cause it : Amiodarone and lithium.
Invs : TFTS. Low TSh and high T4.
Treatment :
1. Carbimazole for 12 to 18 months.
Pregnancy - Propylthiouracil
2. Radioiodine. Should not be pregnant and conceive for at least 6 months. Father at
least 4 months.
3. Surgery
4. Beta blockers (propranolol, atenolol)
Follow up every year is very imp.
4. 79yo anorexic male complains of thirst and fatigue. He has symptoms of frequency,
urgency
and terminal dribbling. His urea and creatinine levels are high. His serum calcium is 1.9
and he is anemic. His BP is 165/95 mmHg. What is the most probable dx?
a. BPH
b. Prostate carcinoma

c. Chronic pyelonephritis
d. Benign nephrosclerosis
Explanation of Question no. 4:
First to say in this case (almost all features goes in favour of prostatic carcinoma like- frequency,
urgency and terminal dribbling are features of prostatism; Age, anorexia and anaemia favours
carcinoma prostate diagnosis and it would be accurate presentation if it was hypercalcaemia. But
given calcium level is of hypocalcaemic level and it is the main cause of discrepancy of this
question). Renal failure can be an association of malignant disease and can cause high
BP. Thirst is a feature of hypercalcaemia (here may be erroneously calcium level is given in
hypocalcaemic level ; probably a bad recall). Prostate biopsy is the confirmatory diagnosis and
others like PSA is suggestive. This is what I could pointed out. If there is any better explanation
please place it to correct the answer- any one please.

Calcium

2.1-2.6 mmol/l

I think this patient has CKD secondary to prostate CA which leads to hypocalcemia
due to vit D def.
Osteoblastic metastases Occasional patients with widespread osteoblastic metastases, particularly those
with breast or prostate cancer, have hypocalcemia.
Diagnosis : Prostate Cancer
Most common CA in men of uk. 1 in 8 men. After 65.
Risk factors: Fatty diet, exposure to cadmium, ageing n family history.
Symptoms :
Poor stream, hesitancy, dribbling, frequency, urgency, poor emptying.
Invs :
Examine. PSA levels.
Confirmatory test : Biopsy.
Grading : Gleason Score.
4 or less - well differentiated. 10 yr risk of local progression 25%
5 - 7 - moderately differentiated. 50% risk
Over 7 - poorly differentiated. 75% risk
Risk assessment PSA levels.
Low - <10 and gleason score 6 or below
Intermediate - psa 10 to 20 or gleason score 7
High - psa >20 or gleason 8 to 10.
Staging : MRI preferred over CT.
Treatment : Surgery. Radical prostatectomy. S/E impotence, incontinence of urine.
Radiotherapy. External and internal (brachytherapy)
HRT to stop TESTOSTERONE.
Medicines - LHRH. Goserelin, leuprorelin, triptorelin (act on pitutary) and Flutamide, cyproterone
(anti androgenic)
Prognosis : variable. Depends on the stage.
Complications :
UTi, AKI, CKD, sexual dysfunction, metastasis.
Note : Prostate CA has increased risk with HYPERCALCEMIA. Not hypo. So the statement seems to
be wrong. Even BPH has nothing to do with Ca levels.
Benign nephrosclerosis is due to long standing HTN. No link to prostate found.

5. A 64yo man has recently suffered from a MI and is on aspirin, atorvastatin and ramipril. He has
been having trouble sleeping and has been losing weight for the past 4 months. He doesnt feel
like doing anything he used to enjoy and has stopped socializing. He says he gets tired easily and
cant concentrate on anything. What is the most appropriate tx?
a. Lofepramine
b. Dosulepin
c. Citalopram
d. Fluoxetine
e. Phenelzine
Ans. The key is C. Citalopram. [Citalopram is the antidepressant of choice in IHD]
Citalopram is associated with dose-dependent QT interval prolongation and is contraindicated in patients with known QT interval prolongation or congenital long QT
syndrome.
Sertraline can also be used in patients with IHD for depression. sertraline is considered the
drug of choice post-MI
1. Depression with obesity=fluoxetine (It helps without weight loss)
2. Depression with sexual dysfunction=mirtazapine
3. Post stroke depression use nortriptyline (TCA)
4. Depression with obsessive compulsive disorder=clomipramine (TCA)
5. Depression with ischemic heart disease=SSRI e. g citalopram

6. A 67yo man after a stroke, presents with left sided ptosis and constricted pupil. He also has
loss
of pain and temp on the right side of his body and left side of his face. Which part of the brain is
most likely affected?
a. Frontal cortex
b. Cerebellum
c. Pons
d. Medulla
e. Parietal cortex
Q. 1. What is the key?
Q. 2. What is the name of this condition?
Ans. 1. The key is D. Medulla.
Ans. 2. The name of the condition is Lateral medullary syndrome [ipsilateral Horner syndrome
and contralateral loss of pain and temperature sense]
Lateral medullary syndrome, also known as Wallenberg's syndrome, occurs following occlusion of the
posterior inferior cerebellar artery
Cerebellar features
ataxia
nystagmus
Brainstem features

ipsilateral: dysphagia, facial numbness, cranial nerve palsy e.g. Horner's


contralateral: limb sensory loss

Lateral medullary or Wallenberg's syndrome:

Usually from occlusion of the vertebral artery.


Occasionally from occlusion of the posterior inferior cerebellar artery.
Involvement of the vestibular system causes nausea, vomiting and vertigo.
Ipsilateral features:
o Ataxia from cerebellar involvement.
o Horner's syndrome from damage to descending sympathetic
fibres.

o
o
o
o
o
o
o

Reduced corneal reflex from descending spinal tract damage.


Nystagmus.
Hypacusis.
Dysarthria.
Dysphagia.
Paralysis of palate, pharynx, and vocal cord.
Loss of taste in the posterior third of the tongue.
Contralateral findings:
o Loss of pain and temperature sensation in the trunk and limbs
(anterior spinothalamic tract).
o Tachycardia and dyspnoea (cranial nerve X).
o Palatal myoclonus (involuntary jerking of the soft palate,
pharyngeal muscles and diaphragm).

7. A 60yo man presents with dysphagia and pain on swallowing both solids and liquids. A barium
meal shows gross dilatation of the esophagus with a smooth narrowing at the lower end of the
esophagus. What is the SINGLE most likely cause of dysphagia?
a. Achalasia
b. Myasthenia gravis
c. Esophageal carcinoma
d. Esophageal web
e. Systemic sclerosis
Ans. The key is A. Achalasia.
Achalasia typically presents in middle-age and is equally common in men and women
Investigations
manometry: excessive lower oesophageal sphincter tone which doesn't relax on swallowing considered most important diagnostic test
barium swallow shows grossly expanded oesophagus, fluid level, 'bird's beak' appearance. This

is in contrast to the rat's tail appearance of carcinoma of the oesophagus

CXR: wide mediastinum, fluid level


Gold standard - Manometry

Treatment

intra-sphincteric injection of botulinum toxin


Heller cardiomyotomy for fit young patients.
balloon dilation for old unwell patients.

Complications : Aspiration pneumonia, perforation, GERD, Oesophagus CA.

Dysphagia
The table below gives characteristic exam question features for conditions causing
dysphagia:

Dysphagia may be associated with weight loss, anorexia or


vomiting during eating
Oesophageal
cancer

Oesophagitis

Past history may include Barrett's oesophagus, GORD,


excessive smoking or alcohol use

May be history of heartburn


Odynophagia but no weight loss and systemically well

Oesophageal
candidiasis

There may be a history of HIV or other risk factors such as steroid


inhaler use

Achalasia

Dysphagia of both liquids and solids from the start


Heartburn
Regurgitation of food - may lead to cough, aspiration pneumonia etc

Pharyngeal
pouch

More common in older men


Represents a posteromedial herniation between thyropharyngeus
and cricopharyngeus muscles
Usually not seen but if large then a midline lump in the neck that
gurgles on palpation
Typical symptoms are dysphagia, regurgitation, aspiration and
chronic cough. Halitosis may occasionally be seen

Systemic
sclerosis

Other features of CREST syndrome may be present, namely


Calcinosis, Raynaud's phenomenon, oEsophageal dysmotility,
Sclerodactyly, Telangiectasia

As well as oesophageal dysmotility the lower oesophageal sphincter


(LES) pressure is decreased. This contrasts to achalasia where the
LES pressure is increased

Myasthenia
gravis

Other symptoms may include extraocular muscle weakness or ptosis

Dysphagia with liquids as well as solids

Globus hystericus

May be history of anxiety


Symptoms are often intermittent and relieved by swallowing
Usually painless - the presence of pain should warrant further
investigation for organic causes

8. A man undergoes a pneumonectomy. After surgery, invs show hyponatremia. What could be
the
cause of the biochemical change?
a. Removal of hormonally active tumor
b. Excess dextrose
c. Excess colloid
d. Excessive K+
e. Hemodilution
Ans. The key is A. Removal of hormonically active tumour.
Small cell lung carcinoma produces acth like peptide which stimulates aldosterone secretion causing
hypernatremia. Removal of that will lead to hyponatremia.
OHCM 170...Lung tumors may secrete both ACTH and ADH. If it was an ACTH secreting tumor then it's
removal may cause hyponatremia. As ACTH helps in absorption of Na and water by releasing Aldosterone
from adrenal gland. On the other hand if it was an SIADH secreting tumor then opposite would happen.

9. A pregnant lady came with pain in her calf muscle with local rise in temp to the antenatal clinic.
What tx should be started?
a. Aspirin
b. LMWH
c. Paracetamol
d. Cocodamol
e. Aspirin and heparin
Ans. The key is B. LMWH.
During pregnancy :
Start LMWH and continue throughout pregnancy. Stop the injections 24 hours before labour and
then restart them 4 hours post op. Warfarin is Contraindicated in pregnancy.
If NO pregnancy :
the protocol is different.
- LMWH stat
- Start Warfarin within 24 hours
- Monitor INR and withdraw LMWH when value is 2.0
- Depending on provoked or non provoked, give Warfarin for 3 and 6 months respectively and then
stop.
- IVC filter is used when anticoagulants fail
- Compression stockings to all patients to prevent 'Post-phlebitic limb changes'

Wells' diagnostic algorithm[1]


Score one point for each of the following:
Active cancer (treatment ongoing or within the previous six months, or
palliative).
Paralysis, paresis or recent plaster immobilisation of the legs.
Recently bedridden for three days or more, or major surgery within the
previous 12 weeks, requiring general or regional anaesthesia.
Localised tenderness along the distribution of the deep venous system
(such as the back of the calf).
Entire leg is swollen.
Calf swelling by more than 3 cm compared with the asymptomatic leg
(measured 10 cm below the tibial tuberosity).
Pitting oedema confined to the symptomatic leg.
Collateral superficial veins (non-varicose).
Previously documented DVT.
Subtract two points if an alternative cause is considered at least as likely as DVT.
The risk of DVT is likely if the score is two or more, and unlikely if the score is one or
less.
invs :
initial - Duplex USG
Gold standard - invasive venography

10. A 53yo female presents with an acute painful hot knee joint. She is a known case of
RA. On
examination, the knee is red, tender and swollen. The hamstring muscles are in spasm.
Her
temp is 38.5C and BP is 120/80mmHg. What is the SINGLE best next inv?
a. Joint aspiration for cytology and culture and sensitivity
b. Joint aspiration for positive birefrengent crystals
c. Joint aspiration for negative birefrengent crystals
d. Blood culture
e. Serum uric acid
Q. 1. What is the likely key here?
Q. 2. Is there any link in septic arthritis and rheumatoid arthritis?

Q. 3. What is the likely organism in this age group?


Q. 4. What is the likely organism in younger age group?
Ans. 1. A. Joint aspiration for cytology and culture and sensitivity.
Ans. 2. Any chronically arthritic joint is predisposed to infection.
Moreover chronic use of steroid in Rh. arthritis is one of the important
predisposing factor.
Ans. 3. Staphylococcus
Ans. 4. Neisseria gonorrhoeae

RA always involves bilateral symmetrical joints with morning stiffness. The patient presented with
new complaint which is monoarticular, swollen n hot. It's clearly Septic arthritis n u do joint
aspiration. Chronic use of steroids is one of the important predisposing factors.

Diagnosis : Septic Arthritis due to persistent Rheumatoid Arthritis.


The classic picture is a single swollen joint with pain on active or passive movement.
It is more common in patients with prior joint damage, as in gout, rheumatoid arthritis
and systemic connective tissue disorders.
Fever and rigors. Chest wall pains.

Treatment :
Flucloxacillin and for MRSA - Vancomycin.
Penicillin with Gentamicin is being used as well.
11. An 80yo man presented with pain in his lower back and hip. He also complains of
waking up in
the night to go to the washroom and has urgency as well as dribbling. What is the most
likely dx?
a. BPH
b. Prostatitis
c. UTI
d. Prostate carcinoma
e. Bladder carcinoma
Q. 1. What is the likely key?
Q. 2. What are the points in favour of your diagnosis?
Q. 3. What are the investigations?
Q. 4. What are the treatment options for carcinoma prostate?

DISCUSSED IN MCQ 4.
Ans. 1. D. Prostate carcinoma.
Ans. 2. Age, nocturia, urgency and dribbling points towards prostate pathology. Pain of
lower back and hip points towards bony metastases from prostate cancer.
Ans. 3. Blood test for PSA; Prostate biopsy; MRI [if initial biopsy is negative, to decide
repeat biopsy]. Source NICE.
Ans. 4. Treatment options: 1. Active treatment [i) radical prostatectomy ii) radical
radiotherapy iii) hormone therapy iv) brachytherapy v) pelvic radiotherapy vi)
orchidectomy]
2. Active surveillance
3. Watchful waiting
4. Palliative care [Source: NICE].

12. An 18yo female has periorbital blisters. Some of them are crusted, others secreting
pinkish fluid. What is the most likely dx?
a. Shingles
b. Chicken pox
c. Varicella
d. Rubella
e. Measles
Q.1. What is the likely key?
Q. 2. Which nerve is involved here?
Q. 3. Is this disease unilateral or bilateral?

Ans. 1. A. Shingles
Ans. 2. Ophthalmic division of trigeminal nerve.
Ans. 3. Typically shingles is unilateral.
Short note everywhere. Mcq covers it up.
Treatment :
Refer to ophthalmologist. Ocular lubricants, cool compressors, topical steroids,
Botulinum toxin injection if neurotrophic ulcers form.
13. A 29yo lady who is a bank manager is referred by the GP to the medical OPC due to
a long hx of tiredness and pain in the joints. An autoimmune screen result showed
smooth muscle
antibodies positive. What is the most appropriate next inv?
a. ECG
b. TFT
c. LFT
d. Serum glucose
e. Jejunal biopsy
Q. 1. What is the likely key?
Q. 2. What is the diagnosis?
Q. 3. What is the definitive investigation?
Q. 4. What is the treatment?
Ans. 1. C. LFT
Ans. 2. Autoimmune hepatitis.
Ans. 3. Definitive investigation is liver biopsy
Ans. 4. Steroid [start with high dose prednisolone]. Azathioprine is commonly added with
steroid to reduce its dose as steroid has more side effects than azathioprine.
Diagnosis : Autoimmune hepatitis.
Symptoms :
Tiredness, fatigue, mild pruritus, amenorrhea, pleuritis, abdominal discomfort,
Oedema, Skin rashes, acne, weight loss. Nausea is prominent.
Signs :
Hepatomegaly, splenomegaly, spider angiomata, ascites, encephalopathy,
jaundice in 50%
The autoantibodies present include antinuclear antibody (ANA), anti-smooth
muscle antibody (ASMA), anti-liver-kidney microsomal-1 (anti-LKM-1) antibody,
antibodies against soluble liver antigen (anti-SLA), antimitochondrial antibody
(AMA) and antiphospholipid antibodies.
Invs :
1. Autoantibodies. Typical is ASMA.
2. Ig G is raised. (Can lead to hyperviscosity syndrome)
3. LFTs. ALT and AST are raised. ALP maybe normal.
4. USG abdomen.
5. Liver biopsy - Confirmatory.
Treatment :
Steroids.

Budesonide + Azathioprine
Prednisolone + Azathioprine
+/- methotrexate, Anti TNF alpha drugs
Liver transplant.
14. A 5yo with recurrent chest pain, finger clubbing with offensive stool. Choose the
single most
likely inv?
a. Endomysial/Alpha gliadin antibody
b. Sweat test
c. Barium meal
d. ECG
e. Glucose tolerance test
Q. 1. What is the likely key?
Q. 2. What is the diagnosis?
Q. 3. What is the mode of inheritance?
Ans. 1. B.
Ans. 2. Cystic fibrosis
Ans.3. Autosomal recessive.
DIAGNOSIS : Cystic FIbrosis. Mutation of CFTR on chromosome 7.
It cause dehydration. Hence, bronchiectasis, bowel obstruction and bacterial
growth,

Signs

Finger clubbing.
Cough with purulent sputum.
Crackles.
Wheezes (mainly in the upper lobes).

INVS :
Sweat Test. Chloride concentration > 60. Sinus X ray or CT scan - Opacities of
sinuses.
Lung function tests, LFTs, Sputum microbiology.

17. A man with suspected active TB wants to be treated at home. What should be done
to prevent
the spread of disease?
a. Immediate start of the tx with Anti-TB drugs
b. All family members should be immediately vaccinated with BCG vaccine
c. Patient should be isolated in a negative pressure chamber in his house
d. Universal prevention application protocol
Ans. d. Universal prevention application protocol.
This protocol isnt available anywhere on the internet. Everybody is suggesting D on the
basis of exclusion.

18. A 7yo child is brought to the ED with a 1 day hx of being listless. On examination, the
child is
drowsy with an extensive non-blanching rash. What advice would you give the parents?
a. All family members need antibiotic therapy
b. Only the mother should be given rifampicin prophylaxis
c. All family members need isolation
d. All family members should be given rifampicin prophylaxis
Q. 1. What is the likely key?
Q. 2 What is the probable diagnosis?
Q. 3. What is the diagnostic investigation?
Q. 4. What is the initial management?
Ans. 1. D. All family member should be given rifampicin prophylaxis
Ans. 2. Meningococcal disease.
Ans. 3. Blood or CCF PCR
Ans. 4. Prehospital management: Benzyl penicillin or cefotaxime.
DIAGNOSIS : Meningococcal infection. It can be meningococcal Septicemia which
is without Meningitis, or can be Meningococcal Meningitis. This Mcq doesnt have
meningitis signs so we will stick to M.Septicemia.
Caused by N.Meningitidis
Signs and Symptoms :
Most common and important - Non - blanching rash
Fever, headache
May have : Stiff neck, back rigidity, bulging fontanelle (in infants), photophobia.
Altered mental state, unconsciousness, toxic/moribund state, Kernig's sign (pain and
resistance on passive knee extension with hips fully flexed) and Brudziski's sign
(hips flex on bending the head forward)
Pre-Hospital Management :
Call 999 and give Benzyl penicillin or Cefotaxime

INVESTIGATIONS :
Blood cultures.
FBC (WCC), CRP, U&Es, renal function tests, LFTs.
Blood test for polymerase chain reaction (PCR): perform whole blood realtime PCR testing - (EDTA sample) - for N. meningitidis to confirm a
diagnosis of meningococcal disease
Investigations for disseminated intravascular coagulation: prothrombin
time is elevated, activated partial thromboplastin time (aPTT) is elevated,
platelet count is reduced and the fibrinogen level is low.
Lumbar puncture - once the patient is stable
Aspirate from other sterile sites suspected of being infected (eg, joints) for
microscopy, culture and PCR.
TREATMENT :

Choice of antibiotics in hospital :


o Ceftriaxone is usually given to those over 3 months
o Cefotaxime and amoxicillin are usually given to those under 3
months.

Vancomycin is given in addition, to those who have recently


travelled outside the UK or have had prolonged or multiple
exposure to antibiotics.
CHEMOPROPHYLAXIS :
To close contacts of cases, irrespective of vaccination status - for example,
those who have had prolonged close contact with the case in a household-type
setting during the seven days before onset of illness
Ciprofloxacin and rifampicin are both recommended by Public Health England
(PHE) but ciprofloxacin is the preferred choice for most individuals.
Ciprofloxacin can be used in all ages and in pregnancy; it is easily available in
a single dose and does not interfere with oral contraceptives (but is contraindicated if there has been previous sensitivity):
o Adults and children aged >12 years - 500 mg orally stat.
o Children aged 5-12 years - 250 mg orally stat.
o Children aged <5 years - 30 mg/kg up to a maximum of 125 mg orally
stat.
Rifampicin is no longer the drug of choice as, although it is licensed for
chemoprophylaxis, it has several disadvantages including important drug
interactions

19. A 47yo man has a temp of 39C and is delirious. He has developed blisters mainly on
his trunk,
which appeared a few hours ago. He is well and not on any medications. He last
travelled 5
months ago to Italy. Which of the following is the most likely dx?
a. Shingles
b. Chicken pox
c. Pemphigoid
d. Bullous pemphigus
Q 1. What is the likely key?
Q. 2. Why delirium in this patient?
Q. Is his travel history has any link to the development of this disease?
Ans. 1. B. Chicken pox
Ans. 2. Adults more commonly develop a more generalized brain inflammation
("encephalitis") whose symptoms may include delirium and seizures.
Ans. 3. Incubation period of chicken-pox is 10-21 days. So this travel history is not
significant.
DIAGNOSIS : Chicken POX.
Why not Shingles?
Shingles is painful, while mcq does not mention it.
Shingles have dermatomal distribution while chickenpox is mainly peripheral and
truncal in distribution.
Chicken pox has 10 - 21 days incubation period, and so does shingles, so this 5
months thing is wrongly stated.
The patient is delirious - suggesting ENCEPHALITIS which is the main
complication of C.pox.
INVS :

Scraping the blisters and the skin and checking it with immunohistochemical
and PCR.
CXR and LP for its complications. (Pneumonia and Encephalitis)

Management
Chickenpox in an otherwise healthy individual

Simple advice regarding adequate fluid intake, minimising scratching if


possible and that the first 1-2 days they are most infectious.
Symptomatic treatment - eg, analgesia and antipyretics such as
paracetamol. There is a possible association with non-steroidal antiinflammatory drugs (NSAIDs) and risk of necrotising soft tissue infections.
Pruritus can be helped by sedating antihistamines and emollients.
Calamine lotion is no longer recommended, as when it dries it ceases to
be effective. Secondary infection may require antibiotics.
Aciclovir should be considered if the patient presents within 24 hours
Anyone with encephalitis - should be admitted to hospital.

20. A 64yo pt has been having freq episodes of secretory diarrhea, which is extremely
watery, with large amts of mucus. A dx of villous adenoma was made after endoscopy.
What electrolyte
abnormality is most likely in this pt?
a. Hyperkalemia
b. Hypernatremia
c. Hyponatremia
d. Hypokalemia
e. Hypercalcemia
Q. 1. What is the likely key?
Q. 2. Why this occurs?
Ans. 1. D
Ans. 2. There is active potassium secretion causing high fecal potassium concentration
leading to this hypokalaemia in secretory diarrhea. There is also reduced potassium
absorption in diarrhea.
Electrolyte loss in diarrhoea (Na, K, Mg, Cl). But potassium loss is more marked especially in severe,
chronic and when associated with mucous loss. Villous adenomas secrete protein n potassium rich mucus
so hypoalbuminemia n hypokalemia

Hypokalaemia is usually defined as a serum concentration of potassium <3.5 mmol/L.


It can be classified as follows:

Mild - 3.1 - 3.5 mmol/L


Moderate - 2.5 - 3.0 mmol/L
Severe - <2.5 mmol/L

Typical ECG findings when potassium is <3.0 mmol/L:

Flat T waves
ST depression
Prominent U waves

NB: the QT interval may appear prolonged, but this is usually a pseudo-prolongation
as the flattened T waves merge into the U waves.
MAIN CAUSES OF HYPOKALEMIA.
KIDNEY causes :

Thiazide or loop diuretics (the most common cause)


Renal tubular acidosis
Hypomagnesaemia
Hyperaldosteronism - eg, Conn's syndrome, renal artery stenosis, Cushing's disease
Tubulo-interstitial renal disease due to Sjgren's syndrome or systemic lupus
erythematosus
Excess liquorice ingestion
Activation of the renin-angiotensin system - eg, Bartter's syndrome or Gitelman's
syndrome

Via the GI tract:

Diarrhoea
Vomiting (bicarbonate diuresis)
Intestinal fistulae
Villous adenoma
Pyloric stenosis
Laxative abuse
Bowel preparation with oral sodium phosphate solution

Via the skin:

Burns
Increased sweating - eg, exercising in a hot climate
Increased loss in sweat - eg, cystic fibrosis

21. A pt with an acute gout attack came to the ED. What drug should be given to relieve
symptoms?
a. NSAIDs
b. Allopurinol
c. Ibuprofen
Q. 1. What is the key?
Q. 2. What is the acute management of gout?
Ans. 1. A
Ans. 2. Oral NSAIDs commenced immediately and continue for 1 2 weeks; Colchicine
can be effective alternative but is slower to work than NSAIDs. Intra articular
corticosteroids are highly effective in acute gouty monoarthritis.

DIAGNOSIS : GOUT
Cause : Interleukin - 1

Risk factors

Male sex, Meat, Seafood, Alcohol (10 or more grams per day), Diuretics,
Obesity, Hypertension, Coronary heart disease, Diabetes mellitus, Chronic
renal failure, High triglycerides

Pharmacological therapeutic options include:

Non-steroidal anti-inflammatory drugs (NSAIDs)


Colchicine
Corticosteroids

NOTE : This MCQ has NSAIDS and Ibuprofen in options which is from the same
group
but the preferred NSAIDS are : Diclofenac, naproxen and indomethacin.
Colchicine is particularly appropriate when NSAIDs are poorly tolerated, in patients
with heart failure and in those who are on anticoagulants.
30-35 mg of prednisolone reported a low incidence of side-effects
Allopurinol should never be started during an acute attack. Wait for 1-2
weeks after the attack resolves.
Co-prescribe colchicine or a low dose non-steroidal anti-inflammatory drug
(NSAID) to prevent an attack of gout whilst initiating therapy, and continue
until after hyperuricaemia has settled (usually a total of three months).
If an acute attack develops during treatment, maintain the dose of allopurinol
but add colchicine or NSAIDs.

22. A pt was lying down on the operating table in a position with his arms hanging down
for 3 hours. Soon after he woke up, he complains of numbness and weakness in that
hand and has limited wrist movement/wrist drop and sensory loss over dorsum of that
hand, weakness of extension of the fingers and loss of sensation at the web of the
thumb. What structure is likely to be damaged?
a. Radial nerve
b. Median nerve
c. Ulnar nerve
d. Axillary nerve
e. Suprascapular nerve
ANS is A. Radial Nerve.
Radial nerve, in spiral grove on humerus is pressed and damaged. It is also called saturday night
palsy.

23. A pt who was previously on 120mg slow release oral morphine has had his dose
increased to

200mg. He is still in significant pain. He complains of drowsiness and constipation. What


is the
next step in the management?
a. Increase slow release morphine dose
b. Fentanyl patch
c. Replace morphine with oral hydromorphone
d. Replace morphine with oxycodone
e. Subcutaneous morphine
Q. 1. What is the likely key? D - Replace morphine with oxycodone.
Q. 2. Why not other options? X

Morphine Start with oral solution 510mg/4h PO with an equal breakthrough dose as
often as required. A double dose at bedtime can enable a good nights sleep. Patient
needs will vary greatly and there is no maximum dose; aim to control symptoms with
minimum side-effects. If not effective, increase doses in 3050% increments
(5mg10mg20mg30mg45mg). Change to modified release preparations (eg MST
Continus 12h) once daily needs are known by totalling 24h use and dividing by 2.
Prescribe 1/6th of the total daily dose as oral solution for breakthrough pain. Side
effects (common) are drowsiness, nausea/vomiting, constipation and dry mouth.
Hallucinations and myoclonic jerks are signs of toxicity and should prompt dose review.
If the oral route is unavailable try morphine/diamorphine IV/SC. If difficulty tolerating
morphine/diamorphine, try oxycodone PO/IV/SC/PR, starting at an equivalent dose. It is
as effective as morphine and is a useful 2nd-line opioid with a different range of receptor
activity. There are also fentanyl transdermal patches which should usually be started
under
specialist supervision (after opioid dose requirements have been established). Remove
after 72h, and place a new patch at a different site. 45mg oral morphine/24h is
approximately equivalent to a 12mcg/h fentanyl patch.

24. A 40yo woman notices increasing lower abdominal distention with little/no pain. On
examination, a lobulated cystic mass is felt and it seems to be arising from the pelvis.
What is
the most appropriate inv?
a. CA 125
b. CA 153
c. CA 199
d. CEA
e. AFP
Q. 1. What is the likely key?
Q. 2. What is the likely diagnosis?
Ans. 1. A
Ans. 2. Ovarian ca. X
DIAGNOSIS : Ovarian CA
Age - 60 to 80 women
SYMPTOMS :

Early symptoms are often vague, such as abdominal discomfort,


abdominal distension or bloating, urinary frequency or dyspepsia.
Constitutional symptoms include fatigue, weight loss, anorexia and
depression.
It most commonly presents with a pelvic or abdominal mass that may be
associated with pain. Abdominal, pelvic or back pain is usually a late sign
It may cause abnormal uterine bleeding.
Often associated with ascites. One third of patients with ascites also have
a pleural effusion.

INVESTIGATIONS :

CA-125 test.
If this is reported as raised (35 IU/mL or greater) arrange pelvic and
abdominal ultrasound scans.
CT is the investigation of choice in the UK

Treatment :
The standard comprehensive surgical staging approach consists of a total abdominal
hysterectomy and bilateral salpingo-oophorectomy (TAH and BSO)
The standard regime is paclitaxel and carboplatin given intravenously every three
weeks for six cycles.
Radiotherapy.
PRGNOSIS :

Stage I: 92%
Stage II: 55%
Stage III: 21.9%
Stage IV: 5.6%

25. A resident of a nursing home presented with rashes in his finger webs and also on
his abdomen, with complaints of itching which is severe at night. He was dx with
scabies. What the best tx for
his condition?
a. 0.5% permethrin
b. Doxycycline
c. 5% permethrin
d. Reassure
e. Acyclovir
Q. 1. What is the likely key?
Q. 2. Will you consider any other treatment beside this?
Ans. 1. C

Ans. 2. Scabies outbreaks in nursing homes and cases of crusted scabies may require
combination therapy consisting of topical application of permethrin and 2 oral doses of
ivermectin at 200 mcg/kg (administered 1 wk apart). X
DIAGNOSIS : SCABIES

signs and symptoms develop after 3-4 weeks. Symptoms reappear within 1-3
days if the person is re-infested due to prior sensitisation.
The most common presenting symptom is widespread itching. This is usually
worse at night and when the person is warm. A history of several family
members all suffering with itch is strongly suggestive of scabies. Scratching
predisposes to secondary bacterial infection.
Lesions may be papules, vesicles, pustules, and nodules. Erythematous papular
or vesicular lesions are usually seen in the sites of the burrows. The more
widespread, symmetrical, itchy, papular eruption is not in the areas of burrows or
obvious mite activity. This is most commonly seen around the axillae, the periareolar region of the breasts in women, and the abdomen, buttocks, and thighs.
The 'wake' sign is specific for scabies, can be seen with the naked eye and
points towards the location of the mite.
Hyperkeratotic crusted lesions called as Crusted Norwegian Scabies

INVS :
Ink Burrow Test.
Diagnosis can be confirmed by taking a skin scraping from an affected area.
TREATMENT :
First line : Permethrin 5%
2nd line : malathion 0.5% aqueous liquid

26. A 34yo alcoholic is found passed out in front of a local pub. The ambulance crew
informs you
that he was sweating when they found him and there were cans of cider lying empty
around
him. What is the initial stage of inv?
a. Capillary blood sugar
b. CT head
c. MRI head
d. ABG
e. MCV
key: A
Hypoglycemia: blood glucose <3.0 mmol/L
alcohol exerts hypoglycemic effects through inhibiting both gluconeogenesis and glycogenolysis , especially
when the drinker already has starvation or adrenocortical insufficiency.
Here we need to find the causes of passed out..first thing first.glucose for hypo...ABG can help to find
acidosis which is likely to find in alcohol poisoning..MCV it will b increased in alcoholic but we are more likely
to look for anaemia in this as there may b H&M which could lead pt to pass out..CT head for SDH.

Risk factors

Tight glycaemic control.


Malabsorption.
Injection into lipohypertrophy sites.
Alcohol.
Insulin prescription error (notable in hospitalised patients).
Long duration of diabetes.
Renal dialysis.
Drug interactions between hypoglycaemic agents - eg, quinine, selective serotonin reuptake
inhibitors (SSRIs).
Impaired renal function.
Lack of anti-insulin hormone function - eg, Addison's disease, hypothyroidism.

Initially

Glucose 10-20 g is given by mouth, either in liquid form or as granulated sugar (two
teaspoons) or sugar lumps
Repeat capillary blood glucose after 10-15 minutes; if the patient is still hypoglycaemic then
the above can be repeated (probably up to 1-3 times).

If hypoglycaemia causes unconsciousness, or the patient is unco-operative

75-80 ml 20% glucose or 150-160 ml of 10% glucose (the volume will be determined by the
clinical scenario).
25 ml of 50% glucose concentration is viscous, making it more irritant and more difficult to
administer, and is rarely used now.

Once the patient regains consciousness, oral glucose should be administered, as above.

If the patient is at home, or intravenous (IV) access cannot be rapidly


established

Glucagon 1 mg should be given by intramuscular (IM), or subcutaneous (SC) injection.


This dose is used in insulin-induced hypoglycaemia (by SC, IM, or IV injection), in adults and
in children over 8 years (or body weight over 25 kg). NB: 1 unit of glucagon = 1 mg of
glucagon.

The patient must be admitted to hospital if hypoglycaemia is caused by an oral antidiabetic


drug, because the hypoglycaemic effects of these drugs may persist for 12-24 hours and
ongoing glucose infusion or other therapies such as octreotide (see under 'Hypoglycaemia
which causes unconsciousness or fitting is an emergency', below) may be required.

Glucagon
Glucagon can have variable absorption, as it is given SC or IM. It has a relatively slow onset of action and
relies on glycogen stores. Therefore, it may not be effective in cachectic patients, those with liver disease,
and in young children. It is contra-indicated in insulinoma and phaeochromocytoma. It also causes
more insulin to be released and creates the potential for secondary rebound hypoglycaemia.

27. A young boy fell on his outstretched hand and has presented with pain around the
elbow. He
has absent radial pulse on the affected hand. What is the most likely dx?
a. Dislocated elbow

b. Angulated supracondylar fx
c. Undisplaced fx of radial head
d. Posterior dislocation of shoulder
Q. 1. What is the key?
Q. 2. What is the cause of absent radial pulse?
Q. 3. What is the immediate management?
Ans. 1. b.
Ans. 2. Damage or occlusion of the brachial artery is the cause of absent radial pulse.
Ans. 3. Open reduction to fix the occluded artery.
This scenario is classic for supracondylar fx,the distal humerus is displaced posteriorly causes vascular and
nerve injuries which if not addressed properly leads to volkmann contracture
posterior dislocation can compromise vascular supply but falling on outstretched hand causes it unlikely.
Other causes of pulseless radial artery except the two above ?
congenital absence,embolization, cervical rib, thoracic outlet syndrome
falling on an outstretched hands can lead to different type of fractures , like frac of clavicle , radius ,
supracondylar etc. but here it says that radial pulse is not felt ,which is a dreaded complication of
supracondylar frac

Mechanism of injury in elbow fractures and dislocation

Radial head and neck


fractures

Fall on to an outstretched hand

Olecranon fractures

Elderly - indirect trauma by pull of triceps and


brachioradialis
Children - direct blow to elbow

Fractures of the coronoid


process

Fall on to an extended elbow as for elbow dislocation

Fractures of the distal


humerus

Fall on to an extended outstretched hand

Intercondylar fractures

Direct or indirect blow to elbow

Condylar fractures

Direct blow to a flexed elbow

Capitellum fracture

Fall on to an outstretched hand, or direct trauma

Elbow dislocation

Fall on to an extended elbow


Common in sport in the young

28. A 65yo woman presented with transient arm and leg weakness as well as a sudden
loss of vision in the left eye. Her symptoms resolved within the next couple of hours.
What is the most
appropriate next inv?
a. CT brain
b. Echo
c. Doppler USG
d. Arteriography
e. 24h ECG
Q. 1. What is the key?
Q. 2. What is the likely diagnosis?
Q. 3. What will be seen on dopplar USG?
Q. 4. What is the management?
Ans. 1. The key is c.
Ans. 2. TIA
Ans. 3. Carotid artery narrowing
Ans. 4. Aspirin 300 mg daily for 2 weeks then aspirin 75mg daily and modified released
dipiridamol 200mg 12 hourly.
unilateral blindness is typical sign for carotid stenosis

The duration is no more than 24 hours in tia

The most common source of emboli is the carotids, usually at the bifurcation.
People who have had a suspected TIA who need brain imaging (ie the vascular territory or
pathology is uncertain) should undergo diffusion-weighted MRI except where contra-indicated, in
which case CT should be used.
People who have a suspected TIA at high risk of stroke (eg, an ABCD2 score of 4 or above - see
below) in whom vascular territory or pathology is uncertain should undergo urgent brain imaging
(preferably diffusion-weighted MRI).
People with a suspected TIA at low risk of stroke (eg, an ABCD2 score of less than 4) in whom
vascular territory or pathology is uncertain should undergo brain imaging within one week of onset
of symptoms (preferably diffusion-weighted MRI).

Driving
Group 1 (car or motorcycle)

Must not drive for one month.


No need to notify DVLA after a single TIA.
Multiple TIAs over a short period: require three months free from further attacks before
resuming driving and DVLA should be notified.

Group 2 (lorry or bus)

Licence refused or revoked for one year following a stroke or TIA.

Assessment of the risk of stroke


An ABCD2 score of more than 4 suggests high risk of an early stroke.

Scoring System for Risk of Stroke after TIA (ABCD2 Score)

Age

Age >60

Blood pressure

BP>140 systolic and/or >90 diastolic

Clinical features

Unilateral weakness

Speech disturbance without weakness

Other

>60 minutes

10-59 minutes

<10 minutes

Duration of symptoms

Diabetes

Presence of diabetes

RCP recommendations
Patients with suspected TIA who are at high risk of stroke (eg, an ABCD2 score of 4 or above)
should receive:
o Aspirin or clopidogrel (each as a 300 mg loading dose and then 75 mg daily)
and a statin started immediately.
o NB: clopidogrel is not licensed for the management of TIA and therefore the
National Institute for Health and Care Excellence (NICE) and the British
National Formulary (BNF) recommend aspirin plus modified-release
dipyridamole.
o Specialist assessment and investigation within 24 hours of onset of symptoms.
People with crescendo TIA (two or more TIAs in a week), atrial fibrillation or those on
anticoagulants should be treated as being at high risk of stroke even if they may have an
ABCD2 score of 3 or below.
Patients with suspected TIA who are at low risk of stroke (eg, an ABCD2 score of 3 or below)
should receive:
o Aspirin or clopidogrel (each as a 300 mg loading dose and then 75 mg daily)
and a statin.
o NB: clopidogrel is not licensed for the management of TIA and therefore NICE
and the BNF recommend aspirin plus modified-release dipyridamole.
Patients with TIA in atrial fibrillation should be anticoagulated in the TIA clinic once intracranial
bleeding has been excluded and if there are no other contra-indications.

29. A man complains of loss of sensation in his little and ring finger. Which nerve is most
likely to be involved?
a. Median nerve
b. Ulnar nerve
c. Radial nerve
d. Long thoracic nerve
e. Axillary nerve
Ans. b.

30. A young man complains of double vision on seeing to the right. Which nerve is most
likely to be involved?
a. Left abducens
b. Right abducens
c. Left trochlear
d. Right trochlear
e. Right oculomotor
Ans. b.

31. A 45yo man keeps having intrusive thoughts about having dirt under the bed. He
cant keep
himself from thinking about these thoughts. If he tries to resist, he starts having
palpitations.
What is the most likely dx?
a. OC personality
b. OCD
c. Schizophrenia
d. Panic disorder
e. Phobia
Q. 1. What is the key?
Q. 2. What is the managemment?
Ans. 1. b.
Ans. 2. CBT 1st line. SSRIs.
intrusive thoughts and anxiety when trying to fight them mean OCD.

32. A 33yo man presents with an itchy scaly annular rash on his thigh after a walk in the
park. Which of the following drugs will treat his condition?
a. Erythromycin

b. Doxycycline
c. Penicillin
d. Amoxicillin
Q. 1. What is the key?
Q. 2. What is the diagnosis?
Ans. 1. b.
Ans. 2. Lyme disease.
Lyme disease is caused by the spirochaete Borrelia burgdorferi and is spread by ticks
Features

early: erythema chronicum migrans + systemic features (fever, arthralgia)


CVS: heart block, myocarditis
neuro: cranial nerve palsies, meningitis

Investigation
serology: antibodies to Borrelia burgdorferi
Management
doxycycline if early disease. Amoxicillin is an alternative if doxycycline is contraindicated (e.g.
pregnancy)
ceftriaxone if disseminated disease
Jarisch-Herxheimer reaction is sometimes seen after initiating therapy: fever, rash, tachycardia
after first dose of antibiotic (more commonly seen in syphilis, another spirochaetal disease)

33. A pt with cerebral mets has polyuria and polydipsia. What part of the brain would be
affected?
a. Cerebral cortex
b. Cerebellum
c. Diencephalon
d. Pons
e. Medulla
Q. 1. What is the key?
Q. 2. What is the diagnosis?
Ans. 1. c.
Ans. 2. Cranial diabetes insipidus.
diencephalon contains hypothalamus centre for thirst.
Other causes of polyuria and thirst.

Psychogenic or primary polydipsia (PP).


Diabetes mellitus.
Other osmotic diureses - eg, hypercalcaemia.
Diuretic abuse.

Investigation
high plasma osmolality, low urine osmolality
water deprivation test

34. A 32yo man presented with painless hematuria. He is hypertensive but the rest of
the exam is
unremarkable. What is the most likely dx?
a. Polycystic kidneys
b. Ca bladder
c. Ca prostate
d. TTP
e. HUS
Q. 1. What is the key?
Q. 2. What are the points to justify your answer?
Q. What is the investigation of choice?
Ans. 1. A.
Ans. 2. Painless haematuria at an younger age with hypertension.
Ans. 3. Renal ultrasound.

autosomal dominant
Gross haematuria following trauma is a classic presenting feature of ADPKD,Advise
against participating in contact sports which risk abdominal trauma
polycystic kidneys can produce excess erythropoietin and hence raise Hb
Angiotensin-converting enzyme (ACE) inhibitors or angiotensin-II receptor
antagonists are the preferred choice

35. A 45yo female complains of pain in the inner side of her right thigh. She was dx with
benign
ovarian mass on the right. Which nerve is responsible for this pain?
a. Femoral nerve
b. Obturator nerve
c. Iliohypogastric nerve
d. Ovarian branch of splanchic nerve
e. Pudendal nerve
Ans. B. [The Obturator nerve is responsible for the sensory innervation of the skin of the
medial aspect of the thigh].

36. A 37yo lady strongly believes that a famous politician has been sending her flowers
every day
and is in love with her. However, this is not the case. What is the most likely dx?
a. Erotomania
b. Pyromania
c. Kleptomania
d. Trichotillomania
e. Grandiosity
Ans. 1. A. [Erotomania is a type of delusion in which the affected person believes that
another person, usually a stranger, high-status or famous person, is in love with them].
Pyromania fail to resist impulses to deliberately start fires, in order to relieve tension or
for instant gratification.
Kleptomania is the inability to refrain from the urge to steal items.
Trichotillomania compulsive urge to pull out one's hair, leading to noticeable hair loss
and balding.
Grandiosity refers to an unrealistic sense of superiority.
37. A 3yo child has been brought with facial lacerations. On examination he has some
cuts over his right cheek and under the eye. The GCS on initial evaluation is 15. What is
the appropriate next inv?
a. Skull XR
b. Facial XR
c. CT scan
d. MRI
e. Observation
Ans. b.
If u have cuts over ur cheek..there is high chance that there can be a fracture of a facial bone..hence by
observing u dont want to wait for a hematoma to form then go for a facial xr Skull xray isnt of any value as
the gcs is 15!

38. A 73yo woman has lymphadenopathy and splenomegaly. She feels well but has had
recurrent
chest infections recently. Choose the single most likely blood film findings?
a. Atypical lymphocytes
b. Excess of mature lymphocytes
c. Plasma cells
d. Multiple immature granulocytes with blast cells
e. Numerous blast cells
Q. 1. What is the key?
Q. 2. What is the diagnosis?
Q. 3. Points in favour of your answer?
Ans. 1. B.
Ans. 2. CLL
Ans. 3. Age of patient (usually above 50 yrs), lymphadenopathy and splenomegaly,
appearance of lymphocytes (mature lymphocytes but functionally not normal).
Repeated chest infection points towards abnormal function of lymphocytes against
infection.

It's CLL because in CML the risk age is 40-60 years, and in CBC there should be increased myeloid
cells(which is absent in options). If I exclude these factors, then the possible DX would be CLL and film
finding is B.
1. Acute lymphoblastic leukemia- abnormal immature lymphocytes, (can be immature B or T lymphocytes)
called lymphoblasts. 2. Chronic lymphocytic leuaemia- Excess of mature lymphocytes. 3. Acute myeloid
leukaemia- blast cells (abnormal immature white cells) derived from myeloid stem cells. 4. Chronic myeloid
leukaemia- near normal granulocytes developed from abnormal stem cells (these are mature cells).
Also age is a factor ALL in any age but common in child, AML- age over 50, CLL common over age 60,
CML- in adults and commoner with increasing age.

39. A lady presents with itching around the breast and greenish foul smelling discharge
from the
nipple. She had a similar episode before. What is the most likely dx?
a. Duct papilloma
b. Duct ectasia
c. Breast abscess
d. Periductal mastitis
e. Mammary duct fistula
Q. 1. What is the key?
Q. 2. What other options (breast conditions) frequently come in plab mcq?
Ans. 1. Key is b.
Ans. 2. 1. Breast ca 2. Duct papilloma and intraductal papilloma (both are same thing) 3.
Mammary duct fistula 4. Breast abscess. X

Mammary duct
ectasia

Dilatation of the large breast ducts


Most common around the menopause
May present with a tender lump around the areola +/- a green nipple discharge
If ruptures may cause local inflammation, sometimes referred to as 'plasma cell
mastitis'

Duct papilloma

Local areas of epithelial proliferation in large mammary duct


May present with blood stained discharge

Breast abscessMore common in lactating women Red, hot tender swelling purulent discharge.
Periductal mastitis occurs when the ducts under the nipple become inflamed and infected. It's a benign
condition (not cancer), which can affect women of all ages but is more common in younger women.
Symptoms include: the breast becoming tender and hot to the touch. the skin may appear reddened.
Mammary duct fistulaThis is a communication between the skin and a major subareolar breast
duct.
It may occur following incision and drainage of a non-lactating abscess, spontaneous discharge
of a periareolar mass or following biopsy of a periductal inflammatory mass.
Treatment is by excision under antibiotic cover.

40. A young male whose sclera was noted to be yellow by his colleagues has a hx of
taking OTC
drugs for some pain. Tests showed raised bilirubin, ALT and AST normal. The
provocation test
with IV nicotinic acid is positive and produces further rise in the serum bilirubin levels.
What is
the most likely dx?
a. Acute hepatitis
b. Drug hypersensitivity
c. Gilberts syndrome
d. Acute pancreatitis
Q. 1. Does nicotinic acid provocation test can differentiate between CLD and Gilberts?
Q. 1. What is the key?
Q. 2. What are the points in favour of your diagnosis?
Ans. 1. C.
Ans. 2. Only bilirubin is increased but not the liver enzymes. Also positive nicotinic acid
provocation test is in its favour. X
Normal AST and ALT rules out any possible insult to the liver.
drug hypersensitivity will not give positive nicotinic acid provocation test.

autosomal recessive condition of defective bilirubin conjugation due to a deficiency of


UDP glucuronyl transferase
unconjugated hyperbilirubinemia (i.e. not in urine)
diagnosed around puberty, and aggravated by intercurrent illness, stress, fasting or
after administration of certain drugs

Investigation and management


investigation: rise in bilirubin following prolonged fasting or IV nicotinic acid
no treatment required

*********************
41. A 24yo biker has been rescued after being trapped under rocks for almost 12h. He
complains of reddish brown urine. His creatinine is 350umol/L and his urea is 15mmol/L.
What is the most
imp step in the management of this patient?
a. Dialysis
b. IV NS
c. IV dextrose
d. IV KCl
e. Pain relief
Q. 1. What is the key?
Q. 2. What is the likely diagnosis?
Ans. 1. Key is B.
Ans. 2. Rhabdomyolysis. X

Crush syndrome is characterised by:

Hypovolaemic shock
Hyperkalaemia

Metabolic acidosis .
Acute kidney injury.
Disseminated intravascular coagulation (DIC).
ECG may show changes secondary to hyperkalaemia.

In the adult, a saline infusion of 1,500 ml/hour should be initiated during extrication. Early,
vigorous hydration (10 litres/day) helps preserve renal function.
42. A 74yo man who has been a smoker since he was 20 has recently been dx with
SCLC. What
serum electrolyte picture will confirm the presence of SIADH?
a. High serum Na, low serum osmolarity, high urine osmolarity
b. Low serum Na, low serum osmolarity, high urine osmolarity
c. Low serum Na, high serum osmolarity, high urine osmolarity
d. High serum Na, low serum osmolarity, low urine osmolarity
e. High serum Na, high serum osmolarity, low urine osmolarity
Ans. b.
SIADH means excess ADH so water retention which will lead to dilutional hyponatremia and decrease in
serum osmolarity and sebsequent increase in urine osmolarity

Management: treat the cause and restrict fluid.


if severe: salt+- loop diuretics
vaptans can be used.
43. A man brought into the ED after being stabbed in the chest. Chest is bilaterally clear
with
muffled heart sounds. BP is 60/nil. Pulse is 120bpm. JVP raised. What is the most likely
dx?
a. Pulmonary embolism
b. Cardiac tamponade
c. Pericardial effusion
d. Hemothorax
e. Pneumothorax
Ans. b.
Beck's triad : low bp , muffled HS, raised jvp .. Cardiac tamponade

Features
dyspnoea
raised JVP, with an absent Y descent - this is due to the limited right ventricular filling
tachycardia
hypotension
muffled heart sounds
pulsus paradoxus
Kussmaul's sign (much debate about this)
ECG: electrical alternans

44. A 50yo pt is admitted for elective herniorraphy. Which of the following options will
lead to a
postponement of the operation?
a. SBP 110mmHg
b. MI 2 months ago
c. Hgb 12g/dl
d. Pain around hernia
e. Abdominal distention
Key is B. After MI elective surgery should not be done before 6 months post MI.
Criteria for postponing elective surgery. ...Hb <10 ,

Plt count <50000


Systolic BP... <90
Uncontrolled HTN, DM , asthma
MI within 3 months

45. A 32yo woman of 39wks gestation attends the antenatal day unit feeling very unwell
with
sudden onset of epigastric pain associated with nausea and vomiting. Her temp is
36.7C. Exam:
she is found to have RUQ tenderness. Her blood results show mild anemia, low
platelets,
elevated liver enzymes and hemolysis. What is the most likely dx?
a. Acute fatty liver of pregnancy
b. Acute pyelonephritis
c. Cholecystitis
d. HELLP syndrome
e. Acute hepatitis
Q. 1. What is the key?
Q. 2. What is the main treatment
Ans. 1. D.
Ans. 2. The main treatment is to deliver the baby as soon as possible [as early as after
34 weeks if multisystem disease is present].
HELLP syndrome is a group of symptoms that occurs in pregnant women who have preeclampsia or eclampsia and who also show signs of liver damage and abnormalities in blood
clotting.
H aemolysis
EL (elevated liver) enzymes
LP (low platelet) count

46. A woman comes with an ulcerated lesion 3 cm in the labia majorum. What is
the lymphatic drainage of this area?
a. External iliac
b. Superficial inguinal LN
c. Para-aortic
d. Iliac
e. Aortic
Ans. Key is B. Superficial inguinal LN.

47. A man post-cholecystectomy presented with jaundice, fever and dark urine. What is
the most
diagnostic inv?
a. ERCP
b. USG Abdomen
c. CT Scan
d. MRCP
e. MRI

Q. 1. What is the key?


Q. 2. What is the likely diagnosis?
Ans. 1. A. ERCP
Ans. 2. Cholangitis. Post cholescytectomy syndrome?
investigation of choice in post cholecystectomy syndrome is ercp

Charcot's triad of right upper quadrant (RUQ) pain, fever and jaundice
Management
intravenous antibiotics
endoscopic retrograde cholangiopancreatography (ERCP) after 24-48 hours to
relieve any obstruction

ERCP contraindications:

Acute pancreatitis (unless persistently raised or worsening bilirubin suggests


ongoing obstruction)
Previous pancreatoduodenectomy
Coagulation disorder if sphincterotomy planned
Recent myocardial infarction
Inadequate surgical back-up
History of contrast dye anaphylaxis
Poor health condition for surgery
Severe cardiopulmonary disease

48. A 79yo stumbled and sustained a minor head injury 2 weeks ago. He has become
increasingly
confused, drowsy and unsteady. He has a GCS of 13. He takes warfarin for Afib. What is
the most likely dx?
a. Extradural hemorrhage
b. Cerebellar hemorrhage
c. Epidural hemorrhage
d. Subdural hemorrhage
e. Subarachnoid hemorrhage
Q. 1. What is the key?
Q. 2. What is the management?
Ans. 1. D.
Ans. 2. 1 line: Evacuation by burr hole craniostomy. 2 line: Craniotomy if the clot is
organized.
st

nd

The gradual onset of symptoms supports the dx


Subdural hematoma as old shrunken /alcoholic brains are prone to develop tear in the veins which bleed
slowly and eventually the hematoma gets big enough to show the symptoms
Also in this case the patient is on warfarin

Type of injury

Extradural
(epidural)
haematoma

Notes

Often results from acceleration-deceleration trauma or a blow to the


side of the head. The majority of epidural haematomas occur in the
temporal region where skull fractures cause a rupture of the middle
meningeal artery.
Features

Subdural
haematoma

features of raised intracranial pressure


some patients may exhibit a lucid interval

Bleeding into the outermost meningeal layer. Most commonly occur


around the frontal and parietal lobes.
Risk factors include old age, alcoholism and anticoagulation.

Slower onset of symptoms than a epidural haematoma.

Subarachnoid
haemorrhage

Usually occurs spontaneously in the context of a ruptured cerebral


aneurysm but may be seen in association with other injuries when a
patient has sustained a traumatic brain injury
worst headache.

49. A 25yo female complains of intermittent pain in her fingers. She describes episodes
of
numbness and burning of the fingers. She wears gloves whenever she leaves the
house. What is
the most probable dx?
a. Kawasaki disease
b. Takayasu arteritis
c. Buergers disease
d. Embolism
e. Raynauds phenomenon
Ans is e.
wearing glove is the catch phrase
Takayasu;pulseless disease, will present with other symptoms like unequal pulses,diziness,weakness
fr buergers hx of smoking...age usually more then 40.buerger's disease should present with smoking hx of
an aged male.

Management
first-line: calcium channel blockers e.g. nifedipine
IV prostacyclin infusions: effects may last several weeks/months

50. A 22yo lady has been unwell for some time. She came to the hospital with
complaints of fever
and painful vesicles in her left hear. What is the most probable dx?
a. Acne
b. Herpes zoster
c. Chicken pox
d. Insect bite
e. Cellulitis
Q. 1. What is the key?
Q. 2. What is the specific name of the condition?

Ans. 1. Herpes Zoster


Ans. 2. Herpes zoster oticus/Ramsay hunt syndrome.
Ramsey hunt syndrome
Painful vesicles in her left ear
Features
auricular pain is often the first feature
facial nerve palsy
vesicular rash around the ear

other features include vertigo and tinnitus

Management
oral aciclovir and corticosteroids are usually given

51. A 5yo girl had earache and some yellowish foul smelling discharge, perforation at the
attic and
conductive hearing loss. She has no past hx of any ear infections. What is the most
appropriate
dx?
a. Acute OM
b. OM with effusion
c. Acquired cholesteatoma
d. Congenital cholesteatoma
e. Otitis externa
Q. 1. What is the key?
Q.2. What are the points in favour of your diagnosis?
Ans. 1. The key is c. Acquired cholesteatoma.
Ans. 2. Ans. 1. The key is c. Acquired cholesteatoma.
Ans. 2. acquired cholesteatomas develop as a result of chronic middle ear infection and are
usually associated with perforation of the tympanic membrane at the attic (mass is seen in attic
with perforation at pars flaccida- in contrast to medial to tympanic membrane which is in
congenital). Clinical presentation usually consists of conductive hearing loss, often with purulent
discharge from the ear
In congenital
mass medial to the tympanic membrane
normal tympanic membrane
no previous history of ear discharge, perforation or ear surgery.

52. A female with T1DM would like to know about an deficiency of vitamins in pregnancy
that can
be harmful. A deficiency of which vitamin can lead to teratogenic effects in the child?
a. Folic acid
b. Vit B12
c. Thiamine
d. Riboflavin
e. Pyridoxine
Ans. A. Folic acid.

Diet: To prevent neural tube defects (NTD) and cleft lip, all should have folate rich foods
+ folic acid 0.4mg daily >1 month pre-conception till 13wks (5mg/day if past NTD, on anti
epileptics, obese (BMI 30), HIV+ve on co-trimoxazole prophylaxis, diabetic or sickle cell
disease.

Smoking: decreases ovulations, causes abnormal sperm production ( less penetrating


capacity),
rates of miscarriage (2), and is associated with preterm labour and lighter-for-dates
babies placenta praevia and abruption. Reduced reading ability in smokers children up
to 11yrs old shows that long term effects are important.
Alcohol consumption: High levels of consumption are known to cause the fetal alcohol
syndrome. Mild drinking eg 12U/wk has not been shown to adversely affect the fetus.
Especially harmful in weeks 3-8.Miscarriage rates are higher among drinkers of alcohol

53. A 23yo woman has been having pain at the base of her thumb, the pain is
reproduced when
lifting her 3 month old baby or changing diapers and also with forceful abduction of the
thumb
against resistance. What is the likely cause?
a. Avascular necrosis of scaphoid
b. Trigger finger.
c. De Quervains tenosynovitis
Q. 1. What is the key?
Q. 2. How will you diagnose the case?
Ans. 1. The key is c. De Quervains tenosinovitis.
Ans. 2. Can be diagnosed by Finkelsteins test:
The physician grasps the thumb and the hand is ulnar deviated sharply. If sharp pain
occurs along the distal radius (top of forearm, about an inch below the wrist), de
Quervain's syndrome is likely.
De Quervain's tenosynovitis is a common condition in which the sheath containing the
extensor pollicis brevis and abductor pollicis longus tendons is inflamed. It typically affects
females aged 30 - 50 years old
Features
pain on the radial side of the wrist
tenderness over the radial styloid process
abduction of the thumb against resistance is painful
Finkelstein's test: with the thumb is flexed across the palm of the hand, pain is
reproduced by movement of the wrist into flexion and ulnar deviation
Management
analgesia
steroid injection
immobilisation with a thumb splint (spica) may be effective
surgical treatment is sometimes required

54. A 6m child presents with fever and cough. His mother has rushed him to the ED
asking for help. Exam: temp=39C and the child is feeding poorly. Dx?
a. Bronchiolitis
b. Asthma
c. Bronchitis
Q. 1. What is the key?
Q.2. What is the management?
Ans. 1. A. Bronchiolitis.
Ans. 2. Management: 1. Oxygen inhalation 2. Nasogastric feeding. DONT USE: i)
bronchodilator ii) steroid iii) antibiotics routinely. [OHCS, page-160]
Acute bronchiolitis is the big lung infection in infants
Symptoms: coryza precedes cough, low fever, tachypnoea, wheeze, inspiratory
crackles, apnoea, intercostal recession cyanosis.
Cause: Winter respiratory syncytial virus. Others: Mycoplasma, parainfluenza,
adenoviruses. Those <6 months old are most at risk.
Signs prompting admission: Poor feeding, >50 breaths/min, apnoea, dehydration, rib
recession, patient or parental exhaustion
Tests: If severe: CXR (hyperinflation); blood gases/SpO2; FBC.
Treatment: O2 (stop when SpO2 92%); nasogastric feeds. 5% need ventilating
(mortality 1%; 33% if symptomatic congenital heart disease). Dont use bronchiodilators
and steroids routinely

55. A 75yo man collapsed while walking in his garden. He recovered fully within 30 mins
with BP
110/80 mmHg and regular pulse of 70bpm. He has a systolic murmur on examination.
His
activities have been reduced lately which he attributes to old age. What is the definitive
diagnostic inv that will assist you with his condition?
a. ECG
b. Echo
c. 24h ECG monitoring
d. 24h BP monitoring
e. Prv CIN
Q. 1. What is the key?
Q. 2. What are the possible causes of this syncope?
Ans. 1. B. Echo.
Ans. 2. i) Aortic stenosis more likely in elderly. ii) hypertrophic cardiomyopathy less
likely in this age as presentation may present in an earlier age.

Aortic stenosis (AS)


Causes: Senile calcification is the commonest. Others: congenital (bicuspid valve,
Williams syndrome, rheumatic heart disease.
Presentation: Think of AS in any elderly person with chest pain, exertional dyspnoea or
syncope. The classic triad includes angina, syncope, and heart failure (usually after age
60). Also: dyspnoea; dizziness; faints; systemic emboli if infective endocarditis; sudden
death.

Signs: Slow rising pulse with narrow pulse pressure (feel for diminished and delayed
carotid upstrokeparvus et tardus); heaving, non-displaced apex beat; LV heave; aortic
thrill; ejection systolic murmur (heard at the base, left sternal edge and the aortic area,
radiates to the carotids).
There may be an ejection click (pliable valve) or an S4 (said to occur more often with
bicuspid valves, but not in all populations).
Tests: ECG: P-mitrale, LVH with strain pattern; LBBB or complete AV block (calcified
ring). CXR: LVH; calcified aortic valve post-stenotic dilatation of ascending aorta.
Echo: diagnostic.
Doppler echo can estimate the gradient across valves
Cardiac catheter can assess: valve gradient; LV function; coronary artery disease
Management: If symptomatic, prognosis is poor without surgery.
If moderate-to-severe and treated medically, mortality can be as high as 50% at 2yrs,
therefore prompt valve replacement is usually recommended.
In asymptomatic patients with severe AS and a deteriorating ECG, valve replacement is
also recommended. If the patient is not medically fit for surgery, percutaneous
valvuloplasty/replacement (TAVI = transcatheter aortic valve implantation) may be
attempted.
56. A 35yo man with a hx of schizophrenia is brought to the ER by his friends due to
drowsiness. On examination he is generally rigid. A dx of neuroleptic malignant
syndrome except:
a. Renal failure
b. Pyrexia
c. Elevated creatinine kinase
d. Usually occurs after prolonged tx
e. Tachycardia
Ans. Key is D. Usually after prolonged tx. It usually occurs within 10 days of starting
treatment. renal failure is the wrong answer as neuroleptic syndrome can lead to renal
failure so we have to give IV fluids to prevent it.
Cause: antipsychotics or dopamenergic drugs (levodopa)
Management: STOP the drug causing it. IV fluids, Dantrolene, Bromocriptine
57. A 33yo drug addict wants to quit. She says she is ready to stop the drug abuse. She
is supported by her friends and family. What drug tx would you give her?
a. Benzodiazepines
b. Diazipoxide
c. Lithium
d. Methadone
e. Disulfiram
Q.1. What is the key.
Q.2. What drugs should you use in i) tobacco abuse and in ii) alcohol abuse?
Ans. 1. Key is d. Methadone. (used in opiate abuse). Nalexone is the antidote.
Ans. 2. i) tobacco abuse: a) bupropion ii) alcohol: a) acamprosate decreases craving b)
disulfirum is a deterrent.

58. A 16m child presents with drooling, sore throat and loss of voice. He has fever with a
temp of

38.2C. What is your next step towards management?


a. Direct pharynoscopy
b. Call ENT surgeon
c. Call anesthesiologist
d. IV fuilds
e. Start antibiotics
Q. 1. What is the key?
Q. 2. What is the diagnosis?
Q. What is the urgent management?
Ans. 1.Key is C. Call anesthesiologist.
Ans. 2. Diagnosis is Acute epiglottitis.
Ans. 3. In given case urgent intubation is needed to secure airway to prevent blockage
of respiration.
Differential: croup. Croup has barking cough which is worse at night and there is no
drooling of saliva NO COUGH IN EPIGLOTITTIS. Croup caused by parainfluenza while
epiglottitis is caused by H.influenze
Acute epiglottitis is rarer than croup but mortality is high: 1% if respiratory distress. Its
an emergency as respiratory arrest can occur.
Presentation: Often, history is short, septicaemia is rapid, and cough is absent. Also:
sore throat (100%), fever (88%), dyspnoea (78%), voice change (75%), dysphagia
(76%), tender anterior neck cellulitis (27%), hoarseness (21%), pharyngitis (20%),
anterior neck nodes (9%), drooling (head for ward tongue out), prefers to sit, refusal to
swallow,
Cause: Haemophilus (vaccination has reduced prevalence); Strep pyogenes.
Investigation: Fibre-optic laryngoscopy remains the 'gold standard' for diagnosing
epiglottitis
Management: Take to ITU; dont examine throat (causes resp. arrest). Give O2 by
mast, Give nebulized adrenaline, IV dexamethasone, antibiotics, antipyretics until the
anesthetist arrives. Definitive management is intubation

59. A 62yo woman complains of unsteadiness when walking. On examination she has
pyramidal
weakness of her left lower limb and reduced pain and temp sensation on right leg and
right side
of trunk up to the umbilicus. Joint position sense is impaired at her left great toe but is
normal
elsewhere. She has a definite left extensor plantar response and the right plantar
response is
equivocal. Where is the lesion?
a. Left cervical cord
b. Midline mid-thoracic cord
c. Right mid-thoracic cord
d. Left mid-thoracic cord

e. Left lumbo-sacral plexus


Q. 1. What is the key?
Q. 2. What is the name of this condition?
Ans. 1. The key is d. Left mid-thoracic cord.
Ans. 2. Brown-sequard syndrome.

Pain & temperature: carried by lateral spinothalamic tract dicussate to the opposite side
within the spinal cord. Vibration, proprioception: carried by the medial lemniscal
system decussate at the junction of pons and medulla. Upper motor neurons in
pyramidal tract also decussate close to medulla. So from this we know that the lesion is
on the left side. And since the symptoms are below the umbillicus it rules out a cervical
lesion. Lesion of lumbo sacral plexus will impair the whole of the lower limb. So the
correct answer is D.
Brown Sequard syndrome:
A lesion in one half of the spinal cord (due to hemisection or unilateral cord lesion)
Presentation: Ipsilateral UMN weakness below the lesion (severed corticospinal tract,
causing spastic paraparesis, brisk reflexes, extensor plantars) Ipsilateral loss of
proprioception and vibration (dorsal column severed) Contralateral loss of pain and
temperature sensation (severed spinothalamic tract which has crossed over
Causes: Bullet, stab, dart, kick, tumour, disc hernia, cervical spondylosis, MS,
neuroschistosomiasis, myelitis, septic emboli (eg meningococcal).
Imaging: MRI
60. A 26yo man present to ED with increasing SOB on left side and chest pain. He has
been a heavy smoker for the past 4 years. He doesnt have any past med hx. What is
the likely dx?
a. Pulmonary embolism
b. MI
c. Asthma
d. Pleural effusion
e. Pneumothorax
Q. 1. What is the key?
Q. 2. What are the points in favour of your diagnosis?
Q. 3. What is the cause of the disease in this case?
Ans. 1. The key is e. Pneumothorax.
Ans. 2. Increased shortness of breath and chest pain with no past medical history.
Ans. 3. Heavy smoking. Tobacco is a risk factor for spontaneous pneumothorax.

Pneumothorax
Causes Often spontaneous (especially in young thin men) due to rupture of a subpleural
bulla.
Other causes: asthma; COPD; TB; pneumonia; lung abscess; carcinoma; cystic fibrosis;
lung fibrosis; sarcoidosis; connective tissue disorders (Marfans sy., EhlersDanlos sy.),
trauma; iatrogenic (subclavian CVP line insertion, pleural aspiration/ biopsy,
transbronchial biopsy, liver biopsy, +ve pressure ventilation).
Symptoms: There may be no symptoms (especially if fit, young and small pneumothorax)
or there may be sudden onset of dyspnoea and/or pleuritic chest pain. Patients with

asthma or COPD may present with a sudden deterioration. Mechanically ventilated


patients may present with hypoxia or an increase in ventilation pressures.
Signs: Reduced expansion, hyper-resonance to percussion and diminished breath sounds
on the affected side. With a tension pneumothorax, the trachea will be deviated away
from the affected side

Management:

61. A pt with hepatocellular ca has raised levels of ferritin. What is the most probable
cause?
a. Hemochromatosis
b. A1 antitrypsin def

c. Cystic fibrosis
Ans. Haemochromatosis.
Haemochromatosis... Autosomal recessive.
SYMPTOMS bronzing of skin, DM, hepatomegly, arthropathy. Can also cause infertility,
arrhythmias, neurological symptoms. Liver fibrosis, cirrhosis & HCC.
INVESTIGATIONS: Serum ferritin then genetic testing for HFE mutations.
TREATMENT: phlebotomy OR liver transplant
62. A woman has electric pains in her face that start with the jaw and move upwards.
Her corneal
reflexes are normal. What is the most likely dx?
a. Atypical face pain
b. Trigeminal neuralgia
c. Tempero-mandibular joint dysfunction
d. GCA
e. Herpes zoster
Q. 1. What is the key?
Q. 2. What are the options mentioned are possible causes of absent corneal reflex?
Ans. 1. Key is b. Trigeminal neuralgia.
Ans. 2. Possible options are 1. Trigeminal neuralgia 2. Herpes zoster ophthalmicus
Trigerminal Neuralgia...Facial pains. PRESENTATION: The episodes are sporadic and
sudden and often like 'electric shocks', lasting from a few seconds to several minutes.
Pain is unilateral, brief, stabbing, recurrent in the distribution of CN5. Can be provoked
by light touch to the face, eating, cold winds, or vibrations typically occurs after shaving,
brushing teeth.
Cause is a compression of CN5.
No Investigations
TREATMENT: Carbamezapine is the first line. Rhizotomy (surgery) may also be done
63. A 32yo man presented with slow progressive dysphagia. There is past hx of retrosternal
discomfort and he has been treated with prokinetics and H2 blockers. What is the
probably dx?
a. Foreign body
b. Plummer vinson syndrome
c. Pharyngeal pouch
d. Peptic stricture
e. Esophageal Ca
Q. 1. What is the key?
Q. 2. What is the underlying cause of this stricture?
Ans. 1. The key is D. Peptic stricture.
Ans. 2. The underlying cause is Gastro-oesophageal reflux.

Points not in favor of CA: Age (32yrs), no anemia, anorexia, lethargy etc mentioned.

Peptic Stricture
PRESENTATION: heartburn, dysphagia, impaction of food, weight loss, and chest pain.
There can be progressive dysphagia, weight loss & anemia.
CAUSES: History of GERD, corrosive intake, drugs like NSAIDs
INVESTIGATIONS: Endoscopy (risk of perforation) Barium swallow
TREATMENT: Benign: endoscopic baloon dilation. Malignant: oesophagectomy
64. A 56yo man comes with hx of right sided weakness & left sided visual loss. Where is
the
occlusion?
a. Ant meningeal artery
b. Mid meningeal artery
c. Mid cerebral artery
d. Carotid artery
e. Ant cerebral artery
f. Ant communicating artery
Q. 1. What is the key?
Q. 2. How will you differentiate between middle cerebral artery occlusion from anterior
cerebral artery occlusion?
Ans. 1. The key is d. Carotid artery.
Ans. 2.
i)
Middle cerebral artery occlusion: paralysis or weakness of contralateral
face and arm (faciobracheal). Sensory loss of the contralateral face and arm.
ii)
Anterior cerebral artery occlusion: paralysis or weakness of the
contralateral foot and leg. Sensory loss at the contralateral foot and leg.
Carotid Artery occlusion:
PRESENTATION: Patients may present with TIAs or CVEs.
Typical symptoms are contralateral weakness or sensory disturbance, ipsilateral
blindness, and (if the dominant hemisphere is involved) dysphasia, aphasia or speech
apraxia.
Carotid bruit may or may not be present
INVESTIGATIONS: For diagnosis: CAROTID ANGIOGRAPHY GOLD STANDARD. MR
angio and angio CT can also be used.
Echo colour Doppler ultrasonography is the screening method of choice
TREATMENT: Medical: Antiplatelets, Anti HTN, Statins
Surgery: Carotid endartarectomy. Symptomatic patients with greater than 50% stenosis
and healthy, asymptomatic patients with greater than 60% stenosis warrant
consideration for carotid endarterectomy.
65. A young college student is found in his dorm unconscious. He has tachyarrhythmia
and high
fever. He also seems to be bleeding from his nose, which on examination shows a
perforated
nasal septum. What is the most likely dx?
a. Marijuana OD
b. Cocaine OD
c. Heroin OD
d. Alcohol OD
e. CO poisoning

Q. 1. What is the key?


Q. 2. What are the points that favours the diagnosis in given question?
Q. 3. What are other important findings?
Ans. 1. Key is B. Cocaine overdose.
Ans. 2. Points in favour: i) Tachyrhythmia ii) High fever iii) perforated nasal septum iv)
unconsciousness
Ans. 3. Other findings: i) Psychiatric: anxiety, paranoia ii) Tachypnoea iii) Increased
energy and talking rapidly iv) Dilated pupils. Also: [rhabdomyolysis, metabolic acidosis,
convulsion].
COCAINE may be snored, taken via IV or smoked.
PRESENTATION: occasional use produces euphoria, increased alertness and feelings
of self-confidence and competence
frequent repeated use causes tachycardia, twitching, insomnia and anxiety
ADDICTION: can result in perforated nasal septum, psych problems.
The patient may present in anxiety, paranoia, they may ask for help.
MANAGEMENT: CBT, self help groups. Benzodiazepines are first line drugs, anti
depressents like SSRIs but donot use with cocaine (causes SSRI syndrome). Beta
blockers for anxiety

66. A 56yo pt whose pain was relieved by oral Morphine, now presents with
progressively
worsening pain relieved by increasing the dose of oral morphine. However, the pt
complains
that the increased morphine makes him drowsy and his is unable to carry out his daily
activities.
What is the next step in his management?
a. Oral oxycodone
b. Oral tramadol
c. PCA
d. IV Fentanyl
e. Diamorphine
Ans. Key is oral oxycodon.
If there are intolerable side effects to morphine go for oral oxycodone
Pain ladder: NSAIDs, Mild opioids, strong opioids.
Once on one step of the ladder do not go back.
NSAIDs are good for bone pain.
Morphine Start with oral solution 510mg/4h PO with an equal breakthrough dose
as often as required. A double dose at bedtime can enable a good nights sleep. Patient
needs will vary greatly and there is no maximum dose; aim to control symptoms
with minimum side-effects. If not effective, increase doses in 3050% increments
(5mg10mg20mg30mg45mg). Change to modified release preparations (eg
MST Continus 12h) once daily needs are known by totalling 24h use and dividing by
2. Prescribe 1/6th of the total daily dose as oral solution for breakthrough pain. Sideeff
ects (common) are drowsiness, nausea/vomiting, constipation and dry mouth.
Hallucinations and myoclonic jerks are signs of toxicity and should prompt dose review.
If the oral route is unavailable try morphine/diamorphine IV/SC (see BOX for

conversions). If difficulty tolerating morphine/diamorphine, try oxycodone


PO/IV/SC/
PR, starting at an equivalent dose. It is as effective as morphine and is a useful 2ndline
opioid with a different range of receptor activity. 61 OxyNorm is the oral liquid form.
There are also fentanyl transdermal patches which should usually be started under
specialist supervision (after opioid dose requirements have been established). Remove
after 72h, and place a new patch at a different site. 45mg oral morphine/24h
is approximately equivalent to a 12mcg/h fentanyl patch.
67. A 30yo man presents with a 5cm neck mass anterior to the sternocleido-mastoid
muscle on the left side in its upper third. He states that the swelling has been treated
with antibiotics for
infection in the past. Whats the most likely cause?
a. Branchial cyst
b. Parotitis
c. Pharyngeal pouch
d. Thyroglossal cyst
e. Thyroid swelling
Q. 1. What is the key?
Q. 2. Justify your answer.
Ans. 1. The key is A. Branchial cyst.
Ans. 2. i) Branchial cyst is anterior triangular lump. [parotid is also anterior triangular
lump but it regresses with appropriate treatment]. ii) pharyngeal pouch is posterior
triangular lump. iii) Thyroglossal is midline lump. iv) thyroid swelling moves with
swallowing.
Branchial cysts emerge under the anterior border of sternocleidomastoid where the
upper third meets the middle third (age <20yrs).
CAUSE: Due to non-disappearance of the cervical sinus (where 2nd branchial arch
grows down over 3rd and 4th)
TREAT by excision
If lump in the supero-posterior area of the anterior triangle, is it a parotid tumour
68. An 18yo man is rushed into the ER by his friends who left him immediately before
they could be interviewed by staff. He is semiconscious, RR=8/min, BP=120/70mmHg,
pulse=60bpm. He is
noted to have needle track marks on his arms and his pupils are small. What is the
single best
initial tx?
a. Insulin
b. Naloxone
c. Methadone
d. Gastric lavage
Q. 1. What is the key?
Q. 2. What is the diagnosis?
Q. 3. What are the points in favour of the diagnosis?
Ans.1. The key is B. Naloxone.
Ans. 2. The diagnosis is opiate overdose.

Ans. 3. Points in favour are: i) reduced consciousness ii) RR 8/min (12<) iii) hypotension
(here lower normal) iv) miosis v) needle track marks on his arms.
Opioid overdose presents with the usual Adverse effects of opioids. This is a typical
presentation. Treatment IS WITH NALOXONE IV/IM/SC
OPIOID WITHDRAWAL SYMPTOMS:
Sweating. Watering eyes. Rhinorrhoea Yawning Feeling hot and cold. Anorexia
and abdominal cramps. Nausea, vomiting and diarrhoea. Tremor. Insomnia,
restlessness, anxiety and irritability. Generalised aches and pains. Tachycardia,
hypertension. Goose flesh (goosebumps). Dilated pupils. Increased bowel
sounds. Coughing.
COMPLICATIONS: Skin infection at injection sites (can be severe; necrotising
fasciitis can occur). Septicaemia. Infective endocarditis. HIV infection. Hepatitis
A, B and C infection. Tuberculosis infection.
TREATMENT: Methadone or buprenorphine. Stabilize the patient on either of the two.
Naltrexone can be used once the patient is detoxified.

69. A 30yo man and wife present to the reproductive endocrine clinic because of
infertility. The man is tall, has bilateral gynecomastia. Examination of the testes reveals
bilateral small, firm testes.
Which of the following inv is most helpful in dx?
a. CT of pituitary
b. Chromosomal analysis
c. Measure of serum gonadotropins
d. Measure of serum testosterone
Q. 1. What is the key?
Q. 2. What is the diagnosis?
Q. 3. What are the points in favour of your diagnosis?
Ans. 1. The key is B. Chromosomal analysis.
Ans. 2. The diagnosis is Klinefelters syndrome. (xxy)
Ans. 3. The points in favour are: i) Infertility ii) Tall stature iii) Bilateral gynaecomastia iv)
Bilateral small firm testes.
Klinefelters syndrome: (47,XXY, 48,XXYY polysomy or a mosaic 47,XXY/46,XY)
Turners is XO with NO Barr body.
PRESENTATION: Infertility & small testis (most common & most imp) gynecomastia,
lack of secondary sexual characteristics, tall and slender and learning disablities
(delayed speech, behavioral problems)
Investigations: Before birth via amniocentesis or CVS.
Later serum testosterone is low. FSH & LH are high (FSH>LH)
Chromosome karyotyping gives the deifinitive diagnosis
TREATMENT: 1. Testosterone replacement. 2.Intracytoplasmic injection of sperm. 3.
Surgery for gynecomastia
70. An 18yo female just received her A-Level results and she didnt get into the
university of her
choice. She was brought into the ED after ingestion of 24 paracetamol tablets. Exam:
confused

and tired. Initial management has been done. Inv after 24h: normal CBC, ABG = pH7.1,
PT=17s,
Bilirubin=4umol/L, creatinine=83umol/L. What is the next step in management?
a. Observation for another 24h
b. Refer to psychologist
c. Give N-Acetylcysteine
d. Discharge with psychiatry referral
e. Liver transplantation
Q. 1. What is the key?
Q. 2. What are the indications of this management?
Ans. 1. The key is E. Liver transplantation.
Ans. 2. King's College Hospital criteria for liver transplantation in paracetamol-induced acute liver failure.
arterial pH <7.3 or arterial lactate >3.0 mmol/L after adequate fluid resuscitation, OR
if all three of the following occur in a 24-hour period:
Creatinine >300 mol/L.
PT >100 seconds (INR >6.5).
Grade III/IV encephalopathy.

PARACETAMOL POISONING: >150mg/kg or 12g total


PRESENTATION: Hepatic damage shown by deranged LFTs occurs after 24hrs.
Patients may develop encaph, hypoglycemia, ARF
INVESTIGATIONS: Paracetamol levels: 4hrs post ingestion, if time is >4hr or staggered
overdose
Any alcohol taken (acute alcohol ingestion will inhibit liver enzymes and may reduce the
production of the toxin NAPQI, whereas chronic alcoholism may increase it)

MANAGEMENT:
If presentation is within the first 4 hours give activated charcoal
All patients who have a timed plasma paracetamol level plotted on or above the
line drawn between 100 mg/L at 4 hours and 15 mg/L at 15 hours after ingestion,
should receive acetylcysteine.
If time unknown (even in staggered dose) give N-Acetyl cysteine without delay
NAC most effective in the first 8 hrs.
NAC can be given during pregnancy
Beware if the patient is on any P450 enzyme inducer medicines as they increase
the toxicity
Refer to ICU if there is fulminant liver failure - those treated with N-acetylcysteine
(NAC) to the medical team and all para-suicides to the psychiatric team.
71. A 75yo alcoholic presents with a mass up to umbilicus, urinary dribbling,
incontinence, and
clothes smelling of ammonia. What is the next step in management?
a. Urethral catheter
b. Suprapubic catheter
c. Antibiotics
d. Condom catheter
e. Nephrostomy
Q. 1. What is the key?
Q. 2. What is the cause of this retention?
Ans. 1. The key is A. Urethral catheter.
Ans. 2. Alcohol consumption (it is rather a less common cause of urinary retention).

ACUTE URINARY RETENTION

Causes of urinary retention

In men - BPH, meatal stenosis, paraphimosis, penile constricting bands,


phimosis, prostate cancer.
In women - prolapse (cystocele, rectocele, uterine), pelvic mass (gynaecological
malignancy, uterine fibroid, ovarian cyst), retroverted gravid uterus.
In both - bladder calculi, bladder cancer, faecal impaction, gastrointestinal or
retroperitoneal malignancy, urethral strictures, foreign bodies, stones.
Infectious and inflammatory:
In men - balanitis, prostatitis and prostatic abscess.
In women - acute vulvovaginitis, vaginal lichen planus and lichen sclerosis,
vaginal pemphigus.
In both - bilharzia, cystitis, herpes simplex virus (particularly primary infection),
peri-urethral abscess, varicella-zoster virus.
Drug-related:
Up to 10% AUR episodes are thought to be attributable to drugs. Those known to
increase risk include:
Anticholinergics (eg, antipsychotic drugs, antidepressant agents, anticholinergic
respiratory agents). Opioids and anaesthetics. Alpha-adrenoceptor agonists.
Benzodiazepines. NSAIDs Detrusor relaxants. Calcium-channel blockers
Antihistamines. Alcohol.
Neurological:
More often causing chronic retention but may cause AUR:
Autonomic or peripheral nerve (eg, autonomic neuropathy, diabetes mellitus,
Guillain-Barr syndrome, pernicious anaemia, poliomyelitis, radical pelvic
surgery, spinal cord trauma, tabes dorsalis).
Brain (eg, cardiovascular disease (CVD), MS, neoplasm, normal pressure
hydrocephalus, Parkinson's disease).
Spinal cord (eg, invertebral disc disease, meningomyelocele, MS, spina bifida
occulta, spinal cord haematoma or abscess, spinal cord trauma, spinal stenosis,
spinovascular disease, transverse myelitis, tumours, cauda equina).
Other:
In men - penile trauma, fracture, or laceration.
In women - postpartum complications (increased risk with instrumental delivery,
prolonged labour and Caesarean section), urethral sphincter dysfunction
(Fowler's syndrome).
In both - pelvic trauma, iatrogenic, psychogenic.
[2]

MANAGEMENT: Foleys catheter first line. If contraindicated or failed refer to urology or


try suprapubic catheterization
COMPLICATIONS: UTIs, Renal failure, Post retention diureses and hematuria
72. In CRF, main cause of Vit D deficiency is the failure of:
a. Vit D absorption in intestines
b. 25 alpha hydroxylation of Vit D
c. Excess Vit D loss in urine

d. 1 alpha hydroxylation of Vit D


e. Availability of Vit D precursors
Ans. The key is D. 1 alpha hydroxylation of Vit D [kidney] [25 alpha hydroxilation of Vit
D- liver].
VITAMIN D: FUNCTION: Absorption of calcium and phosphorus from GIT
SOURCES: Oily fish, fortified food, skin synthesized VitD using sunlight
Deficiency causes Rickets in children and ostomalacia in adults
Causes of Deficiency: Increased demand in pregnancy, breast feeding, Malabsorptive
diseases like Crohns, coeliac, pancreatic insufficiency, CKD, Liver diseases
Treatment: Take vit D supplements in the form of calciferol. Tablets or injections.
Injections can be effective for upto 6 months.
All pregnant and breastfeeding women should take a daily supplement containing 10
micrograms of vitamin D
All children aged 6months to 6years should take daily vitD supplements in the form of
drops
73. Pt with puffiness of face and rash showing cotton wool spots on fundoscopy. Whats
the dx?
a. Macular degeneration
b. Hypertensive retinopathy
c. Diabetic background
d. Proliferative diabetic retinopathy
e. SLE
Q. 1. What is the key?
Q. 2. Why there is puffyness of face?
Q. 3. Why there is cotton wool spots on fundoscopy? What is the most common ocular
manifestation of SLE?
Ans. 1. The key is SLE.
Ans. 2. Puffiness is due to lupus nephritis.
Ans. 3. SLE, can involve the retina. The classic lesion of SLE is a white fluffy appearing
lesion within the retina known as a cotton wool spot. The most common ocular
manifestation in SLE is Keratoconjunctivits sicca.
SLE. Female male 5:1. More common in asians, afro caribbeans
Cause: HLA DR-2 DR-3 association, Environmental factors like UV rays, EBV and drugs
(chlorpromazine, methyldopa, hydralazine, isoniazid, d-penicillamine and minocycline)
Presentation: SLE is a remitting and relapsing illness
Raynauds phenomenon
Arthritis: Early morning stiffness, non erosive, no swelling peripheral, symmetrical
Photosensivity: malar rash. Precipitated by sunlight, sparing the nasolabial fold.
Erythematous, raised & pruritic
Discoid lupus eryhtamatosus: well damarcated with scaling, on sun exposed areas
Mouth ulcers
Pulmonary: pleurisy, fibrosing alveolitis
Renal: Nephritis is often asymptomatic. Glomerulonephritis is common in lupus pts
Neuro: depression and anxiety are common. There may be seizures, meningitis,
psychosis
Vasculitis

Criteria for diagnosis: if any 4 of these 11 are present not necessarily at the same
time.
Malar rash.
Discoid lupus.
Photosensitivity.
Oral or nasopharyngeal ulcers.
Non-erosive arthritis involving two or more peripheral joints.
Pleuritis or pericarditis.
Renal involvement with persistent proteinuria or cellular casts.
Seizures or psychosis.
Haematological disorder: haemolytic anaemia or leukopenia or lymphopenia or
thrombocytopenia.
Immunological disorder: anti-DNA antibody or anti-Sm or antiphospholipid
antibodies.
A positive antinuclear antibody.
Investigations: FBC: Anemia, thrombocytopenia may be seen.
ESR IS RAISED BUT CRP IS USUALLY NORMAL
Antibodies: ANA: screening test but not diagnostic, 95% sensitive. Anti DNA: diagnostic
and show disease activity. High specificity. Anti-Sm is the most specific antibody but
30-40% sensitive. Anti-SSA (Ro) or Anti-SSB (La) are present in 15% of patients
Anti-RNP may indicate mixed connective tissue disease with overlap SLE, scleroderma,
and myositis
Anti-histone: drug-induced lupus ANA antibodies are often this type
anticardiolipin antibodies and lupus anticoagulant should be checked in lupus patients,
as they are associated with APLS.
Complement C3 and C4 levels are reduced
TREATMENT: Avoid sun exposure
NSAIDs for musculoskeletal pains
Steroids: Effective but can be harmful for CVS, osteoporosis. High-dose
prednisolone is reserved for life-threatening SLE
Hydroxychloroquine remains first-line treatment for patients with mild SLE,
especially for those with arthralgia, skin rashes, alopecia, and oral or genital
ulceration
Cyclophosphamide is reserved for treatment of life-threatening disease,
particularly lupus nephritis, vasculitis and cerebral disease
Mycophenolate mofetil is as effective as cyclophosphamide in inducing
remission in lupus nephritis Mycophenolate mofetil is more effective than
azathioprine in maintenance therapy for preventing relapse
Azathioprine is used as a steroid-sparing agent. As an alternative to
cyclophosphamide, azathioprine is much safer but probably less effective
Intravenous immunoglobulins are increasingly being used in the treatment of
resistant lupus and also have a role in patients who have concomitant infection
and active lupus, for whom immunosuppression treatment is often
inappropriate.
Belimumab is licensed as adjunctive therapy in patients with active,
autoantibody-positive SLE with a high degree of disease activity despite
standard therapy
Fertility is normal and pregnancy is safe in mild or stable lupus
COCP should be used with caution

74. A 35yo man presents with progressive breathlessness. He gave a hx of


polyarthralgia with
painful lesions on the shin. CXR: bilateral hilar lymphadenopathy. Whats the most likely
dx?
a. Bronchial asthma
b. Cystic fibrosis
c. Sarcoidosis
d. Bronchiectasis
e. Pneumonia
Q. 1. What is the key?
Q.2 . What is the specific name of this condition? What is the triad?
Ans. 1. The key is C. Sarcoidosis.
Ans. 2. Lofgren syndrome. The triad is i) Erythema nodosum ii) Bilateral hilar
lymphadenopathy iii) Arthralgia.
Sarcoidosis:
Presentation: Lungs are in involved in more than 90% cases of sarcoidosis. There is
interstitial lung disease. The painful skin lesion is erythema nodosum. Also look for
Lupus pernio (chronic raised hardened, often purple lesion) may be seen on the face.
Lofgren syndrome is often a part of sarcoidosis. The triad is i) Erythema nodosum
ii) Bilateral hilar lymphadenopathy iii) Arthralgia
Sarcoidosis is a multisystem disease and can involve any system/organ
Tests: ESR is often raised. Serum ACE enzyme levels are raised in 60% of times
Plain CXR may show bilateral hilar or paratracheal lymphadenopathy. High resolution
CT should be done. There will be restricitve pattern of disease on pulmonary function
tests.
Transbronchial biopsy can demonstrate the presence of non-caseating granulomata,
giving a more accurate diagnosis
Bronchioalveolar lavage may also be done
75. A child presents with clean wound, but he has never been immunized as his parents
were
worried about it. There is no contraindication to immunization, what is the best
management?
a. Full course of DTP
b. 1 single injection DT
c. 1 single injection DTP
d. Only Ig
e. Antibiotic
Ans. The key is A. Full course of DTP.

Vaccination is at 2,3,4 months of age for children under 10 yrs of age. If a dose is missed
just give the next dose and no need to repeat the previous doses and just complete the 3
doses.
For >10 yrs it is same with an interval of at least 1 month between doses.
BOOSTERS: Age <10 yr should receive boosters 3 years after completing the 3 doses it
is DPT. 2nd booster is 10yrs after the 1st booster.
Age >10 yrs receive boosters 5yrs after completing the initial 3 doses. 2nd booster is
10yrs after the 1st booster.
Where there is no reliable history of previous immunisation, it should be assumed that
they are unimmunised, and the full UK recommendations should be followed

76. A 65yo HTN man presents with lower abdominal pain and back pain. An expansive
abdominal mass is palpated lateral and superior to the umbilicus. What is the single
most discriminating inv?
a. Laparascopy
b. KUB XR
c. Pelvic US
d. Rectal exam
e. Abdominal US
Q. 1. What is the key?
Q. 2. What is the diagnosis?
Q. 3. What are the points given here in favour of your diagnosis?
Ans. 1. The key is E. Abdominal US.
Ans. 2. The diagnosis is Abdominal aortic aneurism.
Ans. 3. Points in favour of AAA are i) hypertension ii) abdominal pain iii) back pain iv)
expansile abdominal mass lateral and superior to the umbilicus.
RISK Factors include : Family Hx , tobacco smoking is an important factor.
Male sex.
Increasing age.
Hypertension.
Chronic obstructive pulmonary disease.
Hyperlipidaemia
UNRUPTURED AAA is commonly asymptomatic and is an accidental finding.
Ruptured AAA may present with:
Pain in the abdomen, back or loin - the pain may be sudden and severe.
Syncope, shock or collapse:

The degree of shock varies according to the site of rupture and whether it is contained eg, rupture into the peritoneal cavity is usually dramatic, with death before reaching
hospital; whereas rupture into the retroperitoneal space may be contained initially by a
temporary seal forming.
Ultrasound is simple and cheap; it can assess the aorta to an accuracy of 3 mm. It is
used for initial assessment and follow-up.
SCANS :
CT Scan provides more anatomical details - eg, it can show the visceral arteries, mural
thrombus, the 'crescent sign' (blood within the thrombus, which may predict imminent
rupture) and para-aortic inflammation. CT with contrast can show rupture of the
aneurysm.
MRI angiography may be used.
If size exceeds 5.5cm, we go for surgery.
77. A 55yo man has had severe pain in the right hypochondrium for 24h. The pain
comes in waves and is accompanied by nausea. Nothing seems to relieve the pain. He
feels hot and sweaty but has normal temp. What is the most appropriate next inv:
A.US Abdomen
b. ERCP
c. MRCP
d. Serum amylase
e. UGI endoscopy
Q. 1. What is the key?
Q. 2. What is the diagnosis?
Q. 3. What are the points in favour of your diagnosis?
Ans. 1. The key is A. US abdomen.
Ans. 2. The diagnosis is biliary colic.

Ans. 3. Points in favour- i) severe right hypochondrial pain. ii) colicky nature of the pain
(comes in waves) iii) nausea iv) absence of fever iv) absence of jaundice.
Biliary colic :
The pain starts suddenly in the epigastrium or right upper quadrant (RUQ) and may
radiate round to the back in the interscapular region.
Contrary to its name, it often does not fluctuate but persists from 15 minutes up to 24
hours, subsiding spontaneously or with analgesics.
Nausea or vomiting often accompanies the pain, which is visceral in origin and occurs as
a result of distension of the gallbladder due to an obstruction or to the passage of a
stone through the cystic duct.
Differential diagnosis include reflux, peptic ulcers, irritable bowel syndrome, relapsing
pancreatitis and tumours - eg, stomach, pancreas, colon or gallbladder. Two or more of
these conditions may overlap, so the diagnosis may not be easy.
ULTRASOUND is the best way to demonstrate stones, being 90-95% sensitive.

78. A 67yo man has deteriorating vision in his left eye. He has longstanding
COPD and is on multiple drug therapy. What single medication is likely to
cause this visual deterioration?
a. B2 agonist
b. Corticosteroid
c. Diuretic
d. Theophylline
Q. 1. What is the key?
Q. 2. What is the cause of deteriorating vision?
Ans. 1. The key is B. Corticosteroid.

Ans. 2. Prolonged corticostiroids [also topical i.e. eye drop] can cause
cataract.
79. A woman who returned from abroad after 3 weeks of holiday complains of severe
diarrhea of 3 weeks. She also developed IDA and folic acid def. What condition best
describes her situation?
a. Jejunal villous atrophy
b. Chronic diarrhea secretions
c. Malabsorption
d. Increased catabolism
e. Increased secretions of acid
Q. 1. What is the key?
Q. 2. What are the points in favour?
Q. 3. What are the signs of deficiency may be present?
Ans. 1. The key is C. Malabsorption.
Ans. 2. Diarrhoea, IDA and folic acid deficiency.

Ans. 3.

Iron-deficiency anaemia.

Folate deficiency or vitamin B12 deficiency.

Bleeding, resulting from low vitamin K.

Oedema, which occurs in protein/calorie malnutrition.

Tropical sprue is seen in residents of, and visitors to, tropical areas and it tends to begin
with an acute episode of diarrhoea, fever and malaise before settling into a more chronic
presentation of steatorrhea, malabsorption, nutritional deficiency, anorexia, malaise and
weight loss. Folate deficiency is a significant part of the clinical picture.
80. A 35yo male is bitterly annoyed with people around him. He thinks that people are
putting ideas into his head. What is the single most likely dx?

a. Thought block
b. Thought insertion
c. Thought broadcasting
d. Thought withdrawal
e. Reference
Q. 1. What is the key?
Q. 2. In which disease you will find this feature?
Ans. 1. The key is B. Thought insertion.
Ans. 2. It is seen in schizophrenia.
Symptoms called disorders of thought possession may also occur in schizophrenia.
These include:
Thought insertion. This is when someone believes that the thoughts in their mind are not
their own and that they are being put there by someone else

.Thought withdrawal. This is when someone believes that thoughts are being removed
from their mind by an outside agency.
Thought broadcasting. This is when someone believes that their thoughts are being read
or heard by others.
Thought blocking. This is when there is a sudden interruption of the train of thought
before it is completed, leaving a blank. The person suddenly stops talking and cannot
recall what he or she has been saying.

81. A 10yo girl presents with hoarseness of the voice. She is a known case of bronchial
asthma and has been on oral steroids for a while. What is the most likely cause of
hoarseness?
a. Laryngeal candidiasis
b. Infective tonsillitis

c. Laryngeal edema
d. Allergic drug reaction
e. Ludwigs angina
Hoarseness may be a feature of laryngeal obstruction - so can be a warning of
impending airway obstruction.

This may occur in:

Infections - acute epiglottitis, diphtheria, croup, laryngeal abscess, laryngitis


Inflammation/oedema - airway burns, anaphylaxis, physical trauma, angio-oedema,
hereditary angio-oedema.
Vocal cord immobility - laryngeal nerve palsy (depending on the position of the cords) or
cricoarytenoid joint disease.
Immuno compromised states lead to fungal infections. These include HIV, Diabetes etc.
Patients taking long term steriods (inhalar or oral), antibiotics n those having vitb12 and
folic acid deficiecy are at a risk of having oral thrush.
Q. 1. What is the key?
Q. 2. What is the reason for this condition?
Ans. 1. The key is A. Laryngeal candidiasis.
Ans. 2. Steroids predisposes to fungal infection.
82. A lady with breast cancer has undergone axillary LN clearance. She develops arm
swelling after being stung by a bee. What is the most likely mechanism responsible for
the swelling?
a. Lymphedema
b. Cellulitis
c. Hypersensitivity reaction
d. DVT

e. Fluid retention
Q. 1. What is the key?
Q. 2. What is the reason for this condition?
Ans. 1. The key is A. Lymphoedema.
Ans. 2. Reason is compromised lymphatic drainage of arm due to axillary LN clearance.
83. A 34yo pt presents with 50% partial thickness burns. What should be the most
appropriate management?
a. IV fluids calculated from the time of hospital arrival
b. IV fluids calculated from the time of burn
c. No IVF
d. IV dextrose stat
e. Burns ointment
Q. 1. What is the key?
Q. 2. How the calculation of fluid is made?
Ans. 1. The key is B. IV fluids calculated from the time of burn.
Ans. 2. Resuscitation fluids required in the first 24 hours from the time of injury.
For adults: 3 ml (in partial thickness burn) of Hartmanns solution/kg body
weight/% total
Body surface area.
Half of this calculated volume is given in the first 8 hours and the other half is given over
the following 16 hours.

84. A 54yo man has recently been dx with moderate depression. He has hx of MI and is
suffering from insomnia. What is the drug of choice for him?
a. Citalopram
b. Lofepramine

c. ECT
d. Haloperidol
e. Diazepam
Ans. Key is A. Citalopram. [Citalopram is the antidepressant of choice post MI].
85. A man presented with cellulitis and swelling. He was started on flucloxacillin. What
other medication do you want to add?
a. Vancomycin
b. Penicillin
c. Metronidazole
d. Ceftriaxone
e. Amoxicillin
Q. 1. What is the key?
Q. 2. Is it justified to add this drug? If justified please mention why?
Ans. 1. The key is B. Penicillin.
Ans. 2. Custom and practice has traditionally combined the use of benzylpenicillin and
flucloxacillin in the management of hospitalised patients with cellulitis. In most cases this
is not seen as practical or necessary. Flucloxacillin covers both beta-haemolytic
streptococci and penicillinase-resistant staphylococci.
But for exam purpose, treatment of cellulitis = Benzylpenicillin + Flucloxacillin.
Drug of choice for cellulitis is flucloxacillin. IT Is sufficient alone. If needed to add
something, add penicilin ,, or add doxycycline if exposed to salt water,,, add
erythromycin if there is penicillin allergy, or add ciprofloxacin if exposed to fresh water.
86. A 24yo college student presents with nausea, vomiting, headache, neck stiffness
and a fever of 38.4C. What is the most appropriate empirical antibiotic to be started?
a. Ceftriaxone
b. Penicillin

c. Gentamicin
d. Tazobactam
e. Meropenem
Ans. The key is A. Ceftriaxone. [In OHCM-Cefotaxime <55yrs and Cefotaxime +
Ampicillin if age >55yrs].

One should start benzyl penicillin before admission. After admission, ideally cefotaxime
should be given as per new guidelines. But, as there was no option of cefotaxime, we
would go for ceftriaxone (also 3rd gen cephalosporin)
87. A man with prosthetic heart valve underwent hemicolectomy and after some days
complains of left hypochondriac pain, fever and has a systolic murmur. What is the next
inv to ascertain the cause of HF?
a. CT
b. Blood culture
c. ECG
d. MRI
e. Radioactive thyroid scan
Infective Endocarditis
Risk factors:
Cardiac conditions considered to increase a patient's risk of developing infective
endocarditis:
Valvular heart disease with stenosis or regurgitation. Valve replacement.
Structural congenital heart disease, including surgically corrected or palliated structural
conditions, but excluding: Isolated atrial septal defect. Fully repaired ventricular septal
defect. Fully repaired patent ductus arteriosus. Closure devices that are judged to be
endothelialized.

Investigations
Nonspecific signs of infection - eg, elevated CRP or ESR, leukocytosis, anaemia and
microscopic haematuria.
CXR: as part of the initial assessment.
Electrocardiogram is useful to detect the 10% of patients who will develop conduction
defects.
Blood cultures:
Should be taken prior to starting treatment in all cases. Meticulous aseptic technique is
required.
Echocardiography
Q. 1. What is the key?
Q. 2. What is the diagnosis?
Q. 3. Why have you made this diagnosis?
Q. 4. What are the important risk factors for this condition?
Ans. 1. The key is B. Blood culture.
Ans. 2. The diagnosis is infective endocarditis.
Ans. 3. Fever + new murmur = endocarditis until proven otherwise.
Ans. 4. Important risk factors: dermatitis, IV injections, renal failure, organ
transplantation, DM, post operative wound. Risk factors for abnormal valves: aortic or
mitral valve disease, tricuspid valve in IV drug users, prosthetic valves.

88. A 45yo man with posterior gastric ulcer presented with severe excruciating pain
which subsided after conservative treatment. 10 days later he developed swinging
pyrexia. US shows a collection in the peritoneum. What will be the most likely location of
the collection?
a. Hepatorenal pouch

b. Left paracolic gutter


c. Subphrenic
d. Pelvic cavity
e. Lesser sac

Ans. The key is E. Lesser sac.


89. A 23yo lady was prescribed with azithromycin 1gm for her chlamydial pelvic
infection. She has got a new boyfriend for the last 2 months. She has recently started
contraception to avoid conception. Which of the following contraception method will be
affected by azithromycin?
a. Barrier
b. IUCD
c. POP
d. COCP
Ans. None of them! Before it was thought that hepatic enzyme inhibitor drugs may affect
COCP but later it was established that actually there is no such significant effect. Only
drugs like rifampicin and rifambuin can cause this. No other antibiotic alters COCP
levels. Moreover, POP is not affected by any antibiotic other than rifampicin. Barrier
method has nothing to do with any antibiotic as its a mechanical method. IUCD has no
proved interaction with antibiotics.

90. An 11yo boy is being checked by the diabetic specialist nurse. His HbA1c was high
and he has been skipping meals recently. He has been unhappy at school. Which single
member of the clinical team would you refer him to next?
a. GP
b. Pediatrician

c. Dietician
d. Clinical psychologist
Ans. The key is D. Clinical psychologist. [Unhappy at school, skipping meals these are
psychological issue. He needs psychological counseling].
There was a discussion on plab forum that the answer should be pediatrician , but here
the problem is psychological. Had he missed any medication, he would have had to see
pediatrician.
91. A 35yo man who has served in the army presents with lack of interest in enjoyable
activities and feeling low. He doesnt feel like reading the news or watching movies as he
believes there is
violence everywhere. What is the most appropriate first line therapy?
a. Citalopram
b. Lofepramine
c. CBT
d. Chlordiazepoxide
e. Desensitization
Q. 1. What is the key?
Q. 2. What is the diagnosis?
Q. 3. What is the first line treatment?
Q. 4. Here why 1 line treatment is not considered?
st

Ans. 1. The key is C. CBT


Ans. 2. The diagnosis is depressive illness.
Ans. 3. In depressive illness 1 line therapy is SSRI
st

Ans. 4. In this patient abnormal thinking of presence of violence everywhere is the


trigger for his depression and in this situation CBT gives the best result.

[It is not post traumatic stress disorder as constant vivid flashbacks of the experience
which is the main feature of PTSD is absent here. In the given scenario depression has
a trigger of abnormal thought process that there is violence everywhere! So cognitive
behavioural therapy is the best treatment here (though in typical depression drug of
first choice is SSRI- according to samson note)].
(personally not sure about this one as pt has all the features of ptsd except the flashbacks.
Though, the answer would still be cbt )
92. A man has reducible bulge below the pubic tubercle, and on occlusion of the deep
inguinal ring, cough impulse is present. What is the most likely dx?
a. Direct inguinal
b. Indirect inguinal
c. Femoral
d. Spigelian
e. Lumbar
Q. 1. What is the key?
Q. 2. What are the points in favour of your answer?
Ans. 1. The key is C. Femoral hernia.
Ans. 2. It is just below the pubic tubercle that is just below the inguinal ligament.
Note: this question is a very bad recall as hernia below pubic tubercle is femoral and
cough impulse felt in occluded deep ring is seen in inguinal hernia. In femoral hernia
positive cough impulse is found in femoral ring.
Features of femoral hernia:
Below and lateral to the pubic tubercle
More common in women, particularly multiparous ones

High risk of obstruction and strangulation


Surgical repair is required
Other hernias :

Types of abdominal wall hernias:

Type of hernia

Details

Inguinal hernia

Inguinal hernias account for 75% of abdominal wall hernias.


Around 95% of patients are male; men have around a 25%
lifetime risk of developing an inguinal hernia.

Above and medial to pubic tubercle

Strangulation is rare

Femoral hernia

Below and lateral to the pubic tubercle

More common in women, particularly multiparous ones

High risk of obstruction and strangulation

Surgical repair is required

Umbilical
hernia

Symmetrical bulge under the umbilicus

Paraumbilical
hernia

Asymmetrical bulge - half the sac is covered by skin of the


abdomen directly above or below the umbilicus

Epigastric
hernia

Lump in the midline between umbilicus and the xiphisternum

Most common in men aged 20-30 years

Incisional
hernia

May occur in up to 10% of abdominal operations

Spigelian
hernia

Also known as lateral ventral hernia

Rare and seen in older patients

A hernia through the spigelian fascia (the aponeurotic layer


between the rectus abdominis muscle medially and the semilunar
line laterally)

Obturator
hernia

A hernia which passes through the obturator foramen. More


common in females and typical presents with bowel obstruction

Richter hernia

A rare type of hernia where only the antimesenteric border of the


bowel herniates through the fascial defect

93. A 48yo woman is admitted to ED with a productive cough and moderate fever. She
has often central chest pain and regurgitation of undigested food most times but doesnt
suffer from acid reflux. These symptoms have been present for the last 3.5 months
which affects both food and drink. A CXR shows an air-fluid level behind a normal sized
heart. What is the most likely dx?
a. Pharyngeal pouch
b. Hiatus hernia

c. Bulbar palsy
d. Achalasia
e. TB
Q. 1. What is the key?
Q. 2. What are the points in favour?
Ans. 1. The key is D. Achalasia.
Ans. 2. Points in favour: Aspiration pneumonia due to retained food and fluid in
oesophagus. Regurgitation of undigested food without acid reflux. Dysphagia for both
food and drink. Air-fluid level behind heart.
Why it is not hiatus hernia? Ans. Differentiating point:-i) In hiatus hernia usually you will
get associated GORD [particularly in sliding hernia which is the most common (99%).
However in rolling hernia there may be no reflux]. ii) In hiatus hernia x-ray chest may
demonstrate a retrocardiac gas-filled structure rather than a air-fluid level iii) Also in
hiatus hernia there may be nausea or vomiting.
Why it is not pharyngeal pouch? Ans. In pharyngeal pouch there will be halitosis.
Achlasia has been discussed before in detail.

94. A 64yo man has been waking up in the middle of the night to go to the bathroom. He
also had difficulty in initiating micturition and complains of dribbling. A dx of BPH was
made after a transrectal US guided biopsy and the pt was prepared for a TURP. What
electrolyte abnormality is highly likely due to this surgery?
a. Hypokalemia
b. Hypocalcemia
c. Hyperkalemia
d. Hyponatremia
e. Hypernatremia

Q. 1. What is the key?


Q. 2. Why this happens?
Ans. 1. The key is D. Hyponatremia.
Ans. 2. Absorption of fluid used for bladder irrigation to flush out blood clots and IV fluids
all may lead to hypervolaemia and dilutional hyponatremia.
95. A 56yo lady has developed severe right sided headache which worsens whenever
she comes to bright light since the last 4 days. She feels nauseated, but doesnt vomit.
What is the most likely
dx?
a. SAH
b. Brain tumor
c. Migraine
d. Cluster headache
e. Subdural headache
Q. 1. What is the key?
Q. 2. What is the type of the given case?
Q. 3. What are the points in favour of mentioned type?
Ans. 1. The key is C. Migraine.
Ans. 2. It is migraine without aura. There is presence of trigger (bright light)
Ans. 3. Criteria of migraine without aura: 5 headaches lasting 4-72 hours +
nausea/vomiting (or photo/phono-phobia) + any 2 of: i) unilateral ii) pulsating iii) worsen
by routine activity [OHCM, 9 edition, page-462].
th

It should be noted that as a general rule 5-HT receptor agonists are used in the acute
treatment of migraine whilst 5-HT receptor antagonists are used in prophylaxis. NICE
produced guidelines in 2012 on the management of headache, including migraines.

Acute treatment

first-line: offer combination therapy with an oral triptan and an NSAID, or an oral
triptan and paracetamol
for young people aged 12-17 years consider a nasal triptan in preference to an
oral triptan
if the above measures are not effective or not tolerated offer a non-oral
preparation of metoclopramide* or prochlorperazine and consider adding a nonoral NSAID or triptan

Prophylaxis

prophylaxis should be given if patients are experiencing 2 or more attacks per


month. Modern treatment is effective in about 60% of patients.
NICE advise either topiramate or propranolol 'according to the person's
preference, comorbidities and risk of adverse events'. Propranolol should be
used in preference to topiramate in women of child bearing age as it may be
teratogenic and it can reduce the effectiveness of hormonal contraceptives
if these measures fail NICE recommend 'a course of up to 10 sessions of
acupuncture over 5-8 weeks' or gabapentin
NICE recommend: 'Advise people with migraine that riboflavin (400 mg once a
day) may be effective in reducing migraine frequency and intensity for some
people'
for women with predictable menstrual migraine treatment NICE recommend
either frovatriptan (2.5 mg twice a day) or zolmitriptan (2.5 mg twice or three
times a day) as a type of 'mini-prophylaxis'
pizotifen is no longer recommend. Adverse effects such as weight gain &
drowsiness are common

*caution should be exercised with young patients as acute dystonic reactions may
develop

96. A 35yo man presented with hematuria, abdominal swelling and has a BP of 190/140.
What is the most diagnostic inv?
a. Cystoscopy
b. USG

c. CT
d. Renal biopsy
e. Urine analysis
Q. 1. What is the key?
Q. 2. What is the diagnosis?
Q. 3. What will be the USG findings to establish diagnosis in given case?
Ans. 1. The key is B. USG.
Ans. 2. The diagnosis is ADPKD.
Ans. 3. In given case patients age is 35. So the USG diagnostic criteria is: Age 18 39
yrs>3 unilateral or, bilateral cysts (here bilateral means if 1 + 1 it is enough).
Autosomal dominant polycystic kidney disease (ADPKD) is the most common inherited
cause of kidney disease, affecting 1 in 1,000 Caucasians. Two disease loci have been
identified, PKD1 and PKD2, which code for polycystin-1 and polycystin-2 respectively

ADPKD type 1

ADPKD type 2

85% of cases

15% of cases

Chromosome 16

Chromosome 4

Presents with renal failure earlier

The screening investigation for relatives is abdominal ultrasound:

Ultrasound diagnostic criteria (in patients with positive family history)

two cysts, unilateral or bilateral, if aged < 30 years


two cysts in both kidneys if aged 30-59 years

four cysts in both kidneys if aged > 60 years


97. A young man is brought to the ED after a RTA. His GCS on initial evaluation is 6.

What is the most appropriate next step?


a. CT
b. MRI
c. IV fluids
d. Skull XR
e. Secure airway
Ans. The key is E. Secure airway.
In a case of Road Traffic Accident. or any trauma, management starts with A- airway
(includes cervical immobility), B- (breathing), C (circulation) , D (disability) . ETT
SHOULD BE CONSIDERED IN ALL PATIENTS WITH GCS BELOW 8.
98. A 65yo man presented with frank hematuria. He has no other urinary symptoms.
What is the most appropriate next step that will lead to the dx?
a. IVU
b. US Abdomen
c. Cystoscopy
d. Mid-stream urine for culture
e. Transrectal US
Q. 1. What is the key?
Q. 2. What is the diagnosis?
Q. 3. What are the reasons for this diagnosis?
Q. 4. If there is painless haematuria in young (say 25-30yrs) what diagnosis will come
first?
Ans.1. Key is C. Cystoscopy.
Ans. 2. Bladder cancer.

Ans. 3. Age 65, asymptomatic haematuria.


Ans. 4. ADPKD [at the beginning there is very few or no symptoms]

Bladder CA has already been discussed in mcq # 1.


99. A 30yo woman had a gradual decrease of visual acuity since the last 3 years. Now
she has a disability due to very low vision. Whats the dx?
a. Glaucoma
b. Cataract
c. Macular degeneration
d. Retinitis pigmentosa
e. Keratitis
Q. 1. What is the key?
Q. 2. Why it is not the other given D/D s?
Ans. 1. The key is D. Retinitis pigmentosa.
Ans. 2. i) It is not angle closure glaucoma as angle closure glaucoma occurs usually
after the age of 50; In open angle glaucoma visual loss is not evenly gradual rather
occurs a bit suddenly at its later part. It is not cataract as cataract occurs usually in
elderly. In macular degeneration near blindness does not occur rather causes inability to
identify face or cannot read small prints; otherwise peripheral vision is not that
depressed. In keratitis will be pain, redness, photophobia and vision is ok.
Retinitis pigmentosa primarily affects the peripheral retina resulting in funnel vision
Features

night blindness is often the initial sign


funnel vision (the preferred term for tunnel vision)
fundoscopy: black bone spicule-shaped pigmentation in the peripheral retina,
mottling of the retinal pigment epithelium
Associated diseases

Refsum disease: cerebellar ataxia, peripheral neuropathy, deafness, ichthyosis


Usher syndrome

abetalipoproteinemia
Lawrence-Moon-Biedl syndrome
Kearns-Sayre syndrome
Alport's syndrome

100. A 27yo lady has had an uncomplicated pregnancy so far. She came to the hospital
2h ago after her water broke. The midwife is looking at her now. She has regular
contractions. P.V exam revealed 2cm dilated cervix. Vital signs are normal. What stage
of labour is she in?

a. Second stage

b. First stage

c. Latent stage

d. Third stage

e. Active phase

Ans. The key is B. First stage starts with softening of cervix with start of opening of

cervix and ends when cervix is fully dilated (i.e. 10 cm dilated). [There is nothing named
latent stage but latent phase which is up to 4cm dilatation. So, the preferred option is
first stage here].
Stages of Labour

First stage
The first stage begins with regular contractions (when the fetal presenting part has
descended into the true pelvis), or on admission to hospital with obvious signs of labour.
The first stage ends when the cervix is fully dilated (10 cm).
First stage can be divided into:

Latent or quiet phase: Contractions are not particularly painful and at 5- to 10-minute
intervals. Contractions become stronger with shorter intervals, although the cervix is still
dilating relatively slowly, with membranes possibly breaking later in this phase.

Active phase:Starts with the cervix 3-4 cm dilated and is associated with more rapid
dilatation normally at 0.5-1.0 cm/hour. Once the cervix is dilated to 9 cm, towards the
end of the active phase, contractions may be more painful and women may want to
push. Pushing is undesirable at this stage; there is the need to establish by vaginal
examination whether the cervix is fully dilated. During this time the fetal head descends
into the maternal pelvis and the fetal neck flexes.

While the length of established first stage of labour varies between women, first labours
last on average 8 hours (unlikely 18 hours). Second and subsequent labours last on
average 5 hours (unlikely 12 hours). However if the first stage does not appear to be
progressing, the cause needs to be determined.

Second stage:
This starts when the cervix is fully dilated and ends with the birth of the baby:

Contractions are stronger, occur at 2- to 5-minute intervals and last 60-90 seconds.
The fetal head descends deeply into the pelvis and rotates anteriorly so that the back of
the fetal head is behind the mother's symphysis pubis (98% of cases).
The second stage is said to be active once the baby is visible and the woman usually
also wants to assist what have become expulsive contractions by pushing.
The fetal head becomes more visible with each contraction until a large part of the head
can be seen.
The head is now born with first the forehead, then the nose, mouth and chin.
The head rotates to allow the shoulders to be born next, followed by the trunk and legs.
After this, the baby should start to breathe and to cry loudly.

Third stage:

This stage starts with the birth of the baby and ends with the delivery of the placenta and
membranes:

Separation of the placenta occurs immediately after birth due to forceful uterine
contractions along with retraction of the uterus, thus greatly reducing the size of the
placental bed.
It normally takes up to 5 minutes, but can take longer.
Haemorrhaging is prevented by the contraction of uterine muscle fibres closing off the
blood vessels that were supplying the placenta.
Without active management, after 10-20 minutes, separation is shown by a gush of
blood, prominence of the fundus in the abdomen and apparent lengthening of the
umbilical cord.
101. A 2yo boy fell off his tricycle and hurt his arm. He got up to start crying, but before
there was
any sound, he went pale, unconscious and rigid. He recovered after 1-2 mins but
remained pale.
After an hour he was back to normal. His mother says she was afraid he was going to
die, and
that he had a similar episode 3 months prior after falling down some steps. What single
inv is
indicated?
a. CT head
b. EEG
c. CBC
d. None
e. Skeletal survey
Q. 1. What is the key?
Q. 2. What is the diagnosis?
Ans. 1. The key is D. None.
Ans. 2. Diagnosis is breath holding spell.
102. A 29yo woman had just delivered a still born vaginally, following a major placental
abruption.
Choose the single most likely predisposing factor for developing PPH in this lady?
a. Retained product
b. DIC
c. Fibroid uterus
d. Uterine infection
e. Large placental site

Q. 1. What is the key?


Q. 2. What are the causes of this condition here?
Ans. 1. The key is B. DIC.
Ans. 2. Pregnancy itself is a risk factor for DIC. Placental abruption is a more common
cause of DIC.
Other causes of pregnancy related DIC are: eclampsia, retention of a dead fetus,
amniotic fluid embolism, retained placenta or bacterial sepsis.
103. A 28yo woman has delivered with rotational forceps after an 8h labor and 3h
second stage.
Choose the single most likely predisposing factor for PPH for this pt?
a. Atonic uterus
b. Cervical/vaginal trauma
c. Retained product
d. Preterm labor
e. Uterine infection
Ans. The key is B. Cervical/vaginal trauma. [complication of forceps delivery].
Primary PPH is the loss of greater than 500mL (defi nitions vary) in the first
24h after delivery

Causes: uterine atony (90%), genital tract trauma (7%), clotting disorders (3%)
Risks: Antenatal Previous PPH or retained placenta BMI>35kg/m2 Maternal
Hb<8.5g/dl at onset of labour Antepartum haemorrhage Multiparity 4+ Maternal age
35y+ Uterine malformation or fibroids A large placental site (twins, severe rhesus
disease, large baby) Low placenta, Overdistended uterus (polyhydramnios, twins)
Extravasated blood in the myometrium (abruption).
In labour Prolonged labour (1st, 2nd or 3rd stage) Induction or oxytocin use
Precipitant labour Operative birth or caesarean section. Book mothers with risk factors
for obstetric unit delivery.
Treatment: Give oxytocin 5U slowly IV for atonic uterus.
Attach oxygen, Give IV fluids, maintain systolic >100mmHg, Transfuse blood.
Is the placenta delivered? If it is, is it complete? If not, explore the uterus. If the
placenta is complete, put the patient in the lithotomy position with adequate analgesia and
good lighting. Check for and repair trauma.
If the placenta has not been delivered but has separated, attempt to deliver it by
controlled cord traction after rubbing up a uterine contraction. If this fails, ask an
experienced obstetrician to remove it under general anaesthesia.Beware renal shut down.
104. A 50yo man has had anterior resection of the rectum for carcinoma. He expressed
concerns
about control of post-op pain in discussions with the anaesthetist before surgery. What is
the
best management strategy?
a. Oral diclofenac
b. Oral codeine
c. IM morphine
d. IM dihydrocodeine
e. Ondansetron oral
Ans. The key is C. IM morphine. [Post operative pain is severe pain which needs strong
opioid analgesics].

Oral route will not be suitable for this patient as he is having a major abdominal surgery
so most probably he will be NPO post operatively. Dihydrocodeine is useful for mild to
moderate pain but since its a major surgery and the patient is himself worried about the
pain good analgesia should be maintained. Ondensetron is an anti emetic not an
analgesic. So the most suitable option here is IM Morphine as it is a strong analgesic
most suitable for severe pain.
105. A 73yo male presents with enlarged cervical nodes. He has had recurrent infections
over the last year. His conjunctiva is pale. Choose the single cell type you will find on the
blood film.
a. Granulocyte without blast cells
b. Myelofibroblasts
c. Plasma cells
d. Mature lymphocytes
Q. 1. What is the key?
Q. 2. What is the diagnosis?
Q. 3. What are the points in favour of your diagnosis?
Ans. 1. The key is D. Mature lymphocytes.
Ans. 2. The diagnosis is CLL.
Ans. 3. It is CLL because of his age (73 yrs). Other supportive features are cervical
lymphadenpathy, recurrent infections (mature but functionally defective lymphocytes),
and pale conjunctiva (anaemia).
It is a typical presentation of CLL with reurrent infections, symmetrical lymphadenopathy,
anemia. There can also be hepatosplenomegaly and thrombocytopenia leading to
patechae
On blood film there will be B cell lymphocytosis often with smudge cells. There are
mature but functionally impaired lymphocytes as they escape apoptosis. In bone marrow
there is lymphocytic replacement of bone marrow cells. Mainstay of treatment is
chemotherapy.
106. A 45yo lady has 10m hx of SOB. She is found to have irregularly irregular pulse
and loud P2 with fixed splitting and ejection systolic murmur in left 2nd ICS. What is the
probable dx?
a. TOF
b. ASD
c. VSD
d. PDA
e. CoA
Q. 1. What is the key?
Q. 2. What is the diagnosis?
Ans. 1. The key is B. Atrial septal defect.
Ans. 2. Diagnosis is ASD with atrial fibrillation. [i) atrial fibrillation = irregularly irregular
pulse. ii) ASD = SOB, fixed splitting with loud P2, ESM in pulmonary area]. This pictures
are of atrial septal defect itself though similar findings we get in pulmonary hypertension.
One should not misdiagnose SOB, ESM in pulmonary area and loud P2 as pulmonary
hypertension in the given case.

Fixed splitting is the clincher in this question.


VSD: Pansystolic murmur
PDA: machinery murmur

ASD is acyanotic condition. Ostium seccundum is the most common cause. There is left
to right shunting of blood leading to dyspnoea/heart failure eg at age 4060. There may
be pulmonary hypertension, cyanosis, arrhythmia, haemoptysis, and chest pain.
SIGNS: AF; raised JVP; wide, fixed split S2; pulmonary ejection systolic murmur
If an embolus from DVT of lower limb passes to the brain and causes ischemia it can
only pass from vein to artery through ASD.
INVESTIGATIONS: Echo is diagnositic
Tx: In children closure by surgery before 10yrs of age, In adults transcatheter closure is
now more common than surgery
107. A 5m baby present with recurrent vomiting. Mother noticed some of the vomitus is
blood
stained. Choose the single most likely inv?
a. Upper GI endoscopy
b. Barium meal
c. US
d. Colonoscopy
e. CT abdomen
Ans. The key is A. upper GI endoscopy.
Haematemesis (unless swallowed blood - eg, following a nosebleed or ingested blood
from a cracked nipple in some breast-fed infants) may suggest an important and
potentially serious bleed from the oesophagus, stomach or upper gut.
Projectile vomiting, non bilious: Pyloric stenosis
Bilious vomiting: Call for senior help, consider duodenal obstruction.
108. A 76yo is treated with HTN. He suffers from pain and redness at the MTP joint of
his right big toe. Which of the following anti-HTN cause this symptoms?
a. Losartan
b. Bendroflumethiazide
c. Ramipril
d. Bisoprolol
e. Verapamil
Q. 1. What is the key?
Q. 2. What is the diagnosis?
Q. 3. What is the cause of the disease?
Ans. 1. The key is B. Bendroflumethiazide
Ans. 2. Diagnosis is acute gout.
Ans. 3. Thiazide diuretics may cause hyperuricemia and thus precipitate acute gout.
Thiazide diuretics are contraindicated in gout!
In gout mostly large joints are involved like ankle, knee, foot. But small joints of hands
can also be involved.
It is caused by deposition of monosodium urate crystals in and near joints, precipitated,
for example, by trauma, surgery, starvation, infection or diuretics.
CAUSES: Hereditary, dietary purines, alcohol excess, diuretics, leukaemia, cytotoxics
(tumour lysis).

INVESTIGATIONS: Polarized light microscopy of synovial fluid: negatively birefringent


crystals (while those of pseudogout are positively birefringenent).... Serum urate may or
may not be raised. Punched out erosions on X Ray in advanced disease.
TREATMENT: ACUTE: NSAIDs (indomethacin), colchicine if NSAIDs are contra
indicated like peptic ulcer, heart disease. In renal failure both are problematic so use
steroids.
CHRONIC: Start if >1 attack in 12 months, tophi or renal stones. Use allopurinol. Aim is
plasma urate <0.3mmol/L
In acute attack allopurinol is CI as it exacerbates the attack, wait until 3 weeks after
acute attack to start allopurinol. But once on allopurinol no need to stop it during acute
attacks.
Febuxostat and probenicid are alternatives.
109. A 33yo male involved in a street fight presents with bruises and deformity in the
upper part of
his leg. XR shows fx of the neck of fibula. What is the single most associated nerve
injury?
a. Sciatic nerve
b. Gluteal nerve
c. Musculocutaneous nerve
d. Lateral peroneal nerve
e. Tibial nerve
f. Femoral nerve
Ans. is D. Lateral peroneal nerve. [Lateral peroneal nerve is other name of superficial
peroneal nerve].
110. A 35yo man presents with hx of dyspepsia. H.Pylori antibodies are negative. No
improvement is seen after 1m of tx. What is the next step?
a. Urea breath test
b. Gastroscopy
c. CT
d. MRI
Q. 1. What is the key?
Q. 2. What may be the D/D here?
Q. 3. At this age what are the indications of this procedure?
Ans. 1. Gastroscopy.
Ans. 2. Not responding to treatment D/D is: i) Zollinger Elison syndrome ii) Ca stomach
Ans. 3. Indications of gastroscopy in a 35 yo man (man of age <50yrs): i) Acute
symptoms with H/O previous episode (PUD) ii) Alarm features [weight loss, anaemia,
vomiting, hematemesis and melaena, dysphagia, palpable abdominal mass], fear of
cancer, evidence of organic disease.

Urgent specialist referral - two-week rule


If the patient has dyspepsia at any age with any of the following alarm symptoms:
Chronic GI bleeding.
Progressive unintentional weight loss.
Progressive dysphagia.
Persistent vomiting.
Iron-deficiency anaemia.
Epigastric mass.

[13]

Suspicious barium meal.


NB: patients aged 55 years or older with unexplained and persistent recent-onset
dyspepsia should be referred urgently for endoscopy

If age less than 55 and no alarm signs, try life style modifications and simple antacids. If
no improvement then do H.pylori testing (antibodies). If it is positive do eradication and
review in 4 weeks.
If resolved, no further action required.
If symptoms are not resolved, do urea breath test.
If it is positive, again eradication for H.pylori
If it is negative, do upper GI endoscopy
If the initial H.pylori testing was negative give PPIs or H2 blockers for 4 weeks and
review if symptoms resolve no action needed if they dont resolve do upper GI
endoscopy.
111. A 15yo male has bilateral ankle edema. His BP=110/70mmHg and urinalysis
shows protein++++.
What is the most likely dx?
a. HUS
b. IgA nephropathy
c. Membranous GN
d. Minimal change GN
e. Nephrotic syndrome
Q. 1. What is the key?
Q. 2. What are the points in favour of your diagnosis?
Q. 3. What is the treatment?
Ans. 1. The key is D. Minimal change disease.
Ans. 2. Points in favour: i) Age 15 ii) Ankle oedema iii) Normotension iv) Heavy
proteinuria.
Ans. 3. Treatment of choice is steroid (prednisolone). Failure of steroid or frequent
relapse (>3) cyclophosphamide.
Most common cause of nephrotic in children is minimal change disease. There will be
hypoalbuminemia and peripheral edema too. Electron microscopy shows effacement of
podocyte foot processes.. MCD has albumin selective proteinuria. Treatment is with
steroids.
IgA nephropathy is nephritic and will also show HTN and microscopic hematuria and
follows upper resp tract infection.
Membranous GN also presents as nephrotic but age and since MCD is most common
we choose MCD.
Nephrotic syndrome itself is not a diagnosis.
112. A 28yo man has developed a red, raised rash on trunk after playing football. His
PMH shows he had childhood asthma. The rash is becoming increasingly itchy. What is
the most appropriate tx?
a. Oral chlorpheneraime
b. Oral amoxicillin
c. IM adrenaline
d. Nebulized salbutamol
e. Histamine
Q. What is the key?

Q. 2. What is the diagnosis?


Ans. 1. The key is A. Oral chlorpheneramine.
Ans. 2. Diagnosis is Atopy (allergy).
Since it is an allergic reaction only 2 options are suitable. A & C. IM adrenaline is used in
anaphylactic shock which can occur due to allergy. But this is just a mild allergic reaction
here so anti histamine (chlorpheneramine) is adequate.
Anaphylaxis presents with:
Sudden onset and rapid progression of symptoms.
Life-threatening airway and/or breathing and/or circulation problems
Patient will be mostly in shock.
Mostly commonly caused by certain foods like peanuts, pulses, fish, eggs. Also by
venom (bee,wasps) and drugs like antibiotics.
Treatment:ABCDE, Oxygen, IM Adrenaline. <6yrs0.15ml, 6-12yrs 0.3ml, >12 yrs 0.5ml
1:1000
113. A 72yo man has been advised to have antibiotic prophylaxis for some years now
before dental tx. He has never experienced chest pain. Three weeks ago, he noticed
breathlessness on exertion and for one week he had orthopnea. His pulse is normal.
What is the most probable dx?
a. Aortic regurgitation
b. Ischemic mitral regurgitation
c. Mitral valve prolapse
d. Pulmonary stenosis
e. Mitral valve stenosis
Ans. The kay is E. Mitral valve stenosis.
The patient has mitral stenosis or Aortic regurgitation. he is given prophylaxis for
infective endocarditis. According to OHCM, such prophylaxis has no benefit and should
not be given.
RISK FACTORS for IE: aortic or mitral valve disease; tricuspid valves in IV drug users;
coarctation; patent ductus arteriosus; VSD; prosthetic valves
Mitral Stenosis: Presentation: dyspnoea; fatigue; palpitations; chest pain; systemic
emboli; haemoptysis; chronic bronchitis-like picture
CAUSES: Rheumatic, congenital, mucopolysaccharidoses, endocardial fibroelastosis,
malignant carcinoid, prosthetic valve.
SIGNS: Malar flush on cheeks (due to cardiac output); low-volume pulse; AF common;
tapping, non-displaced, apex beat (palpable S1). On auscultation: loud S1; opening snap
(pliable valve); rumbling mid-diastolic murmur (heard best in expiration, with patient on
left side
ECG shows P-mitrale ECHO is diagnostic. CXR: left atrial enlargement (double
shadow in right cardiac silhouette)
TREATMENT: balloon valvuloplasty (if pliable, non-calcified valve), open mitral
valvotomy
or valve replacement.
Complications: Pulmonary hypertension, emboli, pressure from large LA on local
structures, eg hoarseness (recurrent laryngeal nerve), dysphagia (oesophagus),
bronchial obstruction; infective endocarditis

AORTIC REGUGITATION: CAUSES Acute: Infective endocarditis, ascending aortic


dissection,
chest trauma.
SYMPTOMS: Exertional dyspnoea, orthopnoea, and paroxysmal nocturnal dyspnoea.
palpitations, angina, syncope, CCF
Signs: Collapsing (water-hammer) pulse (p40); wide pulse pressure; displaced,
hyperdynamic apex beat; high-pitched early diastolic murmur (heard best in expiration,
with patient sitting forward).
The diagnosis here is mitral stenosis because of the normal pulse. I think the
information in the question is too deficient for such a disease and diagnosis!

FEVER + NEW MURMUR IS ENDOCARDITIS UNTIL PROVEN OTHERWISE


114. A 37yo woman presents with fatigue. Exam: angular stomatitis, no koilonychea.
Choose the single cell type you will find on the blood film.
a. Macrocytes
b. Microcytes
c. Granulocytes wthout blast cells
d. Blast cells
Q. 1. What is the key?
Q. 2. What is the cause here?
Q. 3. What are the points in favour of mentioned cause?
Ans. 1. The given key is A. Macrocytes.
Ans. 2. The cause here is VIT. B12 or folate deficiency.
Ans. 3. Points in favour of Vit. B12 or folate deficiency: i) fatigue (anaemia) ii) angular
stomatitis (can be seen in Vit. B12 or folate deficiency) iii) absence of koilonychea is
against IDA.
SIGNS in ANEMIA:
Koilocychia (spoon shaped nails) iron deificiency anemia
atrophic glossitis in iron def.
post cricoid webs (plummer vinson syndrome)
Angular stomatitis (cheilosis) in both vit B12 and iron def.
glossitis (beefy-red sore tongue) Vit. B12 def.
115. A 4yo boy with a febrile convulsion lasting eight minutes has been given IV
lorazepam to control them. What is the single most likely serious side effect?
a. Amnesia
b. Anaphylactic shock
c. Apnea
d. Bronchospasm
e. Cardiac arrhythmia
Ans. The key is C. Apnoea.
Due to respiratory depression caused by benzodiazepines. They can also cause
amnesia but it wont be in acute setting.
116. A 4wk girl has been dx of having breast milk jaundice. She is otherwise well. What
is the single most appropriate management?
a. Continue breastfeeding

b. Exchange transfusion
c. Increase fluid intake
d. Phototherapy
e. Stop breastfeeding
Q. 1. What is the key?
Q. 2. What is breast milk jaundice?
Q. 3. What type of hyperbilirubinemia occurs in breast milk jaundice?
Q. 4. What is the cause of this jaundice?
Ans. 1. The key is A. Continue breast feeding.
Ans. 2. If jaundice lasts past the first week of life in a breastfed baby who is otherwise
healthy, the condition may be called "breast milk jaundice."
Ans. 3. Unconjugated hyperbilirubinaemia.
Ans. 4. Cause of breast milk jaundice: factors in a mother's milk that help a baby absorb

bilirubin from the intestine.


Hyperbilirubinaemia (<200mol/L) after 24h is usually physiological
Visible jaundice within 24h of birth is always abnormal. Causes: Sepsis or Rhesus
haemolytic disease: +ve direct Coombs test.
Prolonged jaundice (not fading after 14 days) Causes: breastfeeding; sepsis, (UTI &
TORCH, hypothyroidism; cystic fibrosis; biliary atresia if conjugated and pale stools.
If the jaundice is between 1-14 days no intervention is needed unless it is severe in which
case phototherapy or exchange transfusion is done.
117. A 12yo girl when playing in the garden accidentally stepped on a hive and was
bitten several
times. She has numerous wheals on her body and complains of severe itching. What is
the single
most appropriate management?
a. Oral antihistamine
b. IV antihistamine
c. IM adrenaline
d. Oral ciprofloxacin
e. Reassurance
Ans. The given key is C. IM adrenaline which is a wrong key. The correct answer is A.
Oral antihistamine.
Followings are the indications of adrenaline in anaphylaxis:
1. Horseness of voice
2. Wheeze
3. Shortness of breath
4. Shock
5. Stridor
6. Swelling of the tongue and cheek
7. Facial swelling

Consider anaphylaxis when there is compatible history of rapid-onset severe allergictype reaction with respiratory difficulty and/or hypotension, especially if there are skin
changes present and the treatment of anaphylaxis is IM adrenaline not anti histamine
Adrenaline can be repeated after 5mins.

And since she is bitten by bee several times it a risk factor for anaphylaxis.
118. A term baby born to a 30yo woman of blood group A-ve develops severe jaundice
within the
first 24h of birth. What is the most likely dx?
a. Hereditary spherocytosis
b. G6PD
c. ABO incompatibility
d. Rh incompatibility
e. Physiological jaundice
Ans. The key is D. Rh incompatibility.
As mentioned in the Q116 neonatal jaundice within 24hrs of birth could be either
because of sepsis or Rh incompatibility.
Mother is always Rh- and the baby is Rh +. Fetal antigen crosses the placenta and the
mother produces the antibodies against the antigen. Which cross the placenta in
subsequent pregnancies as a result of secondary response (greater in magnitude) and
cause hemolysis of the fetal blood.
ABO incompatibility: (mother O; baby A or B, or mother A and baby B, or vice
versa) DCT +ve in 4%; indirect Coombs +ve in 8%. Maternal IgG anti-A or antiB haemolysin is always present
119. A 4yo girl is found to have bounding pulse and continuous machinery murmur.
What is the most probable dx?
a. TOF
b. ASD
c. VSD
d. PDA
e. CoA
Ans. The key is D. PDA.

Machinery murmur is the clincher for PDA.


VSD has a pansystolic murmur
ASD ejection systolic and fixed splitting
PDA PRESENTATION: Usually asymptomatic. Acyanotic disease. A large-shunt PDA
may cause lower respiratory tract infection as well as feeding difficulties and poor growth
during infancy, with failure to thrive because of heart failure.
ECHO IS DIAGNOSTIC
MANAGEMENT: Indomethacin can be used but not useful in term infants. Closure is
indicated if the patient is symptomatic at any stage of life or if asymptomatic but with
great left heart load.
Surgery is used where non surgical method can not be used. In asymptomatic infant we
wait till 1 yr for spontaneous closure of PDA if that does not occur it can be closed by
surgery at any time.
In preterms indomethacin or ibuprofen may be used.
Most common complication is infective endocarditis.

120. A 12yo child with episodes of sudden bluish discoloration and brief loss of
consciousness. Exam: clubbing, central cyanosis, systolic thrill with systolic ejection
murmur in 2nd left ICS. What is the most probable dx?
a. TOF
b. ASD
c. VSD
d. PDA
e. CoA
Ans. The key A. TOF.
ASD, VSD, PDA are all acyanotic congenital heart diseases. TOFF is the most common
cyanotic congenital heart disease that survives to adulthood.
Typical features:
1 Ventricular septal defect (VSD)
2 Pulmonary stenosis (most imp feature)
3 Right ventricular hypertrophy
4 The aorta overriding the VSD
During a hypoxic spell, the child becomes restless and agitated and may cry
inconsolably. Toddlers may squat, which is typical of TOF. Clubbing, difficulty of feeding,
failure to thrive all are features.
Chest Xray Shows BOOT SHAPED HEART. Echo is also done.
In acute stage give O2, place the child in knee chest position, give morphine. Surgery is
required within 1st yr of life otherwise mortality is 95%.
121. An 8yo child who is tall for his age and has a refractory error for which he wears
glasses has
presented with severe crushing chest pain. What is the most likely dx?
a. Fragile X syndrome
b. Prader-willi syndrome
c. DiGeorge syndrome
d. Marfans syndrome
Q. 1. What is the key?
Q. 2. What is the cause of this severe crushing chest pain?
Q. 3. What are the most common cardiac abnormalitis found in this disease?
Ans. 1. The key is D. Marfans syndrome.
Ans. 2. Cause of severe crushing chest pain may be aortic dissection.
Ans. 3. Most common cardiac abnormalities in Marfans syndrome are: dilatation of the
aorta and mitral regurgitation.
Marfans syndrome diagnosis:
Major criteria (diagnostic if >2): Lens dislocation (ectopia lentis) aortic dissection or
dilatation; dural ectasia; skeletal features: arachnodactyly (long spidery fingers),
armspan > height, pectus deformity, scoliosis, pes planus. Minor signs: Mitral valve
prolapse, high-arched palate, joint hypermobility. Diagnosis is clinical.
DANGER IS AORTIC DISSECTION. Surgery is done when aorta >5cm
Can also cause pneumothorax.
122. A 4yo child presents with pain of spontaneous onset in his knee of 2 days duration.
He has
developed mild fever in the 2nd day. He can walk but has a limp. Exam: painful restriction
in the

right hip. What is the most probable dx?


a. Osteosarcoma
b. Septic arthritis
c. TB arthritis
d. Exostosis
e. Osteomyelitis
Q. 1. What is the key?
Q. 2. What are the points in favour of your diagnosis?
Ans. Given key is E. Osteomyelitis which is a wrong key. The correct answer is B. Septic
arthritis.
Ans. Points in favour of diagnosis: i) Pain in joints (knee and hip) ii) Fever iii) Painful
restricted movement of joint.
Not sure about the correct answer here. But i think osteomyelitis.
Osteomyelitis mostly has a primary source of infection via which the infection spreads to
bone.
PRESENTATION: Pain of gradual onset over the course of a few dayswith
tenderness,
warmth, and erythema at the affected part; unwillingness to move. Vertebrae and distal
femur mostly affected.
Diagnosis: FBC, ESR, CRP, blood culture. Bone biopsy and culture is gold standard.
Staph aureus (MR the most common organism found.)
Treatment Drain abscesses and remove sequestra by open surgery. Antibiotics:
vancomycin 1g/12h and cefotaxime 1g/12h IVI until the organism and its sensitivities are
known. Fusidic acid or clindamycin can also be used.
Septic Arthritis: Exclude septic arthritis in any acutely inflamed joint, as it can destroy a
joint
in under 24h. Knee & hip joint are most commonly involved.
Risk factors for septic arthritis include:
Increasing age
Diabetes mellitus
Rheumatoid arthritis
Joint surgery
Hip or knee prosthesis
Skin infection in combination with joint prosthesis
Infection with HIV
Diagnosis: Urgent joint aspiration for synovial fluid microscopy and culture is
the key investigation. The joint is usually swollen, warm, tender and exquisitely painful
on movement.
Flucloxacillin or clindamycin as empirical treatment.
123. A man with anterior resection and end to end anastomosis done complains of
severe pain in the chest and abdominal distension. What is the most appropriate inv
likely to review the cause this deterioration?
a. XR abdomen
b. Exploratory laparoscopy
c. CT
d. US
e. Laparotomy
Ans. The key is E. Laparotomy. [diagnostic and therapeutic].

124. Pt with hx of alcoholism, ataxic gait, hallucinations and loss of memory. He is given
acamprosate. What other drug can you give with this?
a. Chlordiazepoxide
b. Thiamine
c. Diazepam
d. Disulfiram
e. Haloperidol
Q. 1. What is the key?
Q. 2. What is the diagnosis?
Q. 3. What are the points in favour of diagnosis?
Ans. 1. The key is B. Thiamine.
Ans. 2. The diagnosis is Wernickes encephalopathy.
Ans. 3. Points in favour of diagnosis: i) history of alcoholism ii) ataxic gait iii)
hallucination iv) memory loss.
Thiamine (vitamin B1) deficiency with a classical triad of 1 confusion 2 ataxia (widebased gait) and 3 ophthalmoplegia (nystagmus, lateral rectus or conjugate gaze
palsies). Always
consider this diagnosis in alcoholics: it may also present with memory disturbance.
TREATMENT: early treatment is essential to prevent progression to the irreversible
Korsakoff syndrome. Alcoholics can present with hypoglycemia so make sure you give
thiamine BEFORE glucose as glucose can precipitate wernickes encaph.
125. A 35yo male builder presented with sudden onset of severe abdominal pain. He
was previously fit and well other than taking ibuprofen for a long term knee injury. On
examination he is in severe pain, pulse=110bpm, BP=110/70mmHg and has a rigid
abdomen. What is the most likely dx?
a. Biliary peritonitis
b. Ischemic colon
c. Pancreatic necrosis
d. Perforated diverticulum
e. Perforated peptic ulcer
Ans. The key is E. Perforated peptic ulcer. [NSAIDs induced perforation].

Peritonitis (Perforation of peptic ulcer/duodenal ulcer, diverticulum, appendix,


bowel, or gallbladder) Signs: prostration, shock, lying still, +ve cough test tenderness (
rebound/percussion pain), board-like abdominal rigidity, guarding and no bowel sounds.
Erect CXR may show gas under the diaphragm.
NB: acute pancreatitis causes these signs, but does not require a laparotomy
so dont be caught out and always check serum amylase
126. A woman 5 days post-op for bilateral salphingo-oopherectomy and abdominal
hysterectomy has developed abdominal pain and vomiting a/w abdominal distension and
cant pass gas. No bowel sounds heard, although well hydrated. What is the most
appropriate next step?
a. XR abdomen
b. Exploratory laparoscopy
c. CT
d. USG
e. Barium enema

Q. 1. What is the key?


Q. 2. What is the diagnosis?
Q. 3. What are the causes of it?
Q. 4. What is the management?
Ans. 1. The key is A. X-ray abdomen.
Ans. 2. The diagnosis is paralytic ileus.
Ans. 3. Causes of paralytic ileus: i) electrolyte imbalance ii) gastroenteritis iii)
appendicitis iv) pancreatitis v) surgical complications and vi) certain drugs.
Ans. 4. Management of paralytic ileus: i) nil by mouth ii) nasogastric suction to alleviate
the distension and remove the obstruction.
Bowel sounds are absent in paralytic ileus But bowel sounds are exaggerated in
mechanical obstruction.

Ileus and incomplete small bowel obstruction can be conservatively managed while
strangulation large bowel obstruction requires surgery.
CT can confirm the level of obstruction.
127. A 30yo man complains of hoarseness of voice. Exam: unilateral immobile vocal
cord. What is the most probable dx?
a. Graves disease
b. Hematoma
c. Unilateral recurrent laryngeal nerve injury
d. External laryngeal nerve injury
e. Tracheomalacia
Ans. The key is C. unilateral recurrent laryngeal nerve injury.
Causes: 30% are cancers (larynx in ~40%; thyroid, oesophagus, hypopharynx,
bronchus, or malignant node). 25% are iatrogenic, ie after parathyroidectomy. Other
causes: CNS disease (polio; syringomyelia); TB; aortic aneurysm;
Symptoms: Symptoms of vocal cord paralysis are:
Hoarseness with breathy voice with a weak cough.
Repeated coughing/aspiration (weak sphincter + supraglottic sensation).
Exertional dyspnoea (glottis is too narrow to allow much air flow).
Nerve damaged with injury of superior thyroid artery: External laryngeal nerve
Nerve damaged with injury to inferior thyroid artery: Recurrent laryngeal nerve

128. A 38yo woman has delivered after an induced labor which lasted 26h. choose the
single most likely predisposing factor for postpartum hemorrhage?
a. Atonic uterus
b. Cervical/vaginal trauma
c. Rupture uterus
d. Fibroid uterus
e. Age of mother
Ans. The key is A. Atonic uterus.
Primary PPH is the loss of greater than 500mL (definitions vary) in the first 24h after
delivery

Causes: uterine atony (90%), genital tract trauma (7%), clotting disorders (3%)

Risks: Antenatal Previous PPH or retained placenta BMI>35kg/m2 Maternal


Hb<8.5g/dl at onset of labour Antepartum haemorrhage Multiparity 4+ Maternal age
35y+ Uterine malformation or fibroids A large placental site (twins, severe rhesus
disease, large baby) Low placenta, Overdistended uterus (polyhydramnios, twins)
Extravasated blood in the myometrium (abruption).
In labour Prolonged labour (1st, 2nd or 3rd stage) Induction or oxytocin use
Precipitant labour Operative birth or caesarean section. Book mothers with risk factors
for obstetric unit delivery.
Treatment: Give oxytocin 5U slowly IV for atonic uterus.
Attach oxygen, Give IV fluids, maintain systolic >100mmHg, Transfuse blood.
Is the placenta delivered? If it is, is it complete? If not, explore the uterus. If the
placenta is complete, put the patient in the lithotomy position with adequate analgesia and
good lighting. Check for and repair trauma.
If the placenta has not been delivered but has separated, attempt to deliver it by
controlled cord traction after rubbing up a uterine contraction. If this fails, ask an
experienced obstetrician to remove it under general anaesthesia.Beware renal shut down.
129. A 32yo woman in tears describing constant irritability with her 2 small children and
inability to relax. She describes herself as easily startled with poor sleep and disturbed
nightmares following a house fire a year ago, while the family slept. What is the single
best tx?
a. Rassurance
b. Relaxation therapy
c. Quetiapine
d. Lofepramine
e. Fluoxetine
Q. 1. What is the key
Q. 2. What is the diagnosis?
Q. 3. What are the points in favour of your diagnosis?
Ans. 1 The key is E. Fluoxetine. The key is probably a wrong key. Likely correct key is B.
Relaxation therapy
Ans. 2. The diagnosis is post traumatic stress disorder.
Ans. 3. Points in favour of PTSD: i) H/O stressor (house fire a year ago) ii) Nightmares of
the stressor iii) Hyper arousal (very anxious and inability to relax (leading to irritability) iv)
associated depression (poor sleep, tearful).
Note: Fluoxetin and peroxetin are the drugs of choice in PTSD. CBT is the nonpharmacological treatment.
PTSD:
Symptoms: Fearful; horrified; dazed Helpless; numb, detached Emotional
responsiveness
Intrusive thoughts Derealization Depersonalization Dissociative amnesia Reliving
of events Avoidance of stimuli Hypervigilance Lack of Concentration Restlessness
Autonomic arousal: pulse; BP; sweating Headaches; abdo pains
Signs: Suspect this if symptoms become chronic, with these
signs (may be delayed years): difficulty modulating arousal; isolated-avoidant modes of
living; alcohol abuse; numb to emotions and relationships; survivor guilt; depression;
altered world
view in which fate is seen as untamable, capricious or absurd, and life can yield no
meaning

or pleasure.
Treatment: Watchful waiting for mild cases.
For severe cases: CBT or eye movement desensitization and reprocessing is done.
Drug treatment is not recommended but in case it is needed prescribe mirtazapine or
paroxetine.
So i agree in this question it is PTSD and B should be the answer.
130. A 22yo woman with longstanding constipation has severe ano-rectal pain on
defecation. Rectal exam: impossible due to pain and spasm. What is the most probable
dx?
a. Anal hematoma
b. Anal fissure
c. Anal abscess
d. Protalgia fugax
e. Hemorrhoids
Ans. The key is B. Anal fissure.

Anal fissures: Acute If less than 6weeks, >6wks chronic.


Causes: Most are due to hard faeces. Spasm may constrict the inferior rectal artery,
causing ischaemia, making healing difficult and perpetuating the problem.
History of constipation almost always present. Examination is almost impossible due to
severe pain.
Treatment: Acute: Increase fluid intake, fiber diet. Bulk forming laxatives are first line.
Topical anesthetics are used. Lactulose can be tried.
Chronic: Topical GTN is the first line and mainstay of treatment. If ineffective for
>8wks surgical referral for use of botulinum toxin.
131. A 20yo student attends the OPD with complaint of breathlessness on and off,
cough and sputum. His sleep is disturbed and skin is very dry in flexural areas of the
body. Exam: tachypnea, hyperresonant percussion and wheezing on auscultation. What
is the most likely dx?
a. Extrinsic allergic alveolitis
b. Asthma
c. Wegeners granulomatosis
d. COPD
e. Cystic fibrosis
Q. What is the key?
Q. What are the diagnostic criteria?
Ans. The key is B. Asthma.
Ans. 2. Diagnostic criteria of asthma: i) Airway hyper-responsiveness to certain stimuli ii)
Recurrent variable airflow limitation usually reversible iii) presents as wheezing,
breathlessness, chest tightness and cough.
ASTHMA.
Symptoms: Dyspnea, wheeze, cough (with or without sputum), chest tightness (4 most
important) particularly if symptoms are worse at night or early morning and in response
to certain triggers like cold, exercise, allergens. Symptoms exacerbated by use of

NSAIDs and Beta blockers. Mostly there is history of allergy (atopy) as in this question
there is history of dry skin. Try to find the precipitating factor.
Signs Tachypnoea; audible wheeze; hyperinflated chest; hyperresonant percussion
note; reduced air entry ; widespread, polyphonic wheeze.

Management: CHRONIC (LONG TERM)

132. A pt with thought disorder washes hands 6x each time he uses the toilet. What is
the best
management?
a. Psychodynamic therapy
b. CBT
c. Antipsychotics
d. Refer to dermatology
e. Reassure
Q. 1. What is the key?
Q. 2. What is the diagnosis?
Ans. 1. The key is B. CBT.
Ans. 2. The diagnosis is obsessive compulsive disorder.
OCD:
Compulsions are senseless, repeated rituals. Obsessions are stereotyped, purposeless
words, ideas, or phrases that come into the mind.
Repetitive behavior and an urge to do it.
Treatment: CBT is first line. Clomipramine (start with 25mg/day PO) or SSRIs (eg
fluoxetine)
133. A 25yo woman presented to her GP on a routine check up. Upon vaginal exam, she
was fine except for finding of cervical ectropion which was painless but mild contact
bleeding on touch. What is the next management?
a. Endometrial ablation
b. Cervical smear
c. Colposcopy
d. Antibiotics
e. Vaginal US
f. Pack with gauze and leave to dry
Q. 1. What is the key?
Q. 2. Points in favour of key.
Ans. 1. The key is D. Antibiotics. WRONG KEY!
Ans. 2. Points in favour of antibiotic: Ectropion and contact bleeding can occur in
infection. In the given case swab is taken to establish or rule out infection. As this is not
in options then the best response is antibiotics. If improves with antibiotics then repeat
smear in 6 months.
There is a red ring around the os because the endocervical epithelium has extended its
territory over the paler epithelium of the ectocervix. Ectropions extend temporarily under
hormonal influence during puberty, with the combined Pill, and during pregnancy. As
columnar epithelium is soft and glandular, ectropion is prone to bleeding, to excess
mucus production, and to infection. Treatment: Once a normal cervical smear has
been confirmed, it is actively managed only if there are symptoms. After stopping any
oestrogen-containing contraceptive, treatment options are controversial but include
diathermy, cryotherapy, surgery with laser treatment and microwave therapy.
SO THE CORRECT ANSWER IS B.
134. A 32yo had a normal vaginal delivery 10 days ago. Her uterus has involuted
normally. Choose the single most likely predisposing factor for PPH?

a. Retained product
b. DIC
c. Uterine infection
d. Von Willebrand disease
e. Primary PPH
Q. 1. What is the key?
Q. 2. What type of PPH it would be?
Ans. 1. The key is C. uterine infection.
Ans. 2. Secondary PPH
Loss of >500ml blood in the first 24hrs after delivery is PRIMARY PPH.
Secondary PPH: This is excessive blood loss from the genital tract after 24h from
delivery. It usually occurs between 5 and 12 days and is due to infections (most common
cause) (endometritis) or retained placenta.
Look for history of extended labour, difficult third stage, ragged placenta, PPH.
Symptoms: Abdominal pain. Offensive smelling lochia. Abnormal vaginal bleeding PPH. Abnormal vaginal discharge. Dyspareunia. Dysuria.

Signs: are those of sepsis. Tachycardia, fever, rigors, suprapubic tenderness.


Treatment: For endometritis: IV antibiotics if there are signs of severe sepsis. If less
systemically unwell, oral treatment may be sufficient. Piperacilin and tazobectum may be
used.
If RPOC are suspected, elective curettage with antibiotic cover may be required. Surgical
measures should be undertaken if there is excessive or continuing bleeding, irrespective
of ultrasound findings
135. A 37yo man slipped while he was walking home and fell on his out stretched hand.
He complains of pain in the right arm. XR showed fx of the head of radius. What is the
single most associated
nerve injury?
a. Radial nerve
b. Musculocutaneous nerve
c. Median nerve
d. Ulnar nerve
Q. 1. What is the key?
Q. 2. What is the root value?
Ans. 1. The key is A. Radial nerve.
Ans. 2. Root value of radial nerve: C5,6,7,8 and T1.

136. A butcher stabbed accidently his groin. He bled so much that the towel was soaked
in blood and BP=80/50mmHg, pulse=130bpm. What % of circulatory blood did he lose?
a. <15%
b. 15-30%
c. 30-40%
d. 40-50%
e. >50%
Q. 1. What is the key?
Q. 2. What is the classification of blood loss according to vital sign?
Ans. 1. The key is C. 30-40%
Ans. 2. Hypovolemic shock Classification:
1. Class 1 up to 15% of blood volume lost: pulse <100; systolic BP normal; pulse
pressure normal; Respiratory rate 14-20; urine output greater than 30 ml/hour.
2. Class 2 15%-30% blood volume lost: pulse 100-120; systolic blood pressure
normal; pulse pressure decreased; respiratory rate 20-30; urine output 20-30 ml/hour.
3. Class 3 30%-40% blood volume lost: pulse 120-140; systolic BP decreased; pulse
pressure decreased, respiratory rate 30-40; urine output 5-15 ml/hr
4. Class 4, blood loss of greater than 40%: pulse rate >140; systolic BP decreased;
pulse pressure decreased respiratory rate >35; urine output negligible.

137. A 67yo man presents with palpitations. ECG shows an irregular rhythm and
HR=140bpm. He is otherwise stable, BP=124/80 mmHg. What is the most appropriate
management?
a. Bisoprolol
b. ACEi
c. Ramipril
d. Digoxin
Ans. The key is A. Bisoprolol.
The patient has Atrial fibrillation. Irregularly irregular pulse and tachycardia.
Agents used to control rate in patients with atrial fibrillation
beta-blockers
calcium channel blockers
digoxin (not considered first-line anymore as they are less effective at controlling
the heart rate during exercise. However, they are the preferred choice if the
patient has coexistent heart failure)
Agents used to maintain sinus rhythm in patients with a history of atrial fibrillation
sotalol
amiodarone
flecainide
others (less commonly used in UK): disopyramide, dofetilide, procainamide,
propafenone, quinidine
TREATMENT CHOICE:

In the given question since the patient is above the age of 65 so rate control is
done! For which either a beta blocker or calcium channel blocker is used!
138. A 78yo man is depressed after his wifes death. He has been neglecting himself.
His son found him in a miserable state when he went to visit. The son cant deal with his
father. What is the appropriate management?
a. Voluntary admission to psychiatry ward
b. Hand over to social worker
c. Request son to move in with father
d. Send pt to care home
Ans. The key is A. Voluntary admission to psychiatry ward.
139. An old alcoholic presents with cough, fever, bilateral cavitating consolidation. What
is the most probable cause?
a. Gram +ve diplococcic
b. Coagulase +ve cocci
c. Gram ve cocci
d. AFB
e. Coagulase ve cocci
Q. 1. What is the key?
Q. 2. What is the organism?
Ans. 1. The key is B. Coagulase +ve cocci.
Ans. 2. Name of organism is Staphylococcus aureus.

Legionella: hotel stay, foreign travel, flu like symptoms, hyponatremia, pleural effusion.
TEST: urinary antigen. CXR shows bi-basal consolidation

Mycoplasma Pneumonae: Rash (erythema multiforme), unusual symptoms (abd pain, dry
cough), long duration of symptoms, hyponatremia, Diagnosis by serology. CXR: reticularnodular shadowing or patchy consolidation


Staphylococcal pneumonia may complicate influenza infection and is seen most
frequently in the elderly and in intravenous drug users or patients with underlying disease. Shows
bilateral cavitations.

Pneumonia associated with COPD: H.influenze (more likely) or P.aeruginosa

P.aeruginosa: Common in bronchiectasis or CF. Also causes hospital acquired infection.

Klebsiella pneumoniae is classically in alcoholics

Strept pneumonia: Associated with herpes labialis. commoner in the elderly,


alcoholics, post-splenectomy, immunosuppressed and patients with chronic heart failure
or pre-existing lung disease

Pneumocystis pneumonia (PCP) causes pneumonia in the immunosuppressed


(eg HIV). CXR may be normal or show bilateral perihilar interstitial shadowing.
Diagnosis: visualization of the organism in induced sputum, bronchoalveolar lavage, or
in a lung biopsy specimen

SO i think it is either klebsiella (gram - rod) or streptococcus as these are the ones
common in alcoholics but bilateral cavitations do point in favor of staphylococcus.
140. A 67yo man had successful thrombolysis for an inf MI 1 month ago and was
discharged after 5 days. He is now re admitted with pulmonary edema. What is the most
probable dx?
a. Aortic regurgitation
b. Ischemic mitral regurgitation
c. Mitral valve prolapse
d. Pulmonary stenosis
e. Rheumatic mitral valve stenosis
Ans. The key is B. Ischaemic mitral regurgitation. [ Causes of Ischaemic mitral regurgitation: left
ventricular remodeling and dysfunction, annular dilation/dysfunction, and mechanical dyssynchrony].

Complications OF MI:
Cardiac arrest
Unstable angina
Bradycardias or heart block
cardiogenic shock
Tachyarrhythmias:
Consider implantable cardiac defibrillator
Right ventricular failure (RVF)/infarction
Pericarditis
DVT & PE:
Systemic embolism:
Cardiac tamponade
Mitral regurgitation
Ventricular septal defect
Late malignant ventricular arrhythmias
Dresslers syndrome
Left ventricular aneurysm
Mitral regurgitation: May be mild (minor papillary muscle dysfunction) or severe
(chordal or papillary muscle rupture or ischaemia). Presentation: Pulmonary oedema.
Treat LVF and consider valve replacement.

141. A 60yo lady who had stroke 3 years ago now reports having increased dyspnea on
exertion and atrial fibrillation. CXR: straight left border on the cardiac silhouette. What is
the most probable
dx?
a. Aortic regurgitation
b. Ischemic mitral regurgitation
c. Mitral valve prolapse
d. Pulmonary stenosis
e. Rheumatic mitral valve stenosis
Q. 1. What is the key?
Q. 2. What are the points in favour of your answer?
Ans. 1. The key is E. Rheumatic mitral valve stenosis.
Mitral Stenosis: Presentation: dyspnoea; fatigue; palpitations; chest pain; systemic
emboli; haemoptysis; chronic bronchitis-like picture
CAUSES: Rheumatic, congenital, mucopolysaccharidoses, endocardial fibroelastosis,
malignant carcinoid (rare), prosthetic valve.
SIGNS: Malar flush on cheeks (due to inc cardiac output); low-volume pulse; AF
common;
tapping, non-displaced, apex beat (palpable S1). On auscultation: loud S1; opening
snap (pliable valve); rumbling mid-diastolic murmur (heard best in expiration,
with patient on left side
ECG show P-mitrale ECHO is diagnostic. CXR: left atrial enlargement
(double shadow in right cardiac silhouette)
TREATMENT: balloon valvuloplasty (if pliable, non-calcified valve), open mitral
valvotomy
or valve replacement.
Complications: Pulmonary hypertension, emboli, pressure from large LA on local
structures, eg hoarseness (recurrent laryngeal nerve), dysphagia (oesophagus),
bronchial obstruction; infective endocarditis
Ans. 2. Points in favour: i) Dyspnoea on exertion ii) Straight left border of the cardiac
silhouette. Iii) Atrial fibrillation is a common association.
142. A 60yo diabetic complains of pain in thigh and gluteal region on walking up the
stairs for the last 6 months. She is a heavy smoker and has ischemic heart disease.
What is the most appropriate dx?
a. Thromboangitis Obliterans
b. Sciatica
c. DVT
d. Atherosclerosis
e. Embolus
Q. 1. What is the key?
Q. 2. What are the points in favour?
Ans. 1. The key is D. Atherosclerosis.
Ans. 2. i) It is not sciatica as sciatica pain is worse when sitting. There may be
weekness, numbness, difficulty moving the leg or foot. A constant pain on one side of
the rear. A shooting pain that makes it difficult to stand up. ii) It is not DVT as no
swelling, warmth or redness of skin are there iii) It is not thromboangitis obliterans as
pulses are ok, no colour change or reduced hair growth, no ulceration or gangrene iv) no
embolism as no pain (rest pain), no numbness, no redness or itching or rash, no
ulceration of skin.

This patient has intermittent claudication due to atherosclerosis,

Symptoms Cramping pain is felt in the calf, thigh, or buttock after walking for a given
distance (the claudication distance) and relieved by rest. Ulceration, gangrene, and foot
pain at resteg burning pain at night relieved by hanging legs over side of bedare the
cardinal features of critical ischaemia.
Fontaine classification for peripheral arterial disease: 1. Asymptomatic 2. Intermittent
claudication 3.Ischaemic rest pain 4. Ulceration/gangrene (critical ischaemia)
Signs: Absent femoral, popliteal or foot pulses; cold, white leg(s); atrophic skin;
punched out ulcers (often painful); postural/dependent colour change; a vascular
(Buergers) angle of <20 and capillary filling time >15s are found in severe ischaemia
Imaging: Colour duplex USS is 1st line (non-invasive and readily available). If
considering
intervention then MR/CT angiography
MANAGEMENT: Stop smoking, exercise, treat HTN, antiplatelet (clopidogrel). Advise
exercise until maximum tolerable pain.
Vasoactive drugs may be used. If PAD is advancing consider:
Percutaneous transluminal angioplasty (PTA) is used for disease limited to a single
arterial segment.
Surgical reconstruction: arterial bypass
Amputation. If all fail. Knee should be saved if possible.
143. A 3yo child who looks wasted on examination has a hx of diarrhea on and off. The
mother
describes the stool as bulky, frothy and difficult to flush. What is the single inv most likely
to
lead to dx?
a. Sweat chloride test
b. Anti-endomysial antibodies
c. LFT
d. US abdomen
e. TFT
Q. 1. What is the key?
Q. 2. What is the diagnosis?
Ans. 1. The key is B. Anti-endomysial antibody
Ans. 2. The diagnosis is celiac disease.
Coeliac Disease: Suspect this in all those with diarrhoea + weight loss or anaemia (esp.
if iron or B12). It is a T-cell-mediated autoimmune disease of the small bowel in which
prolamin
(alcohol-soluble proteins in wheat, barley, rye oats) intolerance causes villous atrophy
and malabsorption (including of bile acids)

Investigations: FBC, Dec feritin, dec vit.B12


Antibodies: alpha -gliadin, transglutaminase and anti-endomysial 95% specific. Duodenal
biopsy shows subtotal villous atrophy.

Treatment Lifelong gluten-free diet


144. A 45yo woman has had severe epigastric and right hypochondrial pain for a few
hours. She has a normal CBC, serum ALP is raised, normal transaminase. 3 months
ago she had a
cholecystectomy done. What is the most appropriate inv?
a. US abdomen
b. ERCP
c. MRCP
d. CT abdomen
e. Upper GI endoscopy
Q. 1. What is the key?
Q. 2. What is the diagnosis?
Ans. 1. The key is B. ERCP.
Ans. 2. Diagnosis is choledocolithiasis.
Right upper quadrant pain think of gall stones. And since the LFTs here show
obstructive picture ALP increased with normal transaminases the obstruction is most
probably in the biliary tract CBD.
ERCP: Endoscopic retrograde cholangiopancreatography (ERCP)
Indications: No longer routinely used for diagnosis, it still has a significant therapeutic
role: sphincterotomy for common bile duct stones; stenting of benign or malignant
strictures and obtaining brushings to diagnose the nature of a stricture.
MRCP: MRCP (magnetic resonance cholangiopancreatography) gives detail of the
biliary system and the pancreatic duct. MRCP has excellent sensitivity and specificity for
diagnosing common bile duct stoneswhen these are >6mm both are 99% (although
accuracy is lower for stones <6mmand is the imaging modality of choice.
But here since we need to remove the stones as well so we use ERCP.
145. A 53yo woman presented with pain in the eye, blurry vision and clumsiness for 3
months. She has a hx of difficulty in swallowing and weakness in her right upper limb 2y
ago. What is the inv of choice?
a. CSF analysis
b. EEG
c. EMG
d. MRI brain
e. Visual evoked response test
Q. What is the key?
Q. What is the diagnosis?
Ans. 1. The key is D. MRI brain.
Ans. 2. Diagnosis is multiple sclerosis.

Multiple sclerosis: Discrete plaques of demyelination occur at multiple CNS sites. Early
exposure to sunlight/vit. D is important, and vit. D status relates to prevention of MS,
PRESENTATION: Usually monosymptomatic: unilateral optic neuritis (pain on
eye

movement and rapid central vision); numbness or tingling in the limbs; leg
weakness; brainstem or cerebellar symptoms (eg diplopia, ataxia). The disease
has a relapsing - remitting course.

INVESTIGATIONS: This is clinical, as no test is pathognomonic. MRI is sensitive


but not specific for plaque detection.
CSF: Oligoclonal bands of IgG on electrophoresis suggest CNS inflammation but
does not confirm MS.. Delayed visual, auditory, and somatosensory evoked
potentials.
MANAGEMENT: Stress free life. Give vit. D to achieve serum 25(OH)D levels of
50nmol/L
Methylprednisolone shortens relapse doesnt alter overall prognosis.
Beta interferon reduces replaces by 30% but does not reduce overall disability
Monoclonal antibodies: Alemtuzumab and natalizumab reduce relapses by 68%.
Glatiramer in secondary progressive
Azathioprine in relapsing-remiting
Palliation: Spasticity: Baclofen, diazepam, dantrolene or tizanidine.
Tremor: Botulinum toxin
Urgency/frequency: If post-micturition residual urine >100mL, teach intermittent selfcatheterization; if <100mL, try tolterodine
146. A 55yo male presents with malaise and tiredness. Exam: spleen approaching RIF,
no
lymphadenopathy. Choose the single cell type?
a. Helmet shaped cell
b. Sickle cell
c. Granulocyte without blast cells
d. Blast cells
Q. 1. What is the key?
Q. 2. What is the diagnosis?
Q. 3. What are the diagnostic features?

diagnosis is CML.
Ans. 1. The key is C. Granulocyte without blast cells.
Ans. 2. The diagnosis is CML.
Ans. 3. Diagnostic features are i) increased number of mature granulocytes ii) huge
splenomegaly.
Causes of Splenomegaly: If massive, think of: chronic myeloid leukaemia,
myelofibrosis, malaria (or leishmaniasis)

Splenomegaly with fever Infection (malaria, SBE/IE hepatitis, EBV,TB, CMV,


HIV)
With lymphadenopathy Glandular fever, leukemia/lymphoma, Sjogrens syndrome
With purpura Septicemia, typhus, DIC, amyloid, meningococcemia
CML: It is a myeloproliferative disorder, common ge of occurence is 40-60 years.
Philadelphia chromosome (Ph) Present in >80% of those with CML t(9:22)
Symptoms Mostly chronic and insidious: weightloss, tiredness, fever, sweats.
There
may be features of gout (due to purine breakdown), bleeding (platelet
dysfunction),
and abdominal discomfort (splenic enlargement). ~30% are detected by chance.
Signs Splenomegaly (>75%)often massive. Hepatomegaly, anaemia, bruising (fi
g 2).
Tests WBC increased (often >100109/L) with whole spectrum of myeloid cells,
ie increased neutrophils, myelocytes, basophils, eosinophils. Hb dec or normal,
platelets variable. Urate increased B12increased. Bone marrow hypercellular.
Treatment: is by chemotherapy or stem cell transplantation.
147. A 6yo pt comes with easy bruising in different places when she falls. CBC:
WBC=25, Hgb=10.9, Plt=45. Her paul brunnel test +ve. What is the most likely dx?
a. Glandular fever
b. ITP
c. Trauma
d. NAI
e. Septicemia
Q. 1. What is the key?
Q. 2. What are the lab. Values that suggests the diagnosis here?
Ans. 1. The key is A. Glandular fever.
Ans. 2. Suggestive lab. Values: WBC=25 (leucocytosis), Hgb=10.9 (usually patient is not
anaemic), Plt=45 (thrombocytopenia-leading to easy bruising), Positive paul bunnel test.

INFECTIOUS MONONUCLEOSIS
Caused by EBV, spread by saliva or droplets. EBV also causes certain cancers
(Hogdkins, burkitts and nasopharyngeal CA)
Symptoms: Sore throat, inc T, anorexia, malaise, lymphadenopathy (esp.
posterior
triangle of neck), palatal petechiae, splenomegaly, fatigue/mood
Blood film Lymphocytosis and atypical lymphocytes (large, irregular nuclei)

Heterophil antibody test (Monospot, Paul Bunnell) 90% show heterophil


antibodies by 3wks, disappearing after ~3 months PCR may also be done.
Treatment: None usually needed.Avoid contact sports for 8 weeks. Avoid
alcohol. Steroid or acyvlovir may be given but there is not much benefit.
Never give ampicillin or amoxicillin for sore throats as they often cause a severe
rash in those with acute EBV infection
148. A 41yo woman who has completed her family, has suffered from extremely heavy
periods for many years. No medical tx has worked. She admits that she would rather
avoid open surgery.
After discussion, you collectively decide on a procedure that wouldnt require open
surgery or
GA. Select the most appropriate management for this case.
a. Endometrial ablation
b. Hysterectomy
c. Fibroid resection
d. Myomectomy
e. Uterine artery embolization
Ans. The key is uterine artery embolization.
Treating menorrhagia Drugs Progesterone-containing IUCDs, eg Mirena
should be considered 1st line treatment for those wanting contraception.
effective for bleeding and also reduce the size of fibroid uterus.
2nd line recommended drugs are antifibrinolytics, antiprostaglandins or the
Pill. Antifibrinolytics Taken during bleeding these reduce loss (by 49%)eg
tranexamic acid CI: thromboembolic disease
Antiprostaglandins eg mefenamic acid 500mg/8h PO pc (CI: peptic ulceration) taken
during
days of bleeding particularly help if there is also dysmenorrhoea. COCP can also be
used if they are not contraindicated..
3rd line recommendation is progestogens IM or norethisterone
Rarely gonadotrophin (LHRH) releasing hormones are used
Surgery Endometrial resection is suitable for women who have completed
their families and who have <10wk size uterus and fibroids <3cm. Contraception will be
required. For women wishing to retain fertility who have fibroids >3cm consider uterine
artery embolization or myomectomy
Women not wishing to retain fertility, with a uterus >10wk size and fibroids >3cm may
benefit from hysterectomy, vaginal hysterectomy being the preferred route.
149. A girl with hx of allergies visited a friends farm. She got stridor, wheeze and
erythematous rash. What is the most appropriate tx?
a. 0.25ml IM adrenaline
b. 0.25ml PO adrenaline
c. 0.25ml IM adrenaline
d. IV chlorphearamine
Ans. The key is A. 0.25 ml IM adrenaline [Presence of stridor and wheeze are
suggestive of anaphilaxis and treatment option is adrenaline].
Consider anaphylaxis when there is compatible history of rapid-onset severe allergictype reaction with respiratory difficulty and/or hypotension, especially if there are skin
changes present and the treatment of anaphylaxis is IM adrenaline not anti histamine

Treatment:ABCDE, Oxygen, IM Adrenaline. <6yrs0.15ml, 6-12yrs 0.3ml, >12 yrs 0.5ml


1:1000
Since the age of the girl is not mentioned here and options A & C are the same so A or
C could be the answers supposing the girl was 6-12 yrs of age.
150. A 5yo boy is referred to the hospital and seen with his father who is worried that he
has been listless. He is not sure why his GP suggested he should come to the ED and is
keen to get some tablets and go home. Exam: tired and irritable, swelling around eyes.
Renal biopsy: remarkable
for podocyte fusion on EM. What is the most probable dx?
a. NAI
b. Myelodysplastic disease
c. HSP
d. Membranous GN
e. Minimal change GN
Ans. The key is E. Minimal change glomerulonephritis. [Podocyte fusion on electron
microscopy]
Most common cause of nephrotic in children is minimal change disease. There will be
hypoalbuminemia and peripheral edema too. Electron microscopy shows effacement of
podocyte foot processes.. MCD has albumin selective proteinuria. Treatment is with
steroids.
151. A 6yo boy is brought to the hospital for a 3rd episode of sore throat in 1 month. He is
found
bleeding from gums and nose and has pale conjunctiva. Whats the single cell type?
a. Clumped platelets
b. Microcytes
c. Granulocyte without blast cells
d. Blast cells
e. Mature lymphocytes
Q. 1. What is the key?
Q. 2. What is the diagnosis?
Q. 3. What are the points that favour diagnosis?
Ans. 1. The key is D. Blast cells.
Ans. 2. The diagnosis is ALL
Ans. 3. Points in favour: i) Age-6yrs ii) recurrent infection (sorethroat) due to neutrpenia
and abnormal lymphoblasts which cannot protect from infection iii) thrombocytopenia
causing gum and nose bleeding. Iii) anaemia (pale conjunctiva) due to reduced red cell
production from marrow occupation by blast cells. [Here debate came why it is not
aplastic anaemia? There is no risk factor mentioned for this patient for aplastic anaemia.
There may be congenital aplastic anaemia but again it would present earlier in life. So it
goes more with leukaemia but it cannot be confirmed unless we do bone marrow
aspiration.]
ALL: This is a malignancy of lymphoid cells, affecting B or T lymphocyte cell lines,
arresting
maturation and promoting uncontrolled proliferation of immature blast
cells, with marrow failure and tissue infiltration.

Causes: Genetic susceptibility, environmental factors (ionizing radiations) Downs


syndrome.
Commonest cancer of childhood.

Signs and symptoms:


Marrow failure: Anaemia (Hb), infection (WCC), and bleeding (platelets).
Infiltration: Hepatosplenomegaly, lymphadenopathysuperficial or mediastinal,
orchidomegaly, CNS involvementeg cranial nerve palsies, meningism.
INVESTIGATIONS: Characteristic blast cells on blood film and bone marrow
CXR and CT scan to look for mediastinal and abdominal lymphadenopathy.
Lumbar puncture should be performed to look for CNS involvement.
TREATMENT: Blood transfusions, prophylactic antibiotics, IV antibiotics in case of
infection. Main stay of treatment is chemotherapy.
Prognosis Cure rates for children are 7090%; for adults only 40%
152. A 23yo man has been stabbed in the back and has SOB. The trachea is not
deviated, he has engorged neck veins and absent breath sounds on the right. What is
the most appropriate dx?
a. Tension pneumothorax
b. Cardiac tamponade
c. Simple pneumothorax
d. Hemothorax
e. Pleural effusion
Q. 1. What is the key?
Q. 2. What are the point in favour of your answer?
Ans. 1. The key is A. Tension pneumothorax.
Ans. 2. Points in favour: i) Stab wound in the back ii) SOB iii) Engorged neck vein iv)
Absent breath sound.
153. A 44yo pt comes with right hemiparesis. Exam: left sided ptosis and left dilated
pupil. Where is the lesion?
a. Cerebral infarct
b. Cerebellar infarct
c. Medulla oblongata
d. Pons
e. Midbrain
Q. 1. What is the key?
Q. 2. What is the name of this condition?
Ans. 1. The key is E. Midbrain.
Ans. 2. Weber syndrome [presence of ipsilateral oculomotor nerve palsy and
contralateral hemiparesis or hemiplagia].
Ptosis + miosis = horners syndrome
CN3 nucleus lies in the midbrain.
Fore brain: CN 1,2
Mid brain: CN 3,4
Pons: CN 5,6,7,8
Medulla: CN 9,10,12

Ptosis + mydriasis= oculomotor nerve palsy.

Webers syndrome (superior alternating hemiplegia) Ipsilateral oculomotor


nerve palsy with contralateral hemiplegia, due to infarction of one-half of the midbrain,

after occlusion of the paramedian branches of the basilar or posterior cerebral


arteries
154. A 50yo man has a stab wound to his left anterior chest at the level of the 4th ICS. He
has a BP 80mmHg, pulse=130bpm. His neck veins are dilated and his heart sounds are
faint. His trachea is central. What is the most appropriate dx?
a. Cardiac tamponade
b. Diaphragmatic rupture
c. Fractured ribs
d. Tension pneumothorax
e. Traumatic rupture of aorta
Q. 1. What is the Key?
Q. What are the points in favour of your answer?
Ans. 1. The key is Cardiac tamponade.
Ans. 2. Points in favour: i) Systolic BP 80 mmHg ii) Pulse 130 bpm iii) Engorged neck
vein iv) Faint heart sounds v) Trachea is central.
Essence:Pericardial fluid collects intrapericardial pressure rises heart cannot
fill pumping stops.
Causes: Trauma, lung/breast cancer, pericarditis, myocardial infarct, bacteria, eg TB.
Signs: Falling BP, a rising JVP, and muffled heart sounds (Becks triad); JVP on
inspiration
(Kussmauls sign); pulsus paradoxus (pulse fades on inspiration) (also in severe
asthma). Echocardiography may be diagnostic. CXR: globular heart; left heart border
convex or straight; right cardiophrenic angle <90. ECG: electrical alternans
Management: Prompt pericardiocentesis. While waiting give O2, IV fluids, monitor ECG.
155. A 15yo boy has a soft painless swelling in the left scrotum, blue in color and can be
compressed. What is the most appropriate next step?
a. Analgesia
b. Antibiotic
c. Biopsy
d. Immediate surgery
e. Reassurance
Q. 1. What is the key?
Q.2. What is the name of this condition?
Ans. 1. The key is E. Reassurance.
Ans. 2. Name of the condition is Varicocele.
Testicular lump = cancer until proved otherwise.
Acute, tender enlargement of testis = torsion until proved otherwise
Diagnosing scrotal masses
Cannot get above inguinoscrotal hernia or hydrocele extending proximally
Separate and cystic epididymal cyst
Separate and solid epididymitis/varicocele
Testicular and cystic hydrocele
Testicular and solidtumour, haematocele, granuloma, orchitis, gumma. USS may help.
Varicocele Dilated veins of pampiniform plexus. Left side more commonly affected.
Often visible as distended scrotal blood vessels that feel like a bag of worms.
Patient may complain of dull ache. Associated with subfertility, but repair (via surgery

or embolization) seems to have little effect on subsequent pregnancy rates.


156. A 12yo pt presents with copious diarrhea. Exam: urine output=low, mucous
membrane=dry, skin turgor=low. What is the most appropriate initial management?
a. Antibiotic
b. Antimotility
c. Anti-emetic
d. Fluid replacement
e. Reassurance
Q. 1. What is the key?
Q. 2. What is the diagnosis and why?
Ans. 1. The key is D. Fluid replacement.
Ans. 2. Diagnosis is severe dehydration. Points in favour: i) low urine output ii) dry
mucous membrane and iii) low skin turgor.
Gastroenteritis Rotavirus is the most common cause of gastroenteritis in infants and
children.
Norovirus (most common cause in adults)
ASSESSMENT OF DEHYDRATION:
Mild dehydration: Decreased urine output.
5% dehydration: Dry mucous membranes; decreased urine output.
10% dehydration: The above + sunken fontanele, inc pulse; hoarse cry; dec skin turgor.
>10%: The above, but worse, with: shock, drowsiness, and hypotension.
MANAGEMENT
Mild: Treated at home by oral rehydrating therapy.
Moderate: Oral fluids, via NG or IV fluids can be used. Rapid rehydration involves 4
hours of
10mL/kg/h 0.9% NaCl then maintenance after if needed. Monitor U & Es
Severe: If not in shock oral or NG route can be used. If in shock. 0.9% saline 20mL/kg
IVI bolus, while calculations are performed. Continuously monitor pulse, BP, ECG.
Continue with boluses until the signs of shock ease.
Then give the daily requirement + fluid deficit
157. A 60yo smoker presents with cramp-like pain in the calves relieved by rest and nonhealing ulcers. Exam: cold extremities with lack of hair around the ankles, absent distal
pulses. What is the
most probable dx?
a. Intermittent claudication
b. Chronic ischemia of the limbs
c. Buergers disease
d. DVT
e. DM
Q. 1. What is the key?
Q. 2. Points that support your diagnosis.
Ans. 1. The key is B. Chronic ischaemia of the limb.
Ans. 2. Intermittent claudication is a symptom not diagnosis. It is not buergers disease
as buerger occur in more younger heavy smoker (before the age of 50yrs) mostly limited

to the extremities, It is not DVT as dvt pain or tenderness is not of an intermittent


claudication pattern. Again in DM there is no intermittent claudication.
158. An otherwise healthy 13yo boy presents with recurrent episodes of facial and
tongue swelling and abdominal pain. His father has had similar episodes. What is the
most likely dx?
a. C1 esterase deficiency
b. HIV
c. Mumps
d. Sarcoidosis
e. Sjogrens syndrome
Q. 1. What is the key?
Q. 2. What is the name of this condition?
Q. 3. Why it is not acquired?
Ans. 1. The key is A. C1 esterase inhibitor deficiency.
Ans. 2. Hereditary angioedema.
Ans. 3. Acquired angioedema usually manifest after the age of 40 yrs.
The oedema is triggered by increased permeability of the blood vessels.The net result is
episodes of massive local oedema, ie angio-oedema. (In angio-oedema, the swelling is
subcutaneous or submucosal rather than epidermal, so urticaria is absent.) It can mimic
anaphylaxis.
Type I has low levels of C1-INH (C1 esterase inhibitor) (the majority of cases).
Type II has impaired function of C1-INH.

Clinical features
Recurrent episodes of angio-oedema and/or abdominal pain - may involve: Laryngeal
oedema - can be fatal:

Pointers to a diagnosis of HAE are:


Family history.
Recurrent episodes of non-urticarial swelling lasting >24 hours, and
unresponsive to antihistamines.
Laryngeal oedema.

Recurrent, unexplained abdominal pain and vomiting.


Symptoms starting in childhood and worsening in adolescence.
The recommended initial tests are:
Serum complement factor 4 (C4) level.
C1 inhibitor (C1-INH) antigenic protein level.
C1-INH function (if available).
Management involves:
Emergency treatment of attacks
Patient education and awareness; may need own supply of emergency
treatment.
Good links with A&E departments.
Prophylaxis:
o Short-term cover for procedures - eg, dental treatment.
o Long-term prophylactic drugs if required.
o Avoidance of triggers.
Testing of family members is recommended owing to the potential seriousness of
an attack.
Drugs:
Plasma-derived C1-INH, A bradykinin receptor inhibitor, Antifibrinolytic drugs - eg,
tranexamic acid, Attenuated androgens - eg, danazol.

159. A 25yo had an LSCS 24h ago for fetal distress. She now complains of intermittent
vaginal
bleeding. Observations: O2 sat=98% in air, BP=124/82mmHg, pulse=84bpm,
temp=37.8C. The
midwife tells you that she had a retained placenta, which required manual removal in the
OT.
Choose the most appropriate C-Section complication in this case?
a. Retained POC
b. Aspiration pneumonitis
c. Endometritis
d. Uterine rupture
e. DIC
Q. 1. What is the key?
Q. 2. What are the points in favour?
Ans. 1. The key is C. Endometritis.
Ans. 2. More handling of tissue like manual removal of placenta, intermittent vaginal
bleeding and raised temperature points toward infective process like endometritis.
This is secondary PPH.
Secondary PPH: This is excessive blood loss from the genital tract after 24h from
delivery. It usually occurs between 5 and 12 days and is due to infections (most common
cause) (endometritis) or retained placenta.
Look for history of extended labour, difficult third stage, ragged placenta, PPH.
Symptoms: Abdominal pain. Offensive smelling lochia. Abnormal vaginal bleeding PPH. Abnormal vaginal discharge. Dyspareunia. Dysuria.

Signs: are those of sepsis. Tachycardia, fever, rigors, suprapubic tenderness.


Treatment: For endometritis: IV antibiotics if there are signs of severe sepsis. If less
systemically unwell, oral treatment may be sufficient. Piperacilin and tazobectum may be
used.
If RPOC are suspected, elective curettage with antibiotic cover may be required. Surgical
measures should be undertaken if there is excessive or continuing bleeding, irrespective
of ultrasound findings
160. A 30yo woman has brief episodes of severe shooting pain in the rectum. Rectal
examination and flexible sigmoidoscopy are normal. What is the most probable dx?
a. Anal hematoma
b. Anal fissure
c. Rectal carcinoma
d. Proctalgia fugax
e. Piles
Ans. is D. Proctagia fugax [normal rectal examination and flexible sigmoidoscopy
excludes other options].

Shooting pain mostly neuropathic pain.

Proctalgia fugax Idiopathic (could be because of spasm of muscles) , intense, brief,


stabbing/crampy rectal pain,often worse at night. Very short lived pain. The mainstay of
treatment is reassurance. Inhaled salbutamol or topical GTN (0.20.4%) or topical
diltiazem (2%) may help.
161. A 78yo male, DM and HTN, had a fall and since then is unable to walk. He presents
with
deformity and tenderness over the right hip area. XR=fx of femur neck. What is the
single most
associated nerve injury?
a. Sciatic nerve
b. Gluteal nerve
c. Lateral peroneal nerve
d. Tibial nerve
e. Femoral nerve
Ans. The key is A. Sciatic nerve.
Fractures of the femoral neck are far more common in the elderly but fractures of the
femoral shaft and supracondylar fractures most often occur in adolescents and young
adults. Hip fracture is the most common reason for admission to an orthopaedic trauma
ward. Intertrochanteric fractures affect the base of the femoral neck, May disrupt the
blood supply to the femoral head, leading to avascular necrosis.

Posterior dislocation of the hip

This is caused by major force to a flexed knee and hip - eg, when knees strike
the dashboard in a road traffic accident.
Posterior dislocations account for the majority of hip dislocations.
The affected leg is shortened and internally rotated with flexion and adduction at
the hip. This appearance may not occur if there is also a femoral shaft fracture.
Diagnosis is usually obvious on AP X-ray
Treatment: ABC, Pain management, reduction under GA. Allis technique

Complications
These include:
Sciatic nerve injury: pain in the distribution of the sciatic nerve, loss of sensation
in the posterior leg and foot and loss of dorsiflexion (peroneal branch) or plantar
flexion (tibial branch) of the foot.
Vascular injury: not as frequent as with anterior dislocations.
Avascular necrosis of the femoral head: risk increases the longer the hip is
dislocated.
Secondary osteoarthritis.
[1]

Anterior dislocation of the hip

This is much less common.


It causes pain in the hip and inability to walk or adduct the leg.
The leg is externally rotated, abducted, and extended at the hip.

Complications

These include damage to the femoral nerve, artery and vein:


Injury to the femoral nerve may occur, resulting in paralysis and numbness in the
femoral nerve distribution.
Injury to the femoral artery may produce arterial insufficiency in the leg

162. A 20yo man has a head on collision in a car. On presentation his is breathless, has
chest pain and fx of 5-7th rib. CXR confirms this. What is the most appropriate initial
action in this pt?
a. Antibiotics
b. Analgesia
c. O2 by mask
d. Physiotherapy
e. Refer to surgeon
Ans. The key is C. O2 by mask. [There was debate in this forum that pain relief should
be given first which will automatically relieve breathing problem. But others told O2 first].
O2 first is the correct answer!
[http://www.urmc.rochester.edu/encyclopedia/content.aspx?contenttypeid=22&contentid
=flailchest]
ABCDE always comes first.
163. A 28yo man with complains of headache and nose bleeds also has pain in the
lower limbs on
exertion. Exam: radio-femoral delay, cold legs with weak pulse and mild systolic murmur
with
normal S1S2. What is the most probable dx?
a. TOF
b. ASD
c. VSD
d. PDA
e. CoA
Ans. The key is coarctation of aorta. [headache and nosebleeds - >hypertension, pain in
lower limb on exertion -> as reduced blood supply to leg due to coarctation, radiofemoral delay, cold legs with week pulse, mid-systolic murmur are all features of
coarctation of aorta].
Radio femoral delay is a clincher for coarctation of aorta.
Coarctation of the aorta Congenital narrowing of the descending aorta. More common
in boys.
Associations: Bicuspid aortic valve; Turners syndrome. Signs: Radiofemoral delay
(femoral pulse later than radial); weak femoral pulse; BP; scapular bruit; systolic
murmur (best heard over the left scapula).
Complications: Heart failure; infective endocarditis. Coarctation of the aorta is
associated with berry aneurysms which if ruptured cause Sub arachnoid hemorrhage.
Tests: CT or MRI-aortogram, CXR shows rib notching.
Cardiac catheterisation
To confirm the diagnosis when this is not clear at ultrasound.

To determine the gradient across the coarctation (with a gradient in excess of 20


mm Hg considered to be significant).
To assess other abnormalities and the overall haemodynamic picture when
considering therapeutic options in more detail.
Therapeutically using balloon angioplasty with or without stent implantation.

Treatment: Surgery or balloon dilatation stenting.


164. A 23yo male has a tonic clonic seizure whilst at college. His GCS is 12,
BP=120/77mmHg,
HR=99bpm. What is the most appropriate inv for his condition?
a. CT
b. MRI
c. Serum blood glucose
d. Serum drug levels
Ans. The key is C. Serum blood glucose [it is also possible that he may have taken drug,
even though first we have to do serum glucose as its presence can be very easily
managed and it needs urgent management to save life. If it is excluded then we can look
for other causes which may be not fatal in short time as hypoglycaemia].
This case will be treated as status epilepticus. Normally status occurs in a patient with
known epilepsy but here no such history is given. Even in a person who presents with
his first seizure we need to look for hypoglycemia first before going onto later tests.
Status epilepticus:
This means seizures lasting for >30min, or repeated seizures without intervening
consciousness.
Also consider eclampsia in mind if the patient is female and the abdomen is
distended.
Investigations
Bedside glucose, the following tests can be done once treatment has started: lab
glucose,
ABG, U&E, Ca2+, FBC, ECG.
Consider anticonvulsant levels, toxicology screen, LP, culture blood and urine, EEG,
CT, carbon monoxide level. Pulse oximetry, cardiac monitor.
Treatment: Try to control seizure in less than 20mins as there could be permanent
brain damage.
1. IV lorazepam:0.1mg/kg. Repeat if no response in 10mins. Be careful about respiratory
depression. If there is no IV access give PR Diazepam.
2.Buccal midazolam: Alternative oral route. squirt half the volume between the lower
gum and the cheek on each side.
3.Phenytoin infusion: 1520mg/kg IVI (roughly 1g if 60kg, and 1 . 5g if 80kg; max 2g.
2nd line! Dont use if bradycardic or hypotensiv. ECG monitoring is recommended.
Diazepam infusion: eg 100mg in 500mL of 5% dextrose. It is most unusual for seizures
to remain unresponsive following this. If they do, allow the idea to pass through your
mind that they could be pseudoseizures, particularly if there are odd features (pelvic
thrusts; resisting attempts to open lids and your attempts to do passive movements;
arms and legs flailing around).

4. Dexamethasone: 10mg IV if vasculitis/cerebral oedema (tumour) possible.


5. General anaesthesia: For refractory status: get anaesthetist/ICU involved early

165. A 20yo man complains of recent onset of itching which followed a viral infection.
There are
numerous wheals of all sizes on his skin particularly after he has scratched it. These can
last up
to an hour. What is the most probable dx?
a. Uremia
b. Urticaria
c. Psychogenic itching
d. Atopic eczema
e. Primary biliary cirrhosis
Ans. The key is B. Urticaria.
Urticaria Signs: wheals, rapid onset after taking drug association with angio-oedema
/anaphylaxis. It can result from both immunological and non-immunological mechanisms.
Causes: Drugs:morphine & codeine cause direct mast cell degranulation; penicillins &
cefalosporins trigger IgE responses; NSAIDs; ACEi.

Clinical diagnosis. No investigations required.


Management:
Find the cause and avoid/treat it.
Antihistamines:
Non-sedating H1 antihistamines are the mainstay of treatment
In pregnancy chlorphenamine is often the first choice of antihistamine.

166. A 75yo lady who had mitral valve replacement 13 yrs ago has developed recurrent
breathlessness. Her husband has noticed prominent pulsation in her neck. She
complains of

abdominal pain and ankle swelling. What is the most probable dx?
a. Aortic regurgitation
b. Mitral regurgitation
c. Mitral stenosis
d. Tricuspid regurgitation
e. Pulmonary stenosis
Ans. The key is D. Tricuspid regurgitation. [Points in favour: i) recurrent breathlessness
if the cause is LV dysfunction, ii) prominent pulsation in the neck giant v waves, iii)
abdominal pain pain in liver on exertion, ankle swelling; These are features of tricuspid
regurgitation. Reference:- OHCM, 9 edition, page- 142]
th

Pulmonary stenosis is mostly congenital. But it can be caused by rheumatic fever and in
this case the patient could have developed rheumatic fever given his history of valve
replacement. But there is no mention of abdominal pain in pulmonary stenosis so i guess
thats the differentiating point here.
Aortic regurgitation, mitral stenosis and regurgitation donot involve abdominal pain and
ankle swelling. And the JVP will not be raised.
Tricuspid regurgitation
Causes: Functional (RV dilatation; eg due to pulmonary hypertension induced by LV
failure); rheumatic fever; infective endocarditis (IV drug abuser); carcinoid syndrome;
congenital (eg ASD, AV canal, Ebsteins anomaly, ie downward displacement of the
tricuspid valve drugs (eg ergot derived dopamine agonists,fenfluramine).
Symptoms: Fatigue; hepatic pain on exertion; ascites; oedema and also dyspnoea and
orthopnoea if the cause is LV dysfunction.
Signs: Giant v waves and prominent y descent in JVP, RV heave; pansystolic murmur,
heard best at lower sternal edge in inspiration; pulsatile hepatomegaly; jaundice; ascites.
Management: Treat underlying cause. Drugs: diuretics, digoxin, ACE-i. Valve
replacement (~10% 30-day mortality).
.
167. A 45yo T1DM had an annual check up. Ophthalmoscopy showed dot and blot
hemorrhage + hard exudate and multiple cotton wool spots. What is the next step in
management?
a. Reassurance and annual screening only
b. Urgent referral to ophthalmologist
c. Laser therapy
d. Non-urgent referral to ophthalmologist
e. Nothing can be done
Ans. The key is D. Non-urgent referral to ophthalmologist. [It is pre-proliferative
retinopathy so non-urgent referral; If proliferative (with neovascularization) urgent
referral].
Diabetic retinopathy Blindness is preventable. Annual retinal screening mandatory
for all patients not already under ophthalmology care. Pre-symptomatic screening
enables laser photocoagulation to be used, aimed to stop production of angiogenic
factors from the ischaemic retina. Indications: maculopathy or proliferative retinopathy.
Background retinopathy: Microaneurysms (dots), haemorrhages (blots) and
hard exudates (lipid deposits). Refer if near the macula, eg for intravitreal triamcinolone.
Pre-proliferative retinopathy: Cotton-wool spots (eg infarcts), haemorrhages,
venous beading. These are signs of retinal ischaemia. Non urgent Refer to a specialist.

Proliferative retinopathy: New vessels form. Needs urgent referral.


Maculopathy: (hard to see in early stages). Suspect if acuity. Prompt laser, intra vitreal
steroids or anti-angiogenic agents may be needed in macular oedema.
168. A 2m baby who has ambiguous genitalia presents to the ED with vomiting. Labs:
Na+=125mmol/L, K+=6mmol/L. What is the most likely dx?
a. Fragile X syndrome
b. Turners syndrome
c. Noonan syndrome
d. Congenital adrenal hyperplasia
Q. 1. What is the key?
Q. 2. What are the points in favour?
Ans. 1. The key is D. Congenital adrenal hyperplasia
Ans. 2. Points in favour: i) ambiguous genitalia ii) salt wasting manifested as
hyponatremia and hyperkalemia (In mild forms of salt-wasting adrenal hyperplasia, salt
wasting may not become apparent until an illness stresses the child). [here
hyperkalaemia inspite of vomiting is indicating the disease].
Congenital adrenal hyperplasia (From secretion of androgenic hormones deficiency of
21-hydroxylase, 11-hydroxylase, or 3--hydroxysteroid dehydrogenase).
Cortisol is inadequately produced, and the consequent rise in ACTH leads to adrenal
hyperplasia and overproduction of androgenic cortisol precursors. CAH is a leading
cause of male pseudohermaphroditism.
Signs: Vomiting, dehydration, and ambiguous genitalia. Girls may be masculinized.
Boys may seem normal at birth, but have precocious puberty, or ambiguous genitalia
(androgens in 17-hydroxylase deficiency), or incomplete masculinization (hypospadias
with cryptorchidism
from 3-hydroxysteroid dehydrogenase). Hyponatraemia and hyperkalaemia are
common. Plasma 17-hydroxyprogesterone Increased in 90%; Increased urinary 17ketosteroids (not in 17-hydroxylase deficit).
Management of adrenocortical crisi: Urgent treatment is needed
0.9% saline IVI (35g Na+/day), glucose, fludrocortisone and hydrocortisone in neonate
IV stat then maintanance dose.
CAH can lead to addisons disease later in life due to delayed onset.
169. A 40yo man collapsed at home and died. The GPs report says he suffered from
T2DM and
BMI=35. What is the most likely cause of death?
a. Myocardial Infarction
b. Diabetes mellitus
c. Heart failure
d. Pulmonary embolism
e. Renal failure
Q. 1. What is the key?
Q. 2. Why the patients death was unnoticed?
Ans. 1. The key is A. MI.
Ans. 2. In diabetics MI become painless when the patient develop autonomic neuropathy
(till there is no autonomic neuropathy diabetic patients will feel MI pain). In this case the
disease was unnoticed as it was a painless attack.
It is one of the complications of Diabetes.
Vascular disease Chief cause of death. MI is 4-fold commoner in DM and is more
likely to be silent. Stroke is twice as common. Women are at high risk.

Address other risk factorsdiet, smoking, hypertension. Suggest a statin (eg simvastatin
40mg
nocte) for all, even if no overt IHD, vascular disease or microalbuminuria. Fibrates
are useful for triglycerides and reduced HDL. Aspirin 75mg reduces vascular events
(if past stroke or MI) and is good as statin co-therapy (safe to use in diabetic retinopathy;
use in primary prevention is disappointing, at least at 100 mg/day.
170. A 38yo pt presented with tingling, numbness, paraesthesia, resp stridor and
involuntary spasm of the upper extremities. She has undergone surgery for thyroid
carcinoma a week ago. What is the most likely dx?
a. Thyroid storm
b. Hyperparathyroidism
c. Unilateral recurrent laryngeal nerve injury
d. External laryngeal nerve injury
e. Hypocalcemia
Q. 1. What is the key?
Q. 2. What is the cause of this condition?
Q. 3. Why there is respiratory stridor?
Ans. 1. The key is E. Hypocalcaemia.
Ans. 2. Hypocalcaemia may be due to accidental parathyroid gland removal during
thyroidectomy.
Ans. 3. Laryngospasm is a feature in hypocalcaemia which may cause stridor.
Thyroid storm causes thyrotoxicosis (inc heart rate, palpitations, weight loss, tremors,
heat intolerance etc) hyperparathyroidism causes hypercalcemia (bone pains, kidney
stones, confusion, psychosis), nerve injuries only explain the stridor.
Causes of hypocalcemia:

Treatment

Mild symptoms: give calcium 5mmol/6h PO, with daily plasma Ca2+ levels.
In chronic kidney disease:May require alfacalcidol
Severe symptoms: give 10mL of 10% calcium gluconate (2.25mmol) IV over 30min,
and repeat as necessary. If due to respiratory alkalosis, correct the alkalosis
171. A 50yo chronic smoker came to OPD with complaint of chronic productive cough,
SOB and
wheeze. Labs: CBC=increase in PCV. CXR >6ribs seen above the diaphragm in
midclavicular line. ABG=pO2 decreased. What is the most likely dx?
a. Interstitial lung disease
b. Wegeners granulomatosis
c. Ca bronchi
d. COPD
e. Amyloidosis
Q. 1. What is the key?
Q. 2. What are the points in favour?
Ans. 1. The key is D. COPD.
Ans. 2. Points in favour: i) Age 50 yrs ii) Chronic smoker iii) Chronic productive cough,
SOB and Wheeze iv) Raised PCV secondary to chronic hypoxaemia v) Low set
diaphragm and widened horizontal ribs vi) Hypoxaemia on ABG.

COPD:
COPD is a common progressive disorder characterized by airway obstruction (FEV1
<80% predicted; FEV1/FVC <0.7. It includes emphysema and chronic bronchitis.
COPD is favoured by: Age of onset >35yrs Smoking (passive or active) or pollution
related Chronic dyspnoea Sputum production Minimal diurnal or day-to-day FEV1
variation.
Chronic bronchitis is defIned clinically as cough, sputum production on most days for 3
months of 2 successive yrs. Symptoms improve if they stop smoking.
Emphysema is defined histologically as enlarged air spaces distal to terminal bronchioles,
with destruction of alveolar walls.
Pink puffers have inc alveolar ventilation, a near normal PaO2 and a normal or low
PaCO2. They are breathless but are not cyanosed. They may progress to type 1
respiratory failure.
Blue bloaters have decreased alveolar ventilation, with a low PaO2 and a high PaCO2.
They are cyanosed but not breathless and may go on to develop cor pulmonale. Their
respiratory centres
are relatively insensitive to CO2 and they rely on hypoxic drive to maintain respiratory
effort
Symptoms Cough; sputum; dyspnoea; wheeze.
Signs Tachypnoea; use of accessory muscles of respiration; hyperinflation; cricosternal
distance (<3cm); decreased chest expansion; resonant or hyperresonant percussion note;
quiet breath sounds (eg over bullae); wheeze; cyanosis; cor pulmonale.
Complications Acute exacerbations infection; polycythaemia; respiratory failure; cor
pulmonale (oedema; raised JVP); pneumothorax (ruptured bullae); lung carcinoma.

Tests FBC: Raised PCV. CXR: Hyperinflation (>6 anterior ribs seen above diaphragm in
midclavicular line); flat hemidiaphragms; large central pulmonary arteries; peripheral
vascular markings; bullae. ECG: Right atrial and ventricular hypertrophy (cor
pulmonale).
ABG: Reduced PaO2 hypercapnia. Lung function: obstructive + air trapping (FEV1
<80% of predicted. FEV1 : FVC ratio <70%
TREATMENT OF CHRONIC STABLE PATIENTS.

172. A 44yo pt has sudden onset of breathlessness and stridor few minutes after
extubation for

thyroidectomy. The pat had longstanding goiter for which he had the surgery. What is
the most
likely dx?
a. Thyroid storm
b. Hematoma
c. Unilateral recurrent laryngeal nerve injury
d. External laryngeal nerve injury
e. Tracheomalacia
Ans. The key is tracheomalacia.
173. A 15yo boy presents with generalized edema. His urinalysis reveals protein +++,
eGFR =110. What is the most likely dx?
a. IgA nephropathy
b. Membranous nephropathy
c. Minimal change disease
d. PSGN
e. Lupus nephritis
Ans. The key is C. Minimal change disease. [Points in favour: i) Age 15 yrs ii)
Generalized oedema iii) Protein in urine +++ vi) Normal eGFR of 110 (Normal range- 90
to 120 mL/min)].
Most common cause of nephrotic syndrome in children is minimal change disease.
There will be hypoalbuminemia and peripheral edema too. Electron microscopy shows
effacement of podocyte foot processes.. MCD has albumin selective proteinuria.
Treatment is with steroids.
174. A 72yo man is receiving chemotherapy for SCLC. He has his 4th tx 8 days ago. He
has a cough with some green sputum but feels well. Temp=37.6C. Chest exam = few
coarse crepitations in the right base. HR=92bpm. CBC: Hgb=12.5g/dL, WBC=1.1,
Neutrophils=0.6, Plt=89. Sputum, urine and blood culture sent to microbiology. What is
the most appropriate management?
a. Broad spectrum antibiotics IV
b. Broad spectrum antibiotics PO
c. GCSF
d. Postpone tx until bacteriology results available
e. Reassure and send home
Q. 1. What is the key?
Q. 2. What is the Diagnosis?
Q. 3. What is the treatment of low WBC count?
Ans. 1. The key is A. Broad spectrum antibiotics IV
Ans. 2. The diagnosis is lower respiratory tract infection.
Ans. 3. GCSF subcutaneously. [it is the treatment of chemotherapy induced leucopenia]
Use of antibiotics in neutropenia:
Treat any known infection promptly.
If T >38C or T >37.5C on 2 occasions, >1h apart, or the patient is toxic, assume
septicaemia and start blind combination therapyeg piperacillintazobactam(+
vancomycin,if Gram +ve organisms suspected or isolated, eg Hickman line sepsis).
Check local preferences. Continue until afebrile for 72h or 5d course, and until
neutrophils >0.5109/L. If fever persists despite antibiotics, think of CMV, fungi (eg
Candida; Aspergillus) and central line infection.

Consider treatment for Pneumocystis eg co-trimoxazole, ie trimethoprim


20mg/kg + sulfamethoxazole 100mg/kg/day PO/IV in 2 daily doses). Remember TB.
Avoid IM injections as they can lead to hematomas.
In this question the neutrophil count is 0.6 and temp is 37.6. So we should still start Iv
antibiotics as they are dangerously close the ranges given in the text above.
Granulocytes colony stimulating factors are used to produce neutrophils and is used in
preventing sepsis but the patient here is almost in sepsis! GCSF are mostly used in
myeloproliferative disorders.
175. A 25yo woman with T1DM has delivered a baby weighing 4.5kg. Her uterus is well
contracted. Choose the single most likely predisposing factor for PPH from the options?
a. Atonic uterus
b. Cervical/vaginal trauma
c. Retained POC
d. Large placental site
e. Rupture uterus
Q. 1. What is the key?
Q. Reason for your answer.
Ans. 1. The key is B. Cervical/vaginal trauma
Ans. 2. The baby is a big baby. If patients uterus was not well contracted we would fear
of atonic uterus! But as uterus is well contracted it is not atonic uterus. Rather most likely
cause is trauma dring delivery of this big baby.
Primary PPH is the loss of greater than 500mL (definitions vary) in the first 24h after
delivery

Causes: uterine atony (90%), genital tract trauma (7%), clotting disorders(3%)
Risks: Antenatal Previous PPH or retained placenta BMI>35kg/m2 Maternal
Hb<8.5g/dl at onset of labour Antepartum haemorrhage Multiparity 4+ Maternal age
35y+ Uterine malformation or fibroids A large placental site (twins, severe rhesus
disease, large baby) Low placenta, Overdistended uterus (polyhydramnios, twins)
Extravasated blood in the myometrium (abruption).
In labour Prolonged labour (1st, 2nd or 3rd stage) Induction or oxytocin use
Precipitant labour Operative birth or caesarean section. Book mothers with risk factors
for obstetric unit delivery.
Treatment: Give oxytocin 5U slowly IV for atonic uterus.
Attach oxygen, Give IV fluids, maintain systolic >100mmHg, Transfuse blood.
Is the placenta delivered? If it is, is it complete? If not, explore the uterus. If the
placenta is complete, put the patient in the lithotomy position with adequate analgesia and
good lighting. Check for and repair trauma.
If the placenta has not been delivered but has separated, attempt to deliver it by
controlled cord traction after rubbing up a uterine contraction. If this fails, ask an
experienced obstetrician to remove it under general anaesthesia.Beware renal shut down.
176. A 23yo lady presents with headache. Exam: photophobia and generalized rash that
doesnt
blanch on pressure. What must be done immediately?
a. IV benzylpenicillin

b. Isolate pt
c. Gown and mask
d. Blood culture
Ans. The key is A. IV benzylpenicillin.
When to act: Headache, pyrexia, neck stiffness, altered mental state: if any 2 co-exist
and not yet in hospital, give benzylpenicillin 1.2g IM/IV before admitting.
Organisms: Meningococcus or pneumococcus. Less commonly Haemophilus
influenzae;
Listeria monocytogenes. CMV, cryptococcus or TB if immunocompromised eg HIV +ve
organ transplant, malignancy.
Features
Early: Headache, leg pains, cold hands and feet, abnormal skin colour.
Later:
Meningism: neck stiffness, photophobia, Kernigs sign (pain + resistance on passive
knee extension with hip fully flexed).
Decreased Conscious level , coma.
Seizures (~20%) focal CNS signs (~20%) opisthotonus
Petechial rash (non-blanching ; may only be 1 or 2 spots, or none).
Signs of galloping sepsis: slow capillary refill; DIC; dec BP. inc T and pulse: inc or
normal.
Management:
Start antibiotics immediately.
<55yrs: cefotaxime 2g/6h slow IV.
>55yrs: cefotaxime as above + ampicillin 2g IV/4h (for Listeria).
Prophylaxis: (discuss with public health/ID) Household contacts in droplet range. Give
rifampin or ciprofloxacin.
If in this question there was an option for IV cefotaxime that would have been the correct
answer since here the patient has presented in the hospital. Benzylpenicilline is given
before admission or before taking the patient to the hospital
177. A 4yo baby has generalized tonic-clonic seizure and fever of 39C. his mother
informs you that this has happened 3-4x before. What is the most likely dx?
a. Febrile convulsion
b. Absence seizures
c. Epilepsy
d. Partial complex seizure
Ans: The key is C. Epilepsy! Probably wrong key! Epilepsy doesnt occur with fever!
Likely correct key is A. Febrile convulsion.
FEBRILE CONVULSION is a single tonicclonic, symmetrical generalized seizure
lasting <20min, occurring as T rises rapidly in a febrile illnesstypically in a normally
developing child (5yrs old).
Think of meningo-encephalitis, CNS lesion, epilepsy, trauma, hypoglycemia, dec
Ca2+, or dec Mg2+ if: Focal CNS signs or CNS abnormality Previous history of
epilepsy The seizure lasts >15min There is >1 attack in 24h.

Examination: Find any infection; if any neck stiffness consider meningitis. :


Management: Put in recovery position; if fit is lasting >5min: lorazepam IV, buccal
midazolam or diazepam PR. Tepid sponging if hot; paracetamol syrup
Labs: Consider FBC, U&E, Ca2+, glucose, MSU, CXR, ENT swabs.
Avoid LP in the postictal period. If you suspect meningitis, then treat now.
Parental education: Allay fear (a child is not dying during a fit). For the 30% having
recurrences, teach carers to use buccal midazolam or rectal diazepam 0.5mg/ kg
Further prevention: Diazepam PR during fevers has a role; other anticonvulsants are
never needed. Explain that all fevers (eg vaccination-associated) should prompt oral
antipyretics.
Prognosis: In typical febrile convulsions there is no progress to epilepsy in 97%.
178. A middle aged Asian presents with episodes of fever with rigors and chills for last
1y. Blood film: ring form plasmodium with schaffners dots in RBCs. What is the drug to
eradicate this infection?
a. Doxycycline
b. Mefloquine
c. Proguanil
d. Quinine
e. Artesonate
Q. 1. What is the key?
Q. 2. What does Shuffners dot in RBC indicate?
Ans. 1. The key is B. Mefloquine.
Ans. 2. Shuffners dot indicates, it is plasmodium ovale or plasmodium vivax infestation.
MALARIA:
Plasmodium vivax and ovale: cyclical fever every 48 hours.
P.malariae: Cyclical fever every 72 hours. Can cause glomerulonephritis. Rarely fatal.
P. falciparum: fevr 36-48hrs cyclical. Fulminant disease.
Presentation: 3 phases:
1 Shivering (1h): I feel so cold.
2 Hot stage (26h): T 41C, flushed, dry skin; nausea/vomiting; headache.
3 Sweats (~3h) as T falls
Also malaise, fatigue, anorexia, myalgias...
Signs: Anaemia, jaundice, and hepatosplenomegaly. No rash or lymphadenopathy
Protective factors: G6PD lack; sickle- cell trait; melanesian ovalocytosis;
Complications: Hemolytic anemia can occur.
5 grim signs: 1 dec Consciousness/coma (cerebral malaria) 2 Convulsions 3 Coexisting
chronic illness 4 Acidosis (eg esp bad if HCO3 <15mmol/L) 5 Renal failure
Diagnosis: Serial thin & thick blood films.
P. vivax ring forms partly hidden by Schuffners dots. P. malariae: ring and band forms
P. falciparum: sausage-like gametocytes in RBC ghosts.
Treatment: If the patient has taken prophylaxis, dont use the same drug for treatment.
If species unknown or mixed infection, treat as P. falciparum. Nearly all P. falciparum is
resistant to chloroquine and in many areas also to Fansidar (pyrimethamine +
sulfadoxine).
Chloroquine is 1st choice for benign malarias in most parts of the world. Never rely on
chloroquine if used singly for prophylaxis.
Treating uncomplicated P. ovale, P. vivax, & P. malariae: Chloroquine base: 10mg/
kg, then 5mg/kg at 6h, 24h and 48h. In resistant cases, try Malarone (atovaquone and
proguanil) ,quinine, or Riamet. Primaquine dose in P. vivax: 500g/kg (max.30mg)

daily for 14d; P. ovale: 250g/kg (max. 15mg) daily for 14dgiven after chloroquine to
treat liver stage and prevent relapse. Screen for G6PD deficiency first. CI: pregnancy. P.
malariae does not need primaquine.
Treating uncomplicated P.falciparum malaria:
Artemether-lumefantrineif >35kg: 4 tabs stat, then 4 tablets at 8, 24, 36, 48 and 60h.
Artesunate-amodiaquine; if a fixed combination pill is available.
Dihydroartemisinin-naphthoquine Dihydroartemisinin piperaquine. Atovaquoneproguanil. can be used.
In pregnancy: Artemisinins are OK in children and pregnancy from 13 weeks;
(use quinine + clindamycin in 1st trimester).
In addition give symptomatic treatment for fever, blood transfusion if required.

Prophylaxis:
If little/no chloroquine resistance: Proguanil 200mg/24h+chloroquine base 300mg/wk.
If chloroquine-resistant P. falciparum: Mefloquine 250mg/wk (18d before to 4wks after
trip) or doxycycline 100mg/d (1d before to 4wks after) or atovaquone 250mg + proguanil
100mg (Malarone) 1 tab/d (1d before travel to 7d after).

179. A 35yo woman had an uneventful lap chole 18h ago. She has a pulse=108bpm,
temp 37.8C. There are signs of reduced air entry at the right base but the CXR doesnt
show an obvious abnormality. What is the most appropriate management strategy?
a. Cefuroxime PO
b. Ceftriaxone IV
c. Chlorpheniramine PO
d. Chest physiotherapy
e. Reassure
Q. 1. What is the key?
Q. 2. What is the diagnosis?
Ans. 1. The key is D. Chest physiotherapy.
Ans. 2. Atelactasis.
Best visible on CT scan and not on chest xray. Mostly occurs as a complication of
anaesthesia.
Arrange physiotherapy and antibiotics.

180. A 20yo pop star singer complains of inability to raise the pitch of her voice. She
attributes this to the thyroid surgery she underwent a few months back. What is the most
likely dx?
a. Thyroid storm
b. Bilateral recurrent laryngeal nerve injury
c. Unilateral recurrent laryngeal nerve injury
d. External laryngeal nerve injury
e. Thyroid cyst
Ans. The key is D. External laryngeal nerve injury.

Complications of thyroid surgery:


1. Bleeding, which may cause tracheal compression.
2. Recurrent laryngeal nerve injury:
Innervates all of the intrinsic muscles of the larynx, except the cricothyroid muscle.
Patients with unilateral vocal fold paralysis present with postoperative hoarseness.
Presentation is often subacute and voice changes may not present for days or
weeks. Unilateral paralysis may resolve spontaneously.
Bilateral vocal fold paralysis may occur following a total thyroidectomy and usually
presents immediately after extubation. Both vocal folds remain in the paramedian
position, causing partial airway obstruction.
Superior (external) laryngeal nerve injury:
o The external branch provides motor function to the cricothyroid muscle.
o Trauma to the nerve results in an inability to lengthen a vocal fold and
thus to create a higher-pitched sound.
o The external branch is probably the most commonly injured nerve in thyroid
surgery.
o Most patients do not notice any change but the problem may be careerending for a professional singer.
Hypoparathyroidism: the resulting hypocalcaemia may be permanent but is
usually transient. The cause of transient hypocalcaemia postoperatively is not
clearly understood.
Thyrotoxic storm: is an unusual complication of surgery but is potentially lethal.
Infection: occurs in 1-2% of all cases. Peri-operative antibiotics are not
recommended for thyroid surgery.
Hypothyroidism.
Damage to the sympathetic trunk may occur but is rare.
181. A 28yo woman at 39wk gestation is in labor. She develops abdominal pain and
HR=125bpm, BP=100/42mmHg, temp=37.2C and saturation=99%. Exam: lower
abdomen is exquisitely tender. CTG=prv normal, now showing reduced variability and
late deceleration develops with slow recovery. She has had 1 prv LSCS for a breech
baby. Choose the most appropriate CS
complication for this lady?
a. Endometritis
b. UTI
c. Urinary tract injury
d. Pleurisy
e. Uterine rupture
Ans. The key is E. Uterine rupture.
Uterine rupture: Its an obstetrical emergency
Causes: ~70% of UK ruptures are due to dehiscence of caesarean section scars.
Other risk factors: Obstructed labour in the multiparous, especially if oxytocin is used
Previous
cervical surgery High forceps delivery Internal version Breech extraction. Rupture is
usually during the third trimester or in labour.

Vaginal birth after caesarean (trial of scar): Vaginal birth will be successful in 72
76%. Endometritis, need for blood transfusion, uterine rupture and perinatal death are
commoner than repeated elective C section.
Signs and symptoms Rupture is usually in labour. In a few (usually a caesarean scar
dehiscence) rupture precedes labour. Pain is variable, some only having slight pain and
tenderness over the uterus. In others pain is severe.
Vaginal bleeding is variable and may be slight (bleeding is intraperitoneal). Unexplained
maternal tachycardia, sudden maternal shock, cessation of contractions, disappearance
of the presenting part from the pelvis, and fetal distress are other presentations.
Postpartum indicators of rupture: continuous PPH with a well-contracted uterus; if
bleeding continues postpartum after cervical repair; and whenever shock is present.
Management If suspected in labour, perform laparotomy, deliver the baby by caesarean
section, and explore the uterus. If rupture is small Repair or if vagina or cervix are
involved in the tear hysterectomy may be needed.
182. An 8m infant presented with FTT and constipation. Exam: large tongue and fam hx
of prolonged neonatal jaundice. What is the most likely dx?
a. Downs syndrome
b. Fragile X syndrome
c. Praderwilli syndrome
d. DiGeorge syndrome
e. Congenital hypothyroidism
Q. 1. What is the key?
Q. 2. What are the points in favour?

Congenital Hypothyroidism: Thyroid hormone is necessary for growth and


neurological development.
Signs: May be none at birthor prolonged neonatal jaundice, widely opened posterior
fontanelle, poor feeding, hypotonia, and dry skin are common. Inactivity, sleepiness,
slow feeding, little crying, and constipation may occur. Look for coarse dry hair, a flat
nasal
bridge, a protruding tongue, hypotonia, umbilical hernia, slowly relaxing reflxes, pulse,
and poor growth and mental development if it has not been picked up. Other later signs:
dec IQ, delayed puberty (occasionally precocious), short stature, delayed dentition.
Universal neonatal screening: Cord blood or filter paper spots (at ~7 days, from heel
prick) allow early diagnosis (theGuthrie card).
Tests: Decreased T4, Increased TSH (but undetectable in secondary hypothyroidism),
Decreased I131 uptake, dec Hb. Bone age is less than chronological age. As it is unwise
to X-ray the whole skeleton, the left wrist and hand are most commonly used.
Treatment: Levothyroxine (LT4): Start neonates with ~15g/kg/day; adjust by 5g/kg
every 2
weeks to a typical dose of 2050g/day. Avoid high TSH levels.

Ans.1. The key is E. Congenital hypothyroidism.


Ans. 2. Points in favour:i) FTT ii) constipation iii) macroglossia iv) prolonged neonatal
jaundice.
183. A 3m infant has presented with recurrent infections. He has abnormal facies and
CXR shows absent thymic shadow. What is the most likely dx?
a. Downs syndrome
b. Fragile X syndrome
c. DiGeorge syndrome
d. Marfans syndrome
Q. 1. What is the key?
Q. 2. What are the points in favour?
Ans. 1. The key is C. DiGeorge syndrome.
Ans. 2. Points in favour: i) Early age of onset ii) abnormal facies iii) absent thymic
shadow on Chest X-ray iii) history of recurrent infection [in newborne can be recognized
by convulsions from hypocalcaemia due to malfunctioning parathyroid glands and low
level of parathyroid hormones].

DiGeorges syndrome A deletion of chromosome 22q11.2 causes absent thymus,


fits, small parathyroids ( decreased Ca2+), anaemia, lymphopenia, dec growth hormone,
dec T-cell-immunity. It is related to velo-cardiofacial syndrome: characteristic
face, multiple anomalies, eg cleft palate, heart defects, cognitive defects
Management:
Cardiac defects are the usual focus of clinical management.
Hypocalcaemia should be screened for by checking calcium levels three-monthly in infancy
and then annually. Low calcium and high phosphate levels should prompt further testing of
parathyroid hormone and vitamin D levels.
All patients should have baseline immunological testing and annual blood count
Cleft palates may be submucous. In particular, they should be sought if feeding difficulties
are encountered in the neonatal period.
Gastro-oesophageal reflux needs to be managed appropriately with feed thickeners and antireflux medication. Nasogastric tube feeding and occasionally gastrostomy may be needed to
deal with feeding issues.

184. A 30yo man presents with deep penetrating knife wound. He said he had TT when
he left school. What will you do for him now?
a. Human Ig only
b. Human Ig and TT
c. Full course of tetanus vaccine only
d. Human Ig and full course of tetanus vaccine
e. Antibiotic
Ans. The key is B. Human Ig and TT.

185. A 32yo previously healthy woman has developed pain and swelling of both knees
and ankles with nodular rash over her shins. As part of the inv a CXR has been
performed. What is the single
most likely CXR appearance?
a. Apical granuloma
b. Bilateral hilar lymphadenopathy
c. Lobar consolidation
d. Pleural effusion
e. Reticular shadowing in the bases
Q. 1. What is the key?
Q. 2. What is the name of this condition? What are the points in favour?
Ans. 1. The key is B. bilateral hilar lymphadenopathy.
Ans. 2. The name is Lofgrens syndrome. It is the triad of i) erythema nodosum ii)
bilateral hilar lymphadenopathy and iii) arthralgia.
Apical granuloma: apical granuloma modified granulation tissue containing elements of
chronic inflammation located adjacent to the root apex of a toothwith infected necrotic pulp.
Lobar consolidation: pneumonia

Sarcoidosis:
Presentation: Lungs are in involved in more than 90% cases of sarcoidosis. There is
interstitial lung disease. The painful skin lesion is erythema nodosum. Also look for
Lupus pernio (chronic raised hardened, often purple lesion) may be seen on the face.
Lofgren syndrome is often a part of sarcoidosis. The triad is i) Erythema nodosum
ii) Bilateral hilar lymphadenopathy iii) Arthralgia
Sarcoidosis is a multisystem disease and can involve any system/organ
Tests: ESR is often raised. Serum ACE enzyme levels are raised in 60% of times
Plain CXR may show bilateral hilar or paratracheal lymphadenopathy. High resolution
CT should be done. There will be restricitve pattern of disease on pulmonary function
tests.

Transbronchial biopsy can demonstrate the presence of non-caseating granulomata,


giving a more accurate diagnosis
Bronchioalveolar lavage may also be done
186. A neonates CXR shows double bubble sign. Exam: low set ears, flat occiput. What
is the most likely dx?
a. Downs syndrome
b. Fragile X syndrome
c. Turners syndrome
d. DiGeorge syndrome
Q. 1. What is the key?
Q. 2. What double bubble sign indicate?
Ans. 1. The key is A. Downs syndrome.
Ans. 2. Double bubble sign indicate duodenal atresia.
Downs syndrome:
Causes:
Non-disjunction of chromosome >88%
Mosaicism <8%
The robertsonian translocation trisomy 21 is the cause in 4% of Downs syndrome
Presentation:
simian palmar crease, hypotonia, flat face/round head, protruding tongue, broad hands,
upward slanted palpebral fissures and epicanthic folds, speckled irises (Brushfield
spots); mental and growth retardation; pelvic dysplasia, cardiac malformations, short,
broad hands, hypoplasia of middle phalanx of (incurving) 5th finger, intestinal atresia and
high arched palate.
.

Associated problems: Duodenal atresia; VSD; patent ductus; AVSD (foramen primum
defects, and, later, a low IQ and a small stature.
Pr enatal diagnosis:
The combined test: combines nuchal translucency (NT) + free -human chorionic
gonadotrophin (HCG) + pregnancy associated plasma protein (PrAP-A or PAPP-A) + the
womans age. Used between 10 weeks 3 days and 13 weeks 6 days. It achieves detection
rates of 95% of all aneuploides, 86% trisomy-21, and 100% of trisomy-18 and trisomy-13
The quadruple test combines maternal -fetoprotein (AFP) + unconjugated estriol + free
HCG or total HCG + inhibin-A + the womans age in the 2nd trimester. Use between 15
weeks + 0 days and 20 weeks + 0 days so useful for women presenting in the 2nd
trimester
The integrated test: It involves NT + PrAP-A in the 1st trimester + the quadruple test in
the 2nd trimester. Do not use 2nd trimester tests for triplets
187. A 19yo boy complains of itching on the site of insect bite. What is the single most
appropriate management?
a. Penicillin oral

b. Doxycycline oral
c. Oral antihistamine
d. Oral ciprofloxacin
e. Reassurance
Ans. The key is C. Oral antihistamine.
This is a case of simple physical utricaria follwing an insect bite which causes itching.
For which oral antihistamine is most suitable.
188. A man presents with scrotal swelling, the swelling is cystic and is non-tender. It is
located in the upper pole of the posterior part of the testis. What is the most likely dx?
a. Epididymal cyst
b. Testicular ca
c. Hydrocele
d. Teratoma
e. Testicular torsion
Ans. The key is A. Epididymal cyst. [the location of upper pole of the posterior part of
testis is the common site for epididymal pathology].
Diagnosing scrotal masses
Cannot get above inguinoscrotal hernia or hydrocele extending proximally
Separate and cystic epididymal cyst
Separate and solid epididymitis/varicocele
Testicular and cystic hydrocele
Testicular and solidtumour, haematocele, granuloma, orchitis, gumma. USS may help.
Epididymal cysts usually develop in adulthood and contain clear or milky
(spermatocele)
fluid. They lie above and behind the testis. Remove if symptomatic
189. A young footballer has collapsed during a game. During initial evaluation:
RR=14/min,
pulse=88bpm, BP=110/70mmHg. He seems to be sweating and muttering some
incomprehensible words. What is the most imp next step?
a. CT
b. MRI
c. Blood sugar
d. Body temp
e. IV fluids
Ans. The key is C. Blood sugar.
The collapse could be because of hypoglycemia. So we should BSR of the patient first
before taking him for CT scan or any other investigation as it is easy and fast and could
give us a cause of collapse that is quickly reversible.
Even in managing a patient with COMA or seizures. after ABC first thing to do is to
check BSR.
The patient here most probably had an attack of SYNCOPE. If in the question there was
an option for ECG then that would have been the right answer.
Syncope:
Causes:

Neurally mediated syncope (NMS) - also called reflex syncope of which vasovagal is the
most important cause of syncope.
Orthostatic hypotension
Cardiac arrythmias
Structural cardiac or cardiopulmonary disease
Substance abuse
Psychogenic
Presentation:
In some forms of syncope there may be a prodromal period with lightheadedness, nausea, sweating, weakness or visual disturbances, but loss of
consciousness often occurs without warning.
Syncope is usually brief, with complete loss of consciousness in reflex syncope
not lasting more than 20 seconds (but may occasionally be up to several
minutes). Recovery from syncope is usually associated with almost immediate
restoration of appropriate behaviour and orientation, but there may be marked
fatigue. Retrograde amnesia may occur, especially in older individuals.
Investigations: The initial evaluation of a patient presenting with transient loss of
consciousness consists of careful history, physical examination, including orthostatic blood
pressure measurements, and an ECG. Further investigations :

FBC: acute anaemia will cause syncope, but patients adapt in cases of chronic anaemia.
Fasting blood glucose (hypoglycaemia).
Management: Reassure and if an underlying cause is found treat it.
190. A 45yo waitress complains of pelvic pain which worsens pre-menstrually and on
standing and walking. She also complains of post-coital ache. Select the most likely
cause leading to her
symptoms?
a. PID
b. Endometritiosis
c. Pelvic congestion syndrome
d. Adenomyosis
e. Premature ovarian failure
Ans. The key is C. Pelvic congestion syndrome.
PID mostly presents with pelvic pain, fever, spasm of lower abdominal muscles and
cervicitis with profuse, purulent, or bloody vaginal discharge. Heavy menstrual loss
suggests endometritis.
Endometriosis can present as given in this question but it will have menstrual
abnormalities too and the pain will not be related to any specific activity.

Pelvic Congestion Syndrome:


Condition is characterised by the presence of dilated pelvic veins associated with stasis

Aetiology
Considered that ovarian dysfunction is responsible for the excessive production of local
oestrogen, causing dilatation and stasis in the pelvic veins, which leads to pelvic pain
Presentation:
women with this condition commonly complain of a dull, aching pain, exacerbated by
activities that increase intra-abdominal pressure; the pain is relieved by lying down.
other clinical features may also be deep dyspareunia, congestive dysmenorrhoea and
post-coital ache condition usually occurs in the reproductive age group, with a mean age
of 33 years.
Examination may reveal tenderness that is maximal over the ovaries. Vaginal and
cervical examination may reveal an apparent blue colouration due to congestion of the
pelvic veins. The patient may also have varicose veins of the legs
Investigations for endometriosis and pelvic inflammatory disease must be instigated
Venography is still considered the definitive radiological investigation for women with
pelvic congestion syndrome
Radiological features: dilated uterine and ovarian veins with reduced venous clearance
of contrast medium
Management
Medical treatment options include:
Medroxyprogesterone acetate (MPA) suppresses ovarian function and therefore reduces
pelvic congestion and pain, however benefit was not sustained after discontinuing
treatment (1)
Gonadorelin analogues goserelin 3.6 mg per month given for 6 months provided an
alleviation of symptoms, an improvement in sexual functioning and a reduction of anxiety
and depressive states in women with pelvic congestion
Other possible treatment options include:
Bilateral ovarian vein ligation
Hysterectomy plus bilateral salpingo-oophrectomy (with post-operative hormone
replacement therapy)
191. A 37yo female had a fall with outstretched hand, presented with dinner fork
deformity and
tenderness over the right arm. What is the single most associated nerve injury?
a. Axillary nerve
b. Radial nerve
c. Musculocutaneous nerve
d. Median nerve
e. Ulnar nerve
Ans. The key is D. Median nerve. [Median nerve is the nerve injured in Colles fracture].
Distal radial fractures Colles type Fracture: (common in osteoporotic post-menopausal
women who fall on an outstretched hand).
There is dorsal angulation and displacement producing a dinner-fork wrist deformity
(the fingers are the prongs). Avulsion of the ulna styloid process may also occur.

Treatment:
For reduction, Biers block method (= IV regional anaesthesia) is best.
Complications: Median nerve symptoms (should resolve after good reduction); ruptured
tendons
(esp. extensor pollicis longus); malunion & nonunion.
192. A mother comes with her 15m child. Which of the following will bother you?
a. Shies away from strangers
b. Can walk but not run
c. Vocabulary consists of only 2 meaningless words
d. She cant make a sentence
e. None
Q. 1. What is the key?
Q. 2. How many words is told clearly by a 15 month old child?
Ans. 1. The key is C. Vocabulary consists of only 2 meaningless words.
Ans. 2. At 15 months the child can clearly say 5 words and his first meaningful clear
word he says at 12 months.

Developmental Milestones.

193. A 35yo lady who has been using IUCD for one year now complains of pelvic pain
and heavy
painful periods. Select the most likely cause leading to her symptoms?
a. PID
b. Endometriosis
c. Adenomyosis
d. Fibroids
e. Asherman syndrome
Q. 1. What is the key?
Q. 2. What points favour your diagnosis?
Ans. 1. The key is A. PID.

Ans. 2. The given picture may have D/D of PID or fibroid. As IUCD is a risk factor for
PID, it is the most likely diagnosis of given picture.

Fibroids are not related to IUCD. Also in fibroids there will be findings on examination
such as mass in lower abdomen.

194. The dx cells of Hodgkin disease are:


a. T-cells
b. R-S cells
c. B-cells
d. Macrophages
e. Auer rods
Ans. The key is B. R-S cells. [Diagnostic cell in Hodgkins disease is Reed-Sternberg
cells].
Hodgkins lymphoma: Lymphomas are disorders caused by malignant proliferations of
lymphocytes. These accumulate in the lymph nodes causing lymphadenopathy.In
Hodgkins lymphoma,characteristic cells with mirror-image nuclei are found, called
ReedSternberg cells
Cause: 2 peaks of incidence: young adults and elderly. EBV has a role in causing it.
Symptoms Often presents with enlarged, painless, non-tender, rubbery superficial
lymph nodes, typically cervical (6070%), also axillary or inguinal nodes. Nodes may
become matted. 25% have constitutional symptoms.
Mediastinal lymph node involvement can cause features due to mass effect, eg
bronchial or SVC obstruction or direct extension, eg causing pleural effusions.
Signs Lymph node enlargement. Also, cachexia, anaemia, spleno- or hepatomegaly.
Types:

Tests Tissue diagnosis: Lymph node excision biopsy if possible. Inc ESR or dec Hb
show worse prognosis.
Staging is by Ann-Arbor system.
Treatment: Depends on chemotherapy and radiotherapy.
195. A 16yo girl is admitted after taking a paracetamol OD 4 h ago. She has consumed
large amounts of alcohol. Her plasma paracetamol conc is just below the conc that
would suggest tx. What should be the tx option for her?
a. Refer to psychiatry ward
b. Refer to medical ward
c. N-acetylcystine
d. Serum plasma paracetamol
e. No further investigation
Ans. The key is A. Refer to psychiatry ward.
Chronic alcohol consumption is an inducer of P-450 enzyme system while acute alcohol
consumption is inhibitor. Since this lady has consumed large amounts of alcohol recently
(acute) the risk of fatal effects of paracetamol poisoning will be reduced. And all such
patients should be referred to the psych ward.
PARACETAMOL POISONING: >150mg/kg or 12 total
PRESENTATION: Hepatic damage shown by deranged LFTs occurs after 24hrs.
Patients may develop encaph, hypoglycemia, ARF

INVESTIGATIONS: Paracetamol levels: 4hrs post ingestion, if time is >4hr or staggered


overdose
Any alcohol taken (acute alcohol ingestion will inhibit liver enzymes and may reduce the
production of the toxin NAPQI, whereas chronic alcoholism may increase it)

MANAGEMENT:
If presentation is within the first hour give activated charcoal
All patients who have a timed plasma paracetamol level plotted on or above the
line drawn between 100 mg/L at 4 hours and 15 mg/L at 15 hours after ingestion,
should receive acetylcysteine.
If time unknown (even in staggered dose) give N-Acetyl cysteine without delay
NAC most effective in the first 8 hrs.
NAC can be given during pregnancy
Beware if the patient is on any P450 enzyme inducer medicines as they increase
the toxicity
Refer to ICU if there is fulminant liver failure - those treated with N-acetylcysteine
(NAC) to the medical team and all para-suicides to the psychiatric team.

196. A 64yo woman has been on HRT for 9yrs. She had regular withdrawal bleeds until
3 yrs ago and since then has been taking a no bleed prep. Recently she noticed a brown
vaginal discharge.
Choose the single most appropriate initial inv?
a. Cervical smear
b. High vaginal swab
c. TFT
d. Transvaginal US
Q. 1. What is the key?
Q. 2. Why this test will be done?
Ans. 1. The key is D. Transvaginal US.
Ans. 2. To determine the endometrial thickness!
In a postmenopausal woman with vaginal bleeding, the risk of cancer is approximately 7.3% if her
endometrium is thick (> 5 mm) and < 0.07% if her endometrium is thin ( 5 mm).
In postmenopausal women without vaginal bleeding, the risk of cancer is approximately 6.7% if the
endometrium is thick (> 11 mm) and 0.002% if the endometrium is thin ( 11 mm).

Investigate postmenopausal vaginal bleeding promptly as the cause may be


endometrial cancer.
Endometrial Carcinoma:
Most are adenocarcinomas, and are related to excessive exposure to oestrogen unopposed
by progesterone.
Risk Factors: Obesity Unopposed oestrogen Functioning ovarian tumour Family
History of breast, ovary, or colon cancer Nulliparity Late menopause Diabetes
mellitus Tamoxifen, tibolone Pelvic irradiation Polycystic ovaries.

Presentation This is usually as postmenopausal bleeding (PMB). It is initially scanty and


occasional ( watery discharge). Then bleeding gets heavy and frequent. Premenopausal
women may have intermenstrual bleeding, but 30% have only menorrhagia.
Diagnosis: TVUS scan is an appropriate first-line procedure to identify which women with
PMB are at higher risk of endometrial cancer. Endometrial thickness of >5mm warrants
biopsy. The definitive diagnosis is made by uterine sampling or curettage. All parts of

the uterine cavity must be sampled; send all material for histology. Hysteroscopy enables
visualization of abnormal endometrium to improve accuracy of sampling.
Staging The tumour is
Stage I in the body of the uterus only.
Stage II in the body and cervix only.
Stage. III advancing beyond the uterus, but not beyond the pelvis.
Stage: IV extending outside the pelvis (eg to bowel and bladder).
Treatment: Stages I and II may be cured by total hysterectomy with bilateral salpingooophorectomy and/or radiotherapy if unfit for surgery. In advanced diseases consider
radiotherapy and/or high dose progesterone which shrinks the tumor.

197. A

young girl complains of episodic headaches preceded by fortification spectra.


Each episode
last for 2-3 days. During headache pt prefers quiet, dark room. What is the tx of choice
for acute
stage?
a. Paracetamol
b. Aspirin
c. Sumatriptan
d. Gabapentin
e. Cafergot
Ans. The key is B. Aspirin. [OHCM, 9 Eition, page-462 where NSAIDS like ketoprophen
or dispersible aspirin 900 mg/6 hr are recommended as treatment in acute stage].
th

Migraine:
Symptoms Classically: Visual or other aura lasting 1530min followed within 1h by
unilateral, throbbing headache. Or: Isolated aura without headache; Episodic severe
headaches without aura, often premenstrual, usually unilateral, with nausea, vomiting
photophobia/phonophobia (common migraine). There may be allodyniaall stimuli
produce pain: I cant brush my hair, wear earrings or glasses, or shave, its so painful.
Signs: None.
Aura: Visual: chaotic cascading, distorting, melting and jumbling of lines, dots, or
zigzags, scotomata or hemianopia; Somatosensory: paresthesia spreading from fingers
to face; Motor: dysarthria and ataxia (basilar migraine), ophthalmoplegia, or
hemiparesis; Speech: (8% of auras) dysphasia or paraphasia, eg phoneme
substitution.

Criteria for diagnosis if no aura 5 headaches lasting 472h + nausea/vomiting (or


photo/phonophobia) + any 2 of: Unilateral Pulsating Impairs (or worsened by)
routine activity.
Partial triggers Seen in 50%: CHOCOLATE or: Chocolate, Hangovers, Orgasms,
Cheese, Oral contraceptives, Lie-ins, Alcohol, Tumult, or Exercise.
Treatment:
Acute:

Step one: simple analgesic with or without anti-emetic In patients who have tried step
1 and didnt respond and in patients with moderate-to-severe migraine, move to step
three.

Use early in the attack to avoid gastric stasis.


Use soluble aspirin 600-900 mg (not in children) or ibuprofen 400-600 mg.
Use prochlorperazine 3 mg buccal tablet if there is nausea and vomiting.

Step two: rectal analgesia and rectal anti-emetic.


Step three: specific anti-migraine drugs Triptans (5HT1-receptor agonists) or
ergotamine (the use of ergotamine is limited by absorption problems and side-effects
such as nausea, vomiting and abdominal pain
Triptans are Contra indicated if IHD, coronary spasm, uncontrolled BP, recent lithium,
SSRIS,
or ergot use.
Prevention Remove triggers; ensure analgesic rebound headache is not complicating
matters. Drugs eg if frequency equal or >2 a month or not responding to drugs
1st-line: Propranolol, amitriptyline (SE: drowsiness, dry mouth, vision), topiramate (SE:
memory) or Ca2+ channel blockers.
2nd-line: Valproate, pizotifen (effective, but unacceptable weight gain in some),
gabapentin, pregabalin, ACE-i, NSAIDS

198. A 60yo pt recovering from a surgery for toxic goiter is found to be hypotensive,
cyanosed in the the RR. Exam: tense neck. There is blood oozing from the drain. What
is the most likely dx?
a. Thyroid storm
b. Reactionary hemorrhage
c. Secondary hemorrhage
d. Primary hemorrhage
e. Tracheomalacia
Ans. The key is B. Reactionary haemorrhage. [in the recovery room, cyanosis,
hypotension, tense neck, oozing of blood from drain; all these goes in favour of
reactionary haemorrhage].
Primary Haemorrhage: Haemorrhage occurring at the time of Injury/Trauma/Surgery
Reactionary Haemorrhage:
Trauma/Surgery

Haemorrhage

occurring

within

first

24

hrs

following

The causes Reactionary Haemorrhage:


1)
2)
3)
4)

Slipping away of Ligatures


Dislodgement of Clots
Cessation of Reflex vasospasm
Normalization of Blood Pressure

Secondary Haemorrhage: Haemorrhage occurring after 7 -14 days after Trauma/Surgery.


The attributed cause is infection and sloughing away of the blood vessels.
The symptoms tell us that the patient is in hypovolemic shock one of the causes of which is
haemorrhage and Since here the patient is still in the recovery room this type of haemorrhage is
reactionary.
199. A 33yo man is hit by a car. He loses consciousness but is found to be fine by the
paramedics.
When awaiting doctors review in the ED he suddenly becomes comatose. What is the
most likely
dx?
a. SAH
b. Subdural hemorrhage
c. Intracerebral hemorrhage
d. Extradural hemorrhage
Ans. The key is D. Extradural haemorrhage. [Age 33 (younger age), considerable head
trauma, and lucid interval (present in bothe extradural and subdural) are the points in
favour].
Lucid interval can occur both ins Subdural and extra dural haemorrhage. The difference
is that the presentation of the lucid interval (that is the gain of consciousness and the
LOC) in extra dural occurs within hours or 1-2 days while in subdural it can take days to
weeks upto 9 months.
Epidural (extradural) haemorrhage:
Suspect this if, after head injury, conscious level falls or is slow to improve, or
there is a lucid interval. Extradural bleeds are often due to a fractured temporal or
parietal bone causing laceration of the middle meningeal artery and vein, typically
after trauma to a temple just lateral to the eye. Any tear in a dural venous sinus will
also result in an extradural bleed. Blood accumulates between bone and dura.
Presentation: Increasingly severe headache, vomiting, confusion, and fits follow,
hemiparesis with brisk reflexes and an upgoing plantar. If bleeding continues, the
ipsilateral pupil dilates, coma deepens, bilateral limb weakness develops, and breathing
becomes deep and irregular (brainstem compression). Death follows a period of coma
and is due to respiratory arrest. Bradycardia and raised blood pressure are late signs.
Tests CT scan shows a haematoma (often biconvex/lens-shaped; the blood forms a
more rounded shape compared with the sickle-shaped subdural haematoma. Skull X-ray
may be normal or show fracture lines crossing the course of the middle meningeal

vessels. Skull fracture after trauma greatly increases risk of an extradural haemorrhage
and should lead to prompt CT. Lumbar puncture is contraindicated.
Management Stabilize and transfer urgently for clot evacuation ligation of the bleeding
vessel. Care of the airway in an unconscious patient and measures to decrease ICP
often
require intubation and ventilation (+ mannitol IVI
Prognosis Excellent if diagnosis and operation early. Poor if coma, pupil abnormalities,
or decerebrate rigidity are present pre-op.

200. A 77yo male presents with hx of enuresis and change in behavior. Exam: waddling
gait. What is the most likely dx?
a. Subdural hemorrhage
b. Brain tumor
c. Normal pressure hydrocephalus
d. Psychotic depression
Ans. The key is C. Normal pressure hydrocephalus. [age (usually occurs in 60s or 70s),
loss of bladder control (enuresis), waddling gait and behavior change are all features of
normal pressure hydrocephalus].
Normal Pressure Hydrocephalus:
describes the condition of ventricular dilatation in the absence of raised CSF pressure on
lumbar puncture, characterised by a triad of gait abnormality, urinary (usually)
incontinence and dementia.
Cause: Idiopathic or it may be secondary to:
Subarachnoid haemorrhage. Meningitis. Head injury. Central nervous system (CNS)
tumour.
Symptoms: The (gradually progressive) classic triad of symptoms is:

Gait disturbance - this is due to distortion of the corona radiata by the dilated
ventricles. Movements are slow, broad-based and shuffling. The clinical
impression is thus one of Parkinson's disease, except that rigidity and tremor
are less marked and there is no response to carbidopa/levodopa. Gait
disturbance is referred to as gait apraxia.
Sphincter disturbance - this is also due to involvement of the sacral nerve
supply. Urinary incontinence is predominant although bowel incontinence
can also occur.
Dementia - this is due to distortion of the periventricular limbic system. The
prominent features are memory loss, inattention, inertia and bradyphrenia
(slowness of thought). The dementia progresses less rapidly than that seen
with Alzheimer's disease.

Signs

Pyramidal tract signs may be present.


Reflexes may be brisk.

Papilloedema is absent (but there has been found to be an association with


glaucoma, so glaucomatous optic disc changes may be noticed).

Investigations
Neuroimaging - MRI or CT scanning may show ventricular enlargement out
of proportion to sulcal atrophy and periventricular lucency.
CSF: Large-volume lumbar puncture (spinal or CSF tap test) - CSF pressure
will be normal, or intermittently raised.
Intraventricular monitoring
Management:
Medical treatment of NPH includes acetazolamide and repeated lumbar puncture.
Surgical:
The mainstay of treatment is surgical insertion of a CSF shunt. This could be to the
peritoneum, the right atrium or, more recently, via external lumbar drainage.

201. A 29yo teacher is involved in a tragic RTA. After that incident, he has been
suffering from
nightmares and avoided driving on the motorway. He has been dx with PTSD. What is
the most
appropriate management?
a. CBT
b. Diazepam
c. Citalopram
d. Dosalepin
e. Olanzepin
Ans. The key is A. CBT.
PTSD:
Symptoms: Fearful; horrified; dazed Helpless; numb, detached Emotional
responsiveness
Intrusive thoughts Derealization Depersonalization Dissociative amnesia Reliving
of events Avoidance of stimuli Hypervigilance Lack of Concentration Restlessness
Autonomic arousal: pulse; BP; sweating Headaches; abdo pains
Signs: Suspect this if symptoms become chronic, with these
signs (may be delayed years): difficulty modulating arousal; isolated-avoidant modes of
living; alcohol abuse; numb to emotions and relationships; survivor guilt; depression;
altered world
view in which fate is seen as untamable, capricious or absurd, and life can yield no
meaning
or pleasure.
Treatment: Watchful waiting for mild cases.

For severe cases: CBT or eye movement desensitization and reprocesing is done. Drug
treatment is not recommended but in case it is needed prescribe mirtazepine or
paroxetine.

202. A 5yo child presents with fever. He looks pale. His parents say he always feels
tired. On exam: orchidomegaly & splenomegaly. Labs: WBC=1.7, Hgb=7.1, Plt=44. What
is the dx?
a. ALL
b. CLL
c. AML
d. CML
e. Hodgkins
Ans. The key is A. ALL. [normally in ALL CBC shows raised WBC, low RBC and low
platelet; but it is also possible to all cell lines to be depressed, as is the presented case].
ALL: This is a malignancy of lymphoid cells, affecting B or T lymphocyte cell lines,
arresting
maturation and promoting uncontrolled proliferation of immature blast
cells, with marrow failure and tissue infiltration.
Causes: Genetic susceptibility, environmental factors (ionizing radiations) Downs
syndrome.
Commonest cancer of childhood.

Signs and symptoms:


Marrow failure: Anaemia (Hb), infection (WCC), and bleeding (platelets).
Infiltration: Hepatosplenomegaly, lymphadenopathysuperficial or mediastinal,
orchidomegaly, CNS involvementeg cranial nerve palsies, meningism.
INVESTIGATIONS: Characteristic blast cells on blood film and bone marrow
CXR and CT scan to look for mediastinal and abdominal lymphadenopathy.
Lumbar puncture should be performed to look for CNS involvement.
TREATMENT: Blood transfusions, prophylactic antibiotics, IV antibiotics in case of
infection. Main stay of treatment is chemotherapy.
Prognosis Cure rates for children are 7090%; for adults only 40%
203. A 6wk child is brought in with vomiting, constipation and decreased serum K+.
What is the dx?
a. Pyloric stenosis
b. Duodenal atresia
c. Hirschsprung disease
d. Achalasia cardia
e. Tracheo-esophageal fistula
Ans. The key is A. Pyloric stenosis. [why not duodenal atresia? Pyloric stenosis is much
more commoner than duodenal atresia; in duodenal atresia the vomitus should contain
bile, which is not the case in pyloric stenosis].

Pyloric stenosis
Symptoms: Presents at 38 weeks)
with vomiting which occurs after feeds and becomes projectile (eg vomiting over
far end of cot). Pyloric stenosis is distinguished from other causes of vomiting by
the following:

The vomit does not contain bile, as the obstruction is so high.


No diarrhoea: constipation is likely (occasionally starvation stools).
Even though the patient is ill: he is alert, anxious, and always hungryand
possibly malnourished, dehydrated.
The vomiting is extremely large volume and within minutes of a feed.
Try to palpate the olive-sized pyloric mass
There may be severe water & NaCl deficit. The picture is of hypochloraemic,
hypokalaemic
metabolic alkalosis
Imaging: Ultrasound detects early, hard-to-feel pyloric tumours, but is only
needed if examination is ve. Barium studies are never needed.
Management: Correct electrolyte disturbances. Before surgery (Ramstedts
pyloromyotomy/ endoscopic surgery) pass a wide-bore nasogastric tube.
204. A 17 yo girl had an episode of seizure. Contraction of muscles started from around
the
interphalangeal joints, which spread to the muscles of wrist and elbow. Choose possible
type of
seizure?
a. Grand mal
b. Tonic clonic
c. Myoclonic
d. Absent
Ans. The key is C. Myoclonic. [seizers associated with contraction of specific muscle
group is seen in myoclonic seizers].
Types of seizures:

Primary generalized seizures Simultaneous onset of electrical discharge


throughout cortex, with no localizing features referable to only one hemisphere.
Absence seizures: Brief (10s) pauses, They do not fall but may pause in what they
are doing. Their face often looks pale with a blank expression. They may look dazed, the
eyes stare and the eyelids may flutter a little. Sometimes their head may fall down a little, or
their arms may shake once or twice. Each seizure usually starts and finishes abruptly. The
person is not aware of the absence and resumes what they were doing.

Tonicclonic seizures: Loss of consciousness. Limbs stiffen (tonic), then jerk


(clonic). May have one without the other. Post-ictal confusion and drowsiness.
Myoclonic seizures: Sudden jerk of a limb, face or trunk. The patient may be
thrown suddenly to the ground, or have a violently disobedient limb: one patient
described it as my flying-saucer epilepsy, as crockery which happened to be in
the hand would take off .
Atonic (akinetic) seizures: Sudden loss of muscle tone causing a fall, no LOC.
Infantile spasms/West syndrome: Peak age: 5 months. Clusters of head nodding
(Salaam attack) and arm jerks, every 330sec. IQ decrease in ~70%. EEG is
characteristic (hypsarrythmia).
205. 46yo man, known case of chronic GN presents to OPD. He feels well. BP =
140/90mmHg. Urine dipstick: protein ++, blood ++ and serum creatinine=106mmol/L.
Which medication can prevent the progression of this dx?
a. ACEi
b. Diuretics
c. Cytotoxic meds
d. Longterm antibiotics

e. Steroids
Ans. The key is A. ACEI. [renal impairment is delayed by ACEI].
206. A 23 yo girl presented with perioral paresthesia and carpopedal spasm 20 mins
after a huge argument with her boyfriend. What is the next step for this pt?
a. SSRI
b. Diazepam
c. Rebreath into a paper bag
d. Propranolol
e. Alprazolam
Q. 1. What is the key?
Q. 2. What is the likely diagnosis?
Ans. 1. The key is C. Rebreathin in paper bag. [hyperventilation causes CO2 washout and
respiratory alkalosis. If you continue breathing and rebreathing in paper bag it will allow CO2
concentration to rise in paper bag and as you rebreath this again and again you will regain some
washed out CO2 and thus relief to this alkalosis].
Ans. 2. The girl may have anxiety disorder when it precipitates leads to hyperventilation
syndrome.

Anxiety:
Symptoms: Tension, agitation; feelings of impending doom, trembling; a sense
of collapse; insomnia; poor concentration; goose flesh; butterflies in the
stomach; hyperventilation (so tinnitus, tetany, tingling, chest pains); headaches;
sweating; palpitations; poor appetite; nausea;
lump in the throat unrelated to swallowing (globus hystericus); difficulty in
getting to sleep; excessive concern about self and bodily functions; repetitive
thoughts and activities
Childrens symptoms: Thumb-sucking; nail-biting; bed-wetting; foodfads.
Causes Genetic predisposition; stress (work, noise, hostile home), events (losing
or gaining a spouse or job; moving house). Others: Faulty learning or secondary
gain (a husband forced to stay at home with agoraphobic wife).
Treatment:
Symptom control: Listening is a good way to reduce anxiety. Explain that
headaches are not from a tumour, and that palpitations are harmless. Regular
(non-obsessive!) exercise: Beneficial effects appear to equal meditation or
relaxation.
Meditation: Intensive but time-limited group stress reduction intervention based
on mindfulness meditation can have long-term beneficial effects.
Cognitivebehavioural therapy and relaxation appear to be the best specific
measures with 5060% recovering over 6 months.
Behavioural therapy employs graded exposure to anxiety-provoking stimuli.
Drugs augment psychotherapy: 1 Benzodiazepines (eg diazepam) 2 SSRI eg
paroxetine in social anxiety). 3 Azapirones (buspirone, 5HT1A partial agonist;
ess addictive/sedating than diazepam, and few withdrawal issues). 4 Old-style
antihistamines (eg hydroxyzine).
5. Beta blockers.6 Others: pregabalin and venlafaxine.
Progressive relaxation training: Teach deep breathing using the diaphragm,
and tensing and relaxation of muscle groups, eg starting with toes and working
up the body. Practice is essential.
Hypnosis

207. A 25 yo woman has been feeling anxious and nervous for the last few
months. She also
complains of palpitations and tremors. Her symptoms last for a few minutes and
are very hard
to control. She tells you that taking alcohol initially helped her relieve her
symptoms but now
this effect is wearing off and she has her symptoms even after drinking alcohol.
What is the dx?
a. Panic disorder
b. Depression
c. OCD
d. Alcohol addiction
e. GAD
Ans. The key is A. Panic disorder.
Panic Attack:
This condition often co-exists with agoraphobia - the avoidance of exposed situations for
fear of panic or inability to escape

Panic attacks must be associated with >1 month's duration of subsequent,


persisting anxiety about recurrence of the attacks, the consequences of the
attacks, or significant behavioural changes associated with them.
A panic attack is defined as a discrete episode of intense subjective fear, where
at least four of the characteristic symptoms, listed below, arise rapidly and peak
within 10 minutes of the onset of the attack:

Attacks usually last at least 10 minutes but their duration is variable.

The symptoms must not arise as a result of alcohol or substance misuse,

medical conditions or other psychiatric disorders, in order to satisfy the


diagnostic criteria.
Panic disorder manifests as the sudden, spontaneous and unanticipated
occurrence of panic attacks, with variable frequency, from several in a day to just
a few per year:

Palpitations, pounding heart or accelerated heart rate.

Sweating.

Trembling or shaking.

Dry mouth.

Feeling short of breath, or a sensation of smothering.

Feeling of choking.

Chest pain or discomfort.

Nausea or abdominal distress.

Feeling dizzy, unsteady, light-headed or faint.

Derealisation or depersonalisation (feeling detached from oneself).

Fear of losing control or 'going crazy'.

Fear of dying.

Numbness or tingling sensations.

Chills or hot flushes.

Signs: No specific signs


Investigations: Just to rule out any physical illness.
Management: Involve the family, find and avoid any triggers. Find out if the symptoms
are because of alcohol use and treat that.
CBT is the first line.

Drugs: Offer an SSRI licensed for this indication first-line unless contra-

indicated.

Consider imipramine or clomipramine if there is no improvement after 12

weeks and further medication is indicated


If there has been an improvement after 12 weeks, continue for 6 months after the
optimum dose has been reached
At the end of treatment, withdraw the SSRI gradually,
Step 3
Reassess the condition and consider alternative treatments.

Step 4
If two interventions have been offered without benefit, consider referral to
specialist mental health services.

208. A 2yo child is very naughty. His teacher complains that he is easily distracted. His
parents say that he cant do a particular task for a long time. He sometimes hurts himself
and breaks many things. This causes many troubles at home. What is the dx?
a. ASD

b. Dyslexia
c. ADHD
d. Antisocial personality disorder
e. Oppositional defiant
Ans. The key is C. ADHD (Attention deficit hyperreactive disorder).

Attention deficit & hyperactivity disorder (ADHD)


is the most common neurobehavioral disorder of childhood. It has prevalence of
35% in Western nations ADHD is commoner in learning-disabled children, and if
prenatal cannabis exposure. The core diagnostic criteria are: impulsivity,
inattention and hyperactivity. Not all those with ADD are hyperactive. There is no
diagnostic test
Most parents first note hyperactivity at the toddler stage, Family association is
often present. These children are at increased risk of self harm and suicide.
Management:
1st line treatment for pre-school children and school age children with
moderate ADHD/moderate impairment is parent training/education
programmes. Older children may benefit from cognitive behavioural therapy.
Drugs may be useful in school age children if non-drug treatments fail (eg
methylphenidate atomoxetine:
Severe ADHD in school age children methylphenidate and atomoxetine are
1st line
treatments so ensure referral
209. A 79 yo lady who is otherwise well recently started abdominal pain. She is afebrile
and
complains that she passed air bubbles during urination. A urethral catheter showed fecal
leakage in the urinary bag. What is the likely pathology?
a. Diuretics
b. CD
c. Rectosigmoid tumor
d. Large bowel perforation
e. UC
Ans. The key is B. CD. [debate came that Crohns disease cannot occur in 79 yrs but
this is not the case! Crohns disease can occur at any age, but is most frequently
diagnosed in people ages 15 - 35. About 10% of patients are children under age 18.
[http://www.nytimes.com/health/guides/disease/crohns-disease/risk-factors.html]. So
I think it can occur in this age also and the features support the diagnosis of CD.

NEXT PAGE!

210. A 2 month child with diarrhea and vomiting for 6 days is brought in looking lethargic.
What is the appropriate initial inv?
a. BUE
b. Random blood sugar
c. CBC
d. CXR
e. AXR
Ans. The key is A. BUE.
Diarrhea makes the child dehydrated and loss of electrolytes occur. Which are making
the the lethargic so we need to check blood Urea and electrolyes and correct the
electrolyte imbalance.
211. A 72 yo man fell while shopping and hurt his knee. His vitals are fine. He speaks in
a low voice
and is very slow to give answers. What is the most probable dx?
a. Alzheimers
b. Vascular demetia
c. TIA
d. Pseudo-dementia
e. Picks dementia
Q. 1. What is the key?
Q. 2. What are the points in favour?
Ans. 1. The key is A. Alzheimers.
Ans. 2. Points in favour: i) age 72 yrs ii) fall iii) loss or slowness of speech.
why not vascular? in vascular: i) confusion ii) disorientation iii)loss of vision
why not pseudodementia? in pseudo i) onset is short and abrupt ii associated
depression
why not picks i) dementia and aphasia
Why not TIA? In TIA complete resolution of symptom!! But here symptoms are
persistent.

Alzheimers Disease:
This is the leading cause of dementia.
Onset may be from 40yrs (earlier in Downs syndrome, in which AD is inevitable).
Presentation: Suspect Alzheimers in adults with enduring, progressive and
global cognitive impairment (unlike other dementias which may affect certain
domains but not others): visuo-spatial skill (gets lost), memory, verbal abilities
and executive function (planning) are all affected
and there is anosognosiaa lack of insight into the problems engendered by the
disease, eg
missed appointments, misunderstood conversations or plots of films, and
mishandling of money and clerical work. Later there may be irritability; mood
disturbance (depression or euphoria); behavioural change (eg aggression,
wandering, disinhibition); psychosis (hallucinations or delusions); agnosia (may
not recognize self in the mirror). There is no standard natural history. Cognitive
impairment is progressive, but non-cognitive symptoms may come and go over

months. Towards the end, often but not invariably, patients become sedentary,
taking little interest in anything.
Associations: environmental and genetic factors both play a role. Accumulation
of beta-amyloid
peptide, neurofibrillary tangles, increased numbers of amyloid plaques, and loss of the
neurotransmitter acetylcholinethe hippocampus, amygdala, temporal neocortex and
subcortical nuclei (eg nucleus basalis of Meynert) are most vulnerable. Vascular effects
are also important95% of AD patients show evidence of vascular dementia

Risk factors 1st-degree relative with AD; Downs syndrome; homozygosity for
apolipoprotein
e (ApoE) e4 allele;are some of the risk factors
Treatment:
Refer to a specialist memory service.
Acetylcholinesterase inhibitors
Donepezil
Rivastigmine. Patches are also available.
Galantamine
212. A 47 yo man met with a RTA. He has multiple injuries. Pelvic fx is confirmed. He
has not passed urine in the last 4 hrs. What is the next appropriate management for this
pt?
a. Urethral catheter
b. Suprapubic catheter
c. IV fluids
d. IV furosemide
e. Insulin
Q. 1. What is the key?
Q. 2. What is the reason of this management?
Ans. 1. The key is B. Suprapubic catheter.
Ans. 2. In pelvic fracture there is chance of urethral rupture and hence displacement of
urethral catheter.

Indications of urethral catheterization:


Indications Relieve urinary retention, Monitor urine output in critically ill
patients, Collect uncontaminated urine for diagnosis.
It is contraindicated in urethral injury (eg pelvic fracture) and acute
prostatitis.
Suprapubic catheterization: Sterile technique required. Absolutely
contraindicated unless there is a large bladder palpable or visible on ultrasound,
because of the risk of bowel perforation. Be wary, particularly if there is a history
of abdominal or pelvic surgery. Suprapubic catheter insertion is high risk and you
should be trained before attempting it, speak to the urologists first!
213. A 49 yo pt presents with right hypochondriac pain. Inv show a big gallstone. What is
the most appropriate management?
a. Lap Cholecystectomy
b. Reassure
c. Low fat diet
d. Ursodeoxycholic acid
e. Emergency laparotomy
Q. 1. What is the key?

Q. 2. Points in favour?
Ans. 1. The key is A. Lap Cholecystectomy.
Ans. 2. i) as symptomatic only reassurence is not appropriate ii) as big ursodyoxycholic
acid is less effective iii) less invasive is preferred so laparoscopic rather than
laparotomy.

Gall Stones:
Pigment stones: (<10%) Small, friable, and irregular. Causes: haemolysis.
Cholesterol stones: Large, often solitary. Causes: age, obesity (Admirands
triangle: inc risk of stone if dec lecithin, dec bile salts, inc cholesterol).
Mixed stones: Faceted (calcium salts, pigment, and cholesterol).
Gallstone prevalence: 8% of those over 40yrs. 90% remain asymptomatic.
Risk factors for stones becoming symptomatic: smoking; parity.
Acute cholecystitis follows stone or sludge impaction in the neck of the
gallbladder, which may cause continuous epigastric or RUQ pain (referred to the
right shoulder, vomiting, fever, local peritonism, or a GB mass. If the stone
moves to the common bile duct (CBD), obstructive jaundice and cholangitis
may occur
Tests: WCC,
Ultrasound Is the best way to demonstrate stones, being 90-95% sensitive.

it shows a thick-walled, shrunken GB (also seen in chronic disease)


Treatment:
NBM, pain relief, IVI, and antibiotics eg cefuroxime 1.5g/8h IV.
Laparoscopic cholecystectomy is the treatment of choice for all patients fit
for GA.
Open surgery is required if there is GB perforation.
Cholecystostomy is also the preferred treatment for acalculous cholecystitis
214. In a man who is neglected and alcohol dependent, which high suicidal risk, which
factor can
increase this risk further?
a. Alcohol dependence
b. SSRI
c. Smoking
d. Agoraphobia
e. Court involvement
Ans. The key is A. Alcohol dependence. This is considered a wrong key by previous
plabbers and suggested correct key is B. SSRI.

Since the patient is already alcohol dependent so that cannot FURTHER increase the
risk. The correct answer here will be SSRI.
215. A 71 yo man presents with coarse tremor. He is on some meds. Which one can be
the reason for the tremor?
a. Lithium
b. Diazepam
c. Fluoxetine
d. Imipramine
e. Haloperidol
Ans. The key is A. Lithium. [lithium is associated with tremor].

Lithium:
Indications for use:
Acute manic or hypomanic episodes
Prophylaxis for bipolar disease. Significantly reduces suicidal risks.
Prophylaxis for recurrent depression & schizoaffective disorder.
Augments the effect of anti depressants.
Cluster headache prophylaxis.
Control of intentional self harm or suicidal behavior.
Contra-indications:
Cardiac disease
Significant renal impairment
Addison's disease and patients with low body sodium levels
Untreated hypothyroidism
Avoid in first trimester of pregnancy. Can be used in 2nd and 3rd trimesters if necessary.
Avoid during breast feeding.
During treatment: Check lithium levels (12 hours post dose) at least every three months
Common side-effects can usually be reduced or eliminated by lowering the lithium dose or
changing the dosage schedule:

Abdominal pain
Nausea
Metallic taste in the mouth (usually wears off)
Fine tremor
Thirst, polyuria, impaired urinary concentration - avoid fluid restriction
Weight gain and oedema

216. A young woman complains of diarrhea, abdominal cramps and mouth ulcers. AXR
shows
distended transverse colon with goblet cell depletion on rectal biopsy. What is the most
probable dx?
a. CD
b. UC
c. Bowel Ca
d. Bowel obstruction
e. IBS
Q. 1. What is the key?
Q. 2. What are points in favour?
Ans. 1. The key is B. UC.
Ans. 2. In UC there is goblet cell depletion and less mucous production in contrast with
CD where there may be goblet cell hyperplasia and mucous secretion is not reduced.
Refer to the explanation of Question No. 209.
217. After eating a cookie at a garden party, a child began to cough and went blue. The
mother also noticed that there were swollen patches on the skin. What is the dx?
a. Allergic reaction

b. Aspiration of food
c. Cyanotic heart disease
d. Trachea-esophageal fistula
e. Achalasia cardia
Ans. The key is A. Allergic reaction.
This here is anaphylaxis as there are noticable skin changes present.
Urticaria Signs: wheals, rapid onset after taking drug association with angio-oedema
/anaphylaxis. It can result from both immunological and non-immunological mechanisms.
Causes: Drugs:morphine & codeine cause direct mast cell degranulation; penicillins &
cefalosporins trigger IgE responses; NSAIDs; ACEi.

Clinical diagnosis. No investigations required.


Management:
Find the cause and avoid/treat it.
Antihistamines:
Non-sedating H1 antihistamines are the mainstay of treatment
In pregnancy chlorphenamine is often the first choice of antihistamine.

Consider anaphylaxis when there is compatible history of rapid-onset severe allergictype reaction with respiratory difficulty and/or hypotension, especially if there are skin
changes present and the treatment of anaphylaxis is IM adrenaline not anti histamine
Treatment:ABCDE, Oxygen, IM Adrenaline. <6yrs0.15ml, 6-12yrs 0.3ml, >12 yrs 0.5ml
1:1000
218. A 70 yo man presents with balance difficulties, vomiting and nausea. Which of the
following is the best inv?
a. MRI cerebellum

b. CT cerebellum
c. Skull XR
d. LP
e. Blood culture
Ans. The key is A. MRI cerebellum. [posterior fossa lesion MRI is preferred].
219. A 2 yo pt presents with colicky pain which radiates from loin to groin. He complains
of similar episodes in the past. Inv has been done and 7mm stone was found in the
ureter. What is the most appropriate management?
a. Percutaneous nephrolithotomy
b. Open surgery
c. Ureterscopy or laser
d. Conservative tx
e. ESWL
Q. 1. What is the key?
Q. 2. What treatments are recommended for different sized stones?
Ans. 1. The key is E. ESWL.
Ans. 2. Stones < 5mm: pass spontaneously, Increase fluid intake.
Stones 5mm-7mm /pain not resolving: medical expulsive therapy---> Nifedipine or Tamsulosin(and/or
prednisolone).
Stones 7mm-2cm: ESWL or Ureteroscopy using dormia basket.
Stones > 2cm/large/multiple/complex: Percutaneous nephrolithotomy.

Renal Stones (nephrolithiasis): Consist of crystal aggregates. Site: 1 Pelviureteric


junction 2 Pelvic brim 3 Vesicoureteric junction.
Types Calcium oxalate (75%) Magnesium ammonium phosphate (struvite/triple
phosphate; 15%) Also: urate (5%), hydroxyapatite (5%), brushite, cystine (1%), mixed
Presentation: Asymptomatic or: 1 Renal colic: excruciating ureteric spasms loin to
groin (or genitals/inner thigh), with nausea/vomiting. Often cannot lie still (differentiates
from peritonitis). Renal obstruction felt in the loin (like intercostal nerve irritation pain;
the latter is not colicky, and is worsened by specific movements/pressure on a trigger
spot). Obstruction of mid-ureter may mimic appendicitis/diverticulitis. Obstruction of
lower ureter
may lead to symptoms of bladder irritability and pain in scrotum, penile tip, or labia
majora. Obstruction in bladder or urethra causes pelvic pain, dysuria, strangury
(desire but inability to void) interrupted flow. UTI can co-exist (risk if voiding
impaired); pyelonephritis (fever, rigors, loin pain, nausea, vomiting), pyonephrosis
(infected hydronephrosis) 3 Haematuria 4 Proteinuria 5 Sterile pyuria 6 Anuria
Tests: BLIs, Mid stream urine, Urine C&S, urinary pH. Urine dipstick: Usually +ve for
.blood (90%).
Imaging: Spiral non-contrast CT is superior to and has largely replaced IVU for
imaging stones (99% visible). 80% of stones are visible on KUB XR
Management: Initially: Analgesia, eg diclofenac + IV fluids if unable to tolerate PO;
antibiotics (eg cefuroxime 1.5g/8h IV, or gentamicin) if infection.
Stones <5mm in lower ureter: ~9095% pass spontaneously. Increase fluid intake.
Stones >5mm/pain not resolving: Medical expulsive therapy: nifedipine 10mg/8h PO or
alpha-blockers (tamsulosin 0.4mg/d 227) promote expulsion and reduce analgesia

requirements:Most pass within 48h. If not, try extracorporeal shockwave lithotripsy


(ESWL) (if <1cm), or ureteroscopy using a basket.
Percutaneous nephrolithotomy (PCNL): keyhole surgery to remove stones, when large,
multiple, or complex. Open surgery is rare.
Indications for urgent intervention (delay kills glomeruli): Presence of infection
and obstructiona percutaneous nephrostomy or ureteric stent may be needed to relieve
obstruction ; urosepsis; intractable pain or vomiting; impending ARF; obstruction in a
solitary kidney; bilateral obstructing stones.

220. A

footballer has been struck in the groin by a kick and a presents with severe pain
and mild
swelling in the scrotum. What is the most appropriate next step?
a. USG
b. Doppler
c. Exploratory surgery
d. IV fluids
e. Antibiotics
Ans. The key is C. Exploratory surgery. [To exclude torsion].
Testicular Torsion: It is the twisting of the testes and it impairs the blood flow to the
testes.
If in any doubt, surgery is required. If suspected refer immediately to urology.
Symptoms: Sudden onset of pain in one testis, which makes walking uncomfortable.
Pain in the abdomen, nausea, and vomiting are common.
Signs: Inflammation of one testisit is very tender, hot, and swollen. The testis may lie
high and transversely. Torsion may occur at any age but is most common at 1130yrs.
With intermittent torsion the pain may have passed on presentation, but if it was severe,
and the lie is horizontal, prophylactic fi xing may be wise.
: The main differential is epididymo-orchitis but with this the patient tends to be older,
there may be symptoms of urinary infection, and more gradual onset of pain.
Tests: Doppler USS may demonstrate lack of blood flow to testis, as may isotope
scanning. Only perform if diagnosis equivocaldo not delay surgical exploration.
Treatment: Ask consent for possible orchidectomy + bilateral fixation (orchidopexy) At
surgery expose and untwist the testis. If its colour looks good, return it to the scrotum
and fix both testes to the scrotum.
221. A 47 yo ex-soldier suffers from low mood and anxiety. He cant forget the images
he faces
before and has always had flashbacks. He is not able to watch the news because there
are
usually some reports about war. What is he suffering from?
a. Depression

b. PTSD
c. Panic attack
d. Agoraphobia
e. GAD
Ans. The key is B. PTSD. [repeated flashbacks and tendency to avoid the thoughts of
stressor is diagnostic of PTSD].
PTSD:
Symptoms: Fearful; horrified; dazed Helpless; numb, detached Emotional
responsiveness
Intrusive thoughts Derealization Depersonalization Dissociative amnesia Reliving
of events Avoidance of stimuli Hypervigilance Lack of Concentration Restlessness
Autonomic arousal: pulse; BP; sweating Headaches; abdo pains
Signs: Suspect this if symptoms become chronic, with these
signs (may be delayed years): difficulty modulating arousal; isolated-avoidant modes of
living; alcohol abuse; numb to emotions and relationships; survivor guilt; depression;
altered world
view in which fate is seen as untamable, capricious or absurd, and life can yield no
meaning
or pleasure.
Treatment: Watchful waiting for mild cases.
For severe cases: CBT or eye movement desensitization and reprocesing is done. Drug
treatment is not recommended but in case it is needed prescribe mirtazepine or
paroxetine.

222. A 36 yo woman has recently spent a lot of money on buying clothes. She goes out
almost every night with her friends. She believes that she knows better than her friends,
so she should
choose the restaurant for eating out. She gave hx of having low mood at 12 yo. Whats
the dx?
a. Mania
b. Depression
c. Bipolar affective disorder
d. Borderline personality disorder
e. Dysthymia
Ans. The key is C. Bipolar affective disorder. [Initial depressive episode (may be before
a long) followed by mania is bipolar affective disorder].
Signs of mania: Mood: Irritability (80%), euphoria (71%), lability (69%).
Cognition: Grandiosity (78%); flight of ideas/racing thoughts (71%); distractibility/ poor
concentration (71%); confusion (25%), many conflicting lines of thought urgently racing
in contrary directions; lack of insight. Behaviour: Rapid speech (98%), hyperactivity
(87%), reduced sleep (81%), hypersexuality (57%), extravagance (55%). Psychotic
symptoms: Delusions (48%), hallucinations (15%). Less severe states are termed
hypomania. If depression alternates
with mania, the term bipolar affective disorder is used (esp. if there is a
history of this). During mood swings, risk of suicide is high. Cyclical mood
swings without the more florid features (as above) are termed cyclothymia

Causes Infections, hyperthyroidism; SLE; thrombotic thromocytopenic purpura; stroke;


water dysregulation/hyponatremia;. Drugs: Amphetamines, cocaine, antidepressants
(esp. venlafaxine), captopril, steroids, procyclidine, L-dopa, baclofen.
Bipolar disorder: (Age at onset: <25.) In a 1st attack Ask about: Infections, drug use,
and past or family history of psychiatric disorders. Do: CT of the head, EEG, and screen
for drugs/toxins.
Treating acute mania for acute moderate/severe mania: olanzapine 10mg PO(SE:
weight gain; inc glucose), or valproate semisodium, eg 250mg/8h PO
Prophylaxis Those who have bipolar affective disorder after successful treatment of the
manic or depressive episode should have a mood stabilizer for longer-term control. If
compliance is good, and U&E, ECG, and T4 normal, give lithium carbonate.

223. A 28 yo female presents with a 3m hx of diarrhea. She complains of abdominal


discomfort and passing stool 20x/day. Exam=febrile. Barium enema shows cobblestone
mucosa. What is the
most likely dx?
a. Ameoba
b. Colon Ca
c. GE
d. CD
e. UC
Ans. The key is D. CD. [Hx of diarrhea, abdominal discomfort, and patient being febrile
indicate gut inflammation and cobblestone appearance on barium enema is suggestive
of CD].
Refer to the explanation of Q.NO 209

224. A child is brought in with high grade fever, runny nose and bark-like cough. He is
also drooling. What is the most appropriate tx for this child?
a. Corticosteroids
b. Paracetamol
c. Adrenaline nebulizer
d. IV antibiotics
e. Intubation under GA
Ans. The key is E. Intubation under GA. [high fever, bark-like cough, drooling in a child
suggest epiglottitis where urgent intubation is needed to avoid respiratory blockage from
epiglottitis].
It is a very tricky question. Symptoms of both croup and acute epiglottitis are mixed here!
A confirm diagnosis cannot be made.
In favour of croup: runny nose and barking cough.
In favour of acute epiglottitis: high grade fever, drooling.
Perhaps in the original exam question more information was provided which couldnot be
recalled here in the question which would have helped in the diagnosis.
Laryngotracheobronchitis/croup: is the leading cause of stridor with a barking cough.
More common than epiglottitis.

Cause: 95% are viral, eg parainfluenza (ribavirin can help, eg in immunodeficiency).


Bacteria (klebsiella; diphtheria) & fungi are rare.
Presentation: If there is cough and no drooling, croup is almost always the
diagnosis.
Initially presents with symptoms of URTI. Hoarsness and barking cough develop in a couple
of days worse at night and mild to moderate fever. Stridor is also present. Respiratory
distress with marked tachypnoea and intercostal recession may be noted. Drowsiness,
lethargy, and cyanosis despite increasing respiratory distress should be considered as red
flags for impending respiratory failure.
Management: Usually self-limiting; treat at home ( antibiotics). Admit (eg to ITU) if
severe. In children, CXR may show steeple sign of a tapering trachea. Give antibiotics,
humidified O2, + nebulized adrenaline (5mL 1:1000, may buy time in severe disease
needing ventilating), and dexamethasone 150g/kg PO stat or budesonide 2mg
nebulized.
Acute epiglottitis is rarer than croup but mortality is high: 1% if respiratory distress. Its
an emergency as respiratory arrest can occur.
Presentation: Often, history is short, septicaemia is rapid, and cough is absent. Also:
sore throat (100%), fever (88%), dyspnoea (78%), voice change (75%), dysphagia
(76%), tender anterior neck cellulitis (27%), hoarseness (21%), pharyngitis (20%),
anterior neck nodes (9%), drooling (head for ward tongue out), prefers to sit, refusal to
swallow,
Cause: Haemophilus (vaccination has reduced prevalence); Strep pyogenes.
Investigation: Fibre-optic laryngoscopy remains the 'gold standard' for diagnosing
epiglottitis
Management: Take to ITU; dont examine throat (causes resp. arrest). Give O2 by
mast, Give nebulized adrenaline, IV dexamethasone, antibiotics, antipyretics until the
anesthetist arrives. Definitive management is intubation

225. A 78yo lady on warfarin for atrial fibrillation lives in a care home. She presents with
hx of
progressive confusion for three days. She was also noticed to have bruises on her arms.
INR = 7. What is the most probable dx?
a. Alzheimers
b. Delirium
c. Subdural hemorrhage
d. Vascular dementia
e. Picks dementia
Ans. The key is C. Subdural haemorrhage. [Age 78 yrs, living in a care home where
unnoticed trivial injury is possible (like fall), warfarin and high INR is potential risk factor
of subdural haemorrhage suggested by bruises on arms also].
Subdural Haemorrhage: Consider this very treatable condition in all whose conscious
level fluctuates, and also in those having an evolving stroke, especially if on
anticoagulants.
Causes: Bleeding is from bridging veins between cortex and venous sinuses (vulnerable
to deceleration injury), resulting in accumulating haematoma between dura and

arachnoid. Most subdurals are from trauma but the trauma is often forgotten as it was so
minor or so long ago (up to 9 months). Elderly are particularly susceptible. Other risk
factors: falls (epileptics, alcoholics); anticoagulation
Symptoms Fluctuating level of consciousness (seen in 35%) insidious physical or
intellectual slowing, sleepiness, headache, personality change, and unsteadiness.
Signs: Raised ICP; seizures. Localizing neurological symptoms (eg unequal pupils,
hemiparesis) occur late and often long after the injury
Imaging: CT/MRI shows clot midline shift (but beware bilateral isodense clots). Look
for crescent-shaped collection of blood over 1 hemisphere. The sickleshape different
iates subdural blood from extradural haemorrhage.
Treatment Irrigation/evacuation, eg via burr twist drill and burr hole craniostomy, can be
considered 1st-line; craniotomy is 2nd-line if the clot has organized. Address causes of
the trauma (eg falls due cataract or arrhythmia; abuse).

226. A 28 yo drug user presents to the ED with collapse and anuria. His serum K+ =
7.5mmol/L. CXR = early pulmonary edema. What is the next appropriate management?
a. Urgent hemodialysis
b. IV calcium gluconate
c. IV insulin + dextrose
d. Furosemide
e. IV NS 0.9%
Ans. The key is B. IV calcium gluconate. [To correct hyperkalemia to prevent cardiac
arrhythmia].
Hyperkaelemia: A plasma potassium >6.5mmol/L is an emergency and needs urgent
treatment
The worry is of myocardial hyperexcitability leading to ventricular fibrillation and cardiac
arrest.
Concerning signs and symptoms fast irregular pulse, chest pain, weakness,
palpitations, and light-headedness.
ECG: tall tented T waves, small P waves, a wide QRS complex (eventually becoming
sinusoidal), and ventricular fibrillation
Artefactual results: If the patient is well, and has none of the above findings, repeat
the test urgently as it may be artefactual
Causes: Oliguric renal failure K+-sparing diuretics Rhabdomyolysis Metabolic
acidosis(DM)
Excess K+ therapy Addisons disease Massive blood transfusion Burns Drugs, eg
ACE-i, suxamethonium Artefactual result
Management:
Stabilisation of the cardiac membrane
intravenous calcium gluconate

Short-term shift in potassium from extracellular to intracellular fluid compartment


combined insulin/dextrose infusion
nebulised salbutamol
Removal of potassium from the body
calcium resonium (orally or enema)
loop diuretics
dialysis

227. A 32 yo woman suffers an episode of severe occipital headache with vomiting and
loss of
consciousness. She is brought to the hospital where she is found to be conscious and
completely alert. Exam: normal pulse & BP. No abnormal neurological signs. What is the
next step in management?
a. Admission for observation
b. CT brain
c. MRI head
d. Reassurance and discharge
e. XR skull
Ans. The key is B. CT brain. [basilar migraine can cause severe headache and LOC. But
there occurs no neurological deficit and recovering from unconsciousness becomes
completely alert. But to diagnose basilar migraine there should at least history of two
migraine attacks with aura. As here diagnostic criteria of basilar migraine is not fulfilled
we can not discharge the patient without neuroimaging like CT or MRI].
228. A 25 yo woman was brought to the ED by her boyfriend. She has many superficial
lacerations on her forearm. She is so distressed and constantly says her boyfriend is
going to end the relationship. She denies trying to end her life. What is the most likely
dx?
a. Acute psychosis
b. Severe depression
c. Psychotic depression
d. Borderline personality disorder
e. Schizophrenia
Ans. The key is D. Borderline personality disorder. [ Borderline personality disorder: Act
impulsively and develop intense but short-lived emotional attachment to others. They are usually
attention seekers but not suicidal].
Borderline Personality Disorder: There is unstable affect regulation, poor impulse control, and
poor interpersonal relationships/self-image, eg with repeated self-injury, suicidality, and a difficult
life-course trajectory
Associations: ADHD;2 learning difficulties. Genetics and adverse childhood events (eg abuse)
are predispositions. Intervene (and refer) early with specific management plan, addressing work,
Dialectical behaviour therapy, inpatient hospital programmes, and drugs can reduce depression,
anxiety, and impulsive aggression. Eventually, supportive interpersonal dyads are achievable

229. A young woman was brought to the hospital. On exam she has low temperature and

tremor.
She says when she closes her eyes, she can see colors. What drug has been used?
a. Amphetamines
b. LSD
c. Cocaine
d. Heroine
e. Ecstasy
Ans. The key is B. LSD.
LSD stands for its chemical name, lysergic acid diethylamide. It is also commonly called
acid. Other terms include blotter, tripper, flash, stars, rainbows.
What are the harmful effects? It can make you frightened and confused. Sometimes you
can get "flashbacks" when you relive the same experience again. People can be more likely
to self-harm when they have a bad trip.

230. A lady comes in severe liver disease and hematemesis. Her INR is >10. What
should she be given?
a. FFP
b. Steroids
c. Whole blood
d. IV fluids
e. Vit K
Ans. The key is A. FFP.

IN any major bleeding irrespective of INR prothrombin complex concentrate (or FFP) is
given and vit.K is also given! But vit.K takes time to act FFP is a better options.
Situation

Major bleeding

Management

Stop warfarin
Give intravenous vitamin K 5mg
Prothrombin complex concentrate - if not available then FFP*

INR > 8.0

Stop warfarin

Minor bleeding

Give intravenous vitamin K 1-3mg


Repeat dose of vitamin K if INR still too high after 24 hours
Restart warfarin when INR < 5.0

INR > 8.0

Stop warfarin

No bleeding

Give vitamin K 1-5mg by mouth, using the intravenous preparation orally


Repeat dose of vitamin K if INR still too high after 24 hours
Restart when INR < 5.0

INR 5.0-8.0

Stop warfarin

Minor bleeding

Give intravenous vitamin K 1-3mg


Restart when INR < 5.0

INR 5.0-8.0

Withhold 1 or 2 doses of warfarin

No bleeding

Reduce subsequent maintenance dose

231. After eating a cookie at a garden party, a child began to cough and went blue. The
mother also noticed that there were swollen patches on the skin. What is the initial
management?
a. OTC antihistamine
b. Oxygen
c. Bronchodilators
d. Epinephrine IM
e. Nebulized epinephrine
Ans. The key is D. Epinephrine IM [anaphylaxis with partially blocked airway].
Already explained in previous questions.

232. A 63 yo female is noted to have left pupil unresponsive to light and is dilated. What
is the most probably dx?
a. Pontine hemorrhage
b. Subdural hemorrhage
c. Cerebellar hemorrhage
d. Extradural hemorrhage
e. Subarachnoid hemorrhage
Ans. The key is D. Extradural hemorrhage. It is a wrong key. In a 63 year old extradural
hemorrhage is extremely unlikely. As no clinical picture is described in question except 3rd nerve
palsy E. SAH is more logical answer!
Sub Arachnoid Haemorrhage
Spontaneous bleeding into the subarachnoid space is often catastrophic
Causes: Rupture of saccular aneurysms (80%); arteriovenous malform ations (AVM; 15%). No
cause

is found in <15%.
Risk factors: Smoking, alcohol misuse, inc BP, bleeding disorders, mycotic aneurysm (SBE),
perhaps post-menopausal oestrogen >45yrs old).
Berry aneurysms Common sites: junction of posterior communicating with the internal carotid or
of the anterior communicating with the anterior cerebral artery or bifurcation of the middle
cerebral artery. Some are hereditary. Associations: Polycystic kidneys, coarctation of the aorta,
EhlersDanlos syndrome
Symptoms Sudden (usually, but not always, within seconds) devastating typically occipital
headacheI thought Id been kicked in the head. Vomiting, collapse, seizures and coma often
follow. Coma/drowsiness may last for days.
Signs Neck stiffness, Kernigs sign (takes 6h to develop), retinal, subhyaloid and vitreous bleeds
(=Tersons syndrome; it carries a worse prognosis) Focal neurology at presentation may suggest
site of aneurysm (eg pupil changes indicating a IIIrd nerve palsy with a posterior communicating
artery aneurysm) or intracerebral haematoma. Later deficits suggest complications
Tests CT detects >90% of SAH within the 1st 48h. LP if CT ve and no contraindication >12h
after headache onset. CSF in SAH is uniformly bloody early on, and becomes xanthochromic
(yellow) after several hours due to breakdown products of Hb (bilirubin). Finding xanthochromia
confirms SAH
Management Refer all proven SAH to neurosurgery immediately.
Re-examine CNS often; chart BP, pupils and GCS. Repeat CT if deteriorating.
Maintain cerebral perfusion by keeping well hydrated, and aim for SBP >160mmHg. Treat BP
only if very severe.
Nimodipine is a Ca2+ antagonist that reduces vasospasm and consequent morbidity from
cerebral ischaemia.
Endovascular coiling is preferred to surgical clipping where possible. Do catheter
or CT angiography to identify single vs multiple aneurysms before intervening.
Intracranial stents and balloon remodelling enable treating wide-necked aneurysms.
Complications Rebleeding is the commonest cause of death, and occurs in 20%,
often in the 1st few days. Cerebral ischaemia due to vasospasm may cause a permanent
CNS deficit, and is the commonest cause of morbidity. Hydrocephalus, due to blockage
of arachnoid granulations, requires a ventricular or lumbar drain. Hyponatraemia
is common but should not be managed with fluid restriction. Seek expert help

233. A 28yo business exec presents at the GP asking for some help because she has
been arguing with her boyfriend frequently. She is worried about her weight, and she
thinks she may be fat. She has been on a diet and lost 7 kgs in the last 2 months on
purpose. She is eating less. She used to do a lot of exercise. Now she says shes feeling
down, has some insomnia and feels tired and without energy. She has not showed up at
work. She is worried because recently she got a loan to buy a luxury car. She cant be
fired. She complains about her low mood. She thinks this is weird because she used to
be extremely productive. She used to work showing an excellent performance at the
office. She even received compliments from her boss. How, she says her boyfriend is
angry because her apartment is a chaos. Usually she spends a lot of time cleaning it,
even upto 3 AM. She liked it to be perfect, but not its a mess. On exam: BMI=23, no
other signs. What is the most probably dx?
a. Anorexia nervosa
b. Bipolar disease

c. Binge eating disorder


d. Hyperthyroidism
e. Schizophrenia
Ans. The key is B. Bipolar disease.
Bipolar Disease:
It is characterised by episodes of mania (or hypomania) and depression. Either one can
occur first and one may be more dominant than the other but all cases of mania
eventually develop depression.
During the manic phase the following may be present:

Grandiose ideas.
Pressure of speech.
Excessive amounts of energy.
Racing thoughts and flight of ideas.
Overactivity.
Needing little sleep or an altered sleep pattern.
Easily distracted - starting many activities and leaving them unfinished.
Bright clothes or unkempt.
Increased appetite.
Sexual disinhibition.
Recklessness with money.

Depressive phase:
In the depressive phase, patients experience low mood with reduced energy. Patients
have no joy in daily activities and have negative thoughts. They lack facial expressions
and have poor eye contact and may be tearful and unkempt. Low mood is worse in the
mornings and is disproportionate to the circumstances. There may be feelings of
despair, low self-esteem and guilt for which there may be no clear reason. There may be
weight loss, reduced appetite, altered sleep pattern with early morning wakening and
loss of libido.
Diagnosis:
ICD-10 requires at least two episodes in which a person's mood and activity levels are
significantly disturbed (one of which must be mania or hypomania)
Three of the following symptoms confirm mania:

Grandiosity/inflated self-esteem.
Decreased need for sleep.
Pressured speech.
Flight of ideas (rapidly racing thoughts and frequent changing of their train of
thought).
Distractibility.
Psychomotor agitation.
Excessive involvement in pleasurable activities without thought for
consequences (eg, spending spree resulting in excessive debts).

There may also be psychotic symptoms - eg, delusions and hallucinations. The manic
episode is mixed if there are associated depressive symptoms
Management: The following are non-pharmacological methods:

Education regarding diagnosis, treatment and side-effects.


Good communication.
Self-help groups.
Support groups.
Self-monitoring of symptoms, side-effects and triggers.
Coping strategies.
Psychological therapy.
Encouragement of engagement in calming activities.
Telephone support.

Psychological therapies have been shown to be beneficial - eg, cognitive behavioural


therapy which helps to identify triggers and how to avoid them.
Pharmacological treatment: For manic attack treatment is same as for mania
If the pt is on antipsychotics and still develops an attack of mania the dose should be
increased. Drugs commonly used are haloperidol, olanzapine, quetiapine and
risperidone. If one antipsychotic is ineffective it is worth changing to a different one.
If ineffective consider adding lithium, if contra indicated add valproate.
Treatment of an acute depressive episode
A risk assessment of suicidal ideation should be made. If it is considered that
compulsory hospital admission would be in the patient's interest, the Mental Health Act
or Common Law may need to be invoked

Patients with moderate-to-severe depression should be offered fluoxetine


combined with olanzapine or quetiapine on its own.
If there is no response, lamotrigine on its own can be tried.
If patients are already taking lithium, the level should be checked and the
dose increased as necessary. If this fails, fluoxetine combined with
olanzapine or quetiapine can be added.,

During an acute mixed episode antidepressants should be avoided and the aim should
be to try to stabilise patients on anti-manic medication.

234. A woman brought her husband saying she wants the thing on his forehead
removed. The
husband is refusing tx saying it improves his thinking. What is the next most appropriate
next

step?
a. Assess his mental capacity to refuse tx
b. Remove lesion
c. Refer to ED
d. Mini-mental state exam
e. Refuse surgery and send pt back
Ans. The key is A. Assess his mental capacity to refuse treatment.
235. A 37 yo man who has many convictions and has been imprisoned many times has
a hx of many unsuccessful relationships. He has 2 boys but doesnt contact them. What
is the most probable
dx?
a. Borderline personality disorder
b. Schizophrenia
c. Avoidant personality disorder
d. Histrionic personality disorder
e. Antisocial behavior disorder
Ans. The key is E. Antisocial behavior disorder.
Antisocial behavior
Features include:

Unstable interpersonal relationships.


Disregard for the consequences of their behaviour.
A failure to learn from experience.
Egocentricity.
A disregard for the feelings of others.
A wide range of interpersonal and social disturbance.
Comorbid depression and anxiety.
Comorbid alcohol and drug misuse

Diagnostic criteria:
People with antisocial personality disorder have a pervasive pattern of disregard for and
violation of the rights of others and the rules of society. A history of conduct disorder before
the age of 15 is a requirement for a diagnosis of antisocial personality disorder (in the DSMIV criteria) and includes the following features:

Repeated breaches of the law. They may well have recurrent criminal
convictions.
Frequent lying and deception, even when there is no obvious gain.
Physical aggression.
Reckless disregard for safety of self or others.
Utter irresponsibility in work and family environments.
Lack of remorse.

Tests: Toxicology screen and HIV screening should be done

Management: All patients diagnosied in primary setting should be referred to specialist


centers.
No drug has UK marketing authorisation specifically for the treatment of antisocial
personality disorder. However, antidepressants and antipsychotics are often used to treat
some of the associated problems and symptoms.
Psychotherapy is at the core of care for personality disorders generally. Cognitive
behavioural therapy and group therapy are perhaps the most widely used and available
forms of psychotherapy

Complications:
Suicide
Substance abuse
Accidental injury
Depression
Homicide

236. A 60 yo man has a pathological rib fx. He also complains of recurrent infection.
BMA is done. Labs: Ca2+ = 3.9mmol/L and ALP = 127u/L. what type of cell would be
found in abdundance in
the marrow smear?
a. Plasma cell
b. Myeloid cell
c. Bence-jones protein
d. Megakaryocytes
e. Reticulocytes
Q. 1. What is the key.
Q. 2. What is the diagnosis?
Q. What are the points in favour of diagnosis?
Ans. 1. The key is A. Plasma cell.
Ans. 2. The diagnosis of multiple myeloma.
Ans. 3. Points in favour: i) age 60 yrs ii) pathological rib fracture (from metastases) iii)
recurrent infection (due to B cell dysfunction (manifested as hypogammaglobulinemia),
numerical and functional abnormalities of T cells, and dysfunction of natural killer cells),
iv) raised calcium level.
Multiple Myeloma: PCDs are due to an abnormal proliferation of a single clone of
plasma or lymphoplasmacytic cells leading to secretion of immunoglobulin occuring as
monoclonal bands or paraprotein in urine or serum electrophoresis.
IgG in ~ ; IgA in ~ ; A very few are IgM or IgD. Other Ig levels are low
(immunoparesis, causing susceptibility to infection). In ~ , urine contains Bence Jones
proteins.
Symptoms:
Osteolytic bone lesions causing backache, pathological fractures (eg long bones
or ribs) and vertebral collapse. Hypercalcaemia may be symptomatic.

Anaemia, neutropenia, or thrombocytopenia may result from marrow infiltration


by plasma cells, leading to symptoms of anaemia, infection and bleeding.
Recurrent bacterial infections due to immunoparesis, and also because of neutropenia
due to the disease and from chemotherapy.
Renal impairment due to light chain deposition.
Diagnostic criteria:
1 Monoclonal protein band in serum or urine electrophoresis.
2 Plasma cells on marrow biopsy
3 Evidence of end-organ damagefrom myeloma:
Hypercalcaemia Renal insufficiency Anaemia
4 Bone lesions: a skeletal survey afterdiagnosis detects bone disease: Xraysof chest; all
of spine; skull; pelvis.
Tests: Rouleaux formation, persistently inc ESR or Polycythemia Vera, inc urea and
creatinine, inc Ca2+ (in ~40%), alk phos usually normal unless healing fracture.
Screening test: Serum and urine electrophoresis. Beta2-microglobulin (as a prognostic
test).
Imaging: X-rays: lytic punched-out lesions, eg pepper-pot skull, vertebral collapse,
fractures or osteoporosis.
CT or MRI may be useful to detect lesions not seen on XR.
Treatment: Symptomatic. For bones give bisphosphonates. For anemia transfusion or
erythropoietin. Keep hydrated to prevent renal failure, dialysis may be needed. Treat
infections.
Chemotherapy: If unsuitable for intensive , melphalan + prednisolone is used.
Complications: Hypercalcemia, Renal injury, cord compression, hyperviscosity.
237. A child presents with blue marks on the sclera, short stature and heart murmur.
What is the dx?
a. Osteogenesis imperfect
b. Hypopituitarism
c. VSD
d. Achondrogenesis
e. Dwarfism
Ans. The key is A. Osteogenesis imperfecta.

Osteogenesis imperfecta:
is an inherited disorder of type I collagen that
results in fragile, low density bones
Types:
I The mildest and most common form. It is autosomal dominant. Associated with blue
sclerae (due to increased corneal translucency) and 50% have hearing loss. Fractures
typically occur before puberty. There is a normal life expectancy.
II Lethal perinatal form with many fractures and dwarfism; it is recessive.
III Severe formoccurs in about 20%. Recessive. Fractures at birth + progressive spinal
and limb deformity, with resultant short stature; blue or white sclera;Life expectancy is
decreased.
IV Moderate form, autosomal dominant. Fragile bones, white sclerae after infancy.

Radiographs: Many fractures, osteoporotic bones with thin cortex, and bowing
deformity of long bones.
Histology: Immature unorganized bone with abnormal cortex.
Treatment: Prevent injury. Physio, rehab and occupational therapy are key.
Bisphosphonates can be used.
238. A 5month child cant speak but makes sounds. She can hold things with palm, not
fingers. Cant sit independently but can hold her head and sit when propped up against
pillows. Hows the
childs development?
a. Normal
b. Delayed speech
c. Delayed sitting
d. Delayed motor development
Ans. The key is A. normal

239. A 27 yo woman has hit her neck in an RTA without complains of tingling or motor
loss. What is the next most appropriate inv?
a. MRI
b. XR
c. CT cervical
d. Diagonal XR
Ans. The key is B. X-ray. [As there is no neurological deficit we can exclude any fracture
by x-ray first].[Diagonal x ray means ,oblique view of cervical spine. By this view we can assess facet
joint arthopathy. This doesn't related to RTA].

240. A young female who has many superficial lacerations was brought into the ED by
her boyfriend for superficially lashing her upper arm. She is adamant and screaming that
she is not suicidal but scared her boyfriend wants to leave her. What is the dx?
a. Acute psychosis
b. Severe depression
c. Obsessive

d. Bipolar
e. Borderline personality
f. Schizophrenia
Ans. The key is acute psychosis.
241. A 22yo woman was brought by her boyfriend with multiple superficial lacerations.
There are
scars of old cuts on her forearms. She is distressed because he wants to end the
relationship.
She denies suicide. What is the most likely dx?
a. Acute psychosis
b. Borderline personality disorder
c. Severe depression
d. Schizoid personality
e. Psychotic depression
Ans. The key is B. Borderline personality disorder.
242. A 31yo single man lives with his mother. He usually drives to work. He always
thinks when the traffic lights change, his mother is calling him, so he drives back home.
What is the dx?
a. OCD
b. GAD
c. Schizophrenia
d. Bipolar
e. Cyclothymia
Ans. The key is C. Schizophrenia. [ delusion of reference - he thinks that the changing traffic lights
are giving message to him].

Schizophrenia is the most common form of psychosis. It is a lifelong, condition, which can
take on either a chronic form or a form with relapsing and remitting episodes of acute illness.
Multiple factors are involved in schizophrenia - eg, genetic, environmental and social.
Risk factors include= family history, intrauterine and perinatal complications,Intrauterine
infection, particularly viral, Abnormal early cognitive/neuromuscular development etc.
Features=
The hallmark symptoms of a psychotic illness are: Delusions,Hallucinations (auditory
hallucination like two or more voices discussing the patient in the third person,thought
echo,voices commenting on the patient's behaviour), Thought disorder (thought
insertion,thought withdrawal, thought broadcasting), Lack of insight.
These first Rank or positive symptoms of schizophrenia are absent in other psychotic
disorders.

Schizophrenia
NICE published guidelines on the management of schizophrenia in 2009.
Key points:
oral atypical antipsychotics are first-line
Examples of atypical antipsychotics
clozapine, olanzapine,risperidone,quetiapine,amisulpride

adverse effects:
weight gain
clozapine is associated with agranulocytosis

cognitive behavioural therapy should be offered to all patients


close attention should be paid to cardiovascular risk-factor modification due to the
high rates of cardiovascular disease in schizophrenic patients (linked to antipsychotic
medication and high smoking rates).

243. A 56yo woman is known case of pernicious anemia. She refuses to take
hydroxycobalamin IM as she is needle shy. She asks for oral medication. Why will oral
meds be not effective?
a. Intrinsic factor def
b. Malabsorption
c. Irritated gastric mucosa
d. Lack of gastric acidity
Ans. The key is A. Intrinsic factor def.
Pernicious Anemia: This is caused by an autoimmune atrophic gastritis, leading to
achlorhydria and lack of gastric intrinsic factor secretion.
Associations Other autoimmune diseases: thyroid disease (~25%), vitiligo, Addisons
disease, hypoparathyroidism. Carcinoma of stomach is ~3-fold more common in
pernicious anaemia
Investigations: Hb (30110g/L) Raised MCV Decreased WCC and platelets if
severe Decreased Serum B12 Reticulocytes reduced or normal as production
impaired Hypersegmented polymorphs Megaloblasts in the marrow
Specific tests: 1.Parietal cell antibodies: found in 90% with PA, 2 Intrinsic factor (IF)
antibodies:
specific for pernicious anaemia, but lower sensitivity.
Treatment Treat the cause if possible. If a low B12 is due to malabsorption, injections
are required. Replenish stores with hydroxocobalamin (B12) 1mg IM alternate days, for
2wks Maintenance: 1mg IM every 3 months for life.
If the cause is dietary, then oral B12 can be given after the initial acute course.
244. An 11m baby had an apnea event. The parents are worried that if something like
this happens in the future, how they are to deal. Advise them about infant CPR.
a. Index and middle finger compression
b. Compression with palm of one hand
c. Compression with palm of two hands
d. Compression with rescue breaths 30:2
e. Compression with rescue breaths 15:2
Ans. The key is A. Index and middle finger compression.
How to give chest compressions: Compress lower half of sternum to of the
chests depth; use the heel of one hand (or, in babies, with both your thumbs,
with your hands encircling the thorax) If >8yrs, the adult 2-handed method is
OK. For an infant, 2 fingers are sufficient, in the middle of a line joining the nipples.
How to give the rescue breaths to a child: Ensure head tilt and chin lift.

Pinch the soft part of his nose. Open his mouth a little, but maintain chin up.
Take a breath, and place your lips around his mouth (good seal). Blow steadily
into his mouth over 11.5sec

245. A teacher brings in a child who says she fell down after hitting a table. On probing
further, you
decide that it was most probably an absence seizure. What led you to this dx?
a. The child had not eaten since morning
b. The child suddenly went blank and there was up-rolling of eyes
c. The child started moving his fingers uncontrollably before he fell
d. The childs body became rigid and then started to jerk
Ans. The key is B. The child suddenly went blank and there was up-rolling of eyes.
Types of seizures:

Primary generalized seizures Simultaneous onset of electrical discharge


throughout cortex, with no localizing features referable to only one hemisphere.
Absence seizures: Brief (10s) pauses,They do not fall but may pause in what they
are doing. Their face often looks pale with a blank expression. They may look dazed, the
eyes stare and the eyelids may flutter a little. Sometimes their head may fall down a little, or
their arms may shake once or twice. Each seizure usually starts and finishes abruptly. The
person is not aware of the absence and resumes what they were doing..

Tonicclonic seizures: Loss of consciousness. Limbs stiffen (tonic), then jerk


(clonic). May have one without the other. Post-ictal confusion and drowsiness.
Myoclonic seizures: Sudden jerk of a limb, face or trunk. The patient may be
thrown suddenly to the ground, or have a violently disobedient limb: one patient
described it as my flying-saucer epilepsy, as crockery which happened to be in
the hand would take off .
Atonic (akinetic) seizures: Sudden loss of muscle tone causing a fall, no LOC.
Infantile spasms/West syndrome: Peak age: 5 months. Clusters of head nodding
(Salaam attack) and arm jerks, every 330sec. IQ decrease in ~70%. EEG is
characteristic (hypsarrythmia)

246. A man has discharge from his left ear after a fight. Where is the discharge coming
from?
a. CSF
b. Inner ear
c. Outer ear
d. Brain
Ans. The key is A. CSF. [probable fracture base of skull]
Ear Discharge & Their Source:
External ear: Inflammation, ie otitis externa produces a scanty watery discharge, as
there are no mucinous glands Blood can result from trauma to the canal. Liquid wax
can sometimes leak out.
Middle ear: Mucous discharges are almost always due to middle ear
disease.Serosanguinous discharge suggests a granular mucosa of chronic otitis media.
An offensive discharge suggests cholesteatoma.
CSF otorrhoea: CSF leaks may follow trauma: suspect if you see a halo sign on filter
paper, or its glucose is increased , or Beta2 (tau) transferrin is present.
.
247. A 40 yo manic depressive is noted to have high serum levels of lithium and
profound
hypokalemia. His GP had started him on anti-HTNs. Choose the single most likely
cause?
a. Verapamil
b. Amiodarone
c. Ranitidine
d. Lithium
e. Thiazide
Ans. The key is E. Thiazide. [Thiazide was prescribed for Hpt and when lithium was
prescribed its level increased due to thiazide and thiazide also caused hypokalemia
resulting the given picture].
Thiazide Diuretics Side Effects:
A possible increase in blood sugar level.
A possible increase in the level of uric acid. So contra indicated in gout.
It can cause a low blood level of potassium, sodium, and magnesium, and a high
level of calcium. These effects may cause weakness, confusion, and rarely,
abnormal heart rhythms to develop.
Other problems, such as:
o Upset stomach.
o Dizziness on standing (due to too low blood pressure).
o Erection problems (impotence) - often reversible on stopping
treatment.
o Skin sensitivity to sunlight.

248. A 74yo man presents with weakness in his arm and leg from which he recovered
within a few days and short term memory loss. He has an extensor plantar response. He
has similar episodes 2 years ago and became unable to identify objects and to make
proper judgment. What is the most appropriate dx?

a. Alcoholic dementia
b. Picks dementia
c. Huntingtons disease
d. Alzheimers disease
e. Vascular dementia
Ans. The key is E. Vascular dementia. [hemiparesis, memory impairment, extensor
plantar reflex, inability to identify objects, poor judgment are features of vascular
dementia].
Vascular dementia: ~25% of all dementias. It represents the cumulative effects of many
small strokes, thus sudden onset and stepwise deterioration is characteristic
Look for evidence of vascular pathology (BP, past strokes, focal CNS signs).
Diagnosis:

Presence of dementia - cognitive decline from higher level of functioning. This


can be demonstrated as memory loss plus impairment in two or more different
cognitive domains . This should be established by clinical examination and
neuropsychological testing. Deficits should be severe enough to interfere with
activities of daily living - not secondary effects of the cerebrovascular event
alone.
Cerebrovascular disease, defined by the presence of signs on neurological
examination and/or by brain imaging.
A relationship between the above two disorders inferred by:
o Onset of dementia within three months following a recognised
stroke.
o An abrupt deterioration in cognitive functions.
o Fluctuating, stepwise progression of cognitive deficits.

Management: Like other dementias the treatment is symptomatic, addressing the


individual's main problems and supporting the carers. Detecting and addressing
cardiovascular risk factors is also very important to try to slow progression

249. A nurse comes to you saying that she has recently developed the habit of washing
her hands after every 15-20 mins. She is unable to conc on her work and takes longer
than before to finish tasks as she must constantly was her hands. What is the most
appropriate management?
a. CBT
b. SSRI
c. ECT
d. Antipsychotics
e. Desensitization
Q. 1. What is the key?
Q. 2. What is the diagnosis?
Ans. 1. The key is A. CBT.
Ans. 2. The diagnosis is OCD.
Compulsions are senseless, repeated rituals. Obsessions are stereotyped, purposeless
words, ideas, or phrases that come into the mind.

Repetitive behavior and an urge to do it.


Treatment: CBT is first line. Clomipramine (start with 25mg/day PO) or SSRIs (eg
fluoxetine)
250. A 61yo man underwent a surgery in which ileal resection had been done. He
complains of
fatigue, headache, and heart racing. Labs: MCV=108fL, Hgb=8.9g/dL. What is the most
likely dx?
a. Vit B12 def
b. Iron def
c. Folate def
d. Hemolytic anemia
e. Anemia of chronic disease
Q. 1. What is the key?
Q. 2. What are the points in favour?
Ans. 1. The key is Vit. B12 deficiency.
Ans. 2. Vit B12 is absorbed mostly in ileum. Megaloblastic anaemia.
Sites of minerals and nutrients absorption:

250. A 61yo man underwent a surgery in which ileal resection had been done. He
complains of
fatigue, headache, and heart racing. Labs: MCV=108fL, Hgb=8.9g/dL. What is the most
likely dx?

a. Vit B12 def


b. Iron def
c. Folate def
d. Hemolytic anemia
e. Anemia of chronic disease
Key : A (Vit B12 def)
Points in favour : typical symptoms of fatigue, headache and palpitations along
with favourable age group. Most important clues lie in Ileal resection (vit b12
absorption occurs in terminal ileum) and secondly increased MCV(showing
macrocytosis).
From symptoms and low HB here we know straight away that the diagnosis here is
some kind of anemia. Next most important thing to look for in anemia questions is the
MCV.Normal MCV is between 76-96 fl.
Microcytic (low MCV) = IDA , Thalassemia , Sideroblastic anemias , Anemia of chronic
disease.
Normocytic (normal MCV) = Acute blood loss , Anemia of chronic disease, bone
marrow failure , Renal failure , hypothyroidism (or increased mcv), haemolysis (or
increased mcv), pregnancy.
Macrocytic (high MCV) = Vit B12 deficiency , Folate deficiency , Alcohol excess or
Liver disease, Myelodysplastic syndromes, Marrow infiltration, hypothyroidism, antifolate
drugs.
Next we look for the cause of the anemia in the question which quite obviously in this
question is the ileal resection. Vitamin B12 is absorbed in the terminal ileum once it is
attached to the intrinsic factor secreted by the parietal cells in the stomach. Intrinsic
factor is deficient in an an autoimmune disease called Pernicious anemia. Other causes
of Vit B12 deficiency include gastrectomy, gastric resection , atrophic gastritis , long term
H.pylori infection, inadequate intake of vit B 12 (vegan diet), malabsorption, Crohns
disease, chronic tropical sprue, DRUGS ( eg, colchicine, neomycin, metformin,
anticonvulsants, long term use of PPIs and H2 receptor blockers).
Ruling out = Except Folate def. other options are easy to rule out. IDA is microcytic
and there is no history of any chronic disease for answer to be D. There is no
evidence of haemolysis in the question like jaundice, retic count and raises LDH etc.
Folate deficiency anemia is mostly due to decreased intake or is pregnancy induced
and has nothing to do with ileum resection.
Topic = Vitamin B12 deficiency and Pernicious anemia
EPI = occurs in all races, peake age is 60, more common in those with blue eyes, early
greying, a positive family history and blood group A. The condition has a female:male ratio of
1.6:1.0.
Presentation = fatigue and lethargy, dyspnoea, faintness, palpitations and headache.
Vitamin B12 deficiency may present with unexplained neurological symptoms - eg,
paraesthesia, numbness, cognitive changes or visual disturbance.
Findings on examination may include pallor, heart failure (if anaemia is severe), lemon tinge
to the skin, glossitis and oral ulceration. Neuropsychiatric features may include irritability,
depression, psychosis and dementia. Neurological features may include subacute combined
degeneration of the spinal cord and peripheral neuropathy.
Investigations = FBC, Blood film , Biochemistry (including serum vit B12 levels
and serum Folate levels),Schilling test and Bone marrow aspiration.

Treatment = For patients with no neurological involvement, treatment is with six injections
of hydroxocobalamin, 1 mg in 1 mL at intervals of between 2-4 days. Subsequently, 1 mg is
usually given at intervals of three months. No NICE guidelines available. Monitory with Vit
B12 levels is not recommended so taper according to severity of symptoms.
For patients with neurological involvement, referral to a haematologist is recommended.
Initial treatment is with hydroxocobalamin 1 mg on alternate days until there is no further
improvement, after which 1 mg should be given every two months for life.
Care should be taken not to give folic acid instead of VitB12 as this may result in
fulminant neurological defcit.
251. A 7yo is brought by his mother who says that he was well at birth but has been
suffering from
repeated chest and GI infections since then. She also says that he is not growing well for
this age.
What is the likely condition of this child?
a. CF
b. SCID
c. Primary Tcell immunodeficiency
d. Primary Bcell immunodeficiency
e. Malabsorption
Key = A (Cystic fibrosis)
Points in favour = recurrent chest and GI infections and child not growing well.

Epidemiology
CF is the most common inherited disease in white populations.[3]
Prevalence is 1 in 2,500 newborn infants, with calculated carrier frequency of 1 in
25.[4] Just over 10,000 people were recorded as having CF in the 2013 UK CF
Registry.[5]
The only risk factor is a family history of the condition.

Pathogenesis
The abnormality in the CFTR gene explains the pathology of CF.

High sodium sweat


Primary secretion of sweat duct is normal but CFTR does not absorb chloride ions,
which remain in the lumen and prevent sodium absorption.

Pancreatic insufficiency
Production of pancreatic enzymes is normal but defects in ion transport produce relative
dehydration of pancreatic secretions, causing their stagnation in the pancreatic ducts.

Biliary disease
Defective ion transfer across the bile duct causes reduced movement of water in the
lumen so that bile becomes concentrated, causing plugging and local damage.

Gastrointestinal disease
Low-volume secretions of increased viscosity, changes in fluid movement across both
the small and large intestine and dehydrated biliary and pancreatic secretions cause
intraluminal water deficiency.

Respiratory disease
Dehydration of the airway surfaces reduces mucociliary clearance and favours bacterial
colonisation, local bacterial defences are impaired by local salt concentrations and
bacterial adherence is increased by changes in cell surface glycoproteins.
Increased bacterial colonisation and reduced clearance produce inflammatory lung
damage due to an exuberant neutrophilic response involving mediators such as IL8 and
neutrophil elastase.

Presentation

As normal digestive function is possible with <5% pancreatic function, CF can


present at any age.
The most common presentation is with respiratory problems - usually recurrent
lower respiratory tract infection (LRTI) with chronic sputum production.
However, immunoreactive trypsinogen (IRT) is now measured on a dried blood spot
obtained on the Guthrie card at day six of life. Samples with abnormally raised IRT levels
will undergo CFTR mutation screening. This was introduced in 2007. This therefore
means that clinical presentation of CF will become rarer. However, screening failures do
sometimes occur. Presentation of CF varies with age.
Presentation and diagnosis
Antenatal

Amniocentesis/chorionic villus sampling (CVS).


Ultrasound demonstration of bowel
perforation/hyperechogenic bowel (4% cases
due to CF).

Perinatal

Screening.
Bowel obstruction with meconium ileus (bowel
atresia).
Haemorrhagic disease of the newborn.
Prolonged jaundice.

Infancy and childhood

Recurrent respiratory infections.


Diarrhoea.
Failure to thrive (thriving does not exclude
diagnosis).
Rectal prolapse.
Nasal polyps (in children, nearly always due to
CF).
Acute pancreatitis.
Portal hypertension and variceal haemorrhage.
Pseudo-Bartter's syndrome, electrolyte
abnormality.
Hypoproteinaemia and oedema.

Adolescence/adulthood

Screening.
Recurrent respiratory infections.
Atypical asthma.
Bronchiectasis.
Male infertility with congenital bilateral absence
of the vas deferens.
Heat exhaustion/electrolyte disturbance.
Portal hypertension and variceal haemorrhage.

Signs
These may include:
Finger clubbing.
Cough with purulent sputum.
Crackles.
Wheezes (mainly in the upper lobes).
Forced expiratory volume in one second (FEV1) showing obstruction.
Babies diagnosed with CF will usually have no signs or symptoms.

Investigations

Sweat testing confirms the diagnosis and is 98% sensitive. Chloride


concentration >60 mmol/L with sodium concentration lower than that of chloride
on two separate occasions.
Molecular genetic testing for CFTR gene.
Sinus X-ray or CT scan - opacification of the sinuses is present in almost all
patients with CF.
CXR or CT of thorax.
Lung function testing - spirometry is unreliable before 6 years.
Sputum microbiology - common pathogens include Haemophilus
influenzae,Staphylococcus aureus, Pseudomonas aeruginosa, Burkholderia
cepacia, Escherichia coli and Klebsiella pneumoniae.
Various blood tests including FBC, U&Es, fasting glucose, LFTs and vitamin A, D
and E levels are usually performed.
Semen analysis if appropriate.

Management is done by multi disciplinary team and is problem specific as the


disease has such a wide spectrum of abnormalities including multiple systems.
252. A 3yo child has a high temp for 4 days and he had not seen a doctor. Then mother
notices rashes
on buccal mucosa and some around the mouth. What is the most appropriate dx?
a. Measles
b. Roseola infectiosum
c. Rubella
d. Chicken pox
e. Impetigo
253. A 70yo lady presents with fever for 3d and confusion. There is no significant PMH.
What is the

most probable dx?


a. Delirium
b. Hypoglycemia
c. Alzheimers
d. DKA
Key = A (Delirium)
Points in favour = Old age with acute confusion accompanied by fever for 3
days(causing factor). The fact that there is no other significant PMH.
Acute/sub-acute confusion in an older patient should be considered delirium until proven
otherwise. Almost possibly any underlying medical condition can be the etiology but
most common are infections, drugs and alcohol withdrawal. Differential may include
dementia, Bipolar disorder, depression and functional psychosis.
Depression has its specific set of symptoms such as low mood, sadness, loss of
interest etc and although delirium can present with these features as well along with
confusion, for the dx to be depression the symptoms must be at least 2 weeks long and
cause significant impairment.
Bipolar disorder, Dementia and Schizophrenia all have a chronic history with specific
symptoms for each one of them.
Ruling out = DKA is not common in elderly and there is no history of DM.
While hypoglycemia can present with acute confusion but not for 3 days for
it to be hypoglycemia we need more symptoms and an earlier duration of onset.
Alzheimer's has to have a chronic history of dementia and can not present with acute
confusion only without any PMH.
Dx = Delirium
ICD-10 definition : An etiologically nonspecific organic cerebral syndrome characterized
by concurrent disturbances of consciousness and attention, perception, thinking,
memory, psychomotor behaviour, emotion, and the sleep-wake schedule. The duration
is variable and the degree of severity ranges from mild to very severe.
EPI = There is increase in incidence with age reaching to 13.6% in those over 85 years.
The prevalence is high in those with malignancy and HIV and in patients with preexisting cognitive impairment.
Presentation : The diagnosis of delirium is clinical. The following features may be
present:
Usually acute or subacute presentation.
Fluctuating course.
Consciousness is clouded/impaired cognition/disorientation.
Poor concentration.
Memory deficits - predominantly poor short-term memory.
Abnormalities of sleep-wake cycle, including sleeping in the day.
Abnormalities of perception - eg, hallucinations or illusions.
Agitation.
Emotional lability.
Psychotic ideas are common but of short duration and of simple content.
Neurological signs - eg, unsteady gait and tremor.

There are no specific investigations and we can order possibly the whole battery of tests
to find the underlying cause.
Management comprises of supportive and environmental measure along with medical
therapy consisting of antipsychotics such as haloperidol and olanzapine. Then comes
the management post-discharge.
Delirium resulting from alcohol discharge is called Delirium tremens and can be
treated with benzodiazepines such as diazepam and chlordiazepoxide.

254. An obese mother suffers from OSAS. Which of the following inv is best for her?
a. ABG
b. Overnight pulse-oximetry
c. Polysomnography
d. EEG
Key = C (Polysomnography)
Points in favour = It is the gold standard investigation for OSAS
In PSG various physiological recordings are taken whilst the patient is asleep overnight.
It cosists of at least an EEG, two elctro-oculograms to measure eye movement and an
electromyogram on the chin to monitor muscle movement.
Overnight pulse-oximetry is a cheaper and more readily available method of diagnosing
OSAS but PSG remains to the best one possible.
ABGs are done to rule out daytime hypoxia or hypercapnia in patients presenting with
symptoms of heart failure. EEG alone is of no diagnostic help.
Dx = Obstructive Sleep Apnea Syndrome
EPI = Incidence is increasing as the incidence of obesity is increasing. Highly prevalent
in patients with type 2 DM and related metabolic conditions.
Risk factors include:
Obesity (strongest risk factor).
Male gender.
Middle age (55-59 in men, 60-64 in women).
Smoking.
Sedative drugs.
Excess alcohol consumption.
Family history
Possibly genetic tendency related to jaw morphology.
Obese children - they have a higher prevalence and severity of OSAS.
Presentation : The following may be suggestive OSAS

Impaired concentration.
Snoring.
Unrefreshing sleep.
Choking episodes during sleep.
Witnessed apnoeas.
Restless sleep.

Irritability/personality change.
Nocturia.
Decreased libido.
Excessive daytime sleepiness.

Along with obesity, increased neck circumference and certain craniofacial or pharyngeal
abnormalities may be notes in examination.
Treatment : CPAP remains to be the treatment of choice along with some behaviour
modification which include sleeping posture, smoking and alcohol cessation and weight
loss. Oral appliances such as mandibular advance splits can also be used for mild to
moderate OSAS.
Surgery is considered only for patients for whom CPAP and oral appliances have failed.
255. A 28yo business man came to the sexual clinic. He was worried that he has HIV
infection. 3 HIV
tests were done and all the results are negative. After a few months, he comes back
again and
claims that he has HIV. What is the dx?
a. Somatization
b. Hypochondriac
c. Mancheusens
d. OCD
e. Schizophrenia
Key = B (Hypochondriasis)
Points in favour = Belief/delusion of having a serious/life threatening disease
which persists after being ruled out by investigations.
Somatoform disorders consists of a group psychiatric illnesses in which patients either
feel or make up signs and symptoms without existence of an organic disease. It includes
the following disorders :
Somatization disorder : Characterised by occurrence of chronic multiple somatic
symptoms for which there is no physical cause.
Hypochondriasis : Patients have a strong fear or belief that they have a serious often
fatal disease that persists despite appropriate medical reassurance. (like in this case).
Body dysmorphic disorder : A preoccupation with bodily shape or appearance with
belief that one is disfigured in some way.
Dissociative (conversion) disorder : Characterised by loss or distortion of neurological
function not fully explained by organic disease. Previously known as Hysteria.
Somatoform Pain disorder: Severe, persistent pain which cannot be explained by
medical condition.
Munchausens syndrome : Severe chronic form of factitious disorder usually older
males who travel widely sometimes in several hospitals in one day. They are convincing
enough to persuade doctors to undertake investigations but no underlying condition is
found. Previous similar hospital visits can be traced.
This cannot be Schizophrenia because of absence of specific symptom featuring in
schizophrenia.

Management : Cognitive behavioural therapy is the main course in management.


Antipsychotics may help but main line of treatment remains to be reassurance and CBT.
256. A 6wk child presents with progressive cyanosis, poor feeding, tachypnea over the
first 2 wks of
life and holosystolic murmur. What is the most appropriate condition?
a. ASD
b. VSD
c. Tricuspid atresia
d. PDA
e. TOF
257. A 29yo woman who was dx to have migraine presents with severe onset of occipital
headache.
She lost her consciousness. CT=normal. Neurological exam=normal. What is the most
appropriate management?
a. Repeat CT
b. MRI
c. LP
d. XR
e. No inv required
Key = E (no investigation required)
Points in favour = occipital headache is common in migraine. Normal CT and
neuro exam means there is no need for any further investigations.
Loss of consciousness in patients with migraine is an alarming sign and should prompt
for imaging straight away. A normal CT and no focal neurological deficit on examination
reassure that there is no new pathology. The young age of the patient gives us a clue as
well. So there is no need of any intervention in this scenario.
Dx = Migraine
Classification = Migraine with aura, Migraine without aura and Chronic migraine
EPI = Common in women than men (vice versa in children)
First attack often in childhood with over 80% having their first attack before the
age of 30. If onset is at age over 50 other pathology should be sought.
Family history is usually positive.
Severity decreases with advancing age.
Presentation : Typically the headaches last between 4 and 72 hours and have at least
two of the following features:
Unilateral.
Pulsating.
Moderate or severe intensity of pain.
Aggravated by, or resulting in the avoidance of, routine physical activity.
In addition, there is at least one of:
Nausea and vomiting during migraine attacks. These are common symptoms that
affect at least 60% of patients suffering from migraines.
Photophobia and phonophobia, which are also very common.
Aura is highly variable in nature (visual,sensory etc) but tends to be consistent for an
individual. Headache starts maximum within one hour of an aura.

Investigations : Diagnosis is purely clinical. Investigations are only done to rule out
secondary causes of headache or when alarming symptoms like loss of consciousness,
seizure. memory loss etc present.
Treatment : Acute attack = Step one = Simple analgesics with or without anti emetics
Step two = Anal analgesics and anal anti emetics
Step three = Triptans or ergotamine ( Patients having
moderate to severe migraine should be moved directly to step 3 ).
Prophylaxis = The NICE guidelines and CKS suggests the following indications for
prophylaxis :
Frequent attacks are two or more attacks per month that produce disability
lasting for three days or more.
Medication overuse is a risk when medication is used on more than two days per
week on a regular basis. Overuse needs to be addressed before further
treatment can begin.
Prophylaxis should be used when standard analgesia and triptans are either
contra-indicated or ineffective.
Beta blockers, amitriptyline, sodium valproate and Botulinum toxin A all can be
used as drugs for prophylaxis. Identifying and refraining from triggering factor (if
any) can also be used to avoid attacks such as stress, dietary factors like
cheese,chocolate,alcohol etc.

258. A 19yo man has been happier and more positive than usual, with more energy than
he has ever
felt before for no particular reason. He has been getting more work done at the office
today and
has been socializing with his friends as usual. What is the most likely dx?
a. Atypical depression
b. Marked depression
c. Bipolar syndrome
d. Psychosis
e. Hypomania
Key = E
Points in favour = There is elevation of mood and energy, increase in activity but
not to severity of Mania and there is no evidence of delusions or hallucinations.
Hypomania is a milder form of mania. There is elevation of mood and energy , increase
in activity and socializing, inability to concentrate and flight of ideas but without
hallucinations or delusions.
Mania on the other hand has more severe symptoms like pressured speech, grandiosity,
increase in sexual activity, insomnia, flight of ideas, psychomotor agitation along with
hallucinations and delusions.
For the dx of Bipolar disorder there needs to be a documented episode of depression
along with mania (bipolar 1) or hypomania (bipolar 2).
Depression has opposite symptoms of mania like low mood and energy , low activity and
interest etc.

Manic episodes can be treated by atypical antipsychotics like olanzapine and


risperidone.
Lithium is used but it has slower onset of action. Lorazepam can be used to
sedate and decrease agitation. Carbamazepine is sometimes used as first line
treatment.
259. A 35yo female attempts suicide 10x. There is no hx of psychiatric problems and all
neurological
exams are normal. What is the best tx?
a. Problem focused tx
b. CBT
c. Antipsychotic
d. Antidepressant
e. ECT
260. A 57yo man presents with weight loss, tiredness, fever and abdominal discomfort.
Exam: spleen
palpable up to the umbilicus. Labs: WBC=127, Hgb=8.7, Plt=138. What is the most likely
dx?
a. CML
b. AML
c. CLL
d. AML
e. Polycythemia
Key = A (CML)
Points in favour = Splenomegaly and raised WBC along with typical signs and
symptoms.
261. A baby born at 34 weeks with a heart murmur is kept in the incubator for almost 4
weeks. There
is no murmur at discharge. What is the likely cause of this murmur?
a. PDA
b. TOF
c. Aneurysm of sinus of Valsalva
d. Aorto-pulmonary septal defect
e. AVM
Key = A (PDA)
Points in favour = PDA can be found in pre mature babies which closes after birth.
The ductus arteriosus is, in developmental terms, a remnant of the sixth aortic arch and
connects the pulmonary artery to the proximal descending aorta just after the left
subclavian artery origin. It is a normal structure in fetal life.
In utero the lungs are not expanded. Gas exchange occurs at the placenta and only
about 10% of the circulation passes through the lungs. The ductus arteriosus connects
the pulmonary artery to the aorta to shunt most of the blood away from the lungs. After
delivery it closes and the blood passes through the opened lungs. Failure of the ductus
arteriosus to close can lead to overloading of the lungs. The shunt is left to right unless
pulmonary hypertension occurs and pulmonary pressure exceeds systemic pressure.

After birth the ductus closes functionally in 12-18 hours and anatomically in 2-3 weeks. If
it remains open beyond three months of life in preterm infants and beyond one year of
life in full-term infants it is termed as persistent patency of ductus arteriosus because the
incidence of spontaneous closure beyond these time limits is very low.
Murmurs in all other cases will persist.
PDA:
History : Patients with a small PDA are usually asymptomatic.A large-shunt PDA may
cause lower respiratory tract infection as well as feeding difficulties and poor growth
during infancy, with failure to thrive because of heart failure.
Examination :
If the pulmonary circulation is markedly overloaded there will be tachycardia,
tachypnoea and a wide pulse pressure.
The precordium is hyperactive and a systolic thrill may be present at the upper
left sternal border.
The first heart sound is normal but the second is often obscured by the murmur.
A grade 1 to 4/6 continuous ('machinery') murmur is best audible at the left
infraclavicular area or upper left sternal border.
In the case of a large PDA shunt, a diastolic mitral rumble may be heard because
of the high flow rate across the mitral valve.
Patients with a small PDA do not have the above-mentioned findings.
Peripheral pulses are bounding as the run-off into the pulmonary circulation
drops the diastolic pressure and causes a wide pulse pressure.
Investigations : ECG, CXR and Echocardiography
Management : Indometacin is ineffective in term infants with PDA and should not be
used. Medical management is limited to use of decongestive measures such as diuretics
in those with features of heart failure.
PDA closure is indicated for any symptomatic infant, child or adult (with exclusion
of those with fixed high pulmonary vascular resistance). Closure is also indicated
in asymptomatic patients with left heart volume load. This can be done either by
surgery or interventional techniques at any age.[4]
Surgical closure is reserved for patients in whom a non-surgical closure
technique is not considered applicable. In infants with heart failure or pulmonary
hypertension, surgery is performed on an urgent basis. The standard surgical
procedure is ligation and division of the ductus through left posterolateral
thoracotomy without cardiopulmonary bypass.[3]
In asymptomatic well infants current practice is to wait until 1 year of age, with
regular echocardiographic evaluation to check for spontaneous closure of the
PDA. If the duct is still patent at 1 year of age it can be closed usually by
occlusion at cardiac catheterisation (endovascular occlusion). National Institute
for Health and Care Excellence (NICE) guidance has been produced and
considers that current evidence on the safety and efficacy of endovascular
occlusion of PDA appears to support the use of this procedure.[5] The procedure
should be performed in units where there are arrangements for cardiac surgical
support in the event of complications. The choice of device depends largely on
the size of PDA. Coils are suitable for closing of small- to medium-sized PDAs
while larger PDAs require other devices such as the Amplatzer patent ductus
arteriosus device.

Serious complications of transcatheter closure of PDA are rare and include


device embolisation, femoral artery or vein thrombosis related to vascular access
and infection.[4]
Whilst the ductus arteriosus is patent then the risk of endocarditis should be
considered (there is no increased risk of endocarditis once repair is complete).
Routine antibiotic prophylaxs is not indicated but during invasive procedures (eg,
urinary or gastrointestinal procedures) involving areas of sepsis, suitable
antibiotics should be given promptly (to cover all the likely organisms, including
any known to cause endocarditis).[6]

262. A 6yo girl who has previously been well presented with a hx of tonic-clonic seizures
lasting
4mins. Her mother brought he to the hospital ad she appeaed ell. She is afeile
ad didt
lose consciousness during the episode of seizure. She has no neurologic deficit. What is
the most
appropriate inv for her?
a. ABG
b. Serum electrolytes
c. ECG
d. Blood glucose
263. A 60yo woman was found by her son. She was confused and had urinary
incontinence. She has
recovered fully after 6h with no neurological complaints. What is the most likely dx?
a. Stroke
b. Vestibular insufficiency
c. TIA
d. Intracranial hemorrhage
Key = C (TIA)
Points in favour = Complete recovery in less than 24 hours
A transient ischaemic attack (TIA) is a temporary inadequacy of the circulation in part of
the brain (a cerebral or retinal deficit) that gives a clinical picture similar to a stroke
except that it is transient and reversible. Hence, TIA is a retrospective diagnosis. The
duration is no more than 24 hours and a deficit that lasts longer than 24 hours is defined
as a stroke. The majority of TIAs last for less than 30 minutes.

Management
Secondary prevention (see below) includes the use of antiplatelet therapy,
antihypertensive, and lipid-modifying treatments, the management of atrial fibrillation if
present and the management of any other underlying or risk factors, including diabetes.

Driving[4]

Group 1 (car or motorcycle)


Must not drive for one month.
No need to notify DVLA after a single TIA.
Multiple TIAs over a short period: require three months free from further attacks
before resuming driving and DVLA should be notified.
Group 2 (lorry or bus)
Licence refused or revoked for one year following a stroke or TIA.

Assessment of the risk of stroke


An ABCD2 score of more than 4 suggests high risk of an early stroke.[5][6]

Scoring System for Risk of Stroke after TIA (ABCD2 Score)

Age

Age >60

Blood pressure

BP>140 systolic and/or >90 diastolic

Clinical features

Unilateral weakness

Speech disturbance without weakness

Other

>60 minutes

10-59 minutes

<10 minutes

Duration of symptoms

Diabetes

Presence of diabetes

RCP recommendations[1]

All patients with a TIA should be seen by a specialist in neurovascular disease


(eg, in a specialist neurovascular clinic or an acute stroke unit).
People with a suspected TIA should be assessed as soon as possible for their
risk of subsequent stroke by using a validated scoring system such as ABCD2
(as above).
Patients with suspected TIA who are at high risk of stroke (eg, an ABCD2 score
of 4 or above) should receive:
o Aspirin or clopidogrel (each as a 300 mg loading dose and then 75 mg
daily) and a statin started immediately.
o NB: clopidogrel is not licensed for the management of TIA and therefore
the National Institute for Health and Care Excellence (NICE) and the
British National Formulary (BNF) recommend aspirin plus modifiedrelease dipyridamole.[7]
o Specialist assessment and investigation within 24 hours of onset of
symptoms.
o Measures for secondary cardiovascular prevention introduced as soon as
the diagnosis is confirmed, including discussion of individual risk factors.
People with crescendo TIA (two or more TIAs in a week), atrial fibrillation or
those on anticoagulants should be treated as being at high risk of stroke even if
they may have an ABCD2 score of 3 or below.
Patients with suspected TIA who are at low risk of stroke (eg, an ABCD2 score of
3 or below) should receive:
o Aspirin or clopidogrel (each as a 300 mg loading dose and then 75 mg
daily) and a statin.
o NB: clopidogrel is not licensed for the management of TIA and therefore
NICE and the BNF recommend aspirin plus modified-release
dipyridamole.[7]
o Specialist assessment and investigations as soon as possible, but
definitely within one week of onset of symptoms.
o Measures for secondary prevention introduced as soon as the diagnosis
is confirmed, including discussion of individual risk factors.
People who have had a TIA but present late (more than one week after their last
symptom has resolved) should be treated as though they are at a lower risk of
stroke.
Patients with TIA in atrial fibrillation should be anticoagulated in the TIA clinic
once intracranial bleeding has been excluded and if there are no other contraindications.
If the patient is in atrial fibrillation, management of that condition is required. In persistent
atrial fibrillation there is benefit from anticoagulation but there is no evidence of any such
benefit in the absence of atrial fibrillation.
All people with TIA, who after specialist assessment are considered candidates for
carotid endarterectomy, should have carotid imaging conducted urgently to facilitate

carotid surgery, which should be undertaken within seven days of the onset of
symptoms.

264. A 34yo woman presents 3 weeks after childbirth. She has had very low mood and
has been
suffering from lack of sleep. She also has thought of harming her little baby. What is the
most
appropriate management for this pt?
a. ECT
b. CBT
c. IV haloperidol
d. Paroxethine
e. Amitryptiline
Key = A (ECT)
Points in favour = This is postpartum psychosis since the onset is more than 2
weeks after delivery. There is evidence of depression and psychosis so ECT
should be done.
265. A 65yo woman presents with headache. She also complains of dizziness and
tinnitus. He has
recently realized she has visual problems. There is hx of burning sensation in fingers
and toes.
On exam: splenomegaly, itchy after hot bath. Labs: RBC=87, Hgb=31.9, Plt=796. What
is the dx?
a. CML
b. CLL
c. Polycythemia vera
d. Myelofibrosis
e. NHL
Key = C (Polycythemia vera)
Points in favour = Headache, burning finger along with visual problems. High hb
266. A 29yo male brought to ED in conscious state. There is no significant past hx.
Which of the
following should be done as the initial inv?
a. CT
b. Blood glucose
c. ABG
d. MRI
e. CBC
Key = B (blood glucose)
Points in favour = Always check for hypoglycemia in unconscious patient first
especially young patients with no significant past history.
Assessing BSR levels is much easier and less invasive than other tests and
quicker to rule out.

Diagnostic criteria

The World Health Organization reclassified chronic myeloproliferative diseases as


myeloproliferative neoplasms in 2008.[4] The criteria for the diagnosis of PRV requires
two major criteria and one minor criterion, or the first major criterion and two minor
criteria.
Major criteria:
o Haemoglobin of more than 18.5 g/dL in men, 16.5 g/dL in women, or
elevated red cell mass greater than 25% above mean normal predicted
value.
o Presence of JAK2 617V F mutation or other functionally similar mutations,
such as the exon 12 mutation of JAK2.
Minor criteria
o Bone marrow biopsy showing hypercellularity with prominent erythroid,
granulocytic, and megakaryocytic proliferation.
o Serum erythropoietin level below normal range.
o Endogenous erythroid colony formation in vitro.
Other confirmatory findings no longer required for diagnosis include:
o Oxygen saturation with arterial blood gas greater than 92%.
o Splenomegaly.
o Thrombocytosis (>400,000 platelets/mm3).
o Leukocytosis (>12,000/mm3).
o Leukocyte alkaline phosphatase (>100 units in the absence of fever or
infection).

Investigation

Initial blood tests:


o FBC in PCV will show not only elevated Hb and packed cell volume but
WCC and platelets will be elevated too. In secondary polycythaemia only
red blood cells are raised.
o Ferritin is often low in primary polycythaemia because of increased
demand for iron. In secondary causes it is usually normal..
Radiology:
o Radioisotopes can be used to measure circulating volumes. Red cells can
be labelled with 51Cr and albumin with 131I. This is expensive, needs
skill and is not widely available.
o CT, MRI or ultrasound scanning of the abdomen may show enlargement
of the spleen as is often found in PRV. It should also check for
abnormalities of the renal system.
Bone marrow and aspirate:
o Tend to be hypercellular in PRV.
o In the plethoric phase, the blood smear shows normal erythrocytes,
variable neutrophilia with myelocytes, metamyelocytes, and varying
degrees of immaturity, basophilia, and increased platelet counts.
o In the spent phase, the blood smear shows abundant teardrop cells,
leukocytosis, and thrombocytosis.
o Generally the findings are not specific to PRV. The bone marrow can be
normal in PRV.
Serum erythropoietin levels are often low in PRV. This can differentiate
secondary erythrocytosis and pseudoerythrocytosis from PRV, but there is
overlap in the levels found and it cannot reliably differentiate.

Cytogenetic studies. Karyotyping can detect fewer than 30% of patients with
PRV. An abnormal test is useful, but a normal test does not exclude PRV.
Clonal assays (using glucose-6-phosphate dehydrogenase (G6PD) markers) are
not generally available for clinical use. Even if it were available it is only of use in
female patients.
Research markers include the thrombopoietin receptor MPL expression and the
PRV1 mRNA in granulocytes.[5]

JAK2 testing
With the development of new techniques for detecting the Janus kinase 2 (JAK2) V617F
mutation this may become a clinically useful marker for PRV. It has been recommended
as a diagnostic marker.[6][5]
JAK2-positive polycythaemia vera is diagnosed if:[2]
The JAK2 mutation is identified; and
The haematocrit is more than 0.48 in women or more than 0.52 in men, or the
red cell mass is 25% higher than normal.
JAK2-negative polycythaemia vera is diagnosed if:[2]
The JAK2 mutation is not identified; and
The haematocrit is more than 0.56 in women or more than 0.60 in men, or the
red cell mass is 25% higher than normal; and
There is no identifiable secondary cause for polycythaemia; and either
o There is palpable splenomegaly or the presence of an acquired genetic
abnormality in the haematopoietic stem cells or both; or
o Any two of the following clinical features are identified: an abnormally
increased platelet count, an abnormally increased neutrophil count,
radiological evidence of splenomegaly, and abnormally low serum
erythropoietin.

Management
The main concern with the management of the disease is the prevention of thrombosis,
which is the main cause of morbidity and mortality. Fibrotic and leukaemic disease also
raises mortality and morbidity.
Intermittent long-term phlebotomy to maintain the haematocrit below 45% (lower
target level may be appropriate for women). Phlebotomy may cause progressive
and sometimes severe thrombocytosis and iron deficiency. Splenomegaly and
pruritus may persist despite control of the haematocrit by phlebotomy.[7]
Low-dose aspirin produces a small reduction in thrombotic events, including
myocardial infarction and stroke, whilst not increasing the risk of
haemorrhage.[8][9]
If it is not possible to control thrombotic events with phlebotomy alone then
myelosuppression must be considered. However, this is not without risk and
increases the risk of leukaemic transformation. Risks and benefits have to be
balanced.
Chemotherapy options include:[2]
o For people younger than 40 years of age: first-line is interferon; secondline is hydroxycarbamide or anagrelide.
o For people 40-75 years of age: first-line is hydroxycarbamide; second-line
is interferon or anagrelide.

For people older than 75 years of age: first-line is hydroxycarbamide;


second-line is radioactive phosphorus or busulfan.
Pruritus can be quite disabling:
o Taking baths or showers at lower temperatures and patting the skin dry,
to avoid rubbing, may help.
o Antihistamines, including H2 receptor antagonists (H2RAs), are useful in
refractory cases.
o Selective serotonin reuptake inhibitors (SSRIs) - eg, paroxetine or
fluoxetine.
Elevated uric acid may require allopurinol.
It may be necessary to consider splenectomy when there is painful splenomegaly
or there are repeated episodes of splenic infarction.

267. A 45yo woman comes with red, swollen and exudating ulcer on the nipple and
areola of right
breast with palpable lump under the ulcer. What do you think is nthis skin condition?
a. Inflammatory cells releasing cytokines
b. Infiltration of the lymphatics by the carcinomatous cells
c. Infiltration of the malignant skin cells to the breast tissue
Key = B
Points in favour = This is a case of CA breast in which infiltration of the
lymphatics cause the ulceration.
268. A 20yo young lady comes to the GP for advice regarding cervical ca. she is worried
as her mother
past away because of this. She would like to know what is the best method of
contraception in
her case?
a. POP
b. Barrier method
c. IUCD
d. COCP
e. IUS
Key = B (barrier method)
Points in favour = barrier method can help prevent catching HPV infection which
is the main etiology behind CA cervix. Other methods may provide with better
contraception but are not good means of preventing hpv infections.
269. A 66yo man, an hour after hemicolectomy has an urine output of 40ml. However,
an hour after
that, no urine seemed to be draining from the catheter. What is the most appropriate
next step?
a. IV fluids
b. Blood transfusion
c. Dialysis
d. IV furosemide
e. Check catheter
Key = E
Points in favour = Always check catheter for any obstruction or other abnormality
before iv fluids.

270. A 24yo pt presented with anaphylactic shock. What would be the dose of
adrenaline?
a. 0.5ml of 1:1000
b. 0.5ml of 1:10000
c. 1ml of 1:500
d. 5ml of 1:1000
e. 0.05ml of 1:100
Key = A
271. A 44yo woman complains of heavy bleeding per vagina. Transvaginal US was done
and normal.
Which of the following would be the most appropriate inv for her?
a. Hysterectomy
b. Endometrial biopsy
c. CBC
d. High vaginal swab
e. Coagulation profile
Key = E (coagulation profile)
Points in favour = After normal vaginal US coagulation profile should be done to
rule out systemic causes of heavy bleeding first. Endometrial biopsy will be
needed if ultrasound shows some endometrial abnormality. Before considering
steps like hysterectomy, systemic causes of bleeding must be ruled out by
checking coagulation profile.
CBC and high vaginal swab will not help much in finding the cause of bleeding.
272. A 60yo woman presented to OPD with dysphagia. No hx of weight loss of
heartburn. No change
in bowel habits. While doing endoscopy there is some difficulty passing through the LES,
but no
other abnormality is noted. What is the single most useful inv?
a. CXR
b. MRI
c. Esophageal biopsy
d. Esophageal manometry
e. Abdominal XR
Key = D
Points in favour = This can be a case of achalasia or esophageal spasms. In both
cases manometry is the gold standard investigation. Another investigation which
could have been done before manometry and even endoscopy is The Barium
Swallow.
Remaining inx given in the question will not help is in the diagnosis.
Achalasia is primarily a disorder of motility of the lower oesophageal or cardiac
sphincter. The smooth muscle layer of the oesophagus has impaired peristalsis and
failure of the sphincter to relax causes a functional stenosis or functional oesophageal
stricture. Most cases have no known underlying cause, but a small proportion occurs
secondary to other conditions - eg, oesophageal cancer.
It tends to present in adult life and is very rare to present in children.

Presentation

The most common presenting feature is dysphagia. This affects solids more than
soft food or liquids.
Regurgitation may occur in 80-90% and some patients learn to induce it to
relieve pain.
Chest pain occurs in 25-50%. It occurs after eating and is described as
retrosternal. It is more prevalent in early disease.
Heartburn is common and may be aggravated by treatment.
Loss of weight suggests malignancy (may co-exist).
Nocturnal cough and even inhalation of refluxed contents is a feature of later
disease.
Examination is unlikely to be revealing although loss of weight may be noted.
Rarely, there may be signs of an inhalation pneumonia.

Treatment : Calcium channel blockers and nitrates can be used.


Pneumatic dilatation or endoscopic botulinum toxin injection can be
used as well.
Heller myotomy remains to be best treatment of choice in patients who are
fit for surgery.
273. A 24yo woman presents with deep dyspareunia and severe pain in every cycle.
What is the
initial inv?
a. Laparoscopy
b. Pelvic US
c. Hysteroscopy
d. Vaginal Swab
Key = B (Pelvis ultrasound)
Points in favour = To rule out cervical abnormalities, endometriosis. ovarian cysts
etc.
274. A 38yo woman, 10d postpartum presents to the GP with hx of passing blood clots
per vagina
since yesterday. Exam: BP=90/40mmhg, pulse=110bpm, temp=38C, uterus tender on
palpation
and fundus 2cm above umbilicus, blood clots +++. Choose the single most likely dx/
a. Abruption of placenta 2nd to pre-eclampsia
b. Concealed hemorrhage
c. Primary PPH
d. Secondary PPH
e. Retained placenta
f. Scabies
275. A 32yo female with 3 prv 1st trimester miscarriages is dx with antiphospholipid
syndrome.
Anticardiolipin
antibodies +ve. She is now 18wks pregnant. What would be the most appropriate
management?
a. Aspirin
b. Aspirin & warfarin
c. Aspirin & heparin
d. Heparin only

e. Warfarin only
Key = C
Points in favour = More than 3 prev miscarriages due to APLS - LMWH plus aspirin
throughout pregnancy is indicated.
Antiphospholipid syndrome (APS) is an autoimmune disorder characterised by arterial
and venous thrombosis, adverse pregnancy outcomes (for mother and fetus), and raised
levels of antiphospholipid (aPL) antibodies.

Presentation:
APS has varied clinical features and a range of autoantibodies. Virtually any system can
be affected, including:[1][4][5]
Peripheral artery thrombosis, deep venous thrombosis.
Cerebrovascular disease, sinus thrombosis.
Pregnancy loss: loss at any gestation - recurrent miscarriage or prematurity can
be seen in APS.
Pre-eclampsia, intrauterine growth restriction (IUGR).
Pulmonary embolism, pulmonary hypertension.
Livedo reticularis (persistent violaceous, red or blue pattern of the skin of the
trunk, arms or legs; it does not disappear on warming and may consist of regular
broken or unbroken circles), purpura, skin ulceration.
Thrombocytopenia, haemolytic anaemia.
Libman-Sacks endocarditis and cardiac valve disease:
o Usually mitral valve disease or aortic valve disease and usually
regurgitation with or without stenosis.
o Mild mitral regurgitation is very common and is often found with no other
pathology. There may also be vegetations on the heart and valves.
Myocardial infarction.
Retinal thrombosis.
Nephropathy: vascular lesions of the kidneys may result in chronic kidney
disease.
Adrenal infarction.
Avascular necrosis of bone.

Investigations
Young adults (50 years old) with ischaemic stroke and women with recurrent
pregnancy loss (3 pregnancy losses) before 10 weeks of gestation should be screened
for aPL antibodies.[3]
Levels of aCL, anti-beta2 GPI or lupus anticoagulant (LA) on two occasions at
least 12 weeks apart.
FBC; thrombocytopenia, haemolytic anaemia.
Clotting screen.
CT scanning or MRI of the brain (cerebrovascular accident), chest (pulmonary
embolism) or abdomen (Budd-Chiari syndrome).
Doppler ultrasound studies are recommended for possible detection of deep vein
thrombosis.

Two-dimensional echocardiography may demonstrate asymptomatic valve


thickening, vegetations or valvular insufficiency.

Management in Pregnancy :
APS in pregnancy may affect both mother and fetus throughout the entire
pregnancy and is associated with high morbidity. Clinical complications are
variable and include recurrent miscarriage, stillbirth, IUGR and pre-eclampsia.
For women with APS with recurrent (3) pregnancy loss, antenatal
administration of low molecular weight heparin combined with low-dose
aspirin is recommended throughout pregnancy. Treatment should begin as
soon as pregnancy is confirmed.
For women with APS and a history of pre-eclampsia or IUGR, low-dose
aspirin is recommended.
Women wit aPL antibodies should be considered for postpartum
thromboprophylaxis.
276. A 23yo presents with vomiting, nausea and dizziness. She says her menstrual
period has been
delayed 4 weeks as she was stressed recently. There are no symptoms present. What is
the next
appropriate management?
a. Refer to OP psychiatry
b. Refer to OP ENT
c. CT brain
d. Dipstick for B-hCG
Key = D
Points in favour = Test for pregnancy first in case of amenorrhea
e. MRI brain
277. A 16yo girl came to the sexual clinic. She complains of painful and heavy bleeding.
She says she
doest a egula le. What is the ost appropriate management?
a. Mini pill
b. Combined pill
c. IUS
d. Anti-prostoglandins
e. Anti-fibrinolytics
278. A 36yo man walks into a bank and demands money claiming he owns the bank. On
being denied,
he goes to the police station to report this. What kind of delusions is he suffering from?
a. Delusion of reference
b. Delusion of control
c. Delusion of guilt
d. Delusion of persecution
e. Delusion of grandeur
Key = E
Points in favour = Delusion of grandeur is defined as delusion of exaggerated self
worth. Hence the answer.
279. Which method of contraception can cause the risk of ectopic pregnancy?

a. COCP
b. IUCD
c. Mirena
d. POP
Key = B (IUCD)
280. A woman has pernicious anemia. She has been prescribed parenteral vitamin B12
tx but she is
needle phobic. Why is oral tx not preferred for this pt?
a. IM B12 is absorbed more
b. Intrinsic factor deficiency affects oral B12 utilization
c. IM B12 acts faster
d. IM B12 needs lower dosage
e. Pernicious anemia has swallowing difficulties
Key = B
Points in favour = There is def of intrinsic factor dt autoimmune causes in
pernicious anemia. VitB12 can not be absorbed without binding to intrinsic
factor.Refer to the explanation of answer to question number 250.
281. An old man comes to the doctor complaining that a part of this body is rotten and
he wants it
removed. What is the most likely dx?
a. Guilt
b. Hypochondriasis
. Muhauses
d. Nihilism
e. Capras syndrome
282. A 31yo woman who is 32weeks pregnant attends the antenatal clinic. Labs:
Hgb=10.7, MCV=91.
What is the most appropriate management for this pt?
a. Folate supplement
b. Ferrous sulphate 200mg/d PO
c. Iron dextran
d. No tx req
Key = D (no tx required)
Explanation : This is dilutional anemia hence no treatment required.
283. A 47yo man who is a chronic alcoholic with established liver damage, has been
brought to the
hospital after an episode of heavy drinking. His is not able to walk straight and is
complaining of
double vision and is shouting obscenities and expletives. What is the most likely dx?
a. Korsakoff psychosis
b. Delirium tremens
c. Wernickes encephalopathy
d. Tourettes syndrome
e. Alcohol dependence
Key = C (Wernickes encephalopathy)
Points in favour = Chronic alcoholic - thiamine deficiency - double vision, unable
to walk.

In case of delirium tremens , there is history of alcohol consumption around 42-72 hours
back. There are symptoms like hallucination, confusion and severe agitation and
sometimes seizures as well.
There is history of vocal or neurological tics for it to be tourettes syndrome.
Korsakoffs syndrome is a late complication of untreated Wernickes. They are both
together known as wernicke-korsakoff syndrome.
Wernicke-korsakoff syndrome:- Wernicke-Korsakoff syndrome (WKS) is a spectrum
of disease resulting from thiamine deficiency, usually related to alcohol abuse.
Presentation :
Vision changes:
o Double vision
o Eye movement abnormalities
o Eyelid drooping
Loss of muscle co-ordination:
o Unsteady, unco-ordinated walking
Loss of memory, which can be profound.
Inability to form new memories.
Hallucinations.
Examination of the nervous system may show polyneuropathy.
Reflexes may be decreased (or of abnormal intensity), or abnormal reflexes may
be present.
Gait and co-ordination are abnormal on testing.
Muscles may be weak and may show atrophy.
Eyes show abnormalities of movement - nystagmus, bilateral lateral rectus palsy
and conjugate gaze palsy.
Blood pressure and body temperature may be low.
Pulse may be rapid.
The person may appear cachectic.
Confabulation
Memory loss
Retrograde amnesia

Encephalopathy
At least two of the four following criteria should be present to diagnose
encephalopathy:[7]
Dietary deficiencies.
Oculomotor abnormalities.
Cerebellar dysfunction.
Either an altered mental state or mild memory impairment.
Treatment :
Thiamine orally (IM or IV may be used in secondary care) plus vitamin B complex
or multivitamins, which should be given indefinitely. Treatment with thiamine is
often started under specialist care, although when deficiency is suspected, it
should be started in primary care.
Offer oral thiamine to harmful or dependent drinkers if either of the following
applies:[9]

They are malnourished (or have a poor diet); prescribe oral thiamine 50
mg per day (as a single dose) for as long as malnutrition may be present.
o They have decompensated liver disease.
A Cochrane review found there was insufficient evidence from randomised
controlled clinical trials to guide clinicians in the dose, frequency, route or
duration of thiamine treatment of WKS due to alcohol abuse.[10] However, more
recent work states that the route of administration and dose depend on the
severity of dependence and overall physical health of the patient.[11]
Although potentially serious allergic adverse reactions may (rarely) occur during,
or shortly after, parenteral administration, the Commission on Human Medicines
has recommended that:[12]
o This should not preclude the use of parenteral thiamine in patients where
this route of administration is required, particularly in patients at risk of
WKS where treatment with thiamine is essential.
o IV administration should be by infusion over 30 minutes.
o Facilities for treating anaphylaxis (including resuscitation facilities) should
be available when parenteral thiamine is administered.

284. A 32yo woman of 39wks gestation attends the antenatal day unit feeling very
unwell with
sudden onset of epigastric pain a/w nausea and vomiting. Temp 36.7C. Exam: RUQ
tenderness.
Bloods: mild anemia, low plts, elevated LFT and hemolysis. What is the most likely dx?
a. Acute fatty liver of pregnancy
b. Acute pyelonephritis
c. Cholecystitis
d. HELLP syndrome
e. Acute hepatitis
Key = D (HELLP syndrome)
Points in favour = hemolysis, elevated LFTs and low platelets

Presentation

HELLP syndrome is a serious form of pre-eclampsia and patients may present at


any time in the last half of pregnancy.
One third of women with HELLP syndrome present shortly after delivery.
Symptoms of HELLP syndrome are usually nonspecific.
Initially, women may report nonspecific symptoms including malaise, fatigue,
right upper quadrant or epigastric pain, nausea, vomiting, or flu-like symptoms.
Hepatomegaly can occur.
Some women may have easy bruising/purpura.
On examination, oedema, hypertension and proteinuria are present.
Tenderness over the liver can occur.

Investigations

There needs to be a high index of clinical suspicion in order to avoid diagnostic


delay and improve outcome.
Haemolysis with fragmented red cells on the blood film
Raised LDH >600 IU/L with a raised bilirubin.
Liver enzymes are raised with an AST or ALT level of >70 IU/L.
Levels of AST or ALT >150 IU/L are associated with increased maternal
morbidity and mortality.

Management

The main treatment is to deliver the baby as soon as possible, even if premature,
since liver function in the mother gets worse very quickly.
Problems with the liver can be harmful to both mother and child.
Definitive treatment of HELLP syndrome requires delivery of the fetus and is
advised after 34 weeks of gestation if multisystem disease is present.
There is no clear evidence of any effect of giving corticosteroids on clinical
outcomes for women with HELLP syndrome.[2]
Transfusion of red cells, platelets, fresh frozen plasma and cryoprecipitate or
fibrinogen concentrate are required as indicated clinically and by blood and
coagulation tests.
Postpartum HELLP syndrome may be treated with steroids and plasma
exchange.
If the fetus is less than 34 weeks of gestation and delivery can be deferred,
corticosteroids should be given.
Blood pressure control is very important.
Women with severe liver damage may need liver transplantation.

285. A 57yo woman presents with dysuria, frequency and urinary incontinence. She
complains of
dyspareunia. Urine culture has been done and is sterile. What is the most appropriate
step?
a. Oral antibiotics
b. Topical antibiotics
c. Topical estrogen
d. Oral estrogen
e. Oral antibiotics and topical estrogen
Key = C (topical estrogen)
Explanation = The problem here is vaginal dryness for which the age and
symptoms are a good clue. Topical estrogen or HRT can be given to treat vaginal
dryness, vaginal discharge and recurrent UTIs in post menopausal women.
286. A pt came to the ED with severe lower abdominal pain. Vitals: BP=125/85mmHg,
Temp=38.9C.
Exam: abdomen rigid, very uncomfortable during par vaginal. She gave a past hx of PID
3 years
ago which was successfully treated with antibiotics. What is the appropriate inv?
a. US
b. Abdomen XR
c. CT
d. High vaginal
e. Endocervical swab

287. A pregnant woman with longterm hx of osteoarthritis came to the antenatal clinic
with
complaints of restricted joint movement and severe pain in her affected joints. What is
the
choice of drug?
a. Paracetamol
b. Steroid
c. NSAID
d. Paracetamol+dihydrocoiene
e. Pethadine
Key = A (paracetamol)
Explanation = Safest drug in pregnancy is paracetamol among the choices given
288. A 24yo 18wk pregnant lady presents with pain in her lower abdomen for the last
24h. She had
painless vaginal bleeding. Exam: abdomen is tender, os is closed. What is the most
probable dx?
a. Threatened miscarriage
b. Inevitable miscarriage
c. Incomplete miscarriage
d. Missed miscarriage
e. Spontaneous miscarriage
Key = A
Points in favour = painless vaginal bleeding , tender abdomen , os closed
Classification of miscarriage is as follows:
Threatened miscarriage: mild symptoms of bleeding. Usually little or no pain.
The cervical os is closed.
Inevitable miscarriage: usually presents with heavy bleeding with clots and
pain. The cervical os is open. The pregnancy will not continue and will proceed to
incomplete or complete miscarriage.
Incomplete miscarriage: this occurs when the products of conception are
partially expelled. Many incomplete miscarriages can be unrecognised missed
miscarriages.
Missed miscarriage: the fetus is dead but retained. The uterus is small for
dates. A pregnancy test can remain positive for several days. It presents with a
history of threatened miscarriage and persistent, dirty brown discharge. Early
pregnancy symptoms may have decreased or gone.
Habitual or recurrent miscarriage : three or more consecutive miscarriages.
. A o hild plaig i the gade had a lea ut. She didt hae a
aiatios. Also, thee is
no contraindication to vaccinations. Parents were worried about the vaccine side effects.
What
will you give?
a. Clean the wound and dress it
b. Give TT only
c. Give DPT only
d. Give DPT and tetanus Ig
e. Give complete DPT vaccine course
290. A 32yo female who has had 3 prv miscarriages in the 1st trimester now comes with
vaginal

bleeding at 8wks. US reveals a viable fetus. What would be the most appropriate
definitive
management?
a. Admit
b. Aspirin
c. Bed rest 2 weeks
d. Cervical cerclage
e. No tx
Key = B (Aspirin)
Explanation = This is a case of APS probably as evident by 3 prev miscarriages
and vaginal bleeding now. Aspirin should be advised throughout the pregnancy
(along with LMWH).
291. A 6yo girl started wetting herself up to 6x/day. What is the most appropriate tx?
a. Sleep alarms
b. Desmopressin
c. Reassure
d. Behavior training
e. Imipramine
Key = A (sleep alarms)
Explanation = Alarm training is a first line treatment for nocturnal enuresis in
children and is the most effective long term strategy. Desmopressin can be used
in children above the age of 7 but is indicated in case of rapid control or when
alarms are inappropriate and is usually used second line to alarm training.
Imipramine is another option approved for 6 year olds but is reserved for resistant
cases only because of its side effects. Behavior therapy is often considered
inferior to these other confirmed methods of treatment.
Reassurance is for children under the age of 5.
292. A 27yo 34wk pregnant lady presents with headache, epigastric pain and vomiting.
Exam:
pulse=115, BP=145/95mmHg, proteinuria ++. She complains of visual disturbance. What
is the
best medication for the tx of the BP?
a. 4g MgSO4 in 100ml 0.9%NS in 5mins
b. 2g MgSO4 IV bolus
c. 5mg hydralazine IV
d. Methyldopa 500mg/8h PO
e. No tx
Key = C
Explanation = This is a case of severe pre-eclampsia as evident by pregnancy of
more than 20 weeks, sustained bp of more than 140/90 mmhg and headache,
epigastric pain and visual disturbances. Treatment of choice for Bp control in
severe pre-eclampsia and eclampsia is IV hydralazine or labetalol or nifedipine
orally.
MgSO4 is for control of fits not bp.

Management of severe pre-eclampsia[1]

Delivery of the fetus and placenta is the only cure. However, preterm delivery may
adversely affect neonatal outcome, with complications resulting from prematurity and low
birth weight.
Blood pressure:
o Antihypertensive treatment should be started in women with a systolic
blood pressure over 160 mm Hg or a diastolic blood pressure over 110
mm Hg. In women with other markers of potentially severe disease,
treatment can be considered at lower degrees of hypertension.
o Labetalol (given orally or intravenously), oral nifedipine or intravenous
hydralazine are usually given for the acute management of severe
hypertension.
o Atenolol, angiotensin-converting enzyme (ACE) inhibitors, angiotensin-II
receptor antagonists and diuretics should be avoided.
o Antihypertensive medication should be continued after delivery, as
dictated by the blood pressure. It may be necessary to maintain treatment
for up to three months, although most women can have treatment
stopped before this.
Prevention of seizures:
Magnesium sulfate should be considered when there is concern about the risk of
eclampsia.
In women with less severe disease, the decision is less clear and will depend on
individual case assessment.
Control of seizures:
o Magnesium sulfate is the therapy of choice to control seizures. A loading
dose of 4 g is given by infusion pump over 5-10 minutes, followed by a
further infusion of 1 g/hour maintained for 24 hours after the last seizure.
o Recurrent seizures should be treated with either a further bolus of 2 g
magnesium sulfate or an increase in the infusion rate to 1.5 g or 2.0
g/hour.
Fluid balance:
o Fluid restriction is advisable to reduce the risk of fluid overload in the
intrapartum and postpartum periods. Total fluids should usually be limited
to 80 ml/hour or 1 ml/kg/hour.
Delivery:
o The decision to deliver should be made once the woman is stable and
with appropriate senior personnel present.
o If the fetus is less than 34 weeks of gestation and delivery can be
deferred, corticosteroids should be given, although after 24 hours the
benefits of conservative management should be reassessed.
o Conservative management at very early gestations may improve the
perinatal outcome but must be carefully balanced with maternal wellbeing.
o The mode of delivery should be determined after considering the
presentation of the fetus and the fetal condition, together with the
likelihood of success ofinduction of labour after assessment of the
cervix.[5]
o The third stage should be managed with 5 units of intramuscular/slow
intravenous Syntocinon. Ergometrine and Syntometrine should not be
given for prevention of haemorrhage, as this can further increase the
blood pressure.
o Prophylaxis against thromboembolism should be considered.

Management of eclampsia

Resuscitation:
o The patient should be placed in the left lateral position and the airway
secured.
o Oxygen should be administered.
Treatment and prophylaxis of seizures:
o Magnesium sulfate is the anticonvulsant drug of choice.
o Intubation may become necessary in women with repeated seizures in
order to protect the airway and ensure adequate oxygenation.
Treatment of hypertension:
o Reduction of severe hypertension (blood pressure >160/110 mm Hg or
mean arterial pressure >125 mm Hg) is essential to reduce the risk of
cerebrovascular accident. Treatment may also reduce the risk of further
seizures.
o Intravenous hydralazine or labetalol are the two most commonly used
drugs. Both may precipitate fetal distress and therefore continuous fetal
heart rate monitoring is necessary.
Fluid therapy:
o Close monitoring of fluid intake and urine output is mandatory.
o Pre-loading the circulation with 400-500 ml colloid prior to regional
anaesthesia or vasodilatation with hydralazine may reduce the risk of
hypotension and fetal distress.
Delivery:
o The definitive treatment of eclampsia is delivery. Attempts to prolong
pregnancy in order to improve fetal maturity are unlikely to be of value.
o However, it is unsafe to deliver the baby of an unstable mother even if
there is fetal distress. Once seizures are controlled, severe hypertension
treated and hypoxia corrected, delivery can be expedited.
o Vaginal delivery should be considered but Caesarean section is likely to
be required in primigravidae, well before term and with an unfavourable
cervix.
o After delivery, high-dependency care should be continued for a minimum
of 24 hours.
All patients need careful follow-up and a formal postnatal review to establish if there is
chronic hypertension, proteinuria or liver damage.

Complications

Eclampsia is usually part of a multisystem disorder. Associated complications


include haemolysis, HELLP syndrome (3%), disseminated intravascular
coagulation (3%), renal failure (4%) and adult respiratory distress syndrome
(3%).
Pre-eclampsia can progress to eclampsia with epileptic fits and sometimes other
neurological symptoms, including focal motor deficits and cortical blindness.
Cerebrovascular haemorrhage is a complicating factor in 1-2%.

293. A 24yo lady who is 37wk pregnant was brought to the ED. Her husband says a few
hours ago she
complained of headache, visual disturbance and abdominal pain. On arrival at the ED
she has a

fit. What is the next appropriate management for this pt?


a. 4g MgSO4 in 100ml 0.9%NS in 5mins
b. 2g MgSO4 IV bolus
c. 2g MgSO4 in 500ml NS in 1h
d. 4g MgSO4 IV bolus
e. 10mg diazepam in 500ml 0.9%NS in 1h
Key = A
Explained in the previous question.
. What is the pathologial hage i Baets esophagitis?
a. Squamous to columnar epithelium
b. Columnar to squamous epithelium
c. Dysplasia
d. Metaplasia
e. Hyperplasia
Key = A (squamous to columnar)
295. A 34yo male presents with hx of headache presents with ataxia, nystagmus and
vertigo. Where
is the site of the lesion?
a. Auditory canal
b. 8th CN
c. Cerebellum
d. Cerebral hemisphere
e. Brain stem
Key = Cerebellum
Points in favour = ataxia, nystagmus and vertigo
296. A 24yo girl comes to the woman sexual clinic and seeks advice for contraception.
She is on
sodium valproate.
a. She at use COCP
b. She can use COCP with extra precaution
c. She can use COCP if anticonvulsant is changed to carbamezapin.
d. She can use COCP with estrogen 50ug and progesterone higher dose
e. She can use COCP
Key = E
Points in favour = Women using anticonvulsants that do not induce live liver
enzyme cytochrome - P450 can use OCPs without any restriction.
Anticonvulsants not inducing liver enzymes = gabapentin, levetiracetam, valproate and
vigabatrin.
Anticonvulsants inducing liver enzymes = phenytoin, carbamazepine, barbiturates,
primidone, topiramate and oxcarbazepine. May use depot medroxyprogesterone acetate,
copper intrauterine contraceptive devices, the levonorgestrel-releasing intrauterine system,
barrier methods and natural family planning methods.
297. A 27yo lady came to the ED 10 days ago with fever, suprapubic tenderness and
vaginal discharge.
PID was dx. She has been on the antibiotics for the last 10days. She presents again with
lower
abdominal pain. Temp=39.5C. what is the most appropriate next management?
a. Vaginal swab
b. Endocervical swab

c. US
d. Abdominal XR
e. Laparoscopy
298. An 18yo man complains of fatigue and dyspnea, he has left parasternal heave and
systolic thrill
with a harsh pan-systolic murmur at left parasternal edge. What is the most probable dx?
a. TOF
b. ASD
c. VSD
d. PDA
e. TGA
Key = C (VSD)
Points in favour = Age , Left parasternal heave, pan systolic murmur at left
parasternal edge.
Transposition of great arteries presents in the infants and not that late in life.
TOF may be left undiagnosed this late but patients present with severe cyanosis and
other typical features of TOF.
ASD has a soft systolic ejection murmur in the pulmonic area and diastolic rumble at left
sternal border.
VSD :-

Epidemiology[3]

VSDs are the most common congenital heart defect in children, occurring in 50%
of all children with congenital heart disease and in 20% as an isolated lesion.
The incidence of VSDs has increased significantly with advances in imaging and
screening of infants and ranges from 1.56 to 53.2 per 1,000 live births. The ease
with which small muscular VSDs can now be detected has contributed to this
increase in incidence.
In the adult population VSDs are the most common congenital heart defect,
excluding bicuspid aortic valve.

Presentation
How haemodynamically significant a VSD is depends on its size, pressure in the
individual ventricles and pulmonary vascular resistance.[3] The presence of a VSD may
not be obvious at birth because of nearly equal pressures in both the ventricles with little
or no shunting of blood. As the pulmonary vascular resistance drops, the pressure
difference between the two ventricles increases and the shunt becomes significant
allowing the defect to become clinically apparent. An exception to this rule is Down's
syndrome where the pulmonary vascular resistance may not fall and the VSD may not
become clinically apparent, first presenting with pulmonary hypertension. All babies with
Down's syndrome should therefore be screened for congenital heart disease no later
than 6 weeks of age.[8]
The clinical presentation varies with the severity of the lesion:
With a small VSD, the infant or child is asymptomatic with normal feeding and
weight gain and the lesion may be detected when a murmur is heard at a routine
examination.

With a moderate-to-large VSD, although the babies are well at birth, symptoms
generally appear by 5 to 6 weeks of age. The main symptom is exercise
intolerance and since the only exercise babies do is feeding, the first impact is on
feeding. Feeding tends to slow down and is often associated with tachypnoea
and increased respiratory effort. Babies are able to feed less, and weight gain
and growth are soon affected. Poor weight gain is a good indicator of heart
failure in a baby. Recurrent respiratory infections may also occur.
With very large VSDs the features are similar but more severe. If appropriate
management is not carried out promptly in infants with large VSDs excessive
pulmonary blood flow may lead to increase in pulmonary vascular resistance and
pulmonary hypertension. These babies may develop a right to left shunt with
cyanosis or Eisenmenger's syndrome.

Physical signs
Again, these depend on the severity of the lesion with, one exception, the loudness of
the murmur. Murmurs are caused by turbulence of blood flow. There may be more
turbulence with a small hole than with a large defect. The loudness of the murmur gives
no indication of the size of the lesion. Even the adage 'the louder the sound, the smaller
the lesion' is untrue.
With a small VSD the infant is well developed and pink. The precordial impulse
may be greater than usual but is usually normal. If it can be heard, the
physiological splitting of the second sound is normal but there is a harsh systolic
murmur that is best heard at the left sternal edge, which may obliterate the
second sound. The murmur tends to be throughout systole but, if the defect is in
the muscular portion, it may be shorter as the hole is closed as the muscle
contracts.
With a moderate or large VSD there is enhanced apical pulsation as well as a
parasternal heave. A grade 2 to 5/6 systolic murmur is audible at the lower left
sternal border. It may be pansystolic or early systolic. A prominent third sound
with a short early mid-diastolic rumble is audible at the apex with a moderate-tolarge shunt (because of increased flow through the mitral valve during diastole).
S2 is loud and single in patients with pulmonary hypertension.
Large defects with no shunts or those with Eisenmenger physiology and right-toleft shunt may have no murmur.
Investigations = ECG, CXR, ECHOCARDIOGRAPHY and Cardiac catheterization.

Management[10]
Medical management

Management in the infant and child depends on symptoms, with small


asymptomatic defects needing no medical management, and unlikely to need
any intervention.
First-line treatment for moderate or large defects affecting feeding and growth is
with diuretics for heart failure and high-energy feeds to improve calorie intake.
Angiotensin-converting enzyme inhibitors are used to reduce afterload which
promotes direct systemic flow from the left ventricle, thus reducing the shunt.
Digoxin can also be given for its inotropic effect.

Any patient needing significant medical management should be referred for


surgical assessment.

Surgical management

Surgical repair is required if there is uncontrolled heart failure, including poor


growth. Even very small babies may be considered for surgery.
Infundibular defects may be considered for closure even if they are asymptomatic
because of their location.
Development of aortic valve prolapse and aortic regurgitation in perimembranous
VSDs may be an indication for surgical closure.
Most defects are closed nowdays by directly placing a patch from the right
ventricular side, usually with the surgeon working through the tricuspid valve.
Patients with large muscular VSDs which are difficult to see or those with multiple
holes (Swiss cheese septum) presenting as neonates or infants need initial
palliation in the form of pulmonary artery banding followed many months later by
corrective surgery and removal of the pulmonary artery band.

Catheter closure

Advances in catheter techniques and devices mean that many muscular and
perimembranous VSDs can now be closed percutaneously. This is in the setting
of normal atrioventricular and ventriculoarterial connections and absence of any
atrioventricular or arterial valve override.
Transcatheter techniques are useful because they avoid cardiopulmonary
bypass. There are, however, recognised complications for device closure of
perimembranous VSDs, including complete heart block needing permanent
pacemaker.[11]
The National Institute for Health and Care Excellence (NICE) has provided
detailed guidance on indications, efficacy and complications of the procedure.[12]
It is safer to close muscular VSDs using a device but muscular VSDs which are
haemodynamically significant are likely to be seen in only young infants, making
catheterisation difficult and challenging. Hybrid procedures increasingly being
used involve insertion of the device in the operation theatre after surgical
exposure of the defect.[13]

299. A young girl presenting with fever, headache, vomiting, neck stiffness and
photophobia. She has
no rashes. What is the most appropriate test to confirm dx?
a. Blood culture
b. Blood glucose
c. LP
d. CXR
e. CT
Key = C
Points in favour = This is suspected meningitis. Do Lp to confirm.

300. A 65yo HTN man wakes up in the morning with slurred speech, weakness of the
left half of his
body and drooling. Which part of the brain is affected?
a. Left parietal lobe
b. Right internal capsule
c. Right midbrain
d. Left frontal lobe
Key = B

351. A 35yo lady presents with painful ulcers on her vulva, what is the appropriate inv
which will lead to the dx?
a. Anti-HSV antibodies
b. Dark ground microscopy of the ulcer
c. Treponema palladium antibody test
d. Rapid plasma regain test
e. VDRL
Dx genital herpes
Ans. key A. Anti-HSV antibodies. [Genital Herpes may be asymptomatic or may remain
dormant for months or even years. When symptoms occur soon after a person is
infected, they tend to be severe. They may start as multiple small blisters that eventually
break open and produce raw, painful sores that scab and heal over within a few weeks.
The blisters and sores may be accompanied by flu-like symptoms with fever and swollen
lymph nodes.
treatment : There are three major drugs commonly used to treat genital herpes
symptoms: acyclovir (Zovirax), famciclovir (Famvir), and valacyclovir(Valtrex). These are
all taken in PO. Severe cases may be treated with the intravenous (IV) drug acyclovir].
options B C D & E are tests for syphilis which presents with single painless ulcer
(canchre)
352. A 53yo man presents with a longstanding hx of a 1cm lesion on his arm. It has
started
bleeding on touch. What is the most likely dx?
a. Basal cell carcinoma
b. Kaposis sarcoma
c. Malignant melanoma
d. Squamous cell carcinoma
e. Kerathoacanthoma
Ans. D Squamous cell carcinoma. [SSCs Arises in squamous cells. SCCs may occur on all
areas of the body including the mucous membranes and genitals, but are most common in areas frequently
exposed to the sun, such as the rim of the ear, lower lip, face, balding scalp, neck, hands, arms and legs.

SCCs often look like scaly red patches, open sores, elevated growths with a central
depression, or warts; they may crust or bleed.
investigation: tissue sample (biopsy) will be examined under a microscope to arrive at
a diagnosis.

prognosis : Squamous cell carcinomas detected at an early stage and removed


promptly are almost always curable and cause minimal damage].
basal cell carcinoma is usually on face with inverted margins
malignant melanoma is on sun exposed parts and is dark (black colored) ulcer
353. A

47yo man with hx of IHD complains of chest pain with SOB on exertion over the
past few days. ECG normal, Echo= increased EF and decreased septal wall thickness.
What is the most likely dx?
a. Dilated CM
b. Constrictive pericarditis
c. Amyloidosis
d. Subacute endocarditis
Ans. The key is A. Dilated CM.
points in fav: sob, palpitation, dec septal wall thinning
treatment : beta blocker, acei, diuretics
Constrictive pericarditis doesnt fits because it starts with urti has pain on lying flat which
is relieved by leaning forward
ecg shows wide spread st elevation
Amyloid deposition in the heart can cause both diastolic and systolic heart failure.
EKG changes may be present, showing low voltage and conduction abnormalities
like atrioventricular block or sinus node dysfunction. On echocardiography the
heart shows restrictive filling pattern, with normal to mildly reduced ejec fraction
354. An elderly pt who is known to have DM presents to the hospital with drowsiness,
tremors and confusion. What inv should be done to help in further management?
a. Blood sugar
b. ECG
c. Standing and lying BP
d. Fasting blood sugar
e. CT
Ans. The key is A. Blood sugar.since he is known diabetic he may have gotten
hypoglycemic d/t his meds
355. A 28yo pregnant woman with polyhydramnios and SOB comes for an anomaly scan
at 31 wks. US= absence of gastric bubble. What is the most likely dx?
a. Duodenal atresia
b. Esophageal atresia
c. Gastrochiasis
d. Exomphalos
e. Diaphragmatic hernia
Ans. The key is B. Oesophageal atresia.
This condition is visible, after about 26 weeks, on an ultrasound. On antenatal
USG, the finding of an absent or small stomach in the setting of polyhydramnios
used to be considered suspicious of esophageal atresia. However, these findings
have a low positive predictive value. The upper neck pouch sign is another sign

that helps in the antenatal diagnosis of esophageal atresia and it may be detected
soon after birth as the affected infant will be unable to swallow its own saliva.
Also, the newborn can present with gastric distention, cough, apnea, tachypnea,
and cyanosis. In many types of esophageal atresia, a feeding tube will not pass
through the esophagus.
356. A 1m boy has been brought to the ED, conscious but with cool peripheries and has
HR=222bpm. He has been irritable and feeding poorly for 24h. CXR=borderline enlarged
heart with clear lung fields. ECG=regular narrow complex tachycardia, with difficulty
identifying p wave. What is the single most appropriate immediate tx?
a. Administer fluid bolus
b. Administer oxygen
c. Oral beta-blockers
d. Synchronized DC cardio-version
e. Unilateral carotid sinus massage

The key is D. Synchrnized DC cardioversion.


reason: As the patient is in probable hemodynamic instability (suggested by cool
peripheries) so we should go for DC cardioversion.
diagnosis SVT.
357. A 7yo child presented with chronic cough and is also found to be jaundiced on
examination.
What is the most likely dx?
a. Congenital diaphragmatic hernia
b. Congenital cystic adenematoid malformation
c. Bronchiolitis
d. RDS
e. Alpha 1 antitrypsin deficiency

The key is E. Alpha 1 antitrypsin deficiency.


REASON. Unexplained liver disease with respiratory symptoms are very suggestive of
AATD.
liver disease occurs because of the accumulation AAT in it
where as d/t inability to be transported out of liver AATD causes emphysema hence the
resp problems
358. A 35yo construction worker is dx with indirect inguinal hernia. Which statement
below best
describes it?
a. Passes through the superficial inguinal ring only
b. Lies above and lateral to the pubic tubercle
c. Does not pass through the superficial inguinal ring
d. Passes through the deep inguinal ring
Ans. The key is D. Passess through the deep inguinal ring.
direct hernia passes directly through the posterior wall of inguinal canal whereas indirect
can only do so via deep ring

359. A woman has numerous painful ulcers on her vulva. What is the cause?
a. Chlamydia
b. Trichomonas
c. Gardenella
d. HSV
e. EBV
Ans. The key is D. HSV. reason has been explained in q 351
360. A 72 yo man has been on warfarin for 2yrs because of past TIA and stroke. What is
the most important complication that we should be careful with?
a. Headache
b. Osteoporosis
c. Ear infection
d. Limb ischemia
e. Diarrhea
Ans. key is wrong
right key is A Headache, as there are chances of SAH or generally ICH
361. A 55yo man has been admitted for elective herniorraphy. Which among the
following can be the reason to delay his surgery?
a. Controlled asthma
b. Controlled atrial fib
c. DVT 2yrs ago
d. Diastolic BP 90mmHg
e. MI 2 months ago
Ans. E SAFER TO DO SURGERY AFTER 6 MONTHS
362. A 65yo known case of liver ca and metastasis presents with gastric reflux and
bloatedness. On bone exam there is osteoporosis. He also has basal consolidation in
the left lung. What is the next appropriate step?
a. PPI IV
b. Alendronate
c. IV antibiotics
d. Analgesic
e. PPI PO
IN THIS case reflux is the cause of recurrent pneumonia so both C AND E can be
right but to chose single one E is more appropriate
363. A 66yo man has the following ECG. What is the most appropriate next step in
management?
a. Metoprolol
b. Digoxin
c. Carotid sinus massage
d. Adenosine
e. Amiodarone.
Ans. A beta blocker for A FIB

364. A 22yo sexually active male came with 2d hx of fever with pain in scrotal area.
Exam: scrotal skin is red and tender. What is the most appropriate dx?
a. Torsion of testis
b. Orchitis
c. Inguinal hernia
d. Epididymo-orchitis
D Epididymo-orchitis.
In orchitis there should be fever, elevation of testes reduces pain (positive prehn sign), In
torsion testis lies at a higher level. In torsion urinalysis negative but in orchitis it is
positive. Orchitis usually occurs in sexually active man. X
365. A man on warfarin posted for hemicolectomy. As the pt is about to undergo surgery.
What
option is the best for him?
a. Continue with warfarin
b. Continue with warfarin and add heparin
c. Stop warfarin and add aspirin
d. Stop warfarin and add heparin
e. Stop warfarin
D Stop warfarin and add heparin
5 DAYS BEFORE SURGERY WARFARIN MUST BE REPLACED BY HEPARIN,
366. A 65yo known alcoholic is brought into hospital with confusion, aggressiveness and
ophthalmoplegia. He is treated with diazepoxide. What other drug would you like to
prescribe?
a. Antibiotics
b. Glucose
c. IV fluids
d. Disulfiram
e. Vit B complex
E Vitamin B complex. [confusion and ophthalmoplegia points towards the diagnosis of
Wernickes encephalopathy].
which occurs d/t thiamine def.
367. A 32yo woman has severe right sided abdominal pain radiating into the groin which
has lasted for 3h. She is writhering in pain. She has no abdominal signs. What is the
most likely cause of her abdominal pain?
a. Appendicitis
b. Ruptured ectopic pregnancy
c. Salpingitis
d. Ureteric colic
e. Strangulated hernia
D Ureteric colic.
It indicate stone at lower ureter. [i) Pain from upper ureteral stones tends to radiate to the flank and
lumbar areas. ii) Midureteral calculi cause pain that radiates anteriorly and caudally. This midureteral pain in

particular can easily mimic appendicitis on the right or acute diverticulitis on the left. iii) Distal ureteral stones
cause pain that tends to radiate into the groin or testicle in the male or labia majora in the female.

368. A 39yo coal miner who smokes, drinks and has a fam hx of bladder cancer is
suffering from BPH. The most important risk factor for his bladder carcinoma is?
a. Fam hx
b. Smoking
c. Exposure to coal mine
d. BPH

B smoking.
. Risk factors of bladder cancer:
i) Smoking ii) Exposure to chemicals used in dye industry iii) Whites are more likely to
develop bladder cancer iv) Risk increases with age v) More common in men vi) Chronic
bladder irritation and infections (urinary infections, kidney and bladder stones, bladder
catheter left in place a long time.) vii) Personal history of bladder or other urothelial
cancer viii) Family history ix) Chemotherapy or radiotherapy x) Pioglitazone for more
than one year and certain herb xi) Arsenic in drinking water xii) Low fluid consumption.
369. A 34yo woman is referred to the endocrine clinic with a hx of thyrotoxicosis. At her
1st
appointment she is found to have a smooth goiter, lid lag and bilateral exophthalmos
with puffy
eyelids and conjunctival injection. She wants to discuss the tx of her thyroid prb as she is
keen to become pregnant. What is the most likely tx you would advise?
a. 18m of carbimazole alone
b. 18m of PTU alone
c. A combo od anti-thyroid drug and thyroxine
d. Radioactive iodine
e. Thyroidectomy

B 18m of PTU alone.


Other drug option i.e Carbamazepine is teratogenic [can cause i) spina bifida
ii)cardiovascular malformations ETC . PTU is on the other hand relatively safe in
pregnancy.
370. A child living with this stepfather is brought by the mother with multiple bruises,
fever and
fractures. What do you suspect?
a. NAI
b. Malnutrition
c. Thrombocytopenia
d. HIV
Ans. The key is A. NAI. [H/O living with stepfather, multiple bruises, fever and fractures
are suggestive of NAI]. OTHER possible points can include hx not matching with
bruises, wounds which are a day or two older at the time of presentation.

371. A young man who was held by the police was punched while in custody. He is now
cyanosed and unresponsive. What is the 1st thing you would do?
a. IV fluids
b. Clear airway
c. Turn pt and put in recovery position
d. Give 100% oxygen
e. Intubate and ventilate
B. Clear airway. [ABC protocol].
372. A HTN male loses vision in his left eye. The eye shows hand movement and a light
shined in the eye is seen as a faint light. Fundus exam: flame shaped hemorrhages. The
right eye is normal. What is the cause of this pts unilateral blindness?
a. HTN retinopathy
b. CRA thrombosis
c. CRV thrombosis
d. Background retinopathy
e. Retinal detachment
key is wrong.
right ans is C ( unilateral blindness with flame shaped hemorrhages are
characteristic of CRVO). Flame shaped hemorrhages are seen in HTN and diabetic
retinopathy too but they will cause bilateral damage.
373. A mentally retarded child puts a green pea in his ear while eating. The carer
confirms this.
Otoscopy shows a green colored object in the ear canal. What is the most appropriate
single
best approach to remove this object?
a. By magnet
b. Syringing
c. Under GA
d. By hook
e. By instilling olive oil
C UNDER GA
[Pea is not a magnetic material and hence it cannot be removed by magnet, it will
swell up if syringing is attempted, as hook placement is likely with risk of pushing
the pea deeper it is not also suitable in a mentally retarded child, and olive oil is
not of help in case of pea. So to avoid injury it is better to remove under GA
374. A pt presents with longstanding gastric reflux, dysphagia and chest pain. On barium
enema, dilation of esophagus with tapering end is noted. He was found with Barretts
esophagus. He had progressive dysphagia to solids and then liquids. What is the single
most appropriate dx?
a. Achalasia
b. Esophageal spasm
c. GERD
d. Barretts esophagus
e. Esophageal carcinoma

E Oesophageal carcinoma. [there is dilatation in oesophagus which is seen both in


achalasia and carcinoma. Dysphagia to solid initaially is very much suggestive of
carcinoma and also barretts change is a clue to carcinoma]
Progressive dysphagia with h/o barrett esophagus are the key indicators
375. A 48yo lady presents with itching, excoriations, redness, bloody discharge and
ulceration around her nipple. What is the most likely dx?
a. Pagets disease of the breast
b. Fibrocystic dysplasia
c. Breast abscess
d. Duct papilloma
e. Eczema
A Pagets disease of the breast.
TYPICAL manifestation of pagets disease
eczema like rash involving nipple and areola with straw or bloody discharge. Eczema is
bilateral.
Also nipple turns inward in advances stages pt complaints of burning sensation at the
site of lesion
DX mammography and biopsy
TX surgery + chemo or radio may be needed
376. Pt with widespread ovarian carcinoma has bowel obstruction and severe colic for
2h and was normal in between severe pain for a few hours. What is the most appropriate
management?
a. PCA (morphine)
b. Spasmolytics
c. Palliative colostomy
d. Oral morphine
e. Laxatives
C. Palliative colostomy.
Cancer or chemotherapy induced obstructions are unlikely to respond to conservative
management [NBM, IV fluid, nasogastric suction] and hence only analgesia will not
relieve it. So in such cases we have to go for palliative colostomy.
377. A 70yo man admits to asbestos exposure 20yrs ago and has attempted to quit
smoking. He has noted weight loss and hoarseness of voice. Choose the single most
likely type of cancer a.w risk
factors present.
a. Basal cell carcinoma
b. Bronchial carcinoma
c. Esophageal carcinoma
d. Nasopharyngeal carcinoma
e. Oral carcinoma

B. Bronchial carcinoma. [Asbestos exposure is a risk factor for lung cancer


and also has a synergistic effect with cigarette smoke].

#. Conditions related to asbestos exposure: i) Pleural plaques (after a latent period of


20-40 yrs) ii) Pleural thickening iii) Asbestosis (latent period is typically 15-30 yrs) iv)
Mesothelioma (prognosis is very poor) v) Lung cancer.
378. A 32yo woman had progressive decrease in vision over 3yrs. She is no dx as
almost blind. What would be the mechanism?
a. Cataract
b. Glaucoma
c. Retinopathy
d. Uveitis
e. Keratitis

B. Glaucoma.
. Cataract is unlikely at this age. Nothing in the history suggests retinopathy. Uveitis and
iritis doesnt have such degree of vision loss and iritis and anterior uveitis have pain,
redness and photophobia. Open angle glaucoma is likely cause.
379. A child during operation and immediately after showed glycosuria, but later his
urine sugar was normal. Choose the most probable dx.
a. Pre-diabetic state
b. Normal finding
c. Low renal tubular threshold
d. DM
B Normal finding.
Stress during operation can cause transient hyperglycemia causing glycosuria
secondary to stress induced rise of cortisol which becomes normal after some time.
380. A pt presented with hx of swelling in the region of the sub-mandibular region, which
became
more prominent and painful on chewing. He also gave hx of sour taste in the mouth, the
area is
tender on palpation. Choose the most probable dx?
a. Chronic recurrent sialadenitis
b. Adenolymphoma
c. Mikuliczs disease
d. Adenoid cystic carcinoma
e. Sub-mandibular abscess
A Chronic recurrent sialadenitis. [pain, swelling, more pain on chewing, tenderness, and
submandibular region suggests diagnosis of submandibular chronic recurrent
sialadenitis, usually secondary to sialolithiasis or stricture].
381. ECG of an 80yo pt of ICH shows saw-tooth like waves, QRS complex of 80ms
duration,
ventricular rate=150/min and regular R-R interval. What is the most porbable dx?
a. Atrial fib
b. Atrial flutter
c. SVT
d. Mobitz type1 second degree heart block
e. Sinus tachycardia

B Atrial flutter. [Saw-tooth like waves, normal QRS comples of 80 ms (normal range 70100 ms), ventricular rate of 150/min and regular R-R interval is diagnostic of atrial
flutter].
FOR AFIB THERE WD BE IRREGULARARLY IRREGULAR RHYTHM
382. A 50 yo woman who was treated for breast cancer 3 yrs ago now presents with
increase thirst and confusion. She has become drowsy now. What is the most likely
metabolic abnormality?
a. Hypercalcemia
b. Hyperkalemia
c. Hypoglycemia
d. Hyperglycemia
e. Hypercalcemia.
E HYPERCALCEMIA
Ans. 2. Increased thirst, confusion, drowsiness these are features of hypercalcemia. Any
solid organ tumour can produce hypercalcemia. Here treated Ca breast is the probable
cause of hypercalcemia.
383. A 29yo woman presents to her GP with a hx of weight loss, heat intolerance, poor
conc and
palpitations. Which of the following is most likely to be a/w dx of thyroiditis a/w viral
infection?
a. Bilateral exophthalmos
b. Diffuse, smooth goiter
c. Reduced uptake on thyroid isotope scan
d. Positive thyroid peroxidase antibodies
e. Pretibial myxedema
C. Reduced uptake on thyroid isotope scan.
DX De Quervains or subacute thyroiditis.
. Viral or subacute thyroiditis: diagnostic criteria: i) Features of hyperthyroidism present.
ii) Pain thyroid, not mentioned. iii) Investigations: high esr (60-100) not mentioned,
Reduced uptake of radioactive iodine by the gland.
384. A lady, post-colostomy closure after 4 days comes with fluctuating small swelling in
the stoma.
What is the management option for her?
a. Local exploration
b. Exploratory laparotomy
c. Open laparotomy
d. Reassure
A Local exploration.
THERE MUST BE SOME LOCAL WOUND PROBLEM
385. A 65yo female pt was given tamoxifen, which of the following side effect caused by
it will
concern you?
a. Fluid retention
b. Vaginal bleeding
c. Loss of apetite
d. Headache and dizziness
e.
B Vaginal bleeding.

. Tamoxifen can promote development of endometrial carcinoma. So vaginal bleeding


will be of concern for us.
386. A 39yo man with acute renal failure presents with palpitations. His ECG shows tall
tented T
waves and wide QRS complex. What is the next best step?
a. Dialysis
b. IV calcium chloride
c. IV insulin w/ dextrose
d. Calcium resonium
e. Nebulized salbutamol

B. IV calcium chloride (both IV calcium gluconate or IV calcium chloride can be used)


when there is ECG changes.
DX The ECG changes are suggestive of Hyperkalemia.
At potassium level of >5.5mEq/L occurs tall tented T waves and at potassium level
>7mEq/L occurs wide QRS complex with bizarre QRS morphology.
387. A 54yo pt 7 days after a total hip replacement presents with acute onset
breathlessness and
raised JVP. Which of the following inv will be most helpful in leading to a dx?
a. CXR
b. CTPA
c. V/Q scan
d. D-Dimer
e. Doppler US of legs

The key is B. CTPA.


The patient has a +ve two level PE Wells score (if it was negative we should do DDimer) and there is no renal impairment or history suggestive of allergy to contrast
media (if these present we should have go for VQ scan) the investigation of choice is
PTCA. NICE guideline.
388. A 7yo girl has been treated with penicillin after sore throat, fever and cough. Then
she
develops skin rash and itching. What is the most probable dx?
a. Erythema nodosum
b. Erythema multiforme
c. SJS
d. Erythema marginatum
e. Erythema gangernosum

. The key is B. Erythema multiforme.


Common drugs causing erythma multiforme are: antibiotics (including, sulphonamides,
penicillin), anticonvulsants (phenytoin,barbiturates), aspirin, antituberculoids, and allopurinol.
CLOSELY related option is SJS which would have muco cutaneous rash but in here we hav only
cutaneous rash.

Nodosum is in diseases like Tb, sarcoidosis, IBD


Marginatum is the rash of acute rheumatic fever

389. A 60yo man presented with a lump in the left supraclavicular region. His appetite is
decreased and he has lost 5kg recently. What is the most probably dx?
a. Thyroid carcinoma
b. Stomach carcinoma
c. Bronchial carcinoma
d. Mesothelioma
e. Laryngeal carcinoma
The key is B. Stomach carcinoma. [Mentioned lump in the left supraclavicular region is
Vershows gland, has long been regarded as strongly indicative of the presence of cancer in the
abdomen, specifically gastric cancer].
390. A 64yo man has presented to the ED with a stroke. CT shows no hemorrhage.
ECG shows atrial fib. He has been thrombolysed and hes awaiting discharge. What
prophylactic regimen is best for him?
a. Warfarin
b. Heparin
c. Aspirin
d. Statins
e. Beta blockers
The key is A. Warfarin. [Atrial fibrillation: post stroke- following a stroke or TIA warfarin
should be given as the anticoagulant of choice. NICE guideline].
391. A 54yo man after a CVA presents with ataxia, intention tremors and slurred speech.
Which part of the brain has been affected by the stroke?
a. Inner ear
b. Brain stem
c. Diencephalon
d. Cerebrum
e. Cerebellum
The key is E. Cerebellum.
i) Ataxia ii) slurred speech or dysarthria iii) dysdiadochokinesia iv) intention tremor v)
nystagmus. are the signs of cerebellar defect
392. A 57yo man with blood group A complains of symptoms of vomiting, tiredness,
weight loss and palpitations. Exam: hepatomegaly, ascites, palpable left supraclavicular
mass. What is the most likely dx?
a. Gastric carcinoma
b. Colorectal carcinoma
c. Peptic ulcer disease
d. Atrophic gastritis
e. Krukenberg tumor
Ans. The key is A. Gastric carcinoma. [i) blood group A is associated with gastric cancer
ii) vomiting, tiredness, weight loss are general features of gastric cancer iii) palpitation
from anemia of cancer iv) hepatomegaly and ascites are late features of gastric cancer.

v) palpable left supraclavicular mass- is Vershows gland, has long been regarded as
strongly indicative of gastric cancer].
393. A 21yo girl looking unkempt, agitated, malnourished and nervous came to the
hospital asking
for painkillers for her abdominal pain. She is sweating, shivering and complains of joint
pain.
What can be the substance misuse here?
a. Alcohol
b. Heroin
c. Cocaine
d. LSD
e. Ecstasy
The key is B. Heroin. [agitation, nervousness, abdominal cramp, sweating, shivering and
piloerection, arthralgia these are features of heroin withdrawal].
Also asking for painkillers. Probably looking for morphine.

394. A child presents with increasing jaundice and pale stools. Choose the most
appropriate test?
a. US abdomen
b. Sweat test
c. TFT
d. LFT
e. Endomyseal antibodies
The key is A. US abdomen. [This is a picture suggestive of obstructive jaundice. LFT can
give clue like much raised bilirubin, AST and ALT not that high and raised alkaline
phosphatase but still USG is diagnostic in case of obstructive jaundice].
395. A 32yo man presents with hearing loss. AC>BC in the right ear after Rinne test. He
also
complains of tinnitus, vertigo and numbness on same half of his face. What is the most
appropriate inv for his condition?
a. Audiometry
b. CT
c. MRI
d. Tympanometry
e. Webers test
The key is C. MRI. [features are suggestive of acoustic neuroma, so MRI is the preferred
option]. it involves basically 8th nerve but 6 7 9 and 10th nerves are also involved with it
396. A 56 yo lady with lung cancer presents with urinary retention, postural hypotension,
diminished reflexes and sluggish pupillary reaction. What is the most likely explanation
for her symptoms?
a. Paraneoplastic syndrome
b. Progression of lung cancer
c. Brain metastasis
d. Hyponatremia
e. Spinal cord compression
The key is A. Paraneoplastic syndrome.

s/s are of autonomic neuropathy which occurs in paraneoplastic syndrome


397. An old woman having decreased vision cant see properly at night. She has
changed her glasses quite a few times but to no effect. She has normal pupil and
cornea. What is the most likely dx?
a. Cataract
b. Glaucoma
c. Retinal detachment
d. Iritis
e. GCA
key is wrong
correct key is A cataract
old age and progressive weakness supports Cataract
Not glaucoma...as pupil would be mid dilated and sluggish reaction and in acute attack corneal
edema
Not RD...as pupil would be yellowish in color and there would be RAPD in massive RD and vision
would be dropped in day and night
Not iritis..as pupil would be constricted and cornea would have precipitation on its back (keratic
precipitate)
Not GCA(giant cell arteritis) as vision on it is suddenly dropped to HM up to LP and vision dropped
day and night

398. A pt comes with sudden loss of vision. On fundoscopy the optic disc is normal.
What is the
underlying pathology?
a. Iritis
b. Glaucoma
c. Vitreous chamber
d. Retinal detachment

Ans. 1. The Key is D. Retinal detachment.


#Causes of sudden painless loss of vision:
1.
2.
3.
4.
5.
6.

Retinal detachment
Vitreous haemorrhage
Retinal vein occlusion
Retinal artery occlusion
Optic neuritis
Cerebrovascular accident

remember retinal detachment has vision loss as if curtain is coming down


399. A child was woken up from sleep with severe pain in the testis. Exam: tenderness
on palpation and only one testis was normal in size and position. What would be your
next step?
a. Analgesia
b. Antibiotics
c. Refer urgently to a surgeon
d. Reassurance
e. Discharge with analgesics

Ans. The key is A. Analgesia. [According to some US sites it is analgesia but no UK site
support this!!! So for Plab exam the more acceptable option is C. Refer urgently to a
surgeon].
IN TORSION THE SOONER THE SURGICAL INTERVENTION DONE, THE BETTER
THE RESULTS ARE
400. A child suffering from asthma presents with Temp 39C, drooling saliva on to the
mothers lap,
and taking oxygen by mask. What sign will indicate that he is deteriorating?
a. Intercostal recession
b. Diffuse wheeze
c. Drowsiness
The key is A. Intercostal recession. [ here intercostals recession and drowsiness both
answers are correct. Hope in exam there will be one correct option]. but to chose
among them, better go with A
401. A 12yo boy presents with painful swollen knew after a sudden fall. Which bursa is
most likely tobe affected?
a. Semimembranous bursa
b. Prepatellar bursa
c. Pretibial bursa
d. Suprapatetaller bursa
. The key is B. Prepatellar bursa. [A fall onto the knee can damage the prepatellar bursa. This
usually causes bleeding into the bursa sac causing swollen painful knee. Prepatellar bursitis that is caused
by an injury will usually go away on its own. The body will absorb the blood in the bursa over several weeks,
and the bursa should return to normal. If swelling in the bursa is causing a slow recovery, a needle may be
inserted to drain the blood and speed up the process. There is a slight risk of infection in putting a needle
into the bursa].
402. A

61yo man has been referred to the OPD with frequent episodes of breathlessness
and chest pain a/w palpitations. He has a regular pulse rate=60bpm. ECG=sinus rhythm.
What is the
most appropriate inv to be done?
a. Cardiac enzymes
b. CXR
c. ECG
d. Echo
e. 24h ECG
The key is E. 24h ECG.
Indications of 24 h ambulatory holter monitoring:

To evaluate chest pain not reproduced with exercise testing

To evaluate other signs and symptoms that may be heart-related, such as fatigue,
shortness of breath, dizziness, or fainting

To identify arrhythmias or palpitations

To assess risk for future heart-related events in certain conditions, such as idiopathic
hypertrophic cardiomyopathy, post-heart attack with weakness of the left side of the
heart, or Wolff-Parkinson-White syndrome

To assess the function of an implanted pacemaker

To determine the effectiveness of therapy for complex arrhythmias

403. A woman dx with Ca Breast presents now with urinary freq. which part of the brain
is the
metastasis spread to?
a. Brain stem
b. Pons
c. Medulla
d. Diencephalon
e. Cerebral cortex
The key is D. Diencephalon. [diencephalon is made up of four distinct components: i) the
thalamus ii) the subthalamus iii) the hypothalamus and iv) the epithalamus. Among these
the hypothalamus has crucial role in causing urinary frequency].
404. A man is very depressed and miserable after his wifes death. He sees no point in
living now that his wife is not around and apologises for his existence. He refuses any
help offered. His son has brought him to the ED. The son can.t deal with the father any
more. What is the most
appropriate next step?
a. Voluntary admission to psychiatry ward
b. Compulsory admission under MHA
c. Refer to social services
d. Alternate housing
e. ECT
Ans. The key is B. Compulsory admission under MHA. [This patient is refusing any help
offered! And his son cannot deal with him anymore! In this situation voluntary admission
to psychiatry ward is not possible and the option of choice is compulsory admission
under MHA].
405. A 31yo man has epistaxis 10 days following polypectomy. What is the most likely
dx?
a. Nasal infection
b. Coagulation disorder
c. Carcinoma
The key is A. Nasal infection.
HEMORRHAGE AFTER 7 TO 14 DAYS IS SECONDARY HEMORRHAGE [Infection is
one of the most important cause of secondary hemorrhage].
406. A woman had an MI. She was breathless and is put on oxygen mask and GTN, her
chest
pain has improved. Her HR=40bpm. ECG shows ST elevation in leads I, II, III. What is
your next step?

a. LMWH
b. Streptokinase
c. Angiography
d. Continue current management
e. None

Ans. The key is B. Streptokinase


algorithm for st elevation MI
angioplasty/thrombolysis
b blocker
acei
clopidogrel
407. A 67yo male presents with polyuria and nocturia. His BMI=33, urine culture =
negative for
nitrates. What is the next dx inv?
a. PSA
b. Urea, creat and electrolytes
c. MSU culture and sensitivity
d. Acid fast urine test
e. Blood sugar
The key is E. Blood sugar. [Age at presentation and class1 obesity favours the diagnosis
of type2 DM].
since culture is -ve for nitrates, so uti is ruled out
408. A pt from Africa comes with nodular patch on the shin which is reddish brown. What
is the
most probable dx?
a. Lupus vulgaris
b. Erythema nodosum
c. Pyoderma gangrenosum
d. Erythema marginatum
e. Solar keratosis
The key is B. Erythema nodosum. [Causes of erythema nodosum: MOST COMMON
CAUSES- i) streptococcal infection ii) sarcoidosis. Other causes- tuberculosis,
mycoplasma pneumonia, infectious mononucleosis, drugs- sulfa related drug, OCP,
oestrogen; Behcets disease, CD, UC; lymphoma, leukemia and some others].
#Nodes are mostly on anterior aspect of shin
409. A 29yo lady came to the ED with complaints of palpitations that have been there for
the past 4 days and also feeling warmer than usual. Exam: HR=154bpm, irregular
rhythm. What is the tx for her condition?
a. Amiadarone
b. Beta blockers
c. Adenosine
d. Verapamil
e. Flecainide

The key is B. Beta blockers [the probable arrhythymia is AF secondary to


thyrotoxicosis(heat intolerance). So to rapid control the symptoms of thyrotoxicosis Beta
blocker should be used].
410. A T2DM is undergoing a gastric surgery. What is the most appropriate pre-op
management?
a. Start him in IV insulin and glucose and K+ just before surgery
b. Stop his oral hypoglycemic on the day of the procesure
c. Continue regular oral hypoglycemic
d. Stop oral hypoglycemic the prv night and start IV insulin with glucose and K+ before
surgery
e. Change to short acting oral hypoglycemic
The key is D. Stop oral hypoglycemic the prv night and start IV insulin with glucose and
K+ before surgery.
411. A 19yo boy is brought by his mother with complaint of lack of interest and no social
interactions. He has no friends, he doesnt talk much, his only interest is in collecting
cars/vehicles having around 2000 toy cars. What is the most appropriate dx?
a. Borderline personality disorder
b. Depression
c. Schizoaffective disorder
d. Autistic spectrum disorder
The key is D. Autistic spectrum disorder.
Autism spectrum disorders affect three different areas of a child's life:
Social interaction
Communication -- both verbal and nonverbal
Behaviors and interests
In some children, a loss of language is the major impairment. In others, unusual
behaviors (like spending hours lining up toys) seem to be the dominant factors.
412. A 45yo man who is diabetic and HTN but poorly compliant has chronic SOB,
develops severe SOB and chest pain. Pain is sharp, increased by breathing and
relieved by sitting forward. What is the single most appropriate dx?
a. MI
b. Pericarditis
c. Lung cancer
d. Good pastures syndrome
e. Progressive massive fibrosis
The key is B. Pericarditis. [Nature of pain i.e. sharp pain increased by breathing and
relieved by sitting forward is suggestive of pericarditis].
Nature of pericardial pain: the most common symptom is sharp, stabbing chest pain
behind the sternum or in the left side of your chest. However, some people with acute
pericarditis describe their chest pain as dull, achy or pressure-like instead, and of
varying intensity.

The pain of acute pericarditis may radiate to your left shoulder and neck. It often
intensifies when you cough, lie down or inhale deeply. Sitting up and leaning forward can
often ease the pain.
Ecg widespread st elevation
Tx: ansaid

413. A 6m boy has been brought to ED following an apneic episode at home. He is now
completely well but his parents are anxious as his cousin died of SIDS at a similar age.
The parents ask for guidance on BLS for a baby of his age. What is the single most
recommended technique for cardiac compressions?
a. All fingers of both hands
b. All fingers of one hand
c. Heel of one hand
d. Heel of both hand
e. Index and middle fingertips of one hand
The key is E. Index and middle fingertips of one hand.
414. A 70yo man had a right hemicolectomy for cecal carcinoma 6days ago. He now has
abdominal distension and recurrent vomiting. He has not opened his bowels since
surgery. There are no bowel sounds. WBC=9, Temp=37.3C. What is the single most
appropriate next management?
a. Antibiotic therapy IV
b. Glycerine suppository
c. Laparotomy
d. NG tube suction and IV fluids
e. TPN

1.
2.
3.
4.

1.
2.

The key is D. NG tube suction and IV fluids. [The patient has developed paralytic ileus
which should be treated conservatively].
s/s of paralytic ileus
diffuse abd pain
constipation
abd distension
nausea vomitis may contain bile
INV : abd x ray errect+ serum electrolytes
TX : conservative
npo
ng +iv fluids
215. A 60yo man with a 4y hx of thirst, urinary freq and weight loss presents with a deep
painless
ulcer on the heel. What is the most appropriate inv?
a. Arteriography
b. Venography
c. Blood sugar
d. Biopsy for malignant melanoma
e. Biopsy for pyoderma
The key is C. Blood sugar. [The patient probably developed diabetic foot].
the next step wd be doppler scan to assess the vascular status
416. A 16yo boy presents with rash on his buttocks and extensor surface following a
sore throat.
What is the most probable dx?
a. Measles
b. Bullous-pemphigoid
c. Rubella

d. ITP
e. HSP
it's a wrong key
right ans is E
# In HSP rash typically found in buttocks, legs and feets and may also appear on the
arms, face and trunk.
in ITP it mostly occurs in lower legs. #HSP usually follow a sore throat and ITP follow
viral infection like flue or URTI.
# HSP is a vasculitis while ITP is deficiency of platelets from more destruction in spleen
which is immune mediated].
417. A 34yo man with a white patch on the margin of the mid-third of the tongue. Which
is the single most appropriate LN involved?
a. External iliac LN
b. Pre-aortic LN
c. Aortic LN
d. Inguinal LN
e. Iliac LN
f. Submental LN
g. Submandibular LN
h. Deep cervical LN
The key is G. Submandibular LN.
418. A 50yo lady presents to ED with sudden severe chest pain radiating to both
shoulder and
accompanying SOB. Exam: cold peripheries and paraparesis. What is the single most
appropriate
dx?
a. MI
b. Aortic dissection
c. Pulmonary embolism
d. Good pastures syndrome
e. Motor neuron disease
The key is B. Aortic dissection. [Usual management for type A dissection is surgery and
for type B is conservative].
Dissecting aortic aneurysm. pt history of chest pain & interscapular back pain indicate dissecting aneurysm
in the descending thoracic aorta can causes interference with the blood supply to the anterior spinal artery
and causes the infarction of the ant aspect of the spinal artery that is anterior spinal artery syndrome and
paraparesis. Treatment- Type A: Immediately IV labetalol for control of HTN to reduces the extension of
dissection then for surgical Mx but Type-B for only conservative Mx.

STANFORD CLASSIFICATION
1. TYPE A : INVOLVING ASCENDING AORTA
2. TYPE B: DOES NOT INVOLVE ASCENDING AORTA
419. A 54yo myopic develops flashes of light and then sudden loss of vision. That is the
single most appropriate tx?
a. Panretinal photocoagulation
b. Peripheral iridectomy
c. Scleral buckling

d. Spectacles
e. Surgical extraction of lens
The key is C. Scleral buckling.
DX: RETINAL DETACHMENT
420. A 40yo chronic alcoholic who lives alone, brought in the ED having been found
confused at
home after a fall. He complains of a headache and gradually worsening confusion. What
is the
most likely dx?
a. Head injury
b. Hypoglycemia
c. Extradural hematoma
d. Subdural hematoma
e. Delirium
The key is D. Subdural hematoma. [subdural hematoma may be acute or chronic. In
chronic symptoms may not be apparent for several days or weeks. Symptoms of
subdural hematomas are: fluctuating level of consciousness, insidious physical or
intellectual slowing, sleepiness, headache, personality change and unsteadiness.
TX: SURGERY e.g. via barr twist drill and burr hole craniostomy 1 line. Craniotomy if
the clot organized 2 line].
MOST COMMON IN OLD PEOPLE AND DRUNKS WITH H/O FREQUENT FALLS
st

nd

421. A 54yo man with alcohol dependence has tremor and sweating 3days into a hosp
admission for a fx femur. He is apprehensive and fearful. What is the single most
appropriate tx?
a. Acamprossate
b. Chlordiazepoxide
c. Lorazepam
d. Lofexidine
e. Procyclidine
Ans. The key is B. Chlordiazepoxide. [This is a case of alcohol withdrawal syndrome.
Chlordiazepoxide when used in alcohol withdrawal it is important not to drink alcohol
while taking Chlordiazepoxide.
Chlordiazepoxide should only be used at the lowest possible dose and for a maximum of
up to four weeks. This will reduce the risks of developing tolerance, dependence and
withdrawal].
422. A 5yo child complains of sore throat and earache. He is pyrexial. Exam: tonsils
enlarged and
hyperemic, exudes pus when pressed upon. What is the single most relevant dx?
a. IM
b. Acute follicular tonsillitis
c. Scarlet fever
d. Agranulocytosis
e. Acute OM
Ans. The key is B. Acute follicular tonsillitis. [Tonsillitis is usually caused by a viral
infection or, less commonly, a bacterial infection. The given case is a bacterial

tonsillitis (probably caused by group A streptococcus). There are four main signs
that tonsillitis is caused by a bacterial infection rather than a viral infection. They are:
a high temperature
white pus-filled spots on the tonsils
no cough
swollen and tender lymph nodes (glands).
423. A man with a fam hx of panic disorder is brought to the hosp with palpitations,
tremors,
sweating and muscles tightness on 3 occasions in the last 6 wks. He doesnt complain of
headache and his BP is WNL. What is the single most appropriate long-term tx for him?
a. Diazepam
b. Olanzapine
c. Haloperidol
d. Fluoxetine
e. Alprazolam
Ans. The key is D. Fluoxetine. [Recommended treatment for panic disorder is i) CBT ii)
Medication (SSRIs or TCA). NICE recommends a total of seven to 14 hours of CBT to
be completed within a four month period. Treatment will usually involve having a weekly
one to two hour session. When drug is prescribed usually a SSRI is preferred.
Antidepressants can take two to four weeks before becoming effective].
424. A 28yo man presents with rapid pounding in the chest. He is completely conscious
throughout. The ECG was taken (SVT). What is the 1st med to be used to manage this
condition?
a. Amiodarone
b. Adenosine
c. Lidocaine
d. Verapamil
e. Metoprolol
Ans. The key is B. Adenosine. [Management of SVT: i) vagal manoeuvres (carotid sinus
message, valsalva manoeuvre) transiently increase AV-block, and unmask the
underlying atrial rhythm. If unsuccessful then the first medicine used in SVT is
adenosine, which causes transient AV block and works by i) transiently slowing
ventricles to show the underlying atrial rhythm ii) cardioverting a junctional tachycardia to
sinus rhythm. OHCM].
425. A 56yo woman who is depressed after her husband died of cancer 3m ago was
given
amitryptaline. Her sleep has improved and she now wants to stop medication but she
still
speaks about her husband. How would you manage her?
a. CBT
b. Continue amitryptaline
c. Psychoanalysis
d. Bereavement counselling
e. Antipsychotic

Ans. The key is B. Continue amitriptyline. [depression is important feature of


bereavement. Patient may pass sleepless nights. As this patients sleep has improved it
indicate he has good response to antidepressant and as he still speaks about her
husband there is chance to deterioration of her depression if antidepressant is stopped.
For depressive episodes antidepressants should be continued for at least 6-9 months
351. A 35yo lady presents with painful ulcers on her vulva, what is the appropriate inv
which will lead to the dx?
a. Anti-HSV antibodies
b. Dark ground microscopy of the ulcer
c. Treponema palladium antibody test
d. Rapid plasma regain test
e. VDRL
Dx genital herpes
Ans. key A. Anti-HSV antibodies. [Genital Herpes may be asymptomatic or may remain
dormant for months or even years. When symptoms occur soon after a person is
infected, they tend to be severe. They may start as multiple small blisters that eventually
break open and produce raw, painful sores that scab and heal over within a few weeks.
The blisters and sores may be accompanied by flu-like symptoms with fever and swollen
lymph nodes.
treatment : There are three major drugs commonly used to treat genital herpes
symptoms: acyclovir (Zovirax), famciclovir (Famvir), and valacyclovir(Valtrex). These are
all taken in PO. Severe cases may be treated with the intravenous (IV) drug acyclovir].
options B C D & E are tests for syphilis which presents with single painless ulcer
(canchre)
352. A 53yo man presents with a longstanding hx of a 1cm lesion on his arm. It has
started
bleeding on touch. What is the most likely dx?
a. Basal cell carcinoma
b. Kaposis sarcoma
c. Malignant melanoma
d. Squamous cell carcinoma
e. Kerathoacanthoma
Ans. D Squamous cell carcinoma. [SSCs Arises in squamous cells. SCCs may occur on all
areas of the body including the mucous membranes and genitals, but are most common in areas frequently
exposed to the sun, such as the rim of the ear, lower lip, face, balding scalp, neck, hands, arms and legs.

SCCs often look like scaly red patches, open sores, elevated growths with a central
depression, or warts; they may crust or bleed.
investigation: tissue sample (biopsy) will be examined under a microscope to arrive at
a diagnosis.
prognosis : Squamous cell carcinomas detected at an early stage and removed
promptly are almost always curable and cause minimal damage].
basal cell carcinoma is usually on face with inverted margins
malignant melanoma is on sun exposed parts and is dark (black colored) ulcer
353. A 47yo man with hx of IHD complains of chest pain with SOB on exertion

over the
past few days. ECG normal, Echo= increased EF and decreased septal wall thickness.
What is the most likely dx?
a. Dilated CM
b. Constrictive pericarditis
c. Amyloidosis

d. Subacute endocarditis
Ans. The key is A. Dilated CM.
points in fav: sob, palpitation, dec septal wall thinning
treatment : beta blocker, acei, diuretics
Constrictive pericarditis doesnt fits because it starts with urti has pain on lying flat which
is relieved by leaning forward
ecg shows wide spread st elevation
Amyloid deposition in the heart can cause both diastolic and systolic heart failure.
EKG changes may be present, showing low voltage and conduction abnormalities
like atrioventricular block or sinus node dysfunction. On echocardiography the
heart shows restrictive filling pattern, with normal to mildly reduced ejec fraction
354. An elderly pt who is known to have DM presents to the hospital with drowsiness,
tremors and confusion. What inv should be done to help in further management?
a. Blood sugar
b. ECG
c. Standing and lying BP
d. Fasting blood sugar
e. CT
Ans. The key is A. Blood sugar.since he is known diabetic he may have gotten
hypoglycemic d/t his meds
355. A 28yo pregnant woman with polyhydramnios and SOB comes for an anomaly scan
at 31 wks. US= absence of gastric bubble. What is the most likely dx?
a. Duodenal atresia
b. Esophageal atresia
c. Gastrochiasis
d. Exomphalos
e. Diaphragmatic hernia
Ans. The key is B. Oesophageal atresia.
This condition is visible, after about 26 weeks, on an ultrasound. On antenatal
USG, the finding of an absent or small stomach in the setting of polyhydramnios
used to be considered suspicious of esophageal atresia. However, these findings
have a low positive predictive value. The upper neck pouch sign is another sign
that helps in the antenatal diagnosis of esophageal atresia and it may be detected
soon after birth as the affected infant will be unable to swallow its own saliva.
Also, the newborn can present with gastric distention, cough, apnea, tachypnea,
and cyanosis. In many types of esophageal atresia, a feeding tube will not pass
through the esophagus.
356. A 1m boy has been brought to the ED, conscious but with cool peripheries and has
HR=222bpm. He has been irritable and feeding poorly for 24h. CXR=borderline enlarged
heart with clear lung fields. ECG=regular narrow complex tachycardia, with difficulty
identifying p wave. What is the single most appropriate immediate tx?
a. Administer fluid bolus

b. Administer oxygen
c. Oral beta-blockers
d. Synchronized DC cardio-version
e. Unilateral carotid sinus massage

The key is D. Synchrnized DC cardioversion.


reason: As the patient is in probable hemodynamic instability (suggested by cool
peripheries) so we should go for DC cardioversion.
diagnosis SVT.
357. A 7yo child presented with chronic cough and is also found to be jaundiced on
examination.
What is the most likely dx?
a. Congenital diaphragmatic hernia
b. Congenital cystic adenematoid malformation
c. Bronchiolitis
d. RDS
e. Alpha 1 antitrypsin deficiency

The key is E. Alpha 1 antitrypsin deficiency.


REASON. Unexplained liver disease with respiratory symptoms are very suggestive of
AATD.
liver disease occurs because of the accumulation AAT in it
where as d/t inability to be transported out of liver AATD causes emphysema hence the
resp problems
358. A 35yo construction worker is dx with indirect inguinal hernia. Which statement
below best
describes it?
a. Passes through the superficial inguinal ring only
b. Lies above and lateral to the pubic tubercle
c. Does not pass through the superficial inguinal ring
d. Passes through the deep inguinal ring
Ans. The key is D. Passess through the deep inguinal ring.
direct hernia passes forectly through the posterior wall of inguinal canal whereas indirect
can only do so via deep ring
359. A woman has numerous painful ulcers on her vulva. What is the cause?
a. Chlamydia
b. Trichomonas
c. Gardenella
d. HSV
e. EBV
Ans. The key is D. HSV. reason has been explained in q 351

360. A 72 yo man has been on warfarin for 2yrs because of past TIA and stroke. What is
the most important complication that we should be careful with?
a. Headache
b. Osteoporosis
c. Ear infection
d. Limb ischemia
e. Diarrhea
Ans. key is wrong
right key is A Headache, as there are chances of SAH or generally ICH
361. A 55yo man has been admitted for elective herniorraphy. Which among the
following can be the reason to delay his surgery?
a. Controlled asthma
b. Controlled atrial fib
c. DVT 2yrs ago
d. Diastolic BP 90mmHg
e. MI 2 months ago
Ans. E SAFER TO DO SURGERY AFTER 6 MONTHS
362. A 65yo known case of liver ca and metastasis presents with gastric reflux and
bloatedness. On bone exam there is osteoporosis. He also has basal consolidation in
the left lung. What is the next appropriate step?
a. PPI IV
b. Alendronate
c. IV antibiotics
d. Analgesic
e. PPI PO
IN THIS case reflux is the cause of recurrent pneumonia so both C AND E can be
right but to chose single one E is more appropriate
363. A 66yo man has the following ECG. What is the most appropriate next step in
management?
a. Metoprolol
b. Digoxin
c. Carotid sinus massage
d. Adenosine
e. Amiodarone.
Ans. A beta blocker for A FIB
364. A 22yo sexually active male came with 2d hx of fever with pain in scrotal area.
Exam: scrotal skin is red and tender. What is the most appropriate dx?
a. Torsion of testis
b. Orchitis
c. Inguinal hernia
d. Epididymo-orchitis
D Epididymo-orchitis.

In orchitis there should be fever, elevation of testes reduces pain (positive prehn sign), In
torsion testis lies at a higher level. In torsion urinalysis negative but in orchitis it is
positive. Orchitis usually occurs in sexually active man. X
365. A man on warfarin posted for hemicolectomy. As the pt is about to undergo surgery.
What
option is the best for him?
a. Continue with warfarin
b. Continue with warfarin and add heparin
c. Stop warfarin and add aspirin
d. Stop warfarin and add heparin
e. Stop warfarin
D Stop warfarin and add heparin
5 DAYS BEFORE SURGERY WARFARIN MUST BE REPLACED BY HEPARIN,
366. A 65yo known alcoholic is brought into hospital with confusion, aggressiveness and
ophthalmoplegia. He is treated with diazepoxide. What other drug would you like to
prescribe?
a. Antibiotics
b. Glucose
c. IV fluids
d. Disulfiram
e. Vit B complex
E Vitamin B complex. [confusion and ophthalmoplegia points towards the diagnosis of
Wernickes encephalopathy].
which occurs d/t thiamine def.
367. A 32yo woman has severe right sided abdominal pain radiating into the groin which
has lasted for 3h. She is writhering in pain. She has no abdominal signs. What is the
most likely cause of her abdominal pain?
a. Appendicitis
b. Ruptured ectopic pregnancy
c. Salpingitis
d. Ureteric colic
e. Strangulated hernia
D Ureteric colic.
It indicate stone at lower ureter. [i) Pain from upper ureteral stones tends to radiate to the flank and
lumbar areas. ii) Midureteral calculi cause pain that radiates anteriorly and caudally. This midureteral pain in
particular can easily mimic appendicitis on the right or acute diverticulitis on the left. iii) Distal ureteral stones
cause pain that tends to radiate into the groin or testicle in the male or labia majora in the female.

368. A 39yo coal miner who smokes, drinks and has a fam hx of bladder cancer is
suffering from BPH. The most important risk factor for his bladder carcinoma is?
a. Fam hx
b. Smoking
c. Exposure to coal mine
d. BPH

B smoking.
. Risk factors of bladder cancer:
i) Smoking ii) Exposure to chemicals used in dye industry iii) Whites are more likely to
develop bladder cancer iv) Risk increases with age v) More common in men vi) Chronic
bladder irritation and infections (urinary infections, kidney and bladder stones, bladder
catheter left in place a long time.) vii) Personal history of bladder or other urothelial
cancer viii) Family history ix) Chemotherapy or radiotherapy x) Pioglitazone for more
than one year and certain herb xi) Arsenic in drinking water xii) Low fluid consumption.
369. A 34yo woman is referred to the endocrine clinic with a hx of thyrotoxicosis. At her
1st
appointment she is found to have a smooth goiter, lid lag and bilateral exophthalmos
with puffy
eyelids and conjunctival injection. She wants to discuss the tx of her thyroid prb as she is
keen to become pregnant. What is the most likely tx you would advise?
a. 18m of carbimazole alone
b. 18m of PTU alone
c. A combo od anti-thyroid drug and thyroxine
d. Radioactive iodine
e. Thyroidectomy

B 18m of PTU alone.


Other drug option i.e Carbamazepine is teratogenic [can cause i) spina bifida
ii)cardiovascular malformations ETC . PTU is on the other hand relatively safe in
pregnancy.
370. A child living with this stepfather is brought by the mother with multiple bruises,
fever and
fractures. What do you suspect?
a. NAI
b. Malnutrition
c. Thrombocytopenia
d. HIV
Ans. The key is A. NAI. [H/O living with stepfather, multiple bruises, fever and fractures
are suggestive of NAI]. OTHER possible points can include hx not matching with
bruises, wounds which are a day or two older at the time of presentation.
371. A young man who was held by the police was punched while in custody. He is now
cyanosed and unresponsive. What is the 1st thing you would do?
a. IV fluids
b. Clear airway
c. Turn pt and put in recovery position
d. Give 100% oxygen
e. Intubate and ventilate
B. Clear airway. [ABC protocol].

372. A HTN male loses vision in his left eye. The eye shows hand movement and a light
shined in the eye is seen as a faint light. Fundus exam: flame shaped hemorrhages. The
right eye is normal. What is the cause of this pts unilateral blindness?
a. HTN retinopathy
b. CRA thrombosis
c. CRV thrombosis
d. Background retinopathy
e. Retinal detachment
key is wrong
right ans is C ( unilateral blindness with flameshaped hemorrhages are
characteristic of CRVO)
373. A mentally retarded child puts a green pea in his ear while eating. The carer
confirms this.
Otoscopy shows a green colored object in the ear canal. What is the most appropriate
single
best approach to remove this object?
a. By magnet
b. Syringing
c. Under GA
d. By hook
e. By instilling olive oil
C UNDER GA
since child is retard, if he wasnt then the ans would be D
olive oil is for insects
374. A pt presents with longstanding gastric reflux, dysphagia and chest pain. On barium
enema, dilation of esophagus with tapering end is noted. He was found with Barretts
esophagus. He had progressive dysphagia to solids and then liquids. What is the single
most appropriate dx?
a. Achalasia
b. Esophageal spasm
c. GERD
d. Barretts esophagus
e. Esophageal carcinoma
E Oesophageal carcinoma. [there is dilatation in oesophagus which is seen both in
achalasia and carcinoma. Dysphagia to solid initaially is very much suggestive of
carcinoma and also barretts change is a clue to carcinoma]
Progressive dysphagia with h/o barrett esophagus are the key indicators
375. A 48yo lady presents with itching, excoriations, redness, bloody discharge and
ulceration around her nipple. What is the most likely dx?
a. Pagets disease of the breast
b. Fibrocystic dysplasia
c. Breast abscess
d. Duct papilloma
e. Eczema

A Pagets disease of the breast.


TYPICAL manifestation of pagets disease
eczema like rash involving nipple and areola with straw or bloody discharge
Also nipple turns inward in advances stages pt complaints of burning sensation at the
site of lesion
DX mammography and biopsy
TX surgery + chemo or radio may be needed
376. Pt with widespread ovarian carcinoma has bowel obstruction and severe colic for
2h and was normal in between severe pain for a few hours. What is the most appropriate
management?
a. PCA (morphine)
b. Spasmolytics
c. Palliative colostomy
d. Oral morphine
e. Laxatives
C. Palliative colostomy.
Cancer or chemotherapy induced obstructions are unlikely to respond to conservative
management [NBM, IV fluid, nasogastric suction] and hence only analgesia will not
relieve it. So in such cases we have to go for palliative colostomy.
377. A 70yo man admits to asbestos exposure 20yrs ago and has attempted to quit
smoking. He has noted weight loss and hoarseness of voice. Choose the single most
likely type of cancer a.w risk
factors present.
a. Basal cell carcinoma
b. Bronchial carcinoma
c. Esophageal carcinoma
d. Nasopharyngeal carcinoma
e. Oral carcinoma

B. Bronchial carcinoma. [Asbestos exposure is a risk factor for lung cancer


and also has a synergistic effect with cigarette smoke].
#. Conditions related to asbestos exposure: i) Pleural plaques (after a latent period of
20-40 yrs) ii) Pleural thickening iii) Asbestosis (latent period is typically 15-30 yrs) iv)
Mesothelioma (prognosis is very poor) v) Lung cancer.
378. A 32yo woman had progressive decrease in vision over 3yrs. She is no dx as
almost blind. What would be the mechanism?
a. Cataract
b. Glaucoma
c. Retinopathy
d. Uveitis
e. Keratitis

B. Glaucoma.

. Cataract is unlikely at this age. Nothing in the history suggests retinopathy. Uveitis and
iritis doesnt have such degree of vision loss and iritis and anterior uveitis have pain,
redness and photophobia. Open angle glaucoma is likely cause.
379. A child during operation and immediately after showed glycosuria, but later his
urine sugar was normal. Choose the most probable dx.
a. Pre-diabetic state
b. Normal finding
c. Low renal tubular threshold
d. DM
B Normal finding.
Stress during operation can cause transient hyperglycemia causing glycosuria
secondary to stress induced rise of cortisole which becomes normal after some time.
380. A pt presented with hx of swelling in the region of the sub-mandibular region, which
became
more prominent and painful on chewing. He also gave hx of sour taste in the mouth, the
area is
tender on palpation. Choose the most probable dx?
a. Chronic recurrent sialadenitis
b. Adenolymphoma
c. Mikuliczs disease
d. Adenoid cystic carcinoma
e. Sub-mandibular abscess
A Chronic recurrent sialadenitis. [pain, swelling, more pain on chewing, tenderness, and
submandibular region suggests diagnosis of submandibular chronic recurrent
sialadenitis, usually secondary to sialolithiasis or stricture].
381. ECG of an 80yo pt of ICH shows saw-tooth like waves, QRS complex of 80ms
duration,
ventricular rate=150/min and regular R-R interval. What is the most porbable dx?
a. Atrial fib
b. Atrial flutter
c. SVT
d. Mobitz type1 second degree heart block
e. Sinus tachycardia
B Atrial flutter. [Saw-tooth like waves, normal QRS comples of 80 ms (normal range 70100 ms), ventricular rate of 150/min and regular R-R interval is diagnostic of atrial
flutter].
FOR AFIB THERE WD BE IRREGULARARLY IRREGULAR RHYTHM
382. A 50 yo woman who was treated for breast cancer 3 yrs ago now presents with
increase thirst and confusion. She has become drowsy now. What is the most likely
metabolic abnormality?
a. Hypercalcemia
b. Hyperkalemia
c. Hypoglycemia
d. Hyperglycemia
e. Hypercalcemia.
E HYPERCALCEMIA

Ans. 2. Increased thirst, confusion, drowsiness these are features of hypercalcemia. Any
solid organ tumour can produce hypercalcemia. Here treated Ca breast is the probable
cause of hypercalcemia.
383. A 29yo woman presents to her GP with a hx of weight loss, heat intolerance, poor
conc and
palpitations. Which of the following is most likely to be a/w dx of thyroiditis a/w viral
infection?
a. Bilateral exophthalmos
b. Diffuse, smooth goiter
c. Reduced uptake on thyroid isotope scan
d. Positive thyroid peroxidase antibodies
e. Pretibial myxedema
C. Reduced uptake on thyroid isotope scan.
DX De Quervains or subacute thyroiditis.
. Viral or subacute thyroiditis: diagnostic criteria: i) Features of hyperthyroidism present.
ii) Pain thyroid, not mentioned. iii) Investigations: high esr (60-100) not mentioned,
Reduced uptake of radioactive iodine by the gland.
384. A lady, post-colostomy closure after 4days comes with fluctuating small swelling in
the stoma.
What is the management option for her?
a. Local exploration
b. Exploratory laparotomy
c. Open laparotomy
d. Reassure
A Local exploration.
THERE MUST BE SOME LOCAL WOUND PROBLEM
385. A 65yo female pt was given tamoxifen, which of the following side effect caused by
it will
concern you?
a. Fluid retention
b. Vaginal bleeding
c. Loss of apetite
d. Headache and dizziness
e.
B Vaginal bleeding.
. Tamoxifen can promote development of endometrial carcinoma. So vaginal bleeding
will be of concern for us.
386. A 39yo man with acute renal failure presents with palpitations. His ECG shows tall
tented T
waves and wide QRS complex. What is the next best step?
a. Dialysis
b. IV calcium chloride
c. IV insulin w/ dextrose
d. Calcium resonium
e. Nebulized salbutamol

B. IV calcium chloride (both IV calcium gluconate or IV calcium chloride can be used)


when there is ECG changes.
DX The ECG changes are suggestive of Hyperkalemia.
At potassium level of >5.5mEq/L occurs tall tented T waves and at potassium level
>7mEq/L occurs wide QRS complex with bizarre QRS morphology.
387. A 54yo pt 7 days after a total hip replacement presents with acute onset
breathlessness and
raised JVP. Which of the following inv will be most helpful in leading to a dx?
a. CXR
b. CTPA
c. V/Q scan
d. D-Dimer
e. Doppler US of legs

The key is B. CTPA.


The patient has a +ve two level PE Wells score (if it was negative we should do DDimer) and there is no renal impairment or history suggestive of allergy to contrast
media (if these present we should have go for VQ scan) the investigation of choice is
PTCA. NICE guideline.
388. A 7yo girl has been treated with penicillin after sore throat, fever and cough. Then
she
develops skin rash and itching. What is the most probable dx?
a. Erythema nodosum
b. Erythema multiforme
c. SJS
d. Erythema marginatum
e. Erythema gangernosum

. The key is B. Erythema multiforme.


Common drugs causing erythma multiforme are: antibiotics (including, sulphonamides,
penicillin), anticonvulsants (phenytoin,barbiturates), aspirin, antituberculoids, and allopurinol.
CLOSELY related option is SJS which would have muco cutaneous rash but in here we hav only
cutaneous rash

389. A 60yo man presented with a lump in the left supraclavicular region. His appetite is
decreased and he has lost 5kg recently. What is the most probably dx?
a. Thyroid carcinoma
b. Stomach carcinoma
c. Bronchial carcinoma
d. Mesothelioma
e. Laryngeal carcinoma
The key is B. Stomach carcinoma. [Mentioned lump in the left supraclavicular region is
Vershows gland, has long been regarded as strongly indicative of the presence of cancer in the
abdomen, specifically gastric cancer].

390. A 64yo man has presented to the ED with a stroke. CT shows no hemorrhage.
ECG shows atrial fib. He has been thrombolysed and hes awaiting discharge. What
prophylactic regimen is best for him?
a. Warfarin
b. Heparin
c. Aspirin
d. Statins
e. Beta blockers
The key is A. Warfarine. [Atrial fibrillation: post stroke- following a stroke or TIA warfarine
should be given as the anticoagulant of choice. NICE guideline].
391. A 54yo man after a CVA presents with ataxia, intention tremors and slurred speech.
Which part of the brain has been affected by the stroke?
a. Inner ear
b. Brain stem
c. Diencephalon
d. Cerebrum
e. Cerebellum
The key is E. Cerebellum.
i) Ataxia ii) slurred speech or dysarthria iii) dysdiodokokinesis iv) intention tremor v)
nystagmus. are the signs of cerebellar defect
292. A 57yo man with blood group A complains of symptoms of vomiting, tiredness,
weight loss and palpitations. Exam: hepatomegaly, ascites, palpable left supraclavicular
mass. What is the most likely dx?
a. Gastric carcinoma
b. Colorectal carcinoma
c. Peptic ulcer disease
d. Atrophic gastritic
e. Krukenburg tumor
Ans. The key is A. Gastric carcinoma. [i) blood group A is associated with gastric cancer
ii) vomiting, tiredness, weight loss are general features of gastric cancer iii) palpitation
from anemia of cancer iv) hepatomegaly and ascites are late features of gastric cancer.
v) palpable left supraclavicular mass- is Vershows gland, has long been regarded as
strongly indicative of gastric cancer].
293. A 21yo girl looking unkempt, agitated, malnourished and nervous came to the
hospital asking
for painkillers for her abdominal pain. She is sweating, shivering and complains of joint
pain.
What can be the substance misuse here?
a. Alcohol
b. Heroin
c. Cocaine
d. LSD
e. Ecstasy

The key is B. Heroin. [agitation, nervousness, abdominal cramp, sweating, shivering and
piloerection, arthralgia these are features of heroin withdrawal].
394. A child presents with increasing jaundice and pale stools. Choose the most
appropriate test?
a. US abdomen
b. Sweat test
c. TFT
d. LFT
e. Endomyseal antibodies
The key is A. US abdomen. [This is a picture suggestive of obstructive jaundice. LFT can
give clue like much raised bilirubin, AST and ALT not that high and raised alkaline
phosphatase but still USG is diagnostic in case of obstructive jaundice].
395. A 32yo man presents with hearing loss. AC>BC in the right ear after Rhine test. He
also
complains of tinnitus, vertigo and numbness on same half of his face. What is the most
appropriate inv for his condition?
a. Audiometry
b. CT
c. MRI
d. Tympanometry
e. Webers test
The key is C. MRI. [features are suggestive of acaustic neuroma, so MRI is the preferred
option]. it involves basically 8th nerve but 6 7 9 and 10th nerves are also involved with it
396. A 56 yo lady with lung cancer presents with urinary retention, postural hypotension,
diminished reflexes and sluggish pupillary reaction. What is the most likely explanation
for her symptoms?
a. Paraneoplastic syndrome
b. Progression of lung cancer
c. Brain metastasis
d. Hyponatremia
e. Spinal cord compression
The key is A. Paraneoplastic syndrome.
s/s are of autonomic neuropathy which occurs in paraneoplastic syndrome
397. An old woman having decreased vision cant see properly at night. She has
changed her glasses quite a few times but to no effect. She has normal pupil and
cornea. What is the most likely dx?
a. Cataract
b. Glaucoma
c. Retinal detachment
d. Iritis
e. GCA
key is wrong
correct key is A cataract

old age and progressive weakness supports Cataract


398. A pt comes with sudden loss of vision. On fundoscopy the optic disc is normal.
What is the
underlying pathology?
a. Iritis
b. Glaucoma
c. Vitreous chamber
d. Retinal detachment

Ans. 1. The Key is D. Retinal detachment.


#Causes of sudden painless loss of vision:
1.
2.
3.
4.
5.
6.

Retinal detachment
Vitreous haemorrhage
Retinal vein occlusion
Retinal artery occlusion
Optic neuritis
Cerebrovascular accident

remember retinal detachment has vision loss as if curtain is coming down


399. A child was woken up from sleep with severe pain in the testis. Exam: tenderness
on palpation and only one testis was normal in size and position. What would be your
next step?
a. Analgesia
b. Antibiotics
c. Refer urgently to a surgeon
d. Reassurance
e. Discharge with analgesics
Ans. The key is A. Analgesia. [According to some US sites it is analgesia but no UK site
support this!!! So for Plab exam the more acceptable option is C. Refer urgently to a
surgeon].
IN TORSION THE SOONER THE SURGICAL INTERVENTION DONE, THE BETTER
THE RESULTS ARE
400. A child suffering from asthma presents with Temp 39C, drooling saliva on to the
mothers lap,
and taking oxygen by mask. What sign will indicate that he is deteriorating?
a. Intercostal recession
b. Diffuse wheeze
drooling means the age is less than 3, so drowsiness is ruled out because of the age of the baby
c. Drowsiness
The key is A. Intercostal recession. [ here intercostals recession and drowsiness both
answers are correct. Hope in exam there will be one correct option]. but to chose
among them, better go with A
401. A 12yo boy presents with painful swollen knew after a sudden fall. Which bursa is
most likely tobe affected?
a. Semimembranous bursa
b. Prepatellar bursa

c. Pretibial bursa
d. Suprapatetaller bursa
. The key is B. Prepatellar bursa. [A fall onto the knee can damage the prepatellar bursa. This
usually causes bleeding into the bursa sac causing swellen painful knee. Prepatellar bursitis that is caused
by an injury will usually go away on its own. The body will absorb the blood in the bursa over several weeks,
and the bursa should return to normal. If swelling in the bursa is causing a slow recovery, a needle may be
inserted to drain the blood and speed up the process. There is a slight risk of infection in putting a needle
into the bursa].
402. A

61yo man has been referred to the OPD with frequent episodes of breathlessness
and chest pain a/w palpitations. He has a regular pulse rate=60bpm. ECG=sinus rhythm.
What is the
most appropriate inv to be done?
a. Cardiac enzymes
b. CXR
c. ECG
d. Echo
e. 24h ECG
The key is E. 24h ECG.
Indications of 24 h ambulatory holter monitoring:

To evaluate chest pain not reproduced with exercise testing

To evaluate other signs and symptoms that may be heart-related, such as fatigue,
shortness of breath, dizziness, or fainting

To identify arrhythmias or palpitations

To assess risk for future heart-related events in certain conditions, such as idiopathic
hypertrophic cardiomyopathy, post-heart attack with weakness of the left side of the
heart, or Wolff-Parkinson-White syndrome

To assess the function of an implanted pacemaker

To determine the effectiveness of therapy for complex arrhythmias

403. A woman dx with Ca Breast presents now with urinary freq. which part of the brain
is the
metastasis spread to?
a. Brain stem
b. Pons
c. Medulla
d. Diencephalon
e. Cerebral cortex
The key is D. Diencephalon. [diencephalon is made up of four distinct components: i) the
thalamus ii) the subthalamus iii) the hypothalamus and iv) the epithalamus. Among these
the hypothalamus has crucial role in causing urinary frequency].

404. A man is very depressed and miserable after his wifes death. He sees no point in
living now that his wife is not around and apologises for his existence. He refuses any
help offered. His son has brought him to the ED. The son can.t deal with the father any
more. What is the most
appropriate next step?
a. Voluntary admission to psychiatry ward
b. Compulsory admission under MHA
c. Refer to social services
d. Alternate housing
e. ECT
Ans. The key is B. Compulsory admission under MHA. [This patient is refusing any help
offered! And his son cannot deal with him anymore! In this situation voluntary admission
to psychiatry ward is not possible and the option of choice is compulsory admission
under MHA].
405. A 31yo man has epistaxis 10 days following polypectomy. What is the most likely
dx?
a. Nasal infection
b. Coagulation disorder
c. Carcinoma
The key is A. Nasal infection.
HEMORRHAGE AFTER 5 TO 7 DAYS IS SECONDARY HEMORRHAGE [Infection is
one of the most important cause of secondary hemorrhage].
406. A woman had an MI. She was breathless and is put on oxygen mask and GTN, her
chest
pain has improved. Her HR=40bpm. ECG shows ST elevation in leads I, II, III. What is
your next step?
a. LMWH
b. Streptokinase
c. Angiography
d. Continue current management
e. None

Ans. The key is B. Streptokinase


algorithm for st elevation MI
angioplasty/thrombolysis
b blocker
acei
clopidogrel
407. A 67yo male presents with polyuria and nocturia. His BMI=33, urine culture =
negative for
nitrates. What is the next dx inv?
a. PSA
b. Urea, creat and electrolytes
c. MSU culture and sensitivity
d. Acid fast urine test
e. Blood sugar

The key is E. Blood sugar. [Age at presentation and class1 obesity favours the diagnosis
of type2 DM].
since culture is -ve for nitrates, so uti is ruled out
408. A pt from Africa comes with nodular patch on the shin which is reddish brown. What
is the
most probable dx?
a. Lupus vulgaris
b. Erythema nodosum
c. Pyoderma gangrenosum
d. Erythema marginatum
e. Solar keratosis
The key is B. Erythema nodosum. [Causes of erythema nodosum: MOST COMMON
CAUSES- i) streptococcal infection ii) sarcoidosis. Other causes- tuberculosis,
mycoplasma pneumonia, infectious mononucleosis, drugs- sulfa related drug, OCP,
oestrogen; Behcets disease, CD, UC; lymphoma, leukemia and some others].
#Nodes are mostly on anterior aspect of shin
409. A 29yo lady came to the ED with complaints of palpitations that have been there for
the past 4 days and also feeling warmer than usual. Exam: HR=154bpm, irregular
rhythm. What is the tx for her condition?
a. Amiadarone
b. Beta blockers
c. Adenosine
d. Verapamil
e. Flecainide
The key is B. Beta blockers [the probable arrhythymia is AF secondary to
thyrotoxicosis(heat intolerance). So to rapid control the symptoms of thyrotoxicosis Beta
blocker should be used].
410. A T2DM is undergoing a gastric surgery. What is the most appropriate pre-op
management?
a. Start him in IV insulin and glucose and K+ just before surgery
b. Stop his oral hypoglycemic on the day of the procesure
c. Continue regular oral hypoglycemic
d. Stop oral hypoglycemic the prv night and start IV insulin with glucose and K+ before
surgery
e. Change to short acting oral hypoglycemic
The key is D. Stop oral hypoglycemic the prv night and start IV insulin with glucose and
K+ before surgery.
411. A 19yo boy is brought by his mother with complaint of lack of interest and no social
interactions. He has no friends, he doesnt talk much, his only interest is in collecting
cars/vehicles having around 2000 toy cars. What is the most appropriate dx?
a. Borderline personality disorder
b. Depression
c. Schizoaffective disorder

d. Autistic spectrum disorder


The key is D. Autistic spectrum disorder.
Autism spectrum disorders affect three different areas of a child's life:
Social interaction
Communication -- both verbal and nonverbal
Behaviors and interests
In some children, a loss of language is the major impairment. In others, unusual
behaviors (like spending hours lining up toys) seem to be the dominant factors.
412. A 45yo man who is diabetic and HTN but poorly compliant has chronic SOB,
develops severe SOB and chest pain. Pain is sharp, increased by breathing and
relieved by sitting forward. What is the single most appropriate dx?
a. MI
b. Pericarditis
c. Lung cancer
d. Good pastures syndrome
e. Progressive massive fibrosis
The key is B. Pericarditis. [Nature of pain i.e. sharp pain increased by breathing and
relieved by sitting forward is suggestive of pericarditis].
Nature of pericardial pain: the most common symptom is sharp, stabbing chest pain
behind the sternum or in the left side of your chest. However, some people with acute
pericarditis describe their chest pain as dull, achy or pressure-like instead, and of
varying intensity.

The pain of acute pericarditis may radiate to your left shoulder and neck. It often
intensifies when you cough, lie down or inhale deeply. Sitting up and leaning forward can
often ease the pain.
Ecg widespread st elevation
Tx: ansaid
413. A 6m boy has been brought to ED following an apneic episode at home. He is now
completely well but his parents are anxious as his cousin died of SIDS at a similar age.
The parents ask for guidance on BLS for a baby of his age. What is the single most
recommended technique for cardiac compressions?
a. All fingers of both hands
b. All fingers of one hand
c. Heel of one hand
d. Heel of both hand
e. Index and middle fingertips of one hand
The key is E. Index and middle fingertips of one hand.
414. A 70yo man had a right hemicolectomy for ceacal carcinoma 6days ago. He now
has abdominal distension and recurrent vomiting. He has not opened his bowels since
surgery. There are no bowel sounds. WBC=9, Temp=37.3C. What is the single most
appropriate next management?
a. Antibiotic therapy IV
b. Glycerine suppository
c. Laparotomy
d. NG tube suction and IV fluids
e. TPN

1.
2.
3.
4.

1.
2.

The key is D. NG tube suction and IV fluids. [The patient has developed paralytic ileus
which should be treated conservatively].
s/s of paralytic ileus
diffuse abd pain
constipation
abd distension
nausea vomitis may contain bile
INV : abd x ray errect+ serum electrolytes
TX : conservative
npo
ng +iv fluids
215. A 60yo man with a 4y hx of thirst, urinary freq and weight loss presents with a deep
painless
ulcer on the heel. What is the most appropriate inv?
a. Ateriography
b. Venography
c. Blood sugar
d. Biopsy for malignant melanoma
e. Biopsy for pyoderma
The key is C. Blood sugar. [The patient probably developed diabetic foot].
the next step wd be doppler scan to assess the vascular status
416. A 16yo boy presents with rash on his buttocks and extensor surface following a
sore throat.
What is the most probable dx?
a. Measles
b. Bullous-pemphigoig
c. Rubella
d. ITP
e. HSP
its a wrong key
right ans is E
# In HSP rash typically found in buttocks, legs and feets and may also appear on the
arms, face and trunk.
in ITP it mostly occurs in lower legs. #HSP usually follow a sorethroat and ITP follow
viral infection like flue or URTI.
# HSP is a vasculitis while ITP is deficiency of platelets from more destruction in spleen
which is immune mediated].
417. A 34yo man with a white patch on the margin of the mid-third of the tongue. Which
is the single most appropriate LN involved?
a. External iliac LN
b. Pre-aortic LN
c. Aortic LN
d. Inguinal LN
e. Iliac LN
f. Submental LN

g. Submandibular LN
h. Deep cervical LN
The key is G. Submandibular LN.
418. A 50yo lady presents to ED with sudden severe chest pain radiating to both
shoulder and
accompanying SOB. Exam: cold peripheries and paraparesis. What is the single most
appropriate
dx?
a. MI
b. Aortic dissection
c. Pulmonary embolism
d. Good pastures syndrome
e. Motor neuron disease
The key is B. Aortic dissection. [Usual management for type A dissection is surgery and
for type B is conservative].
STANFORD CLASSIFICATION
1. TYPE A : INVOLVING ASCENDING AORTA
2. TYPE B: DOESNOT INVOLVE ASCENDING AORTA
419. A 54yo myopic develops flashes of light and then sudden loss of vision. That is the
single most appropriate tx?
a. Pan retinal photo coagulation
b. Peripheral iridectomy
c. Scleral buckling
d. Spectacles
e. Surgical extraction of lens
The key is C. Scleral buckling.
DX: RETINAL DETACHMENT
420. A 40yo chronic alcoholic who lives alone, brought in the ED having been found
confused at
home after a fall. He complains of a headache and gradually worsening confusion. What
is the
most likely dx?
a. Head injury
b. Hypoglycemia
c. Extradural hematoma
d. Subdural hematoma
e. Delirium
The key is D. Subdural hematoma. [subdural hematoma may be acute or chronic. In
chronic symptoms may not be apparent for several days or weeks. Symptoms of
subdural hematomas are: fluctuating level of consciousness, insidious physical or
intellectual slowing, sleepiness, headache, personality change and unsteadiness.
TX: SURGERY e.g. via barr twist drill and barr hole craniostomy 1 line. Craniotomy if
the clot organized 2 line].
MOST COMMON IN OLD PEOPLE AND DRUNKS WITH H/O FREQUENT FALLS
st

nd

421. A 54yo man with alcohol dependence has tremor and sweating 3days into a hosp
admission for a fx femur. He is apprehensive and fearful. What is the single most
appropriate tx?
a. Acamprossate
b. Chlordiazepoxide
c. Lorazepam
d. Lofexidine
e. Procyclidine
Ans. The key is B. Chlordiazepoxide. [This is a case of alcohol withdrawal syndrome.
Chlordiazepoxide when used in alcohol withdrawal it is important not to drink alcohol
while taking Chlordiazepoxide.
Chlordiazepoxide should only be used at the lowest possible dose and for a maximum of
up to four weeks. This will reduce the risks of developing tolerance, dependence and
withdrawal].
422. A 5yo child complains of sore throat and earache. He is pyrexial. Exam: tonsils
enlarged and
hyperemic, exudes pus when pressed upon. What is the single most relevant dx?
a. IM
b. Acute follicular tonsillitis
c. Scarlet fever
d. Agranulocytosis
e. Acute OM

Ans. The key is B. Acute follicular tonsillitis. [Tonsillitis is usually caused by a viral
infection or, less commonly, a bacterial infection. The given case is a bacterial
tonsillitis (probably caused by group A streptococcus). There are four main signs
that tonsillitis is caused by a bacterial infection rather than a viral infection. They are:
a high temperature
white pus-filled spots on the tonsils
no cough
swollen and tender lymph nodes (glands).
423. A man with a fam hx of panic disorder is brought to the hosp with palpitations,
tremors,
sweating and muscles tightness on 3 occasions in the last 6 wks. He doesnt complain of
headache and his BP is WNL. What is the single most appropriate long-term tx for him?
a. Diazepam
b. Olanzapine
c. Haloperidol
d. Fluoxetine
e. Alprazolam
Ans. The key is D. Fluoxetine. [Recommended treatment for panic disorder is i) CBT ii)
Medication (SSRIs or TCA). NICE recommends a total of seven to 14 hours of CBT to
be completed within a four month period. Treatment will usually involve having a weekly
one to two hour session. When drug is prescribed usually a SSRI is preferred.
Antidepressants can take two to four weeks before becoming effective].

424. A 28yo man presents with rapid pounding in the chest. He is completely conscious
throughout. The ECG was taken (SVT). What is the 1st med to be used to manage this
condition?
a. Amiodarone
b. Adenosine
c. Lidocaine
d. Verapamil
e. Metoprolol
Ans. The key is B. Adenosine. [Management of SVT: i) vagal manoeuvres (carotid sinus
message, valsalva manoeuvre) transiently increase AV-block, and unmask the
underlying atrial rhythm. If unsuccessful then the first medicine used in SVT is
adenosine, which causes transient AV block and works by i) transiently slowing
ventricles to show the underlying atrial rhythm ii) cardioverting a junctional tachycardia to
sinus rhythm. OHCM].
425. A 56yo woman who is depressed after her husband died of cancer 3m ago was
given
amitryptaline. Her sleep has improved and she now wants to stop medication but she
still
speaks about her husband. How would you manage her?
a. CBT
b. Continue amitryptaline
c. Psychoanalysis
d. Bereavement counselling
e. Antipsychotic
Ans. The key is B. Continue amitriptyline. [depression is important feature of
bereavement. Patient may pass sleepless nights. As this patients sleep has improved it
indicate he has good response to antidepressant and as he still speaks about her
husband there is chance to deterioration of her depression if antidepressant is stopped.
For depressive episodes antidepressants should be continued for at least 6-9 months

426. A 64yo man presents with a hx of left sided hemiparesis and slurred
speech. He was absolutely fine 6h after the episode. What is the most
appropriate prophylactic regimen?
a. Aspirin 300mg for 2 weeks followed by aspirin 75mg
b. Aspirin 300mg for 2 weeks followed by aspirin 75mg and dipyridamole
200mg
c. Clopidogrel 75mg
d. Dipyridamole 200mg
e. Aspirin 300mg for 2 weeks
KEY- B
Dx- TIA.
What is TIA?

Inadequate circulation in part of the brain, gives a picture similar to stroke


but duration < 24 hours.
Common in old age. Men > women. ^ in black race.
Important risk factors- HTN, smoking, DM, Hyperlipidemia, Heart
disease. Management is by: Antiplatelets, anti HTN, lipid modifying ttt, AF
ttt and any risk factors like DM.
Treatment: Aspirin + dypiridamole (each as 300mg loading then 75mg
daily) + statin. [NICE guidelines]
427. A 63yo lady with a BMI=32 comes to the ED with complaints of
pigmentation on her legs. Exam: dilated veins could be seen on the lateral
side of her ankle. Which of the following is involved?
a. Short saphenous vein
b. Long saphenous vein
c. Deep venous system
d. Popliteal veins
e. Saphano-femoral junction
KEY- A
Short saphenous vein- lateral side
Long saphenous vein- medial side
*Long saphenous vein is the vessel of choice used for autotransplantation in
coronary artery bypass. It is also a common site for varicose vein formation.

428. A 55yo man presents with hx of weight loss and tenesmus. He is dx


with rectal carcinoma. Which risk factors help to develop rectal carcinoma
except following?
a. Smoking
b. Family hx
c. Polyp
d. Prv carcinoma
e. High fat diet
f.
High fibre diet
KEY- F
All options except High fiber diet are risk factors for developing rectal
carcinoma.
*Other risk factors for Rectal Carcinoma are:
-IBD
-Nulliparity and early menopause
-Diet rich in meat and fat, poor in folate and Calcium
-Sedentary lifestyle, obesity, smoking and high alcohol intake.
-Diabetes
-Radiation and asbestos exposure

429. A pt presents with a painful, sticky red eye with a congested


conjunctiva. What is the most suitable tx?
a. Antibiotic PO
b. Antihistamine PO
c. Antibiotic drops
d. Steroid drops
e. IBS
KEY- C
Dx- Bacterial Conjunctivitis.
Painful eye, usually bilateral. Smearing of vision on waking up.
Mild photophobia. If severe, indicates corneal involvement or adenoviral
conjunctivitis.
Thick yellowish-white mucopurulent discharge. Visual acuity is normal
Symptoms- Red eye, difficult to open in the morning, glued together by
discharge. Presence of follicles on the conjunctiva- More likely viral
conjunctivitis.

Treatment:
Topical broad spectrum antibiotics. Drug of choice is chloramphenicol
drops. If pregnant, intolerant to chloramphenicol or history of aplastic
anemia or blood dyscrasia, use fusidic acid.
430. A 45yo woman complains of pain in her hands precipitated by
exposure to the cold weather. She is breathlessness on walking. When she is
eating, she can feel food suddenly sticking to the gullet. It seems to be in the

middle of the esophagus but she cant localize exactly where it sticks. It is
usually relieved with a drink of water. Choose the single most likely cause
of dysphagia from the options?
a. Esophageal carcinoma
b. Systemic sclerosis
c. SLE
d. Pharyngeal carcinoma
e. Globus hystericus
KEY-B
We can rule out option A and D simply because she presents with systemic
complaints, and these two will cause only local signs.
*Globus hystericus is when a patient feels like they have a lump in their
throat, when infact they dont. Examination is completely normal.
*This leaves SLE and systemic sclerosis. In SLE, there is the condition
mentioned in this question (Raynauds phenomenon) but NO DYSPHAGIA.
->Systemic sclerosis (SS) is classified into 2 types- Limited cutaneous SS
(70%) and Diffuse cutaneous SS (30%) according to extent of skin
involvement.
-Limited SS formerly called CREST syndrome
Calcinosis
Raynauds phenomenon- cardinal sign, early and very common presentation.
Esophageal dysmotility
Sclerodactyly
Telangiectasia
431. A 3yo child brought to the ED with a swelling over the left arm. XR
shows multiple callus formation in the ribs. Exam: bruises on child's back.
What is the most appropriate next step?
a. Check child protection register
b. Coagulation profile
c. Skeletal survey
d. Serum calcium
e. DEXA scan
KEY- C
Dx- This is a case of Non accidental injury (NAI) i.e. child abuse.
*The clinchers are the multiple calluses in the ribs and the bruises on the
childs back, denoting repeated trauma. The injuries are often multiple,
frequent or of different ages. Abusers almost always go to the ED and not

their family GP since the chances of meeting the same ER doctor twice is
less, hence the chance of someone detecting the abuse is less.
*Initial investigations include FBC, clotting screen, skeletal survey (X-ray
series to detect any other injuries), brain imaging and retinal exam if there is
head injury, and sexual health test. Next, check child protection register.
*Other options:
-Checking child protection register is not done until confirmation or
suspicion is made (not initial step).
-Serum calcium has no benefit here (serum sodium is sometimes checked if
Salt poisoning is suspected)
-DEXA scan has no role here since it is used for diagnosis and follow up of
osteoporosis.
432. A 35yo woman has had bruising and petechiae for a week. She has
also had recent menorrhagia but is otherwise well. Blood: Hgb=11.1,
WBC=6.3, Plt=14. What is the single most likely dx?
a. Acute leukemia
b. Aplastic anemia
c. HIV infection
d. ITP
e. SLE
KEY- D
*Patient only presents with petechiae and menorrhagia, but is othwerwise
well. Hence all other options are unlikely. Also aplastic anaemia will result
in pancytopenia, but WBCs and Hb is normal here.
*What is ITP? Immune thrombocytopenic purpura.
-Autoimmune, destruction or decreased reduction of platelets. Hence
decreased platelets.
-Classified into primary (isolated) or secondary (in association with other
disease).
>Secondary ITP causes:
_Autoimmune disorders (Antiphospholipid AB syndrome, SLE)
_Viral ( CMV, VZ, HepC, HIV)<-- Most common in children at around 6
years.
_H.pylori
_Drugs
>Presentation:Petichae, epistaxis, hematuria or menorrhagia. Rarely intracranial bleeds.
>Investigations:

FBC, peripheral blood smear. Screen for HIV, HepC and other underlying
cause.
>Treatment:
-Only if symptomatic.
-Avoid NSAIDs and aspirin.
-First line tt is Prednisolone for 3 weeks, then taper off, IVIG and give IV
anti-D in Rh +ve and non-splenectomised people.
-Second line Splenectomy. Complications- infection, bleeding, thrombosis,
relapse.
-Refractory ITP- Romiplostim and Eltrombopag (thrombopoetin receptor
agonists)
433. A 30yo man complains of episodes of hearing music and sometimes
threatening voices within a couple of hours of heavy drinking. What is the
most likely dx?
a. Delirium tremens
b. Wernickes encephalopathy
c. Korsakoffs psychosis
d. Alcohol hallucinosis
e. Temporal lobe dysfunction
KEY- D
*Alcohol withdrawal presents in the following stages:
-Minor withdrawal symptoms- [Appear 6-12 hours after alcohol has
stopped.] Insomnia, tremors, mild anxiety, mild agitation or restlessness,
nausea, vomiting, headache, excessive sweating, palpitations, anorexia,
depression and craving.
-Alcohol hallucinosis- Visual, auditory or tactile hallucinations that can
occur either during acute intoxication or withdrawal. During withdrawal,
they [occur 12-24 hours after alcohol has stopped.]
-Withdrawal seizures are generalized tonic-clonic seizures that [appear 2448 hours after alcohol has stopped.]
-Delirium tremens appears [48-72 hours after alcohol has stopped]. Altered
mental status in the form of confusion, delusions, severe agitation and
hallucinations. Seizures can occur. Examination might reveal stigmata of
chronic alcoholic liver disease.
>Investigation: FBC, LFTs, clotting, ABG to look for metabolic acidosis,
Glucose, blood alcohol levels, U&E, creatinine, amylase, CPK and blood

culture. CXR to check for aspiration pneumonia. CT scan if seizures or


evidence of head trauma. ECG-arrhythmia.
>Management of alcohol withdrawal-ABC
-Treat hypoglycemia
-Sedation: Benzodiazepine (chlordiazepoxide). Alternative- diazepam.
-Carbamezapine or Mg if history of withdrawal seizures.
-IV Thiamine to prevent or treat Wernickes encephalopathy that might lead
to korsakoff syndrome.
*Wernickes encephalopathy- Triad of ataxia, ophthalmoplegia and mental
confusion). If left untreated, leads to Korsakoffs syndrome (Wernickes
plus confabulation, antero or retrograde amnesia and telescoping of events)
>Investigations: FBC (^MCV), LFTs, Glucose, U&E (^Na, ^Ca,
^Uricaemia), ABG (^Carbia and Hypoxia), Serum thiamine (low).
434. A pt had TIA which he recovered from. He has a hx of stroke and
exam shows HR in sinus rhythm. He is already on aspirin 75mg and antiHTN drugs. What other action should be taken?
a. Add clopidogrel only
b. Increase dose of aspirin to 300mg
c. Add warfarin
d. Add clopidogrel and statin
e. Add statin only
KEY- D
TIA Prophylaxis: Aspirin, clopidogrel and statin.
TIA ttt: Aspirin and dypiridamole.
435. A 40yo woman suddenly collapsed and died. At the post-mortem
autopsy, it was found that there a bleed from a berry aneurysm from the
circle of Willis. In which space did the bleeding occur?
a. Subarachnoid
b. Subdural
c. Extradural
d. Subparietal
e. Brain ventricles
KEY- A.
Berry (or saccular) aneurysms are found in the circle of willis which is found
in the subarachnoid space. They are the most common form of cerebral
aneurysms. They present with sudden severe headache and gold standard for
diagnosis is CT. Gold standard for treatment is surgical clipping, done after

restoration of respiration and reduction of ICP. Berry aneurysms are often


associated with APCKD.

436. A schizophrenic pt hears people only when he is about to fall asleep.


What is the most likely dx?
a. Hypnopompic hallucinations
b. Hyponogogic hallucinations
c. Hippocampal hallucinations
d. Delirious hallucinations
e. Auditory hallucinations
KEY- B
Hypnopompic hallucinations- While waking up.
Hyponogogic hallucinations- While falling asleep.
Hippocampal hallucination- Photographic, animated or film-like clarity of
people, animals, faces, flowers, insects etc.
Auditory hallucinations- hearing voices that arent present.
437. A pt who came from India presents with cough, fever and enlarged
cervical LN. Exam: caseating granulomata found in LN. What is the most
appropriate dx?
a. Lymphoma
b. TB adenitis

c. Thyroid carcinoma
d. Goiter
e. Thyroid cyst
KEY- B
Points in favour- Traveling to India, cough, LN and caseating granulomata,
which is unique for TB.
438. A 44yo man comes with hx of early morning headaches and
vomiting. CT brain shows ring enhancing lesions. What is the single most
appropriate option?
a. CMV
b. Streptococcus
c. Toxoplasmosis
d. NHL
e. Pneumocystis jerovii
KEY- C
*Causes of ring enhancing lesions on CT brain:
-Brain abscess
-Primary or secondary tumour
-CNS lymphoma
-CNS toxoplasmosis
-Nocardia infection.
>Out of the options, toxoplasmosis is the right answer, and it is commonly
found in HIV patients. TREATMENT with pyrimethamine/sulfadiazine and
folinic acid. OR clindamycin if intolerant FOR 4-6 WEEKS.
If immunocompromised, PROPHYLAXIS with
Trimethoprim+sulfamethoxazole.
439. A 72yo man is found to be not breathing in the CCU with the
following rhythm. What is the most likely dx?
a. SVT
b. VT
c. VF
d. Atrial fib
e. Atrial flutter
KEY- C
VFib- Chaotic depolarisation of ventricles. Atrial rate 60-100. Ventricular
rate 400-600. Irregular. Ttt by immediate defibrillation

VTach- Sequence of 3 or more ventricular beats. Atrial rate 60-100.


Ventricular rate 110-250. Regular. Can progress to VFib and cardiac arrest.
Ttt if pulse present, cardioversion. If pulseless, defibrillation.
440. A 65yo man with difficulty in swallowing presents with an
aspiration pneumonia. He has a bovine cough and fasciculating tongue.
Sometimes as he swallows food it comes back through his nose. Choose the
single most likely cause of dysphagia from the given option?
a. Bulbar palsy
b. Esophageal carcinoma
c. Pharyngeal pouch
d. Pseudobulbar palsy
e. Systemic sclerosis
KEY- A
Bulbar palsy - Relates to medulla. Affection of lower cranial nerves (VIIXII). Dysphagia, dysphonia, dysarthria, tremulous lips, FASICULATIONS.
Pseudobulbar palsy - Affection of corticobulbar tracts. Dysphagia,
dysphonia. Donald duck speech, unable to protrude tongue. NO
FASICULATIONS
441. A 16yo teenager was brought to the ED after being stabbed on the
upper right side of his back. Erect CXR revealed homogenous opacity on the
lower right lung, trachea was centrally placed. What is the most probable
explanation for the XR findings?
a. Pneumothorax
b. Hemothorax
c. Pneumonia
d. Tension pneumothorax
e. Empyema
KEY- B
Sharp stabbing wound- Hemothorax. Clincher- homogenous opacity; not
seen with pneumothorax. Also since trachea is not displaced from the centre,
it is simple, not tension hemothorax.
Treatment- Chest drain insertion in the 5th intercostal space, mid-axillary
line.
For tension hemo/pneumothorax, needle thoracostomy insertion in the 2nd
intercostal space, mid-clavicular line.
442. A 55yo woman complains of retrosternal chest pain and dysphagia
which is intermittent and unpredictable. The food suddenly sticks in the

middle of the chest, but she can clear it with a drink of water and then finish
the meal without any further problem. A barium meal shows a corkscrew
esophagus. What is the single most likely dysphagia?
a. Esophageal candidiasis
b. Esophageal carcinoma
c. Esophageal spasm
d. Pharyngeal pouch
e. Plummer-vinson syndrome
KEY- C.
**Esophageal spasm- Oesophageal motility disorder. Dysphagia,
regurgitation and chest pain. corkscrew oesophagus on Barium swallow Xray. Ttt- Nitroglycerin, CCB, PPI. Botulinum toxin, balloon dilatation.

**Plummer vinson syndrome- triad of iron deficiency, esophegeal webs and


dysphagia. Premalignant - squamous cell carcinoma of oesophegus. Also
presents with cheilitis, koilonychia, glossitis and splenomegaly. Patient
complains of burning sensation in tongue and oral mucosa. Ttt is iron
supplementation and endoscopic dilation for webs
**Oesophageal candidiasis- Immunocompromised like HIV or renal
transplant. Odynophagia, with oral thrush. maybe weight loss. Ttt

fluconazole for atleast 21 days or atleast 14 days after disappearance of


symptoms.
**Oesophageal carcinoma- Dysphagia to colod foods then later to liquids.
Weight loss, hoarseness of voice(if involving the recurrent laryngeal nerve),
hematemesis, hemoptysis, nausea and vomiting.
Risk factors- smoking and unhealthy diet.
Diagnosis- Endoscopy and biopsy.
Treatment- Surgery, radio and chemotherapy depending on stage.
**Pharyngeal pouch (Zenckers diverticulum)- Common above 70. M:F is
5:1.
Presentation: Dysphagia, regurgitation, aspiration, chronic cough and weight
loss.Neck lump that gurgles on palpation. Halitosis from food decaying in
the pouch. Investigation: Barium swallow shows residual contrast pool
within the pouch. Aspiration from the pouch might cause inhalation
pneumonia. Ttt cricopharyngeal myotomy.
443. A 38yo female presents with sudden loss of vision but fundoscopy is
normal. She a similar episode about 1 y ago which resolved completely
within 3m. Exam: mild weakness of right upper limb and exaggerated
reflexes. What is the single most appropriate tx?
a. Pan retinal photo coagulation
b. Pilocarpine eye drops
c. Corticosteroids
d. Peripheral iridectomy
e. Surgical extraction of lens
KEY- C
> This is a case of optic neuritis caused by Multiple sclerosis. Steroids are
the answer here. They are given during acute symptomatic attacks of MS.
During relapse or remission, disease modifying agents like interferons are
given.
> Pan retinal photocoagulation is done for diabetic retinopathy where parts
on the retina are burned in order to reduce the Oxygen demand.
Lens extraction is done mainly for cataract to remove the opacified lens that
disturbs the vision
> Peripheral iridectomy is done by making a hole in the iris for open angle
glaucoma in order to provide an alternative drainage for the fluid
accumulating inside the eye, thus decreasing the IOP.

> Pilocarpine is a parasympathomimetic given for open angle glaucoma in


order to contract the ciliary muscles and to open the trabecular meshwork,
allowing increased outflow of the aqueous humour
>Surgical extraction of the lens is done for cataract where the opacified lens
that disturbs the vision is removed

444. A 15yo boy presents with a limp and pain in the knee. Exam: leg is
externally rotated and 2cm shorter. There is limitation of flexion, abduction
and medial rotation. As the hip is flexed external rotation is increased.
Choose the most likely dx?
a. Juvenile rheumatoid arthritis
b. Osgood-schlatter disease
c. Reactive arthritis
d. Slipped femoral epiphysis
e. Transient synovitis of the hip
KEY- D
> Slipped femoral epiphysis- Fracture through the growth plate (physis),
which results in slippage of the overlying end of the femur (epiphysis).
Symptoms include gradual, progressive onset of thigh or knee pain with a
painful limp. Hip motion will be limited, particularly internal rotation.
> Osgood Schlatter disease- Inflammation of the patellar ligament at the
tibial tuberosity. Painful lump just below the knee, often seen in young
adolescents. Risk factors- overuse (especially in sports involving running,
jumping and quick changes of direction) & adolescent growth spurts.
> Reactive arthritis or Reiter's syndrome- Autoimmune reaction to an
infection somewhere else in the body. Triad- arthritis, uveitis,
urethritis\cervicitis
445. A 64yo woman has difficulty moving her right shoulder on
recovering from surgery of the posterior triangle of her neck. What is the
single most appropriate option?
a. Accessory nerve
b. Glossopharyngeal nerve
c. Hypoglossal nerve
d. Vagus nerve
e. Vestibule-cochlear nerve
KEY- A

446. A 37yo man with an ulcer on the medial malleolus. Which of the
following LN is involved?
a.
b.
c.
d.
e.
f.
g.
h.

External iliac LN
Pre-aortic LN
Aortic LN
Inguinal LN
Iliac LN
Submental LN
Submandibular LN
Deep cervical LN

447. A pt presents with weight loss of 5kgs despite good appetite. He also
complains of palpitations, sweating and diarrhea. He has a lump in front of
his neck which moves on swallowing. What is the most appropriate dx?

a. Lymphoma
b. TB adenitis
c. Thyroid Ca
d. Goiter
e. Thyroid cyst
KEY- D
Typical symptoms of hyperthyroidism- Weight loss, palpitations, sweating,
diarrhoea. Goiter lump moves with swallowing.
Thyroglossal cyst moves upwards on tongue protrusion
Thyroid cancer usually presents as a painless, hard and FIXED thyroid mass
enlarging rapidly over a period of a few weeks.
448. A 76yo woman has become tired and confused following an
influenza like illness. She is also breathless with signs of consolidation of
the left lung base. What is the most likely dx?
a. Drug toxicity
b. Delirium tremens
c. Infection toxicity
d. Hypoglycemia
e. Electrolyte imbalance
KEY- C
Infection toxicity is also called Toxic shock syndrome. It is the case here
because of the history of preceding flu-like illness which points towards
toxins (enterotoxin type B) from Staphylococcus aureus]. There is also
consolidation of the lung which is most probably due to the Staph
pneumonia.
Delirium tremens is due to alcohol withdrawal and it usually occurs at
around day 3 of cessation of alcohol intake.
No other choice fits this scenario.
449. A young pt is complaining of vertigo whenever she moves sideways
on the bed while lying supine. What would be the most appropriate next
step?
a. Head roll test
b. Reassure
c. Advice on posture
d. Carotid Doppler
e. CT
KEY- A

Dx? Benign Paroxysmal Positional Vertigo (BPPV)- Most common cause of


vertigo. Vertigo triggered by change in head position. Might be
accompanied by nausea and nystagmus. Less commonly, vomiting and
syncope.
Diagnosis: Dix-Hallpike and Head roll test.
Management: Epley and Semont Maneuver.
450. A 32yo man has OCD. What is the best tx?
a. CBT
b. SSRI
c. TCA
d. MAO inhibitors
e. Reassure
KEY- A
OCD is treated initially with individual CBT (Cognitive Behavioural
therapy) plus exposure and response prevention. If symptoms become severe
or do not improve, SSRIs like fluoxetine or Citalopram etc are introduced.
Recent studies have shown that there is no superiority of one over the other
(CBT over SSRIs), but CBT remains the initial management plan, This
question is quite deficient, and the original key is B. SSRI, but Im sure in
the exam, it will be more detailed; but this is how OCD is managed.
Reference: Patient.co.uk. Link- http://patient.info/doctor/obsessivecompulsive-disorder-pro
451. A 65yo woman says she died 3m ago and is very distressed that
nobody has buried her. When she is outdoors, she hears people say that she
is evil and needs to be punished. What is the most likely explanation for her
symptoms?
a. Schizophrenia
b. Mania
c. Psychotic depression
d. Hysteria
e. Toxic confusional state
KEY- C
Psychotic depression consists of a major depressive episode plus psychotic
symptoms like hallucinations or delusions (in this case nihilistic delusions).
Toxic confusional state can be eliminated since there is no history of
infection.
452. A 50yo woman presents following a fall. She reports pain and
weakness in her hands for several months , stiff legs, swallowing difficulties,

and has bilateral wasting of the small muscles of her hands. Reflexes in the
upper limbs are absent. Tongue fasciculations are present and both legs
show increased tone, pyramidal weakness and hyper-reflexia with extensor
plantars. Pain and temp sensation are impaired in the upper limbs. What is
the most likely dx?
a. MS
b. MND
c. Syringobulbia
d. Syringomyelia
e. Myasthenia gravis
KEY- C
In MS, there are characteristic relapse and remission which is absent here.
MND is purely motor, there is no sensory deficit; In myasthenia gravis there
is muscular weakness without atrophy.
Syringomyelia is a condition in which there is fluid-filled tubular cyst
(syrinx) within the central, usually cervical, spinal cord. The syrinx can
elongate, enlarge and expand into the grey and white matter and, as it does
so, it compresses the nervous tissue of the corticospinal and spinothalamic
tracts and the anterior horn cells. This leads to various neurological
symptoms and signs, including pain, paralysis, stiffness and weakness in the
back, shoulders and extremities. It may also cause loss of extreme
temperature sensation, particularly in the hands, and a cape-like loss of pain
and temperature sensation along the back and arms.
** If the syrinx extends into the brainstem, syringobulbia results. This may
affect one or more cranial nerves, resulting in facial palsies. Sensory and
motor nerve pathways may be affected by interruption and/or compression
of nerves.
453. Which of the following formulas is used for calculating fluids for burn
pts?
a. 4 x weight(lbs) x area of burn = ml of fluids
b. 4 x weight(kgs) x area of burn = L of fluids
c. 4 x weight(kgs) x area of burn = ml of fluids
d. 4 x weight(lbs) x area of burn = L of fluids
e. 4.5 x weight(kgs) x area of burn = dL of fluids
KEY- C
>Burns are injuries caused by thermal, chemical, electrical or radiation
energy.

Start with ABCs. Establish the time of the injury- from the time the injury
happened, not from the time the patient presents. Give strong analgesia. Rule
out Non accidental injury. Avoid hypothermia.
>Fluid Requirements = Body area burned(%) x Wt (kg) x 4mL. This is
called Parkland formula. Give 1/2 of total requirements in 1st 8 hours, then
give 2nd half over next 16 hours. Area of body burn is calculated by addition
of percentage of burn in each area, by rule of 9s:
9% head and neck, 9% each upper limb, 18% each lower limb, 18% front of
trunk, 18% back of trunk, 1% Palmar surface of the hand, including fingers,
1% Perineum

454. A 65yo male presents with dyspnea and palpitations. Exam:


pulse=170bpm, BP=120/80mmHg. Carotid massage has been done as first
instance. What is the next step of the management?
a. Adenosine
b. Amilodipine
c. DC cardioversion
d. Lidocaine
e. Beta blocker
KEY- A
Likely diagnosis SVT. Initially, vagal manoeuvres, if fails iv adenosine.
Vagal manoeuvres (carotid sinus massage, Valsalva manoeuvre)
transiently increase AV block, and may unmask an underlying atrial rhythm.
If unsuccessful, give adenosine, which causes transient AV block.
455. A 48yo farmer presented with fever, malaise, cough and SOB.
Exam: tachypnea, coarse end-inspiratory crackles and wheeze throughout,
cyanosis. Also complaint severe weight loss. His CXR shows fluffy nodular
shadowing and there is PMN leukocytosis. What is the single most
appropriate dx?
a. Ankylosing spondylitis
b. Churg-strauss syndrome
c. Cryptogenic organizing
d. Extrinsic allergic alveolitis
e. Progressive massive fibrosis
KEY- D
Dx- Farmers lung/ Hypersensitivity penumonitis/ Extrinsic allergic
penumonitis.
It is diffuse granulomatous inflammation of the lung in patients who are
allergic to organic antigens present in dust particles. On chest X-ray, diffuse
nodular opacities are seen.
456. A 35yo lady is admitted with pyrexia, weight loss, diarrhea and her
skin is lemon yellow in color. CBC = high MCV. What is the most probably
dx?
a. Aplastic anemia
b. Pernicious anemia
c. Leukemia
d. ITP
e. Lymphoma
KEY- B

Clincher- High MCV. It may be graves with pernicious anemia. Lemon


yellow pallor occurs in pernicious anemia. Hyperthyroidism may cause
persistently raised body temperature
457. A 72yo woman who had a repair of strangulated femoral hernia 2
days ago becomes noisy, aggressive and confused. She is febrile, CBC
normal apart from raised MCV. What is the most likely dx?
a. Electrolyte imbalance
b. Delirium tremens
c. Wernickes encephalopathy
d. Infection toxicity
e. Hypoglycemia
KEY- B
Delirium tremens occurs after alcohol withdrawal, usually 3 to 4 days after
cessation of alcohol. Altered mental status in the form of confusion,
delusions, severe agitation and hallucinations. Seizures can occur.
Examination might reveal stigmata of chronic alcoholic liver disease.
Alcohol also typically raises MCV.
Wernickes encephalopathy- Triad of ataxia, ophthalmoplegia and mental
confusion). If left untreated, leads to Korsakoffs syndrome (Wernickes
plus confabulation, antero or retrograde amnesia and telescoping of events)
Electrolyte imbalance may cause confusion but not aggressiveness.
Infection toxicity will cause high fever, low BP, rash etc which is absent
here.
Hypoglycemia can occur with alcohol intake but it does not present this way.
It presents with sweating, pallor, shakiness etc.
458. An old lady had UTI and was treated with antibiotics. She then
developed diarrhea. What is the single most likely tx?
a. Co-amoxiclav
b. Piperacillin + tazobactam
c. Ceftriaxone
d. Vancomycin
KEY- D
This is a case of pseudomembraneous colitis. It is caused by Clostridium
difficile. It occurs after use of antibiotics. Treated with Vancomycin or
Metronidazole.
459. A 56yo man has symptoms of sleep apnea and daytime headaches
and somnolence. Spirometry shows a decreased tidal volume and vital
capacity. What is the single most appropriate dx?

a. Ankylosing spondylitis
b. Churg-strauss syndrome
c. Good pasture syndrome
d. Motor neuron disease
e. Progressive massive fibrosis
f.
Spinal cord compression
KEY- D
Involvement of respiratory muscles in Motor Neuron Disease is associated
with poor respiration causing sleep apnoea.
460. A 55yo man presents with mild headache. He has changed his
spectacles thrice in 1 yr. there is mild cupping present in the disc and sickle
shaped scotoma present in both eyes. What is the single most appropriate tx?
a. Pan retinal photo coagulation
b. Pilocarpine eye drops
c. Corticosteroids
d. Scleral buckling
e. Analgesics alone
KEY- B
> Sickle-shaped scotoma or siedel sign is often seen in glaucoma. That along
with the fact that he keeps changing his spectacles denotes that this is a case
of progressive open angle glaucoma. It can also present with nausea,
vomiting, headache and ocular pain. Treated with Carbonic anhydrase
inhibitors like acetazolamide, Miotic agents (parasympathomimetics) such
as pilocarpine, Alpha2-adrenergic agonists like brimonidine, or
Prostaglandin analogs like latanoprost.
> Pan retinal photocoagulation is done for diabetic retinopathy where parts
on the retina are burned in order to reduce the Oxygen demand.
> Scleral buckling is done for retinal detachment to put the retina back in
place.
461. A 55yo woman was found collapsed at home, paramedics revived
her but in the ambulance she had a cardiac arrest and couldnt be saved. The
paramedics report tells that the woman was immobile lately due to hip pain
and that they found ulcers on the medial side of ankle. She had DM and was
on anti-diabetics. What is the cause of her death?
a. Acute MI
b. DKA
c. Pulmonary embolism
d. Acute pericarditis

e. Cardiac tamponade
KEY- C
This is a case of collapse due to PE following DVT caused by the patients
immobilization due to hip pain.
Cardiac tamponade- Triad of hypotension, distended engorged neck veins,
and muffled JVP.
Pericarditis- Chest pain worse with inspiration and lying down, relieved by
lying forward.
No history supporting DKA or MI.
462. An 18yo previously well student is in his 1 year at uni. He has been
brought to the ED in an agitated, deluded and disoriented state. What is the
most probable reason for his condition?
a. Drug toxicity
b. Delirium tremens
c. Infection toxicity
d. Electrolyte imbalance
e. Head injury
KEY- A
Clinchers are teenage, and 1st year of university, where students tend to
experiment with drugs.
Infection toxicity can be ruled out due to lack of any signs of infection like
fever. Lack of history of trauma rules out head injury, and delirium tremens
is due to alcohol withdrawal.
st

463. A young adult presents to the ED after a motorcycle crash. The pt


has bruises around the left orbital area. GCS=13, examination notes
alcoholic breath. Shortly afterwards, his GCS drops to 7. What is the single
most important initial assessment test?
a. MRI brain
b. CT brain
c. CXR
d. CT angio brain
e. Head XR
KEY- B
This is a typical case of Epidural hematoma. It is usually due to trauma, and
has a period of lucidity before collapse. Due to the sudden drop in GCS, CT
brain should be immediately done.
464. A 30yo female attends OPD with a fever and dry cough. She says
that she had headache, myalgia and joint pain like one week ago. Exam:

pulse=100bpm, temp=37.5C. CXR: bilateral patchy consolidation. What is


the single most likely causative organism?
a. Pneumococcal pneumonia
b. Legionella
c. Mycoplasma
d. Klebsiella
e. Chlamydia pneumonia
KEY- C
> Mycoplasma pneumonia- Atypical pneumonia. Slow onset, dry cough,
pleuritic pain, myalgia, arthralgia, malaise.
> Legionella- history of travel and stay in hotel- atypical symptoms plus GI
manifestations.
> Klebsiella- commonly associated with alcohol.
465. A 46yo man is being investigated for indigestion. Jejunal biopsy
shows deposition of macrophages containing PAS (Periodic acid-schiff) +ve
granules. What is the most likely dx?
a. Bacterial overgrowth
b. Celiac disease
c. Tropical sprue
d. Whipples disease
e. Small bowel lymphoma
KEY- D
Periodic acid-schiff positive granules containing macrophages in jejunal
biopsy is diagnostic of whipples disease.
Coeliac disease is gluten sensitivity.
466. A 32yo woman of 38wks gestation complains of feeling unwell with
fever, rigors and abdominal pains. The pain was initially located in the
abdomen and was a/w urinary freq and dysuria. The pain has now become
more generalized specifically radiating to the right loin. She says that she
has felt occasional uterine tightening. CTG is reassuring. Select the most
likely dx?
a. Acute fatty liver of pregnancy
b. Acute pyelonephritis
c. Round ligament stretching
d. Cholecystitis
e. UTI
KEY- B
This is a case of UTI followed by ascending infection leading to
pyelonephritis. Fever, rigors and abdominal pain are typical symptoms.

467. A 32yo pt presents with cervical lymphadenopathy and


splenomegaly. What is the single most appropriate option?
a. Hemophilus
b. Streptococcus
c. Toxoplasmosis
d. NHL
e. Pneumocystis jerovcii
KEY- D
Non Hodgkins lymphoma is the only option here that will have both
lymphadenopathy and splenomegaly (although splenomegaly is not a
common presentation). Pneumocystis jerovici and Toxoplasmosis are
common in HIV patients.
468. A 62yo man who was admitted for surgery 3days ago suddenly
becomes confused. His attn span is reduced. He is restless and physically
aggressive and picks at his bed sheets. What single aspect of the pts hx
recovered in his notes is most likely to aid in making the dx?
a. Alcohol consumption
b. Head trauma
c. Hx of anxiety
d. Prescribed med
e. Obvious cognitive impairment
KEY- A
This is a typical case of Delirium tremens. It appears [48-72 hours after
alcohol has stopped]. Altered mental status in the form of confusion,
delusions, severe agitation and hallucinations. We should ask alcohol
history. Examination might reveal stigmata of chronic alcoholic liver
disease.
469. A 10yo girl presents with pallor and features of renal failure. She
has hematuria as well as proteinuria. The serum urea and creat are elevated.
These symptoms started after an episode of bloody diarrhea 4 days ago.
What is the most probable dx?
a. TTP
b. HUS
c. ITP
d. HSP
e. ARF
KEY- B

Haemolytic Uraemic Syndrome (HUS) is a triad of Haemolytic anaemia,


thrombocytopaenia and Renal failure. It is said to be caused most commonly
by E.coli O:157H7 which binds to endothelial receptors in the GIT, Renal
and central nervous system. Symptoms [ abdominal pain, pallor due to
anaemia, hematuria and proteinuria, features of renal failure likenausea/vomiting, swelling of face, hand, feet or entire body etc. and elevated
urea and creatinine etc.] start around two weeks after an episode of bloody
diarrhea. The diarrheoa is charactised to get bloody after 1-3 days. This
scenario is typical for HUS. It is also known to be precipitated by strept
pneumonia and some drugs like cyclosporin and tacrolimus.
470. A 40yo woman has had intermittent tension, dizziness and anxiety
for 4 months. Each episode usually resolves after a few hours. She said she
takes alcohol to make her calm. She is in a loving relationship and has no
probs at work or home. What is the next step in her management?
a. Collateral info
b. CT brain
c. CBC
d. LFT
e. TFT
KEY- A Collateral info. Likely diagnosis is panic disorder. Collateral info
from family, friends & other peers should be asked to find out the cause for
her anxiety.
471. A 45yo IV drug abuser is brought into the ED with complaint of
fever, shivering, malaise, SOB and productive cough. Exam: temp=39C,
pulse=110bpm, BP=100/70mmHg. Inv: CXR=bilateral cavitating
bronchopneumonia. What is the single most likely causative organism?
a. Mycoplasma
b. Staphylococcus
c. Chlamydia pneumonia
d. Pseudomonas
e. PCP
KEY- B
Staphylococcus and PCP are common in IV drug abusers. Both are also
recognized cause of cavitating pneumonia. This case is with productive
cough which goes more with staphylococcus as PCP is not productive, but is
rather associated with dry cough.
Mycoplasma pneumonia- Atypical pneumonia. Slow onset, dry cough,
pleuritic pain, myalgia, arthralgia, malaise.

472. A 71yo woman looks disheveled, unkempt and sad with poor eye
contact. She has recently lost her husband. Which of the following describes
her condition?
a. Anxiety
b. Hallucination
c. Mania
d. High mood
e. Low mood
KEY- E
Dx- Depression. Disheveled and unkempt because she doesnt take care of
herself, plus the loss of her husband, points towards depression.
473. A 62yo male comes to the GP complaining of double vision while
climbing downstairs. Which of the following nerve is most likely involved?
a. Abducens nerve
b. Trochlear nerve
c. Oculomotor nerve
d. Optic nerve
e. Trigeminal nerve
KEY- B
This is a lesion in the Trochlear nerve affecting the Superior oblique muscle.
All extrinsic muscles of the eye are supplied by the Oculomotor nerve
except the Lateral rectus by the Abducens nerve and the Superior oblique by
the trochlear (mnemonic LAST).
Oculomotor nerve affection causes palsy of inferior rectus, medial rectus and
superior rectus manifesting as double vision in multiple gaze. But trochlear
involving superior oblique only causes diplopia in downgaze only.

474. L1 level, what is the most appropriate landmark?


a. Mcburneys point
b. Stellate ganglion
c. Deep inguinal ring
d. Termination of the spinal cord
e. Transpyloric plane

KEY- E

475. A 32yo woman presents to the ED with headache and vomiting. She
was decorating her ceiling that morning when the headache began, felt
mainly occipital with neck pain. Some 2hs later she felt nauseated, vomited
and was unable to walk. She also noticed that her voice had altered. She
takes no reg meds and has no significant PMH. Exam: acuity, field and fundi
are normal. She has upbeat nystagmus in all directions of gaze with normal
facial muscles and tongue movements. Her uvulas deviated to the right and
her speech is slurred. Limb exam: left arm past-pointing and
dysdiadochokinesia with reduced pin prick sensation in her right arm and
leg. Although power is normal, she cant walk as she feels too unsteady.
Where is the most likely site of lesion?
a. Right medial medulla
b. Left medial pons
c. Left cerebellar hemisphere
d. Right lateral medulla
e. Left lateral medulla
KEY- E
Lateral medullary syndrome affects:
-Contralateral spinothalamic tract (loss of pain and temperature on the
opposite side of the body)
-Ipsilateral Sympathetic tract- Horners syndrome.
-Ipsilateral Spinal trigeminal nucleus (loss of pain,temperature and corneal
reflex on same side of the face)
-Nucleus ambigous- Dysphagia and Dysarthria
-Inferior cerebellar peduncle- Ataxia
Ipsilateral Cranial nerves- IX, X and XI (dysphagia, loss of gag reflex, palate
paralysis)
Cause- Occlusion of PICA (posterior inferior cerebellar artery)
Medial medullary syndrome affects:
Contralateral corticospinal tract/pyramids- weakness of arms and legs
opposite side.
Contralateral Medial lemniscus/dorsal column- loss of proprioception and
vibration.
Ipsilateral hypoglossal nerve- weakness of tongue on the same side.
[Hypoglossal nerve affection manifests as protrusion of the tongue to the
side of the weakness while at rest, it deviates to the contralateral side)
Cause- Occlusion of Anterior spinal artery.
476. A 28yo female presents with 1 wk hx of jaundice and 2d hx of
altered sleep pattern and moods. She was dx with hypothyroidism for which

she is receiving thyroxine. TFT showed increased TSH. PT=70s. What is the
most probable dx?
a. Acute on chronic liver failure
b. Hyper-acute liver failure
c. Autoimmune hepatitis
d. Acute liver failure
e. Drug induced hepatitis
KEY- C
Autoimmune hepatitis may present as acute hepatitis, chronic hepatitis, or
well-established cirrhosis. Autoimmune hepatitis rarely presents as
fulminant hepatic failure. One third may present as acute hepatitis marked
by fever, hepatic tenderness and jaundice. Non specific features are
anorexia, weight loss and behavioural change (here altered sleep pattern and
moods). There may be coagulopathy (here PT=70s.) leading to epistaxis,
gum bleeding etc. Presence of other autoimmune disease like
hypothyroidism supports the diagnosis of autoimmune hepatitis.
477. A 55yo man has a chronic cough and sputum, night sweats and
weight loss. What is the single most likely causative organism?
a. Coagulase +ve cocci in sputum
b. Gram -ve diplococci in sputum
c. Gram +ve diplococci in sputum
d. Pneumocystis carinii in sputum
e. Sputum staining for mycobacterium tuberculosis
KEY- E
Classic features of TB- Chronic cough and sputum, night sweats and weight
loss. Organism is Acid fast bacilli mycobacterium tuberculosis.
478. A 20yo pregnant 32wks by date presents to the antenatal clinic with
hx of painless vaginal bleeding after intercourse. Exam: P/A soft and
relaxed, uterus=dates, CTG=reactive. Choose the single most likely dx?
a. Abruption of placenta 2 to pre-eclampsia
b. Antepartum hemorrhage
c. Placenta previa
d. Preterm labor
e. Placenta percreta
KEY- C
Clincher- painless bleeding, typical presentation of placenta previa. Uterus is
soft and relaxed and theres no pain, so we rule out placental abruption.
nd

479. A 30yo man presents to the ED with difficulty breathing. He has


returned from India. Exam: throat reveals grey membranes on the tonsils and
uvula. He has mild pyrexia. What is the single most relevant dx?
a. Diphtheria
b. IM
c. Acute follicular tonsillitis
d. Scarlet fever
e. Agranulocytosis
KEY- A
Clinchers- History of travel to India, and greyish membrane.
Infectious mononucleosis will typically present in a teenager, with enlarged
cervical lymphadenopathy and fever.
Acute follicular tonsillitis as the name suggests, will have follicles on the
tonsils.
Scarlet fever presents with rash and strawberry tongue
480. A 23yo man comes to the ED with a hx of drug misuse. He
recognizes that he has a prb and is willing to see a psychiatrist. Which of the
following terms best describes this situation?
a. Judgement
b. Thought insertion
c. Thought block
d. Mood
e. Insight
KEY- E
Insight is the patient's awareness and understanding of the origins and
meaning of his attitudes, feelings, and behavior and of his disturbing
symptoms, basically, he is aware that he has a problem.
481. A pt with hodgkins lymphoma who is under tx develops high fever.
His blood results show WBC <2800 and has a chest infection. Choose the
most likely tx?
a. Co-amoxiclav
b. Piperacillin+tazobactam
c. Erythromycin
d. Piperacillin+Co-amoxiclav
e. Penicillin+tazobactam
KEY- B

This patient with Hodgkins lymphoma has a severe infection and his WBC
count is very low, so he needs to be covered with broad spectrum antibiotics,
hence piperacillin and tazobactam.
482. A 25yo woman presents with urinary freq, dysuria and fever. Urine
microscopy shows 20-50 RBC and 10-20 WBC in each field. What is the
most probable dx?
a. Schistosmiasis
b. Kidney trauma
c. Ureteric calculus
d. Bladder calculi
e. Cystitis
KEY- E
Clincher- WBCs.
These are typical Symptoms of UTI- fever, frequency, dysuria. Urine
microscopy here (hematuria and presence of WBCs) indicate cystitis.
Schistosomiasis can present with hematuria and fever but also with
additional symptoms like diarrhea, abdominal pain, hepatosplenomegaly,
cough and history of travel would likely be given.
Kidney trauma can present with hematuria, but not dysuria, frequency and
fever.
Calculi can present with severe lower abdominal and back pain, difficult
urination, frequency, fever, dysuria and haematuria. The pain, that comes in
waves, may also be associated with nausea, vomiting and chills. WBCs
though, will not be present.
483. A 65yo presents with dyspareunia after sex. She in menopause. She
complains of bleeding after sex. What is the most probably dx?
a. Cervical ca
b. Endometrial ca
c. Ovarian ca
d. Breast ca
e. Vaginal ca
KEY- B
RULE- Postmenopausal bleeding, be it post coital or not, is Endometrial
cancer unless proven otherwise.
Clinchers- Post menopausal, and age 65.
Cervical cancer is common in women aged 25-34 years, while 90% of
women with endometrial cancer are over 50 years of age. [Sourcepatient.co.uk]

484. A 45yo man underwent an emergency splenectomy following a fall


from his bicycle. He smokes 5 cigarettes/day. Post-op, despite mobile, he
develops swinging pyrexia and a swollen painful left calf. His CXR shows
lung atelectasis and abdominal US demonstrates a small subphrenic
collection. What is the single most likely risk factor for DVT in this pt?
a. Immobility
b. Intraperitoneal hemorrhage
c. Smoking
d. Splenectomy
e. Sub-phrenic collection
KEY- D
Since it is stated that patient is mobile. option A can be eliminated. Option B
and E are not known to predispose to Thromboembolism. This leaves C and
D, and splenectomy is a stronger link to DVT due to it being:
1. A recent major surgery(within 12 weeks), which is a big risk factor
for DVT.
2. The surgery itself- Vascular events after splenectomy are likely
multifactorial, probably resulting from some combination of
hypercoagulability, platelet activation, disturbance and activation of
the endothelium, and altered lipid profiles. The spleen's primary
phagocytic function is to remove infectious organisms, other insoluble
cellular debris, and senescent or abnormal red cells and platelets. This
filtration function results from the blood moving slowly through the
splenic sinusoids in the red pulp lined with macrophages actively
ingesting that which does not easily pass around them. Absence of this
extremely sensitive filter may permit particulate matter and damaged
cells to persist in the bloodstream, therefore perturbing and activating
the vascular endothelium leading to a shift in vascular homeostasis
toward enhanced coagulation.
485. A 6m baby had LOC after which he had jerky movement of hands
and feet. What is the most probable dx?
a. Infantile spasm
b. Absence
c. Partial simple seizure
d. Atonic seizure
e. Partial complex
KEY- E
> Generalised- Entire body is involved.
> Focal/Partial- Not the entire body is involved.

> Complex- Loss of consciousness.


> Simple- No loss of consciousness.
> Absence seizures- The person has a brief loss of consciousness (an
absence) for a few seconds. They do not fall but may pause in what they are
doing. Their face often looks pale with a blank expression. They may look
dazed, the eyes stare and the eyelids may flutter a little. Sometimes their
head may fall down a little, or their arms may shake once or twice. Each
seizure usually starts and finishes abruptly. The person is not aware of the
absence and resumes what they were doing.
> Infantile spasm (West Syndrome) occurs in the first year of life (3-8
months) and is not associated with LOC. Its nature is more generalized
rather than the focal nature described here. Infantile spasm can also be called
salaam spasms, because the appearance of the seizures is like a bowing
forwards or backwards movement.
486. A 24yo primigravida who is 30wk pregnant presents to the labor
ward with a hx of constant abdominal pain for the last few hours. She also
gives a hx of having lost a cupful of fresh blood per vagina before the pain
started. Abdominal exam: irritable uterus, CTG=reactive. Choose the single
most likely dx?
a. Abruption of placenta 2 to pre-eclampsia
b. Antepartum hemorrhage
c. Placenta previa
d. Vasa previa
e. Revealed hemorrhage
KEY- B
Presentation indicates abruption of the placenta, but not confirmed yet.
Generally bleeding during this time is given a general diagnosis of
antepartum haemorrhage. There is no history or features suggestive of of
hypertension or pre-eclampsia so A is not the choice. Abruption can be
either concealed or revealed abruption.
Placenta praevia is painless bleeding.
487. A 62yo lady presents with right sided headache and loss of vision.
What is the single most inv?
a. ESR
b. BUE
c. CT head
d. XR orbit
e. IOP
KEY- A
nd

> This is most probably Giant cell arteritis/Temporal arteritis. It is common


in females and elderly people and should always be considered in cases of
new-onset headache in patients 50 years of age or older. Initial investigation
is ESR which will be raised (>40mm/hr), and confirmatory diagnosis is
temporal artery biopsy. Patient should be started on steroids immediately if
GCA is suspected, even if diagnosis is not confirmed, as delay in treatment
might lead to blindness due to occlusion of the ophthalmic artery.
> IOP (Intraocular pressure) is used to investigate glaucoma.
488. A 24yo man asks his GP for a sick note from work. He says that
feels down, is lethargic and has stopped enjoying playing the piccolo (his
main hobby). He was admitted to the psychiatry ward last year following an
episode of overspending, promiscuity and distractibility. What is the most
probable dx?
a. Psychosis
b. Cyclothymia
c. Bipolar affective disorder
d. Seasonal affective disorder
KEY- C
> Features of Mania (overspending, promiscuity and distractibility) plus
Depression (low mood, lethargy and anhedonia) denote Bipolar Disorder.
> Cyclothymia is a mild form of Bipolar disorder [Mild depression without
somatic symptoms + Hypomania] that often goes unnoticed.
489. A 42yo female who is obese comes with severe upper abdominal
pain with a temp=37.8C. She has 5 children. What is the most probable dx?
a. Ectopic pregnancy
b. Ovarian torsion
c. Hepatitis
d. Biliary colic
e. Cholecystitis
KEY- E
> This is cholecystitis, or non-alcoholic steatohepatitis. The 5 Fs of
cholecystitis are- Fat
Female
Fare
Forty
Fertile.
> Ovarian torsion and ectopic pregnancy will have lower abdominal pain.

490. A child has just recovered from meningitis. What inv will you do
before discharge?
a. CT scan
b. EEG
c. Blood culture
d. Repeat LP
e. Hearing test
KEY- E
** Patient is already recovering from meningitis, so none of options A, B, C
or D are indicated.
** Since hearing loss is the most common complication of meningitis,
people recovering from the condition will usually have a hearing test. The
test should be carried out before you're discharged, or within 4 weeks of
being well enough to have the test. Children and young people should
discuss the results of their hearing test with a paediatrician between 4 and 6
weeks after being discharged from hospital. In cases where hearing is
severely affected, cochlear implants may be needed. [NICE Guidelines and
NHS].
> Complications of Meningitis:
*Immediate: septic shock, DIC, coma with loss of protective airway
reflexes, cerebral oedema and raised ICP, septic arthritis, pericardial
effusion and haemolytic anaemia (H. influenzae).
Subdural effusions: reported in 40% of children aged 1-18 months with
bacterial meningitis.
Syndrome of inappropriate antidiuretic hormone secretion (SIADH).
Seizures
*Delayed: decreased hearing, or deafness; other cranial nerve dysfunction,
multiple seizures, focal paralysis, subdural effusions, hydrocephalus,
intellectual deficits, ataxia, blindness, Waterhouse-Friderichsen syndrome
and peripheral gangrene.
491. A primiparous woman with no prv infection with herpes zoster is
18wk pregnant. She had recent contact with a young 21yo pt having
widespread chicken pox. What is the most suitable management for the
pregnant lady?
a. Acyclovir PO
b. Acyclovir IV +IVIG
c. Acyclovir IV
d. Reassure
e. IVIG

KEY- E
*If the pregnant woman is not immune to VZV and she has had a significant
exposure, she should be offered varicella-zoster immunoglobulin (VZIG) as
soon as possible. VZIG is effective when given up to 10 days after contact
(in the case of continuous exposures, this is defined as 10 days from the
appearance of the rash in the index case).
*If she had no previous infection and develops a rash (got infected) and
comes within 24 hour of development of rash- acyclovir is given. [MRCOG
Guideline].
492. A 40yo woman presents to the GP with low mood. Of note, she has
an increased appetite and has gone up 2 dress sizes. She also complains that
she cant get out of bed until the afternoon. What is the most likely dx?
a. Pseudo depression
b. Moderate depression
c. Severe depression
d. Dysthymia
e. Atypical depression
KEY- E
Atypical depression is a subtype of major depression or dysthymic disorder
that involves several specific symptoms, including increased appetite or
weight gain, hypersomnia, marked fatigue or weakness, moods that are
strongly reactive to environmental circumstances, and feeling extremely
sensitive to rejection, or feeling of being weighed down, paralyzed, or
"leaden.
493. An 8yo boy is clinically obese. As a baby he was floppy and
difficult to feed. He now has learning difficulties and is constantly eating
despite measures by his parents to hide food out of reach. What is the most
probable dx?
a. Cushings syndrome
b. Congenital hypothyroidism
c. Prader Willi syndrome
d. Lawrence moon biedel syndrome
e. Downs syndrome
KEY- C
Prader Willi syndrome- congenital disorder caused by defect in gene on
chromosome 15. Characterized by hypotonia at birth, feeding difficulties,
poor growth and delayed development. At the beginning of childhood, they
present with obsessive eating and obesity, learning difficulties, behavioural

problems and compulsive behavior such as picking on skin. Distinctive


facial features like triangular mouth, unusually fair skin and light-coloured
hair, almond shaped eyes and short forehead.
494. A 20yo lady is suffering from fever and loss of appetite. She has
been dx with toxoplasmosis. What is the tx?
a. Pyrimethamine
b. Pyrimethamine + sulfadiazine
c. Clindamycin
d. Spiramycin
e. Trimethoprim + sulfamethoxazole
KEY- B
Toxoplasmosis:
TREATMENT with pyrimethamine/sulfadiazine and folinic acid. OR
clindamycin if intolerant FOR 4-6 WEEKS.
If immunocompromised, PROPHYLAXIS with
Trimethoprim+sulfamethoxazole.
495. A 68yo woman has a sudden onset of pain and loss of hearing in her
left ear and unsteadiness when walking. There are small lesions visible on
her palate and left external auditory meatus. What is the single most likely
dx?
a. Acute mastoiditis
b. Cholesteatoma
c. Herpes zoster infection
d. Oropharyngeal malignancy
e. OM with infusion
KEY- C
Clincher- lesions which are probably vesicles.
Herpes zoster oticus (Ramsay Hunt syndrome) occurs when latent varicella
zoster virus reactivates in the geniculate ganglion of the 7th cranial nerve.
Symptoms: Painful vesicular rash on the auditory canal on drum, pinna,
tongue, palate or iris with ipsilateral facial palsy, loss of taste, vertigo,
tinnitus, deafness, dry mouth and eyes. OHCM 9th edition, page 505.
496. A 45yo woman has been dx with Giant Cell A and is being treated
with steroids. What is the other drug that can be added to this?
a. ACEi
b. Beta blockers
c. Aspirin

d. Interferons
e. IVIG
KEY- C
For GCA, along with Steroids, 3 other medications are required:
1). Low-dose aspirin: Start aspirin 75 mg daily unless there are contraindications - eg, active peptic ulceration or a bleeding disorder. Low-dose
aspirin has been shown to decrease the rate of visual loss and strokes in
patients with GCA.
2). Start gastroprotection with a proton pump inhibitor in view of added risk
of peptic ulceration with high-dose steroids and aspirin.
3). Start Osteoporosis prophylaxis with bisphosphonates since patient in on
long-term steroid treatment.
497. A 17yo man has acute pain and earache on the right side of his face.
Temp=38.4C and has extensive pre-auricular swelling on the right, tender on
palpation bilaterally. What is the single most likely dx?
a. Acute mastoiditis
b. Acute otitis externa
c. Acute OM
d. Mumps
e. OM with effusion
KEY- D
> Mumps- Prodromal malaise, fever, painful parotid swelling, becoming
bilateral in 70%. OHCS 9th edition, page 142.
> Otitis externa typically presents after swimming, and involves only the
external ear. Treated with topical antibiotic drops (aminoglycosides) and
acetic acid 2% ear drops.
> Otitis media will involve the tympanic membrane and the pain will be in
the ear, not pre-auricular. Treated with analgesics and antipyretics for adults,
and antibiotics for children.
498. An ECG of an elderly lady who collapsed in the ED shows rapid
ventricular rate of 220 bpm, QRS=140ms. What is the most probable dx?
a. Atrial fibrillation
b. VT
c. SVT
d. Mobitz type1 2 degree heart block
e. Sinus tachycardia
KEY- B
nd

VTach- Sequence of 3 or more ventricular beats. Atrial rate 60-100.


Ventricular rate 110-250. Wide QRS complex (>120ms). Regular. Can
progress to VFib and cardiac arrest. Ttt- if pulse present, cardioversion. If
pulseless, defibrillation.
SVT and AFib will have narrow QRS complexes.

499. A pt presents with purple papular lesions on his face and upper trunk
measuring 1-2 cm across. They arent painful or itchy. What is the single
most likely dx?
a. Kaposis sarcoma
b. Hairy leukoplakia
c. Cryptosporidium
d. CMV infection
e. Cryptococcal infection
KEY- A
Kaposis sarcoma is a spindle-cell tumour derived from capillary endothelial
cells or from fibrous tissue, caused by human herpes virus. It presents as non
painful purple papules ( to 1 cm) or plaques on skin and mucosa (any
organ). It is not itchy, and it metastasizes to nodes. Associated with AIDS
infection. OHCM 9th edition, page 716.
500. A 6yo boy is clinically obese, his BMI >95 centile. He has no other
medical prbs, examination is unremarkable. His mother says that she has
tried everything to help him lose weight. What is the most probable dx?
a. Cushings syndrome
b. Congenital hypothyroidism
c. Downs syndrome
d. Lawrence moon biedel syndrome
e. Primary obesity
KEY- E
Features support primary childhood obesity. No other associated signs or
symptoms except obesity. Its not cushing (No moon face, pigmentation,
hyperglycaemia) etc. Its not congenital hypothyroidism, (No weight loss
despite increased appetite), not Down syndrome (No features of Down) or
Lawrence moon biedel syndromes (No learning difficulties).
th

501. A 20yo boy is brought by his parents suspecting that he has taken some drug. He is
agitated,
irritated and cant sleep. Exam: perforated nasal septum. Which of the following is the
most
likely to be responsible for his symptoms?
a. Heroine
b. Cocaine
c. Ecstasy/MDMA/amphetamine
d. Alcohol

e. Opioids
B. Cocaine
perforated nasal septum
Heroine: pinpoint pupils, dec consciousness, bradycardia, resp depression, hypoxia.
antidote: naloxone
Ecstasy/MDMA/amphetamine: agitation, anxiety, confusion, ataxia, tachycardia,
hypertension, hyponatraemia, hyperthermia, rhabdomyolysis
Mechanism of action
cocaine blocks the uptake of dopamine, noradrenaline and serotonin
The use of cocaine is associated with a wide variety of adverse effects:
Cardiovascular effects
myocardial infarction
both tachycardia and bradycardia may occur
hypertension
QRS widening and QT prolongation
aortic dissection
Neurological effects
seizures
mydriasis
hypertonia
hyperreflexia
Psychiatric effects
agitation
psychosis
hallucinations
Others

hyperthermia
metabolic acidosis
rhabdomyolysis

Management of cocaine toxicity


in general benzodiazipines are generally first-line for most cocaine related problems
chest pain: benzodiazipines + glyceryl trinitrate. If myocardial infarction develops then primary
percutaneous coronary intervention
hypertension: benzodiazipines + sodium nitroprusside
the use of beta-blockers in cocaine-induced cardiovascular problems is a controversial issue. The
American Heart Association issued a statement in 2008 warning against the use of beta-blockers
(due to the risk of unopposed alpha-mediated coronary vasospasm) but many cardiologists since
have questioned whether this is valid. If a reasonable alternative is given in an exam it is

probably wise to choose it.

502. For a pt presenting with Parkinsons disease which of the following drugs is most
useful in the management of the tremor?

a. Apomorphine
b. Cabergoline
c. Selegiline
d. Amantadine
e. Benzhexol
e. Benzhexol
Antimuscarinics
block cholinergic receptors
now used more to treat drug-induced parkinsonism rather than idiopathic Parkinson's
disease
help tremor and rigidity
e.g. procyclidine, benzotropine, trihexyphenidyl (benzhexol)
503. A 26yo woman has become aware of increasing right sided hearing deficiency
since her recent pregnancy. Her eardrums are normal. Her hearing tests show: BCnormal. Weber test lateralizes to the right ear. What is the single most likely dx?
a. Encephalopathy
b. Functional hearing loss
c. Tympano-sclerosis
d. Otosclerosis
e. Sensorineural deafness
key is D. Otosclerosis. [There are no features of encephalopathy. As Weber test is lateralized it is unlikely to
be functional hearing loss. In tympanosclerosis ear drum becomes chalky white. So as the ear drum is
normal it is not tympanosclerosis. Weber test is lateralized to right and deafness is also on the right. So it not
sensorineural deafness but conductive deafness which makes otosclerosis as the most likely diagnosis.
Rinne's test
air conduction (AC) is normally better than bone conduction (BC)
if BC > AC then conductive deafness
Weber's test
in unilateral sensorineural deafness, sound is localised to the unaffected side
in unilateral conductive deafness, sound is localised to the affected side

504. A 58yo T1DM on anti-HTN therapy for 13yrs developed central chest pain for 45
mins while
driving a/w cold sweating and dyspnea. What is the single most appropriate dx?
a. MI
b. Pericarditis
c. Pulmonary embolism
d. Costochondritis
e. Pneumothorax
a. MI

characteristic central or epigastric chest pain radiating to the arms, shoulders, neck, or jaw.
The pain is described as substernal pressure, squeezing, aching, burning, or even sharp pain.
Radiation to the left arm or neck is common.
Chest pain may be associated with sweating, nausea, vomiting, dyspnoea, fatigue, and/or
palpitations.

Pericarditis chest pain: may be pleuritic. Is often relieved by sitting forwards


Pulmonary embolism pleuritic chest pain, dyspnoea and haemoptysis
Costochondritis Chest wall pain with a history of repeated minor trauma or unaccustomed activity
(eg, painting, moving furniture) is common

505. A man was brought to the ED from a shopping mall after collapsing there. He is
conscious and answering questions now. His ECG shows irregular rhythm. Your choice
of inv:
a. CT
b. MRI
c. 24h ECG
d. Echo
d. Echo
The man had a syncopial attack ..the most valvular cause for it is aortic stenosis which needs an
Echo to diagnose it or if there is any other valvular lesion or ventricular dysfunction
go for echo just to exclude any structural abnormalities.
holter- ecg is already said to be irregular. we already know that there is an rhythm problem so no
use to holter.
If in history something indicated towards TIA or stroke then CT or MRI would be considered.

506. A 10yo boy is clinically obese and the shortest in his class. He had a renal
transplant last year and his mother is worried that he is being bullied. What is the most
probable dx?
a. Cushings syndrome
b. Congenital hypothyroidism
c. Pseudocushings syndrome
d. Lawrence moon biedel syndrome
e. Downs syndrome
a. Cushings syndrome
he's on steroids post-renal transplant, Oral steroids is the chief cause of Cushing's syndrome (OHCM, 8th,
page 124).
Laurance-moon synd. Night blindness due to retinitis pigmentosa, polydactyly are important features
(OHCS/8th/648). With no emphasis on more common features, Oral-steroid induced (post renal transplant)
Cushing makes more sense.
Congenital hypothyroidism Feeding difficulties, Somnolence, Lethargy, Low frequency of crying,
Constipation
Downs syndrome he is clinically obese not conganital case,down syndrome has cardaic problem and
characteristic facial feature and mentalyy retarded so it cant be option,these features are same for lawrence
moon but ptnt are mentally retarded whereas kid is studying in normal school rather than special one
Pseudocushings syndromeit is mainly an idiopathic condition.Some frequently occurring illnesses can
induce a phenotype that largely overlaps with Cushing syndrome and is accompanied by hypercorticolism

507. A 45yo man had cancer of head of pancreas which has been removed. He has a hx
of
longstanding heartburn. He now comes with rigid abdomen which is tender, temp 37.5C,
BP=90/70mmHg, pulse=120bpm. What is the next step of the inv?
a. CT abdomen

b. XR abdomen
c. MRI abdomen
d. US abdomen
e. Endoscopy
b. XR abdomen
Long standing Heart burn - peptic ulcer disease resulting into hollow viscous perforation leading to
gas under diaphragm in x-ray abdomen!
where are all those sign of acute pancreatitis in this case except hypotension and tachycardia which can
occur with perotinitis.suppose its pancreatitis ,do u think cT is nxt step after presentation? Nxt step must be
serum amylase and lipase and cT is most accurate.

Laparoscopy has become a routine procedure in the management of acute


abdominal disease

508. A 50yo man presents to the ED with acute back pain radiating down to his legs.
Pain which is usually relieved by lying down and exacerbated by long walks and prolong
sitting. What inv
would be the best option?
a. MRI
b. CT spine
c. XR spine
d. Dual energy XR abruptiometry
e. Serum paraprotein electrophoresis
a. MRI
Diagnosis most likely Prolapsed Intervertebral Disc. Sudden onset acute back pain radiating down the leg,
and it is relieved on lying down and exacerbated by prolonged walks and on coughing and moving the back.
Investigation done is MR Spine to look for prolapsed disc and nerve root compression.
Never think of CT in case of spinal cord compression
mri... better for visualisin soft tissue.. ct better if bony detail is desired.. this is lumbar degenerative disc
disease most likely due to a herniated nucleus pulposus at l4/5 or l5/s1

Lumbosacral disc herniation: (patient.co.uk)


If there is nerve entrapment in the lumbosacral spine, this leads to symptoms of
sciatica which include:
o Unilateral leg pain that radiates below the knee to the foot/toes.
o The leg pain being more severe than the back pain.
o Numbness, paraesthesia, weakness and/or loss of tendon reflexes,
which may be present and are found in the same distribution and
only in one nerve root distribution.
o A positive straight leg raising test (there is greater leg pain and/or
more nerve compression symptoms on raising the leg).
o Pain which is usually relieved by lying down and exacerbated by
long walks and prolonged sitting.
o MRI is very sensitive in showing disc herniations

Management
Simple analgesics as first line
Pain due to a herniated lumbosacral disc may settle within six weeks. If it does not, or there

are red flag signs such as the possibility of cauda equina syndrome, referral to an
orthopaedic or neurosurgeon should be considered.

509. What is the most appropriate antibiotic to treat uncomplicated chlamydial infection
in a 21yo
female who isnt pregnant?
a. Erythromycin
b. Ciprofloxacin
c. Metronidazole
d. Cefixime
e. Doxycycline
e. Doxycycline
Chlamydia is the most prevalent sexually transmitted infection in the UK.
Management
doxycycline (7 day course) or azithromycin (single dose). The 2009 SIGN guidelines
suggest azithromycin should be used first-line due to potentially poor compliance
with a 7 day course of doxycycline
if pregnant then erythromycin or amoxicillin may be used.
Potential complications

epididymitis
pelvic inflammatory disease
endometritis
increased incidence of ectopic pregnancies
infertility
reactive arthritis
perihepatitis (Fitz-Hugh-Curtis syndrome)

510. A 45yo manual worker presented with a 2h hx of chest pain radiating to his left arm.
His ECG is normal. What is the single most appropriate inv?
a. Cardiac enzymes
b. CXR
c. CT
d. ECG
e. V/Q scan
a. Cardiac enzymes
to rule out NSTEMI.

Non-ST-elevation ACS (NSTE-ACS): patients present with acute chest pain but without
persistent ST-segment elevation. The ECG shows persistent or transient ST-segment
depression or T-wave inversion, flat T waves, pseudo-normalisation of T waves, or no ECG
changes at presentation.
Management:
All patients should receive
aspirin 300mg
nitrates or morphine to relieve chest pain if required

Antithrombin treatment. Fondaparinux should be offered to patients who are not at a high risk of bleeding
and who are not having angiography within the next 24 hours. If angiography is likely within 24 hours or a
patients creatinine is > 265 mol/l unfractionated heparin should be given.

Clopidogrel 300mg should be given to all patients and continued for 12 months.

Intravenous glycoprotein IIb/IIIa receptor antagonists (eptifibatide or tirofiban) should be given to


patients who have an intermediate or higher risk of adverse cardiovascular events (predicted 6-month
mortality above 3.0%), and who are scheduled to undergo angiography within 96 hours of hospital
admission.

Coronary angiography should be considered within 96 hours of first admission to hospital to patients who
have a predicted 6-month mortality above 3.0%. It should also be performed as soon as possible in patients
who are clinically unstable.

511. A 26yo woman had bipolar disorder for 10yrs and is on Lithium for it. She is
symptom free for
the past 4 years. She is now planning her pregnancy and wants to know whether she
should
continue taking lithium. What is the single most appropriate advice?
a. Continue lithium at the same dose and stop when pregnancy is confirmed
b. Continue lithium during pregnancy and stop when breast feeding
c. Reduce lithium dosage but continue throughout pregnancy
d. Reduce lithium gradually and stop when pregnancy is confirmed
e. Switch to sodium valproate
d. Reduce lithium gradually and stop when pregnancy is confirmed
symptom free for last 4 years. Lithium is teratogenic.
Adverse effects

nausea/vomiting, diarrhoea
fine tremor
polyuria (secondary to nephrogenic diabetes insipidus)
thyroid enlargement, may lead to hypothyroidism
ECG: T wave flattening/inversion
weight gain

Monitoring of patients on lithium therapy


inadequate monitoring of patients taking lithium is common - NICE and the National Patient Safety
Agency (NPSA) have issued guidance to try and address this. As a result it is often an exam hot
topic
lithium blood level should 'normally' be checked every 3 months. Levels should be taken 12 hours
post-dose
thyroid and renal function should be checked every 6 months
Pregnancy: avoid in first the trimester (teratogenic). Only use in the second and third trimester if
considered essential, ie a severe risk to the patient, and monitor levels closely, as dose
requirements may alter.
Breast-feeding: avoid, as present in milk, and there is risk of toxicity in an infant. Bottle-feeding is
advisable.

Withdrawal
Abrupt withdrawal (both because of poor compliance or rapid change in dose) can precipitate
relapse. Withdraw lithium slowly over several weeks, watching for relapse.

512. A pt presents with dysphagia and pain on swallowing. He has sore mouth and
soreness in the corners of the mouth. What is the single most likely dx/
a. Kaposis sarcoma
b. Molluscum contagiosum
c. CMV infection
d. Candida infection
e. Toxoplasma abscess
d. Candida infection
Pain on swallowing is classic for candida.
Kaposi's sarcoma
caused by HHV-8 (human herpes virus 8)
presents as purple papules or plaques on the skin or mucosa (e.g. gastrointestinal and respiratory
tract)
skin lesions may later ulcerate
respiratory involvement may cause massive haemoptysis and pleural effusion
radiotherapy + resection
Molluscum contagiosum
The majority of cases occur in children (often in children with atopic eczema), with the maximum incidence
in preschool children aged 14 years.characteristic pinkish or pearly white papules with a central umbilication,
which are up to 5 mm in diameter. Lesions appear in clusters in areas anywhere on the body (except the
palms of the hands and the soles of the feet).

CMV infection
Infection is worldwide and usually asymptomatic . The most common disease manifestation is
gastrointestinal disease. CMV pneumonia is the most serious complication, but has become less common
with prevention strategies for at-risk patients. Rare manifestations include retinitis and encephalitis.

ToxoplasmaThis is asymptomatic in most cases.Toxoplasmic chorioretinitis.Toxoplasmic


encephalitis.Pneumonitis.Multiorgan involvement with respiratory failure and shock.

513. A 30yo lady has epistaxis for 30mins. Her Hgb is normal, MCV normal, WBC
normal,
PT/APTT/Bleeding time are normal. Where is the defect?
a. Plts
b. Coagulation factor
c. Sepsis
d. Anatomical
e. RBC
d. Anatomical
All labs normal.

Trauma to the nose (the most common cause) - especially nose picking!
Insertion of foreign bodies and excessive nose blowing may also be seen as
trauma. The latter is likely to occur with a cold when the nasal mucosa is
congested. Sinusitis causes nasal congestion.
Disorders of platelet function - thrombocytopenia and other causes of abnormal
platelets, including splenomegaly and leukaemia. Waldenstrm's
macroglobulinaemia may present with nosebleeds. Idiopathic thrombocytopenic
purpura (ITP) can occur in children and young adults.
Drugs - aspirin and anticoagulants.
Disorders of platelets are more likely to be a problem than clotting factor deficiency.
Abnormalities of blood vessels in the elderly arteriosclerotic vessels prolong
bleeding. Hereditary haemorrhagic telangiectasia (Osler-Rendu-Weber syndrome)
causes recurrent epistaxis from nasal telangiectases.
Malignancy of the nose may present with bleeding - juvenile angiofibroma is a
highly vascular benign tumour that typically presents in adolescent males.
Cocaine use - if the septum looks sloughed or atrophic ask about use of cocaine.
Other conditions - Wegener's granulomatosis and pyogenic granuloma can present
as an epistaxis.

514. Midpoint between the suprasternal notch and pubic symphysis. What is the single
most
appropriate landmark?
a. Fundus of the gallbladder
b. Mcburneys point
c. Stellate ganglion
d. Deep inguinal ring
e. Transpyloric plane
e. Transpyloric plane
An upper transverse, the transpyloric, halfway between the jugular notch and the upper border of the
symphysis pubis; this indicates the margin of the transpyloric plane, which in most cases cuts through the
pylorus, the tips of the ninth costal cartilages and the lower border of the first lumbar vertebra;

Fundus located at the tip of the 9 costal cartilage


th

Mcburneys point one-third of the distance from the anterior superior iliac spine to the umbilicus

Stellate ganglion located at the level of C7 (7th cervical vertebrae), anterior to the transverse process
of C7, superior to the neck of the first rib, and just below the subclavian artery

Deep inguinal ring immediately above the midpoint of the inguinal ligament (midway
between the anterior superior iliac spine and the pubic tubercle)

Structures crossed
The transpyloric plane is clinically notable because it passes through several important
abdominal structures. These include:
lumbar vertebra 1 and hence passes just before the end of the spinal cord in
adults.
the fundus of the gallbladder
the end of the spinal cord
the Neck of pancreas

the origin of the superior mesenteric artery from the abdominal aorta and
termination of the superior mesenteric vein at the hepatic portal vein
the left and right colic flexure
hilum of the kidney on the left
hilum of the kidney on the right
the root of the transverse mesocolon
duodenojejunal flexure
the 1st part of the duodenum
the upper part of conus medullaris
the spleen
the pylorus of the stomach which will lie at this level approximately 5 cm to the
right of the midline.
cisterna chyli (which drains into the thoracic duct)

515. Tip of the 9th costal cartilage. What is the single most appropriate landmark?
a. Fundus of the gallbladder
b. Deep inguinal ring
c. Termination of the spinal cord
d. Transpyloric plane
e. Vena cava opening in the diaphragm
a. Fundus of the gallbladder
spinal cord around the L1/L2 vertebral level, forming a structure known as the conus medullaris.

Apertures through the diaphragm


1. Vena caval hiatus (vena caval foramen)
at the level of T8 and transmits the IVC and occasionally the phrenic nerve.
2. Esophageal hiatus
at the level of T10 and transmits the esophagus and vagus nerves.
3. Aortic hiatus
at the level of T12 and transmits the aorta, thoracic duct, azygos vein, an
occasionally greater splanchnic nerve.

516. A child complains of RIF pain and diarrhea. On colonoscopy, granular transmural
ulcers are seen near the ileo-cecal junction. What should be the management?
a. Sulfasalazine
b. Oaracetamol
c. Ibuprofen
d. Metronidazole
a. Sulfasalazine
Crohns disease-transmural ulcers

metronidazole is often used for isolated peri anal disease


Remission
glucocorticoids (oral, topical or intravenous) are generally used to induce remission.
5-ASA drugs (e.g. mesalazine) are used second-line to glucocorticoids but are not as
effective

stopping smoking is a priority (remember: smoking makes Crohn's worse, but may help
ulcerative colitis)
Maintenance
azathioprine or mercaptopurine is used first-line to maintain remission
methotrexate is used second-line
517. A 60yo woman presents with acute onset of bone and back pain following a rough
journey in a car. Exam: tenderness at mid-thoracic vertebra with spasm, she feels better
once she bends
forward. What is the single most probable dx?
a. Osteoporotic fx verterbra
b. Myofacial pain
c. Whiplash injury
d. MI
e. Pancreatitis
b. Myofacial pain
Myofascial pain syndrome typically occurs after a muscle has been contracted
repetitively.
Only myofascial pain/muscle sprain relieves on change of position.
Whiplash is only for cervical vertebrae
osteoporotic fx has dull pain and persistent or even worse on movement.
After repeated contraction of a muscle there occur spasm of the muscle and often the
pain may felt in some other part (referred pain). Here rough journey and associated
spasm is clincher. In vertebral fracture you will find neurological features which is absent
here.
Spinal Stenosis pain also Relieved by sitting down, leaning forwards and crouching
down

518. A 70yo woman presents with recurrent episodes of parotid swelling. She complains
of difficulty in talking and speaking and her eyes feel gritty on waking in the morning.
What is the single most likely dx?
a. C1 esterase deficiency
b. Crohns disease
c. Mumps
d. Sarcoidosis
e. Sjogrens syndrome
e. Sjogrens syndrome
main symptoms of xerophthalmia (dry eyes), xerostomia (dry mouth) and enlargement of the
parotid glands.
Difficulty eating dry food, typically cracker biscuits.
Difficulty with dentures.
Complaint of the tongue sticking to the roof of the mouth.
Speaking for long periods of time causes hoarseness.
Oral candidiasis and angular cheilitis.
Dry eyes tend to cause a gritty sensation. There is a predisposition to blepharitis and the
eyes may be sticky in the morning.

There may be recurrent parotitis, usually bilateral. Glands are usually enlarged
but this is not often the presenting feature.
Dryness of the mucosa of the trachea and bronchi may present as a dry cough.
Dryness of the pharynx and oesophagus may cause difficulty in swallowing, and
lack of saliva and secretions may predispose to gastro-oesophageal reflux.
There can be dry skin and vaginal dryness causing dyspareunia
Disease of the pancreas can lead to malabsorption and even acute pancreatitis
or chronic pancreatitis but a more likely cause of elevated serum amylase is
parotitis.
Fatigue is a common feature.
About 20% have Raynaud's phenomenon.

Associated diseases
There may be a number of associated autoimmune conditions, such as the variant of
scleroderma: calcinosis, Raynaud's phenomenon, (o)esophageal motility disorder,
sclerodactyly and telangiectasia (CREST). There may be joint pain, swelling and fatigue
rrecurrent miscarriage with antiphospholipid syndrome.

Investigations

Rheumatoid factor
Antinuclear antibodies
Schirmer test

519. A 39yo woman has not had her period for 10months. She feels well but is anxious
as her mother had an early menopause. Choose the single most appropriate initial inv?
a. Serum estradiol conc.
b. Serum FSH/LH
c. Serum progesterone conc.
d. None
e. Transvaginal US
b. Serum FSH/LH
Premature Menopause (ovarian faililure)shoud be ruled out.So FSH and LH(very high)
Premature Ovarian Failure (menopause before 40 years of age).

climacteric symptoms: hot flushes, night sweats


infertility
secondary amenorrhoea
raised FSH, LH levels
serum estradiol reflects primarily the activity of the ovaries. useful in the detection of
baseline estrogen in women with amenorrhea or menstrual dysfunction, and to detect the
state of hypoestrogenicity and menopause.
Serum progesterone: indicates if failure to ovulate
7 days prior to expected next period
520. A 50yo man with DM suddenly develops persistent crushing central chest pain
radiating to the neck. What is the single most appropriate dx?
a. Angina
b. Costochondritis (tietzs disease)

c. Dissecting aneurysm
d. MI
e. Pulmonary embolism
c. Dissecting aneurysm
Pain can radiate to back (classically described in questions) or to the neck as well. MI is an important
differential but usually MI in diabetics is silent one.
Angina does not radiate, costochondritis mostly have localised pain
In aortic dissection, pain is abrupt in onset and maximal at the time of onset. In contrast, the pain associated
with acute myocardial infarction starts slowly and gains in intensity with time. It is usually more oppressive
and dull.
Although tearing is the classical description, the pain is described as sharp more often than tearing, ripping,
or stabbing.

Investigations
Often the first problem is to distinguish aortic dissection from myocardial infarction. Both conditions may
exist if the dissection involves the coronary ostium. For this reason, the electrocardiogram (ECG) is very
important.

Best: MRI
521. A 22yo man has rushed into the ED asking for help. He describes recurrent
episodes of
fearfulness, palpitations, faintness, hyperventilation, dryness of the mouth with peri-oral
tingling and cramping of the hands. His symptoms last 5-10 mins and have worsened
since their
onset 3months ago. He is worried he may be having a heart attack. An ECG shows
sinus
tachycardia. What is the single most appropriate immediate intervention?
a. High flow oxygen
b. IV sedation
c. Rebreathe into a paper bag
d. Refer for anxiety management course
e. Refer for urgent cardiology opinion
c. Rebreathe into a paper bag

Characteristic symptoms experienced during


panic attacks
Panic disorder manifests as the sudden, spontaneous and unanticipated occurrence of panic
attacks, with variable frequency, from several in a day to just a few per year:
Palpitations, pounding heart or accelerated heart rate.
Sweating.
Trembling or shaking.
Dry mouth.
Feeling short of breath, or a sensation of smothering.
Feeling of choking.
Chest pain or discomfort.
Nausea or abdominal distress.

Feeling dizzy, unsteady, light-headed or faint.


Derealisation or depersonalisation (feeling detached from oneself).
Fear of losing control or 'going crazy'.
Fear of dying.
Numbness or tingling sensations.
Chills or hot flushes.

522. An 8yo boy has longstanding asthma. He has admitted with a severe episode and
is tired and drowsy. He has not improved on oxygen, inhaled B2 agonist and IV
hydrocortisone. CXR shows bilateral hyperinflation. He is too breathless to use a
peakflow meter and is O2 sat <90%. What is the single most appropriate inv?
a. CBG
b. CXR
c. CT chest
d. Pulse oximetry
e. Spirometry
a. CBG
cbg=capillary blood gas, abg= arterial blood gas any one can be used.
ABG to assess the severity to guide oxygen therapy and to decide if intubation is needed or not.

Clinical assessment of the severity of an acute asthma attack in those aged over 2
years
Acute severe
Unable to complete sentences in one breath.
Unable to feed or talk.
Pulse >125 in those aged over 5 years or
>140 in 2- to 5-year-olds.
Respiratory rate >30 in those aged over 5
and >40 in 2- to 5-year-olds.

Life-threatening
Silent chest.
Cyanosis.
Poor respiratory
effort.
Hypotension.
Exhaustion.
Confusion.
Coma.

523. A man was operated for colorectal ca. His pain is relieved with morphine 60mg bd
PO but now he cant swallow medications. What will be the next regimen of analgesic
administration?
a. Oxycodone
b. Fentanyl patch
c. Morphine 60mg IV/d
d. Morphine 240mg IV/d
b. Fentanyl patch

A regular 4-hourly starting dose for opioid-naive patients is usually 5-10 mg


morphine.
Once pain relief is at a satisfactory and stable level, sustained-release
preparations can be substituted to allow od or bd dosing
Any breakthrough pain not associated with unusual activity should be treated
with morphine elixir or ordinary tablets at 1/6 total daily dose.
the breakthrough dose of morphine is one-sixth the daily dose of morphine
When increasing the dose of opioids the next dose should be increased by 3050%.
If vomiting, dysphagia or increasing weakness prevent patients from taking oral
morphine then usual practice is to convert to a subcutaneous infusion of opioid
via a device such as a syringe driver.( in whom oral opioids are not suitable and
analgesic requirements are unstable) Injection site should be changed every 2-3
days.
An alternative to both oral morphine and subcutaneous diamorphine in patients with
stable pain is transdermal fentanyl or buprenorphine patches. (in whom oral opioids
are not suitable and analgesic requirements are stable) They can be useful in
ambulatory patients where the following exist:
-Problems with the oral route.
-Intractable constipation or subacute obstruction.
-Morphine intolerance.
Consider if agitated confusion is due to opioid toxicity rather than uncontrolled pain
before giving further opioids. Consider switching to an alternative strong opioid.
Alternatives include hydromorphone, methadone and oxycodone
Consider changing the route of administration - eg where gastrointestinal absorption
is poor, consider switching to skin patches.
opioids should be used with caution in patients with chronic kidney disease.
Alfentanil, buprenorphine and fentanyl are preferred

Oral to oral route conversions[2]

Converting

Converting to:

Divide 24-hour dose of current opioid by figure


below to calculate initial 24-hour dose of new opioid

from:
(new opioid)
(current
opioid)

oral codeine

oral morphine

Divide by 10

oraltramadol

oral morphine

Divide by 5

oral morphine

oral oxycodone

Divide by 2

oral morphine

oralhydromorphone

Divide by 7.5

524. Just above the mid-inguinal point. What is the single most appropriate landmark?
a. Femoral artery pulse felt
b. Mcburneys point
c. Stellate ganglion
d. Deep inguinal ring
e. Transpyloric plane
d. Deep inguinal ring

525. 5th ICS in the ant axillary line. What is the single most appropriate landmark?
a. Apex beat
b. Chest drain insertion
c. Stellate ganglion
d. Transpyloric plane
e. Vena cava opening into the diaphragm
b. Chest drain insertion
Surface anatomy of Apex beat is left 5th ICS midclavicular line.

526. A 34yo man with MS has taken an OD of 100 tablets of paracetamol with intent to
end his life. He has been brought to the ED for tx but is refusing all intervention.
a. Assessment
b. Evaluate pts capacity to refuse tx
c. Establish if pt has a prv mental illness
b. Evaluate pts capacity to refuse tx

Urgent treatment

Consent not needed when urgent treatment is required:

To save the patient's life.


To prevent a serious deterioration in the patient's condition, so long as the
treatment is not irreversible.
To alleviate serious suffering so long as the treatment is neither irreversible nor
hazardous.
To prevent the patient from behaving violently or being a danger to self or others
so long as the treatment is neither irreversible nor hazardous, and represents
the minimum interference necessary.

527. A 23yo woman with painless vaginal bleeding at 36wks pregnancy otherwise
seems to be
normal. What should be done next?
a. Vaginal US
b. Abdominal US
c. Vaginal exam
d. Reassurance
b. Abdominal US
to assess fetal being and check placenta previa
vaginal US is more accurate but not initial when bleeding.
No PV until no PP
Placenta previa
Painless bleeding starting after the 28th week (although spotting may occur earlier) is usually
the main sign.

Acute bleeding
Admit the patient to hospital.
DO NOT PERFORM A VAGINAL EXAMINATION, as this may start torrential bleeding in the
presence of placenta praevia.

Blood loss is assessed and cross-matched for possible transfusion.


Resuscitation if indicated; the mother is the priority and should be stabilised prior
to any assessment of the fetus.
Appropriate surgical intervention may be required:
o In severe bleeding the baby is delivered urgently whatever its
gestational age.
o Hysterectomy should also be considered in severe cases.
If immediate delivery is not likely, maternal steroids may be indicated in order to
promote fetal lung development and reduce the risk of respiratory distress
syndrome and intraventricular haemorrhage.

528. A 29yo lady admitted with hx of repeated UTI now developed hematuria with loin
pain. What is the most probable dx?
a. Acute pyelonephritis
b. Chronic pyelonephritis
c. UTI
d. Bladder stone
a. Acute pyelonephritis

Presentation
Onset is usually rapid with symptoms appearing over a day or two. There is unilateral or
bilateral loin pain, suprapubic or back pain. Fever is variable but can be high enough to
produce rigors. Malaise, nausea, vomiting, anorexia and occasionally diarrhoea occur.
There may or may not be accompanying lower urinary tract symptoms with frequency,
dysuria, gross haematuria or hesitancy. Gross haematuria occurs in 30-40% of young
women. The patient looks ill and there is commonly pain on firm palpation of one or both
kidneys and moderate suprapubic tenderness without guarding.
Investigation of choice:
Contrast-enhanced helical/spiral CT (CECT) scan is the best investigation in adults
In children, the choice is between ultrasound and CT scanning. CT is more sensitive but
the exposure to radiation may make ultrasound a safer option.
Treatment: ciprofloxacin for seven to ten days

529. A 45yo chronic smoker attends the OPD with complaints of persistent cough and
copious
amount of purulent sputum. He had hx of measles in the past. Exam: finger clubbing and
inspiratory crepitations on auscultation. What is the single most likely dx/
a. Interstitial lung disease
b. Bronchiectasis
c. Asthma
d. COPD
e. Sarcoidosis
b. Bronchiectasis
Bronchiectasis -(clubbing is not present in pure COPD)
Plus all the signs and symptoms are characteristic of bronchiectasis.
- Copious and purulent sputum
- Finger clubbing
- Post infective occurrence (as can be noted with pertussis, measles, recurrent childhood bronchiolitis etc)
That being said, he probably has underlying COPD owing to the history of chronic smoking, which makes
him susceptible to repeated viral infections of the respiratory tract and consequently bronchiectasis.
If COPD ever presents with finger clubbing, we investigate for underlying bronchogenic Ca or
bronchiectasis.

Interstitial lung disease The most common symptom of all forms of interstitial lung disease is
shortness of breath. dry nonproductive cough.
Asthma wheeze, breathlessness, chest tightness and cough, particularly if:
symptoms worse at night and in the early morning
symptoms in response to exercise, allergen exposure and cold air

symptoms after taking aspirin or beta blockers

COPD patients over 35 years of age who are smokers or ex-smokers and have symptoms such as
exertional breathlessness, chronic cough or regular sputum production.

Bronchiectasis
Bronchiectasis describes a permanent dilatation of the airways secondary to
chronic infection or inflammation.
Post-infection - eg, childhood respiratory viral infections (measles, pertussis,
influenza, respiratory syncytial virus), tuberculosis, bacterial pneumonia. Infection
is the most common cause.
persistent daily expectoration of large volumes of purulent sputum.
dyspnoea, chest pain and haemoptysis. Bronchiectasis may progress to
respiratory failure and cor pulmonale.
Coarse crackles are the most common finding
The gold standard for diagnosis is HRCT of the chest.
first-line treatment is amoxicillin 500 mg three times a day or clarithromycin 500
mg twice daily
All children and all adults up to the age of 40, presenting with bronchiectasis,
should have investigations for cystic fibrosis.

530. A 68yo man has had malaise for 5 days and fever for 2 days. He has cough and
there is dullness to percussion at the left lung base. What is the single most appropriate
inv?
a. Bronchoscopy
b. CXR
c. CT
d. MRI
e. V/Q scan
b. CXR
If a person is sufficiently sick to require hospitalization, a chest radiograph is
recommended.
A CT scan can give additional information in indeterminate cases.

Pneumonia
Presentation
Symptoms: cough, purulent sputum which may be blood-stained or rustcoloured, breathlessness, fever, malaise.
Diagnosis is unlikely if there are no focal chest signs and heart rate, respiratory
rate and temperature are normal.
The elderly may present with mainly systemic complaints of malaise, fatigue,
anorexia and myalgia.
Signs: tachypnoea, bronchial breathing, crepitations, pleural rub, dullness with
percussion.
CURB-65 criteria of severe pneumonia
Confusion (abbreviated mental test score <= 8/10)
Urea > 7 mmol/L
Respiratory rate >= 30 / min
BP: systolic <= 90 or diastolic <= 60 mmHg
age >= 65 years

Management
low or moderate severity CAP: oral amoxicillin. A macrolide should be added for
patients admitted to hospital
high severity CAP: intravenous co-amoxiclav + clarithromycin OR cefuroxime +
clarithromycin OR cefotaxime + clarithromycin

531. A 5yo child was admitted with hx of feeling tired and lethargic all the time, bleeding
gums and
sore throat since the last 3months. Exam: hepatosplenomegaly. What is the most
probable dx?
a. ALL
b. AML
c. CML
d. CLL
e. Lymphoma
a. ALL
Acute lymphoblastic leukaemia (ALL) is the most common type in children, primarily
affecting 2 to 8 year-olds.
Approximately 2/3s of AML patients are over 60.
hepatosplenomegaly in both. lymphadenopathy in ALL.
AML presents with bleeding gums, but bleeding tendency in ALL too. ALL also has gum
hypertrophy.
ALL
Symptoms
Fatigue, dizziness and palpitations
bone and joint pain
Recurrent and severe infections (oral, throat, skin, perianal infections commonly)
Fever
Left upper quadrant fullness and early satiety due to splenomegaly (10-20%)
Haemorrhagic or thrombotic complications due to thrombocytopenia or
disseminated intravascular coagulopathy (DIC) - for example, menorrhagia,
frequent nosebleeds, spontaneous bruising
Signs
Pallor
Tachycardia and a flow murmur
Nonspecific signs of infection
Petechiae,purpura or ecchymoses
hepatosplenomegaly
Lymphadenopathy
Gum hypertrophy
Cranial nerve palsy (especially III, IV, VI and VIII) in mature B-cell ALL

532. A 65yo man presents with back pain. Exam: splenomegaly and anemia. Blood:
WBC=22,
Hgb=10.9, Plt=100, ESR=25. He has been found to have Philadelphia chromosome.
What is the
single most likely dx?
a. ALL
b. AML
c. CML
d. CLL
e. Lymphoma
c. CML
CML is characterised by a consistent cytogenetic abnormality - a reciprocal translocation
between the long arms of chromosomes 22 and 9, t(9;22). The result is a shortened
chromosome 22, known as the Ph chromosome.

Symptoms

Fatigue.
Night sweats.
Weight loss.
Abdominal fullness or abdominal distension.
Left upper quadrant pain due to splenic infarction.

Signs

Splenomegaly - the most common physical finding


Hepatomegaly.
Enlarged lymph nodes are also a possibility.
Anaemia
Easy bruising.
Fever.
Gout due to rapid cell turnover.
Hyperviscosity syndrome due to leukocytosis - visual disturbance (fundoscopy
may show papilloedema, venous obstruction and retinal haemorrhages),
priapism,cerebrovascular accident (CVA), confusion.

FBC:
o
o

Leukocytosis is common.
Differential shows granulocytes at all stages of development and
increased numbers of eosinophils and basophils.
o Platelets may be elevated, decreased or normal levels.
o A mild-to-moderate, usually normochromic and normocytic, anaemia
is common.
Peripheral blood smear - all stages of maturation seen; often resembles a bone
marrow aspiration.

533. A 24yo woman has 8wk amenorrhea, right sided pelvic pain and vaginal bleeding.
She is
apyrexial. Peritonism is elicited in the RIF. Vaginal exam reveals right sided cervical
excitation.
What is the most probable dx?
a. Ectopic pregnancy
b. Salpingitis
c. Endometriosis
d. Ovarian torsion
e. Ovarian tumor
a. Ectopic pregnancy
amenorrea, pelvic pain n vaginal bleeding clinical triad of ectopic pregnancy
cervical excitation is sign in ectopic pregnancy and PID
cervical motion tenderness which is when bilateral we suspect PID n when unilateral ectopic most likely
since the pt is apyrexial this rules out PID

The most common symptoms are


Abdominal pain.
Pelvic pain.
Amenorrhoea or missed period.

Examination

There may be some tenderness in the suprapubic region.


Peritonism and signs of an acute abdomen may occur.
Women with a positive pregnancy test and any of the following need to be
referred immediately to hospital:
o Pain and abdominal tenderness.
o Pelvic tenderness.
o Cervical motion tenderness.
Vaginal bleeding (with or without clots).
The most accurate method to detect a tubal pregnancy is transvaginal ultrasound.
Human chorionic gonadotrophin (hCG) levels are performed in women with pregnancy of
unknown location who are clinically stable.

534. A 64 yo woman has been treated for breast cancer with tamoxifen. What other drug
should be added to her tx regime?
a. Bisphosphonates
b. Calcium
c. Vit D
d. Calcitonin
e. Phosphate binders
a. Bisphosphonates
bisphosphonates reduce the risk of bone metastasis in cancers and is normally taken as adjuvant therapy in
many types of tumours including breast cancer

Plus it prevents bone resorption


breast cancer can easily metastasise to bones and that in turn will lead to bone lysis so maybe thats why we
need bisphosphonates.
since the patient has been treated with TAMOXIFEN , we conclude the fact that she has a metastatic
disease.
According to BNF , the use of BISPHOSPHONATE in patients with metastatic breast cancer may reduce
pain and prevent skeletal complications of bone metastases.
The other choices like calcitonin and VIT D are of little value in postmenopausal women with metastases.

Bisphosphonates
Bisphosphonates decrease demineralisation in bone. They inhibit osteoclasts.
Clinical uses
prevention and treatment of osteoporosis
hypercalcaemia
Paget's disease
pain from bone metatases
Adverse effects
oesophagitis, oesophageal ulcers
osteonecrosis of the jaw
increased risk of atypical stress fractures of the proximal femoral shaft.
535. A 6yo woman with regular menses and her 28yo partner comes to the GP surgery
complaining of primary infertility for 2yrs. What would be the single best investigation to
see whether she is ovulating or not?
a. Basal body temp estimation
b. Cervical smear
c. Day2 LH and FSH
d. Day21 progesterone
e. Endometrial biopsy
d. Day21 progesterone
To check for whether it is ovulatiry or anovulatiry cycles. Progesterone level inceases and peaks 5 to six
days post ovulation. Which is 21 day progesterone levels

Infertility affects around 1 in 7 couples. Around 84% of couples who have regular sex will
conceive within 1 year, and 92% within 2 years
Causes
male factor 30%
unexplained 20%
ovulation failure 20%
tubal damage 15%

other causes 15%

Basic investigations
semen analysis
serum progesterone 7 days prior to expected next period (day 21 of 28 day
cycle)
indicates ovulation.
Interpretation of serum progestogen
Level

Interpretation

< 16 nmol/l

Repeat, if consistently low refer to specialist

16 - 30 nmol/l

Repeat

> 30 nmol/l

Indicates ovulation

Key counselling points


folic acid
aim for BMI 20-25
advise regular sexual intercourse every 2 to 3 days
smoking/drinking advice

536. A 10yo boy who takes regular high dose inhaled steroids for his longstanding
asthma has been advised to use bronchodilators to control his acute attacks. His parents
are unsure when should he use his bronchodilator. What is the single most appropriate
inv?
a. CXR
b. None
c. Peak flow rate diary
d. Pulse oximetry
e. Spirometry
c. Peak flow rate diary
spirometry is to make the diagnosis while Peak flow rate is for monitoring
PEF decides the severity of asthma attack..so when PEF will decreas during day he could take
bronchodilato
A peak flow diary is a useful way for you to write down and record your peak flow scores on a regular basis.
A peak flow is a measurement of how much air you can blow out of your lungs in a set amount of time. Peak
flow scores help you to see when your asthma is getting better of worse.

537. A woman presented with blurred vision and intermittent clumsiness for 3m.
Reflexes are brisk in her arm and optic disc is pale. What is the single most appropriate
test to confirm dx?
a. CSF analysis
b. CT
c. MRI
d. EEG
e. EMG
c. MRI
multiple sclerosis, investigation of choice is gadolinium enhanced mri.

Multiple sclerosis: features

3 times more common in women


most commonly diagnosed in people aged 20-40 years

Patient's with multiple sclerosis (MS) may present with non-specific features, for
example around 75% of patients have significant lethargy.
Visual

optic neuritis: common presenting feature


optic atrophy
Uhthoff's phenomenon: worsening of vision following rise in body temperature
internuclear ophthalmoplegia

Sensory
pins/needles
numbness
trigeminal neuralgia
Lhermitte's syndrome: paraesthesiae in limbs on neck flexion
Motor

spastic weakness: most commonly seen in the legs

Cerebellar
ataxia: more often seen during an acute relapse than as a presenting symptom
tremor
Others

urinary incontinence
sexual dysfunction
intellectual deterioration

538. A 63yo man presents after having a seizure. Exam: alert, orientated, inattention on
the left side and hyperreflexia of the arm. What is the most probable dx?

a. Cerebral tumor
b. Pituitary adenoma
c. Cerebellar abscess
d. Huntingtons chorea
e. Parkinsonism
a. Cerebral tumor
Inattention or neglect is a feature of parietal lobe lesion. If lesion is on right side there will be left sided
inattention. That is patient is unaware of his left side and he when shaves do it only to right half of face,
during eating eats only from the right half of plate and can not drive as he only aware of his right side and
totally unaware of left side of the road
nothing else fit thats the best expl.. pituitary will give bitemporal vision loss cerebellar signs are nystagmus
ataxia etc. Chorea is repeated movements.. its focal ant lobe lesion.
Seizure n hyperflexia (exaggerated reflexes) are indicators of space occupied lesion (SOL)
Because the pt has got upper motor neuron signs.
Its not pitutry because no signs of optic n compression and its not bilateral.
Its not cerebellar dis other wise he should have ataxia, pass pointing rombergism.
Its not chorea cuz no symptoms of chorea.
Not parkinson because no hypokinesia, tremor
seizure causing neuro deficit in elderly... first D/D should b cerebral tumour unless specified otherwise..

539. A 40yo man with a 25y hx of smoking presents with progressive hoarseness of
voice, difficulty swallowing and episodes of hemoptysis. He mentioned that he used to
be a regular cannabis user. What is the single most likely dx?
a. Nasopharyngeal cancer
b. Pharyngeal carcinoma
c. Sinus squamous cell carcinoma
d. Squamous cell laryngeal cancer
e. Hypopharyngeal tumor
d. Squamous cell laryngeal cancer
Hoarseness of voice is localizing the problem to the larynx

The symptoms of cancer of the pharynx differ according to the type:


Oropharynx: common symptoms are a persistent sore throat, a lump in the
mouth or throat, pain in the ear.
Hypopharynx: problems with swallowing and ear pain are common symptoms
and hoarseness is not uncommon.
Nasopharynx: most likely to cause a lump in the neck but may also cause nasal
obstruction, deafness and postnasal discharge.

Laryngeal Cancer

Smoking is the main avoidable risk factor for laryngeal cancer


Chronic hoarseness is the most common early symptom.
urgent CXR to decide where to refer
Flexible laryngoscopy is the best way to inspect the larynx
staging include CT and/or MRI scans.

540. A 30yo lady complains of intermittent diarrhea, chronic abdominal and pelvic pain
and
tenesmus. Sometimes she notices blood in her stool. Select the most likely cause
leading to her
symptoms?

a. Inflammatory bowel disease


b. Diverticulosis
c. Irritable bowel disease
d. Adenomyosis
e. UTI
a. Inflammatory bowel disease
classic features.

Diverticulosis is defined as the presence of diverticula which are asymptomatic.


The diagnosis of IBS should be considered if the patient has had the following for
at least 6 months:
abdominal pain, and/or
bloating, and/or
change in bowel habit
Adenomyosis is the invasion of the myometrium by endometrial tissue.
Extrauterine endometrial tissue causes inflammation, pain and the formation of
adhesions. Clinically its significance is as a cause of chronic pelvic pain,
dyspareunia and female infertility.

541. A 50yo lady with weak limbs when examined was found to have burn marks on
finger tips,
wasted and weak hands with diminished reflexes. She also has weak spastic legs and
dissociated
sensory loss. What is the dx?
a. MS
b. Syringomyelia
c. MND
d. Guillian-barre
e. Freidriechs ataxia
b. Syringomyelia
weakness and wasting of muscle, diminished or loss of tendon reflex, loss of pain and temperature sense,
experience of pain on touching skin, there are burn marks on finger due to loss of temperature sense, these
all points towards syringomyelia
MS- causes UMNL with brisk reflex not causes dissociated sensory loss. most common cause of dissociated
sensory loss - syringomyelia,& Brown-sequard syndrome.
motor+sensory-syringomyelia
motor+tongue-syringobulbia
motor only MND
Friedreich's ataxia is a progressive neurodegenerative disorder, typically with onset before 20 years of age.
Signs and symptoms include progressive ataxia, ascending weakness and ascending loss of vibration and
joint position senses, pes cavus, scoliosis, cardiomyopathy, and cardiac arrhythmias.
If the syrinx extends into the brainstem, syringobulbia results. This may affect one or more cranial nerves,
resulting in facial palsies.
Syringomyelia is more common in men

Sensory features
Pain and temperature sensation are lost due to spinothalamic tract damage.

Classically, the sensation loss is experienced in a shawl-like distribution over the


arms, shoulders and upper body.
Dysaesthesia (pain experienced when the skin is touched) is common.
Light touch, vibration and position senses in the feet are affected as the syrinx
enlarges into the dorsal columns.
Painless ulcers on the hands.
Motor features
These begin to occur as the syrinx extends and damages the lower motor
neurons of the anterior horn cells.
Muscle wasting and weakness begins in the hands and then affects the forearms
and shoulders.
Tendon reflexes are lost.
Claw hand may be present.
There may be respiratory muscle involvement.
Autonomic features
Bladder, bowel and sexual dysfunction can occur.
Horner's syndrome may be present.

Investigations

MRI is now primarily used for diagnosis


CT scanning is better at showing abnormalities of bony spinal canal, whilst MRI
scanning is better at showing soft tissue.
Plain X-rays may show a widened cervical canal.
Lumbar puncture is best avoided because of risk of herniation.

542. A 23yo woman is being followed up 6wks after a surgical procedure to evacuate the
uterus
following a miscarriage. The histology has shown changes consistent with a hydatidiform
mole.
What is the single most appropriate inv in this case?
a. Abdominal US
b. Maternal karyotype
c. Paternal blood group
d. Serum B-HCG
e. Transvaginal US
d. Serum B-HCG
Complete hydatidiform mole
Occurs when an empty egg is fertilized by a single sperm that then duplicates its own
DNA, hence the all 46 chromosomes are of paternal origin
Features
bleeding in first or early second trimester
exaggerated symptoms of pregnancy e.g. hyperemesis
uterus large for dates
very high serum levels of human chorionic gonadotropin (hCG)

hypertension and hyperthyroidism* may be seen

Management
urgent referral to specialist centre - evacuation of the uterus is performed
effective contraception is recommended to avoid pregnancy in the next 12
months
Around 2-3% go on to develop choriocarcinoma
In a partial mole a normal haploid egg may be fertilized by two sperms, or by one sperm
with duplication of the paternal chromosomes. Therefore the DNA is both maternal and
paternal in origin. Usually triploid - e.g. 69 XXX or 69 XXY. Fetal parts may be seen
*hCG can mimic thyroid-stimulating hormone (TSH)

543. A 67yo man with hx of weight loss complains of hoarseness of voice. CT reveals
opacity in the right upper mediastinum. He denied any hx of difficulty breathing. What is
the single most
appropriate inv?
a. Laryngoscopy
b. Bronchoscopy
c. LN biopsy
d. Bronchoalveolar lavage
e. Barium swallow
c. LN biopsy
Hoarseness + weight loss.. suspicion of malignancy.

hoarseness due to Malignancy


Laryngeal cancer - smoking is major risk factor.
Other neck or chest tumours - eg, lung cancer, lymphoma, thyroid cancer.
Hoarseness persisting for >3 weeks requires investigation to exclude malignancy:
Carcinomas of larynx and lung must be considered, so CXR and/or laryngoscopy
are indicated.
National Institute for Health and Care Excellence (NICE) guidance on suspected
cancer states that for patients with hoarseness persisting for >3 weeks,
particularly smokers aged 50 years and heavy drinkers:
o Arrange urgent CXR.
o Refer patients with positive findings urgently to a team specialising in the
management of lung cancer.
o Refer patients with a negative finding urgently to a team specialising in
head and neck cancer.

544. A 52yo man whose voice became hoarse following thyroid surgery 1 wk ago shows
no
improvement. Which anatomical site is most likely affected?

a. Bilateral recurrent laryngeal nerve


b. Unilateral recurrent laryngeal nerve
c. Unilateral external laryngeal nerve
d. Bilateral external laryngeal nerve
e. Vocal cords
b. Unilateral recurrent laryngeal nerve
bilateral injury of the RLN leads to aphonia.
In unilateral damage, the patient voice is still preserved but it's harsh ( hoarse ) due to unilateral paralysis of
the vocal cords.
Direct injury to the vocal cords is unlikely in thyroid procedures since the larynx isn't opened.
The external laryngeal nerves are more frequently damaged than the RLN , but they cause only minor
changes in voice quality ( pitch changes).
Bilateral rln palsy will cause emergency airway obstruction and stridor
Vocal cord inj should be transient and improving
External laryngeal inj doesnt cause hoarseness
B due to close relation of the inferior thyroid artery to the recurrent laryngeal nerve the clamping of artery
during surgery might accidentally injured the nerve causing hoarseness of voice if bilaterally affected the
nerve it will most likely causing acute respiratory distress

545. A 73yo male presents with a 12m hx of falls. His relatives have also noticed rather
strange
behavior of late and more recently he has had episodes of enuresis. Exam:
disorientation to
time and place, broad-based, clumsy gait. What is the most probable dx?
a. Dementia
b. Pituitary adenoma
c. CVD
d. Syringomyelia
e. Normal pressure hydrocephalus
e. Normal pressure hydrocephalus
clincher: Gait, dementia and enuresis
the wet, wobbly and wacky grandpa
It is a reversible cause of dementia. Seen in elderly patients. Secondary to reduced CSF absorption at the
arachnoid villi.
Classical triad of features:1. Urinary incontinence
2. Dementia
3. Bradyphrenia
4. Gait abnormality
Imaging:
Hydrocephalus with an enlarged 4th ventricle
Management:
Ventriculoperitoneal shunting
one in which person talks vulgar things---> frontotemporal dementia
One in which dementia fluctuates---> lewy body dementia
One asso with parslysis---> vascular dementia
One with stepwise deterioration---> Alzheimer's disease

546. A 75yo nursing home resident complains of headache, confusion and impaired
vision for 4days. She has multiple bruises on her head. What is the most likely cause of
confusion in this pt/
a. Alcohol intoxication

b. Infection
c. Subdural hematoma
d. Hypoglycemia
e. Hyponatremia
c. Subdural hematoma
multiple bruises on her head
Subdural
haematoma

Bleeding into the outermost meningeal layer. Most commonly


occur around the frontal and parietal lobes.
Risk factors include old age, alcoholism and anticoagulation.
Slower onset of symptoms than a epidural haematoma.

547. A 50yo woman returned by air to the UK from Australia. 3days later she presented
with sharp chest pain and breathlessness. Her CXR and ECG are normal. What is the
single most
appropriate inv?
a. Bronchoscopy
b. Cardiac enzymes
c. CT
d. MRI
e. Pulse oximetry
f. V/Q scan
g. CTPA
g. CTPA
Long flight and sharp chest pain along with breathlessness points towards PE
As per NICE guidelines the most appropriate investigation is CTPA if ur suspecting PE. V/Q scan is
preferred in only few situation like pregnancy, Ckd patients, or ctpa n/a. U have to do wells scoring of the
patient and if its > 4, u straight away do CTpa .. Dont even wait for d dimer. Definitely CTPA in this case

medical student textbook triad of pleuritic chest pain, dyspnoea and haemoptysis.
computed tomographic pulmonary angiography (CTPA) is now the recommended gold
standard
Management:
Low molecular weight heparin (LMWH) or fondaparinux should be given initially after a
PE is diagnosed.An exception to this is for patients with a massive PE where
thrombolysis is being considered. In such a situation unfractionated heparin should be
used.
a vitamin K antagonist (i.e. warfarin) should be given within 24 hours of the
diagnosis
the LMWH or fondaparinux should be continued for at least 5 days or until the
international normalised ratio (INR) is 2.0 or above for at least 24 hours,
whichever is longer, i.e. LMWH or fondaparinux is given at the same time as
warfarin until the INR is in the therapeutic range
warfarin should be continued for at least 3 months.

NICE advise extending warfarin beyond 3 months for patients with


unprovokedPE.
for patients with active cancer NICE recommend using LMWH for 6 months

Thrombolysis
thrombolysis is now recommended as the first-line treatment for massive PE
where there is circulatory failure (e.g. hypotension). Other invasive approaches
should be considered where appropriate facilities exist

548. A tall thin young man has sudden pain in the chest and becomes breathless while
crying. What is the single most appropriate inv?
a. Cardiac enzymes
b. CXR
c. CT
d. ECG
e. V/Q scan
b. CXR
spontaneous pneumothorax .most often in young thin male ..due to rupture of bullous emphysema
patient may have marfans syndrome or alpha-1 antitrypsin deficiency

Secondary pneumothorax
Recommendations include:
if the patient is > 50 years old and the rim of air is > 2cm and/or the patient is
short of breath then a chest drain should be inserted.
otherwise aspiration should be attempted if the rim of air is between 1-2cm. If
aspiration fails (i.e. pneumothorax is still greater then 1cm) a chest drain should
be inserted. All patients should be admitted for at least 24 hours
if the pneumothorax is less the 1cm then the BTS guidelines suggest giving
oxygen and admitting for 24 hours
549. A 21yo woman has had several sudden onset episodes of palpitations, sweating,
nausea and
overwhelming fear. On one occasion she was woken from sleep and feared she was
going insane. There is no prv psychiatric disorder. What is the most probable dx?
a. Pheochromocytoma
b. Panic disorder
c. GAD
d. Phobia
e. Acute stress disorder
b. Panic disorder
panic attack is MORE likely... it can be pheochromocytoma but "overwhelming fear" makes panic
attack look more fitting... It cant be phobia, because the attacks are just random without any known
trigger

Pheochromocytoma pressure symptoms

Treatment in primary care


NICE recommend either cognitive behavioural therapy or drug treatment
SSRIs are first-line. If contraindicated or no response after 12 weeks then
imipramine or clomipramine should be offered
550. A 55yo woman with a persistent cough and hx of smoking develops left sided chest
pain
exacerbated by deep breathing with fever and localized crackles. What is the single
most
appropriate dx?
a. Dissecting aneurysm
b. Pericarditis
c. Pneumonia
d. Pneumothorax
e. Pulmonary embolism
c. Pneumonia
dissec aneurysm will have a sharp pain radiating to the back.
pericarditis has similiar features , but i dont see a cardiac cause.
pneumothorax is seen in smokers , as a complication to COPD , but it wont have fever and above features. (
reduced air entry is seen in it, with hyper resonance on percussin ).
pulm embloism also seems unlikely , should have travel history .
looks like pneumonia then - pain on inspiration ( pleural pain ), fever , cough, and crackles !

Klebsiella pneumoniae is classically in alcoholics


Streptococcus pneumoniae (pneumococcus) is the most common cause of communityacquired pneumonia
Characteristic features of pneumococcal pneumonia
rapid onset
high fever
pleuritic chest pain
herpes labialis
Management
CURB-65 criteria of severe pneumonia
Confusion (abbreviated mental test score <= 8/10)
Urea > 7 mmol/L
Respiratory rate >= 30 / min
BP: systolic <= 90 or diastolic <= 60 mmHg
age >= 65 years
Patients with 3 or more (out of 5) of the above criteria are regarded as having a severe
pneumonia
The British Thoracic Society published guidelines in 2009:

low or moderate severity CAP: oral amoxicillin. A macrolide should be added for
patients admitted to hospital
high severity CAP: intravenous co-amoxiclav + clarithromycin OR cefuroxime +
clarithromycin OR cefotaxime + clarithromycin

551. A 40yo woman complains of dysphagia for both solids and liquids. She sometimes
suffers from
severe retrosternal chest pain. Barium swallow reveals a dilated esophagus which
tapers to a
fine distal end. What is the best management strategy?
a. Reassurance
b. Antispasmodics
c. Dilatation of the LES
d. Endoscopic diverticulectomy
e. Barium swallow
c. Dilatation of the LES
Achalasia typically presents in middle-age and is equally common in men and women
Investigations
manometry: excessive LOS tone which doesn't relax on swallowing - considered most

important diagnostic test

barium swallow shows grossly expanded oesophagus, fluid level, 'bird's beak' appearance. This

is in contrast to the rat's tail appearance of carcinoma of the oesophagus

CXR: wide mediastinum, fluid level


Gold standard - Manometery

Treatment

intra-sphincteric injection of botulinum toxin


Heller cardiomyotomy for fit young patients.
balloon dilation for old unwell patients.

Complications : Aspiration pneumonia, perforation, GERD, Oesophagus CA.

552. A 38yo female G4 at 32wks of pregnancy presented with thick white marks on the
inside of her mouth for 3wks. Her mouth including her tongue appeared inflamed on
examination. She
smokes 20 cigarettes/day despite advice to quit. She attends her ANC regularly. What is
the
most probable dx?
a. Lichen planus
b. Aphthous ulcer
c. Smoking
d. Candidiasis
e. Leukoplakia
d. Candidiasis
pregnancy is one of the risk factors, as well as smoking.
Oral fluconazole 50 mg/day for 7 days.

Prevention of oral candidiasis

Patients taking oral/inhaled steroids - good inhaler technique, spacer device,


rinse mouth with water after use.
Denture wearers - thorough cleaning of dentures, leave them out at night, ensure
they fit correctly.
Smoking cessation.

Lichen planus is a flat topped violaceous skin lesion not associated with smoking.
Classically, white slightly raised lesions with a trabecular, lacy appearance on the inside
of the cheeks.Can be precipitated by trauma (Kbner's phenomenon).Topical steroids
are considered to be the first-line treatment for oral lichen planus. Topical
immunomodulators (eg, imiquimod) may be useful as second-line treatment in severe
oral lichen planus.
Aphthous ulcer clearly defined, painful, shallow rounded ulcers not associated with
systemic disease. They are not infective.
Leukoplakia This is a white patch adhering to oral mucosa that cannot be removed by
rubbing. It is usually a diagnosis of exclusion.

553. A 69yo woman has had a stroke. Her left upper and lower limbs are paralyzed and
she is having difficulty in speaking. Which anatomical site is most likely affected?
a. Hippocampus
b. Cerebellum
c. Internal capsule
d. Thalamus
e. Brain stem
c. Internal capsule
Internal capsule. Its lacunar infarct. Internal capsule has both corticospinal and corticobulbar fibers. Infarct
results in hemiparesis/ hemiplegia with dysphagia/ dysarthria
Right middle cerebral artery territory is affected that irrigates internal capsule structure and broca area too
by exclusion method for me:
A would've caused memory deficit.
B some typical signs.
D sensory loss ipsilat.
E at least 2 cranial nerves involved

554. A 72yo man brought to the ED with onset of paraplegia following a trivial fall. He
was treated for prostatic malignancy in the past. What is the single most probable dx?
a. Pagets disease
b. Osteoporotic fx of vertebre
c. Secondary
d. Multiple myeloma
e. Spondylosis
c. Secondary
Bones weakened from metastatic cancer may break (fracture). The fracture can happen with a fall or injury,
but a weak bone can also break during everyday activities. These fractures often cause sudden, severe
pain. The pain may keep you from moving much at all. In some cases, a fracture is the first sign of bone
metastasis.
The most common sites of fractures are the long bones of the arms and legs and the bones of the spine.
Sudden pain in the middle of the back, for example, is a common symptom of a bone in the spine breaking
and collapsing from cancer.
c.a prostate led to vertebral mets(most common bone secondary in c.a prostate),that lead to fracture on
trivial injury and paraplegia due to spinal cord compression

Multiple myeloma can present with a wide variety of symptoms including


hypercalcaemia, anaemia, renal impairment and bone pain.

Spondylosis inflammatory back pain and enthesitis (inflammation at the site of bone
insertion of ligaments and tendons) or arthritis with radiological findings.
Pagets disease most common complaints are bone pain and/or deformity.pathological
fractures elevated serum alkaline phosphatase or characteristic abnormality on X-ray.
555. A 14yo girl has developed an itchy, scaly patch on her scalp. She had a similar
patch that cleared spontaneously 2yrs ago. Her aunt has a similar undiagnosed rash on
the extensor aspects of her elbows and knees. What is the single most likely dx?
a. Eczema
b. Fungal infection
c. Impetigo
d. Lichen planus
e. Psoriasis
e. Psoriasis
HINT: Someone with psoriasis may have other family members with the same problem. Psoriasis is a skin
condition that tends to flare up from time to time
Psoriasis. Rash Always on extensors.

Eczema on flexors.
onset below age 2 years, flexural involvement, generally dry skin, other atopic disease
Impetigo 'golden', crusted skin lesions typically found around the mouth, very
contagious
Lichen planus itchy, papular rash most common on the palms, soles, genitalia and
flexor surfaces of arms, 'white-lace' pattern on the surface (Wickham's striae)

Psoriasis
presents with red, scaly patches on the skin although it is now recognised that patients
with psoriasis are at increased risk of arthritis and cardiovascular disease.
plaque psoriasis: the most common sub-type resulting in the typical well demarcated
red, scaly patches affecting the extensor surfaces, sacrum and scalp
flexural psoriasis: in contrast to plaque psoriasis the skin is smooth
guttate psoriasis: transient psoriatic rash frequently triggered by a streptococcal
infection. Multiple red, teardrop lesions appear on the body
pustular psoriasis: commonly occurs on the palms and soles
Management of chronic plaque psoriasis
regular emollients may help to reduce scale loss and reduce pruritus
first-line: NICE recommend a potent corticosteroid applied once daily plus vitamin
D analogue applied once daily (applied separately, one in the morning and the
other in the evening) for up to 4 weeks as initial treatment
second-line: if no improvement after 8 weeks then offer a vitamin D analogue
twice daily
third-line: if no improvement after 8-12 weeks then offer either: a potent
corticosteroid applied twice daily for up to 4 weeks or a coal tar preparation
applied once or twice daily
short-acting dithranol can also be used

556. A pt after transurethral prostatic biopsy. What electrolyte imbalance can he


develop?
a. Hyperkalemia
b. Hyponatremia
c. Hypocalcemia
d. Hypernatremia
e. Hypercalcemia
b. Hyponatremia
Transurethral Resection of the Prostate (TURP) Syndrome is a rare but potentially lifethreatening complication of a transurethral resection of the prostate procedure. It occurs as a
consequence of the absorption into the prostatic venous sinuses of the fluids used to irrigate
the bladder during the operation. Symptoms and signs are varied and unpredictable, and
result from fluid overload and disturbed electrolyte balance and hyponatraemia. Treatment is
largely supportive and relies on removal of the underlying cause, and organ and
physiological support. Preoperative prevention strategies are extremely important.

557. A 28yo woman has been admitted at 38wks gestation. Her BP=190/120mmHg and
proteinuria +++. Immediately following admission she has a grand-mal seizure. What is
the single most
appropriate initial management?
a. Diazepam IV
b. Fetal CTG
c. Hydralazine IV
d. Immediate delivery
e. Magnesium sulphate IV
e. Magnesium sulphate IV
Eclampsia is defined as the occurrence of one or more convulsions superimposed on
pre-eclampsia.

Management of eclampsia

Resuscitation
Treatment and prophylaxis of seizures:
o Magnesium sulfate is the anticonvulsant drug of choice.
o Intubation may become necessary in women with repeated seizures
Treatment of hypertension:
o Reduction of severe hypertension (blood pressure >160/110 mm Hg
or mean arterial pressure >125 mm Hg) is essential to reduce the
risk of cerebrovascular accident. Treatment may also reduce the
risk of further seizures.

Intravenous hydralazine or labetalol are the two most commonly


used drugs. Both may precipitate fetal distress and therefore
continuous fetal heart rate monitoring is necessary.
Fluid therapy:
o Close monitoring of fluid intake and urine output is mandatory.
Delivery:
o The definitive treatment of eclampsia is delivery.
o However, it is unsafe to deliver the baby of an unstable mother even
if there is fetal distress. Once seizures are controlled, severe
hypertension treated and hypoxia corrected, delivery can be
expedited.
o Vaginal delivery should be considered but Caesarean section is
likely to be required in primigravidae, well before term and with an
unfavourable cervix.
o After delivery, high-dependency care should be continued for a
minimum of 24 hours

558. A 27yo woman had pre-eclampsia and was delivered by C-section. She is now
complaining of RUQ pain different from wound pain. What inv will you do immediately?
a. Coagulation profile
b. LFT
c. Liver US
d. MRCP
e. None
b. LFT
HELLP syndrome

Presentation

HELLP syndrome is a serious form of pre-eclampsia and patients may present at


any time in the last half of pregnancy.
One third of women with HELLP syndrome present shortly after delivery.
Initially, women may report nonspecific symptoms including malaise, fatigue,
right upper quadrant or epigastric pain, nausea, vomiting, or flu-like symptoms.
Hepatomegaly can occur.
Some women may have easy bruising/purpura.
On examination, oedema, hypertension and proteinuria are present.
Tenderness over the liver can occur.

Investigations

There needs to be a high index of clinical suspicion in order to avoid diagnostic


delay and improve outcome.
Haemolysis with fragmented red cells on the blood film
Raised LDH >600 IU/L with a raised bilirubin.
Liver enzymes are raised with an AST or ALT level of >70 IU/L.

Definitive treatment of HELLP syndrome requires delivery of the fetus and is advised
after 34 weeks of gestation if multisystem disease is present.

559. A 10yo girl has been referred for assessment of hearing as she is finding difficulty
in hearing her teacher in the class. Her hearing tests show: BC normal, symmetrical AC
threshold reduced
bilaterally, weber test shows no lateralization. What is the single most likely dx?
a. Chronic perforation of tympanic membrane
b. Chronic secretory OM with effusion
c. Congenital sensorineural deficit
d. Otosclerosis
e. Presbycusis
b. Chronic secretory OM with effusion
B/L conductive deafness
glue ear/ OM e effusion
Bc normal means no sn deafness .. there is conductive deafness .. otosclerosis has cd but it usually appears
in 3rd decade of life n associated with tinnitus .. perforation on both sides is uncommon .. so we're left with
csom with effusion which is most common cause of cd in school going age
Glue Ear/ otitis media with effusion: recurrent ear infections, poor speech development, and failing
performances at school, typically in children between the ages of 2 and decreasing with advancement of
age.. .Causes conductive hearing loss.
The clincher also is 'child finding difficulty in hearing in classroom/turning up the volume of Tv'

Chronic suppurative otitis media (CSOM) is a chronic inflammation of the middle ear and
mastoid cavity. Clinical features are recurrent otorrhoea through a tympanic perforation, with
conductive hearing loss of varying severity.

CSOM presents with a chronically draining ear (>2 weeks), with a possible
history of recurrent AOM, traumatic perforation, or insertion of grommets.
The otorrhea should occur without otalgia or fever.
Fever, vertigo and otalgia should prompt urgent referral to exclude intratemporal
or intracranial complications.
Hearing loss is common in the affected ear

Treatment options include:


grommet insertion - to allow air to pass through into the middle ear and hence do
the job normally done by the Eustachian tube. The majority stop functioning after
about 10 months
adenoidectomy
560. A thin 18yo girl has bilateral parotid swelling with thickened calluses on the dorsum
of her hand. What is the single most likely dx?
a. Bulimia nervosa
b. C1 esterase deficiency
c. Crohns disease
d. Mumps
e. Sarcoidosis
a. Bulimia nervosa
Clincher for a is calluses over dorsum; (chronic inducing vomiting) ,parotid swelling

Thickened calluses at back of hand (Russel's sign -tooth mark on finger for induced vomiting) +parotid
enlargment Bulimia

C1 esterase deficiency autosomal dominant condition associated with low plasma


levels of the C1 inhibitor (C1-INH) protein. attacks may be proceeded by painful macular
rash
painless, non-pruritic swelling of subcutaneous/submucosal tissues
may affect upper airways, skin or abdominal organs (can occasionally present as
abdominal pain due to visceral oedema)
Mumps can be asymptomatic.fever, headache, malaise, myalgia and anorexia, can
precede parotitis.Parotitis is usually bilateral although it can be unilateral.
Sarcoidosis
acute: erythema nodosum, bilateral hilar lymphadenopathy, swinging fever,
polyarthralgia
insidious: dyspnoea, non-productive cough, malaise, weight loss
skin: lupus pernio(chronic raised hardened, often purple lesion)
hypercalcaemia
Heerfordt's syndrome (inflammation of submaxillary/parotid glands with uveitis
and facial nerve palsy) may accompany constitutional presentation.

Bulimia nervosa
Presentation

The history often dates back to adolescence.


o Regular binge eating.
o Attempts to counteract the binges - eg, vomiting, using laxatives,
diuretics, dietary restriction and excessive exercise.
o Preoccupation with weight, body shape, and body image.
o low self-esteem, and self-harm.
o Periods may be irregular.
Physical examination is usually normal and is mainly aimed at excluding
medical complications such as dehydration or dysrhythmias (induced by
hypokalaemia).
o Examination must include height and weight (and calculation of the BMI)
and blood pressure.
o Salivary glands (especially the parotid) may be swollen.
o There may be oedema if there has been laxative or diuretic abuse.
o Russell's sign may be present (calluses form on the back of the hand,
caused by repeated abrasion against teeth during inducement of
vomiting).

561. A 48yo presents with severe chest pain since the last 40mins. In the ED he is given
oxygen, GTN, morphine. ECG=ST elevation. Bloods=increased troponin levels. What is
the next step of
management?
a. Beta blockers
b. Percutaneous angiography
c. Anticoagulant & heparin
d. Clopidogrel
e. Aspirin

b. Percutaneous angiography
Technically speaking trops are raised which means 2-3 hours have passed. PCI is indicated within 120
mins.
It should be E
PCI is more appropriate as the clinch is the time 40 mins that's why i went for B.
I would go for B
In case of ST elevation MI.. mx include aspirin at first usually given by GP or paramedic then morphine with
metoclopramide. GTN not used routinely except in case of HTN or severe LVF. Next step is PCI if available
within 120mins of hospital contact. If not available .fibrinolysis done if no CI and later proceed for PCI.
As here time frame of 4o mins mentioned..i guess its PCI..B
Its B. ST elevation MI means that the thrombus clot has already been stabilized and occluded the
vessel...that is why we give t-PA (thrombolytic) in ST elevation MI only... aim is to break down the thrombus,
not stop it from forming...aspirin only stops it from forming it doesn't break it down... smile emoticon also
raised troponin levels mean that the patient has a previous infarct at most 10 days before (since it raised in
only 40 minutes, not 4 hours), having previous infarct means has 2 or 3 vessel disease and not 1 vessel
disease, he is high risk patient and should have angioplasty as soon as possible...answer is B, angiography
is done right before angioplasty

Acute Myocardial Infarction

Pre-hospital management
first line management is MONA (Morphine, O2, Nitrates and Aspirin)
Sublingual glyceryl trinitrate and intravenous morphine + metoclopramide should be
given to help relieve the symptoms.
Aspirin 300mg should be given to all patients (unless contraindicated)
Pre-hospital thrombolysis is indicated if the time from the initial call to arrival at hospital
is likely to be over 30 minutes.

Primary percutaneous coronary intervention (PCI)


Door (or diagnosis) to treatment time should be less than 90 minutes, or less than 60
minutes if the hospital is PCI ready and symptoms started within 120 minutes
If they cannot be transferred to a larger hospital for PCI within 120 minutes then
fibrinolysis should be given. If the patient's ECG taken 90 minutes after fibrinolysis failed
to show resolution of the ST elevation then they would then require transfer for PCI.
PCI should be considered if there is an ST elevation acute coronary syndrome, if
symptoms started up to 12 hours previously

562. A 34yo female presents with a foul smelling discharge. What set of organisms are
we looking for to be treated here?
a. Chlamydia, gonorrhea
b. Chlamydia, gardenella
c. Chlamydia, gonorrhea, gardenella
d. Gonorrhea, gardenella
e. Gardenella only
e. Gardenella only

Chlamydia is usually asymptomatic (no odour) and generally goes with gonorrhea. BV will give the grey fishsmelling discharge
Bacterial vaginosis and Trichomonas vaginalis give foul smelling discharge. In BV its grey white fishy and in
TV it can be greenish frothy fihy alongwith vulvovginitis i-e strawberry cervix. The discharge of Chlamydia
and Gonorrhea is not foul smelling but gives dysuria.

Bacterial vaginosis (BV) describes an overgrowth of predominately anaerobic organisms


such as Gardnerella vaginalis.
Amsel's criteria for diagnosis of BV - 3 of the following 4 points should be present
thin, white homogenous discharge
clue cells on microscopy: stippled vaginal epithelial cells
vaginal pH > 4.5
positive whiff test (addition of potassium hydroxide results in fishy odour)
Management
oral metronidazole for 5-7 days

563. A 6wk formula fed baby boy is found at the child health surveillance to be deeply
jaundiced. His weight gain is poor and his stools are pale. What is the most likely dx?
a. Galactosemia
b. Biliary atresia
c. G6PD deficiency
d. Rh incompatibility
e. Congenital viral infection
b. Biliary atresia
pale stools , dark urine- biliary atresia
pale stools, straw urine - galactosemia
galactosemia presents with vomitting,diarrhea,failure to thrive and jaundice...progressive n deep jaundice is
biliary atresia

Biliary atresia presents shortly after birth, with persistent jaundice, pale stools and dark
urine. All term infants who remain jaundiced after 14 days (and preterm infants after 21
days) should be investigated.
Galactosemia There is often feeding difficulty, with vomiting and failure to gain weight,
with poor growth in the first few weeks of life.
G6PD deficiency
neonatal jaundice is often seen
intravascular haemolysis
gallstones are common
splenomegaly may be present
Heinz bodies on blood films

564. A 45yo man with colon cancer now develops increased thirst, increased frequency
in urination and weight loss. His fasting blood glucose=9mmol/L. what is the most
appropriate management?
a. Oral hypoglycemic
b. Insulin long acting
c. Short acting insulin before meal
d. IV insulin
e. Subcutaneous insulin
a. Oral hypoglycemic
colon cancer is assoc with hyperinsulinemia or insulin resistance..so oral hypoglycemics preferred
because oral hypoglycemic (metformin) has anticancerogenic effect.
A.. first line treatment dont get confused by colon cancer..
Metformin is the first drug of choice for the management of type 2 diabetes. It has two main antidiabetic
mechanisms of action, both of which have also been implicated as anticarcinogenic mechanisms. Firstly,
metformin inhibits hepatic glucose production through an LKB1/AMP-activated protein kinasemediated
mechanism which has been shown to adversely affect the survival of cancer cell lines. Secondly, metformin
improves insulin sensitivity in peripheral tissues reducing hyperinsulinemia. Insulin resistance and
hyperinsulinemia have been associated with increased risk of several types of neoplasm and specifically
with colorectal cancer.

565. A 34yo man from Zimbabwe is admitted with abdominal pain to the ED. An AXR
reveals bladder calcification. What is the most likely cause?
a. Schistosoma mansoni
b. Sarcoidosis
c. Leishmaniasis
d. TB
e. Schistosoma haematobium
e. Schistosoma haematobium
Schistosoma Hematobium (Bilhaarziasis). CA urinary bladder and vesicolithiasis are the two main concern
here

S. haematobium causes urinary schistosomiasis, and is the most prevalent and


widespread species in Africa and the Middle East.
Schistosomiasis is associated with anaemia, chronic pain, diarrhoea, exercise intolerance,
and malnutrition.
The first sign may be swimmer's itch
Fever.
Hepatosplenomegaly.
Right upper quadrant pain or tenderness.
Urticaria may be seen occasionally.
Lymphadenopathy.
Praziquantel is the drug of choice
Oxamniquine is the only alternative
Complications:
renal stones
increased risk of squamous cell carcinoma of bladder that has been noticed
especially in Egypt. It is possible that the infestation and the carcinogens in
tobacco smoke have a synergistic effect.

Hydronephrosis
renal failure may occur
iron-deficiency anaemia
Portal hypertension

566. A 6yo came with full thickness burn. He is crying continuously. What is the next
step of
management?
a. Refer to burn unit
b. IV fluid stat
c. Antibiotic
d. Analgesia
e. Dressing
d. Analgesia
In NHS,, making comfortable to patient is vital. Here question ask for initial management, hence analgesia is
the most here then after treat accordingly, either refer to burn unit or give if fluids using parklands.
D. Then iv fluids then refer to burn unit.

Referral to secondary care


all deep dermal and full-thickness burns.
superficial dermal burns of more than 10% TBSA in adults, or more than 5%
TBSA in children
superficial dermal burns involving the face, hands, feet, perineum, genitalia, or
any flexure, or circumferential burns of the limbs, torso, or neck
any inhalation injury
any electrical or chemical burn injury
suspicion of non-accidental injury
Management of burns
initial first aid as above
review referral criteria to ensure can be managed in primary care
superficial epidermal: symptomatic relief - analgesia, emollients etc
superficial dermal: cleanse wound, leave blister intact, non-adherent dressing,
avoid topical creams, review in 24 hours
567. A 78yo nursing home resident is revived due to the development of an intensely
itchy rash.
Exam: white linear lesions are seen on the wrists and elbows and red papules are
present on the
penis. What is the most appropriate management?
a. Topical permethrin
b. Referral to GUM clinic
c. Topical betnovate
d. Topical ketoconazole
e. Topical selenium sulfide hyoscine
a. Topical permethrin

Red papule on penis typical with wrist and elbow lesion goes with Scabies, topical permethrin once wk and
repeat if symptoms remain.

white linear lesions


Features
widespread pruritus
linear burrows on the side of fingers, interdigital webs and flexor aspects of the wrist
Nodules may develop. These occur particularly at the elbows, anterior axillary folds,
penis, and scrotum.
in infants the face and scalp may also be affected
secondary features are seen due to scratching: excoriation, infection
Management
permethrin 5% is first-line
malathion 0.5% is second-line
give appropriate guidance on use
pruritus persists for up to 4-6 weeks post eradication

568. A 4yo has earache and fever. Has taken paracetamol several times. Now its
noticed that he
increases the TV volume. His preschool hearing test shows symmetric loss of 40db.
What is the
most likely dx?
a. OM with effusion
b. Otitis externa
c. Cholesteatoma
d. CSOM
e. Tonsillitis
a. OM with effusion
see Q. 559
569. A pt presents with gradual onset of headache, neck stiffness, photophobia and
fluctuating LOC. CSF shows lymphocytosis but no organism on gram stain. CT brain is
normal. What is the single most likely dx?
a. Hairy leukoplakia
b. TB
c. CMV infection
d. Candida infection
e. Cryptococcal infection
b. TB
TB as there is lymphocytosis and no organism on gram staining Zn staining or AFB can detect
mycobacterium TB

Viral meningitis may be clinically indistinguishable from bacterial meningitis but features
may be more mild and complications (eg, focal neurological deficits) less frequent. Any
person presenting with suspected meningitis should therefore be managed as having
bacterial meningitis until proved otherwise.

classic triad of fever, neck stiffness and a change in mental status was present in only
44% of adults presenting with community-acquired acute bacterial meningitis. However,
95% had at least two of the four symptoms of headache, fever, neck stiffness and
altered mental status.
Most patients with viral meningitis present with subacute neurological symptoms
developing over 1-7 days. Chronic symptoms lasting longer than one week suggest
meningitis caused by some viruses as well as TB, syphilis or fungi.

Bacterial

Viral

Tuberculous

Appearance

Cloudy

Clear/cloudy

Slight cloudy, fibrin


web

Glucose

Low (< 1/2 plasma)

60-80% of plasma
glucose*

Low (< 1/2 plasma)

Protein

High (> 1 g/l)

Normal/raised

High (> 1 g/l)

White cells

10 - 5,000
polymorphs/mm

15 - 1,000
lymphocytes/mm

10 - 1,000
lymphocytes/mm

570. An 18m boy has been brought to the ED because he has been refusing to move his
left arm and crying more than usual for the past 24h. He has recently been looked after
by his mothers new bf while she attended college. Assessment shows multiple bruises
and a fx of the left humerus
which is put in plaster. What is the single most appropriate next step?
a. Admit under care of pediatrician
b. Discharge with painkillers
c. Follow up in fx clinic
d. Follow up in pediatric OPD
e. Follow up with GP
a. Admit under care of pediatrician
Non accidental injury
The most common manifestation of abuse is bruising
An estimated 15-25 % of pediatric burns are the result of abuse.
Fractures are the second most common manifestation of child abuse after soft tissue
injuries.
Any fracture in a young child should be concerning, especially if the child is not
ambulating.
Abusive head trauma, also known as shaken baby syndrome, is the most common
cause of child abuse death, usually occurring during the first year of life.

571. A 74yo female presents with headache and neck stiffness to the ED. Following a
LP the pt was started on IV ceftriaxone. CSF culture = listeria monocytogenes. What is
the appropriate tx?
a. Add IV amoxicillin
b. Change to IV amoxicillin + gentamicin
c. Add IV ciprofloxacin
d. Add IV co-amoxiclav
e. Continue IV ceftriaxone as mono-therapy
b. Change to IV amoxicillin + gentamicin
Meningitis caused by meningococci
Intravenous ceftriaxone for at least seven days is usually used..
Prevention of secondary case of meningococcal meningitis is usually with
rifampicin or ciprofloxacin.
Meningitis caused by pneumococci
Vancomycin and a third-generation cephalosporin (either cefotaxime or ceftriaxone)
Benzylpenicillin may be given if the organism is penicillin-sensitive but penicillin
resistance is becoming an increasing problem.
Meningitis caused by H. influenzae type b
Children aged 3 months and older and young people - intravenous ceftriaxone for
10 days.
Meningitis caused by group B streptococci
This mainly occurs in babies between the ages of 7-90 days. Intravenous
cefotaxime for at least 14 days should be given.
Meningitis caused by listeriosis
For children under the age of 3 months, intravenous amoxicillin or ampicillin for
21 days in total, plus gentamicin for at least the first seven days.

572. A pt presents with fever, dry cough and breathlessness. He is tachypneic but chest
is clear.
Oxygen saturation is normal at rest but drops on exercise. What is the single most likely
dx?
a. CMV infection
b. Candida infection
c. Pneumocystis carinii infection
d. Cryptococcal infection
e. Toxoplasma abscess
c. Pneumocystis carinii infection
This kind of history about oxygen desaturation on exercise is typical for PCP. Dry cough along with that
supports that.
patient is immunocompromised. Mostly in HIV patients we see that pt becomes breathless after a walk or
exercise. Also fever with dry cough is there. Most likely pathogen is PCP.
CMV affects retina n brain
Toxoplasmosis..brain
Candida. .mouth and esophagus
Cryptococcus..meningitis

Whilst the organism Pneumocystis carinii is now referred to as Pneumocystis jiroveci,


the term Pneumocystis carinii pneumonia (PCP) is still in common use

PCP is the most common opportunistic infection in AIDS


all patients with a CD4 count < 200/mm should receive PCP prophylaxis
Features:
dyspnoea
dry cough
fever
very few chest signs
Pneumothorax is a common complication of PCP.
Extrapulmonary manifestations are rare (1-2% of cases), may cause
hepatosplenomegaly
lymphadenopathy
choroid lesions
Investigation
CXR: typically shows bilateral interstitial pulmonary infiltrates but can present with other
x-ray findings e.g. lobar consolidation. May be normal
exercise-induced desaturation
sputum often fails to show PCP, bronchoalveolar lavage (BAL) often needed to
demonstrate PCP (silver stain shows characteristic cysts)
Management
co-trimoxazole
IV pentamidine in severe cases
steroids if hypoxic (if pO2 < 9.3kPa then steroids reduce risk of respiratory failure by
50% and death by a third)

573. A 14yo boy fell and hit his head in the playground school. He didnt lose
consciousness. He has
swelling and tenderness of the right cheek with a subconjuctival hemorrhage on his right
eye.
What is the most appropriate initial inv?
a. CT brain
b. EEG
c. MRI
d. Skull XR
e. Facial XR
e. Facial XR
there's no indication of CT scan here- he is conscious and has not vomited

The difference between Skull and facial x ray is view. Skull PA view is done in prone
position for seeing Skull bones. Facial is simply reverse i.e AP view done in supine
position and gives more clear view of facial bones. Suspected injury to facial bones is a

CONTRAINDICATION for PA view as patient cannot be asked to lie down in prone


position/ or to lean forward with face down.
A facial or sinus X-ray may be done to:
Find problems of the sinuses of the face and nose, such as sinusitisor abnormal
growths (polyps or tumors).
Find fractures of the facial bones and nose.
Check the bones around the eye (orbital cavity).
Check the sinuses before surgery.
Check for metal objects around the eyes before a magnetic resonance imaging
(MRI) test.
Look for the cause of pain in the face.
574. A 15m child is due for his MMR vaccine. There is a fam hx of egg allergy. He is
febrile with acute OM. What is the single most appropriate action?
a. Defer immunization for 2wks
b. Dont give vaccine
c. Give half dose of vaccine
d. Give paracetamol with future doses of the same vaccine
e. Proceed with standard immunization schedule
a. Defer immunization for 2wks
egg allergy is not contraindication for MMR ...therefore if pt is febrile then wait for the next two weeks until he
is afebrile and give the normal dose of immunization

Children in the UK receive two doses of the Measles, Mumps and Rubella (MMR)
vaccine before entry to primary school. This currently occurs at 12-15 months and 3-4
years as part of the routine immunisation schedule
Contraindications to MMR
severe immunosuppression
Acute illness
allergy to neomycin
children who have received another live vaccine by injection within 4 weeks
pregnancy should be avoided for at least 1 month following vaccination
immunoglobulin therapy within the past 3 months (there may be no immune
response to the measles vaccine if antibodies are present)
Adverse effects
malaise, fever and rash may occur after the first dose of MMR. This typically
occurs after 5-10 days and lasts around 2-3 days
Note that the following are NOT contra-indications:
Family history of any adverse reactions following immunisation.
Previous history of infection with pertussis, measles, rubella or mumps.
Contact with an infectious disease.
Asthma, eczema, hay fever or rhinitis.
Treatment with antibiotics or locally acting (eg, topical or inhaled) steroids.
The child's mother being pregnant.
The child being breast-fed.
History of jaundice after birth.
Being over the age recommended in the immunisation schedule.

'Replacement' corticosteroids.
Allergy to eggs
Neurological conditions are not a contra-indication although, if the condition is
poorly controlled (eg, epilepsy), immunisation should be deferred.
MMR should ideally be given at the same time as other live vaccines, such as
BCG. However, if live vaccines cannot be administered simultaneously, a fourweek interval is recommended.

575. A 33yo lady with Hodgkins lymphoma presents with temp=40C, left sided
abdominal pain and lymphadenitis. Blood was taken for test. What will you do next?
a. Wait for blood test
b. Start broad spectrum IV antibiotics
c. Oral antibiotics
d. CBC
e. Monitor pyrexia
b. Start broad spectrum IV antibiotics
the patient has an immune compromising disease ,you cant wait until you get lab results or give oral
antibiotics, you shuld give systemic antibiotic to treat any possible infectons
Chemotherapy causes imunosuppresion so increased chance of infections,as in this case temp 40,and
lymphadenitis so broad spectrum antibiotics

576. A 40yo man with marked weight loss over the preceding 6m has bilateral white,
vertically
corrugated lesion on the lateral surfaces of the tongue. What is the single most likely dx?
a. C1 esterase deficiency
b. Crohns disease
c. HIV disease
d. Sarcoidosis
e. Sjogrens syndrome
c. HIV disease

'Hairy' leukoplakia
This is associated with Epstein-Barr virus (EBV) and occurs mostly in people with HIV,
both immunocompromised and immunocompetent.
The natural history of hairy leukoplakia is variable. Lesions may frequently appear and
disappear spontaneously. Hairy leukoplakia is often asymptomatic and many patients
are unaware of its presence. Some patients with hairy leukoplakia do experience
symptoms including mild pain, dysaesthesia, alteration of taste and the psychological
impact of its unsightly cosmetic appearance.
Systemic antiviral therapy, which usually achieves resolution of the lesion within
1-2 weeks of therapy.
Topical therapy with podophyllin resin 25% solution, which usually achieves
resolution after 1-2 treatment applications.
Topical therapy with retinoic acid (tretinoin), which has been reported to resolve
hairy leukoplakia.
Ablative therapy, which can also be considered for small hairy leukoplakia
lesions. Cryotherapy has been reported as successful but is not widely used.

577. A 3m baby was miserable and cried for 2h following his 1st routine immunization
with DTP, HiB and meningitis. What is the single most appropriate action?
a. Defer immunization for 2wks
b. Dont give vaccine
c. Give half dose of vaccine
d. Give paracetamol with future doses of the same vaccine
e. Proceed with standard immunization schedule
e. Proceed with standard immunization schedule
General contraindications to immunisation
confirmed anaphylactic reaction to a previous dose of a vaccine containing the
same antigens
confirmed anaphylactic reaction to another component contained in the relevant
vaccine (e.g. egg protein)
Situations where vaccines should be delayed
febrile illness/intercurrent infection
Contraindications to live vaccines
pregnancy
immunosuppression
Specific vaccines
DTP: vaccination should be deferred in children with an evolving or unstable
neurological condition
Not contraindications to immunisation
asthma or eczema
history of seizures (if associated with fever then advice should be given regarding
antipyretics)
breastfed child
previous history of natural pertussis, measles, mumps or rubella infection
history of neonatal jaundice
family history of autism
neurological conditions such as Down's or cerebral palsy
low birth weight or prematurity
patients on replacement steroids e.g. (CAH)
578. A 65yo man with HTN develops gingival hyperplasia. What is the single most likely
dx?
a. ACEi
b. Beta blockers
c. Crohns disease
d. Nifedipine
e. Sarcoidosis
d. Nifedipine

Side effect of CCB


also due to cyclosporin, phenytoin, AML.

579. A 65yo woman is undergoing coronary angiography. What measure will protect her
kidneys
from contrast?
a. Furosemide
b. Dextrose
c. 0.45% saline
d. 0.9% saline
d. 0.9% saline
post contrast nephropathy due to contrast induced or cholesterol embolisation.
adequately hydrated pt prior to procedure reduces the complications.

580. An 83yo woman who is a resident in a nursing home is admitted to hospital with a
4d hx of
diarrhea. She has had no weight loss or change in appetite. She has been on analgesics
for 3wks
for her back pain. She is in obvious discomfort. On rectal exam: fecal impaction. What is
the
single most appropriate immediate management?
a. Codeine phosphate for pain relief
b. High fiber diet
c. Oral laxative
d. Phosphate enema
e. Urinary catheterization
d. Phosphate enema
Codiene
Laxative
Fiber will increase gut motility
Where as
Phosphate enema will act locally
Helpful in clearing
Fecal impaction too

Bulk producers:
Increase faecal mass, which stimulates peristalsis.
They must be taken with plenty of fluid
Contra-indications: difficulty in swallowing; intestinal obstruction; colonic atony;
faecal impaction.
Stool softeners:
Side-effects can include: anal seepage, lipoid pneumonia, malabsorption of fatsoluble vitamins
Stimulants:
Increase intestinal motility and should not be used in intestinal obstruction.
Prolonged use should be avoided, as it may cause colonic atony and
hypokalaemia (but there are no good, long-term follow-up studies).
Osmotic agents:

Retain fluid in the bowel.


Enemas and suppositories - useful additional treatment.

581. A 26yo woman being treated for a carcinoma of the bronchus with steroids presents
with
vomiting, abdominal pain and sudden falls in the morning. What is the most specific
cause for
her symptoms?
a. Steroid side effects
b. Postural hypotension
c. Adrenal insufficiency
d. Conns disease
e. Cushings disease
c. Adrenal insufficiency
Streoids causing suppression of acth. In turn causing mineralcorticoid deficiency so adrenal insufficiency
high dose sterods suppresss adrenals...cause hyponatraemia..hypotension..hypoglycaemia..hyperkalaemia
Exogenous steroids can suppress the pituitary adrenal axis leading to adrenal insufficiency. Symptoms
include weakness, anorexia, dizzy,
Faints,nausea,vomiting,abd pain
Steroid does not cause vomit or falls so A can be excluded.
Postural hypotension does not cause abd. Pain so B excluded .
Conn's syndrome is hyperaldosteronism which would cause hypernatremia and hypokalemia and
hypertension.
So D excluded
Cushing causes hypertension so E Excluded
C is the right answer cuz adrenal insufficiency due to prolonged steroid intake would cause addison
syndrome which is hyponatremia hypotension abdominal pain .

582. A 78yo woman presents with unilateral headache and pain on chewing.
ESR=70mm/hr. She is on oral steroids. What is the appropriate additional tx?
a. Bisphosphonates
b. HRT
c. ACEi
d. IFN
e. IV steroids
a. Bisphosphonates
She getting treated for temporal arteritis, therefore steroid will cause osteoporosis. So additional therapy is A
It appears GCA. We first do ESR and start steroids. If symptoms are not resolved then we up the dose of
steroids.
Additional treatment would be bisphosphonate to reduce risk of osteoporosis.

Bisphosphonates
Bisphosphonates decrease demineralisation in bone. They inhibit osteoclasts.
Clinical uses
prevention and treatment of osteoporosis
hypercalcaemia

Paget's disease
pain from bone metatases

Adverse effects
oesophagitis, oesophageal ulcers
osteonecrosis of the jaw
increased risk of atypical stress fractures of the proximal femoral shaft.
583. A 30yo man is suffering from fever, rash and photophobia. Doctors are suspecting
he is suffering from meningitis. Which is the best medication for this condition?
a. Ampicilling
b. Cefotaxime
c. Tetracycline
d. Acyclovir
e. Dexamethasone
b. Cefotaxime
Initial 'blind' therapy
Children 3 months and older and young people should be given intravenous
ceftriaxone as empirical treatment before identification of the causative
organism. If calcium-containing infusions are required at the same time,
cefotaxime is preferable.
584. A 15yo girl was admitted with anemia, chest infection and thrombocytopenia. She
was treated and her symptoms had regressed. She was brought again with fever and
the same symptoms a few days later. She also seems to have features of meningitis.
What is the most likely dx?
a. AML
b. ALL
c. Aplastic anemia
d. CML
e. CLL
b. ALL
Young, anemia, thrombocytopenia, recurrent infectionswith/without cns involvement and testicular swelling...
always go for All..if not treated completed can appear again...in aplastic anemia the cell count of all cell
types is low with a mention of some predisposing factor..like drugs, radiation or a dry tap of bone marrow.
Patients with ALL frequently have meningeal leukaemia at the time of relapse (50-75% at one year in
the absence of CNS prophylaxis) and a few have meningeal disease at diagnosis (<10%).

ALL
Symptoms
Fatigue, dizziness and palpitations
bone and joint pain
Recurrent and severe infections (oral, throat, skin, perianal infections commonly)
Fever
Left upper quadrant fullness and early satiety due to splenomegaly (10-20%)

Haemorrhagic or thrombotic complications due to thrombocytopenia or


disseminated intravascular coagulopathy (DIC) - for example, menorrhagia,
frequent nosebleeds, spontaneous bruising

Signs

Pallor
Tachycardia and a flow murmur
Nonspecific signs of infection
Petechiae,purpura or ecchymoses
hepatosplenomegaly
Lymphadenopathy
Gum hypertrophy
Cranial nerve palsy (especially III, IV, VI and VIII) in mature B-cell ALL

585. A pt was admitted to the ED after a head injury. When examined on arrival his
GCS=15 and then at night his GCS deteriorated to 12. What investigation should be
done?
a. CT head
b. XR skull
c. IV mannitol
d. Drill a burr hole
e. Shift to OR
a. CT head

In patients with normal or near-normal GCS and who are alert

Haemodynamic status - pulse rate, blood pressure, fluid status.


Neurological assessment - full history and examination, make notes of pupil size
and reaction to light.
Look for other possible injuries and any other relevant examination.

In patients with reduced GCS

Resuscitate but make a quick assessment of GCS and pupils. The priority is to
get the patient to hospital and CT scanned within the first hour after injury.

Selection of adults for CT scan

CT scan of the brain within one hour (with a written radiology report within one
hour of the scan being undertaken):
Glasgow Coma Scale (GCS) <13 when first assessed or GCS <15 two
hours after injury
Suspected open or depressed skull fracture
Signs of base of skull fracture*
Post-traumatic seizure
Focal neurological deficit
>1 episode of vomiting
All patients with a coagulopathy or on oral anticoagulants should have a CT
brain scan within eight hours of the injury, provided there are no other identified
risk factors, as listed above.

586. A 4yo boy who prv had normal hearing, has a mild earache relieved by
paracetamol. He has
been noticed to turn up the vol on the TV. He has bilateral dull tympanic membranes. His
preschool hearing test shows symmetrical loss of 40dB. What is the single most likely
dx?
a. Acute otitis externa
b. Acute OM
c. Ear wax
d. Foreign body
e. OM with effusion
e. OM with effusion
Glue Ear/ otitis media with effusion: recurrent ear infections, poor speech development, and failing
performances at school, typically in children between the ages of 2 and decreasing with advancement of
age.. .Causes conductive hearing loss.
The clincher also is 'child finding difficulty in hearing in classroom/turning up the volume of Tv'

Chronic suppurative otitis media (CSOM) is a chronic inflammation of the middle ear and
mastoid cavity. Clinical features are recurrent otorrhoea through a tympanic perforation, with
conductive hearing loss of varying severity.

CSOM presents with a chronically draining ear (>2 weeks), with a possible
history of recurrent AOM, traumatic perforation, or insertion of grommets.
The otorrhea should occur without otalgia or fever.
Fever, vertigo and otalgia should prompt urgent referral to exclude intratemporal
or intracranial complications.
Hearing loss is common in the affected ear

Treatment options include:


grommet insertion - to allow air to pass through into the middle ear and hence do
the job normally done by the Eustachian tube. The majority stop functioning after
about 10 months
adenoidectomy

587. An 18yo man presents to his GP with thirst and polyuria. Some 6m ago he had a
significant head injury as the result of a RTA. He is referred to the local endocrine clinic.
Which of the following results would be the most useful in confirming the dx of diabetes
insipidus after a water
deprivation test (without additional desmopressin)?
a. Plasma sodium of 126mmol/l
b. Plasma sodium of 150mmol/l
c. Plasma osmolality of 335mosmol/kg and urine osmolality of 700mosmol/kg
d. Plasma osmolality of 280mosmol/kg and urine osmolality of 700mosmol/kg
e. Plasma osmolality of 335mosmol/kg and urine osmolality of 200mosmol/kg
e. Plasma osmolality of 335mosmol/kg and urine osmolality of 200mosmol/kg
normal osmolality of plasma is somewhere around 285 to 295. Since water is being lost plasma will
be more concentrated hence osmolality more than 300. Urines normal osmolality is more than 600.
Since urine is dilute osmolality is much lower..so E is correct
In DI: urine osmolarity is low while plasma osmolarity is high.
In contrast, in SIADH, urine osmolarity is high while plasma osmolarity is low.

Classification of causes of diabetes insipidus on basis of water


deprivation and DDAVP response
Urine osmolality after
fluid deprivation
(mOsm/kg)

Urine osmolality after


DDAVP (mOsm/kg)

Likely diagnosis

<300

>800

CDI

<300

<300

NDI

>800

>800

Primary/psychogenic
polydipsia

<300

>800

Partial CDI or NDI or PP or


diuretic abuse

588. A 75yo man has left-sided earache and discomfort when he swallows. There is
ulceration at the back of his tongue and he has a palpable non-tender cervical mass.
What is the single most likely dx?
a. Acute mastoiditis
b. Dental abscess
c. Herpes zoster infection
d. Oropharyngeal malignancy
e. Tonsillitis
d. Oropharyngeal malignancy
Nontender lymphadenopathy usu suggests neoplasia.And she has ulcer near to the root of the tongue.out of
the given option closest diagnosis seems to be orppharyngeal Carcinoma which is option D.

The symptoms of cancer of the pharynx differ according to the type:


Oropharynx: common symptoms are a persistent sore throat, a lump in the
mouth or throat, pain in the ear.
Hypopharynx: problems with swallowing and ear pain are common symptoms
and hoarseness is not uncommon.
Nasopharynx: most likely to cause a lump in the neck but may also cause nasal
obstruction, deafness and postnasal discharge.
589. A 42yo man has been tired and sleepy for the last few weeks in the morning. His
work has starte getting affected as he feels sleepy in the meetings. His BMI=36. What is
the single most likely dx?
a. Idiopathic hypersomnia
b. Narcolepsy
c. Chest hyperventilation syndrome
d. OSAS
e. REM-related sleep disorder
d. OSAS
Obstructive sleep apnea syndrome

Risk factors include:


Obesity (strongest risk factor).
Male gender.
Middle age (55-59 in men, 60-64 in women).
Smoking.
Sedative drugs.
Excess alcohol consumption.
All patients with OSA causing excessive daytime or awake time sleepiness need to
cease driving until satisfactory control of symptoms has been attained.
Narcolepsy tetrad of classic symptoms: excessive daytime sleepiness (EDS), cataplexy,
hypnagogic hallucinations and sleep paralysis.
590. A 35yo pregnant woman has been having tingling and numbness of her thumb,
index and
middle fingers for a while. She has been treated with local steroids but it hasnt helped
her
much and now she has planned to undergo a surgical procedure. Which of the following
structures will be incised?
a. Flexor digitorum profundus
b. Transverse carpel ligament
c. Palmar aponeurosis
d. Extensor retinaculum
b. Transverse carpel ligament
carpal tunnel syndrome
in pregnancy, if carpal tunnel syndrome occurs: you first go for topical steroids.. then splint.. and then
surgery: and you incise the transverse carpal lig.

carpal tunnel, an anatomical compartment bounded by the bones of the carpus and the
flexor retinaculum.

CTS is characterised by tingling, numbness, or pain in the distribution of the median nerve
(the thumb, index, and middle fingers, and medial half the ring finger on the palmar aspect)
that is often worse at night and causes wakening.

Positive Phalen's test: flexing the wrist for 60 seconds causes pain or
paraesthesia in the median nerve distribution.
Positive Tinel's sign: tapping lightly over the median nerve at the wrist causes a
distal paraesthesia in the median nerve distribution.
Positive carpal tunnel compression test: pressure over the proximal edge of
the carpal ligament (proximal wrist crease) with thumbs causes paraesthesia to
develop or increase in the median nerve distribution.

Electroneurography (ENG) - This is the gold standard investigation for CTS.

591. A 58yo pt presents with altered bowel habits and bleeding per rectum. Exam and
sigmoidoscopy showed an ulcer. What is the single most likely dx?
a. Colorectal carcinoma
b. Celiac disease
c. Crohn's disease
d. UC
e. IBS
a. Colorectal carcinoma
Older patient. Altered bowel habits and bleeding PR is Ca unless proven otherwise.
Alter bowl habbit+ bleeding are a red flag for CA colon>>>> colonoscopy >>>> one ulcer >>> CA. UC
Usually there is no alteration in bowel habit, usually boodly diarrhoea >>> colonoscopy >>>>
multiple ulceration

third most common cancer in the UK


second most common cause of cancer death in the UK.
Presentation:
Right colon cancers: weight loss, anaemia, occult bleeding, mass in right iliac
fossa, disease more likely to be advanced at presentation.
Left colon cancers: often colicky pain, rectal bleeding, bowel obstruction,
tenesmus, mass in left iliac fossa, early change in bowel habit, less advanced
disease at presentation.
The most common presenting symptoms and signs of cancer or large polyps are
rectal bleeding, persisting change in bowel habit and anaemia.
Colonoscopy is the gold standard for diagnosis of colorectal cancer.

592. A mother is concerned that her 18m son has a vocabulary of ten words but cant
form a
sentence. What is the best management strategy?
a. Arrange hearing test
b. Assess developmental milestones
c. Reassurance
d. Refer to speech therapist
e. MRI brain
c. Reassurance

Age

Milestone

3 months

Quietens to parents voice


Turns towards sound
Squeals

6 months

Double syllables 'adah', 'erleh'

9 months

Says 'mama' and 'dada'


Understands 'no'

12 months

Knows and responds to own name

12-15 months

Knows about 2-6 words (Refer at 18 months)


Understands simple commands - 'give it to mummy'

2 years

Combine two words


Points to parts of the body

2 years

Vocabulary of 200 words

3 years

Talks in short sentences (e.g. 3-5 words)


Asks 'what' and 'who' questions
Identifies colours
Counts to 10 (little appreciation of numbers though)

4 years

Asks 'why', 'when' and 'how' questions

593. A 55yo man has weight loss, dyspnea and syncope. He smokes 20 cigarettes/day.
Inv confirms squamous cell carcinoma in the left bronchus. What is the single most likely
biochemical
abnormality to be a/w the condition?
a. Hypercalcemia
b. Hyperkalemia
c. Hypoernatremia
d. Hypocalcemia
e. Hypomagnesium
a. Hypercalcemia
Squamous cell ca causes hypercalcemia...whereas small cell ca causes SIADH
paraneoplastic syndrome causing ectopic production of parathyroid hormone-related protein (PTHrP),
resulting in "hypercalcemia"

There are three main subtypes of non-small cell lung cancer:


Squamous cell cancer
typically central

associated with parathyroid hormone-related protein (PTHrP) secretion


hypercalcaemia
strongly associated with finger clubbing

Adenocarcinoma
typically peripheral
most common type of lung cancer in non-smokers, although the majority of
patients who develop lung adenocarcinoma are smokers
Large cell lung carcinoma
typically peripheral
anaplastic, poorly differentiated tumours with a poor prognosis
may secrete -hCG
594. A 72yo man presents with intermittent difficulty in swallowing with regurgitation of
stale food
materials. Sometimes he wakes up at night with a feeling of suffocation. Choose the
single most
likely cause of dysphagia?
a. Benign structure
b. Esophageal carcinoma
c. Esophageal spasm
d. Pharyngeal pouch
e. Systemic sclerosis
d. Pharyngeal pouch
Pharyngeal
pouch

More common in older men


Represents a posteromedial herniation between thyropharyngeus
and cricopharyngeus muscles
Usually not seen but if large then a midline lump in the neck that
gurgles on palpation
Typical symptoms are dysphagia, regurgitation, aspiration and
chronic cough. Halitosis may occasionally be seen

595. A 9m child is brought to the ED with an irreducible firm swelling which descended
into the left
groin when the child has been crying. Exam: both testicles are palpable in the scrotum.
What is
the most appropriate management strategy?
a. Reassurance
b. Emergency herniotomy
c. Elective herniotomy
d. Emergency herniotomy + orchidopexy
e. Elective herniotomy + orchidopexy

c. Elective herniotomy
As the swelling descended into the left scrotum when the child was crying so probably it is no more
irreducible
So elective herniotomy
strangulated hernia:

Examination reveals a firm lump in the groin of a crying child, which may extend into the
scrotum. The child may have vomited but is usually well.
Paediatric surgeons will undertake repair soon after diagnosis, regardless of age
or weight, in healthy full-term infant boys with asymptomatic reducible inguinal
hernias. Emergency surgery is twenty times more likely to cause complications
than an elective procedure.
Immediate surgery is not always necessary in a case of strangulation: four out of
five can be reduced manually.
Tachycardia, fever or signs or obstructions are indications for surgery.
596. A 37yo woman was admitted for femur fx repair after a RTA. On the 4th post-op
day she became confused and starts picking on her bed sheets and complains of seeing
spiders all over. What is the most likely dx?
a. Delirium tremens
b. Wernickes encephalopathy
c. Korsakoffs psychosis
d. Psychotic depression
a. Delirium tremens
Delirium tremens usually begins 24-72 hours after alcohol consumption has been
reduced or stopped.
The symptoms/signs differ from usual withdrawal symptoms in that there are
signs of altered mental status. These can include:
o Hallucinations (auditory, visual, or olfactory).
o Confusion.
o Delusions.
o Severe agitation.
Seizures can also occur.
Examination may reveal signs of chronic alcohol abuse/stigmata of chronic liver
disease. There may also be:
o Tachycardia.
o Hyperthermia and excessive sweating.
o Hypertension.
o Tachypnoea.
o Tremor.
o Mydriasis.
o Ataxia.
o Altered mental status.
o Cardiovascular collapse.

Withdrawal symptoms:

Symptoms typically present about eight hours after a significant fall in blood
alcohol levels. They peak on day 2 and, by day 4 or 5, the symptoms have
usually improved significantly.
Minor withdrawal symptoms (can appear 6-12 hours after alcohol has stopped)
Alcoholic hallucinosis (can appear 12-24 hours after alcohol has stopped):[6]
o Includes visual, auditory or tactile hallucinations.
Withdrawal seizures (can appear 24-48 hours after alcohol has stopped):
o These are generalised tonic-clonic seizures.
Alcohol withdrawal delirium or 'delirium tremens' (can appear 48-72 hours
after alcohol has stopped)

597. A 36yo pt came with diarrhea, bleeding, weight loss and fistula. What is the single
most likely dx?
a. Celiac disease
b. Crohns disease
c. UC
d. IBS
b. Crohns disease
fistula due to transmural involvement, weight loss in crohns.
598. A 72yo woman who is taking look diuretics is suffering from palpitations and muscle
weakness.
What is the electrolyte imbalance found?
a. Na+ 130mmol/l, K+ 2.5mmol/l
b. Na+ 130mmol/l, K+ 5.5mmol/l
c. Na+ 140mmol/l, K+ 4.5mmol/l
d. Na+ 150mmol/l, K+ 3.5mmol/l
e. None
a. Na+ 130mmol/l, K+ 2.5mmol/l
Hypokalaemia can occur with loop or thiazide diuretics.

599. A 60yo diabetic pt on anti-diabetic medication developed diarrhea. What is the most
likely cause for his diarrhea?
a. Autonomic neuropathy
b. Infective
c. Celiac disease
d. Crohns disease
a. Autonomic neuropathy
In general people no autonomic neuropathy usually. Once a diabetic develop autonomic neuropathy he gets
diarrhea off and on due to his autonomic neuropathy and it becomes the most comon cause for his diarrhea.
So most likely cause in this case would be Autonomic neuropathy.
Autonomic neuropathy{GI tract} causes dysfunctional motility / secretions /absorption. ..leads to
gasteroparesis(damage to vagus nerve ), ch: diarrhea(neuronal damage to small intestine)
,,/constipation(colon nerves damage )...

>diarrhea can be due to metformin] it has very common GI side effects ..

Autonomic neuropathy

Risk factors include hypertension and dyslipidaemia. It is more common in


females.
May present with:
o Cardiac autonomic neuropathy, which has been linked to:[5]
Resting tachycardia, postural hypotension, orthostatic bradycardia
and orthostatic tachycardia.
Exercise intolerance.
Decreased hypoxia-induced respiratory drive.
Increased incidence of asymptomatic myocardial ischaemia,
myocardial infarction, decreased rate of survival after myocardial
infarction.
Congestive heart failure.
o Genitourinary:
Impotence, retrograde ejaculation, urinary hesitancy, overflow
incontinence.
At least 25% of men with diabetes have problems with sexual
function.
There is often no association with glycaemic control, duration or
severity of diabetes.
Risk factors for erectile dysfunction include increasing age,
alcohol, initial glycaemic control, intermittent claudication and
retinopathy.
o Gastrointestinal:
Nausea and vomiting.
Abdominal distension.
Dysphagia.
Diarrhoea.
o Gustatory sweating, anhidrosis.

600. Which artery runs in the anterior inter-ventricular groove?


a. Acute marginal branch
b. Left ant descending artery
c. Septal branches
d. Circumflex artery
e. Right coronary artery
b. Left ant descending artery
601. A mother presents with her 12m daughter. The child has no meaningful words, is
unable to sit unaided and cant play with her toys. She doesnt laugh and has poor
interaction with her
siblings. What is the best management strategy?
a. Arrange hearing test
b. Assess developmental milestones
c. Reassure
d. Refer to speech therapist

e. MRI brain
b. Assess developmental milestones
1st we will do assessment of all developmental milestones thn will go for ct or mri

602. A pt presents with progressive visual deterioration. Exam: large, multiple cotton
wool spots in
both eyes. What is the single most likely dx?
a. Kaposis sarcoma
b. Cryptosporidium
c. CMV infection
d. Pneumocystis carinii infection
e. Cryptococcal infection
c. CMV infection
Retinitis:
Retinitis is the most common manifestation of CMV disease in patients who are
HIV positive.
It presents with decreased visual acuity, floaters, and loss of visual fields on one
side.
Ophthalmological examination shows yellow-white areas with perivascular
exudates. Haemorrhage is present. Lesions may appear at the periphery of the
fundus, but they progress centrally.
It begins as a unilateral disease, but in many cases it progresses to bilateral
involvement. It may be accompanied by systemic CMV disease.

Ganciclovir has been used to treat retinitis, but it only slows the progression of
the disease. The optimal treatment is using ganciclovir implants in the vitreous,
accompanied by intravenous ganciclovir therapy.
Oral ganciclovir may be used for prophylaxis of CMV retinitis. It should not be
used for treatment.

603. A 53yo had a dental extraction after which he recently had a mitral valve prolapse,
high temp of 39C, cardiac failure and new cardiac murmur. What is the single most likely
dx?
a. Atheroma
b. Congenital
c. Regeneration
d. Infection
e. Neoplastic
d. Infection
infective endocarditis?

604. A 12yo boy with a hx of fall on an outstretched hand was brought to the ED with
swelling and
pain around the elbow. His radial nerve was affected. What is the type of fx?
a. Angulated fx
b. Epiphyseal fx
c. Compound fx
d. Spiral fx
d. Spiral fx

Humeral shaft fractures


Complications

Radial nerve injury: occurs in 11.8% of fractures. It is most common in distal third
fractures.It is more common in transverse or spiral fracture. Spontaneous
recovery occurs in 70.7% treated conservatively. Initial expectant treatment may
avoid unnecessary operations.
Brachial artery injury.
Non-union.

605. A 32yo lady complains that she hears everyone saying that she is an evil person.
What type of hallucinations is she suffering from?
a. 2nd person auditory hallucinations
b. 3rd person auditory hallucinations
c. Echo de la pense
d. Gedankenlautwerden

b. 3rd person auditory hallucinations


She hears everyone
talking ABOUT her. So it's third person hallucination. If she had been hearing
everyone talking TO her, it would've been second person hallucination.

606. A 65yo woman had an excision of colonic tumor 3yrs ago. Now she is losing weight
and feels
lethargic. Exam: pale but no abdominal findings. What is the most appropriate inv?
a. CA 125
b. CA 153
c. CA 199
d. CEA
e. AFP
d. CEA
607. A 46yo African-Caribbean man is found to have BP=160/90mmHg on 3 separate
occasions. What is the best initial tx?
a. ACEi
b. Beta-blockers
c. ARBs
d. None
e. CCB
e. CCB
Step 1 treatment
patients < 55-years-old: ACE inhibitor (A)
patients > 55-years-old or of Afro-Caribbean origin: calcium channel blocker
Step 2 treatment
ACE inhibitor + calcium channel blocker (A + C)
Step 3 treatment
add a thiazide diuretic (D, i.e. A + C + D)
NICE now advocate using either chlorthalidone (12.5-25.0 mg once daily) or
indapamide (1.5 mg modified-release once daily or 2.5 mg once daily) in
preference to a conventional thiazide diuretic such as bendroflumethiazide
608. A 39yo woman will undergo tubal sterilization and she wants to know the failure
rate of this
type of sterilization.
a. 1:50
b. 1:200
c. 1:500
d. 1:1000
e. 1:5000

b. 1:200
Male sterilisation - vasectomy
failure rate: 1 per 2,000*
semen analysis needs to be performed twice following a vasectomy before a
man can have unprotected sex (usually at 16 and 20 weeks)
Female sterilisation
failure rate: 1 per 200*
609. Which of the following reflexes and innervating spinal nerves are correctly paired?
a. Anal reflex S1
b. Ankle jerk L5
c. Biceps jerk C7 & C8
d. Knee jerk L3 & L4
e. Triceps jerk T1
d. Knee jerk L3 & L4
bicep: C5-C6
Tricep: C6-C7
Ankle: S1

610. A 62yo man with rheumatoid arthritis struck his hand against a door. He
subsequently found
that although he could extend the interphalangeal joint of his right thumb, the MCP joint
of the
thumb remained flex. What is the single most likely tendon to have been damaged?
a. Extensor carpi ulnaris
b. Extensor digitorum
c. Extensor indicis
d. Extensor pollicis brevis
e. Extensor pollicis longus
d. Extensor pollicis brevis
Remember pollicis is for thumb. Extensir pollicis brevis is inserted at the base of proximal phalanx of thumb .
Extensor pollucis longus at the interphalangeal joint of thumb. As she is unable to extend MCP joint so
brevus tendon is ruptured.
( Make it simple.. from longus remember it would be long and go more distal to get inserted at the IP joint.
And brevis ends short at MCP joint)

611. A 68yo lady complains of falls to the ground without any warning, maintains
consciousness and no confusion. She says this has occurred at number of times. What
is the dx?
a. Stokes Adams attack
b. Hypoglycemia
c. Vasovagal syncope
d. Drop attacks
e. Epilepsy
d. Drop attacks
Sudden falling to ones knees without LOC and without warning. Recovery is immediate
brief LOC in SAA without warning,while vv syncope- triggers with loc

because she maintains consciousness. Stokes Adams and Vaso-vagal--> brief loss of consciousness.
Stokes Adams patient gets pale before attack. Vaso-vagal mostly in young girls
age is not for epilepsy , other than a,b,c preceding particular s/s with brief loc in a&b ,
Adams is associated Av heart block bradyarrhythmia,hypoglycemia has hx of dm or sweating
tachycardia,vasovagal is suddenly change in position and epilepsy is hx and initial convulsive and postictal
phase

A classic Stokes-Adams attack is a collapse without warning, associated with


loss of consciousness for a few seconds. Typically, complete (third-degree) heart
block is seen on the ECG during an attack (but other ECG abnormalities such as
tachy-brady syndrome have been reported)
Syncope is a transient loss of consciousness caused by transient global cerebral
hypoperfusion characterised by rapid onset, short duration, and spontaneous
complete recovery
Hypoglycaemia
Shaking and trembling
Sweating, pins and needles in the lips and tongue
Hunger, palpitations
Headache (occasionally), double vision, difficulty in concentrating
Slurring of speech, confusion, change of behaviour
Stupor, coma
612. A 50yo man complains of being pursued by the police for a crime he denies
committing. He has poor concentration and impaired short-term memory. He admits to
drinking large amounts of
alcohol for the last 20yrs. What is the most probable dx?
a. Dementia
b. Hallucination
c. Wernickes encephalopathy
d. Schizophrenia
e. Korsakoff psychosis
e. Korsakoff psychosis
ConfabulationFalsification of memory in clear consciousness - very characteristic of the
syndrome.
Anterograde amnesia is the main feature of the syndrome.
Retrograde amnesiaTelescoping of events is characteristic - eg, the patient says
something happened recently when it took place many years ago.
Wernicke-Korsakoff syndrome (WKS) is a spectrum of disease resulting from thiamine
deficiency, usually related to alcohol abuse.
Wernicke's encephalopathy: a classic triad of symptoms (mental confusion, ataxia and
ophthalmoplegia)
Korsakoff's syndrome: late manifestation of the condition, where Wernicke's
encephalopathy has not been adequately treated

Serum thiamine levels (vitamin B1) levels may be low.

613. A pt with prv hx of HTN, the membranes have ruptured and the cervix is 3cm
dilated. 4h later on examination showed that the cervix was still 3cm dilated. What is the
single most appropriate
management for her labor?
a. Repeat vaginal examination in 4h
b. CTG
c. C-section
d. External rotation
e. IV syntocin drip
e. IV syntocin drip
Prolonged labour first line is to augment labour, either artficial rupture of membranes followed by IV
syntocin. In this case patient membranes have ruptured spontaneously and there is no contraindication to
syntocin. So augment labour first if patient still had poos progress after augmenting labour then C section.

Active management of labour has been modified significantly over time but the core
principles remain:
Early diagnosis following strict criteria, by a senior midwife.
Vaginal examination hourly for three hours, then every two hours, at least. This
allows the rate of progress to be plotted on a partogram.
Amniotomy one hour after admission.
Augmentation with Syntocinon if not dilating at rate of 1 cm/hour.
614. A 6yo girl has had 2 short episodes of cough and wheeze over the last 12m. These
2 acute
episodes responded quickly to bronchodilator, she has no symptoms or abnormal
physical signs.
She has slight eczema and her mother has asthma. What is the single most appropriate
inv?
a. CXR
b. Peak flow rate diary
c. Pulse oximetry
d. Spirometry
e. Sweat test
d. Spirometry
Diagnostic test for asthma is Spirometry. And if it was a known asthmatic case then our investigation wud be
pefr....if the ques mentioned something abt known asthma on beta agonist and or steroid...yt having
symptoms and exacerbations provided she is taking inhaler properly...best advice wud be to maintain a pef
diary so tht we can adjust treatment options.
Spirometry
If FEV1/ FBV > 80% ( Restrictive lung disease)
If FEV1/FVC < 80% ( obstructive lung disease)
If reversible > 12% after brochochodilator ( reversibilty test) its asthma otherwise copd

Spirometry
With older children with an intermediate probability of asthma, diagnostic tests such as
PEFR and forced expiratory volume in one second (FEV1) can provide objective
measures of airways obstruction but these may be normal between episodes of
bronchospasm and provide poor discrimination with other conditions that also cause
airways obstruction.Spirometry is usually possible from about 5 years old, although there

is wide variation, and is dependent on the child's co-operation and comprehension of the
task.
Where there is evidence of airways obstruction, looking for changes in PEFR or FEV1
10 minutes after the use of a bronchodilator (reversibility usually taken as >12%
subsequent improvement in lung function). Also, look for response to a treatment trial
over a defined time period, as this adds further weight to the diagnosis of asthma.

615. A 45yo man had recently started taking anti-HTN therapy. 6m later his
RBS=14mmol/l. Which
single drug is most likely to have caused this?
a. Amlodipine
b. Bendroflumethiazide
c. Doxazosin
d. Losartan
e. Ramipril
b. Bendroflumethiazide
Pt was not diabetic, but he develops DM after taking medication...Thiazide and B-blockers increases risk of
DM OHCM-134
The connection between diuretics and hyperglycemia involves intracellular K+ levels. Intracellular K+ is
involved in the secretion of a lot of hormones...including insulin. Some diuretics cause hypokalemia,
(decrease inK+) like thiazides. This inhibits insulin secretion and can lead to hyperglycemia.

Common adverse effects


dehydration
postural hypotension
hyponatraemia, hypokalaemia, hypercalcaemia
gout
impaired glucose tolerance
impotence
Amlodipine Flushing, headache, ankle swelling
Doxazosin postural hypotension, drowsiness, dyspnoea, cough
Losartan Like ACE inhibitors they should be used with caution in patients with renovascular
disease. Side-effects include hypotension and hyperkalaemia.
Ramipril cough, angioedema, hyperkalaemia, first-dose hypotension.

616. A 27yo waitress has pelvic pain, dysmenorrhea and increasingly heavy periods.
She also
complains of dyspareunia. There is generalized pelvic tenderness without peritonism.
Pelvic US
is normal. What is the most likely dx?
a. Endometriosis
b. Uterine fibroid
c. Pelvic congestion syndrome
d. PID
e. Tubal pregnancy
c. Pelvic congestion syndrome (or A?)

Here, Profession , Waitress, is the clue , prolonged standing is the risk factor for pelvic congestion
syndrome!
It is pelvic venous congestion, dilated pelvic veins cause cyclic dragging pain, worse menstrually and after
prolonged standing,walking. Dyspareunia. Air hostess, waiters.
Also pelvis us free suggest absent of endometriosis plus waitress !! But if didn't mentioned pelvic us it will be
typically endometriosis

Ultrasound examination may be useful (US). This imaging test uses sound waves to detect the
abnormal veins. It is good at showing the blood flow through the veins and is non-invasive.
However, sometimes the veins in the pelvis are difficult to see through the abdomen, therefore a
special ultrasound where a small probe is placed into the vagina to see the veins, called a
transvaginal ultrasound may be required.
Duplex ultrasound scanning =golden standard diagnosis

617. A 14yo girl is clinically obese. She has not started her periods yet and has severe
acne. Among
her inv, a high insulin level is found. What is the most probable dx?
a. Cushings syndrome
b. Graves disease
c. Acquired hypothyroidism
d. PCOS
e. Addisons disease
d. PCOS
14 yr...primary amenorrhea, severe acne ,high insulin. ...All favours PCOS.
obesity hyperandrogenism(acne) hyperinsulinemia and amenorrhea
P' AM + Insulin Resistance. Both Cushing n PCOS have hyperglycaemia but in Cushing's $, it's due to high
glucocorticoids lvl, not coz of Insulin Resistance.

Features
subfertility and infertility
menstrual disturbances: oligomenorrhea and amenorrhoea
hirsutism, acne (due to hyperandrogenism)
obesity
acanthosis nigricans (due to insulin resistance)
Investigations
pelvic ultrasound: multiple cysts on the ovaries
FSH, LH, prolactin, TSH, and testosterone are useful investigations: raised
LH:FSH ratio is a 'classical' feature but is no longer thought to be useful in
diagnosis. Prolactin may be normal or mildly elevated. Testosterone may be
normal or mildly elevated - however, if markedly raised consider other causes
check for impaired glucose tolerance
618. An 18yo girl with primary amenorrhea complains of severe abdominal pain every 48weeks
which is now getting worse. Exam: lower abdominal mass is felt. What is the most
probable dx?
a. Ectopic pregnancy
b. Ovarian carcinoma
c. Hematometrium

d. Biliary colic
e. Renal carcinoma
c. Hematometrium
hematometrium, may be septate vagina
She has cyclical bleed every month as scenario tells. Examination shows lower abdmonal mass likely blood
accumulation everytime. Cause is likely imperforate hymen or transvaginal septum

619. A 14yo boy with asthma suddenly developed chest pain and increasing
breathlessness during a
game of football. When seen in the ED he was not cyanosed. He has reduced breath
sounds on
the right side. His oxygen saturation is 94% on air. What is the single most appropriate
inv?
a. Capillary blood gases
b. CXR
c. CT chest
d. Exercise challenge
e. MRI chest
b. CXR
spontaneous pneumothorax
young pt, sudden cp, sob, decreased breath sounds -> spont pneumothorax on rt. side ... CXR- if >2cm...do
aspiration

if the rim of air is < 2cm and the patient is not short of breath then discharge
should be considered
otherwise aspiration should be attempted
if this fails (defined as > 2 cm or still short of breath) then a chest drain should be
inserted

620. A 36yo woman was recently admitted to a psychiatric ward. She believes that the
staff and
other pts know exactly what she is thinking all the time. What is the most likely symptom
this pt
is suffering from?
a. Thought insertion
b. Thought withdrawal
c. Thought block
d. Though broadcasting
e. Hallucination
d. Though broadcasting
Thought insertion, removal or interruption - delusions about external control of thought
Thought broadcasting - the delusion that others can hear one's thoughts

thought withdrawal is the delusional belief that thoughts have been 'taken out' of the
patient's mind, and the patient has no power over this. It often accompanies thought
blocking.
Thought blocking is a thought condition usually caused by a mental health condition
such as schizophrenia. During thought blocking, a person stops speaking suddenly and
without explanation in the middle of a sentence.
hallucination is a perception in the absence of external stimulus that has qualities of
real perception.
621. A 60yo woman is admitted to the hospital after a fall. She is noted to have poor eye
contact.
When asked how she is feeling, she admits to feeling low in mood and losing enjoyment
in all
her usual hobbies. She has also found it difficult to concentrate, feels that she is not
good at
anything, feels guilty over minor issues and feels very negative about the future. What is
the
most likely dx?
a. Mild depression
b. Moderate depression
c. Severe depression
d. Psychosis
e. Seasonal depression
a. Mild depression
NICE use the DSM-IV criteria to grade depression:
1. Depressed mood most of the day, nearly every day
2. Markedly diminished interest or pleasure in all, or almost all, activities most of
the day, nearly every day
3. Significant weight loss or weight gain when not dieting or decrease or increase
in appetite nearly every day
4. Insomnia or hypersomnia nearly every day
5. Psychomotor agitation or retardation nearly every day
6. Fatigue or loss of energy nearly every day
7. Feelings of worthlessness or excessive or inappropriate guilt nearly every day
8. Diminished ability to think or concentrate, or indecisiveness nearly every day
9. Recurrent thoughts of death, recurrent suicidal ideation without a specific plan,
or a suicide attempt or a specific plan for committing suicide
Subthreshold
depressive symptoms
Mild depression

Fewer than 5 symptoms


Few, if any, symptoms in excess of the 5 required to make
the diagnosis, and symptoms result in only minor functional
impairment

Moderate depression

Symptoms or functional impairment are between 'mild' and


'severe'

Severe depression

Most symptoms, and the symptoms markedly interfere with


functioning. Can occur with or without psychotic symptoms

622. A 70yo woman lives in a nursing home following a stroke has developed reddish
scaly rash on
her trunk. She has many scratch marks on her limbs and trunk with scaling lesions on
her hands
and feet. What is the single most appropriate initial tx?
a. Aqueous cream
b. Chlorphenaramine
c. Coal tar
d. 1% hydrocortisone ointment
e. Permethrin
e. Permethrin
Nursing home, multiple scratch marks :scabies

Features
widespread pruritus
linear burrows on the side of fingers, interdigital webs and flexor aspects of the
wrist
Nodules may develop. These occur particularly at the elbows, anterior axillary
folds, penis, and scrotum.
in infants the face and scalp may also be affected
secondary features are seen due to scratching: excoriation, infection
Management
permethrin 5% is first-line
malathion 0.5% is second-line
give appropriate guidance on use (see below)
pruritus persists for up to 4-6 weeks post eradication
623. A 16yo boy following a RTA was brought to the ED with a swelling and deformity in
his right
thigh. Exam: airway is patent and is found to have a pulseless leg. Which structure is
involved in
this fx?
a. Femoral artery
b. Posterior tibial artery
c. Common peroneal nerve
d. Dorsalis pedis
a. Femoral artery

624. A man sat cross-legged for about 30mins. After this he was unable to dorsiflex his
left foot and
had loss of sensation in the web space between the big toe and the 2nd toe. He also has
sensory
loss on the same side of the foot after 2h. Which of the following was affected?
a. Femoral nerve
b. Sural nerve
c. Peroneal nerve
d. Sciatic nerve
c. Peroneal nerve
Chronic peroneal neuropathy can result from, among other conditions, bed rest of long
duration, hyperflexion of the knee, peripheral neuropathy, pressure in obstetric stirrups,
and conditioning in ballet dancers. The most common cause is habitual leg crossing that
compresses the common peroneal nerve as it crosses around the head of the
fibula.Transient trauma to the nerve can result from peroneal strike.
Damage to this nerve typically results in foot drop, where dorsiflexion of the foot is
compromised and the foot drags (the toe points) during walking; and in sensory loss to
the dorsal surface of the foot and portions of the anterior, lower-lateral leg.

625. A 25yo woman is presenting with diarrhea and abdominal bloating over the last 4m.
Exam: she
has blistering rash over her elbows. Biochemistry: low serum albumin, calcium and folate
conc.
On jejunal biopsy, there is shortening of the villi and lymphocytosis. What is the most
likely dx?
a. Celiac disease
b. Whipples disease
c. Crohns disease
d. Tropical sprue
e. Giardiasis
f. Cystic fibrosis
a. Celiac disease
Patient with chronic diarrhoea, villus atrophy, lymphocytosis, Dermatitis herpetiformis. All with Celiac
disease.
Coeliac Disease ( Endoscopic small bowel biopsy- subtotal villus atrophy and lymphocytic infiltration )
The blistering rash is dermatitis herpetiformis..associated with coeliac

Coeliac disease is caused by sensitivity to the protein gluten. Repeated exposure leads to villous
atrophy which in turn causes malabsorption. Conditions associated with coeliac disease include
dermatitis herpetiformis (a vesicular, pruritic skin eruption) and autoimmune disorders (type 1
diabetes mellitus and autoimmune hepatitis).
Signs and symptoms

Chronic or intermittent diarrhoea


Failure to thrive or faltering growth (in
children)
Persistent or unexplained
gastrointestinal symptoms including
nausea and vomiting
Prolonged fatigue ('tired all the time')
Recurrent abdominal pain, cramping or
distension
Sudden or unexpected weight loss
Unexplained iron-deficiency anaemia, or
other unspecified anaemia

Conditions

Autoimmune thyroid disease


Dermatitis herpetiformis
Irritable bowel syndrome
Type 1 diabetes
First-degree relatives (parents,
siblings or children) with coeliac
disease

Complications
anaemia: iron, folate and vitamin B12 deficiency (folate deficiency is more common than
vitamin B12 deficiency in coeliac disease)
hyposplenism
osteoporosis, osteomalacia
lactose intolerance
enteropathy-associated T-cell lymphoma of small intestine
subfertility, unfavourable pregnancy outcomes

Immunology
tissue transglutaminase (TTG) antibodies (IgA) are first-choice according to
NICE
endomyseal antibody (IgA)
anti-gliadin antibody (IgA or IgG) tests are not recommended by NICE
anti-casein antibodies are also found in some patients
Jejunal biopsy
villous atrophy
crypt hyperplasia
increase in intraepithelial lymphocytes
lamina propria infiltration with lymphocytes
Whipples disease diarrhea, steatorrhea, abdominal pain, weight loss, migratory
arthropathy, fever, and neurological symptoms. duodenal endoscopy, which reveals
PAS-positive macrophages
626. A 19yo man presents for the 1st time with a firm and unshakable belief that he is
being followed
by terrorists who are plotting against him. What is the single best term for this mans
condition?
a. Delusion of persecution
b. Delusion of grandeur
c. Delusion of control
d. Delusion of reference
e. Delusion of nihilism
a. Delusion of persecution
most common types of delusions, centering around a person's fixed, false belief that
others aim to obstruct, harm, or kill him/her.
Delusion of grandeur fixed, false belief that one possesses superior qualities such as
genius, fame, omnipotence, or wealth.
Delusion of control false belief that another person, group of people, or external force
controls one's general thoughts, feelings, impulses, or behavior.
Delusion of reference A neutral event is believed to have a special and personal
meaning. For example, a person with schizophrenia might believe a billboard or a
celebrity is sending a message meant specifically for them.
Delusion of nihilism the delusion that things (or everything, including the self) don't
exist. a sense that everything is unreal.
627. A 19yo female is brought in by her parents. They are concerned about her BMI
which is 12. She
is satisfied with it. What is the next step?
a. Psychiatric referral for admission
b. Family counselling

c. Social service
d. Start antidepo
e. Medical admission
e. Medical admission
bmi s 12..so next step s medical admission.

The defining clinical features are:


Refusal to maintain a normal body weight for age and height.
Weight below 85% of predicted. This means in adults a body mass index (BMI)
below 17.5 kg/m2.
Having a dread of gaining weight.
Disturbance in the way weight or shape is experienced, resulting in overevaluation of size.
Amenorrhoea for three months or longer
fatigue, hypothermia, hypotension, peripheral oedema, gaunt face, lanugo hair,
scanty pubic hair, acrocyanosis (hands or feet are red or purple), and bradycardia
Enhanced weight loss by over-exercise, diuretics, laxatives and self-induced vomiting
need for urgent referral and appropriate medical intervention

Nutrition: BMI below 14; weight loss more than 0.5 kg per week.
Circulation: systolic BP below 90; diastolic BP below 70; postural drop greater
than 10 mm Hg.
Squat test: unable to get up without using arms for balance or leverage.
Core temperature below 35C.
Blood tests: low potassium, sodium, magnesium or phosphate. Raised urea or
LFTs. Low albumin or glucose.
ECG: pulse rate below 50; prolonged QT interval.

628. A lady who works at a nursing home presents with itching. Exam: linear tracks on
the wrist. She
says that 2d ago she had come in contact with a nursing home inmate with similar
symptoms.
What is the mechanism of itching?
a. Infection
b. Destruction of keratinocytes
c. Allergic reaction
d. Immunosuppression
e. None
c. Allergic reaction
Scabies. pruritis due to allergic reaction.
629. A teacher had a respiratory infection for which she was prescribed antibiotics. After
the
antibiotic course when she rejoined school, she lost her voice completely. What is the
single
most appropriate dx?
a. Recurrent laryngeal nerve palsy
b. Angioedema
c. Laryngeal obstruction by medication
d. Laryngitis

e. Functional dysphonia/vocal cords


e. Functional dysphonia/vocal cords
Can't be recurrent laryngeal nerve because major reasons are it's either trauma via
surgery for thyroid or any neck surgery, tumour of neck,tumour superior vena cava,
tumour of mediastinal,metastasis etc.
can't b angioedema because it presents acutely with in minutes and usually superficial ie
visible sites though also involve deeper respiratory structures. Drug's notorious to cause
angioedema are A.C.E.i and in rare cases A.R.B.s and very rarely antibiotics antifungals
and can be any drug acute reaction.
Can't be laryngeal obstruction because it will eventually block your respiratory intake ie
breathing which this patient seems to having no problem with.
Similarly Laryngitis is an infection for which doctors do prescribe antibiotics but main
treatment is voice rest and gargles and further question says after use of antibiotics in
laryngitis voice hoarseness or loss is pre treatment mostly.
Antibiotics rarely cause voice loss.
Where no organic cause is found - a diagnosis of exclusion.
A common cause of hoarseness. There are various forms (below).
Infections
Acute laryngitis (common), often with upper respiratory infection. Usually viral (may have
secondary infection with staphylococci or streptococci).
Other infections - fungal or tuberculous.
Benign laryngeal conditions
Voice overuse - common.
Benign lesions of the vocal cords - eg, nodules (singer's nodes), polyps and papillomas.
630. A 43yo lady is admitted with pyrexia, arthropathy, breathlessness and syncope.
She was recently
dx with pulmonary emboli. There is an early diastolic sound and a mid-diastolic rumble.
Her JVP
is elevated with prominent a-waves. What is the most likely cause?
a. Mitral regurgitation
b. Ventricular ectopics
c. Pulmonary regurgitation
d. Atrial myxoma
e. Complete heart block
d. Atrial myxoma
Atrial Myxoma presents mostly with signs and symptoms of mitral stenosis if in left atrium, so there's mid
diastolic murmur and the other early diastolic sound is called tumour plop which is characteristic to the
impact of the tumour on the mitral valve in diastole as the valves in systole were closed and holding the

tumour above but when they open the Myxoma falls on the valve producing early diastolic plop.On top we
have extra cardiac symptoms
depending upon location,if myxoma obstructs the valve, then its diastolic rumble...if leaflets r damaged, then
its a systolic rumble(due to regurgitation)

Jugular venous pressure may be elevated, and a prominent A wave may be present.
A loud S1 is caused by a delay in mitral valve closure due to the prolapse of the tumor into the mitral
valve orifice (mimicking mitral stenosis).
P2 may be delayed. Its intensity may be normal or increased, depending on the presence of
pulmonary hypertension.
In many cases, an early diastolic sound, called a tumor plop, is heard. This sound is produced by the
impact of the tumor against the endocardial wall or when its excursion is halted.
An S3 or S4 may be audible.
A diastolic atrial rumble may be heard if the tumor is obstructing the mitral valve.
If there is valve damage from the tumor, mitral regurgitation may cause a systolic murmur at the
apex.
A right atrial tumor may cause a diastolic rumble or holosystolic murmur due to tricuspid
regurgitation.
General examination may reveal fever, cyanosis, digital clubbing, rash, or petechiae.

631. A 28yo man presents with a maculopapular rash over his trunk and palms. He also
has
numerous mouth ulcers. He had a penile ulcer which healed 2wks ago. What will you do
to
confirm the dx?
a. PCR for treponemal and non-treponemal antibiodies
b. Dark ground microscopy from mouth ulcer
c. Blood culture for treponema
d. Dengue fever
a. PCR for treponemal and nontreponemal antibodies
because it has now progressed to secondary syphilis and the investigation of choice is PCR
Dark ground microscopy is done from the chancre fluid and not from mouth ulcers
stage 1/ primary= dark field microscopy. 2= treponeme specific and non- antibodies, treponemes r seen in
the lesions too. Late secondary= organisms can no longer be seen but the AB tests are still +. tertiary= look
for FTA and TPHA antibodies in CSF.
PCR is the best for confirmation of any diagnosis??

Management
benzylpenicillin
alternatives: doxycycline
the Jarisch-Herxheimer reaction is sometimes seen following treatment. Fever,
rash, tachycardia after first dose of antibiotic. It is thought to be due to the
release of endotoxins following bacterial death and typically occurs within a few
hours of treatment.

632. A 34yo man complains of arthralgia, abdominal pain and vomiting, a facial rash that
is worse in
the summer and hematuria. Urea and creatinine are slightly elevated with urinalysis
demonstrating red cell casts. PMH is remarkable for childhood eczema. Which inv is
most likely
to lead to a dx?
a. US KUB
b. Joint aspiration
c. Auto antibodies
d. IVU
e. Renal biopsy
c. Autoantibodies
HSP mostly in children after a viral infection and with a palpable purpura on buttocks and extensor surfaces.
Si can't be HSP.
SLE yes I agree a lot of presentations going in favour of SLE. arthralgias rash photosensitivity renal
involvement. I don't recall abdominal pain and vomiting in SLE.
SLE - Facial Rash, worsen by sunlight, arthralgia, Nephritis, Other immune problems ( eczema )

General features
fatigue
fever
mouth ulcers
lymphadenopathy
Skin
malar (butterfly) rash: spares nasolabial folds
discoid rash: scaly, erythematous, well demarcated rash in sun-exposed areas.
Lesions may progress to become pigmented and hyperkeratotic before becoming
atrophic
photosensitivity
Raynaud's phenomenon
livedo reticularis
non-scarring alopecia
Musculoskeletal
arthralgia
non-erosive arthritis
Cardiovascular
myocarditis
Respiratory
pleurisy
fibrosing alveolitis
Renal

proteinuria
glomerulonephritis (diffuse proliferative glomerulonephritis is the most common
type)

Neuropsychiatric
anxiety and depression

psychosis
seizures

Autoantibodies:
ANA: screening test with a sensitivity of 95% but not diagnostic in the absence of
clinical features.
Anti-dsDNA: high specificity but sensitivity is only 70%.
633. A 56yo woman has had severe abdominal pain for 24h radiating to her back and is
accompanied
by nausea and vomiting. She appears to be tachycardic and in shock. She was found to
have
gallstones, 2yrs ago. What is the most likely inv to confirm dx?
a. US abdomen
b. LFT
c. Serum lipase
d. Angiography
e. CT abdomen
c. Serum lipase
lipase is done for diagnosing pancreatiitis.... CT is done to find the complications of pancreatitis..

Serum amylase 3 or more times normal is the traditional way of diagnosing acute
pancreatitis. However, lipase levels are more sensitive and more specific.

634. A 32yo female with axillary freckles and caf au lait spots wants to know the
cahnces of her child
also having similar condition.
a. 1:2
b. 1:4
c. No genetic link
d. 1:16
e. Depends on the genetic make up of the partner
a. 1:2 (or E)?
neurofibromatosis..autosomal dominant..so 1:2
if her partner has also same condition than there 75 %chances while he is normal than 50 % chances
we cant tell her for sure that there is 50 % of getting her baby affected when he husband is diseased.
So i think option E is correct answer in such scenerio

635. A 40yo man has pain, redness and swelling over the nasal end of his right lower
eyelid. The eye is watery with some purulent discharge. The redness extends on to the
nasal peri-orbital area
and mucoid discharge can be expressed from the lacrimal punctum. What is the single
most
appropriate clinical dx?
a. Acute conjunctivitis
b. Acute dacrocystitis
c. Acut iritis
d. Retrobulbar neuritis
e. Scleritis
b. Acute dacryocystitis
Dacryocystitis is infection of the lacrimal sac
Features
watering eye (epiphora)
swelling and erythema at the inner canthus of the eye
Management is with systemic antibiotics. Intravenous antibiotics are indicated if there is
associated periorbital cellulitis

636. A 60yo lady has severe chest pain. ECG shows changes of inferior wall MI. ECG
also shows
progressive prolongation of PR interval until a QRS complex is dropped. What is the
most
probable dx?
a. Atrial fibrillation
b. VT
c. SVT
d. Mobitz type I 2nd degree heart block
e. Mobitz type II 2nd degree heart block
d. Mobitz type I 2nd degree heart block
First degree heart block
PR interval > 0.2 seconds
Second degree heart block
type 1 (Mobitz I, Wenckebach): progressive prolongation of the PR interval until a
dropped beat occurs
type 2 (Mobitz II): PR interval is constant but the P wave is often not followed by
a QRS complex
Third degree (complete) heart block
there is no association between the P waves and QRS complexes
637. A 52yo woman speaks rapidly without any pause and ignores interruptions. She
doesnt even
pause to take enough breaths. What term best describes this kind of speech?
a. Flight of ideas
b. Brocas aphasia
c. Wernickes aphasia
d. Pressure of speech
e. Verbal dysphasia
d. Pressure of speech
Flight of ideas a nearly continuous flow of rapid speech that jumps from topic to topic
Brocas aphasia When a stroke injures the frontal regions of the left hemisphere, different kinds of
language problems can occur. This part of the brain is important for putting words together to form complete
sentences. Injury to the left frontal area can lead to what is called Brocas aphasia.
Wernickes aphasia People with serious comprehension difficulties have what is called Wernickes
aphasia.

638. A 30yo woman has been feeling low and having difficulty in concentrating since her
mother
passed away 2m ago. She feels lethargic and tends to have breathlessness and tremors
from
time to time. What is the most likely dx?
a. Adjustment disorder

b. PTSD
c. Panic disorder
d. GAD
e. Bereavement
a. Adjustment disorder
Adjustment disorder is a short-term condition that occurs when a person has great difficulty coping with, or
adjusting to, a particular source of stress, such as a major life change, loss, or event.
Unlike major depression, however, an adjustment disorder doesn't involve as many of the physical and
emotional symptoms of clinical depression (such as changes in sleep, appetite and energy) or high levels of
severity (such as suicidal thinking or behavior).
adj disorder starts within 3 m of the stress and does not last more than 6 m while bereavement starts w the
stress and does not last more than 2 months
It is not PTSD as in aetiology death of near one is not included and given case doesn't have the diagnostic
feature of repeated memory flashbacks or dream. Panic disorder does not occur in response to any external
or internal stress! So it is not panic disorder. Similarly GAD is chronic anxiety which is not directly related to
any object or situation. Which also does not explain death as a point in its favour! Beyond 2 months
bereavement is considered to be either pathological bereavement or major depression. So given time period
of 2 months indicates it is no more normal bereavement. So by exclusion I think it is a case of adjustment
disorder (of which death of near one is considered as an etiologic factor).
Bereavement is the time spent adjusting to loss. It has four stages accepting that your loss really happened
experiencing the pain that comes with grief
trying to adjust to life without the person who died
putting less emotional energy into your grief and finding a new place to put it i.e. moving on.
Since this woman is not able to adjust to the loss and is having physical symptoms I think that's the reason
it's adjustment disorder.

639. A 32yo man on psychiatric medications complains of inability to ejaculate. Which


drug is most
likely to cause these symptoms?
a. Lithium
b. Haloperidol
c. Chlorpromazine
d. Fluoxetine
e. Clozapine
d. Fluoxetine
SSRI's ( fluoxetine ) cause sexual dysfunction

gastrointestinal symptoms are the most common side-effect

640. A 4yo boy is brought by his parents with complains of wetting his bed at night and
whenever he
gets excited. What would be the most appropriate management for this child?
a. Desmopressin
b. Oxybutynin
c. Behavioural therapy
d. Tamsulosin
e. Restrict fluid intake
c. Behavioural therapy
Children below 7yrs : sleep alarms or behavioural therapy

Children above 7yrs : Desmopressin

Alarm training is a first-line treatment for nocturnal enuresis and is the most effective longterm strategy
Desmopressin should be offered first-line to children aged over 7 where rapid control is
needed or an alarm is inappropriate. Otherwise it should be used second-line after an alarm
has been tried. It may be used in children aged 5-7 if treatment is required under the same
circumstances.

641. A 34yo DM pt is undergoing contrast radiography. What measure should be taken


to prevent
renal damage with contrast dye?
a. Reduce contrast dye
b. Plenty of fluids
c. NSAIDS
d. ACEi
e. IV dextrose
b. Plenty of fluids
642. A 75yo woman presents to the breast clinic having noticed that she has had a blood
stained
discharge from the left nipple, together with dry skin over the left areola. Exam: blood
stained
discharge with dry flaky skin noted on the left areola. The nipple was noted to be
ulcerated. Wht
is the most appropriate inv?
a. FNAC
b. MRI
c. Punch biopsy
d. Open biopsy
e. Stereotactic biopsy
c. Punch biopsy
Pagets disease
A punch biopsy is when the doctor removes a small circle of skin tissue to biopsy. You might have this type
of biopsy if your doctor thinks you could have inflammatory breast cancer or Paget's disease of the nipple.

643. A 50yo man presents with low mood, poor concentration, anhedonia and insomnia.
He has had
2 episodes of increased activity, promiscuity and aggressive behavior in the past. He
was arrest
8m ago for trying to rob a bank claiming it as his own. Which drug is most likely to
benefit him?
a. Haloperidol
b. Citalopram
c. Desipramine
d. Carbamazepine
e. Ethosuximide
d. Carbamazepine
bipolar disorder

Lithium should be considered first-line, with the addition of valproate if ineffective.


Valproate or olanzapine should be considered for patients intolerant of lithium or
who are not prepared to undergo regular monitoring.
If symptoms still continue then the patient should be referred to a mental health
specialist. Medications that might be used in this situation are lamotrigine
(especially in bipolar II disorder) or carbamazepine.
If medication is stopped, patients should be made aware of early warning
symptoms of recurrence. Medication should be tailed off gradually

644. A 25yo woman complains of dizziness, nausea, vomiting, visual disturbances and
anxiety which
keep coming from time to time. Most of the attacks are a/w sudden change in posture.
What is
the most likely dx?
a. Panic disorder
b. Carotid sinus syncope
c. BPPV
d. Vertebrobasilar insufficiency
e. Postural hypotension
c. BPPV

vertigo triggered by change in head position (e.g. rolling over in bed or gazing
upwards)
may be associated with nausea
each episode typically lasts 10-20 seconds
positive Dix-Hallpike manoeuvre

BPPV has a good prognosis and usually resolves spontaneously after a few weeks to
months. Symptomatic relief may be gained by:
Epley manoeuvre (successful in around 80% of cases)
teaching the patient exercises they can do themselves at home, for example
Brandt-Daroff exercises
Medication is often prescribed (e.g. Betahistine) but it tends to be of limited value.

645. A 56yo man was recently put on anti-HTN meds and recent biochemistry on 2
occasions showed: Na+=132, K+=7.6, Urea=11.3, Creat=112. Which of the following
drugs is responsible for this
result?
a. Amlodipine
b. Bendroflumethiazide
c. Doxazosin
d. Atenolol
e. Ramipril
e. Ramipril
no Angiotensin II >> no Aldosterone >> hyponatremia, hyperkalemia.

Side-effects:

cough: occurs in around 15% of patients and may occur up to a year after
starting treatment. Thought to be due to increased bradykinin levels
angioedema: may occur up to a year after starting treatment
hyperkalaemia
first-dose hypotension: more common in patients taking diuretics

Cautions and contraindications


pregnancy and breastfeeding - avoid
renovascular disease - significant renal impairment may occur in patients who
have undiagnosed bilateral renal artery stenosis
aortic stenosis - may result in hypotension
patients receiving high-dose diuretic therapy (more than 80 mg of furosemide a
day) - significantly increases the risk of hypotension
hereditary or idiopathic angioedema
646. A 46yo woman has offensive yellow discharge from one nipple. She had a hx of
breast abscess
3yrs ago. What is the possible dx?
a. Duct papilloma
b. Duct ectasia
c. Duct fistula
d. Breast cancer
c. Duct fistula
as there is H/O abscesses which might have led to fistula formation. Clincher here is previous H/O abscess
In cancer n papilloma discharge is blood stained mainly and ectasia it is greenish clear discharge from
multiple ducts

Breast cancer Characteristically a hard, irregular lump. There may be associated nipple inversion or skin
tethering
Paget's disease of the breast - intraductal carcinoma associated with a reddening and thickening (may
resemble eczematous changes) of the skin/areola
Mammary duct ectasia Dilatation of the large breast ducts
Most common around the menopause
May present with a tender lump around the areola +/- a green nipple discharge
If ruptures may cause local inflammation, sometimes referred to as 'plasma cell mastitis'
Duct papilloma Local areas of epithelial proliferation in large mammary ducts
Hyperplastic lesions rather than malignant or premalignant
May present with blood stained discharge
Breast abscess More common in lactating women
Red, hot tender swelling

647. A 35yo woman undergoing tx for TB presents with malar rash, photosensitivity and
hematuria.
What is the single most likely positive antibody?
a. Anti Ds DNA
b. Anti Sm
c. Anti Histone
d. Anti La
e. Anti centromere

c. Anti Histone
drug induced lupus.
Most common causes
procainamide
hydralazine
Less common causes
isoniazid
minocycline
phenytoin
648. A 6wk child with profuse projectile vomiting. What is the first thing you will do?
a. US
b. Check serum K+ level
c. ABG
d. NG tube
e. IV fluids
b. Check serum K+ level
vomiting causes metabolic alkalosis.initially hypokalemia and later hyponatremia
This seems a case of Pyloric stenosis
Definitive diagnosis is done by US showing olive like
But on blood tests it shows low blood levels of potassium and chloride in association with an increased
blood pH and high blood bicarbonate level due to loss of stomach acid (which contains hydrochloric acid)
from persistent vomiting.
6 weeks is the typical age for pyloric stenosis presentation

649. A 55yo woman who attends the clinic has recently been dx with a depressive
episode. She
complains of unintentionally waking early in the morning, a recent disinterest in sex and
a loss
of appetite, losing 5kg weight in the last month. She feels that her mood is worse at the
beginning of the day. What is the most likely dx for this pt?
a. Mild depression
b. Moderate depression
c. Severe depression
d. Low mood
e. Pseudo depression
b. Moderate depression
Physical symptoms like weight loss and early morning insomnia makes it moderate as opposed to mild
Pseudodepression
"A condition of personality following frontal lobe lesion in which apathy, indifference and a loss of initiative
are apparent symptoms but are not accompanied by a sense of depression in the patient."

Q.621 for more


650. An employer sent his worker to the ED after having hit his head on a machine.
Exam: normal. What is the single most likely inv you would do?

a. Skull XR
b. CT head
c. MRI head
d. Reassure
a. Skull XR
CT head only if
Loss of consciousness more than 5 minutes,
More than two episodes of vomiting,
Signs of base of skull fracture ( peri orbital haematoma, bleeding or CSF leak from nose or ears),
Fits,
Double vision,
Headache not easing off with paracetamol or ibuprofen,
Unusual drowsiness/lOW GCS,
If none of the above discharge with head injury advise which means if any of the above develop in next 24
he pt should come back to A&E,
This is current NICE head injury guideline.

651. A lady with fam hx of ovarian carcinoma has a pelvis US that fails to reveal any
abnormality.
What is the single most appropriate inv?
a. Pelvic CT
b. CA 125
c. CA 153
d. Laparoscopy
e. MRI
b. CA 125
Pelvic usg is not as sensitive as ca 125. So even if pelvic usg fails to detect a small lesion, it can still be
detected by a rise in ca 125.

652. A 10yo boy is taken to his GP by his parents with behavioural prbs. He attends a
special school
due to inappropriate behavior and during the interview with his parents the boy barks at
infrequent episodes and shouts expletives. What is the most likely dx?
a. Asperger syndrome
b. Cotard syndrome
c. Rett syndrome
d. Ekbom syndrome
e. Tourettes syndrome
e. Tourettes syndrome
tourette syndrome (multiple motor tics + vocal / phonetic tics)

Asperger syndrome: autism spectrum disorder (autism: a mental condition, present from early
childhood, characterized by great difficulty in communicating and forming relationships with other people and
in using language and abstract concepts.) The main difference from classic autism is a lack of delayed or
retarded cognition and language. Those with AS are also more likely to seek social interaction and share
activities and friendships.
In classic autism, children tend to be spotted earlier (18-30 months) because of impaired communication. In
Asperger's syndrome (AS), the diagnosis comes later - usually at school entry, when socialisation becomes
necessary. Many people with AS may learn to mask their problems. They may present as patients with no

serious mental health problem, but who are anxious, lonely, have a poor employment record and just don't
seem to fit in.

Cotard syndrome: afflicted person holds the delusion that he or she is dead, either figuratively or
literally.

Rett syndrome: small feet ,hands & head ,no speech no walking, repeated hand movement. Onset
occurs between 6 and 18 months of age. deceleration of the rate of head growth

ekbom syndrome: delusional parasitosis delusional belief that they are infested with parasites,
whereas in reality no such parasites are present.
Wittmaack-Ekbom syndrome: a synonym of restless legs syndrome

Tourettes syndrome Tics can be defined as sudden, purposeless, repetitive, non-rhythmic,


stereotyped movements or vocalisations - eg, eye twitching or blinking. Examples of vocal tics are throat
clearing, grunting and barking.

Other features that may be seen in Tourette's syndrome

Echolalia - involuntary copying of other's' words.


Palilalia - repeating one's own words.
Coprolalia - compulsory saying of dirty words, which is pathognomonic of the
syndrome and is seen in about 10% of patients.
Copropraxia - making obscene gestures.
Echopraxia - involuntary copying of other's movements.
Difficulty concentrating or easily distracted.

653. A 52yo male presents with sudden complete loss of vision from right eye. He also
had been
complaining of right sided headaches which would come up more on chewing. On
fundoscopy,
the retina was pale and a cherry red spot could be seen in the macular region. What
caused this
vision loss?
a. CRAO
b. CRVO
c. Branch RAO
d. Branch RVO
e. Circumciliary vein occlusion
a. CRAO
pale optic disc, cherry red spot on macula

The most common causes of a sudden painless loss of vision are as follows:
ischaemic optic neuropathy (e.g. temporal arteritis or atherosclerosis)
occlusion of central retinal vein
occlusion of central retinal artery
vitreous haemorrhage
retinal detachment

Ischaemic optic neuropathy


may be due to arteritis (e.g. temporal arteritis) or atherosclerosis (e.g. hypertensive,
diabetic older patient)
due to occlusion of the short posterior ciliary arteries, causing damage to the optic
nerve
altitudinal field defects are seen
Central retinal vein occlusion
incidence increases with age, more common than arterial occlusion
causes: glaucoma, polycythaemia, hypertension
severe retinal haemorrhages are usually seen on fundoscopy
Central retinal artery occlusion
due to thromboembolism (from atherosclerosis) or arteritis (e.g. temporal arteritis)
features include afferent pupillary defect, 'cherry red' spot on a pale retina
Vitreous haemorrhage
causes: diabetes, bleeding disorders
features may include sudden visual loss, dark spots
Retinal detachment
features of vitreous detachment, which may precede retinal detachment, include
flashes of light or floaters

654. A 48yo woman presents with left-sided severe headache. She also has a red,
watering eye and complains of seeing colored haloes in her vision. What is the most
appropriate next step?
a. Measure IOP
b. Relieve pain with aspirin
c. 100% oxygen
d. CT
e. Relieve pain with sumatriptan
a. Measure IOP
Red watery eye point to cluster headache but that's occur in young males...all other points to glaucoma
haloes seen in glaucoma. Measure iop

Features
severe pain: may be ocular or headache
decreased visual acuity
symptoms worse with mydriasis (e.g. watching TV in a dark room)
hard, red eye
haloes around lights
semi-dilated non-reacting pupil
corneal oedema results in dull or hazy cornea
systemic upset may be seen, such as nausea and vomiting and even abdominal pain

Management
urgent referral to an ophthalmologist
management options include reducing aqueous secretions with acetazolamide and
inducing pupillary constriction with topical pilocarpine

655. A 31yo woman presents with 7-10days following childbirth, with loss of feeling for
the child, loss
of appetite, sleep disturbance and intrusive and unpleasant thoughts of harming the
baby. What
is the best tx for this pt?
a. Fluoxetine
b. Haloperidol
c. CBT
d. Reassurance
e. ECT
e. ECT
Antipsychotics are given taking into account the breastfeeding factor. Usually lithium is given but ECT is
better way to go. Why we need ECT? As this is psychosis and there is danger to both mother and child we
need to control the situation rapidly with high intensity psychological intervention

Postpartum psychosis is a psychiatric emergency. It requires urgent assessment, referral,


and usually admission, ideally to a specialist mother and baby unit.[8]
Management is primarily pharmacological, using the same guidance as for other causes of
psychosis. Medication would normally involve an antipsychotic and/or mood stabilising drug.
However, choice of medication must take breastfeeding into account. Mothers requiring
lithium treatment should be encouraged not to breast-feed, due to potential toxicity in the
infant. Most antipsychotics are excreted in the breast milk, although there is little evidence of
it causing problems. Where they are prescribed to breast-feeding women, the baby should
be monitored for side-effects. Clozapine is associated with agranulocytosis and should not
be given to breast-feeding women. Electroconvulsive therapy (ECT) may also be considered
in some cases

656. A 56yo male pt presents with intermittent vertigo, tinnitus and hearing loss. What is
the best
drug tx for this pt?
a. Buccal prochlorperazine
b. Oral flupenphenazine
c. TCA
d. Gentamicin patch on the round window
e. No med tx available
a. Buccal prochlorperazine
meiners disease-t/t oral percholperazine

Features
recurrent episodes of vertigo, tinnitus and hearing loss (sensorineural). Vertigo is
usually the prominent symptom
a sensation of aural fullness or pressure is now recognised as being common
other features include nystagmus and a positive Romberg test
episodes last minutes to hours
typically symptoms are unilateral but bilateral symptoms may develop after a number
of years
Natural history
symptoms resolve in the majority of patients after 5-10 years
the majority of patients will be left with a degree of hearing loss
psychological distress is common
Management
ENT assessment is required to confirm the diagnosis
patients should inform the DVLA. The current advice is to cease driving until
satisfactory control of symptoms is achieved
acute attacks: buccal or intramuscular prochlorperazine. Admission is sometimes
required
prevention: betahistine may be of benefit

657. An 82yo woman has developed painful rash on one side of her forehead and ant
scalp. Lesions
have also affected her cornea. What is the single most appropriate option?
a. Accessory nerve
b. Facial nerve
c. Olfactory nerve
d. Optic nerve
e. Trigeminal nerve
e. Trigeminal nerve
Corneal involvement. Forehead n scalp. All are area of supply of trigeminal

658. A 24yo woman presents with episodes of peri-oral tingling and carpo pedal spasms
every time
she has to give a public talk. This also happens to her before interviews, exams and
after
arguments. What is the best management strategy for this pt?
a. Diazepam
b. Rebreathe in a paper bag
c. Desensitization
d. Buspirone
e. Propranolol
b. Rebreathe in a paper bag??? many confusing answers
She has co2 washout which results in hypocalcemia hence the Peri oral tingling and carpo pedal spasms

Desensitization is for phobias...like arachnophobia


buspirone is for smoking cessation
For acute attack.. Rebreathe into paper bag
For prophylaxis just like when a pt has to give a public talk or appear in an interview..beta blocker.
.propanolol.
Best Mx is CBT. Desensitization
If CBT doesn't help we go for medical Mx.. SSRI
questions mentions all the events she is worried about and get symptoms from.
What I think is that this is panic disorder which starts at the time public speaking and ends after its over. As
opposed to GAD which is persistent and long lasting.
And if we go according to management of panic attack. Step 1 is education
Step 2 is cbt or ssri.
So i think in this case cbt is the best answer.
If it would be that the patient needs something to calm down while an exam then propranolol would be best.
I mean for one time event.
And if it would be acute and first time episode then rebreathing would be way to go.

659. A 32yo woman P3 of 39wks gestation reports having spontaneous ROM 4days
ago. She didnt attend the delivery suite as she knew that would happen and had already
decided on a home
birth. Today she feels very hot and sweaty. She thought that she was starting to have
labour
pains but she describes the pain as more constant. Exam: uterus is tender throughout.
Blood
tests show raised CRP and WBC. Select the most likely dx?
a. Round ligament stretching
b. Chorioamnionitis
c. Uterine rupture
d. Labor
e. DIC
b. Chorioamnionitis
The characteristic clinical signs and symptoms of chorioamnionitis include the following:

Maternal fever (intrapartum temperature >100.4F or >37.8C): Most frequently observed sign
Significant maternal tachycardia (>120 beats/min)
Fetal tachycardia (>160-180 beats/min)
Purulent or foul-smelling amniotic fluid or vaginal discharge
Uterine tenderness
Maternal leukocytosis (total blood leukocyte count >15,000-18,000 cells/L)

The standard drug treatment in the mother with chorioamnionitis includes ampicillin and an
aminoglycoside (ie, usually gentamicin), although clindamycin may be added for anaerobic pathogens.
Clindamycin may also be used if the mother is allergic to penicillin

660. A 63yo man continues to experience chest pain and has a temp of 37.8C 2 days
after an acute
MI. His ECG shows widespread ST elevation with upward concavity. What is the single
most
likely explanation for the abnormal inv?
a. Acute pericarditis

b. Cardiac tamponade
c. Atrial thrombus
d. Left ventricular aneurysm
e. Dressler syndrome
a. Acute pericarditis
Pericarditis in the first 48 hours following a transmural MI is common
Features
chest pain: may be pleuritic. Is often relieved by sitting forwards
other symptoms include non-productive cough, dyspnoea and flu-like symptoms
pericardial rub
tachypnoea
tachycardia
Causes

viral infections (Coxsackie)


tuberculosis
uraemia (causes 'fibrinous' pericarditis)
trauma
post-myocardial infarction, Dressler's syndrome
connective tissue disease
hypothyroidism

ECG changes
widespread 'saddle-shaped' ST elevation
PR depression: most specific ECG marker for pericarditis

Dressler's syndrome tends to occur around 2-6 weeks following a MI. The underlying
pathophysiology is thought to be an autoimmune reaction against antigenic proteins formed
as the myocardium recovers. It is characterised by a combination of fever, pleuritic pain,
pericardial effusion and a raised ESR. It is treated with NSAIDs.

661. A 55yo man presents with an ulcer of the scrotum. Which of the following LN is
involved?
a. External iliac LN
b. Pre-aortic LN
c. Aortic LN
d. Inguinal LN
e. Iliac LN
f. Submental LN
g. Submandibular LN
h. Deep cervical LN
D. Inguinal LN

662. A 35yo woman has butterfly rash on her face and she suffers symmetrical joint
pains on knee
and elbow, ESR is raised. What is the most discriminative inv for dx?
a. Anti DNA antibodies
b. Anti Jo1 antibodies
c. Anti nuclear antibodies

d. Anti centromere antibodies


e. Anti la antibodies
a. Anti DNA antibodies
in diagnostic critera for SLE,we have four antibody.Among them most specific is anti double stranded DNA
antibody.So answer is a.

Immunology
99% are ANA positive
20% are rheumatoid factor positive
anti-dsDNA: highly specific (> 99%), but less sensitive (70%)
anti-Smith: most specific (> 99%), sensitivity (30%)
also: anti-U1 RNP, SS-A (anti-Ro) and SS-B (anti-La)
Monitoring
ESR: during active disease the CRP is characteristically normal - a raised CRP may
indicate underlying infection
complement levels (C3, C4) are low during active disease (formation of complexes
leads to consumption of complement)
anti-dsDNA titres can be used for disease monitoring (but note not present in all
patients)

663. Pt had a fight following which he developed bleeding, ringing and hearing loss from
one ear.
What is the inv of choice?
a. CT
b. XR skull
c. Otoscopy
d. MRI vestibule
e. Coagulation study
a. CT
CT scan to rule out basilar skull fracture esp when there is history of fight, bleeding from ear
(hemotympanum)
If bleeding and tinnitus is present it is almost certain that the patient has a traumatic perforation...
So now our main aim is to rule out fracture base of skull which can be best done by a CT scan.

Selection of adults for CT scan

CT scan of the brain within one hour (with a written radiology report within one
hour of the scan being undertaken):
Glasgow Coma Scale (GCS) <13 when first assessed or GCS <15 two
hours after injury
Suspected open or depressed skull fracture
Signs of base of skull fracture*
Post-traumatic seizure
Focal neurological deficit
>1 episode of vomiting
All patients with a coagulopathy or on oral anticoagulants should have a CT
brain scan within eight hours of the injury, provided there are no other identified
risk factors, as listed above. Again, a written radiology report should be
available within one hour of the scan being undertaken.

664. A 35yo IVDA on penicillin and flucloxacillin for cellulitis now presents with jaundice,
pale stools
and dark urine. What is the single most likely dx?
a. Hep A
b. Cholestatic jaundice
c. Chronic active hepatitis
d. Primary biliary cirrhosis
e. Hep B
b. Cholestatic jaundice
Cholestatic jaundice associated with flucloxacillin therapy.
Although it's a vague scenario, the diagnosis of drug induced hepatic injury ( DIHI ) requires these attributes
1) history of drug exposure 2) exclusion of other causes of hepatic injury , like hepatitis etc 3) improvement
after stopping the suspected drug 4) exacerbation/recurrence after the use of offending drug
Pale stool = cholestasis

665. A 79yo woman has been dx with T2DM. Her BMI=22. RBS are 8 and 10mmol/l. Her
BP=130/80mmHg. Her fasting cholesterol=5.7mmol/l. She is currently symptom-free but
has
microalbuminuria. What is the single most appropriate drug management?

a. ACEi and glibenclamide


b. ACEi and metformin
c. Statin and ACEi
d. Statin and glibenclamide
e. Statin and metformin
c. Statin and ACEi
Statins with acei to decrease cholesterol and prevent protinurea.
We are giving ACE I because of microalbuminuria. And Statins because of high cholesterol. You never start
off with oral hypoglycemics straightaway. Her BMI is ok. Lifestyle modifications n exercise would be advised
if microalbuminuria is present which is a microvascular complication- nephropathy, v add ACEIs/sartan to
inhibit renin angiotensin system to protect kidneys even if BP is normal ! plus statin here for raised chol
levels

ACE inhibitors are also used to treat diabetic nephropathy and have a role in secondary
prevention of ischaemic heart disease.
People with IFG should then be offered an oral glucose tolerance test to rule out a diagnosis
of diabetes. A result below 11.1 mmol/l but above 7.8 mmol/l indicates that the person
doesn't have diabetes but does have IGT.'
The diagnosis of type 2 diabetes mellitus can be made by either a plasma glucose or a
HbA1c sample. Diagnostic criteria vary according to whether the patient is symptomatic
(polyuria, polydipsia etc) or not.
If the patient is symptomatic:
fasting glucose greater than or equal to 7.0 mmol/l
random glucose greater than or equal to 11.1 mmol/l (or after 75g oral glucose
tolerance test)

666. A 68yo woman is unable to extend the IP joint of her right thumb 7wks following a fx
of the right radius. Other finger and thumb movements are normal. What is the single
most likely tendon to be damaged?
a. Abductor pollicis longus
b. Extensor pollicis brevis
c. Extensor pollicis longus
d. Flexor digitorum profundus
e. Flexor pollicis longus
C. Extensor pollicis longus
Extensor pollicis brevis is inserted at the proximal phalanx of thumb meaning at MCP joint. Its
function is to extend MCP of thumb. However extensor pollicis longus is inserted at distal phalanx of
thumb so it causes extension at Interphalangeal joint and also at MCP joint of thumb.

667. A mother presents her 6m son who is vocalizing. She has noticed that he doesnt
respond to
loud noises. His motor milestones are normal. What is the best management strategy?
a. Arrange hearing test
b. Assess development milestones
c. Reassure
d. Refer to speech therapist

e. MRI brain
a. Arrange hearing test
3 months

Quietens to parents voice


Turns towards sound
Squeals

668. A 39yo man presents to the ED with persistent cough, sputum and dyspnea. He
gave a hx of
smoking 20 cigarettes/d for the last 10 years. Pt was given oxygen in ambulance but he
is not
improving. What is the next step?
a. Prednisolone
b. Salbutamol
c. Check ABG
d. CXR
e. ECG
c. Check ABG
to assess hypoxia and co2 levels. it seems like this man has a chest infection. Salbutamol would be more
appropriate if they mentioned wheezy chest
COPD,with type two respiratory failure, patient is not getting better O2, so ABGs to proceed further to
confirm dx and treat.
>> whenever you initiate or change oxygen therapy, do an ABG within the next hour or sooner, if the patient
is deteriorating.

NICE guidelines from 2010 recommend the following:


increase frequency of bronchodilator use and consider giving via a nebuliser
give prednisolone 30 mg daily for 7-14 days
it is common practice for all patients with an exacerbation of COPD to receive
antibiotics. NICE do not support this approach. They recommend giving oral
antibiotics 'if sputum is purulent or there are clinical signs of pneumonia'
669. A 66yo woman has been brought to the hospital on a number of occasions with a
hx of loss of memory. Her PMH is significant for a MI 6yrs ago. It is noted that she has a
step wise decline of
her cognitive functions. What is the most likely dx?
a. Alzhemiers
b. Vascular dementia
c. Picks dementia
d. Huntingtons disease
e. Lewy body dementia
b. Vascular dementia
step-wise is a clincher for vascular
History of multiple loss of memory is vascular dementia. She also has vascular problems indicates by MI.
Step wise decline is characteeistic of it as well.

Step laddar detoriation of cognitive function is characteristic feature of vascular dementia which is also
supported by the h/o MI, a vascular condition.
Alzheimers ussually has an insidious onset
Step wise deterorition in memory >h/o multiple TIAs> vascular
Gradual >alZihemer
Fluctuating > features of parkinsonism> lewy body dementia
Associted with depression >pseudo dementia
Strong family history>hungtington's
Impaired cognition> memory is usually normal> pick's dementia

670. A 55yo man returns for routine follow up 6wks after a MI. He gets breathless when
walking
uphill. His ECG shows ST elevation in leads V1, V2, V3 and V4. What is the single most
likely
explanation for the abnormal investigation?
a. Heart block
b. Right ventricular strain
c. Atrial thrombus
d. Left ventricular aneurysm
e. Dresslers syndrome
d. Left ventricular aneurysm
S t elevation 6 weeks after mi...with no other major symptoms... Also a continuous St elevation and a history
of mi points towards an aneurysm
persistent ST elevation after few months of acute MI,D/D-1.ventricular aneurysm 2.underkinetic wall motion
disorder. N.B -contrary to sounding fatal,there is neither predisposition nor any association of cardiac rupture
in ventricular aneurysm.
Ventricular aneurysm occurs at the site of previous STEMI. V1-V4 involvement indicates previous
anteroseptal MI ( caused by LAD obstruction ) . This localized involvement of leads almost excludes
Dressler's syndrome where pericarditis causes ST elevation in all but aVR leads.

Left ventricular aneurysm


The ischaemic damage sustained may weaken the myocardium resulting in aneurysm
formation. This is typically associated with persistent ST elevation and left ventricular
failure. Thrombus may form within the aneurysm increasing the risk of stroke. Patients
are therefore anticoagulated.
Dressler's syndrome tends to occur around 2-6 weeks following a MI. The underlying
pathophysiology is thought to be an autoimmune reaction against antigenic proteins
formed as the myocardium recovers. It is characterised by a combination of fever,
pleuritic pain, pericardial effusion and a raised ESR. It is treated with NSAIDs.

671. A 4m girl has severe FTT and increasing jaundice which was 1st noticed at 1wk of
age. She has an enlarged liver and scratches on her skin. Her parents have been unable
to seek medical care.
What is the most likely dx?
a. Biliary atresia
b. G6PD deficiency
c. Hep B

d. Spherocytosis
a. Biliary atresia
gradual increasing of jaundice with obstructive jaundice indicate this. so USG to confirm this.G6PD def is
hereditary disease of XLR inheritance and always presented when aggravating factor present (some
medication) and otherwise asymptomatic. congenital spherocytosis not presented with this early stage upto
development of gallstone due to haemolysis. hepB not possible due to it causes hepatocellular picture.
Galactosemia: Poor weight gain, poor feeding, irritable, jaundice.
B. Atresia: Jaundice noticed usually at 1 wk, progressive, pale stools, dark urine.
G6PD Def: Jaundice comes usu after/ during illness, off and on.

672. A 76yo man suddenly collapsed and died. At post mortem exam, a retroperitoneal
hematoma
due to ruptured aortic aneurysm was noted. What is the most likely underlying cause of
the
aortic aneurysm?
a. Atheroma
b. Cystic medial necrosis
c. Dissecting aneurysm
d. Polyarteritis nodosa
e. Syphilis
a. Atheroma

risk factors include:

Severe atherosclerotic damage of the aortic wall; however, new evidence


suggests this is not the only factor, and aneurysmal disease is probably a
distinct arterial pathology.
Family history - there are probably strong genetic factors. About 15% of firstdegree relatives of a patient with an AAA, mainly men, will develop an
aneurysm.[6]
Tobacco smoking is an important factor.
Male sex.
Increasing age.
Hypertension.
Chronic obstructive pulmonary disease.
Hyperlipidaemia.
In population-based studies, people with diabetes have a lower incidence of
aneurysms than people without diabetes.

673. A 33yo male came to the hospital with complaint of occasional left sided chest pain
that lasts
<30mins, following exercise, which relieves upon taking rest. What is the most probable
dx?
a. Unstable angina
b. Decubitus angina
c. Stable angina
d. Coronary spasm

e. MI
c. Stable angina

Stable angina is when the pain is precipitated by predictable factors - usually


exercise.
Unstable angina: angina occurs at any time and should be considered and
managed as a form of acute coronary syndrome.

Decubitus angina occurs when the patient lies down. It is usually a complication of cardiac
failure due to the strain on the heart resulting from the increased intravascular volume.
Patients usually have severe coronary artery disease.

A full 12-lead ECG may show some ischaemic changes but a normal ECG does not rule out
a diagnosis of angina. Changes on a resting 12-lead ECG that are consistent with CAD
include:
Pathological Q waves.
Left bundle branch block (LBBB).
ST-segment and T-wave abnormalities (eg flattening or inversion).
when an attack of angina occurs, they should:
Stop what they are doing and rest.
Use glyceryl trinitrate (GTN) spray or tablets as instructed.
Take a second dose of GTN after 5 minutes if the pain has not eased.
Take a third dose of GTN after a further 5 minutes if the pain has still not eased.
Call 999/112/911 for an ambulance if the pain has not eased after another 5
minutes (ie 15 minutes after onset of pain), or earlier if the pain is intensifying or
the person is unwell.
Medication
all patients should receive aspirin and a statin in the absence of any
contraindication
sublingual glyceryl trinitrate to abort angina attacks
NICE recommend using either a beta-blocker or a calcium channel blocker first-line
based on 'comorbidities, contraindications and the person's preference'
if a calcium channel blocker is used as monotherapy a rate-limiting one such as
verapamil or diltiazem should be used. If used in combination with a beta-blocker
then use a long-acting dihydropyridine calcium-channel blocker (e.g. modifiedrelease nifedipine). Remember that beta-blockers should not be prescribed
concurrently with verapamil (risk of complete heart block)
if there is a poor response to initial treatment then medication should be increased to
the maximum tolerated dose (e.g. for atenolol 100mg od)
if a patient is still symptomatic after monotherapy with a beta-blocker add a calcium
channel blocker and vice versa
if a patient is on monotherapy and cannot tolerate the addition of a calcium channel
blocker or a beta-blocker then consider one of the following drugs: a long-acting
nitrate, ivabradine, nicorandil or ranolazine

if a patient is taking both a beta-blocker and a calcium-channel blocker then only add
a third drug whilst a patient is awaiting assessment for PCI or CABG

674. A 42yo overweight smoker comes with heavy periods. A scan reveals a normal
uterus. She
would like a long term tx with minimal side effects that would offer tx for the menorrhagia
and
provide contraception. She is unsure whether she would like more children. She is
adamant that
she doesnt want surgery as she is terrified of the prospect. Select the best management
for her
menorrhagia?
a. COCP
b. GrH analogues
c. IU/systemic progesterone
d. NSAIDs
e. Copper containing IUCD
c. IU/systemic progesterone
IUS. mirena coil. it provides relief of the menorrhagia and provides long term contraception for up to 5 years.
As pt is smoker COCP is not suitable and patient has menorrhagia. In this case C is better option i think.
Copper containing IUCD is not indicated in unexplained/undiagnosed vaginanl bleeding.
Gnrh analogues mostly used for menorehagia n not contracep

Menorrhagia was previously defined as total blood loss > 80 ml per menses, but it is
obviously difficult to quantify.
Does not require contraception
either mefenamic acid 500 mg tds (particularly if there is dysmenorrhoea as well) or
tranexamic acid 1 g tds. Both are started on the first day of the period
if no improvement then try other drug whilst awaiting referral
Requires contraception, options include
intrauterine system (Mirena) should be considered first-line
combined oral contraceptive pill
long-acting progestogens
Norethisterone 5 mg tds can be used as a short-term option to rapidly stop heavy menstrual
bleeding.

675. A 10yo male child was brought by his mother complaining that her child watches TV
at very high volumes, doesnt like to play outside and instead has become more sincere
with reading. She
also says that her son doesnt respond to her. What do you expect to see on otoscopy?

a. Flamingo pink tympanic membrane


b. Attic perforation
c. Bluish grey tympanic membrane with air fluid levels
d. Inflamed tympanic membrane with cart wheel appearance of vessels
e. Red and inflamed tympanic membrane
c. Bluish grey tympanic membrane with air fluid levels
serous otitis media or OM with effusion...often first presenting symptom is watching tv at high volume

676. A lady underwent debulking surgery for ovarian carcinoma. Soon after the surgery
she presents
with signs of intestinal obstruction. What is the single most appropriate inv?
a. Pelvic CT
b. CA 125
c. Laparotomy
d. Laparoscopy
e. Abdominal US
c. Laparotomy?
Abdominal CT then Laparotomy, while abdominal CT is not mentioned here so I will choose laparotomy.
since the patient has stage 4 cancer, further management with the patient is focused on palliative care. with
intestinal obstruction, laparotomy would be able to locate the level of obstruction, and place a colostomy/
ileostomy to relieve the obstruction
Laparotomy.
Or CECT abdomen which is not in the choice.
Better never open without imaging.
In Ca Ovary, Omental cake is one of the biggest challenge which leads to recurrent obstruction.
And also loculated ascites.

677. A 45yo woman undergoing tx for RA for the last 5yrs presents with dizziness, easy
fatigabiliy and lack of energy. A blood film shows MCV 106. What could be the most
probable reason for her
anemia?
a. Steroids
b. Chronic disease
c. NSAIDs
d. Methotrexate
e. Sulfasalazine

d. Methotrexate
due to def of folate megaloblastic anemia.as methotrexate is an anti folate drug
methotrexate cause many side effects including macrocytic anemia, liver & pulmonary toxicity, BM supp.
,stomatitis, rash, nausea and alopecia ..
Anaemia of chronic disease is either normocytic normochronic or micro hypo
-MTX & sulfasalazine can both give rise to high-MCV anemia, MTX by anti-folate effect & salfasalazine by
hemolytic process which result in high reticulocyte count (which are large cells) and hence high MCV
values...but statistically speaking, MTX anemia & myelosuppression is more common that salfasalazineinduced hemolytic anemia (about 10 times).
-NSAID & steroids can cause microcytic anemia (low MCV, low ferritin & iron,high TIBC and high soluble
transferrin receptor) due to GI bleeding and iron deficiency.
-Long standing RA can cause also anemia of chonic disease (low MCV,high ferritin, low iron,low TIBC and
low soluble transferrin receptor).
-Miscellaneous causes of anemia in RA includes: GI bleeding due to vasculitis, Felty syndrome, non-hodgkin
lymphoma (T-large granular lymphocyte lymphoma), all of which are microcytic (i.e low MCV).

Antifolate drugs:
With uncertain mechanism of action (e,g anticonvulsants and possibly alcohol
and nitrofurantoin)
Causing malabsorption of folate (eg, cholestyramine, sulfasalazine,
methotrexate)
Trimethoprim may exacerbate pre-existing folate deficiency but does not cause
megaloblastic anaemia

678. A 62yo man who has recently had flu-like illness has woken to find his speech
altered.
Movement of his eyelids and lips are weak on the right side. Exam: normal. Which
anatomical
site is most likely to be affected?
a. Facial nerve
b. Hypoglossal nerve
c. Oculomotor nerve
d. Trigeminal nerve
e. Glossopharyngeal nerve
a. Facial nerve
bell's palsy
679. A 5yo girl has had an URTI for 3 days and has been treated with paracetamol by
her mother. For the last 12h she has been hot and irritable with severe pain in her right
ear. What is the most
likely dx?
a. Herpes zoster infection
b. Impacted ear wax
c. Mumps
d. OM
e. Perforation of eardrum

d. OM
AOM is a condition in which there is inflammation of the middle ear, frequently in association
with an upper respiratory tract infection (URTI). It commonly presents with:

Pain
Malaise
Irritability
Fever
Vomiting

The fever is often very high and may be associated with febrile convulsions.
A well-recognised complication is that a child who is screaming and in a great deal of pain
finally settles and the ear starts to discharge green pus. The eardrum has burst, releasing
the pressure and relieving the pain.
The Health Protection Agency recommends a five-day course of amoxicillin or
erythromycin for patients allergic to penicillin

680. A 35yo man has a temp=39C, cough with purulent sputum and right sided chest
pain on
inspiration. He has herpes labialis. What is the single most likely causative organism?
a. Coagulase +ve cocci in sputum
b. Gram -ve diplococci in sputum
c. Gram +ve diplococci in sputum
d. Pneumocystis carinii in sputum
e. Serology for legionella
c. Gram +ve diplococci in sputum
Pneumococcus is the most common , and presents with herpes labialis apart from other symptoms
Herpes labialis is a benign viral infection. If this is associated with a pneumonia, it is usually a pneumococcal
pneumonia. It is a self limiting condition that RESOLVES in 10-15 days.
which is Streptococcus pneumoniae (Pneumococcus) which is coagulase & catalase (-)ve, alpha hemolytic
Gram (+)ve diplococci (Inulin, bile fermentation & optochin sensitivity distinguishes it from Viridans group
which is quite similar and is out of the scope of the question).
The key to answer is again "statistical" one (i.e the most common cause of community-acquired pneumonia
in this age group).
The herpes labialis is often associated with streptococcal pneumonia which is another key feature.
Pneumocystis carinii would cause a noticeable dyspnea in an immunosuppressed individual or known case
of HIV with CD4 count less than 200.
Legionella would cause diarrhea, altered level of consciousness & SIADH (hyponatremia) in an elderly.
Coagulase (+)ve is staphylococcal group in which S.aureus genus is responsible for a pneumonia following
influenza in a debilitated elderly patient which is multilobular, pneumatocele, abscesses and necrotizing with
dissemination, endocarditis and osteomyelitis.

681. A 27yo female was brought to the ED by her friend from a movie theatre. She
complains of
sudden severe pain in the eye followed by vomiting and also was seeing colored halos.
She gives
a past hx of recurrent headaches which used to resolve spontaneously. Exam: fixed,
dilated
ovoid pupil seen. What is the first inv?
a. CT head
b. MRI orbits
c. Blood culture and sensitivity
d. Toxicology screen
e. Applanation tonometry
e. Applanation tonometry
With Halos with eye pain acute angle closure glaucoma should be rule out. So option should be E. Here in
theatre(usually dark) , her pupil have dilated and aggravated the symptom of glaucoma.
movie theatre --->lights off ----> pupil fully dilated----> angle closed--->obstruction of fluid flow from anterior
chamber---> increased pressure in anterior chamber---> AACG
Diagnosis is Acute Angle Closure glaucoma. It is based on the finding of two symptoms of ocular pain,
nausea/vomiting, and a history of intermittent blurring of vision with haloes and at least three signs of the
following: IOP greater than 21 mm Hg, conjunctival injection, corneal epithelial oedema, mid-dilated nonreactive pupil and shallower chamber in the presence of occlusion.

Features
severe pain: may be ocular or headache
decreased visual acuity
symptoms worse with mydriasis (e.g. watching TV in a dark room)
hard, red eye
haloes around lights
semi-dilated non-reacting pupil
corneal oedema results in dull or hazy cornea
systemic upset may be seen, such as nausea and vomiting and even abdominal pain
Management
urgent referral to an ophthalmologist
management options include reducing aqueous secretions with acetazolamide and
inducing pupillary constriction with topical pilocarpine

682. An 82yo male suddenly becomes unconscious and fell down. He recovered
completely within
minutes. What is the best inv you to to dx the case?
a. ECG
b. EEG
c. Blood glucose level
d. CT
e. CXR
a. ECG
in old age arrhythmias are common like stokes adam..so ecg

Sudden loss of consciousness with sudden gain is always cardiac until proved otherwise (except in diabetics
)
The immediate recovery is the trick. If it was hypoglycemia or head injury, the patient would not wake up
quickly. The only condition where the patient gets loss of conscious and recover quickly is in cardiac
conditions
If hypoglycemia recovery without glucose would be difficult. There would be some other features like
palpitation sweating etc.
For exam purposes.. A pt in hypo won't recover until glucose is administered

A classic Stokes-Adams attack is a collapse without warning, associated with loss of


consciousness for a few seconds. Typically, complete (third-degree) heart block is seen on
the ECG during an attack (but other ECG abnormalities such as tachy-brady syndrome have
been reported)

683. A child admitted with progressive muscle weakness and frequent falls. What is the
most
probable dx?
a. Duchennes MD
b. Beckers MD
c. Polymyositis
d. Dermatomyositis
e. Polymyalgia rheumatic
a. Duchennes MD
Duchenne is an x linked recessive disorder where muscle degeneration occurs and present at a very early
age ( before 6 yrs usually) with proximal limb and pelvis muscle weakness..later progressing to the arms etc.
Beckers is same as duchenne but a much milder form so present at a later age.
Duchene ...starts early in life 5-6 years.
B...starts later after 10 years
b is almost same but less severe,c and d are inflammatory but there is rash involved in d..and e is entirely
different related to giant cell arteritis

There is progressive proximal muscular dystrophy with characteristic pseudohypertrophy of


the calves. All patients have symptoms by age 3 years, but diagnosis is often delayed.
Presenting features are:

Motor milestones delayed


Inability to run - waddling gait when attempting to do so
Other gait signs - no spring in the step, cannot hop or jump; toe walking; falls
Gower's sign - 'climbing up legs' using the hands when rising from the floor
Hypertrophy of calf muscles (and possibly other muscles too, including the
deltoid, quadriceps, tongue and masseters)

Non-locomotor presenting symptoms:

Speech delay or global developmental delay


Failure to thrive
Abnormal LFTs (raised AST or ALT)
Anaesthetic complications - eg, myoglobinuria, rhabdomyolysis or malignant
hyperthermia after certain anaesthetics
Fatigue - this is common

The initial investigation is serum CK


The precise diagnosis is best achieved by a combination of:
Genetic analysis - can identify most (but not all) of the DMD mutations.
Muscle biopsy - with assay for dystrophin protein.
Clinical observation of muscle strength and function.

684. A 56yo man presents to the ED with chest pain. The following ECG was taken.
What is the most likely dx?

a. Anterior MI
b. Inferior MI
c. Lateral MI
d. Posterior MI
e. NSTEMI
a. Anterior MI

ECG changes

Coronary artery

Anteroseptal

V1-V4

Left anterior descending

Inferior

II, III, aVF

Right coronary

Anterolateral

V4-6, I, aVL

Left anterior descending or left circumflex

Lateral

I, aVL +/- V5-6

Left circumflex

Posterior

Tall R waves V1-2

Usually left circumflex, also right coronary

685. A schizophrenic says: life is unfair. I like fairs. Fairs have food. It must be good.
What term
describes this pts speech?
a. Neologism
b. Flight of ideas
c. Brocas aphasia
d. Wernickes aphasia
e. Clang association
e. Clang association
rhytmic speech is Clang association
association of words based upon sound rather than concepts
Neologism: is the name for a relatively new or isolated

term, word, or phrase that


may be in the process of entering common use, but that has not yet been
accepted into mainstream language
686. A man comes to the ED with hx of pulsatile swelling in the abdomen, he has hx of
HTN and exam: pulse=120bpm, BP=70/40mmHg. He is restless and in shock. What
emergency management should be done on priority basis?
a. Urgent abdominal CT
b. Urgent abdominal US
c. IV fluids 0.9% NS crystalloids to bring SBP to 90mmHg
d. IV fluids 0.9% NS crystalloids to bring SBP to 120mmHg
e. Dopamine inj
c. IV fluids 0.9% NS crystalloids to bring SBP to 90 mmHg
Aortic aneurism rupture with hypovolemic shock.. first step is to give crystalloids for resus to bring the
systolic bp up to a min of 90
our target sbp should be <100 here bcoz he's hypertensive and the autoregulatory mech in chronic HTN wl
maintain higher bp once sbp is corrected to 90...again if sbp is corrected to higher levels by crystalloids, it
may increase preload and predispose to MI, CVA in AAA rupture
C, then A, then surgery

687. A 5yo boy has cough and swelling at the knee after falling on the ground with
rashes on the
buttocks which are non-blanching. PT=13, APTT=71, Hgb=11, WBC=8, Plt=200. Choose
the most likely dx?

a. NAI
b. Hemophilia
c. HSP
d. Osler weber rendu syndrome
e. Von-Willebrand disease
b. Hemophilia
APTT is prolonged
haemarthrosis is a feature of hemophilia
there is also prolonged in aptt which indicates that there is something wrong with internal pathway
of coagulation process. This is true from hemophilia because the factor 8 deficiency is affecting the
internal pathway
HSP purpura is also non blanchable though mostly present on buttocks and limbs..... but here APTT
IS RAISED...in HSP the cause is vasculitis due IGA deficiency....so diagnosis is Haemophilia.aided
further by joint swelling. In hsp it is arthritis and arthralgia.
Labs of HSP :
ESR raised ( as it is vasculitis )
IgA raised
Proteinuria
Nothing happens to PT , APTT
As they are significantly deranged in this scenario suggesting Haemophilia.

Haemophilia A is due to a deficiency of factor VIII whilst in haemophilia B (Christmas


disease) there is a lack of factor IX
Features
haemarthrosis, haematomas
prolonged bleeding after surgery or trauma
Blood tests
prolonged APTT
bleeding time, thrombin time, prothrombin time normal

688. A 45yo woman presents with discharge from the left nipple. The discharge is
brownish-green
and foul smelling. What is the most likely dx?
a. Duct papilloma
b. Intra-ductal papilloma
c. Duct ectasia
d. Mammary duct fistula
e. Breast abscess
c. Duct ectasia
brownish-green
and foul smelling
689. A 10yo boy presents with generalized swelling. This has been present for 4days
and included
swollen ankles and puffiness of the face. It started a few days after he had a mild cold
with

runny nose. His only PMH was eczema. Urine analysis: hematuria, proteinuria 10g/24h,
creat
60umol/l and albumin=15g/l. What is the single most likely dx?
a. IgA nephropathy
b. HSP
c. Minimal change nephropathy
d. Wilsons disease
e. Cardiac failure
a. IgA nephropathy
the typical presentation of nephritic syndrome as well as we know the patient upper respiratory tract infection
then it lead to immune response
no hematuria in minimal change
HSP presents typically with the following JAKS joints abdomen kidney and skin
arthritis, abdominal pain, haematuria and petechiae
In hsp there is purpura on extensor surfaces, abdominal pain and nephritis
iga nephropathy after 2-3 days of cold infection, post strep glomerulonephritis after 1-12wks of URTI

690. A 28yo man complains of vertigo, nausea and vomiting for more than 30 mins and
tinnitus,
hearing loss in the left ear. What is the tx for this pt?
a. Buccal prochlorperazine
b. Metachlorpromide
c. Cyclazine
d. Cotrimazole
e. Ondansetron
a. Buccal prochlorperazine
1st line-if vomiting
or betahistine
or chlorthalidone (OHCS)
691. A 16yo girl has been unwell for 5days with malaise, headache and dry cough. She
has a few
crackles in her chest. Her CXR shows patchy consolidation in the lower lobes. What is
the single
most likely causative organism?
a. Cold agglutinins
b. Gram ve diplococci in sputum
c. Gram +ve diplococcic in sputum
d. Serology for legionella
e. Sputum staining for mycobacterium TB
a. Cold agglutinins
Mycoplasma
malaise, headache, dry cough n patchy consolidation all go i favour of mycoplasma....legionella shows
bibasal consolidation
patchy consolidation and the features go in favour of mycoplasma and since mycoplasma can casue
autoimmune hemolytic anemia we go for testing cold agglutinins
dry cough is either mycoplasma or legionella (patchy consolidation : mycoplasma +no history of travel ) so
cold agglutination . treatment clarythro or doxy

Mycoplasma pneumoniae is a cause of atypical pneumonia which often affects younger


patients. It is associated with a number of characteristic complications such as erythema
multiforme and cold autoimmune haemolytic anaemia.
Features
the disease typically has a prolonged and gradual onset
flu-like symptoms classically precede a dry cough
bilateral consolidation on x-ray
complications may occur as below
Complications
cold agglutins (IgM) may cause an haemolytic anaemia, thrombocytopenia
erythema multiforme, erythema nodosum
meningoencephalitis, Guillain-Barre syndrome
bullous myringitis: painful vesicles on the tympanic membrane
pericarditis/myocarditis
gastrointestinal: hepatitis, pancreatitis
renal: acute glomerulonephritis
Investigations
diagnosis is generally by Mycoplasma serology
positive cold agglutination test
Management
erythromycin/clarithromycin
tetracyclines such as doxycycline are an alternative

692. A child with increasing jaundice and pale stools. Choose the appropriate test:
a. Endomyseal antibodies
b. Sweat test
c. TFT
d. LFT
e. US
e. US
Biliary atresia. Us abdomen
US abdomen. [This is a picture suggestive of obstructive jaundice. LFT can give clue like much raised
bilirubin, AST and ALT not that high and raised alkaline phosphatase but still USG is diagnostic in case of
obstructive jaundice].
clinically pale stool, dark urine and itching is suggestive of obstructive jaundice, high bilirubin, slight rise in
ast and alt but alp is very high. bilirubin is predominantly conjugated

693. A 73yo woman with RA is unable to extend the fingers of her right hand at the MCP
joint and IP joints following a fall. What is the single most likely tendon to have been
damaged?
a. Extensor carpi radialis
b. Extensor carpi ulnaris
c. Extensor digitorum

d. Extensor indicis
e. Flexor digitorum profundum
c. Extensor digitorum
Extensor digitorum makes an extensor expansion and is inserted in lateral four fingers on the dorsal aspect.
It gives off 3 slips. Main attachment is at dorsal side of the base of proximal phalanx. Middle slip is attached
on the sides middle phalanx and lateral slips at the sides of distal phalanx. Pt is unable to extend both the
MCP and IP joints of all fingers

694. You are called to see a 20yo woman 2h post-LSCS. She has not passed urine
since her operation. She denied any urinary symptoms preoperatively. Exam: appears
unwell, temp=37.5C,
BP=94/73mmHg, pulse=116bpm, sat=97%. Her abdomen is distended with tenderness
in the left
flank and suprapubic region. Bowel sounds are not audible. Choose the most
appropriate post Csection
complication for this lady?
a. UTI
b. Urinary tract injury
c. Pleurisy
d. Acute pyelonephritis
e. Paralytic ileus
b. Urinary tract injury
it is one of the common complication of CS. here patient does not have urination. and all the gut symptoms
is because of release of urine from injured urether to the abdomen

695. A 58yo man has a headache and confusion of 3 days after slipping and hitting his
head in the
garden. What is the most appropriate initial inv?
a. XR skull
b. XR face
c. CT brain
d. MRI brain
e. EEG
c. CT brain

Selection of adults for CT scan

CT scan of the brain within one hour (with a written radiology report within one hour
of the scan being undertaken):
Glasgow Coma Scale (GCS) <13 when first assessed or GCS <15 two
hours after injury
Suspected open or depressed skull fracture
Signs of base of skull fracture*
Post-traumatic seizure
Focal neurological deficit
>1 episode of vomiting
All patients with a coagulopathy or on oral anticoagulants should have a CT brain
scan within eight hours of the injury, provided there are no other identified risk
factors, as listed above. Again, a written radiology report should be available within
one hour of the scan being undertaken.

696. A 4yo boy has a cough and arthritis followed by rash on legs which are nonblanching on glass test. No hx of fever. PT=13, APTT=31, Hgb=12, WBC=6.5, Plt=300.
Whats the most likely dx?
a. Meningitis septicemia
b. Hemophilia
c. HSP
d. ITP
e. TTP
c. HSP
HSP henoch scholene pupura rashes on buttock plus arthritis
and age grp in children
Triad of upper respiratory or git infection with arthritis and non blanching rash.
aptt normal is HSP,APTT RAISED IS HEMOPHILIA
HSP,actually 4 systems involves-GIT,RENAL,SKIN AND JOINTS
triad of purpura, arthritis n abd pain mostly precipitated by a RTI ..purpura involving extensor surface of legs
and buttocks... kidney involvement later mostly presenting as hematuria
differentiate btw itp n hsp?
1.HSP is vasculitis with immune complex deposition in vessels /ITP is due to antibodies against platelet 2.
HSP normal or raised plt count /ITP low plt count 3.HSP specific distribution of rash, joint, kidney , GIT
involved /ITP bleeding through most orifices n other mucocutaneous sites... 4. HSP usually precipitated by
infection /ITP precipitated by infection, trauma etc purpura in ITP generalised

Features

palpable purpuric rash (with localized oedema) over buttocks and extensor surfaces
of arms and legs
abdominal pain
polyarthritis
features of IgA nephropathy may occur e.g. haematuria, renal failure

Treatment

analgesia for arthralgia


treatment of nephropathy is generally supportive. There is inconsistent evidence for
the use of steroids and immunosuppressants

697. A 72yo man presents to the ED with chest pain. The following ECG was taken
What is the most likely dx?

a. Anterior MI
b. Inferior MI
c. Lateral MI
d. Posterior MI

e. NSTEMI
b. Inferior MI

698. A young man has diarrhea, loss of weight and flatulence for 3 days. What is the
most imp tx?
a. Metronidazole
b. Fluconazole
c. Vancomycin
d. Amoxicillin
a. Metronidazole
only suitable option for diarrhea
699. A 6yo child presented with drooling of saliva and severe stridor. He is febrile and
sick looking. XR Neck in extension shows a thumb sign. Choose the single most likely
dx.
a. Croup
b. URTI
c. Diphtheria
d. Acute epiglottitis
d. Acute epiglottitis
most common in children between the ages of 2 and 8

The most common symptoms

Sore throat.
Odynophagia (painful swallowing).
Drooling (inability to swallow secretions).
Fever.
Anterior neck tenderness over the hyoid bone.

Fibre-optic laryngoscopy remains the 'gold standard' for diagnosing epiglottitis


radiograph of the neck may show the 'thumbprint sign'.

700. A mother presents with her 3yo son who has indistinct nasal speech. He snores at
night and has restless sleep. He is tired by day. What is the best management strategy?
a. Arrange hearing test
b. Assess development milestones
c. Refer to ENT surgeon
d. Refer to speech therapist
e. MRI brain
c. Refer to ENT surgeon
The symptoms points to adenoid hypertrophy so for confirmation refer to ENT surgeon

Swollen adenoids may not cause any symptoms or problems. However, symptoms may
develop in some cases, especially if the adenoids become very large. Possible
symptoms include the following:
Breathing through the nose may be noisy or rattly. This may get worse and cause
difficulty breathing through the nose. The child then mainly mouth breathes.
A constantly runny nose.
Snoring at night. In severe cases sleep may be disrupted by the blocked nose
and there is difficulty with breathing.
Swollen adenoids may block the entrance of the Eustachian tube. This is the
tube that goes from the back of the nose to the middle ear. It normally allows air
to get into the middle ear. If this tube is blocked it may contribute to the formation
of glue ear

701-730: Wajiha
731-765: Sonya Hashmi
766-790: Samrah
790-800: Murk Memon
701. A 17yo boy while playing football got a kick and now he is
complaining of severe pain and swelling of the left side of his scrotum. What
inv is the most appropriate to dx?
a. Needle aspiration of scrotum
b. US scrotum
c. MSU
d. Surgical exploration of scrotum
e. Urine test for hematuria
Ans: D
Painful swelling after trauma in teenage boys indicates torsion of testis
which is a surgical emergency.
If the blood supply to the testis is cut off for more than 6 hours can lead to
permanent damage.
More accurately torsion of testis is called torsion of spermatic cord which
causes occlusion of testicular blood vessels
Occurs in one in 4,000 males under 25 years of age
Left side is more common
In patients who present within 24 hours of torsion, the preferred option is
exploration of the scrotum, detorsion and orchidopexy if the testis is viable.

US scrotum with Doppler will help in detection of presence/absence of


intratesticular blood flow for the early identification of testicular torsion
Urinalysis may be helpful in borderline cases, to exclude urine infection and
epididymitis
702. A 50yo man has had hoarseness of voice and drooping eyelid for 2m.
a mass is palpable in the right supraclavicular fossa. He smokes 20
cigarettes/day for the last 30yrs. What is the most likely dx?
a. Carcinoma larynx
b. Carcinoma thyroid
c. Carcinoma right bronchus
d. Mesothelioma
e.
Pancoast tumor
Drooping of eyelid points towards horners syndrome. Pancoast tumor can
also lead to hoarsness of voice by compressing the recurrent laryngeal nerve.
There could be weakness of small muscles of the hand by involvement of the
C8-T1 of the brachial plexus.
703. An 84yo man got surgical pain which is well controlled by oral
morphine 60mg BD. However, now this pt is unable to swallow. What is the
most appropriate next step?
a. Morphine 60mg state
b. Morphine 60mg TDS
c. Oxycodone 10mg OD
d. Morphine 60mg IV
e.
Fentanyl patches
Fentanyl patches are used to relieve severe pain in people who are expected to need pain
medication around the clock for a long time and who cannot be treated with other
medications. Fentanyl is in a class of medications called opiate (narcotic) analgesics. It
works by changing the way the brain and nervous system respond to pain

Since the patient is pain free and we know dose of morphine to be given we
can switch to fentanyl patches.
704. A 19yo man has exercise induced asthma. This has prv been
controlled using a salbutamol inhaler as req, but he now gets attacks with
exercise. What is the single most appropriate tx?
a. Regular salbutamol
b. Regular salbutamol and budesonide

c.
Sodium cromoglycate
d. Oral steroid
e. Inhaled steroid
Answer here should be B or E. Inhaled salbutamol + inhaled ICS are
required.

Exercise-induced asthma
For most, exercise-induced asthma indicates poorly controlled asthma and will require
regular inhaled steroid treatment beyond the anticipatory use of a bronchodilator when
preparing for sport. Where exercise poses a particular problem and patients are already on
inhaled corticosteroids, consider the addition of long-acting beta agonists, leukotriene
inhibitors, chromones, oral beta2 agonists or theophyllines.

705. A 3yo boy has a sudden onset of fever, vomiting and bilateral face
swelling. Few days earlier the GP saw him for bilateral parotid pain and
gave analgesics. What is the most appropriate next step?
a. Analgesic
b. Antibiotic
c. Biopsy
d. Immediate surgery
e.
Reassurance
This is a case of mumps. It can be unilateral or bilateral. Can cause
distortion of the face due to swelling. Pain is usually around the angle of the
jaw. Fever can be as high as 39.5 and it can cause dryness of the mouth due
to blockage of the salivary ducts. No specific treatment is required. Just keep
the patient hydrated and treat symptomatically.
706. A 75yo man with adenocarcinoma of the prostate which has spread
outside the capsule of the gland has ARF. What is the most appropriate next
inv?
a. MRI spine
b. Radionuclide bone scan
c. Trans rectal US
d. US pelvis
e.
US KUB
This is to look for renal status and metastatic invasion.

707. A 57yo male presents with sudden onset severe abdominal pain and
rigidity against a 4d background of LIF pain and pyrexia. He has no
PM/SHx of note and isnt on any meds. What is the most likely dx?
a. Intussusception
b. Ischemic colon
c. Sigmoid volvulus
d.
Perforated diverticulum
e. Perforated meckels diverticulum
It is a case of peritonitis.
Pain in LIF points to one of the following:
Gastroenteritis (will have more of a generalized abdominal pain)
Constipation (Will not be this serious unless there is obstruction which will
be apyrexial)
Sigmoid volvulus (again it will cause intestinal obstruction)
Hernia (may lead to obstruction or strangulation)
Diverticulitis (90% have pain LIF)
Appendicitis (mostly RIF veryyy rarely pain is in LIF.)
Meckels diverticulum mimics appendicitis and in most of the cases it is
asymptomatic.
This is perforated diverticulitis as there is evidence of peritonitis because of
severe abdominal pain and rigidity and pyrexia.
708. A 46yo woman has weight gain, sensitivity to cold, pulse=50bpm,
heart is enlarged with murmur. What is the single most likely dx?
a.
Hypothyroidism
b. Hyperthyroidism
c. Cushings syndrome
d. Addisons disease
e. Pheochromocytoma
Typical presentation of hypothyroidism.
Hyperthyroidisim has opposite symptoms.
Cushings has moon face, hypertension, weight gain csn occur, abdominal
striae.
Addisons has pigmentation, hyponatremia, hypotension, hyperkalemia
Pheochromocytoma presents with episodic headaches and HTN.

709. An alcoholic who has completely given up drinking hears voices.


What is the most appropriate tx?
a.
Olanzapine
b. Diazepam
c. Acamprosate
d. Disulfiram
e. Haloperidol
This is psychosis. The patient has auditory hallucinations. Diazepam is used
in acute alcohol withdrawal. Acamprosate is to reduce cravings and maintain
remission. Disulfiram is a deterrent. Olanzapine is preferred over haloperidol
as it is an atypical antipsychotic which has fewer side effects.
710. A 6yo boy has completed an induction course of chemo for ALL. He
has an enlarged left scrotum. What is the most appropriate next step?
a. Herniotomy
b.
CT abdomen
c. Biopsy
d. Immediate surgery
e. Reassurance
711. A 32yo miner is rescued after being trapped under a fallwn rock for
4h. After applying a bladder catether, 15-20ml of reddish brown urine was
obtained. HR=120bpm, SBP=100mmHg. What would be the next
appropriate step?
a. Dopamine IV
b.
Fluid challenge
c. Furosemide IV
d. 20% Mannitol IV
e. Antibiotics
The patient has developed rhabdomyolysis (trapped under a fallen rock,
reddish brown urine due to myoglobin in urine) in which there is a great
danger of acute kidney failure as is the case in this patient due to decreased
urine output so we have to give this patient IV fluids to avoid further renal
compromise.
712. A 60yo man has had spontaneous painful swelling of his right knee for
3days. 5days prv he had an inguinal hernia repaired as a day case. He takes
bendroflumethiazide 2.5mg daily. He is apyrexial. What is the single most
appropriate diagnostic inv?

a.
b.
c.
d.
e.

Blood culture
CRP
D-dimer
XR knee
Serum uric acid

There is a suspicion of gout for which serum uric acid needs to be checked.
The patient takes a thiazide diuretic which predisposes to gout.
713. A 27yo woman with anxiety and weight loss has tachycardia, tremor
and mild proptosis. What single mechanism accounts for her weight loss?
a. Deficiency in thyroid hormone
b. Increased level of calcitonin
c.
Increased metabolic rate
d. Insulin resistance
e. Reduced caloric intake
The patient has thyrotoxicosis (increased T3 & T4) which are catabolic
enzyme increasing the metabolic rate leading to the weight loss. The
symptoms given point towards Graves disease.
714. A man with carcinoma and multiple metastasis presents with
intractable nausea and vomiting. He has become drowsy and confused. What
is the most appropriate management?
a. Dexamethasone IM
b. Dexamethasone PO
c.
Ondansetron IM
d. Ondansetron PO
e. Morphine oral
Ondansetron is a 5HT-3 blocker mostly used for chemotherapy induced
vomiting. Dexa and morphine have no role in this case. PO route cannot be
used as the patient is vomiting.
715. A 19yo man presents with weight loss, increasing thirst and increasing
frequency of going to the washroom. His father, grandfather and 2 sisters
have been dx with DM. What is the most likely type of DM this pt suffers
from?
a. IDDM
b. NIDDM
c. LADA

d.
MODY
e. DKA
Diabetes at such a young age with a positive family history of early diabetes
always think of Maturity onset diabetes of the young. It has autosomal
dominant mode of inheritance.
716. A 42yo woman with a PMH of severe headache treated in the ED
presents with signs and symptoms of renal failure. She has been seen by her
GP for HTN and abdominal pain with OP inv pending. Which inv is most
likely to lead to a dx?
a.
US KUB
b. CT brain
c. IVU
d. Renal artery Doppler
e. Renal biopsy
Loin pain, gross hematuria, nocturia, HTN all can be the features of
polycystic kidney disease. Headaches could be because of HTN. or it could
also be renal artery stenosis for which US has to be done.
717. In perforation of a post gastric ulcer, where will the fluid accumulate
in the peritoneal cavity?
a. Left paracolic gutter
b. Pelvic cavity
c.
Lesser sac
d. Under the diaphragm
e. Right paracolic gutter
Behind the stomach is the lesser sac. All other are irrelevant here.
718. A 62yo male is brought to the ED by his daughter because of his
persistent lying. He is a known alcoholic and has been admitted recently
with delirium tremens. On questioning, he denies any problem with memory.
He knows his name and address and states that was at the betting shop this
morning, but his daughter interjects calling him a liar explaining that he was
at her home. What is the most likely dx?
a. Ganser syndrome
b. Cotard syndrome
c. Wernickes encephalopathy
d.
Korsakoff psychosis

e.

Alcohol withdrawal

Answer should be D.

Symptoms of Korsakoff syndrome

Vision changes:
o Double vision
o Eye movement abnormalities
o Eyelid drooping
Loss of muscle co-ordination:
o Unsteady, unco-ordinated walking
Loss of memory, which can be profound.
Inability to form new memories.
Hallucinations.

719. A 70yo man presented with muscle weakness and inability to climb
the stairs. Inv: CPK raised, ESR 15. What is the most likely dx?
a.
Polymyositis
b. Polymyalgia rheumatic
c. Reactive arthritis
d. RS
e. Duchennes MD
Proximal muscle weakness raised CPK point towards the diagnosis. Anti Jo
antibodies are raised in polymyositis. Polymyalgia rheumatica presents with
shoulder girlde ache, morning stiffness. Duchene will not present at the age
of 70 yrs it presents in childhood.
720. A 65yo known alcoholic is brought into the hospital with confusion,
aggressiveness and ophthalmoplegia. He is treated with diazepoxide. What
other drug would you like to prescribe?
a. Antibiotics
b. Glucose
c. IV fluids
d. Disulfiram
e.
Vit B complex
An alcoholic who presents with the triad of ophthalmoplegia, ataxia and
confusion is wernickes syndrome. It occurs due to vitamin B1 deficieny
(thiamine def).

721. A pt suffering from schizophrenia laughs while talking about his


fathers death. Which term best describes his condition?
a. Depression
b. Flat affect
c. Emotional liability
d.
Incongruent affect
e. Clang association
Flat affect, which is also called blunted affect, is one of the negative symptoms
of schizophrenia. A person with negative symptoms lacks a normal range of
feelings and behaviors
Emotional liability also known as pseudobulbar affect is characterized by
involuntary crying or uncontrollable episodes of crying and/or laughing, or other
emotional displays.
Symptoms are said to be mood-congruent if they are consistent with a patient's mood or
mental disorder. Conversely, they are said to be mood-incongruent if they are inconsistent
with their primary mood.

clanging refers to a mode of speech characterized by association of words


based upon sound rather than concepts

722. A 72yo man has been on warfarin for 2yrs because of past TIAs and
strokes. What is the most imp complication the pt should be careful with?
a. Headache
b.
Osteoporosis
c. Ear infection
d. Limb ischemia
e. Diarrhea
The answer should either be headache or diarrhea. diarrhea is one of the
direct adverse effects of warfarin. Since the patient is on warfarin there is a
chance of intracranial bleed which can present as a headache so it could be
an indirect effect of warfarin use. Warfarin has nothing to do with
osteoporosis.
723. A 24yo woman is afraid to leave her house as whenever she goes out,
she tends to have SOB and sweating. She has stopped going out except with
her husband. What is the most likely dx?
a. Social phobia
b. Claustrophobia
c. Depression
d. Panic disorder

e.

Agoraphobia

Social phobia is when the person is afraid to go to places where he/she may
be judged, commented on or criticized by others like parties.
Claustrophobia is fear of a tight space, small room etc.
Agoraphobia is fear of large open spaces.
724. A pt on HTN drugs develops hyperkalemia. Which anti-HTN is likely
to cause it?
a.
Ramipril
b. Lorsartan
c. Thiazide
d. Nifedipine
e. Furosemide
Just an adverse effect of ace inhibitors. Diuretics cause hypokalemia.
725. A young man develops itching worse at night and following bathing.
Exam: greysish white linear rash can be seen on the wrist and periumbilical
area. What is the dx?
a.
Scabies
b. Polycythemia
c. Urticarial
d. Atopic eczema
e. Lichen planus
Widespread itching, worse at night and when the person is warm. Skin signs
can vary.
726. A 40yo lady who has been a smoker since she was a teenager has the
following blood result: Hgb=19. What hormone should you check?
a. Aldosterone
b. Cortisol
c.
Erythropoietin
d. T4
e. TSH
The person has polycythemia which means increased RBC mass.
Erythropoietin is responsible for the production of RBCs. No other
mentioned hormone here has anything to do with RBC production.

727. A 25yo man presents with hoarseness of voice. He has swollen vocal
cords. His BMI=32 and he smokes 20-25 cigarettes/day. What would you
advise him?
a.
Stop smoking
b. Lose weight
There are several reasons for hoarsness of voice on of which is CA larynx.
Smoking is a big risk factor for it.
728. A 64yo male was admitted to the medical ward with complaint of
diarrhea, abdominal pain and weight loss for few months. Exam: clubbing,
perianal skin tags and abdominal tenderness. Colonscopy reveals transmural
granulomatous inflammation involving ileocecal junction. He was dx with
what?
a.
CD
b. UC
c. Bowel cancer
d. Gastric cancer
e. IBD
Peri anal tag, transmural granulomatous disease disease take the diagnosis
towards crohns disease.
729. A pt presents with hemoptysis 7d after tonsillectomy. What is the next
step?
a. Packing
b. Oral antibiotics and discharge
c.
Admit and IV antibiotics
d. Return to OT and explore
e. Ice cream and cold fluids
This is a case of secondary hemorrhage (mostly between 5-14 days) the most
common cause of which is infection.
730. A 55yo man presents with HTN. He complains of headache and visual
disturbances. He also reports itching after a hot bath and burning sensation
in finger and toes. His face is flushed red. PE: mild splenomegaly. Inv:
Hgb=20g/dl, WBC=20, plt=500, EPO normal. What is the likely dx?
a. Myelofibrosis
b.
Polycythemia rubra vera
c. Essential thrombocythemia

d. CML
e. CLL
Increased haemoglobin, flushing, itching, burning sensation, visual
disturbance (amaurosis fugax) all are symptoms of polycythemia rubra vera.
731. An old man having T2DM with increased skin tanning, heart failure
and having high ferritin (hemochromatosis) level is refusing tx. Where is the
first site of cancer?
a. Testes
b. Adrenal
c.
Liver
d. Pancreas
It is a case of hemochromatosis so the primary site is liver. It can lead to
HCC.
732. A 60yo DM lady presents with severe peri-anal pain and swelling.
Whats the cause?
a. Anal carcinoma
b. Anal fissure
c. Hemorrhoid
d.
Anal abscess
Q1. What is the key?
Q2. What is the supporting evidence?
A1. The answer is anal abscess.
A2. The lady is 60 years old and presents with perianal pain and swelling.
Firstly it is important to think of an infection in case of diabetics.
An anorectal abscess is a collection of pus in the anal or rectal region. It may
be caused by infection of anal fissure/STD/blockage of anal glands.
The following anatomical types have been identified.
Perianal abscess: the most common (60%) - caused by direct
extension of sepsis in the intersphincteric plane caudal to the perianal
skin.
Ischiorectal abscess: (20%) - results from extension of sepsis through
the external sphincter into the ischiorectal space.
Intersphincteric abscess: (5%) - depending on the effort made to find
them, sepsis confined to the intersphincteric space.

Supralevator abscess: (4%) - produces horseshoe abscess track.


Postanal abscess: posteriorly based below the level of the anococcygeal ligament.

High Risk group includes diabetics, immunocompromised, anal sex, patients


of inflammatory bowel disease.
Age group 20-60 with mean age 40, M:F=2:1
SYMPTOMS: pain, lump, nodule, hardened area in perianal region,
tenderness at edge of anus, fever, constipation or pain associated with bowel
movements.
Pain is usually constant, throbbing and worse on sitting down.
INVESTIGATIONS: Digital rectal examination is sometimes enough to
diagnose.
PROCTOSIGMOIDOSCOPY: to rule out associated diseases.
MRI: for assessment of location of fistular tracts and their openings, location
of deep abscesses, peri-rectal space, any damage to anal sphincter.
TRANSPERINEAL USG: may help to locate deep abscesses or an
associated pathology.
SCREENING for STD, inflammatory bowel disease, diverticular disease or
lower GI malignancy.
TREATMENT:
prompt surgical drainage
pain relief
Antibiotics only if diabetic or immunosuppression.
COMPLICATIONS: systemic infection, recurrence, scarring, fistula in ano
in 30% of patients, reduces with early surgery.
To differentiate from anal carcinoma, it may also present with anal swelling
and pain, but there is usually associated bleeding and faecal incontinence.
Anal fistula usually presents with excruciating pain while defecating like
shreds of glass, and may be associated with rectal bleeding usually bright red
and not mixed with stools.
For haemorrhoids, bright red, painless rectal bleeding with defecation is
usually the most common symptom.

733. A woman is sad, fatigues and she is eating more and also has sleeping
disturbance and hears the voice of her husband who died 3yrs ago. What is
the dx?
a. OCD

b.
Psychotic depression
c. Grieving
d. Severe depression
Q1. What is the key?
Q2. Why is it so.
Q1. The key is Psychotic depression.
Q2. Why..
Depression refers to both negative affect (low mood) and/or absence of
positive affect (loss of interest and pleasure in most activities) and is usually
accompanied by an assortment of emotional, cognitive, physical and
behavioural symptoms.It is currently ranked the third most prevalent
moderate and severe disabling condition globally by the World Health
Organization (WHO).

Classification:
(NICE) guidance uses the Diagnostic and Statistical Manual Fourth Edition
(DSM-IV) classification.
To diagnose major depression, this requires at least one of the core symptoms:
Persistent sadness or low mood nearly every day.
Loss of interests or pleasure in most activities.

Plus some of the following symptoms:


Fatigue or loss of energy.
Worthlessness, excessive or inappropriate guilt.
Recurrent thoughts of death, suicidal thoughts, or actual suicide
attempts.
Diminished ability to think/concentrate or increased indecision.
Psychomotor agitation or retardation.
Insomnia/hypersomnia.
Changes in appetite and/or weight loss.
Symptoms should have been present persistently for at least two weeks and
must have caused clinically significant distress and impairment.
Rule out other organic/physical factor like substance abuse, or chronic
illness.

DSM-5 was published in 2013. It proposes the following changes to the


classification of depressive disorders:
Persistent depressive disorder - this term is proposed to
encompass both chronic major depressive disorder and
dysthymia.
Removal of the major depression bereavement exclusion - the
diagnosis of major depression was excluded in people who had
recently been bereaved. This has been removed, leaving more
leeway for clinical judgement.
A new category of mixed anxiety/depressive disorder.
The NICE guidelines encourage a case-finding approach with at-risk groups
(individuals with a past history of depression or a chronic health problem
with associated functional impairment) using a two question approach:
During the past month, have you:
o Felt low, depressed or hopeless?
o Had little interest or pleasure in doing things?
ASSESSMENT:
The Patient Health Questionnaire (PHQ-9)
The Hospital Anxiety and Depression (HAD) Scale
Beck's Depression Inventory
Full history and examination +mental state examination
enquire about delusions/hallucinations and suicidal ideas.
Rule out organic causes of depression like hypothyroidism/drug side
effects
Medications that may cause depressed mood include:
Centrally acting antihypertensives (eg, methyldopa).
Lipid-soluble beta-blockers (eg, propranolol).
Benzodiazepines or other central nervous system depressants.
Progesterone contraceptives, especially medroxyprogesterone
injection.

Differential diagnosis

Bipolar disorder.
Schizophrenia (depression may co-exist).
Dementia may occasionally present as depression and vice versa.
Seasonal affective disorder.

Dysthymia (recently classified by DSM-5 as a subtype of persistent


depressive disorder) is a chronic depressive state of more than two
years in duration.
Other psychiatric conditions may co-exist with depression (eg,
generalised anxiety disorder, panic disorder, obsessive-compulsive
disorder, personality disorders).
Bereavement: depressive symptoms begin within 2-3 weeks of a death
(uncomplicated bereavement and major depression share many
symptoms but active suicidal thoughts, psychotic symptoms and
profound guilt are rare with uncomplicated bereavement).

INVESTIGATIONS:
Rule out organic causes
Blood tests may include blood glucose, U&Es, LFTs, TFTs, calcium levels,
FBC and inflammatory markers, HIV OR SYPHILIS SEROLOGY, drug
screening.
MANAGEMENT:
Consider watchful waiting, assessing again normally within two
weeks.
Guided self-help based on cognitive behavioural therapy (CBT)
principles
Antidepressants are not recommended for the initial treatment of mild
depression, because the risk:benefit ratio is poor. However they are
recommended in moderate to severe depression.
Selective serotonin reuptake inhibitors (SSRIs) are used as first-line
antidepressants in routine care because they are as effective as
tricyclic antidepressants and less likely to be discontinued because of
side-effects; also because they are less toxic in overdose.
Citalopram, fluoxetine, paroxetine, or sertraline have equal efficacy
however a recent meta-analysis suggested that escitalopram had the
highest probability of remission and is the most effective and costeffective pharmacological treatment in a primary care setting
Fluoxetine is the antidepressant of choice for children and young
people.
Where a patient has concurrent physical health problems, citalopram
or sertraline may be preferred, as they have less risk of significant
drug interactions

Treatments such as dosulepin, phenelzine, combined antidepressants


and lithium augmentation of antidepressants should be initiated only
by specialist mental healthcare professionals.
Electroconvulsive therapy (ECT) may be used to gain fast and shortterm improvement of severe symptoms after all other treatment
options have failed, or when the situation is thought to be lifethreatening.

What is Psychosis;
Psychosis is a severe mental disorder in which there is extreme impairment
of ability to think clearly, respond with appropriate emotion, communicate
effectively, understand reality and behave appropriately.
Psychosis occurs in a number of serious mental illnesses and not just
schizophrenia, eg depression, bipolar disorder (manic-depressive illness),
puerperal psychosis and sometimes with drug and alcohol abuse
Disabling symptoms include delusions and hallucinations:
A delusion is a false, fixed, strange, or irrational belief that is firmly
held. The belief is not normally accepted by other members of the
same culture or group. There are delusions of paranoia (plots against
them), delusions of grandeur (exaggerated ideas of importance or
identity) and somatic delusions (false belief in having a terminal
illness).
An hallucination is sensory perception (seeing, hearing, feeling,
smelling) without an appropriate stimulus, like hearing voices when
no one is talking. Not all hallucination suggests psychosis.
So to conclude we can see that the patient is the question was suffering from
complicated bereavement associated with psychotic symptoms.

734. A 40yo teetotaler woman is recovering from a hysterectomy 2days ago.


At night she becomes agitated and complains of seeing animals and children
walking around the ward. What is the most likely dx?
a. Delirium tremens
b.
Toxic confusional state
c. Hysteria

d. Mania
e. Drug induced personality disorder
The key says toxic confusional state.
Confusion and agitated behaviour are common complications after
operation,
especially in elderly patients. It is customary to prescribe sedation, instead of
regarding confusion as a symptom whose causes must be , diagnosed, and
preferably anticipated and prevented.
This is the most common psychosis seen in hospital with 20% of elderly
patients having some form of delirium. Also know as toxic confusional state.
A simple scheme of management is outlined, with special emphasis on
common conditions.Often it is a symptom of an acute underlying disorder
which needs early treatment. It may in turn cause its own complications,
such as injury, disturbance of healing tissues or of equipment, or pneumonia
due to mistaken over-use of sedation in treatment. The turmoil caused by a
confused patient is distressing to other patients, and creates hard work for
nurses and junior medical staff.
CAUSES OF POST OPERATIVE CONFUSION:
1. Common:
Cerebrovascular disease.
Drugs, delirium tremens.
Chest infection or atelectasis.
Renal infection.
Abdominal sepsis, superficial or deep.
Over-full bladder or rectum.
2. Less common:
Anaemia (especially vitamin B12 deficiency), unrecognized blood
loss, and other forms of anoxia.
Hypothyroidism.
Hepatic or renal failure.
Subdural haematoma.
3. Psychological factors:
Sensory distortion by bombardment or deprivation.
Sleep disturbance and loss.
Depression, anxiety, schizophrenia.
4. Aggravation by noxious stimuli:

Fear, discomfort, pain, thirst, hunger.


5. Rare causes:
Hypoglycaemia, fat embolism, hypernatraemia.
Drugs BZDs, opiates, tricyclics, alcohol, L-Dopa
Signs
Disordered thinking incoherent, slow, irrational, jumbled, rambling
often worse at night
Euphoric, fearful, angry or depressed
Language impaired reduced speech, repetetive or disruptive
Illusions/delusion/hallucinations think tactile or visual (auditory
more suggests psychosis)
Reversal of sleep patterns drowsy by day > awake & vigilant at
night
Inattention
Unaware/disorientated doesnt know time of day, name or address
Memory deficit may be amnesic
TREATMENT:
Management of the confused postoperative patient may be quite easy or very
difficult, depending on the underlying causes of the condition. Most often
there is a single predominating factor which is temporary, such as retention
of urine, or pneumonia with congestive cardiac failure, to be treated along
accepted lines.
Sedative drugs are best withdrawn unless there is uncontrollable agitation,
when the most useful are diazepam (Valium), paraldehyde, or
chlorpromazine.
identify and treat underlying cause
careful nursing and rehydration
nurse in quiet, single room, with window
review current drug therapy
if disruptive can use haloperidol
can persist for 1-2 week
As the clue given is the patient being post-op , so we have to consider acute
post op toxic confusional state as the most likely diagnosed compared to
others and since the patient is non alcoholic there is no chance of delirium
tremens.

735. A woman with a hx of drug abuse and increased alcohol intake, now
comes for help and she is concerned about her problem. What is the most
appropriate management option?
a. Voluntary admission
b.
Psychiatry team
c. Mental health team
d. Psychiatry voluntary admission
Key: The important point in this scenario is that the patient is willing for
treatment. NICE has published guidelines for drug misuse and treatment and
for every patient these protocols need to be followed. The main points from
the guidelines regarding assessment and decision making as as follows. The
only problem with this key was that psychiatry team and mental health team
seem to be the same thing and usually patients are sent to organizations such
as alcoholics anonymous etc who have specially designed plans for
management so they are neither psychiatric teams, nor mental health teams.
Anyway have a look at the basic framework of the management plan and
then decide about answer according the patients scenario.
Treatment provision: key points
The needs of drug misusers should be assessed in terms of their
health, social functioning and criminal involvement.
All GPs have a duty to provide basic medical services to people who
are dependent on opioids and they should screen patients for drug
misuse.
A good initial assessment is essential:
o This may involve a multidisciplinary team.
o Good assessment is vital to the continuing care of the patient.
o It can enable the patient to become engaged in treatment and
may begin a process of change even before a full assessment is
completed.
Confirmation of drug taking should be gained (through history,
examination and drug testing).
Any risks to their children should be assessed and child protection
services involved as appropriate.

Emergency or acute problems should be treated (for example, access


to clean needles and equipment).
Testing for blood-borne infections should be arranged as appropriate.
A physical and psychological health assessment should be carried out.
Any ongoing criminal involvement or offences should be determined.
The drug misuser's expectations and desire to change should be
assessed.
The degree of dependence and need for substitute medication should
be assessed.
An individual care and treatment plan should be drawn up and
reviewed regularly.
A named person should manage and deliver an individual's care (eg
the GP or drugs worker). They may be known as the 'keyworker'.
If detoxification and/or substitute prescribing are requested, after an
initial assessment, GPs can refer to local specialist community drug
services and there are usually locally agreed shared care guidelines. A
care plan between the drug misuser and the service provider can then
be drawn up.
A GP may have a special clinical interest in the management of
substance misuse in primary care and may be able to take more
responsibility in the treatment of patients, particularly in complex
cases.
Drug testing can help to monitor compliance and treatment outcome.

Psychosocial components of treatment: key points


A keyworker with a good therapeutic alliance is best placed to provide
psychosocial assessment and support.
Counselling, cognitive behavioural therapy and supportive help (for
example with housing and benefits) are examples of psychosocial
treatment strategies.
If the keyworker does not have the full range of competencies to
deliver psychosocial interventions, other professionals may be
involved.
Common social problems among drug misusers include housing,
employment and financial difficulties. Criminal convictions are also
common.
Mental health problems such as depression and anxiety can co-exist
with drug misuse.

In cannabis, hallucinogen and stimulant abuse (including cocaine),


psychosocial interventions are the main treatment.
For opioid, alcohol and polydrug misuse, they can be used in
conjunction with drug treatment.
Patients should also be advised about support groups such as
Alcoholics Anonymous and Narcotics Anonymous.
Some patients find that self-help approaches work for them and these
should be discussed.
In other countries, couple- and family-based interventions and
contingency management have been found to be helpful. These
approaches are not commonly used in the UK at present but they
should be considered (provided the appropriate training has been
received).
The NICE guidelines support a number of formal psychosocial
treatments and detail the evidence that supports them. These include:
brief motivational interventions, self-help groups and contingency
management (eg, incentives contingent on each presentation of a
negative drug test).

736. A 28yo woman who is 32 wks pregnant in her 3 pregnancy is


diagnosed as a case of placental abruption. After all the effective measures,
she is still bleeding. What is the underlying pathology?
a.
Clotting factor problem
b. Clausers syndrome
c. Platelet problem
d. Succiturate lobe
e. Villamentous insertion of placenta
rd

Ans: Placental abruption has been defined as the complete or partial


separation of a normally implanted placenta from its uterine site before the
delivery of the fetus. This definition differentiates this process from placenta
previa, in which the placenta is implanted in an abnormal anatomical
position covering the internal cervical os.
Grade I: Mild. This group accounts for 40% of all cases; it includes
antepartum hemorrhage of uncertain cause. There may be slight vaginal
bleeding and uterine irritability. Maternal blood pressure usually is normal,
and there is no maternal coagulopathy or fetal distress. The diagnosis of this

class of abruptio placentae is confirmed on postpartum detection of a small


retroplacental clot.
Grade II: Intermediate. This accounts for 45% of all cases. This diagnosis is
based on the classic features of abruptio placentae with uterine
hypertonicity, but the fetus still is alive. There is a greater amount of vaginal
bleeding (mild to moderate), hypofibrinogenemia, and fetal distress. Blood
pressure is maintained, but the pulse rate may be elevated and postural blood
volume deficits may be present.
Grade III: Severe. This accounts for 15% of all cases. In such cases, the
fetus is always dead. Usually, heavy vaginal bleeding occurs, although in
some cases this may be concealed. Maternal hypotension,
hypofibrinogenemia, and thrombocytopenia are present, along with a tetanic
uterus. This type is further subdivided into grade IIIA, in which overt
coagulopathy is not present, and grade IIIB, when an overt coagulopathy
results.
This grading system may be helpful in establishing a therapeutic plan.
2

Clinicians and investigators have observed the presence of a bleeding


diathesis accompanying some cases of premature separation of the placenta.
In addition to its association with severe placental abruption, acquired
hypofibrinogenemia also has been reported in cases of amniotic fluid
embolism, long-standing fetal death in utero, septic abortion, eclampsia, and
delayed postpartum hemorrhage.
The phenomenon of consumption coagulopathy leads to patient injury
because of two problems: the bleeding diathesis caused by diminished
coagulation factors and elevated FDP and the localized tissue necrosis in
target organs because of fibrin deposition in small blood vessels. As a
protective mechanism, the fibrinolytic system is activated secondarily, and
dissolution of the fibrin clots by plasmin may protect the local tissue from
anoxia. This delicate balance between fibrin deposition and degradation is
present in the body at these times. If this secondary fibrinolysis and
fibrinogenolysis become excessive, the decreased coagulation factors
resulting from consumption are augmented by destruction of the same
factors by plasmin, and the hemorrhagic situation is made worse. The
degradation products of fibrin and fibrinogen also interfere with fibrinogen
conversion to fibrin. Fibrinolytic problems in abruptio placentae are mostly
secondary, with progressive activation of the system occurring only after an
initial phase of intravascular coagulation.

The complications of hypofibrinogenemia have been detected in one third to


one half of patients having signs and symptoms of severe placental
separation. Most patients with abruptio placentae have the milder form and
exhibit no clinical difficulties with the clotting mechanism.
So as it says in the mcq that all other measures to stop the bleeding have
failed then we should start thinking on the lines of clotting factor problems.
Even in the most severe cases, clinically evident coagulopathy usually
resolves by 12 hours after delivery.

737. An old woman having decreased vision cant see properly at night. She
has changed her glasses quite a few times but with no effect. She has normal
pupils and cornea. What is the most likely dx?
a.
Cataract
b. Glaucoma
c. Retinal detachment
d. Iritis
e. GCA
Vision loss among the elderly is a major health care problem.
Approximately one person in three has some form of vision-reducing eye
disease by the age of 65. The most common causes of vision loss among the
elderly are age-related macular degeneration, glaucoma, cataract and
diabetic retinopathy. Age-related macular degeneration is characterized by
the loss of central vision. Primary open-angle glaucoma results in optic
nerve damage and visual field loss. Because this condition may initially be
asymptomatic, regular screening examinations are recommended for elderly
patients. Cataract is a common cause of vision impairment among the
elderly, but surgery is often effective in restoring vision. Diabetic
retinopathy may be observed in the elderly at the time of diagnosis or during
the first few years of diabetes. Patients should undergo eye examinations
with dilation when diabetes is diagnosed and annually thereafter.
Many causes of gradual visual loss can be diagnosed on history and
examination alone with only the most basic additional investigations. For
any patient with a gradual loss in visual acuity ,the following protocol
should be followed to reach to an initial diagnosis.

Assessment
History

A routine history is mandatory and will often guide you to a possible cause.
Specifically ask about:
The nature of the problem:
o Unilateral versus bilateral.
o Blurred vision: whether this is the whole field, close, distance
or both.
o Restricted visual field: often noted following difficulties in
driving, knocking into things at the periphery of vision
o Distorted rather than blurred vision (eg, dent in printed words,
door/window frames, objects appearing smaller or larger). If so,
check with an Amsler grid (see under 'Further reading &
references', below). Distortion of straight lines indicates
serious macular pathology and needs urgent referral.
o Bits of visual field missing altogether: central versus peripheral;
establish what the remainder of the vision is like.

Progression: ask whether there has been a slow and steady decline;
whether there have been step-wise drops in visual acuity, or whether
the problem has been intermittent. If intermittent, think of transient
ischaemic attacks (TIAs) or impending acute angle-closure glaucoma.
Ask whether there have been any associated factors. For example,
pain (very important diagnostically, see 'Painful loss', below), redness,
or visual phenomena - eg, haloes, flashes of light, new floaters. These
symptoms usually merit an urgent referral.
Ask whether there are any precipitating factors. Specifically ask about
changes over the course of a day and whether vision is better in the
day or at night.

Examination

Important points to note on examination are:


The visual acuity of both eyes. Note whether this improves using a
pinhole.
The red reflex: a media opacity (appears black against the red reflex)
suggests a corneal, lens or vitreous problem. To localise the site of the
opacity with respects to the pupil (lens):
o Slowly shift the direction of your ophthalmoscope light.

o
o
o
o
o
o
o
o

Look at the direction in which the opacity appears to move in


relationship to the pupillary (central) axis.
If there is no 'movement' of the opacity, it lies within the pupil
(lens).
If the opacity 'moves' in the same direction, it is anterior to the
lens (cornea).
If the opacity 'moves' in the opposite direction, it is posterior to
the pupil (posterior lens or vitreous).
If the media is clear, it is more likely to be a retinal or optic
nerve disorder.
If there is a normal red reflex, take a good look at the fundus.
Do a functional testing of visual field, pupils, optic nerve and
macula.
Use an Amsler grid to look for distortion of straight lines - if
present, this indicates serious macular pathology and needs
urgent referral

Interpreting the findings - some clinical patterns

Loss of vision in one eye - implies the problem is in the eye itself or in
the optic nerve before it reaches the optic chiasm.
Loss of vision in both temporal fields (bitemporal hemianopia) occurs from lesions compressing the optic chiasm (eg, pituitary
tumour or craniopharyngioma).
Loss of left or right visual field (homonymous hemianopia) - implies
the lesion is somewhere between the optic chiasm and the occiput.
Chronic glaucoma tends to cause tunnel vision (loss of peripheral
visual field).
Macular degeneration causes loss of central vision (central scotoma)
and may cause distortion of straight lines.
Cataracts - the patient often complains of glare in dark conditions
(and so difficulty in driving at night) and may complain that colours
appear more dull than they used to. There may be an abnormal red
reflex and, in advanced cases, the cataract may be visible to the naked
eye (this is increasingly rare these days). Other aspects of the
examination should be normal unless there is concurrent pathology.
Refer routinely.

738. A 53yo man was admitted to the hospital for inv of hemoptysis. 3 days
after admission he developed alternating state of consciousness, ataxic gait
and some visual problems. What is the most appropriate management of this
pt?
a. Acamprosate
b. Chlordiazepoxide
c. Diazepam
d.
High potent vitamins
e. Disulfiram
Ans: As we can see from this question that it is not important to find out the
reason for haemoptysis , the reason may be any, he was admitted for
investigations and then around 3 days later he started having some
symptoms. In a hospital setting, if a patient starts having symptoms like
these, one should always think about alcohol withdrawal.
Alcohol withdrawal syndrome is a potentially life-threatening condition that
can occur in people who have been drinking heavily for weeks, months, or
years and then either stop or significantly reduce their alcohol
consumption.When heavy drinkers suddenly stop or significantly reduce
their alcohol consumption, the neurotransmitters previously suppressed by
alcohol are no longer suppressed. They rebound, resulting in a phenomenon
known as brain hyperexcitability. So, the effects associated with alcohol
withdrawal -- anxiety, irritability, agitation, tremors, seizures, and DTs Presentation

This may be in a number of different ways:


A patient may present in acute alcohol withdrawal.
A patient may be admitted to hospital for another reason and thus an
unplanned alcohol withdrawal may be precipitated. Alcohol-use
disorders can complicate the assessment and treatment of other
medical and psychiatric problems.
A patient may present wishing to abstain from alcohol but be seen as
at risk of acute alcohol withdrawal.
Alcohol withdrawal symptoms can begin as early as two hours after the last
drink, persist for weeks, and range from mild anxiety and shakiness to
severe complications, such as seizures and delirium tremens (also called
DTs).

Withdrawal symptoms:
Symptoms typically present about eight hours after a significant fall in
blood alcohol levels. They peak on day 2 and, by day 4 or 5, the
symptoms have usually improved significantly.
Minor withdrawal symptoms (can appear 6-12 hours after alcohol has
stopped):
o Insomnia and fatigue.
o Tremor.
o Mild anxiety/feeling nervous.
o Mild restlessness/agitation.
o Nausea and vomiting.
o Headache.
o Excessive sweating.
o Palpitations.
o Anorexia.
o Depression.
o Craving for alcohol.
Alcoholic hallucinosis (can appear 12-24 hours after alcohol has
stopped):
o Includes visual, auditory or tactile hallucinations.
Withdrawal seizures (can appear 24-48 hours after alcohol has
stopped):
o These are generalised tonic-clonic seizures.
Alcohol withdrawal delirium or 'delirium tremens' (can appear 48-72
hours after alcohol has stopped)

Delirium tremens
This is a medical emergency. A hyperadrenergic state is present.
Clinical features

Delirium tremens usually begins 24-72 hours after alcohol


consumption has been reduced or stopped.
The symptoms/signs differ from usual withdrawal symptoms in that
there are signs of altered mental status. These can include:
o Hallucinations (auditory, visual, or olfactory).
o Confusion.
o Delusions.
o Severe agitation.
Seizures can also occur.

Examination may reveal signs of chronic alcohol abuse/stigmata of


chronic liver disease. There may also be:
o Tachycardia.
o Hyperthermia and excessive sweating.
o Hypertension.
o Tachypnoea.
o Tremor.
o Mydriasis.
o Ataxia.
o Altered mental status.
o Cardiovascular collapse.
MANAGEMENT:
The goals of treatment are threefold: reducing immediate withdrawal
symptoms, preventing complications, and beginning long-term therapy to
promote alcohol abstinence.
Prescription drugs of choice include benzodiazepines, such as diazepam
(Valium), chlordiazepoxide (Librium), lorazepam (Ativan), and oxazepam
(Serax). Such medications can help control the shakiness, anxiety, and
confusion associated with alcohol withdrawal and reduce the risk of
withdrawal seizures and DTs. In patients with mild to moderate symptoms,
the anticonvulsant drug carbamazepine (Tegretol) may be an effective
alternative to benzodiazepines, because it is not sedating and has low
potential for abuse.
Thiamine
Thiamine deficiency is common in people who are alcohol-dependent,
due to their poor diet, the presence of gastritis which can affect its
absorption and also the fact that it is a coenzyme in alcohol
metabolism.
Deficiency can cause Wernicke's encephalopathy, which if left
untreated, can lead to Korsakoff's syndrome.
Oral thiamine is poorly absorbed in dependent drinkers. For this
reason, all those undergoing detoxification in the community should
be considered for admission for parenteral high-potency B complex
vitamins (Pabrinex) as prophylactic treatment. However, because of
the risk of anaphylaxis, resuscitation facilities need to be available at
the time of administration. The risk of anaphylaxis is lower if the drug
is given intramuscularly (IM).
As prophylactic treatment, one pair of ampoules of Pabrinex should
be given IM or intravenously (IV) once a day for three to five days. A
pair of ampoules contains 250 mg of thiamine.

MANAGEMENT OF DT:
This should be in a hospital setting. Intensive care may be needed for
very unwell patients.
It should first include assessment and management of 'Airway,
Breathing and Circulation (ABC)' .
Any hypoglycaemia should be treated.
Sedation with benzodiazepines is suggested. Diazepam has a rapid
onset of action.
Addition of barbiturates may also be necessary in those refractory to
benzodiazepine treatment and may reduce the need for mechanical
ventilation in very unwell patients in the intensive care unit.
Patients with delirium tremens may also have Wernicke's
encephalopathy and should be treated for both conditions
o At least two pairs of ampoules of Pabrinex (500 mg thiamine)
should be given IV three times daily for three days.
o If the patient does not respond, treatment should be
discontinued.
o If signs or symptoms respond to treatment, continue with two
ampoules of Pabrinex once daily for five days or for as long
as improvement continues.
Magnesium may also protect against seizures and arrhythmias
Disulfiram is used to treat chronic alcoholism. It causes unpleasant effects
when even small amounts of alcohol are consumed. These effects include
flushing of the face, headache, nausea, vomiting, chest pain, weakness,
blurred vision, mental confusion, sweating, choking, breathing difficulty,
and anxiety. These effects begin about 10 minutes after alcohol enters the
body and last for 1 hour or more. Disulfiram is not a cure for alcoholism, but
discourages drinking.
Acamprosate, is a medication used along with counselling in the long term
treatment of alcohol dependence.
Acamprosate is thought to stabilize the chemical balance in the brain that
would otherwise be disrupted by alcohol withdrawal. Reports indicate that
acamprosate works to best advantage in combination with psychosocial
support and can help facilitate reduced consumption as well as full
abstinence.
739. A pt underwent hip surgery. Later he presents with SOB and chest
pain. What is the dx?

a.
b.
c.
d.
e.

Pulmonary embolism
MI
Tension pneumothorax
Fat embolism
None

Pulmonary embolism is the sudden blockage of a major artery in the lung,


usually by a blood clot. In most cases, the clots are small and are not deadly,
but they can damage the lung. But if the clot is large and stops blood flow to
the lung, it can be deadly. Quick treatment could save your life or reduce the
risk of future problems.
The most common symptoms are:
Sudden shortness of breath.
Sharp chest pain that is worse when you cough or take a deep breath.
A cough that brings up pink, foamy mucus.
In most cases, pulmonary embolism is caused by a blood clot in the leg that
breaks loose and travels to the lungs. A blood clot in a vein close to the skin
is not likely to cause problems. But having blood clots in deep veins (DVT)
can lead to pulmonary embolism. More than 300,000 people each year have
DVT or a pulmonary embolism.Other things can block an artery, such as
tumors, air bubbles, amniotic fluid, or fat that is released into the blood
vessels when a bone is broken. But these are rare.
Other things that can increase your risk include:
Being inactive for long periods. This can happen when you have to
stay in bed after surgery or a serious illness, or when you sit for a long
time on a flight or car trip.
Recent surgery that involved the legs, hips, belly, or brain.
Some diseases, such as cancer, heart failure, stroke, or a severe
infection.
Pregnancy and childbirth (especially after C section).
Taking Oral contraceptive pills or hormone therapy.
Smoking.
So the scenario in this mcq is most likely pulmonary embolism.
740. A 25yo man presents with hx of breathlessness. A transthoracic echo
reveals a patent foramen ovale. What diagnostic inv would you do for a
patent foramen ovale?
a. Transesophageal echo
b.
Bubble echo

c. Transthoracic echo
d. ECG

Patent foramen ovale


Patent foramen ovale (PFO) is a hole between the left and right atria of the
heart. This hole exists in everyone before birth, but usually closes shortly
after being born. PFO is what the hole is called when it fails to close
naturally after a baby is born.
This hole allows blood to bypass the fetal lungs, which cannot work until
they are exposed to air. When a newborn enters the world and takes its first
breath, the foramen ovale closes, and within a few months it has sealed
completely in about 75 percent of us. When it remains open, it is called a
patent foramen ovale. For the vast majority of the millions of people with a
PFO, it is not a problem, even though blood is leaking from the right atrium
to the left. Problems can arise when that blood contains a blood clot.
Diagnosis of PFO

Patent Foramen Ovale can only be detected by specialized testing. If


suspected, your doctor may order:
Echocardiogram
Transesophageal echo (TEE) an ultrasound test used to visualize
the heart and defect, where an imaging probe with a camera is placed
into the esophagus
Bubble study In some cases during an echo or TEE, the doctor may
inject agitated saline through an IV in your arm with bubbles. During
the test, the doctor can watch the echo to see if bubbles pass from the
right to the left side of the heart to reveal the PFO.

741. A 25yo woman with a hx of several episodes of depression is brought


to the ED after she was found with several empty bottles of her meds. She
complains of coarse tremor, nausea and vomiting. Which of the following
drugs is likely to have caused her symptoms?
a. Fluoxetine
b. Amitryptilline

c.
Lithium
d. Phenelzine
e. Olanzapine

742. A 23yo man feels anxious and agitated when faced with stress. He has
an interview in 3days and would like some help in relieving his symptoms.
What is the most appropriate management?
a. SSRI
b. CBT
c.
Propranolol
d. Diazepam
743. An 8yo boy dx with asthma is on salbutamol and beclomethasone.
However, he wakes up at night due to his symptoms. What is the next
appropriate management?
a.
LABA
b. High dose steroid
c. Aminophylline
d. Oral prednisolone
e. Sodium cromoglycate
744. A woman presents with a hx of poisoning 10x with different
substances. There are no obvious signs of depression or suicidal behavior.
What is the best preventive step?
a. Open access to ED
b. 24h help line
c. CBT
d. Anti-depressants
e.
Insight into problem
Is 745. A boy was rushed to the ED unconscious after he had taken
methadone belonging to the sister. He was given naloxone and he
regained consciousness. After a while he started getting drowsy again.
What is responsible for his present drop in level of consciousness?
a. Naloxone is absorbed faster than methadone
b. Methadone is absorbed faster than naloxone
c. He has also taken another substance apart from methadone

d. The methadone had already caused some brain damage


e.
Naloxone is eliminated faster than methadone
746. A 24yo male on remand in prison for murder is referred by the prison
doctor. He is noted to be behaving oddly whilst in prison and complains of
seeing things. He has a prv hx of IV drug abuse. On questioning he provides
inappropriate but approximate answers to all questions stating that Bill
Clinton is the prime minister of England. What is the prisoner suffering
from?
a. Capgras syndrome
b. Cotard syndrome
c.
Ganser syndrome
d. Ekbom syndrome
e. Tourettes syndrome
747. A 32yo lady has recently become more active, sleeps less and bought
a house and 2 new cars. What is the most likely dx?
a. Bipolar disorder
b. Mania
c.
Hypomania
d. Schizophrenia
748. The body of a 65yo man who was treated for TB and bronchitis was
seen at autopsy. His legs were swollen and his liver showed signs of a
transudate fluid. What was the cause of the transudate?
a. Liver cirrhosis
b. Alcoholic liver disease
c.
Cardiac failure
d. Budd-chiari syndrome
e. TB
749. A 60yo woman has tiredness. She has noticed that her skin looks
permenantly tanned and she describes dizziness on standing up. What is the
single most likely electrolyte pattern to be found?
a.
Na+=120mmol/L, K+=5.9mmol/L
b. Na+=125mmol/L, K+=2.9mmol/L
c. Na+=140mmol/L, K+=4.5mmol/L
d. Na+=150mmol/L, K+=3.5mmol/L
e. Na+=150mmol/L, K+=5.9mmol/L
750. A 20yo girl with amenorrhea and BMI of 14 still thinks she has to lose
weight. What is the most likely dx?
a.
Anorexia nervosa
b. Bulimia nervosa
c. OCD

d. Depression
e. Body dysmorphic disorder
751. A guy who has several convictions and has been imprisoned several
times, breaks up with his family and doesnt contact his children. What type
of personality disorder is this?
a. Borderline
b.
Antisocial
c. Schizotypal
d. Schizoid
e. Criminal
Cluster A

They tend to be odd or eccentric:


Paranoid personality disorder They display pervasive distrust and
suspicion. Common beliefs include:
Others are exploiting or deceiving them.
Friends and associates are untrustworthy.
Information confided to others will be used maliciously.
There is hidden meaning in remarks or events others perceive as
benign.
The spouse or partner is unfaithful. Pathological jealousy is
sometimes called the Othello syndrome.
Schizoid personality disorder This type of personality disorder is less
common in clinical settings:
They are detached from others and have little desire for close
relationships.
There is little pleasure in activities.
They appear indifferent to praise or criticism and often seem cold or
aloof.
Schizotypal personality disorder They show marked eccentricities of
thought, perception and behaviour. Typical examples include:
Ideas of reference (believing that public messages are directed
personally at them).
Odd beliefs or magical thinking.
Vague, circumstantial, or stereotyped speech.
Excessive social anxiety that does not diminish with familiarity.
Idiosyncratic perceptual experiences or bodily illusions.
Cluster B

They tend to be dramatic and emotional:


Antisocial personality disorder See separate article Antisocial Personality
Disorder and National Institute for Health and Clinical Excellence (NICE)
guidance. People with antisocial personality disorder have a pervasive
pattern of disregard for and violation of the rights of others and the rules of
society. Onset must occur by age 15 years.( Criminal behavior is central to
its definition)
[5]

Borderline personality disorder See separate article Borderline Personality


Disorder and NICE guidance. The important feature of borderline
personality disorder is a pervasive pattern of unstable and intense
interpersonal relationships, self-perception and moods. Impulses are poorly
controlled. At times they may appear psychotic because of the intensity of
their distortions. It is a commonly overused diagnosis in DSM-IV.
[6]

Histrionic personality disorder


They also display excessive emotion and attention-seeking behaviour.
They are quite dramatic and often sexually provocative or seductive.
Their emotions are labile.
In clinical settings, their tendency to vague and impressionistic speech
is often apparent.
Narcissistic personality disorder Narcissistic patients are grandiose and
require admiration from others. Features include:
Exaggeration of their own abilities or achievements.
Sense of entitlement.
Exploitation of others.
Lack of empathy.
Envy of others.
An arrogant, haughty attitude.
[7]

Cluster C

They tend to be anxious and fearful:


Avoidant personality disorder
They are generally very shy.
There is a pattern of social inhibition, feelings of inadequacy and
hypersensitivity to rejection.

Unlike patients with schizoid personality disorder, they do desire


relationships but are paralysed, by their fear and sensitivity, into social
isolation.
Dependent personality disorder Many people exhibit dependent
behaviours and traits but people with dependent personality disorder have an
excessive need to be cared for and that results in submissive and clinging
behaviour, regardless of consequences. Diagnosis requires at least five of the
following features:
Difficulty making decisions without guidance and reassurance.
Need for others to assume responsibility for most major areas of the
person's life.
Difficulty expressing disagreement with others.
Difficulty initiating activities because of lack of confidence.
Excessive measures to obtain nurture and support.
Discomfort or helplessness when alone.
Urgent seeking for another relationship when one has ended.
Unrealistic preoccupation with fears of being left to self-fend.
Obsessive-compulsive personality disorder
People with obsessive-compulsive personality disorder are markedly
preoccupied with orderliness, perfectionism and control.
They lack flexibility or openness.
Their preoccupations interfere with efficient function despite their
focus on tasks.
They are often scrupulous and inflexible about matters of morality,
ethics and values, to a point beyond cultural norms.
They are often 'stingy' as well as stubborn.

752. A 17yo lady presents with a worm in her ear. She is very agitated and
anxious. What is the next step?
a. Remove under GA
b. Suction
c.
Alcohol drops
d. Forceps
753. A 20yo male smoker is noted to have intense rubor of the feet and
absent foot pulse. Exam: amputated right 2 toe. What is the most probable
dx?
a. Intermittent claudication
b. Cardiovascular syphilis
nd

c.
Buergers disease
d. Embolism
e. Acute limb ischemia
754. A young lady after a heavy bout of drinking last night comes to the
ED with dizziness, abdominal pain, vomiting blood with cool peripheries.
After initial resuscitation, oxygen and fluids, she still continues to bleed with
pulse=130bpm and BP=85/58mmHg. What would be your next best
management?
a. Clotting screen
b. US
c. CT
d.
Endoscopy
e. Omeprazole
755. A 12yo boy complains of acute development of purpura on the
dependent areas of his body 2wks after a URTI. The purpura doesnt blanch
on pressure, tests reveal: Hgb=12, plts=50, WBC=5. Bleeding time=10mins,
APTT=40s, PT=1.02. What is the most likely dx?
a.
ITP
b. TTP
c. Von Willebrands disease
d. Hemophilia A
e. Hemophilia B
756. A woman presents with hx of falls, becomes pale and clumsy. She is
hypertensive and takes atenolol, bendroflumethiazide and amlodipine. What
inv is needed?
a. 24h ECG
b.
24h BP monitoring
c. ECG
d. Echo
e. CT head
757. A 43yo woman has been feeling lethargic and tired. Her
BP=160/90mmHg. Bloods: Na+=140mmol/L, K+=3.1mmol/L. What is the
most likely dx?
a. Cushings syndrome
b.
Conns syndrome
c. Hyperparathyroidism
d. Renal disease
e. Pheochromocytoma
Consider Conn's syndrome if hypertension, hypokalemia or alkalosis in patient not on diuretics.
Causes of hypertension with hypokalemia
1. Essential HTN with diuretic use

2. Conns
3. Cushings
4. Pheochromocytoma
5. Malignant hypertension

758. A 2yo child aspirated a foreign object which was removed at the hosp.
the parents are now asking how to remove it if that ever happens at home.
What do you advise?
a. Hemlich maneuver
b. Bring to the hospital
c.
Turn the child on his back and give thumps
d. CPR
e. Remove manually by fingers
759. A 28yo pregnant lady presents with severe lower abdominal pain with
excessive per vaginal bleeding at 34wks gestation. What should be the initial
inv of choice?
a. Coagulation profile
b.
US abdomen
c. CT pelvis
d. D-dimer
e. Kleiuber test
760. A 3yo child with severe diarrhea and vomiting, looks lethargic, has
sunken eyes and a feeble cry. What is the choice of fluids?
a.
0.9%NS
b. 0.9%NS + 5%Dextrose
c. 0.45%NS + 5%Dextrose
d. 0.45%NS
761. A lady with depression has a bag full of meds. She now presents with
coarse tremors. Which drug caused her symptoms?
a.
Lithium
b. Thyroxine
c. Amitriptyline
d. Sodium valproate
e. Tetrabenazine
762. A 38yo man has had a liver biopsy as part of inv for abnormal LFTs.
The p athologist report states: special stains demonstrate the presence of a
very large amount of iron pigment within the hepatocytes. What single
condition is identified by the pathologists report?
a. Alpha 1 antitrypsin deficiency
b. Hemangioma
c. Hemochromatosis

d.
Hemosiderosis
e. Wilsons disease
The term "hemosiderosis" is used to denote a relatively benign accumulation of iron. The term
"hemochromatosis" is used when organ dysfunction occurs..A Prussian blue iron stain demonstrates the
blue granules of hemosiderin in hepatocytes and Kupffer cells. Hence i think pathologist will describe it as
hemosiderosis whereas clinicians after ruling out the organ dysfunction termed it as a hemochromatosis..
SO i think answer here would be D ( hemosiderosis)because question is asking about pathologist report.

763. A 27yo man presents to the ED after a RTA where his foot was stuck
under a truck for several hours. He now has swelling on the foot. Exam: foot
tenderness, loss of sense in the space between the 3 metatarsal and big toe
and his dorsalis pedis is not felt. What is the most likely dx?
a.
Compartment syndrome
b. Arterial rupture
c. Arterial embolus
d. DVT
e. Fibular fx
764. You are a FY doctor in the ED when a mother brings her 2yo son to
you with a 1h hx of noisy breathing. She state that although he had mild
coryza over the last week, he was improving and so they had gone to a
childrens picnic with nursery friends. Another parent had found him
coughing and spluttering, and ever since his breathing has remained noisy.
Though he appears well in the ED, his current observations demonstrate a
raised RR and sat=91% on air. What is the most likely dx?
a. Anaphylaxis
b. Croup
c.
Foreign body aspiration
d. Epiglottitis
765. A pt taking doxycycline complains of nausea, indigestion, abdominal
pain and vomiting. What will you advise?
a.
Take it after meals
b. Take it before meals
c. Stop the drug
d. Take antiacids
e. Take antiemetic
rd

766. A 49yo man lost his job and now is homeless. He was found
wandering in the park. He is muttering that some people are after him.
Alcohol was tested and it was negative. What will your next step be?
a. Thiamine
b.
Neuropsycho analysis

c. Mini mental state


d. CT head
e. MRI head
ans: Middle aged man, presenting with persecutory delusions. Age is not in favour of
schizophrenia. There are stressors present, loss of job and homeless. Substance abuse
ruled out for alcohol. Not other drugs ruled out. So go for neuropsychiatric analysis to
find out cause. Ct would have been indicated after trauma, mri for focal symptoms.
Thiamine in alcohol intoxication.Minimental is specific for cognition and wouldnt point
towards diagnosis.

767. A pt with SNHL and loss of corneal reflex on the left side. What is
the most definitive inv?
a. CT of internal auditory meatus
b. Nuclear imaging of brain
c.
MRI of internal auditory meatus
d. Radio isotope scan
e. XR skull
Unilateral sensorineural hearing loss and loss of corneal reflex indicate
involvement of facial, trigeminal and vestibulocochlear nerve which is
common in acoustic neuroma. MRI of internal auditory meatus and CP
angle would show the tumour.
Acoustic Neuroma:
Presentation: progressive ipsilateral tinnitus, sensorineural deafness. Big
tumours may cause ipsilateral cerebellar signs or raised ICP. 5, 6, 7
cranial nerves at risk.
Investigations: MRI
TREatment: surgery
768. A middle aged man complains of a node which has been growing on
his nose for several months. Now its firm with central depression. It is
0.6cm in size. What is the single most likely dx?
a.
Basal cell carcinoma
b. Squamous cell carcinoma
c. LN
d. Melanoma
e. Kaposis sarcoma
f. Keratoacanthoma
g. Molluscumcontagiosum
Basal Cell carcinoma presents with pearly nodule with rolled
telangiectatic edge on the face. Lesions on trunk can appear as red scaly
plaques with raised smooth edge.

Causes: uv exposure
Treatment: excision and radiotherapy
769. A 45yo woman presents with rotational vertigo, nausea and
vomiting, especially on moving her head. She also had a similar episode
2yrs back. These episodes typically follow an event of runny nose, cold,
cough and fever. What is the most probable dx?
a. Acoustic neuroma
b. Menieres disease
c. Labyrinthitis
d. BPPV
e.
Vestibular neuronitis
above scenario is that of vestibular neuronitis.
Symptoms of rotational vertigo nausea and vomiting followed an event of
viral illness.
Acoustic neuroma, would have some other cranial nerve involvement.
Menieres is characterised by spisodes of vertigo, sensorineural hearing
loss, sense of aural fullness and tinnitus.
BPPV is characterised by vertigo and dizziness related to head movement.
Labyrinthitis resembles vestibular neuronitis but with additional hearing
loss.
Presentation: nausea, vomiting and vertigo .moving head aggravates
symptoms.
Nystagmus
Investigations: clinical diagnosis.
MRI ct to rule out other causes
Differntials: BPPV, labyrinthitis, Meniers
Management: reassurance. Bed rest.If severe,
antiemetics.Prochlorperazine.
770. A 65yo man with cancer of middle 1/3 of the esophagus presents
with dysphagia. What is the most immediate management?
a. Chemotherapy
b. Radiotherapy
c.
Stenting
d. Gastrostomy
e. TPN
Squamous cell carcinoma of the esophagus.Presenting with dysphagia.
Treatment: for early esophageal cancer, endoscopic resection or
submucosal dissection

Endoscopic Esophagectomy TOC for most advanced


Stenting is the first line approach to assist swallowing.
Radio chemo may be of used primarily in reducing bulk of tumour,
Nutritional status: use liguid feeds or PEG tubes.
771. A 1yo child is brought to ED. He woke up in the middle of the night
crying severely. What initial measure should be taken for this child?
a. Refer to surgeon
b. Discharge with advice
c.
Analgesia
d. Antibiotic
information provided seems insufficient.
However, an infant can only express underlying pain by crying. So
immediate treatment could be analgesia to relax and baby and proceed
with further examination to find out the cause.
772. A 30yo lady was playing volleyball when her hand got injured with
the ball. The right hand is not swollen and there is tenderness under the
root of the thumb. XR is normal. What is the most appropriate next
management?
a. Arm sling for 1 wk
b. Raise had for 2d
c.
Repeat XR
d. Full arm cast
773. A 33yo female presents with sudden severe colicky abdominal pain
in her RIF. A mobile mass is felt on examination. What is the most likely
dx?
a. PID
b. Appendicitis
c.
Ovarian torsion
d. Constipation
Severe colicky abdominal pain in rif could be ureteric colic or ovarian
torsion.
Mobile mass and female gender indicates its ovarian torsion
Ovarian Torsion: Usually happens when ovaries enlarge via cysts. There
may be adnexal tenderness.
Ultrasound: torsion may be intermittent presenting with intermittent pain.
Treatment: initially by laparoscopy and possible oophoropexy.
If severe vascular compromise or peritonitis: salpingo oophorectomy.

774. A middle aged male is feeling unwell after a recent MI. The recent
ECG shows prolonged QRS complex and tented T wave.
Na+=136mmol/l, K+=6.2mmol/l, urea=5mmol/l. What is the most
appropriate management?
a.
Calcium gluconate
b. Calcium resonium
c. Calcium with vit D supplement
d. Vit D
e. Calcium
Ecg abnormalities and a raised potassium level needs immediate
treatment of hyperkalemia.
Symptoms: fast irregular pulse, chest pain, palpitations or weakness
Ecg: tall tented T waves, small P waves, wide QRS, vfib
Causes: oliguric renal failure, potassium sparing diuretics,
rhabdomyolysis, metabolic acidosis, burn
Treatment:
Urgent or Non urgent
Urgent: 10 ml calcium gluconate over 2 minutes
Insulin+ glucose
Nebulise with salbutamol
Calcium resonium
Dialysis
775. A 68yo man with DM and HTN was noted to have cholesterol level
of 3.4mmol/l. he was also noted to have microalbuminuria. What is the
best drug to add to his regimen?
a.
ACEi
b. Statin
c. Amylodipine
d. Biguanides
Cholesterol levels are normal.
In hypertension and diabetes, acei are the drugs employed to slow down
renal progression of disease. They are the DOC for proteinuria.
776. A child playing with toys suddenly develops breathlessness and
stridor. Which inv will lead to the dx?
a.
Laryngoscopy
b. CXR
c. Peak flow meter
d. ABG

scenario of foreign body intake by the child. He has developed


breathlessness and stridor indicating airway obstruction. So laryngoscopy
will help in visualising and extraction of the foreign body.
777. Anxious parents ask you for resus technique for their 3yo. What do
you tell them?
a. 5 compression: 1 breath
b. 5 compression: 2 breaths
c.
15 compression: 2 breaths with nose pinched
d. 15 compression: 2 breaths without nose pinched
e. 30 breaths: 2 compressions
778. A 56yo man complains of increased vol of sputum with specks of
blood and chest pain. He has ahx of DVT. Exam: clubbing. What is the
cause of blood in his sputum?
a. Pulmonary thrombosis
b. Bronchial carcinoma
c.
Bronchiectasis
d. Pulmonary TB
Bronchiectasis
permanent dilatation and thickeningof airways characterised by cough,
copious purulent sputum, recurrent infection
signs: coarse crackles, often in lower zones.
Ronchi
Wheeze
Inv:
CXR
HRCT gold standard
Sputum microbiology
Differentials: COPD, Asthma, TB, pneumonia, sinusitis
Treatment: chest physio, postural drainage
Stop smoking
Antibiotics
Amoxicillin is first line
Cipro in pseudomonas patients for 14 days
779. A 32yo female has ahx of SOB and fever. Pre-broncho-dilation test
was done and it was 2/3.5 and post-bronchodilator was 3/3.7. The pt was
dx of eczema and TB. What is the possible dx?
a. COPD
b.
Asthma

c. Pneumonia
d. Bronchiectasis
Asthma:
Shortness of breath and fever alongwith eczema hx. Points toward asthma
Precipitants: cold exercise nsaid b blockers smoking infection
Associated with acid reflux and other atopic disease
Diagnosis:
Fev1 is reduced more than fvc and ratio is less than 75: obstructive defect
Greather than 15% improvent in fev1 with bronchodilators as in this
scenario.
Management
Step 1: short acting b agonist inhaler, salbutamol
If not controlled or night symptoms, use more than twice a week
Step two: add low dose inhaled corticosteroid
Step 3 add long acting b agonist. If little improvement inc dose of inhaled
corticosteroid. If no improvement, stop LABA. And add leukotriene
receptor antagonist
Step 4: increase inhaled corticosteroid or oral t5heophylline
Step 5 oral corticosteroid
780. A 2yo male pt was brought by his mother with a swelling in the
right side of his neck extending from the angle of the mouth to the middle
1/3 of the sternocleidomastoid muscle. The swelling was on the
anterolateral side of the sternocleidomastoid and was brilliantly
transilluminant. What is the likely dx?
a. Lymphangioma
b.
Branchial cyst
c. Thyroglossal cyst
d. Ranula
e. Thyroid swelling
Branchial Cyst:
Commonly presents as painless mass in the neck of a child. Discharge
maybe associated if present with sinus tract. Smooth non tender fluctuant
masses along the lower one thord of anterior border of SCM
Differentials: lymphadenopathy
Carotid body tumour
Cystic hygroma
Ectopic thyroid
Investigation

Ultrasound
Ct
Surgical excision
781. A 50yo newly dx with HTN complains of urinary freq and dysuria.
The urinalysis reveals presence of white cells and protein. Choose the
single most appropriate tx?
a. Imipramine
b. Adjust diuretics
c. Vaginal estrogen
d.
Trimethoprim
bacterial growth of 10 power 5 org/ml. in MSU
urethritis, cystitis,
upperuti: pyelonephritis
e.coli main organism, strep, proteus
PC: frequency, urgency, dysuria, hematuria, suprapubic pain.
Inv: urine dipstick
MSU
Urine CS
Rx: trimethoprim or nitrofurantoin
2 line: co amoxiclav
782. A boy injured his ear during a rugby match. He reported it being
painful. Exam: red and tender pinna. Tympanic membrane was normal.
What would be the next appropriate step?
a. Topical gentamicin
b. Oral flucloxacillin
c. IV flucloxacillin
d. Refer to ENT specialist
e.
No further intervention needed
scenario most probably of perichondrial hematoma.
If not surgically explored and aspirated can lead to malformed ear or
cauliflower ear which is cosmetically undesirable. Common in boxers
783. A 2yo girl prv well presents with ahx of vomiting and diarrhea for
4hrs. What is the most suitable indication for IV fluid administration?
a.
Capillary refill time >4s
b. HR >90bpm
c. Increased RR
d. Stool >10x/d
e. Weight of child = 10kgs
nd

784. A 44yo woman with breast cancer had an extensive removal and
LN clearance. She needs an adjunctive tx. Her mother had cancer when
she was 65. Which of the following factors will be against the tx?
a. Famhx
b. Tumor grading
c. LN involvement
d.
Her age
adjunctivetx is given in LN involvement. Tumor grading should also be done to stage it.

785. A 45yo man presents with hearing loss and tinnitus in the right ear.
Exam: weber test lateralizes to the left. Audiometry: AC > BC in both
ears. What is the next best inv?
a. CT
b.
MRI brain
c. Angiogram
d. Otoscopy
Unilateral sensorineural hearing loss and loss of corneal reflex indicate
involvement of facial, trigeminal and vestibulocochlear nerve which is
common in acoustic neuroma. MRI of internal auditory meatus and CP
angle would show the tumour.
Acoustic Neuroma:
Presentation: progressive ipsilateral tinnitus, sensorineural deafness. Big
tumours may cause ipsilateral cerebellar signs or raised ICP. 5, 6, 7
cranial nerves at risk.
Investigations: MRI
TREatment: surgery
786. A 74yo lady called an ambulance for an acute chest pain. She has
ahx of DM and HTN, and is a heavy smoker. Paramedics mentioned that
she was overweight and recently immobile because of a hip pain. She
collapsed and died in the ambulance. What is the most likely cause of
death?
a.
Pulmonary embolism
b. MI
c. Stroke
d. Cardiac arrhythmia
e. Cardiac failure
acute onset of chest pain and hx of immobility point towards pul
embolism.
Causes: DVT
After long bone fracture

Amniotic fluid
Air embolism
Immobility
Risk factors for venous thromboembolism 3]
[

Major risk factors: relative risk of 5-20

Minor risk factors: relative risk of 2-4

Cardiovascular:
Surgery:

Major abdominal/pelvic

surgery
or hip/knee replacement
(risk lower if prophylaxis used).

Postoperative intensive care.

Obstetrics:

Congenital heart disease.

Congestive cardiac failure.

Hypertension.

Paralytic stroke.

Oestrogens:

Pregnancy (but see major risk

factors for late pregnancy and


puerperium).

Late pregnancy.

Combined oral contraceptive.

Puerperium.

Hormone replacement

Caesarean section.

therapy.

Lower limb problems:

Haematological:

Fracture.

Varicose veins - previous

detailed list is available)

varicose vein surgery;

Thrombotic disorders (a

Consider this in cases of PE aged


<40 years, recurrent VTE or a
positive family history.

superficial thrombophlebitis;
varicose veins per se are not a risk

Myeloproliferative disorders.

Renal:

factor.

Nephrotic syndrome.

Malignancy:

Chronic dialysis.
Paroxysmal nocturnal

Abdominal/pelvic.

Advanced/metastatic.

haemoglobinuria.

Reduced mobility:

Miscellaneous:

Hospitalisation.

Institutional care.

disease (COPD).

Previous proven VTE:

Chronic obstructive pulmonary

Neurological disability.

Occult malignancy.

(could be major or

Long-distance sedentary

minor risk factor:

travel.

no data on relative risk).

Intravenous (IV) drug use

Other:

Obesity.

Other chronic diseases:


inflammatory bowel disease,
Behet's disease.

Major trauma.

Spinal cord injury.

Central venous lines.

Dyspnoea.

Pleuritic chest pain, retrosternal chest pain.

Cough and haemoptysis.

Any chest symptoms in a patient with symptoms suggesting a deep vein

thrombosis (DVT).

In severe cases, right heart failure causes dizziness or syncope.

Signs include:

Tachypnoea, tachycardia.

Hypoxia, which may cause anxiety, restlessness, agitation and impaired

consciousness.

Pyrexia.

Elevated jugular venous pressure.

Gallop heart rhythm, a widely split second heart sound, tricuspidregurgitant

murmur.

Pleural rub.

Systemic hypotension and cardiac arrest


Offer patients in whom PE is suspected and with a likely two-level PE Wells' score
either an immediate computed tomography pulmonary angiogram (CTPA) or
immediate interim parenteral anticoagulant therapy followed by a CTPA, if a CTPA
cannot be carried out immediately. Consider a proximal leg vein ultrasound scan if
the CTPA is negative and DVT is suspected.

Offer patients in whom PE is suspected and with an unlikely two-level PE Wells'


score a D-dimer test and, if the result is positive, offer either an immediate CTPA or
immediate interim parenteral anticoagulant therapy followed by a CTPA, if a CTPA
cannot be carried out immediately.

For patients who have an allergy to contrast media, or who have renal impairment, or
whose risk from irradiation is high:
o

Assess the suitability of a ventilation/perfusion single-photon emission


computed tomography (V/Q SPECT) scan or, if a V/Q SPECT scan is not
available, a V/Q planar scan, as an alternative to CTPA.

If offering a V/Q SPECT or planar scan that will not be available immediately,
offer immediate interim parenteral anticoagulant therapy.

Diagnose PE and treat patients with a positive CTPA or in whom PE is identified with
a V/Q SPECT or planar scan.

Consider alternative diagnoses in the following two groups of patients:

Patients with an unlikely two-level PE Wells' score and either a negative D-dimer
test, or a positive D-dimer test and a negative CTPA.

Patients with a likely two-level PE Wells' score and both a negative CTPA and no
suspected DVT.

nitial resuscitation

Oxygen 100%.

Obtain IV access, monitor closely, start baseline investigations.

Give analgesia if necessary (eg, morphine).

Assess circulation: suspect massive PE if systolic BP is <90 mm Hg or there is a

fall of 40 mm Hg, for 15 minutes, not due to other causes.

Anticoagulation therapy[4]

Offer a choice of low molecular weight heparin (LMWH) or fondaparinux to

patients with confirmed PE, with the following exceptions:

For patients with severe renal impairment or established chronic kidney

disease (estimated glomerular filtration rate (eGFR) <30 ml/min/1.73 m2) offer
unfractionated heparin (UFH) with dose adjustments based on the activated partial
thromboplastin time (aPTT) or LMWH with dose adjustments based on an anti-Xa
assay.

For patients with an increased risk of bleeding, consider UFH.

For patients with PE and haemodynamic instability, offer UFH and consider

thrombolytic therapy.

Start the LMWH, fondaparinux or UFH as soon as possible and continue it for at

least five days or until the international normalised ratio (INR) is 2 or above for at least
24 hours, whichever is longer.

Offer LMWH to patients with active cancer and confirmed PE, and continue the

LMWH for six months. At six months, assess the risks and benefits of continuing
anticoagulation.

Offer a vitamin K antagonist (VKA) to patients with confirmed PE within 24 hours

of diagnosis and continue the VKA for three months. At three months, assess the risks
and benefits of continuing VKA treatment.

Offer a VKA beyond three months to patients with an unprovoked PE, taking into

account the patient's risk of VTE recurrence and whether they are at increased risk of
bleeding.

Rivaroxaban: 7]
[

Rivaroxaban is recommended by NICE as an option for treating PE and

preventing recurrent DVT and PE in adults.

The duration of treatment recommended depends on bleeding risk and other

clinical criteria.

Short-term treatment (at least three months) is recommended for people with

transient risk factors such as recent surgery and trauma. Longer treatment is
recommended for people with permanent risk factors, or idiopathic (unprovoked)
DVT or PE.

nitial resuscitation

Oxygen 100%.

Obtain IV access, monitor closely, start baseline investigations.

Give analgesia if necessary (eg, morphine).

Assess circulation: suspect massive PE if systolic BP is <90 mm Hg or there is a

fall of 40 mm Hg, for 15 minutes, not due to other causes.

Anticoagulation therapy[4]

Offer a choice of low molecular weight heparin (LMWH) or fondaparinux to

patients with confirmed PE, with the following exceptions:

For patients with severe renal impairment or established chronic kidney

disease (estimated glomerular filtration rate (eGFR) <30 ml/min/1.73 m2) offer
unfractionated heparin (UFH) with dose adjustments based on the activated partial
thromboplastin time (aPTT) or LMWH with dose adjustments based on an anti-Xa
assay.

For patients with an increased risk of bleeding, consider UFH.

For patients with PE and haemodynamic instability, offer UFH and consider

thrombolytic therapy.

Start the LMWH, fondaparinux or UFH as soon as possible and continue it for at

least five days or until the international normalised ratio (INR) is 2 or above for at least
24 hours, whichever is longer.

Offer LMWH to patients with active cancer and confirmed PE, and continue the

LMWH for six months. At six months, assess the risks and benefits of continuing
anticoagulation.

Offer a vitamin K antagonist (VKA) to patients with confirmed PE within 24 hours

of diagnosis and continue the VKA for three months. At three months, assess the risks
and benefits of continuing VKA treatment.

Offer a VKA beyond three months to patients with an unprovoked PE, taking into

account the patient's risk of VTE recurrence and whether they are at increased risk of
bleeding.

Rivaroxaban: 7]
[

Rivaroxaban is recommended by NICE as an option for treating PE and

preventing recurrent DVT and PE in adults.

The duration of treatment recommended depends on bleeding risk and other

clinical criteria.

Short-term treatment (at least three months) is recommended for people with

transient risk factors such as recent surgery and trauma. Longer treatment is
recommended for people with permanent risk factors, or idiopathic (unprovoked)
DVT or PE.

787. A 77yo woman suffered diarrhea 4d ago. What would be her blood
gas result?
a. pH =7.2, PaCO2 =8
b.
pH =7.4, PaCO2 =1.5
c. pH =7.4, PaCO2 =2.6
d. pH =7.4, PaCO2 =2.8
Diarrhea is a cause of metabolic acidosis. In the initial states there will be
respiratory compensation leading to normalisation of ph and low
carbondioxide by washing out carbondioxide. With time, the kidney will
also save bicarbonate and bring ph back to normal. In this question, b, c, d
all are suitable but best is option B.
788. A man presents with abdominal pain, vomiting, pulse=120bpm,
BP=90/60mmHg and a rigid abdomen. His chest is clear. What is the
immediate management?
a. Call radiology
b. Admit to medical ward
c.
Urgent admission to ITU
patient presenting with acute abdomen. Peritonitis. First thing would be
to shift him to ICU and stabilise.
789. A 60yo man presents with weight loss and Hgb=6. Hx reveals that
he has abdominal pain and diarrhea for the past 6m. What is the most
appropriate inv?
a. Barium enema
b.
Colonoscopy
c. Sigmoidoscopy
presenting features are consistent with Colonic carcinoma.

Risk factors

Family history of colorectal neoplasia: carcinoma; adenoma under

the age of 60 years.[3]

Past history of colorectal neoplasm: carcinoma, adenoma.

Inflammatory bowel disease: ulcerative colitis, Crohn's colitis.

Polyposis syndromes: familial adenomatous polyposis (Gardner's

syndrome), Turcot's syndrome, attenuated adenomatous polyposis coli,


flat adenoma syndrome, hamartomatous polyposis syndromes (PeutzJeghers syndrome, juvenile polyposis syndrome, Cowden's syndrome).

Hereditary non-polyposis colorectal cancer (HNPCC).

Hormonal factors: nulliparity, late age at first pregnancy, early

menopause.

Diet: rich in meat and fat; poor in fibre, folate and calcium.

Sedentary lifestyle, obesity, smoking, high alcohol intake.

Diabetes mellitus.

Previous irradiation, occupational hazards - eg, asbestos exposure.

History of small bowel cancer, endometrial cancer, breast cancer or

ovarian cancer.

Presentation

The presentation depends on the site of the cancer:

Right colon cancers: weight loss, anaemia, occult bleeding, mass in right
iliac fossa, disease more likely to be advanced at presentation.

Left colon cancers: often colicky pain, rectal bleeding, bowel obstruction,

tenesmus, mass in left iliac fossa, early change in bowel habit, less advanced
disease at presentation.

The most common presenting symptoms and signs of cancer or large polyps are
rectal bleeding, persisting change in bowel habit and anaemia.
All patients with symptoms suspicious of colorectal cancer must have a thorough
abdominal examination and rectal examination.

Inv:
Colonoscopy and biopsy
Ct for staging
Rx
Surgery
Chemo radio for advanced cases

790. A 35yo primigravida post C-section complains of inability to void.


She denies dysuria but complains of fullness. She was treated with an
epidural for analgesia. What is the single most appropriate inv?
a. MSU
b.
US abdomen
c. US KUB
d. Serum calcium
Answer= C. US KUB. This is the case of postoperative urinary retention (the risk
factors for it include operation itself and spinal anesthesia) so US KUB should be
done to measure the urinary volume of bladder. it will guide in the treatment plan. if
the urine volume is <400ml observe the patient and if the urine volume is > 600ml
catheterize the patient.

791. A 35yo primigravida post C-section complains of inability to void.


She denies dysuria but complains of fullness. She was treated with an
epidural for analgesia. What is the single most appropriate inv?
a. MSU
b.
US abdomen
c. US KUB
d. Serum calcium
Answer= C. US KUB. This is the case of postoperative urinary retention (the risk factors
for it include operation itself and spinal anesthesia) so US KUB should be done to
measure the urinary volume of bladder. it will guide in the treatment plan. if the urine
volume is <400ml observe the patient and if the urine volume is > 600ml catheterize the
patient.

792. A female pt with HTN and serum K+=2.7mmol/l. which hormone


would you be looking for?
a.
Aldosterone

b.
c.
d.
e.

Cortisol
Thyrotoxin
Renin
Testosterone

Answer= A. Aldosterone. hypertension with hypokalemia points towards the diagnosis of


primary hyperaldosteronism so aldosterone levels will be high in this patient.

PRIMARY HYPERALDOSTERONISM
Primary hyperaldosteronism was previously thought to be most commonly caused by an
adrenal adenoma, termed Conn's syndrome. However, recent studies have shown that
bilateral idiopathic adrenal hyperplasia is the cause in up to 70% of cases. Differentiating
between the two is important as this determines treatment. Adrenal carcinoma is an
extremely rare cause of primary hyperaldosteronism
Features
hypertension
hypokalaemia (e.g. muscle weakness)
alkalosis
Investigations
high serum aldosterone
low serum renin
high-resolution CT abdomen
adrenal vein sampling
Management
adrenal adenoma: surgery
bilateral adrenocortical hyperplasia: aldosterone antagonist e.g. spironolactone

793. A 25yo male with a hx of frequent binge drinking presents 4h after


having had a take away meal following a nights heavy drinking. He
complains of nausea and has vomited on several occasions. After the last
vomiting episode, he vomited approximately a cupful of blood. On
admission, he smells of alcohol, pulse=100bpm, BP=140/80mmHg. He has
some tenderness in the epigastrium. What is your dx?
a. Gastric carcinoma
b.
Mallory-weiss tear
c. Esophageal carcinoma
d. Esophageal varices
e. Esophageal varices
f. Peptic ulceration
B. Mallory-weiss tear. Alcoholic patient with the complains of several
episodes of vomiting followed by hematemesis all point towards the
diagnosis of mallory weiss tear.

MalloryWeiss tear
Persistent vomiting/retching causes haematemesis via an oesophageal
mucosal tear
Aetiology:
Haematemesis due to a Mallory Weiss tear usually occurs after a prolonged or forceful bout
of retching, vomiting, coughing, straining or even hiccupping.
Risk factors:
excessive alcohol consumption.
conditions predisposing to retching and vomiting: e.g gastroenteritis, hyperemesis
gravidarum,bulimia, renal disease,hepatitis, migraine, raised icp etc.
Presentation:
The classic presentation is of haematemesis following a bout of retching or vomiting.
However, a tear may occur after a single vomit.
Other symptoms include melaena, light-headedness, dizziness, or syncope, and features
associated with the initial cause of the vomiting - eg, abdominal pain
Diagnosis:
Endoscopy is the primary diagnostic investigation.
Treatment:
Resuscitation is a priority - maintain airway, provide high-flow oxygen, correct fluid losses.
MWS usually follows a benign course but occasionally endoscopic treatment is required to
stop bleeding.

794. A young boy presented with bilateral periorbital edema, ankle


swelling and increase in body weight. What is the most likely dx?
a. Chronic heart failure
b.
Nephrotic syndrome
c. Renal failure
d. Acute heart failure
e. Glomerulonephritis
Answer: B. Nephrotic syndrome.
NEPHROTIC SYNDROME
The nephrotic syndrome is a triad of:
Proteinuria >3.5g/24h (ACR >250 mg/mmol)
Hypoalbuminemia (<25g/L, usually much lower)
Oedema
Severe hyperlipidaemia (total cholesterol >10 mmol/L) is often present
causes:
It can be due to primary renal disease or secondary to a number of systemic
disorders.

Primary causes: Minimal change disease, membranous nephropathy, focal


segmental glomerulosclerosis (FSGS), mesangiocapillary GN (MCGN).
Secondary causes: Hepatitis B/C (usually membranous, hep C can cause
MCGN),
SLE (class V lupus nephritis causes a membranous pattern), diabetic
nephropathy,
amyloidosis, paraneoplastic (usually membranous pattern) or drug related
(again
usually membranousNSAIDS, penicillamine, anti-TNF, gold).
Treatment:
(In children the peak incidence is between 2 and 5 years of age. Around 80% of cases in
children are due to a condition called minimal change glomerulonephritis. The condition
generally carries a good prognosis with around 90% of cases responding to high-dose oral
steroids.)

Reduce oedema: Loop diuretics, eg furosemide are used, often high doses
are
needed.
Reduce proteinuria: ACE-i or ARB should be started in all patients.
Reduce risk of complications
Treat underlying condition.
795. A 53yo man with prv hx of COPD presents with breathlessness and
purulent sputum. O2 stat=85% on air. ABG: PaO2=7.6, PaCOS=7. What is
the appropriate management for his condition?
a.
24% oxygen
b. Mechanical ventilation
c. 100% oxygen
d. Nebulized salbutamol
Answer= A. 24% oxygen. this is acute exacerbation of copd so 24% oxygen
is given first.

796. A 34yo man was involved in a RTA and whilst in the ambulance his
GCS deteriorated and RR increased from 30-48. What is the most
appropriate management for this pt?
a. IV fluid
b. Needle thoracocentesis
c.
100% oxygen
d. Portable XR
Answer= C. 100% oxygen. RTA case so we will follow ABC protocol.
The ATLS programme uses the ABCDE mantra.
It prioritizes direct treatment according to the most life-threatening injury
identified and avoids delay. Remember to act immediately
Primary survey
A=Airway + O2 + cervical spine. Approach the patient with arms ready to
immobilize
Assess the airway; jaw thrust can be used to help maintain patency. Give
100% 02 to all patients.
B=Breathing + ventilation. Check air entry with auscultation; also
auscultate
the heart; inspect, palpate and percuss the chest wall for further evidence of
injury. Check RR.
C=Circulation + haemorrhage control. Check GCS/AVPU, skin perfusion,
BP &
pulse. Control any visible haemorrhage with local pressure, and consider
possible sources of occult haemorrhage if no source identified but the patient
is shocked, ie examine abdo, pelvis, femoral.

Get 2L of warmed Ringers lactate solution/Hartmanns solution/0.9% saline


running stat via 2 separate points of venous access
D=Disability. Check GCS (if not already done), pupillary reflexes, gross
evidence of a lateralizing injury or spinal cord level.
E=Exposure. Check and maintain body temperature using rewarming
methods. Totally undress the patient, cutting all clothes off if necessary.
Adjuncts to the primary survey can add life-saving information: CXR, lateral
C Spine X-ray and pelvic X-ray .
urinary catheter to accurately assess urine output (exclude urethral injury
first);
NGT insertion, O2 sats and then ABG to accurately
assess oxygenation.
Secondary survey:
Now the patient is stabilized. More focused imaging can take
place. Includes more focused examination (eg PR exam, otoscopy) and tests
(eg
limb x-ray, full C-spine series).
797. A 44yo lady who has PCKD is concerned because her 38yo brother
has just died of an intracranial insult. She knows he was not hypertensive.
What was the most likely cause of her brothers death?
a. Subdural hematoma
b.
Subarachnoid hemorrhage
c. Cerebral infarct
d. Epidural hematoma
Answer: B. subarachnoid hemorrhage. subarachnoid hemorrhage is mainly
due to rupture of berry aneurysm ( which is associated with adult polycystic
kidney disease, ehler danlos syndrome and coarctation of aorta)
SUBARACHNOID HEMORRHAGE
Causes
85% are due to rupture of berry aneurysms (conditions associated with berry aneurysms
include adult polycystic kidney disease, Ehlers-Danlos syndrome and coarctation of the
aorta)

AV malformations
trauma
tumours

Investigations
CT: negative in 5%
lumbar puncture: done after 12 hrs (allowing time for xanthochromia to develop)
Complications
rebleeding (in 30%)
obstructive hydrocephalus (due to blood in ventricles)
vasospasm leading to cerebral ischaemia
Management
neurosurgical opinion: no clear evidence over early surgical intervention against
delayed intervention
post-operative nimodipine (e.g. 60mg / 4 hrly, if BP allows) has been shown to
reduce the severity of neurological deficits but doesn't reduce rebleeding.

798. A 36yo male dx with glioblastoma since last 5m has cerebral edema
and is on dexamethasone. He has diarrhea and vomiting for the last 3 days.
He has been suffering from repeated falls today. What could be the possible
cause for his falls?
a.
Adrenal insufficiency
b. Dehydration
c. Dexamethasone therapy
d. Raised ICP
Answer: A. Adrenal insufficiency. diarrhea, vomiting, falls (d/t postural
hypotension) all point towards diagnosis. It is because of the steroid
resistance and these exogenous steroids would have caused atrophy of
adrenal glands.
Adrenal insufficiency is a condition in which there is destruction of the adrenal cortex and
subsequent reduction in the output of adrenal hormones, ie glucocorticoids (cortisol) and/or
mineralocorticoids (aldosterone). There are two types of adrenal insufficiency:
Primary insufficiency (Addison's disease) - there is an inability of the adrenal glands to
produce enough steroid hormones. The most common cause for this in the developed world
is autoimmune disease.
Secondary insufficiency - there is inadequate pituitary or hypothalamic stimulation of the
adrenal glands.
Presentation:
Often diagnosed late: lean, tanned, tired, tearful weakness, anorexia,dizzy, faints, fl u-like
myalgias/arthralgias. Mood: depression, psychosis, low self esteem.
GI: nausea/vomiting, abdominal pain, diarrhoea/constipation. Think of Addisons in all with
unexplained abdominal pain or vomiting. Pigmented palmar creases & buccal mucosa

(ACTH; cross-reacts with melanin receptors). Postural hypotension.Vitiligo. Signs of critical


deterioration: Shock (BP, tachycardia),T, coma.
investigations:
In a patient with suspected Addison's disease the definite investigation is a ACTH stimulation
test (short Synacthen test).If a ACTH stimulation test is not readily available (e.g. in primary
care) then sending a 9 am serum cortisol can be useful:
> 500 nmol/l makes Addison's very unlikely
< 100 nmol/l is definitely abnormal
100-500 nmol/l should prompt a ACTH stimulation test to be performed
Associated electrolyte abnormalities are seen in around one-third of undiagnosed patients:

hyperkalaemia
hyponatraemia
hypoglycaemia
metabolic acidosis

management:
Patients who have Addison's disease are usually given both glucocorticoid and
mineralocorticoid replacement therapy.
This usually means that patients take a combination of:

hydrocortisone: usually given in 2 or 3 divided doses. Patients typically require 20-30 mg


per day, with the majority given in the morning dose

fludrocortisone

799. A 2yo child is brought by his mother. The mother had hearing
impairment in her early childhood and is now concerned about the child.
What inv would you do?
a.
Audiometry
b. Distraction testing
c. Scratch test
d. Tuning fork
Answer: A. Audiometry. Family history of deafness so audiometry should
be done.
800. An 8yo child presents with recurrent abdominal pain, occasional
headaches but maintains a good appetite. Exam: normal. CBC, BUE, etc are
normal. What would you do for her next?
a. US abdomen
b. CT head
c.
Reassure
d. Analgesics

Answer: C reassure. This is the case of abdominal migraine. Reassurance is the most
important treatment in it.
Abdominal migraine. This presents typically as recurrent bouts of generalised abdominal
pain associated with nausea and vomiting but no headache, followed by sleep and recovery.
Typical migraines may occur separately. pain is mostly periumbilical and lasts 1 to 4 hrs
sometimes interfering with normal physical activity and routine.
Diagnosis: clinical. assess triggers like sleep pattern, missing meals etc.
children with abdominal migraine have tendency to develop migraine when they grow up.
Treatment: Reassurance is the most important treatment.
For headaches in children paracetamol can be given.

Muniba / Samar: 814-845


Chishti: 856-865
Hamza: 866-875
Wajiha: 876-950
801. A 78yo gentleman suddenly collapsed. His HR=120bpm,
BP=70/40mmHg. Exam: pulsatile mass in abdomen. What is the most
appropriate dx?
a.
Aortic aneurysm
b. Mesenteric cyst
c. Umbilical hernia
Key is A aortic aneurysm.
Clincher: Pulsatile mass in abdomen is most likely to be an aortic aneurysm.
Its actually a burst aortic aneurysm which can be predicted by collapsed,
tachycardic patient presenting with hypotension.
ABDOMINAL AORTIC ANEURYSM:

An aneurysm is a permanent and irreversible dilatation of a blood vessel by at least 50% of


the normal expected diameter.
An aneurysm is caused by degradation of the elastic lamellae, a leukocytic infiltrate,
enhanced proteolysis and smooth muscle cell loss. The dilatation affects all three layers of
the arterial wall. A false aneurysm (pseudoaneurysm) is caused by blood leaking through the
arterial wall but contained by the adventitia or surrounding perivascular tissue.
The 'normal' diameter of the abdominal aorta is approximately 2 cm; it increases with age.
An abdominal aneurysm is usually defined as an aortic diameter of 3 cm or greater. Most
abdominal aortic aneurysms (AAAs) arise from below the level of the renal arteries.

Presentation
Unruptured AAA

Most patients with unruptured AAA have no symptoms.


AAA may be an incidental finding on clinical examination, or on scans ultrasound, CT or MRI. It may sometimes be visible on a plain X-ray film.
Possible symptoms and signs are:
o Pain in the back, abdomen, loin or groin:
This may be due to pressure on nearby structures. Back
pain may be due to erosion of the vertebral bodies.
NB: severe lumbar pain of recent onset may indicate
impending rupture.
o The patient or doctor may find a pulsatile abdominal swelling
o Distal embolisation may produce features of limb ischaemia. The
appearance of micro-embolic lower limb infarcts in a patient with
easily palpable pedal pulses suggests a popliteal or abdominal
aneurysm.
o Ureterohydronephrosis can also occur with AAA.
o Inflammation or retroperitoneal fibrosis can complicate AAA and
may cause symptoms - eg, back pain, weight loss and symptoms
related to entrapment of adjacent structures.

Ruptured AAA

NB: the diagnosis may not be obvious. Ruptured AAA should be considered in
any patient with hypotension and atypical abdominal symptoms. Similarly,
abdominal pain in a patient with a known aneurysm or pulsatile mass must be
considered as a possible ruptured or rapidly expanding aneurysm and treated
accordingly.
Ruptured AAA may present with:
o Pain in the abdomen, back or loin - the pain may be sudden and
severe.
o Syncope, shock or collapse:

The degree of shock varies according to the site of


rupture and whether it is contained - eg, rupture into the
peritoneal cavity is usually dramatic, with death before
reaching hospital; whereas rupture into the
retroperitoneal space may be contained initially by a
temporary seal forming.

802. A woman prv in good health presents with sudden onset of severe
occipital headache and vomiting. Her only physical sign on exam is a stiff
neck. Choose the most likely dx.

a.
b.
c.
d.
e.

Subarachnoid hemorrhage
Subdural hematoma
Cerebellar hemorrhage
Migraine
Cerebral embolus

Key is A
Any Severe headache of Sudden onset in previously well patient is
Subarachnoid hemorrhage unless proven otherwise.
It may also be described as worst headache of my life.
Subdural hematoma usually presents with history of falls in an alcoholic or
elderly patient and may also shows signs of confusion.
Cerebellar hemorrhage may present with signs and symptoms of cerebellar
lesion like lack of balance , nystagmus e.t.c.
Migraine presents with unilateral severe throbbing pain with photophobia
and phonophobia. It lasts for about 4-72 hours and may occur along with
nausea and vomiting and aura.
Cerebral embolus rarely cause headache and presents with focal neurological
deficits.
SUBARACHNOID HEMORRHAGE:

Presentation
The most characteristic feature is a sudden explosive headache. This may last a few
seconds or even a fraction of a second.

Sudden explosive headache may be the only symptom in a third of patients.


Of patients who present with a sudden explosive headache as the only
symptoms, around 10% have SAH.
It is difficult to suspect SAH without sudden headache, but with seizure or
confusional state - there are many other more common causes of these
presentations, but SAH should be on the list of differential diagnoses.
Vomiting may occur; although this does not distinguish it from other causes of
headache.
Seizures, occur in about 7%. When they do, they are highly suggestive of a
haemorrhage.
1-2% of patients with SAH present with an acute confusional state.
Neck stiffness and other signs of meningism may be present, although it usually
presents around six hours after onset of SAH.
Trauma may be confused with SAH if trauma has also occurred. Patients may also
cause a motor vehicle accident as a result of SAH. SAH needs to be on the
differential diagnostic list for patients with altered consciousness, headache or
seizure after trauma, particularly if there is disproportionate headache or neck
stiffness.
SAH following head injury causes headache, decreased level of consciousness and
hemiparesis. SAH is a frequent occurrence in traumatic brain injury, and carries a
poor prognosis if it is associated with deterioration in the level of consciousness.

Investigations
CT scanning

If SAH is suspected, CT scanning (without contrast) is the first line in


investigation because of the characteristically hyperdense appearance of blood
in the basal cisterns.

Lumbar puncture

CT is negative in 2% of patients with SAH.


If the CT scan is negative but the history is suggestive, lumbar puncture should
be undertaken, providing the scan shows no contra-indications. Around 3% of
patients with a negative CT scan will prove, on lumbar puncture, to have had an
SAH.
Lumbar puncture to remove a CSF sample from the lumbar sac should ideally
take place over 12 hours after the onset of the headache because if there are
red cells in the CSF, sufficient lysis will have taken place during that time for
bilirubin and oxyhaemoglobin to have formed

Angiography

Further investigation should follow immediately acute SAH is confirmed.


After an SAH is confirmed, its origin needs to be determined. If the bleeding is
likely to have originated from an aneurysm, the choice is between cerebral
angiography (injecting radiocontrast through a catheter to the brain arteries) and
CT angiography (visualising blood vessels with radiocontrast on a CT scan) to
identify aneurysms. Catheter angiography also offers the possibility of coiling an
aneurysm.

803. A 34yo housemaid presents with headaches in the back of her head
for several days and pain on flexing her neck. What is the most likely cause?
a. Subdural hemorrhage
b.
Cervical spondylosis
c. Subarachnoid hemorrhage
d. Meningitis
e. Cluster headache
Key is B
Clincher is 34 year old housemaid which shows that patient has to work with
flexed neck for longer time of the day.
pain on flexion of neck shows nerve root irritation due to
compression,herniation or osteophytic outgrowth.
Subarachnoid hemorrhage is usually sudden in onset and does not prevail for
several days.
Migraine is a unilateral throbbing pain and is not associated with neck
flexion
Cluster headache presents with unilateral headache with unilateral
autonomic signs like rhinorrhea, lacrimation and ptosis
CERVICAL SPONDYLOSIS:
Cervical spondylosis is chronic cervical disc degeneration with herniation of disc material,
calcification and osteophytic outgrowths. As with simple back pain, it is multifactorial in origin,
reflecting poor posture, muscle strain, sporting and occupational activities as well as
psychological factors.
Cervical spondylosis undoubtedly contributes to this burden, but may also cause:
Radiculopathy due to compression, stretching or angulation of the cervical nerve
roots.
Myelopathy due to compression, compromised blood supply or recurring minor
trauma to the cord.

Symptoms

Cervical pain worsened by movement.


Referred pain (occiput, between the shoulder blades, upper limbs).
Retro-orbital or temporal pain (from C1 to C2).
Cervical stiffness - reversible or irreversible.
Vague numbness, tingling or weakness in upper limbs.
Poor balance.

Signs

Limited range of movement (forward flexion, backward extension, lateral flexion,


and rotation to both sides).
Minor neurological changes like inverted supinator jerks (unless complicated by
myelopathy or radiculopathy).
Poorly localised tenderness.

Investigations
Most patients do not need further investigation and the diagnosis is made on clinical grounds
alone.

Plain X-ray of cervical spine showing formation of osteophytes, narrowing of disc


spaces with encroachment of intervertebral foraminae. This is not diagnostic as
these findings are common in normal middle-aged patients.
Patients with neurological abnormality will need magnetic resonance imaging
(MRI) of the cervical spine at an early stage, particularly if they have progressive
myelopathy, radiculopathy or intractable pain.

804. A 40yo man complains of thirst and lethargy. His


BP=140/90mmHg, corrected Ca2+=3.7mmol/l. What is the most appropriate
management at this stage?
a.
IV fluids
b. Prednisolone
c. IV hydrocortisone
d. Calcium prep
KEY is A
This is Hypercalcemia since CA2+ levels are increased (normal range 2.252.5mmol/l).
Increasing the circulating volume with 0.9% saline, helping to increase the urinary
output of calcium

Presentation
Presentation of Hypercalcaemia
At levels <2.8 mmol/L

At levels <3.5
mmol/L

At levels >3.5 mmol/L

All of the previous plus:


Abdominal pain
Vomiting
Dehydration

Muscle
Lethargy
weakness
Cardiac
Constipation
arrhythmias,

Anorexia and
shortened QT

nausea
interval
Fatigue
Coma
Pancreatitis
If the hypercalcaemia is long-standing, calcium may be deposited in soft tissues or may
result in stone formation - eg, nephrocalcinosis, nephrolithiasis or chondrocalcinosis.

Polyuria and
polydipsia
Dyspepsia - due to
calcium-regulated
release of gastrin
Depression
Mild cognitive
impairment

All of the previous


plus:

Acute hypercalcaemia[11]
Treatment should be initiated in hospital on the advice of a specialist and should include:

Increasing the circulating volume with 0.9% saline, helping to increase the
urinary output of calcium.
A loop diuretic such as furosemide. This is occasionally used where there is fluid
overload but it does not reduce serum calcium .
After rehydration, bisphosphonates (which act by reducing bone turnover)
should be administered intravenously. Pamidronate and zolendronic acid are
commonly used. Salmon calcitonin may also be given. It has fewer side-effects
than bisphosphonates but is less effective in reducing hypercalcaemia.[12][13]
Glucocorticoids are useful for hypercalcaemia due to vitamin D toxicity,
sarcoidosis and lymphoma.[13]
Gallium was identified as a useful drug when it was found that patients with
malignancy having gallium scans did not develop hypercalcaemia. It may be
given intravenously to patients with malignant hypercalcaemia who do not
respond to bisphosphonates.[14]
Cinacalcet hydrochloride is a calcimimetic (= mimicking the action of calcium)
agent that effectively reduces parathyroid levels in patients with secondary
hyperparathyroidism.[12]
A new experimental approach to malignancy-associated hypercalcaemia
involves the blockade of receptor activator of nuclear factor kappa-B ligand,
usually abbreviated as RANKL. RANKL is a key element in the differentiation,
function and survival of osteoclasts, which plays an essential role in removing
calcium ions from the bone in response to PTH stimulation.[15] Denosumab, a
human monoclonal antibody that acts in this manner, is licensed for the

prevention of osteoporotic fractures but is also useful for patients with persistent
or relapsed hypercalcaemia of malignancy.[16]
Haemodialysis or peritoneal dialysis may be relevant in patients with severe
hypercalcaemia secondary to renal failure.

805. A 75yo man on digoxin develops weakness in the right upper and
lower limbs which resolves within a few hours. What is the most definitive
inv for this condition?
a. Carotid Doppler
b.
Angiography
c. CT head
d. Digoxin level
KEY is B its a wrong key.
Correct Key is A.
This is a case of TIA since focal neurological symptoms vanished within
few hours of onset.
Also supported by use of digoxin which means patient already had some
arrhythmias or heart failure or there can be digoxin induced heart failure due
to which the chance of thromboembolism were increased and TIA occurred.
One of the major complication of these emboli is Carotid artery stenosis
which leads to decreased perfusion of the cerebral cortex and presentation
with stroke or TIA.
Carotid Doppler is the most definitive investigation for this condition.
806. A 35yo man presents with balance problems, headache, SNHL and
loss of coreneal reflex on the left side. What is the most definitive inv?
a. CT scan of internal auditory meatus
b. Nuclear imagine of the brain
c. MRI of internal auditory meatus
d.
MRI brain
KEY is D.
Diagnosis- Acoustic Neuroma.
Clincher: balance problem, headache and unilateral SNHL presenting origin
from Cerbellopontine angle.
Loss of Corneal reflex shows involvment of trigeminal nerve.

Investigation of Choice for Diagnosing Acoustic Neuroma is MRI scan.


ACOUSTIC NEUROMA:
Acoustic neuromas are tumours of the vestibulocochlear nerve (eighth cranial nerve), arising
from the Schwann cells of the nerve sheath. Most arise from the vestibular portion and only a
few arise from the cochlear (auditory) division

Presentation
Any unilateral sensorineural hearing loss should be considered as caused by an acoustic
neuroma until proven otherwise.

Consider the diagnosis of acoustic neuroma in patients with:

Unilateral or asymmetrical hearing loss or tinnitus, whether progressive or acute in


onset.
Impaired facial sensation.

As the tumour spreads, there is an increase in hearing loss and disequilibrium, and
symptoms due to compression of other structures may occur:

Facial pain or numbness due to involvement of the trigeminal nerve.


Facial weakness is uncommon despite the tumour pressing on the facial nerve.
Earache.
Ataxia due to cerebellar compression.
Severe brainstem compression can produce hydrocephalus with visual loss and
persistent headache and even decreased level of consciousness.
Balance problems without other explanation.

807. A 52yo man has a painful, red, photophobic right eye with slightly
blurred vision and watering for 3 days. He has had no such episodes in the
past. On slit lamp examination there are cells and flare in the ant chamber
and pupil is sluggish to react. What is the single most appropriate clinical
dx?

a. Acute close-angle glaucoma


b. Acute conjunctivitis
c. Acute dacrocystitis
d.
Acute iritis
e. Corneal foreign body
KEY is B
This is a clear Picture of Acute Iritis. Iritis forms the part of anterior Uveitis.
Anterior uveitis is the term for inflammation which affects the eye's front (anterior) part of
the uveal tract. This can include the iris (iritis) or the iris and the ciliary body (iridocyclitis). It
is the most common type of uveitis
SYMPTOMS AND SIGNS:
this usually affects one eye. The common symptoms are eye pain (usually felt as a dull ache
in and around the eye), redness of your eye, and photophobia (which means you do not like
bright light). You may develop blurred vision or even some visual loss (usually temporary).
You may develop headaches and notice that the pupil of the affected eye may change shape
slightly. The pupil may not react to light (normally becomes smaller) or it may lose its smooth
round shape. Your eye may become watery. The symptoms tend to develop over a few
hours or days.
FINDINGS on SLIT LAMP EXAMINATION:

The diagnosis of iritis is confirmed by examining the eye with a slit lamp (a
special microscope designed for eye exams). Your ophthalmologist can see cells
(whiteblood cells) and flare (particles of protein) in the fluid that is produced in the
eye.
Two other physical exam findings aid your eye doctor in diagnosing iritis. They
include:

Topical anesthetics do not relieve the pain associated with iritis.


Shining light in the normal, unaffected eye causes pain in the affected eye
if iritis is present. This is because shining light in one eye causes both
pupils to constrict. Movement of the affected iris causes pain

808. A 55yo lady with swelling on the abdomen below the umbilicus on
the right side. What is the single most appropriate lymph node?
a.
b.
c.
d.

External iliac LN
Pre-aortic LN
Aortic LN
Inguinal LN

e.
f.
g.
h.

Iliac LN
Submental LN
Submandibular LN
Deep cervical LN

KEY is D
swelling below the umbilicus in a 55 year old lady is more likely to be of
ovarian origin.

809. A 66yo male presents with painful swallowing. What is the most
likely dx?
a. Neisseria meningitides
b. Cryptococcus neoformans
c.
Candida albicans

d.
e.

Isospora belli
Mycobacterium avium

KEY is C
Candida infections are more common in old age, diabetics and in
immunocompromised
Isospora belli causes an intestinal infection mostly in HIV positive patients.
MAC can cause fevers, diarrhea, malabsorption, as well as loss of appetite and weight loss,
and can disseminate to the bone marrow

Cryptococcus mainly cause fungal meningitis and encephalitis in


immunocompromised and HIV positive.

Oesophageal candidiasis
Presentation

Dysphagia, pain on on swallowing food or fluids and/or retrosternal pain, usually


with oropharyngeal candidiasis.
This combination of symptoms is predictive of oesophageal candidiasis.
It is most often associated with treatment of haematopoietic or lymphatic
malignancies.
In HIV-positive patients, it is an AIDS-defining illness.

Diagnosis

A therapeutic trial of fluconazole for patients is useful; most patients will respond
within 7 days of treatment.
Definitive diagnosis is by endoscopy.

Management of oesophageal candidiasis

Consider admission to hospital - oesophageal candidiasis is a life-threatening


infection.
Treat for 14-21 days; the following treatment options are suggested:
o First-line treatment options:
Oral fluconazole (200-400 mg daily).
Intravenous (IV) fluconazole, an echinocandin - eg,
caspofungin, or amphotericin.[10]
o Second-line drugs are oral itraconazole oral solution, IV
posaconazole or IV voriconazole.

For AIDS patients, HAART is advised to prevent recurrence.

810. A 45yo lady complains of expressible galactorrhea, decreased libido


and amenorrhea, weight gain and easy fatigue. Her serum prolactin levels
are 1100 um/l. what is the likely cause of hyperprolactinemia?
a. Hypothyroidism
b. Stress
c. Pregnancy
d.
Prolactin secreting pituitary tumor
e. PCOS
KEY is D
Prolactin levels greater than 150-200ng/L are almost always due to a
prolactin secreting tumour.
CAUSES OF HYPERPROLACTINEMIA:
'Physiological' causes:

Pregnancy.
Puerperium.
Breast stimulation (including suckling a child).
Stress - physical (including excessive exercise) or psychological - including
venepuncture.
Non-fasting sample.
Macroprolactinaemia:
o This refers to prolactin of high molecular mass, mostly complexes of
monomeric prolactin with immunoglobulins (prolactin auto-antibody
complexes). These larger molecules have no bioactivity and a
prolonged clearance rate similar to that of immunoglobulins.
Depending on the immunoassay used, macroprolactinaemia may
account for 25% of laboratory documented hyperprolactinaemia.
Consider this cause in an asymptomatic patient with
hyperprolactinaemia and consult laboratory staff (see
'Investigations', below).

Intracranial causes:

Pituitary tumours:
o Abnormally high levels of prolactin may be caused by a prolactinsecreting pituitary tumour or by a non-secreting pituitary tumour that
prevents dopamine (prolactin release-inhibiting hormone) from
reaching normal prolactin-producing cells.
o Prolactinomas:

Microprolactinomas (the most common, approximately


90%).
Macroprolactinomas (>10 mm size - approximately
10%).
Pituitary or hypothalamic tumour compressing the
pituitary stalk, egcraniopharyngioma.
Prolactinomas occur in about 20% of patients with
multiple endocrine neoplasia type 1.[4]
Head injury (eg due to disruption of the pituitary stalk).
Brain surgery and radiotherapy.
Post-ictal - within hours of a seizure.

Endocrine and metabolic causes:

Hypothyroidism (due to increased synthesis of TRH).


Cushing's syndrome.
Chronic renal failure.
Severe liver disease.
Polycystic ovarian syndrome.
Coeliac disease (possibly).[5]

Dopamine receptor antagonists, eg domperidone, metoclopramide, neuroleptics.


Dopamine-depleting agents, eg methyldopa.
Antidepressants, e.g tricyclic antidepressants, monoamine-oxidase inhibitors,
serotonin reuptake inhibitors.
Verapamil.
Opiates.
Protease inhibitors.
Bezafibrate.
Omeprazole.
H2-receptor antagonists.
Oestrogens, anti-androgens.
Cyproheptadine.
Cocaine.

Drugs:

Other causes

Chest wall surgery or trauma.


Sarcoidosis.
Langerhan's cell histiocytosis.
Idiopathic - a diagnosis of exclusion.

811. A 42yo female had sudden severe headache and vomiting. She took
paracetamol and then collapsed. What is the most likely dx?
a.
SAH
b. Viral encephalitis
c. Meningitis

d.

Anaphylaxis

Key is A
any sudden severe headache is subarachnoid hemorrhage unless proven
otherwise.
812. Parents of a 3m baby are worried about cot death. What advice would
you give?
a. Lay on the back with feet towards head end
b.
Lay on the back with feet towards feet end
c. Lay on side
d. Lay on stomach
KEY is A
Cot death is more properly known as sudden infant death syndrome (SIDS). It is the term
used to describe the sudden death of a baby in its sleep where no cause or reason can be
found.
Research has shown that the risk of cot death can be reduced. The things you can do fall
into four main categories:

Sleeping - learn the best way for your baby to sleep.


Smoking - create a smoke-free home for your baby.
Symptoms of illness - see a doctor if your baby appears unwell.
Consider breast-feeding.

SLEEPING
The best way for your baby to sleep is as follows. Make sure that any childminders or
babysitters know this too.

Lay your baby on their back to sleep


In other words, do not lay him or her on their front or side. Cot death is less common in
babies who sleep on their back. It is important to put babies on their backs for daytime naps
and any other times of sleep, in addition to the longer night-time sleep.

813. A child born at 36wks developed difficulty in breathing with


intercoastal recession and nasal flaring. His temp is normal but his mother
had PROM 48h ago. What is the most likely inv that will lead to tx?
a. Blood culture
b.
CXR
c. Stool culture
d. Sputum culture
Key is B
This is Infant Respiratory Distress Syndrome.
since the child is afebrile so this is not Bronchiolitis plus the symptoms
appeared right after the delivery.
IRDS:
Infant respiratory distress syndrome (IRDS) is caused by the inadequate production of
surfactant in the lungs

Risk factors

Premature delivery.
Male infants.

Infants delivered via caesarean section without maternal labour.


Hypothermia.
Perinatal asphyxia.
Maternal diabetes.
Multiple pregnancy.
Family history of IRDS.

Secondary surfactant deficiency may occur in infants as a result of:[1]

Intrapartum asphyxia.
Pulmonary infection - eg, group B beta-haemolytic streptococcal pneumonia.
Pulmonary haemorrhage.
Meconium aspiration pneumonia.
Oxygen toxicity along with pressure or volume trauma to the lungs.
Congenital diaphragmatic hernia and pulmonary hypoplasia.

Presentation

Usually preterm delivery.


Presents very soon after birth with respiratory distress: tachypnoea, expiratory
grunting, subcostal and intercostal retractions, diminished breath sounds,
cyanosis and nasal flaring.

Investigations

Blood gases: respiratory and metabolic acidosis along with hypoxia. Metabolic
acidosis results from poor tissue perfusion.
Pulse oximetry is used as a non-invasive tool to monitor oxygen saturation,
which should be maintained at 85-93%.
CXR.
Monitor FBC, electrolytes, glucose, renal and liver function.
Echocardiogram: diagnosing patent ductus arteriosus (PDA), determine the
direction and degree of shunting, making the diagnosis of pulmonary
hypertension and excluding structural heart disease.
Cultures to rule out sepsis.
May rapidly progress to fatigue, apnoea and hypoxia.

814. A 68yo man has had increasing dysphagia for solid food for 3m and
has lost 5kgs in weight. What single inv is most likely to lead to a def dx?
a. Barium swallow
b. CXR
c. CT chest

d.
Endoscopy and biopsy
e. Video-fluoroscopy
815. A 24yo male was trying to move his wardrobe but it fell on his
thigh. It was there for a very long time before someone was able to remove
it. When he was seen in ED he had casts in his urine but no RBCs. Other inv
showed hypocalcemia and high serum creatinine. What is the cause for his
renal failure?
a. Acetylcholine
b.
Myoglobin
c. Myotroponin
d. Acetyl acetate
Answer: B
Rhabdomyolysis can be defined as a clinical syndrome associated with the
breakdown of skeletal muscle fibres and myocyte cell membranes, leading to
release of muscle contents into the circulation, resulting in multiple
complications, including hyperkalaemia. It is a medical emergency and can
lead to cardiac arrest if not promptly treated. Myoglobinuria is an early
feature of rhabdomyolysis, but it may be cleared within a few hours despite
other sequelae continuing.
Pathophysiology:
Myocyte function under normal circumstances is maintained by adenosine
triphosphate-dependent channels which ensure effective cell ion levels and
play a role in calcium efflux from myocytes.[1] Damage to the myocyte
membrane (eg, due to trauma or lack of energy for the cell membrane
channels) causes an increase in the amount of calcium in the cell, which
leads to apoptosis through various proteolytic enzymes.[1][2] This leads to
the muscle necrosis and the release of various substances into the circulation
- eg, myoglobin, potassium, phosphate, creatine kinase (CK) and urate.[3]
Myoglobin is a skeletal muscle protein involved in metabolism and
myoglobinaemia usually occurs before a rise in CK in rhabdomyolysis. Any
myoglobin that reaches the circulation will be filtered by the kidneys and
can lead to acute kidney injury through either direct toxicity or precipitation,
or both.[3] This process is facilitated by an acidotic environment and
hypovolaemia.[1] Myoglobin can appear in the urine (myoglobinuria)
causing 'tea-coloured' urine with a positive urine dipstick for blood. The
latter can cause confusion with haematuria and haemoglobinuria.
816. Anatomical structure to be pierced during surgery midline port
during gallstone removal.

a.
b.
c.
d.
e.
f.
g.
h.

External iliac muscle


Cricoid cartilage
Linea alba
Rectus sheath muscle
Duramater
1 tracheal cartilage
Conjoined tendon
Intercostal muscles
st

Answer: C
In lap. Choly open technique incision is made for entering the port either in
infraumblical or transumblical region which leads to piercing of linea Alba.
817. A 48yo man has continuous ant chest pain which is worse on
inspiration and has temp of 37.5C since 4wks after a MI. His ESR=45mm/h.
What is the single most likely explanation for the abnormal inv?
a. Acute pericarditis
b. Cardiac tamponade
c. Atrial thrombus
d. Left ventricular aneurysm
e.
Dressler syndrome
Answer: E
Dresslers syndrome was first described in 1956. It is characterised by
pleuritic chest pain, low-grade fever and pericarditis (autopsy shows
localised fibrinous pericarditis), which may be accompanied by pericardial
effusion. It tends to follow a benign clinical course. It is thought to be
immune-mediated (antiheart antibodies may be present). The reported
incidence has been declining in recent years.[1]
Risk factors
If a person has had a previous episode, it is more likely to recur. It seems
more likely to occur after a large infarct.
Presentation
It usually presents two to five weeks after the initial episode, with pain and
fever that may suggest further infarction.
The pain is the main symptom, often in the left shoulder, often pleuritic, and
worse on lying down.
There may be malaise, fever and dyspnoea.

Rarely, it may cause cardiac tamponade or acute pneumonitis.


A pericardial friction rub may be heard. The typical sound of pericarditis is
described as like the sound of boots walking over fresh snow.
Differential diagnosis
The pain may initially suggest a further episode of angina or myocardial
infarction. Pleuritic chest pain may also suggest pneumonia or pulmonary
embolism.
Investigations
FBC will show leukocytosis, sometimes with eosinophilia and an elevated
ESR.
Serology may show heart autoantibodies.
ECG may show ST elevation in most leads without reciprocal ST
depression, typical of pericardial effusion.
Echocardiography shows pericardial effusion.
MRI scan may show an effusion and, more recently, has been shown to
reveal pericardial involvement.[8]
CXR shows pleural effusions in 83%, parenchymal opacities in 74%, and an
enlarged cardiac silhouette in 49%.
Management
Aspirin may be given in large doses.
Other non-steroidal anti-inflammatory drugs (NSAIDs) or corticosteroids
may be used, especially if there are severe and recurrent symptoms.
Steroids are particularly valuable where severe symptoms have required
pericardiocentesis, and when infection has been excluded.
In resistant or recurrent cases, colchicine may be useful.
If there is significant pericardial effusion then pericardiocentesis, involving
aspiration of the fluid, may be required to relieve the constriction on the
heart.
dressler occurs after > 2 weeks @ least. before that it can be pericarditis. Dressler & Aneurysm occurs after
2 weeks. wherever there is pleurisy after 2 weeks of MI, most probably Dressler. Autoimmune kind of
disease. only steroids will help.

818. An 8yo child swallowed 12 tablets of paracetamol 4h ago. Serum


paracetamol levels when tested were at critical level. What would you do
next?

a. Activated charcoal
b.
IV N-acetylcysteine
c. Gastric lavage
d. Observation only
Answer: B
NAC is believed to work by a number of protective mechanisms. It acts as a
precursor for glutathione, promoting normal conjugation of any remaining
paracetamol, and also supplies thiols that function as antioxidants. It is
virtually 100% effective in preventing liver damage when given within eight
hours of ingestion.[3] After eight hours, efficacy decreases sharply.
The initial dose of acetylcysteine should be given as an infusion over 60
minutes. This should reduce the number of dose-related adverse effects. The
infusion should be in 5% glucose, with 0.9% sodium chloride as an
alternative. There are now NO specific contra-indications to acetylcysteine
use. Even if there is a previously reported reaction, the benefits of treatment
outweigh the risks.
Specific weight-related dosing tables are available to guide the health
professional.[5] Children receive the same doses and treatment as adults, but
with a reduced quantity of intravenous fluid, as fluid overload is a potential
risk.
A full treatment course comprises three consecutive doses, administered
sequentially, with no break between infusions.
819. A pt dx with DVT is taking warfarin. What is his cut off INR limit?
a. <1
b. 1-2
c.
2-3
d. 3-4
Answer: C
Target INR range and duration of treatment
In most situations the INR target is 2.5 (target range 2.0 3.0). This range is
appropriate for the prophylaxis or treatment of venous thromboembolism
and reduction of the risk of systemic embolism for people with atrial
fibrillation and valvular heart disease.5 In some situations higher ranges are
more appropriate. The target INR may vary depending on individual clinical
situations. The target INR for mechanical prosthetic valves is dependent on

the type of valve replacement used.6


The duration of warfarin therapy for a provoked DVT or PE is 13 weeks. For
unprovoked DVT or PE the duration again is 13 weeks, but for individual
patients within their clinical context, the indefinite use of warfarin may be
appropriate.5 For atrial fibrillation, cardiomyopathy and valvular heart
disease (selected cases) an indefinite period of warfarin treatment is
recommended.
820. Inserting a drain in the mid-axillary line. What is the single most
appropriate anatomical structure?
a. External iliac muscle
b. Linea alba
c. Rectus sheath muscle
d. Conjoined tendon
e. Intercostal muscles
Answer: E
821. A 29yo man presents with hx of cough, weight loss and night
sweats. Exam: pansystolic murmur. What is the most appropriate dx of
underlying cause?
a. Malaria
b. HSP
c.
HIV
d. Dengue fever
Answer: C
Patients with HIV are likely to have heart valve disease (tricuspid mc in drug
abusers) leading to pansystolic murmur. All other options do not have direct
relation to valvular disease.
822. A 15yo boy presents with recurrent breathlessness and wheeze
especially after exercise. What is the most diagnostic inv?
a. CXR
b.
Lung function test
c. PEFR
d. CT scan
Answer: B

What is wheezing?
Wheezing is a whistling sound that occurs during breathing when the
airways are narrowed. Commonly the sound is more prominent when you
breathe out than when you breathe in (although not always). The sound is
caused by air that is forced through airways that are narrower than normal.
Narrowed airways can be due to:
Bronchospasm. This means that the muscle within the lining of the airways
contracts. This has an effect of narrowing (constricting) the airways.
Swelling of the lining of the airways.
A lot of secretions (mucus, etc) in the airways.
An inhaled object. For example, if you inhale a peanut (which is not
uncommon in children).
An abnormal growth in the airway, such as a tumour (cancer).
Wheeze with breathing difficulty
Breathing difficulty can mean breathlessness, fast breathing or difficulty
getting your breath. If you have any breathing difficulty and do not already
have clear instruction as to what to do then see a doctor immediately. (Some
people with asthma have pre-arranged action plans as agreed with their
doctor. For example, if they have wheeze and become breathless then they
have a plan of which medication to take. However, if you do not have such a
plan and become breathless, you need to see a doctor immediately. In
particular, children with any breathing difficulty need to be seen as soon as
possible by a doctor.)
What are the causes of wheezing?
There are many possible causes. Below is a brief overview of the more
common and important causes.
Asthma
Asthma is a condition that affects the smaller airways (bronchioles). From
time to time the airways narrow (constrict) due to bronchospasm (described
earlier). Asthma can start at any age, but it most commonly starts in
childhood. At least 1 in 10 children, and 1 in 20 adults, have asthma. The
common symptoms are cough and wheeze. You may also become
breathless, and develop a feeling of chest tightness. Symptoms can range
from mild to severe between different people, and at different times in the
same person. Each episode of symptoms may last just an hour or so, or
persist for days or weeks unless treated.

Chronic obstructive pulmonary disease (COPD)


COPD is a general term which includes the conditions chronic bronchitis
and emphysema. Chronic means persistent. Bronchitis is inflammation of
the airways of the lungs (bronchi). Emphysema is damage to the smaller
airways and air sacs (alveoli) of the lungs. Pulmonary means 'affecting the
lungs'. Chronic bronchitis and emphysema commonly occur together. The
term COPD is used to describe airflow obstruction due to chronic bronchitis,
emphysema, or both. The main cause of COPD is smoking and it mainly
affects older people.
Bronchiectasis
Bronchiectasis is an abnormal widening of one or more airways. Extra
mucus is made in the abnormal airways, which is prone to infection. The
main symptom is a cough which produces a lot of sputum. You may also
cough up some blood from the abnormal inflamed airway and become
wheezy. There are various underlying causes of bronchiectasis. For
example, a previous severe lung infection, some inherited conditions, and
some other conditions that can damage part of an airway.
Bronchiolitis
Bronchiolitis is an infection of the small airways of the lung (the
bronchioles). It is a common condition of babies. Most affected babies are
not seriously ill, and make a full recovery. Sometimes it becomes more
serious and hospital care may be needed. It is usually caused by a virus
called the respiratory syncytial virus (RSV). Bronchiolitis in the UK usually
occurs in the winter months (November to March).
Other infections
An infection anywhere in the respiratory tract may trigger wheezing. For
example, acute bronchitis (infection of the bronchi) is common in adults and
may cause some wheezing in addition to other symptoms. A viral respiratory
infection is a common cause in young children.
823. A 34yo man was walking along the countryside when an insect bit
him. After which he started to complain of an annular rash spreading
upwards.
a.

Penicillin PO

b.
Doxycycline PO
c. Flucloxacillin PO
d. Gentamicin PO
e. Ciprofloxacin PO
f.
Antihistamine PO
g. Antihistamine IV
h. Corticosteroid IV
i.
Corticosteroid IM
j.
Adrenaline IM
k. Adrenaline IV
l.
Atropine IV
m. Reassurance
Answer: B
Lyme disease can be contracted from a deer tick (Ixodes) bite containing
Borrelia species of bacteria. Erythema migrans is the pathognomonic skin
finding associated with Lyme disease and is often described as a bull's-eye
or target rash. It typically is an annular, erythematous plaque with central
clearing. Lyme disease can be associated with myalgias and arthralgias,
fever, anorexia and nausea, fatigue, and regional lymphadenopathy, but the
rash may be the only finding at presentation.
Lesions are usually 5 to 68 cm, although they can vary in size.1 They appear
three to 30 days after the tick bite, but most commonly within seven to 14
days. The classic lesion occurs in approximately 80% of cases.2 Lyme
disease has been reported in all 50 states but is endemic in the Northeast and
in parts of Minnesota, Wisconsin, and northern California.3 Most cases
occur from May to September when the Ixodes tick is in the nymph stage.
There are three distinct clinical stages of Lyme disease. The early localized
stage (three to 30 days after tick exposure) includes the influenza-like
symptoms of fever, fatigue, arthralgias, and myalgias. The early
disseminated stage (days to weeks after tick exposure) includes multiple
lesions, neurologic symptoms (palsies, radiculopathy, or peripheral
neuropathy), or cardiac symptoms (myocarditis and varying degrees of
atrioventricular block). The late stage (months to years after tick exposure)
includes arthritis, primarily affecting the knee, and possible cognitive
disturbances. Treatment should be initiated promptly to avoid progression to
late stages of the disease.
Erythema multiforme is a hypersensitivity reaction to medication use or an

infection, such as herpes simplex virus infection.4,5 It typically manifests as


papules or plaques with erythematous borders. The target or iris lesions
typically appear on the palms, soles, elbows, or knees.
Granuloma annulare is a benign, self-limited, annular eruption that does not
require treatment. It presents as skin-colored plaques or papules on distal
portions of the extremities, specifically the hands, wrists, and feet. It is
idiopathic and occurs in adults and children, although it is most common
between 40 and 50 years of age. It is more common in women than in men.6
Nummular eczema is an idiopathic papulovesicular dermatitis commonly
associated with asthma and atopic dermatitis. It typically manifests as coinshaped lesions and is most prominent in cold or dry months.
Rheumatic fever is an inflammatory disease that can develop after group A
streptococcal infection and is associated with erythema marginatum. This
annular rash is typically slightly elevated, mildly erythematous, and
nonpruritic, and is primarily found on extensor surfaces of extremities,
sparing the face.
Antibiotics for lyme disease
Lyme disease is curable with several types of antiboitics including common
antibiotics like doxycycline, amoxicillin and azitromycin, which can be
taken orally. Sometimes the antibiotic is administered intravenous (IV),
particularly when the drug of choice is ceftriaxone (Rocephin), which cannot
be taken orally.
Other antibiotics used include: minocycline, tetracycline, cefuroxim,
claritromycin. Some doctors also experiment with other types of antibiotics
or combinations of antibiotics. Many of those treatment have not been
studied, though.

824. A 60yo man was brought in by his wife complaining of ataxia,


urinary incontinence and erectile dysfunction. He also complains of rigidity
and slowing of movement with a pill rolling tremor of the hands. What is the
likely dx?

a.
b.
c.
d.

Parkinsons disease
Idiopathic parkinsons disease
Shy-drager syndrome
Huntingtons disease

KEY is C
SHY DRAGER SYNDROME:
Shy-Drager Syndrome (Multiple System Atrophy) (SDS)(MSA) is a a rare
degenerative condition resulting from degeneration of certain nerve cells in the
brain and spinal cord. Body functions controlled by these areas of the brain and
spinal cord do not function normally in sufferers. These functions include the
autonomic or involuntary nervous system (which controls blood pressure, heart
rate, and bladder function) and the motor system (which controls balance and
muscle movement).
Depending upon which part of the brain is affected first, MSA may appear in
different ways. Sometimes it presents with low blood pressure on standing,
urinary bladder problems, or difficulties with balance and movement that
resemble Parkinson's disease. Early symptoms often include impotence and
urinary incontinence. Most patients complain of severe constipation and later
develop rectal incontinence.
Orthostatic hypotension an excessive drop in blood pressure when the patient
stands up causing light-headedness or dizziness - is a universal feature of SDS.
It is worse after meals and early in the day. When orthostatic hypotension
becomes severe (i.e. blood pressure of less than 70/40) blackouts may result.
When the patient falls to a prone position, the blood pressure returns toward
normal and the patient will regain consciousness.
Many patients also complain of dry mouth and dry skin, and because of
abnormal sweating have problems with relation of body temperature. The
hands and feet will be cold.

Major features supporting diagnosis of probable MSA


System

Autonomic

Feature

Severe (symptomatic or otherwise)


orthostatic hypotension. Commonly
associated symptoms include lightheadedness, dizziness, weakness of

Notes

Blood pressure fall by 30 mm Hg


systolic and 15 mm Hg diastolic
within three minutes of standing from a
previous three-minute supine position.
Associated supine hypertension is

legs, fatigue and syncope. Postprandial


hypotension may be a major feature.

Urogenital

Extrapyramidal
tract

Cerebellar
function

Urinary incontinence or
incomplete emptying.
Erectile dysfunction.

Bradykinesia.
Rigidity.
Postural instability.
Tremor (but not classic
pill rolling).

Gait/limb ataxia.
Ataxic dysarthria.
Oculomotor dysfunction
(sustained gaze-evoked
nystagmus).

common, and is aggravated by


medication used to reduce orthostatic
hypotension.

Urinary dysfunction is
the most frequent initial
complaint in women.
Erectile dysfunction is
the most frequent initial
complaint in men.

Check that postural instability is not


caused by primary visual, vestibular,
cerebellar, or proprioceptive
dysfunction.

825. A 67yo man being managed for a malignancy develops neutropenic


fever. He has been commenced on Ticacillin, Tazobactam and Gentamicin.
He has also recently commenced on Meropenem but on the 3 day his temp
still remains >39C. 2 blood tests and urine cultures show no organism. Inv:
Hgb=104g/dl, WBC=<0.5, Plt=15. What will you do next?
a. Continue IV antibiotics and add oral antifungals
b.
Continue antibiotics and add IV antifungals
c. Stop antibiotics
d. Continue only present antibiotics
rd

826. A young girl who is known to have T1DM presented with


drowsiness and deep breathing. Her sugar level=20. Her BP=120/80 mmHg
and her mucous membranes are dry. What would be the next appropriate
step?
a. Serum urea

b. Blood culture
c. CT
d. HbA1c
e.
ABG
827. Removal of a glioma, which single most appropriate anatomical
structure will be pierced?
a. Cricoid cartilage
b. Rectus sheath muscle
c.
Dura Mater
d. Conjoined tendon
e. Intercostal muscles
828. A child is not breathing and intubation failed. At what anatomical
site should the incision be made?
a.
b.
c.
d.
e.
f.
g.
h.

External iliac muscle


Cricoid cartilage
Linea alba
Rectus sheath muscle
Duramater
1 tracheal cartilage
Conjoined tendon
Intercostal muscles
st

829. A 10yo child who presented with fx of the radius which was treated
with a plaster cast, complains of pain. Exam: limb is swollen but warm and
well perfused, pulses are present. What should you do next?
a. Send for repeat XR
b. Remove cast
c. Replace cast with more padding
d.
Give analgesic
830. A 32yo man has been to Thailand and returned with cervical
lymphadenopathy and fever. What is he most likely suffering from?
a.
HIV
b.
EBV
c.
Typhoid
d.
Measles
key : b
reason : clinical pic fits EBV ( fever + lymphadenopathy )
synonms : glandular fever / infectious mononucleosis

Presentation :

Low-grade fever, fatigue and prolonged malaise.


Lymphadenopathy, especially neck glands

Sore throat; tonsillar enlargement is common, classically exudative


and may be massive; palatal petechiae and uvular oedema.

Fine macular non-pruritic rash


Later signs include:

Mild hepatomegaly and splenomegaly (splenic enlargement returns


to normal or near normal usually within three weeks after the clinical
presentation) with tenderness over the spleen.

Jaundice occurs in fewer than 10% of young adults, but in as many


as 30% of infected elderly patients.
Investigations :
Heterophile antibodies >>> Following IM caused by EBV, 70-90% of
patients produce IM heterophile antibodies (antibodies against an antigen
produced in one species that react against antigens from other species). The
heterophile antibodies are not specific for the virus. These antibodies can be
detected by :
Paul-Bunnell test
Monospot test
EBV-specific antibodies
Management :
Patients are traditionally advised to avoid contact sports for three weeks
because of the risk of splenic rupture
Avoid alcohol for the duration of the illnes
Advise paracetamol for analgesia and control of fever.
No specific antiviral therapy is available.
Short courses of corticosteroids are beneficial for haemolytic anaemia,
central nervous system involvement or extreme tonsillar enlargement.
831. A 6yo child presents with edema and mild proteinuria. No
hematuria. What is the most likely dx?
a.
PSGN
b.
Membranous GN
c.
Minimal change GN
d.
RPGN
key : c
reason : age 6 ys + mild proteinuria
Minimal change disease


Light microscopy is virtually normal but electron microscopy shows
widespread fusion of the epithelial cell foot processes on the outside of the
glomerular basement membrane. Immunofluorescence is usually negative.

Most often presents in children aged between 2 and 4 years. Accounts


for 90% of cases of nephrotic syndrome in children and about 20% of cases
in adults.

Clinical features: nephrotic syndrome with selective proteinuria;


normal renal function, normal blood pressure, normal complement levels;
increased risk of infections, especially urinary tract infections and
pneumococcal peritonitis (therefore give prophylactic penicillin if
oedematous).

Associated with atopy in children, especially those who are HLADR7-positive.

May also be related to underlying Hodgkin's disease in adults.

Usually responds to a course of high-dose prednisolone but relapse is


frequent.

Relapsing disease may go into remission following treatment with


prednisolone and cyclophosphamide or ciclosporin.

One third of patients have one episode, one third develop occasional
relapses and one third have frequent relapses which stop before adulthood.

Minimal change disease does not progress to end-stage chronic kidney


disease.
Membranous nephropathy

Widespread thickening of the glomerular basement membrane occurs.

Immunofluorescence reveals granular deposits of immunoglobulin and


complement.

Although most cases are idiopathic, it may also be secondary to SLE,


hepatitis B, malignancy or the use of gold or penicillamine.

It is more common in men.

It is the most common cause of nephrotic syndrome in adults. It may


present with proteinuria or nephritic syndrome, hypertension. Haematuria is
rare.

The idiopathic form may respond to a treatment regimen involving


alternate months of corticosteroids with chlorambucil or cyclophosphamide,
or to ciclosporin.

It progresses to end-stage kidney disease in 30-50% of patients. The


remainder with idiopathic membranous nephropathy has a complete or
partial spontaneous remission of nephrotic syndrome with stable renal
function.

832. An 80yo woman suffering from RA presents with severe


epigastric pain and vomiting. She also complains of shoulder tip pain. What
is the single most discriminatory inv?
a.
US Abdomen
b.
Sigmoidoscopy
c.
Colonscopy
d.
Barium meal
e.
Upper GI endoscopy
f.
Erect CXR
key : f
reason : acute abdomen + diaphragmatic irritation >>>> gut perforation
mostly peptic ulcer because of RA drugs
Indications of plain abdominal x-ray
Renal colic:

A 'KUB' picture is requested. This is a large film that is designed to


take in the kidneys, ureters and bladder.

About 90% of renal stones are radio-opaque. Uric acid stones may
be missed.

False positives may occur from phleboliths that are most common in
the pelvic veins. False negatives may arise, especially if stones are small.

Calcification may represent gallstones but only a minority of


gallstones are radio-opaque. Gallstones become more frequent with age and
are often asymptomatic.[4]

Doctors in A&E tend to be poor at identifying stones on plain films


but, if urinalysis is negative, the diagnosis is unlikely to be renal colic.
Intestinal obstruction:

Erect and supine films are used to confirm the diagnosis.

Obstruction of the small bowel shows a ladder-like series of small


bowel loops but this also occurs with an obstruction of the proximal colon.
Fluid levels in the bowel can be seen in upright views.

Distended loops may be absent if obstruction is at the upper


jejunum.

Obstruction of the large bowel is more gradual in onset than small


bowel obstruction. The colon is in the more peripheral part of the film and
distension may be very marked.

Fluid levels will also be seen in paralytic ileus when bowel sounds
will be reduced or absent rather than loud and tinkling as in obstruction.

In an erect film, a fluid level in the stomach is normal as may be a


level in the caecum. Multiple fluid levels and distension of the bowel are
abnormal.

Perforation of the intestine:

If the bowel has been perforated and a significant amount of gas has
been released it will show as a translucency under the diaphragm on an erect
film.

Gas will also be found under the diaphragm for some time after
laparotomy or laparoscopy.
Appendicitis

An appendicolith may be apparent in an inflamed appendix in 15%


of cases but as a diagnostic point in the management of appendicitis, the
plain X-ray is of very limited value.[6]

It may be of value in infants.


Intussusception:

Intussusception occurs in adults and children.

A plain abdominal X-ray may show some characteristic gas


patterns.[8]

A sensitivity and specificity of 90% adds to this rather difficult


diagnosis but ultrasound is vastly superior.[9]

Detection of swallowed foreign bodies:

Plain X-ray will detect the presence of radiopaque foreign bodies.

A plain abdominal X-ray will show 90% of cases of 'body packing'


(internal concealment of drugs to avoid detection) but there will be false
positives in 3%.
833. A 44yo man went on holiday to Sudan 5wks ago. He now
presents with red urine and fever. Exam: hepatomegaly. What is the most
likely dx?
a.
Malaria
b.
Brucellosis
c.
Leptospirosis
d.
Schistosomiasis
key : d
Schistosomiasis
Presentation

Infection can be acute or chronic.

Physical findings vary with the stage of illness, worm burden, worm
location, and organs involved.

Schistosomiasis is associated with anaemia, chronic pain, diarrhoea,


exercise intolerance, and malnutrition.[2]

Acute syndrome (Katayama syndrome)


The acute reaction is due to the sudden release of highly antigenic
eggs.

The most common acute syndrome is Katayama fever. It usually


occurs in children or young adults with no past exposure to the disease and
is most likely with S. japonicum.

As travellers present several weeks after contact with infested water, it


is necessary to obtain a careful travel history, including drinking water
sources and activities such as swimming.
Symptoms:

Most acute infections are asymptomatic.

The first sign may be swimmer's itch in which there is an urticarial


response for a few days after the parasite has penetrated the skin.

Malaise.

Arthralgia or myalgia.

Cough.

Diarrhoea.

Right upper quadrant pain.


Signs:

Fever.

Hepatosplenomegaly.

Right upper quadrant pain or tenderness.

Urticaria may be seen occasionally.

Lymphadenopathy.

Initial invasion of skin and infection with non-human species may


cause itching and rash.
Chronic disease

Chronic schistosomiasis can present months to years after exposure,


making diagnosis difficult.

It is endemic in poor, rural areas.

Many patients have not had an acute syndrome.

Symptoms may be few or mild. They may be nonspecific or reflect the


site of egg production in the mesentery or bladder wall, the extent of damage
to liver or spleen, the degree of lung involvement, and possibly other sites
including the central nervous system (CNS).
Symptoms:

Bloody diarrhoea.

Abdominal pain, right upper quadrant pain, cramps.

Haematemesis, which can occur from oesophageal varices with portal


hypertension.

Haematuria, dysuria:


The first feature may be frequency of micturition.

Initially, haematuria is only terminal but, as it becomes more severe,


the blood produces red urine throughout the stream.

There is proteinuria.

Pulmonary hypertension may produce:

Fatigue.

Dyspnoea on exertion.

Cough.

Atypical chest pain.

Hepatosplenomegaly.
Signs:

Abdominal tenderness.

Ascites with portal hypertension.

Seizures and/or altered mental state (with cerebral infection).


Investigations

Microscopic examination of stool or urine is the gold standard for


diagnosis but requires the adult worms to be producing eggs.
Serology can diagnose less advanced infections
FBC shows eosinophilia and anaemia.
Renal function may be impaired if the urinary tract is obstructed.
Management
Praziquantel is the drug of choice in most cases.
The World Health Organization (WHO) believes that praziquantel is safe
in pregnancy, lactation and in children under the age of 24 months.
Oxamniquine is the only alternative
In acute Katayama fever, corticosteroids are very important to subdue the
hypersensitivity reaction.
Corticosteroids and anticonvulsants may be needed as adjuvants to
praziquantel in neuroschistosomiasis.
Surgical

Endoscopy and sclerotherapy can treat oesophageal varices.

A ventriculoperitoneal shunt and corticosteroids are required to treat


hydrocephalus and raised intracranial pressure in cerebral schistosomiasis.
Complications
Urinary tract

Secondary bacterial infection and renal stones may occur.

There is an increased risk of squamous cell carcinoma of bladder


that has been noticed especially in Egypt. It is possible that the infestation
and the carcinogens in tobacco smoke have a synergistic effect.


Hydronephrosis may occur but will reverse if the disease is treated,
suggesting that the renal parenchyma is compressed but not destroyed and
renal function is not markedly impaired.

Schistosomal nephropathy leading to renal failure may occur.

Female urogenital schistosomiasis may be a risk factor for HIV


infection.
Alimentary canal
Gastrointestinal complications include gastrointestinal bleeding,
gastrointestinal obstruction, malabsorption and malnutrition.

Lesions tend to bleed and there is loss of blood and protein, causing
iron-deficiency anaemia and hypoproteinaemia. These lesions are mostly in
the colon and rectum.

Fibrosis of the liver occurs, producing portal hypertension. S. mansoni


infection invariably results in liver fibrosis.

Portal hypertension can produce oesophageal varices that may bleed,


and ascites.

Portocaval shunting predisposes to pulmonary infestation and


problems of pulmonary hypertension.

Co-infection with hepatitis, HIV, and malaria can increase the risk of
hepatocellular carcinoma and increase the risk of mortality.
Other complications

Chronic septicaemic salmonellosis (prolonged fever with enlargement


of the liver and spleen) may occur in schistosoma-infected individuals who
are co-infected with salmonella.

Pulmonary hypertension.

Cor pulmonale.

Neuroschistosomiasis (includes increased intracranial pressure,


myelopathy and radiculopathy).
834. A 32yo homosexual comes with hx of weight loss. Fundoscopy
reveals retinal hemorrhages. What is the single most appropriate option?
a.
Mycobacterium avium
b.
CMV
c.
Hemophilus influenze
d.
NHL
e.
Pneumocystic jerovici
key : B
reason : homosexual +wt loss >>> HIV >>> CMV
Cytomegalovirus and HIV infection
CMV can cause very serious infection in HIV infection.
Retinitis:


Retinitis is the most common manifestation of CMV disease in
patients who are HIV positive.

It presents with decreased visual acuity, floaters, and loss of visual


fields on one side.

Ophthalmological examination shows yellow-white areas with


perivascular exudates. Haemorrhage is present. Lesions may appear at the
periphery of the fundus, but they progress centrally.

It begins as a unilateral disease, but in many cases it progresses to


bilateral involvement. It may be accompanied by systemic CMV disease.

Ganciclovir has been used to treat retinitis, but it only slows the
progression of the disease. The optimal treatment is using ganciclovir
implants in the vitreous, accompanied by intravenous ganciclovir therapy.

Oral ganciclovir may be used for prophylaxis of CMV retinitis. It


should not be used for treatment.
CMV pneumonia in patients who are HIV positive is uncommon. The reason
for this is unknown.
Gastrointestinal tract:

In the upper gastrointestinal tract, CMV has been isolated from


oesophageal, gastric and duodenal ulcers. Patients with oesophageal disease
may present with painful dysphagia.

In the lower gastrointestinal tract, patients with CMV may present


with diarrhoea due to colitis.
CMV may cause disease in the peripheral and central nervous system.
835. A 30yo man comes with hx of early morning back pain and
stiffness. Exam: red eyes. What is the single most appropriate option?
a.
Iris
b.
Ciliary body
c.
Cornea
d.
Conjunctivitis
e.
Sclera
key : A
reason : male pt with early morning back pain and stiffness >> AS
>>>uvitits
Presentation
AS usually presents before the age of 30 years.
Most patients have mild chronic disease or intermittent flares with
periods of remission.
Systemic features are common. Fever and weight loss may occur
during periods of active disease. Fatigue is also prominent.
Morning stiffness is characteristic.

Inflammatory back pain which Often improves with moderate


physical activity.
The spinal disease starts in the sacroiliac joints (bilateral
lumbosacral region) and may be felt as diffuse nonspecific buttock pain.
Peripheral enthesitis:
Occurs in approximately a third of patients.
Common sites - behind the heel (Achilles tendonitis), the heel pad
(plantar fasciitis) and the tibial tuberosity.
Lesions tend to be painful, especially in the morning. There may be
associated swelling of the tendon or ligament insertion.
Peripheral arthritis:
Also occurs in about a third of patients.
Joint involvement is usually asymmetric, involving the hips, shoulder
girdle (glenohumeral, acromioclavicular, and sternoclavicular joints), joints
of the chest wall (costovertebral joints, costosternal junctions) and
symphysis pubis.
Other peripheral joints are less often and less severely affected,
usually as asymmetrical oligoarthritis.
In children, AS tends to commence with arthritis prior to spinal
disease developing.
Temporomandibular joints are occasionally involved.
Extra-articular manifestations of AS
Eye involvement
Acute anterior uveitis occurs in 20-30% of patients. Of all patients
presenting with acute anterior uveitis, a third to a half have or will go on to
develop AS.
Acute anterior uveitis presents with an acutely painful red eye and
severe photophobia and requires emergency treatment to prevent visual loss.
Cardiovascular involvement
This occurs in <10% of patients, usually in those with severe longstanding disease.
Aortitis of the ascending aorta may lead to distortion of the aortic
ring, causing aortic regurgitation.
Fibrosis of the conduction system may result in various degrees of
atrioventricular block, including complete heart block.
Pulmonary involvement
Restrictive lung disease may occur in patients in later stages, with
costovertebral and costosternal involvement limiting chest expansion.
Pulmonary fibrosis of the upper lobes.
Renal involvement

Amyloidosis is a very rare complication in patients with severe, active


and long-standing disease and may cause renal dysfunction with proteinuria
and renal insufficiency or chronic kidney disease.
Immunoglobulin A (IgA) nephropathy is another association.
Neurological involvement
This usually occurs secondary to fractures of a fused spine.
Also, patients with AS are prone to atlanto-axial subluxation, which
may lead to cervical myelopathy.
Cauda equina syndrome may occur in patients with severe longstanding disease.
Metabolic bone disease
Osteopenia and osteoporosis may occur in patients with long-standing
spondylitis, further increasing risk of fracture.
Diagnosis
The British Society for Rheumatology recommends that the modified New
York criteria be used to diagnose AS.
Clinical criteria:

Low back pain, for more than three months; improved by exercise,
not relieved by rest.

Limitation of lumbar spine motion in both the sagittal and the


frontal planes.

Limitation of chest expansion relative to normal values for age and


sex.
Radiological criterion: sacroiliitis on X-ray.
Definite AS is diagnosed if the radiological criterion is present plus at
least one clinical criterion and probable AS if three clinical criteria are
present, or if the radiological criterion is present but no clinical criteria are
present.
investigations :
X-rays are the most helpful imaging modality in established disease,
although they may be normal in early disease.
MRI scanning may be useful in identifying early sacroiliitis. MRI of the
sacroiliac joints is more sensitive than either plain X-ray or CT scan in
demonstrating sacroiliitis
Musculoskeletal ultrasound scanning can help in diagnosing enthesitis.
836. A 70yo woman with longstanding anxiety is seen in the OPD.
She complains of her heart skipping a beat quite often. This particularly
occurs when she is trying to get to sleep. The palpitations are never
sustained. What is the most likely rhythm disturbance?
a.
SVT

b.
c.
d.
e.

VF
VT
V-ectopics
A-fib
837. A 17yo has acute pain around his right eye, pain on one side of
his face and ear ache too. What is the single most dx?
a.
Ear wax
b.
Ear foreign body
c.
Dental abscess
d.
Cellulitis
e.
Herpes zoster
key : e
reason : acute onset + unilatral with sensory nerve distribution
THIS PT IS IN the pre eruptive phase >>> no blisters yet
838. A 12yo boy presented with itching in his hands. Exam: skin is
dry and red. His mother is asthmatic and older brother has hay fever. What is
the single most likely causative factor?
a.
Dermatitis herpitiformis
b.
Scabies
c.
Eczema
d.
Uremia
e.
Drug induced
key : c
Eczema
Diagnostic criteria

Must have an itchy skin condition (or report of scratching or rubbing


in a child) plus three or more of the following:

History of itchiness in skin creases such as folds of the elbows,


behind the knees, fronts of ankles, or around the neck (or the cheeks in
children aged 18 months or under).

History of asthma or hay fever (or history of atopic disease in a


first-degree relative in children aged under 4 years).

General dry skin in the preceding year.

Visible flexural eczema (or eczema affecting the cheeks or forehead


and outer limbs in children aged under 4 years).

Onset in the first two years of life (not always diagnostic in children
aged under 4 years).

If it does not itch it is very unlikely to be eczema.


management :
emolient + topical steroids

839. A 45yo man presented with pruritic purple papules on the flexor
surface of his wrist and some white lacy markings on his buccal mucosa.
What is the single most likely causative factor?
a.
ALL
b.
Lymphoma
c.
Polycythemia
d.
IDA
e.
Lichen planus
key : e
Lichen planus
Presentation
Onset is usually acute, affecting the flexor surfaces of the wrists,
forearms and legs.
The typical lesion is an intensely itchy 2-5 mm red or violet shiny flattopped papule with white streaks ('Wickham's striae').
Mucous membranes are commonly affected >>> Classically, white slightly
raised lesions with a trabecular, lacy appearance on the inside of the cheeks
840. A known DM was admitted with sudden LOC. What is the
initial inv?
a.
CT scan
b.
RBS
c.
MRI
d.
ECG
e.
ABG
key : B
reason : diabetic pt >>> hypogycemia is common >>> RBS is the initial inv
841. A 36yo lady comes with hx of early morning stiffness of her
small joints and with red and painful eye. What is the single most
appropriate option?
a.
Iris
b.
Ciliary body
c.
Cornea
d.
Conjunctivitis
e.
Sclera
f.
Lichen planus
key : E
reason :female pt with early morning stiffness of her small joints >>> RA
>>> scleritis
Scleritis

Scleritis often appears in association with other inflammatory diseases such


as rheumatoid arthritis and granulomatosis - the histopathological changes
are characteristic of a chronic granulomatous disease.
842. A 23yo man comes with 2d hx of sticky greenish discharge
from the eyes with redness. What is the single most appropriate option?
a.
Iris
b.
Ciliary body
c.
Cornea
d.
Conjunctivitis
e.
Sclera
key : D
Bacterial conjunctivitis
Presentation
Symptoms
Discomfort - burning or gritty but not sharp.
Pain is minimal; significant pain suggests a more serious diagnosis.
Vision is usually normal, although 'smearing', particularly on waking,
is common.
Discharge tends to be thick rather than watery.
There may be mild photophobia. Significant photophobia suggests
severe adenoviral conjunctivitis or corneal involvement.
History
Ask about contact lens wear: establish whether this could be (or lead
to) a problem of the (vulnerable) cornea.
Time course: onset, duration - in chronic cases consider venereal
disease in people at a sexually active age.
Use of over-the-counter medication: consider whether this could be a
reaction to previously administered drops or ointment.
Social aspect: establish whether anybody else has had it (family,
school, work). Determine whether there are concerns about working during
the course of the illness.
Findings
'Red eye' with uniform engorgement of all the conjunctival blood
vessels.
Bacterial conjunctivitis may often be distinguished from other types
of conjunctivitis by the presence of a yellow-white mucopurulent discharge.
Eyes may be difficult to open in the morning, glued together by
discharge.

There is also usually a papillary reaction (small bumps on the


palpebral conjunctiva, appearing like a fine velvety surface). The presence
of follicles is more likely to indicate viral conjunctivitis.
Bacterial conjunctivitis is usually bilateral (but often sequential).
Check visual acuity - this should be normal, other than the mild and
temporary blur secondary to the discharge which can be blinked or wiped
away.
843. A pt was admitted with erectile dysfunction, reduced facial hair
and galactorrhea. What is the most probable dx?
a.
Hyperprolactinemia
b.
Cushings syndrome
c.
Pheochromocytoma
d.
Hyperthyroidism
e.
Hypoparathyroidism
key : A
reason : galactorrhea + with erectile dysfunction + reduced facial hair
hyperprolactinaemia
Presentation
Women:
Common symptoms of are amenorrhoea, oligomenorrhoea and
galactorrhoea.
They may also have infertility, hirsutism and reduced libido.
Men:
The hormonal effects of raised prolactin levels are subtle and develop
slowly.
Endocrine symptoms are reduced libido, reduced beard growth and
erectile dysfunction.
Children:
Growth failure and delayed puberty are possible presentations in
children.
Symptoms due to tumour size (usually macroprolactinomas):
Headache.
Visual disturbances (classically, a bitemporal hemianopia (lateral
visual fields) or upper temporal quadrantanopia).
Cranial nerve palsies.
Symptoms and signs of hypopituitarism.
Rarely, cerebrospinal fluid (CSF) leak or secondary meningitis.
Investigations
Initial investigations
TFTs.

Exclude pregnancy.
Basal serum prolactin:
If prolactin is mildly elevated (eg 400-1000 mU/L, normal range <400
mU/L), it should be repeated before referral.
Dynamic prolactin stimulation tests, such as the TRH test, are not
required. Measurement of serum prolactin on three separate occasions (at
least two hours after rising and when the patient is rested) is sufficient.
A prolactin level >5000 mU/L usually indicates a true prolactinoma.
Further investigations
Visual field testing.
Pituitary imaging (preferably MRI).
Assessment of pituitary function
844. A 32yo man has been repeatedly admitted to hospital for what
was described as anxiety or panic attacks and palpitations. On occasions he
is found to be tremulous and hypertensive. A persistent weight loss is noted.
What is the most probable dx?
a.
Hyperthyroidism
b.
Panic attacks
c.
Pheochromocytoma
d.
Cushings disease
e.
GAD
key : c
reason :male young age + anxiety + palpitation +wt loss +HTN
Pheochromocytoma
signs and Symptoms
Headache
Profuse sweating
Palpitations
Tremor
Nausea
Weakness
Anxiety
Sense of doom
Epigastric pain
Flank pain
Constipation
Weight loss
Hypertension but it may be paroxysmal in 50%.
Postural hypotension.

symptoms are intermittent and may vary from once a month to several times
a day with duration from seconds to hours.
Investigations
Blood tests
Blood glucose is often raised.
Calcium may be elevated.
Haemoglobin is elevated
Plasma catecholamines and plasma metanephrines (the o-methylated
metabolites of catecholamines) have both been used in diagnosis
Urine
24-hour urine collection is required for creatinine , total
catecholamines, vanillylmandelic acid (VMA) and metanephrines.
Imaging
After biochemical confirmation of a tumour, imaging is necessary to locate
it.[9]
90% of phaeochromocytomas are in the adrenal glands and 98%
within the abdomen. Common locations for extra-adrenal
phaeochromocytomas include close to the origin of the inferior mesenteric
artery, bladder wall, heart, mediastinum and carotid and glomus jugulare
tumours.
MRI can locate all tumours within the adrenals.
CT is less sensitive and detects around 85-95% of tumours in excess
of 1 cm in diameter.
management :
Surgical resection of the tumour is the treatment of choice and usually
results in cure of the hypertension. Pre-operative treatment with alphablockers and beta-blockers is required to control blood pressure and prevent
intraoperative hypertensive crises.
845. A 35yo man with T1DM is dehydrated with BP of 90/50mmHg.
What is the single most appropriate initial inv?
a.
ABG
b.
CBC
c.
HbA1c
d.
LFT
e.
Serum Urea
key: a
reason : T1DM suspected to have DKA>>> ABG will show acidosis
DKA
DKA is characterised by hyperglycaemia, acidosis, and ketonaemia

DKA is normally seen in type 1 diabetics and may be a presenting feature of


undiagnosed type 1 diabetes, particularly in children. However DKA may
rarely occur in type 2 diabetics
Check capillary blood glucose and blood gases promptly. If these suggest
diabetic ketoacidosis (DKA) then immediately begin resuscitation and
management.
management:

Immediate resuscitation as required


Correct dehydration >>> 0.9% sodium chloride solution is the
recommended fluid of choice
Insulin therapy >>> A fixed-rate IV insulin infusion calculated on 0.1
units/ per kilogram infusion is recommended.
Metabolic treatment
Treat any precipitating illness
846. In OGTT what is the glucose venous plasma level 2h after glucose
intake which indicates impaired glucose tolerance?
a. >11.1mmol/l
b.
Between 7.8-11.0mmol/l
c. Between 8.0-10.9mmol/l
d. Between 10.0-11.0mmol/l
e. Between 7.1-11.0mmol/l
ANSWER is B.

847. A young man who has no PMH presented with jaundice, low Hgb,
retics 8% and other indices WNL but occasional spherocytes were seen on
blood film. What is the single most appropriate inv?
a. G6PD enzyme assay
b.
Direct coombs test
c. Repeat blood film
d. Indirect coombs test
e. BMA
Normal retic count is 1%. Increased reticulocyte count always indicates hemolysis. Since
there are spherocytes on the blood film it means this patient either has autoimmune
hemolytic anemia or hereditary spherocytosis. Direct coombs test will be positive in
autoimmune hemolytic anemia. For HS Flow cytometric analysis of eosin-5-maleimide
(EMA) binding to red cells, and cryohaemolysis test have replaced osmotic fragility tests.

848. A 22yo man came to the hosp after an injury in his hand while
playing basketball. Exam: avulsion of extensor tendon from the distal
phalanx. What is the single most probable deformity?
a. Dinner fork deformity
b. Game keeper thumb
c.
Mallet finger
d. Gun-stock deformity
e. Garden spade deformity
A Colles' fracture (dinner fork deformity) is a fracture of the distal radius in the forearm
with dorsal and radial displacement of the wrist and hand.
Game keepers thumb: This is injury to the ulnar collateral ligament (UCL) of the MCP joint
(on the medial side of the thumb) due to forced abduction of the MCP.
Mallet Finger: There is avulsion of the extensor tendon causing the finger to be stuck in
moderate flexion.
Gunstock deformity: Also known as cubitus varus. Cubitus varus (varus means a
deformity of a limb in which part of it is deviated towards the midline of the body) is a
common deformity in which the extended forearm is deviated towards midline of the body .
Garden spade deformity: This is the Smiths fracture. Reverse of colles. The definition is a
fracture of the distal radius, with or without ulnar involvement, that has volar (anterior)
displacement of the distal fragments
So the ANSWER here is C.

849. A 28yo man is inv for recurrent lower back pain. A dx of


Ankylosing Spondylitis is suspected. Which of the following inv is most
useful?
a. ESR
b.
XR sacro-iliac joints
c. HLA B27
d. XR thoracic spine
e. CT lumbar spine

Tests: Diagnosis is clinical, supported by imaging (MRI is most sensitive


and better at detecting early disease). Sacroiliitis is the earliest X-ray feature,
but may appear late.
In later stages, calcification of ligaments with ankylosis lead to a bamboo
spine appearance.
Also: FBC (normocytic anaemia), ESR, CRP, HLA B27+ve (not diagnostic)
Treatment: Exercise, NSAIDs, TNF blockers etanercept, adalimumab and
golimumab are indicated in severe active AS if NSAIDS fail
850. A 4yo girl is taken by her mother to the ED and complains of feeling
unwell, urinary urgency and temp=39C. What is the single next best inv?
a. Catheter catch of urine
b.
Clean catch of urine
c. US
d. IVU
e. Suprapubic catch of urine
ANSWER B.
Recommended way of getting a urine sample: Dipstick all ward urines. If
nitrites or WCC +ve, get a clean catch (or a suprapubic aspirate or catheter
sample; bag urines have many false positives from vulvitis or balanitis).
Wash the genitals gently with water, and tap repeatedly in cycles of 1min
with 2 fingers just above the pubis, 1h after a feed, and wait for a clean
voided urine (CVU) sample, avoiding the streams 1st part
851. A 2yo girl presents with a 4d hx of fever that started with a cough.
Her RR=45bpm, sat=94%, temp=38.9C, capillary refill time=1s. There are
crepitations at the left base on auscultation. Urine shows negative dipstick.
What is the single inv most likely to lead to dx?
a. Blood for C&S
b. ESR
c.
CXR
d. Urine for C&S
e. CSF analysis
ANSWER C.
The main presenting symptoms here are Cough, fever and tachypnea. Which
means some respiratory problem is present. A cause of fever has been ruled
out by giving the negative urine test which rules out UTI. Plus there are
crepitations at the base of lung. So we will first do a chest X-ray to look for

the cause possibly pnuemonia. Blood C & S takes times. ESR is raised in
sooo many diseases and will not point towards any specific diagnosis. Urine
dipstick is negative so no need to culture that. No signs of neurological
involvement so no need to do CSF analysis.
852. A 3yo girl presents with fever for 2d. She is drowsy and had a
seizure causing twitching of the right side of the body for 4mins. Her RR=30
bpm, sat=90%, temp=38.9C, capillary refill time=2s. Urine negative on
dipstick. What is the single inv most likely to lead to dx?
a. Blood for C&S
b. ESR
c. CXR
d. Urine for C&S
e. CSF analysis
This question is similar to the one given above. But here the patient is
drowsy and had a seizure which shows CNS involvement so we would go
for CSF analysis to rule out meningitis most probably.
853. A 6m boy is admitted with persistent irritability. He is lethargic and
is not feeding as well as usual. His RR=30bpm, sat=97%, temp=38.0C,
capillary refill time=2s. Urine reveals leucocytes on dipstick. What is the
single inv most likely to lead to dx?
a. Blood for C&S
b. ESR
c. CXR
d.
Urine for C&S
e. CSF analysis
Again a similiar sort of a scenario. Only abnormal sign is the temperature
with symptoms of irritability, lethargy and not taking feed. Here the urine
analysis reveals leucocytosis pointing towards a possible diagnosis of UTI
so we go for urine culture and sensitivity to know about the particular
organism and the specific antibiotic for it. All other inv here will be useless
until UTI has been ruled out.
854. A 3yo boy presents with a 1d hx of being unwell. He appears
shocked and has 3h old rash made up of urticarial and purpural spots. His
RR=30bpm, sat=94%, temp=39C, capillary refill time=1s. Urine is clean on
dipstick. What is the single inv most likely to lead to dx?
a.
Blood for C&S

b.
c.
d.
e.

ESR
CXR
Urine for C&S
CSF analysis

Patient with fever and rash could lead to meningitis. In a previous question
when meningitis was suspected we went for CSF analysis but here since
there is a rash LP is contraindicated so we go for Blood C & S.
Contraindications of LP: DIC; purpura or brain herniation is near (odd
posture or breathing; glascow coma scale <13; dilated pupils, dolls eye
reflexes, BP, pulse, papilloedema.
855. A child is dx with VUR. What would you tell his parents?
a. Requires antibiotic prophylaxis
b.
Most will require surgery
c. Most will have kidney scarring by 5yo
d. Nothing can be done
e. Reassure
Well according to the current guidelines there is little benefit from surgery in
patients with VUR. OHCS 9th ed. page 175 states Surgical correction of
moderate reflux is unlikely to be beneficial, and in minor reflux is likely
to be harmful
And for prophylaxis it is recommended that no prophylaxis shall be given
after the first episode of UTI but antibiotic prophylaxis is recommended in
patients with recurrent UTIs. So the correct answer here should be A.
856. A 2yo child presents with severe vomiting. Exam: mass felt in
abdomen. What inv is most appropriate?
a.
US
b. XR
c. CT
d. CBC
Pyloric stenosis dd intussusseption

Presentation

Typical presentation is onset of vomiting at 2-8 weeks of age (late presentation


up to 6 months can occur but is very rare)
o Vomiting: non-bilious, often but not always projectile and usually 3060 minutes after a feed, with the baby remaining hungry.
o Vomiting increases in frequency over several days.
o Vomiting also increases in intensity until it becomes projectile.
o Slight haematemesis may occur.
Persistent hunger, weight loss, dehydration, lethargy, and infrequent or absent
bowel movements may be seen.
Stomach wall peristalsis may be visible.
An enlarged pylorus, classically described as an 'olive', may be palpated in the
right upper quadrant or epigastrium of the abdomen:
o The 'olive' is best palpated at the start of a feed but is often missed.
o With the infant supine and the examiner on the child's left side,
gently palpate the liver edge near the xiphoid process.
o Then displace the liver superiorly; downward palpation should
reveal the pyloric olive just on, or to the right of, the midline.
o It should be possible to roll the pylorus beneath the examining
finger.

Investigations

Serum electrolytes (for correction of imbalances before surgical repair); there is


often metabolic alkalosis with severe potassium depletion. However biochemical
disturbances are now much less common with earlier diagnosis.[8]
Ultrasound is reliable and easily performed and has replaced barium studies as
the main investigation.[9] There is a normal variation of pylorus muscle
measurements with age and gestation but ultrasound has a very high sensitivity
and specificity.[10]

857. A 13yo girl complains of a 2d hx of hoarseness of voice a/w dry


cough. She feels feverish. On direct laryngoscopy, her vocal cords are
grossly edematous. What is the single most appropriate inv?
a.
None req
b. Sputum for AFB
c. Laryngoscopy
d. Bronchoscopy
e. XR cervical spine
acute laryngitis probably due to viralinfection no need of any further investigation

Acute laryngitis

Investigations are rarely helpful in primary care. A swab for microbiological


analysis may be contributory if excessive exudate is present.

Clinicians with the skill to perform indirect laryngoscopy will typically find redness
and small dilated vasculature on the inflamed vocal folds.

858. A 7yo girl is brought by her mother with bright red staining of her
underpants. She also gives a hx that her daughter recently started taking
horse riding lessons. What is the single most appropriate next action?
a. Local exam
b.
Exam under GA
c. Continue regular child care
d. Inform child protection services
e. Coag profile
it will be extremely difficult to examine a child unless under GA. We can't assume that the problem is
superficial so we might need to have a look deeper to exclude any serious problem caused the mother to
bring her child to hospital.

859. A 7d baby whose birth weight was 3.5kg and now is 3kg. What is
the most appropriate next step?
a. Check child protection register
b. Nutritional assessment
c. Skeletal survey
d.
Continue regular child care
e. Inform police
it's normal to lose weight .5 kg during initial period of life and baby will regain weight double the birth weight
by 2-3 month

860. A 6yo child fell on his nose 2d ago. His parents have now brought
him with difficulty in breathing. Exam: fever, nasal bones are straight. What
is the single most likely dx?
a. Nasal polyp
b. Septal hematoma
c.
Septal abscess
d. Deviated nasal septum
e. Fx nose
Fever= abscess.
Hematomas are not associated with fever unless they are infected.

861. A 12yo pt came to the OPD with complains of fever, malaise,


weight loss, anorexia and productive cough. Exam: temp=39C,
pulse=100bpm. His mother says that he has a hx of recurrent chest infections
and he is not thriving well. What is the single most likely causative
organism?

a.
b.
c.
d.
e.

Pneumococcal pneumonia
Staphylococcus
Mycobacterium TB
Pseudomonas
PCP

Pseudomonas common infective agent in CF

862. A 3yo child brought by his mother. Exam: bruises on the buttocks.
Mother also gives hx of runny nose 2wks ago. What is the single most
appropriate next action?
a. Check child protection register
b.
Coag profile
c. Skeletal survey
d. Continue regular child care
e. Inform police
This is HSP.
Answer should be B.
NAI also presents this way with bruise fractures multiple callus formation but the history is a bit different like
step father came to emergency department.
But again most of the options are given for NAI.

863. A 4yo is brought to the ED by ambulance. His mother reports that he


has been unwell with a sore throat for 8h. He is sitting on his mothers knee
and is tolerating an oxygen mask but looks unwell. He has constant noisy
breathing and he is drooling saliva. His temp=39C. What is the most imp
dx?
a. Acute asthma
b. Bronchiolitis
c. Croup
d.
Epiglottitis
e. Tonsillitis
In epiglotittis there is always drooling of saliva

864. A pt with terminal cancer is being treated with chemo develops


tingling and numbness of the fingertips of both arms. What is the single
most likely cause of the symptoms?
a. Bone mets to cervical vertebrae
b.
Post-chemo neuropathy
c. Hyponatremia
d. Hypocalcemia
865. An 80yo man has a permanent catheter. Catheter specimen urine
found lots of e-coli. What is the single most appropriate management as he
wants to attend his daughters wedding next week?
a.
Change the catheter

b.
c.
d.
e.

Prolonged antibiotics
Bladder wash
Repeat MSU after wedding
Reassure

Catheter has become infected due to presence of e coli if it is left aside it will lead to catheter related UTI

866. A 35yo male typist who suffered a scaphoid fx was treated with a
scaphoid cast. After 2wks when the cast was removed for a review XR, it
was found that he had problems in moving the thumb, index and middle
fingers. What would you suggest as the management for the recent prb?
a.
Release of flexor retinaculum
b. Release of common flexor sheath
c. Release of palmar sheath
d. Ulnar nerve release
e. Fasciotomy
867. A pt on insulin is booked in for a hernia operation. What is the most
appropriate management of insulin?
a. Give insulin and saline pre-op
b. Stop insulin for the duration of the op
c.
Give IV insulin + dextrose + saline pre-op
d. Give insulin as usual pre-op
e. None
868. A 35yo male who recently had an appendicectomy has got severe
pain in his right big toe. Joint is red and swollen. He consumes 30 units of
alcohol/week. What is the most probable dx?
a. Rhabdomyosarcoma
b. Osteoarthritis
c.
Gout
d. Pseudogout
e. Arthritis
869. A 25yo male who recently noticed change in his shoe size, he is also
constipated, has a preference to hot weather, his skin is dry, has severe pain
in wrist joint. Joint is red and swollen. What is the most probable dx?
a. Chondro-sarcoma
b. Lipo-sarcoma
c. Gout
d.
Pseudogout
e. Ankylosing spondylitis
870. A 45yo woman had her visual acuity checked at her local optician.
12h later she presents to the ED with severe pain and redness in her eye.
What is the single most appropriate option?

a. Iris
b. Ciliary body
c.
Ant chamber
d. Post chamber
e. Cornea
871. A 75yo man who has DM and HTN experiences acute monocular
blindness which resolves after 1h. What is the most likely dx?
a. GCA
b. Optic neuritis
c. Lacunar infarct
d. Pontine hemorrhage
e.
Amaurosis fugax
872. A 26yo presents with prolonged constipation, blood on side of stool
and very painful defecation. PR exam: very painful. What is the single most
likely dx?
a. Ca Colon
b. UC
c. CD
d.
Anal fissure
e. Constipation
873. A 35yo man with painless left testicular enlargement for the past 6m
which is increasing in size and 3x larger than the right side. There is no
tenderness or redness. What is the most likely dx?
a.
Testicular tumor
b. Hydrocele
c. Epididymal cyst
d. Epididymo-orchitis
e. Reassure
874. A middle aged man who has had a hx of chronic sinusitis, nasal
obstruction and blood stained nasal discharge. He now presents with cheek
swelling, epiphora, ptosis, diplopia, maxillary pain. What is the single most
likely dx?
a. Nasopharyngeal ca
b. Pharyngeal ca
c.
Sinus squamous cell ca
d. Squamous cell laryngeal ca
e. Hypopharyngeal tumor
875. A 60yo man with a long hx of smoking and alcohol presents with
nasal obstruction, epistaxis, diplopia, otalgia and conductive deafness. What
is the single most likely dx?

a.
Nasopharyngeal ca
b. Pharyngeal ca
c. Sinus squamous cell ca
d. Squamous cell laryngeal ca
e. Hypopharyngeal tumor
876. A 60yo is on tx for IHD, HTN and hyperlipidemia. During the night
he complains of wheeze and SOB. Which of the following meds is
responsible for that?
a. Amlodipine
b.
Atenolol
c. Ramipril
d. Simvastatin
e. Bendroflumethiazide
Key : B
Clincher : wheeze and SOB which is a very common side effect of Atenolol,
a beta blocker which are contraindicated in asthma as it causes severe
bronchospasm leading to wheeze and SOB
SOB is less common with calcium channel blockers, ACE inhibitors and anti
hyperlipidemics. Thiazide diuretics do not cause SOB
BETA BLOCKERS
Beta-blockers are medicines that are used to treat a variety of conditions.
Their full correct name is beta-adrenoceptor blocking medicines. There are
several types of beta-blocker. They include acebutolol, atenolol, bisoprolol,
carvedilol, celiprolol, labetalol, metoprolol, nadolol, nebivolol, oxprenolol,
pindolol, sotalol, propranolol and timolol.
Beta-blockers work by blocking the transmission of certain nerve impulses.
The ends of some nerves release a chemical (neurotransmitter) called
noradrenaline when the nerve is stimulated. This chemical then stimulates
beta-adrenergic receptors. These receptors are tiny structures which occur on
cells in various parts of the body including the heart, brain, and blood
vessels. When these receptors are stimulated, they cause various effects. For
example, nerve impulses to the heart can stimulate beta-adrenergic receptors
on heart cells. This causes an increase in the force and rate of the heartbeat.
The beta-adrenergic receptors are also stimulated by adrenaline

(epinephrine), a hormone which circulates in the bloodstream. Adrenaline is


made in the adrenal gland.
The beta-blocker medicine 'sits' on beta-adrenergic receptors and stops
(blocks) the receptor from being stimulated. So, for example, if betaadrenergic receptors in the heart are blocked, the force and rate of the
heartbeat are reduced.
Indications :

prevent angina pains

reduce risk of further MI

control certain arrythmias

treat heart failure

lower bkood pressure

glucoma

anxiety

overactive thyroid gland

migraine
Common side effects include :
Blurred vision
cold hands or feet
confusion
difficult or labored breathing
dizziness, faintness, or lightheadedness
shortness of breath
sweating
tightness in chest
unusual tiredness or weakness
wheezing
Contraindications :

asthma

uncontrolled heart failure

bradycardia

hypotension

certain problems with rythm of heart


patient.co.uk and mayo clinic)

(source:

877. A 15yo boy who complains of pain in his leg which has settled with
aspirin. What is the most probable dx?
a. Leiomyosarcoma
b. Liposarcoma
c. Painful hip
d. Exostosis
e.
Osteoid osteoma
Key : e
Clincher : age and pain settling with aspirin
Leiomyosarcoma is rare occurs over 50yrs of age
Liposarcoma occurs in adults
Exostosis is bone growth commonly in ribs and is not relieved by aspirin
OSTEOID OSTEOMA

Benign bone forming tumors in children particularly adolescents

has a characteristic lucent nidus <2 cm and surrounding solid


periosteal reaction

tumor cells proliferate rapidly producing high amounts of PGE2

it is a very vascular tumor, allowing drugs to penetrate

aspirin inhibits PGE2 synthesis providing analgesia


878. A 20yo fit man suddenly developed severe lower back pain as
getting up from bed. What is the single most probable dx?
a. Pagets disease
b. Multiple myeloma
c.
PID
d. AS
e. Spondylosis
Key : c
Clincher : age and sudden onset of pain
In pagets disease the bone forming process becomes fast and out of control.
Commonly, there are no symptoms and it is diagnosed by chance when x ray
is done for another reason.
Myeloma is a cancer that affects cells in the bone marrow, called plasma

cells. As the cancerous plasma cells fill the bone marrow, you are not able to
make enough normal blood cells. The symptoms and problems which
develop are mainly due to the uncontrolled production of plasma cells in the
bone marrow, and the excess amount of antibody (paraprotein) that the
plasma cells make.
AS is a persistent (chronic) arthritic (rheumatic) disease of unknown cause.
It mainly affects the spine and the sacroiliac joints. The main symptom is
back pain. The pain usually starts in the lower back. You may think of it as
just mild backache at first. It typically becomes gradually worse over several
months.
Spondylosis literally means stiffening or fixation of the bony building blocks
of the spine (vertebrae) as the result of a disease process. Spondylosis refers
to degenerative changes in the spine such as bone spurs and degenerating
intervertebral discs. Many people with spondylosis on X-ray do not have any
symptoms. In fact, lumbar spondylosis (spondylosis in the low back) is
present in 27%-37% of people without symptoms.
PID ( prolapsed intervertebral disc) (source : patient.co.uk)
A 'slipped' (prolapsed) disc often causes severe lower back pain. The disc
often presses on a nerve root which can cause pain and other symptoms in a
leg. In most cases, the symptoms ease off gradually over several weeks.
Bouts of back pain are very common. However, less than 1 in 20 cases of
sudden-onset (acute) back pain are due to a 'slipped' (prolapsed) disc. The
most common age to develop a prolapsed disc is between 30 and 50 years.
Twice as many men as women are affected.
symptoms of a prolapsed disc
Back pain : The pain is often severe and usually comes on suddenly. The
pain is usually eased by lying down flat and is often made worse if you
move your back, cough or sneeze.
Nerve root pain (usually sciatica) : Nerve root pain is pain that occurs
because a nerve coming from the spinal cord is pressed on (trapped) by a
'slipped' (prolapsed) disc, or is irritated by the inflammation caused by the
prolapsed disc.
The irritation or pressure on the nerve next to the spine may also cause pins
and needles, numbness or weakness in part of a buttock, leg or foot. The
exact site and type of symptoms depend on which nerve is affected.

Your doctor will normally be able to diagnose a 'slipped' (prolapsed) disc


from the symptoms and by examining you. (It is the common cause of
sudden back pain with nerve root symptoms.) In most cases, no tests are
needed, as the symptoms often settle within a few weeks. Tests such as Xrays or scans may be advised if symptoms persist. In particular, an MRI scan
can show the site and size of a prolapsed disc

TREATMENT :

Exercise and keep going

Painkillers, if needed should be taken regularly

Some people visit a physiotherapist, chiropractor or osteopath


for manipulation and/or other physical treatments.

surgery may be considered if the symptoms have not settled


after about six weeks or so.

879. A 60yo man brought to the ED with fx hip, he is deaf and has
bilateral pedal edema. What is the single most probable dx?
a.
Pagets disease
b. Osteoporotic fx vertebra
c. Secondary
d. Multiple myeloma
e. Spondylosis
Key : a
Clincher : bilateral pedal edema and deafness
Osteoporosis, multiple myeloma and spondylosis do not cause pedal edema
as their complication.
PAGET'S DISEASE

In Paget's disease, the bone-making process (bone turnover) becomes faster


and out of control. Affected areas of bone form new bone material in an
abnormal way. The newly formed bone is thicker than normal and the bone
may become wider. However, the bone is not made properly and it is weaker
than normal bone. This can cause deformity of the affected bone,
particularly in weight-bearing bones such as the leg bones, which may bend.
Affected bone is also more likely to break (fracture).
In affected areas of bone there are abnormal osteoclasts which are bigger
than normal and more in number. They dissolve more than normal bone. As
a response to this, the osteoblasts increase in activity to make new bone
material. But this increase in bone turnover leads to badly structured areas of
bone that are wrongly woven.
It mainly affects people aged over 50. The disease affects three men for
every two women.
Symptoms :
One or more bones may be affected. The bones most commonly affected are:
The pelvis
The thigh bone in the upper leg (femur)
The bones (vertebrae) of the spine
The skull
The shin bone (tibia).
In many cases there are no symptoms
More than 9 in 10 people with Paget's disease have no symptoms.
The disease is commonly found by chance when an X-ray is taken for
another reason.
Pain
Deformity
Fractures
(Affected bones are more liable to break (fracture). A fracture after a minor
fall or injury may be the first indication that Paget's disease has developed.)
Nerve compression
Joint inflammation (arthritis)
Complications :

Heart failure
Vascular steal syndrome
(If the skull is extensively affected, the extra blood flow needed to supply
the enlarged skull may be 'stolen' from the blood supply going to the brain.
This may result in you becoming tired and listless. )
Bone cancer
Hypercalcemia
Gout
Investigations :
Xray
Bone scan
Blood tests ( alkaline phosphatase rise with increase in bone turnover)
Biopsy, if the diagnosis is uncertain
Treatment :
No treatment is required if pt. is asymptomatic
Treatment is advised when there is pain, bone deformity or complication
Treatment is also advised if paget's disease is in a site where it may cause
problem. (At base of the skull it can compress the
Ear nerve and lead to deafness)
Bisphosphonates given for several weeks or months reduce the abnormal
bone turnover
Painkillers
Calcium and vitamin D
Walking sticks and shoe raises
Calcitonin
Surgery
880. An 80yo lady presents with pain on left 6 rib for a week. It is nontender on examination. What is your most likely dx?
a. Herpes zoster
b. Costochondritis
c.
Bone degeneration
d. Thoracic vertebra compression
th

Key : c
Clincher : age and non tender
Herpes zoster ... there's no vesicular eruption that occurs in 2-3days
Costochondritis .... it is tender to touch
Thoracic vertebra compression ... this will cause limb weakness and bowel
involvement
So in an 80yrs old its common to have bone degenerative disease, which
includes both osteopenia and osteoporosis.
Both men and women start having age-related bone loss at about age 50, but
bone loss can be accelerated in individuals who didn't develop maximum
peak bone mass. There are two levels of bone loss that can occur and are
associated with an increased risk for fractures:
Osteopenia: Osteopenia, which is loss of bone mineral density (BMD), is the
warning siren that the bones are thinning. This phase begins when existing
bone breaks down faster than the body can replace it. Preventing transition
into osteoporosis takes the combination of exercise, calcium, and possibly
some medications prescribed by your doctor.
Osteoporosis: Osteoporosis Latin for porous bone takes years to
develop as bones slowly lose minerals, density, and structure, which makes
them weaker. If left untreated, osteoporosis can lead to stooped posture, loss
of height, and broken bones.The good news is that not everyone ends up
with osteoporosis and there are tests to determine how dense your bones are.
Women's bones are smaller and less dense than men's, and women are four
times more likely than men to suffer from osteoporosis. This is because men
in their 50s don't experience the rapid loss of bone mass that women do in
the years following menopause. By age 65 or 70, however, men and women
are losing bone mass at the same rate, and the absorption of calcium
decreases in both sexes. Excessive bone loss causes bone to become fragile
and more likely to fracture.
881. A 68yo DM, HTN with a 45pack/year smoking hx, has left sided
chest pain increased with breathing. Exam: myosis on left side and wasting
of small muscles of left hand. What is the single most appropriate dx?
a. Costochondritis
b.
Lung cancer

c.
d.
e.

Good pastures syndrome


MND
Progressive massive fibrosis

Key : b
Clincher : age, long h/o smoking, myosis, wasting of small muscles of hand
Costochondritis ... chest pain is close to sternum mostly 4,5,6th ribs. There is
tenderness
Good pasture's syndrome ... Goodpasture's Syndrome is an uncommon
autoimmune disease that affects both the kidneys and the lungs. Doesn't
cause myosis or wasting of muscles
MND ... Motor neurone disease (MND) causes a progressive weakness of
many of the muscles in the body.
The main types of MND are:
Amyotrophic lateral sclerosis (ALS), Progressive bulbar palsy (PBP),
Progressive muscular atrophy (PMA) &
Primary lateral sclerosis (PLS).
PANCOAST TUMOR (Lung cancer)
Classically caused by an apical (superior pulmonary sulcus) malignant
neoplasm of the lung. The neoplasm is usually bronchogenic in origin (most
commonly squamous cell carcinoma, sometimes adenocarcinoma and largecell carcinoma).
This syndrome results from the invasion of a number of structures and
tissues around the thoracic inlet and may be characterised by:An ipsilateral
invasion of the cervical sympathetic plexus leading to Horner's syndrome
(miosis, enophthalmos, ptosis; in 14-50% of patients). Ipsilateral reflex
sympathetic dystrophy may occur.Shoulder and arm pain (brachial plexus
invasion C8-T2) leading to wasting of the intrinsic hand muscles and
paraesthesiae in the medial side of the arm.Less commonly, unilateral
recurrent laryngeal nerve palsy producing unilateral vocal cord paralysis
(hoarse voice bovine cough), and/or phrenic nerve involvement.There may
be arm oedema secondary to the compression of blood vessels.Superior vena
cava syndrome may also occur.
This is similar to other lung cancer emphasising imaging, including CXR
and CT scan of the lungs and abdomen, and also possibly positron emission

tomography (PET). MRI is the imaging of choice to assess structures at the


thoracic inlet prior to surgery. Brain CT or MRI are required, as it is the
most common site of metastases. Good biopsy results are usually achieved
by percutaneous methods.
Originally, Pancoast's tumour was fatal due to involvement of vital
structures at the thoracic inlet. This has improved with multimodality
treatment, including induction chemoradiotherapy (usually cisplatin-based)
followed by resection. Resection may involve a wedge resection or a
lobectomy.
882. A 34yo man had a cold 2d back. He now presents with right sided
facial pain. What is the single most likely dx?
a.
Maxillary sinus
b. Ethmoid sinus
c. Septal hematoma
d. Septal abscess
e. Allergic rhinitis
Key : a
Clincher : age, h/o cold, facial pain
Ethmoid sinus ... it is more common in children
Septal hematoma ... collection of blood within the septum of the nose, it
doesnt cause facial pain
Septal abcess ... a serious condition that is caused by bacteria. Trauma to the
nose, or even nasal surgery, can leave the patient prone to develop a nasal
abscess, which is basically a pocket filled with blood (haematoma) which
has become affected by bacteria. Symptoms include nasal blockage, pain,
redness over the nasal bridge, difficulty in breathing and fever.
Allergic rhinitis ... doesnt cause facial pain
MAXILLARY SINUS (source : patient.co.uk)
The sinuses are small, air-filled spaces inside the cheekbones and forehead.
They make some mucus which drains into the nose through small channels.
Sinusitis means inflammation of a sinus. Most bouts of sinusitis are caused
by an infection. The cheekbone (maxillary) sinuses are the most commonly
affected.
Acute sinusitis means that the infection develops quickly (over a few days)

and lasts a short time. Many cases of acute sinusitis last a week or so but it is
not unusual for it to last 2-3 weeks.
Chronic sinusitis means that a sinusitis becomes persistent and lasts for
longer than 12 weeks. Chronic sinusitis is uncommon.
Risk factors :
Cold/flu
Dental infection
Allergic rhinitis
Nasal polyp
Facial injury or surgery
Foreign body
Smoking
Asthma
Pregnancy
Poor immune system
Symptoms :
The most typical symptoms are headache and pain in the face, with the latter
increasing when the patient bends down, lifts something, coughs etc., that is,
when pressure increases in the sinuses. In case of acute maxillary sinusitis
pain is much stronger than in the case of the chronic type.
Blocked/runny nose
Toothache
Fever
Headache
Cough
Tiredness
Treatment :
Antibiotics are needed only :If your symptoms are severe or if you are very
unwell.If you have another illness such as cystic fibrosis, heart problems or a
weakened immune system.If your symptoms are not settling within seven
days, or are worsening.
Treatment to relieve symptoms :
Painkillers
Decongestant nasal spray or drop
Keeping hydrated

Warm face packs


Saline nasal drops
Steam inhalation
883. A 29yo man with hx of asthma comes with post nasal discharge and
bilateral painless nasal blockage. What is the single most likely dx?
a.
Nasal polyp
b. Septal hematoma
c. Septal abscess
d. Atopic rhinitis
e. Allergic rhinitis
Key : a
Clincher : h/o asthma
Septal hematoma and septal abcess are not associated with asthma
Atopic and allergic rhinitis will have watery discharge but no PND
NASAL POLY (source:patient.co.uk)
Nasal polyps are soft fleshy swellings that grow inside the nose. They may
be yellowish, grey or pink in colour. They are common and non-cancerous
(benign). Nasal polyps can vary greatly in size. There may be only one but
sometimes several grow like a small bunch of grapes on a stem.
Certain conditions make nose inflammation and polyps more likely. These
include asthma, an allergy to aspirin, cystic fibrosis and some rare conditions
of the nose (such as allergic fungal sinusitis and Churg-Strauss syndrome).
Symptoms :
blocked feeling in the nose.
difficult to breathe through your nose.
Watering from the nose (rhinorrhoea) is common.
A postnasal drip may occur.
An ENT surgeon can usually diagnose nasal polyps based on your symptoms
and on examination of your nose.
Occasionally a CT scan or MRI scan may be needed. These scans may show

more detail about where the polyps are and what effects they might have had
on other parts of the face, sinuses and skull.
Treatment :
Steroid nose drops
Steroids tablets
Surgery ( polypectomy / endoscopic sinus surgery)
884. A 24yo man has been found unconscious in an alleyway with a
RR=6bpm and HR=60 bpm. His pupils are constricted. What is the best tx?
a. Methadone
b.
Naloxone
c. Naltrexone
d. Thiamine
e. Glucose
Key : b
Clincher : miosis, respiratory depression
Methadone is used for severe refractory pain
Naltrexone is used for opioid dependence
Thiamine deficiency, which is found in a large number of alcoholics, is an
important contributor to alcoholrelated brain damage of all kinds.
Depending on the level of thiamine deficiency, symptoms can vary greatly.
There are two primary types of thiamine deficiency: wet beriberi and dry
beriberi.
Glucose is given in hypoglycemia which will not present with constricted
pupils
OPIOID POISONING ( morphine toxicity here )
Opiate poisoning can occur at any time from birth (when pethidine given to
the mother in labour may suppress ventilation) to terminal care. The
outcome can range from minor adverse effects such as constipation to death
from respiratory depression.
Symptoms
Opiate poisoning may be a chronic problem, in which case the main
complaint will be of constipation. There may be nausea, vomiting or just

loss of appetite. There may be sedation and craving for the next dose.Acute
toxicity presents with drowsiness that will be more severe if there is also
alcohol involved, or involvement of other sedatives. There may be nausea or
vomiting.
Signs
Respiratory depression may be apparent. Hypotension and tachycardia are
possible. There are usually pinpoint pupils but this sign may be absent if
other drugs are involved.The 'post-mortem sole incision' sign has been
identified. This is an incision made in the sole by an acquaintance in the
belief that the subsequent blood loss will reduce the likelihood of death in an
individual who has taken an accidental overdose of an opiate.
Management
Do not delay establishing a clear airway, adequate ventilation and
oxygenation if consciousness is impaired.
Give naloxone intravenously (IV) (0.4-2 mg for an adult and 0.01 mg/kg
body weight for children) if coma or respiratory depression is present.
Give intramuscularly (IM) if no vein is available. Repeat the dose if there is
no response within two minutes. Naloxone is a competitive antagonist and
large doses (4 mg) may be required in a severely poisoned patient.
885. A 23yo female presents with back pain and early morning stiffness,
also complaining of eye problem and her sister has a similar condition. What
is the single most probable dx?
a. Pagets disease
b. PID
c. Myofacial pain
d.
AS
e. Spondylosis
Key : d
Clincher : back pain, early morning stiffness, eye problem
Paget's disease ... pain is mostly worse at night
PID ... no stiffness
Myofascial pain ... Myofascial pain syndrome is a chronic pain disorder. In
myofascial pain syndrome, pressure on sensitive points in your muscles

(trigger points) causes pain in seemingly unrelated parts of your body. This
is called referred pain.
Spondylosis ... Spondylosis literally means stiffening or fixation of the
vertebrae as the result of a disease process. There is no treatment to reverse
the process of spondylosis, because it is a degenerative process. The
treatments of spondylosis target the back pain and neck pain that spondylosis
can cause.
ANKYLOSING SPONDYLITIS
AS is a persistent (chronic) arthritic (rheumatic) disease of unknown cause.
It mainly affects the spine and the sacroiliac joints. Sometimes other joints
and other parts of the body are affected.
AS most commonly begins between 20 and 30 years of age, but sometimes
first develops in children and older adults. It is three times more common in
men than in women. There may be a family history with two or more
members of a family being affected.
There is a strong hereditary (genetic) tendency. For example, there is a
strong association with a gene called HLA-B27.
Symptoms :
Back pain
Rest does not make it better. In fact, the pain may wake you from sleep.
Instead, exercise and movement usually ease the pain. The pain tends to be
worse first thing in the morning. Lying in bed after waking is often
uncomfortable. The pain tends to ease as the day goes on.
Stiffness in the lower spine
Inflammation of tendons and ligaments
Inflammation of part of the eye (uveitis)
Until recently, the X-ray changes were the only way to confidently confirm
AS. More recently, an MRI scan of the sacroiliac joints has been used to
confirm the diagnosis at an earlier stage. An MRI scan can give a much
more detailed view of a joint than a traditional X-ray picture and can detect
inflammation in the sacroiliac joints.
Symptoms can vary in severity and usually wax and wane. Flare-ups of
inflammation which cause periods of worse pain and stiffness tend to occur
from time to time. If joints outside your spine are affected, they tend to flare

up at the same time as back symptoms. The number of flare-ups that occur,
how severe they are, and how long they last, can vary greatly from person to
person.
886. A 63yo female with a hx of osteoporosis suddenly falls on her
outstretched hand while shopping. XR shows fx at distal radius with
backward shift of the distal fragment. What is the single most probable
deformity?
a.
Dinner fork deformity
b. Coxavara
c. Mallet finger
d. Cubitus valgus
e. Garden spade deformity
Key : a
Clincher : backward shift of distal fragment
Coxavara ... a deformity of the hip
Mallet finger ... an injury to the thin tendon that straightens the end joint of a
finger or thumb, also known as "baseball finger".
Cubitus valgus ... a deformity in which the forearm is angled away from the
body to a greater degree than normal when fully extended.
Garden spade deformity ... fracture at distal radius with forward shift of the
distal fragment
Dinner fork deformity
Fracture at distal radius with backward shift of the distal fragment caused
due to sudden fall on an outstretched hand.
887. A 60yo man presents with severe colicky pain from his right flank
radiating to his groin. His urinalysis reveals trace blood cells. What is the
single most discriminatory inv?
a.
US abdomen
b. XR KUB
c. Colonoscopy
d. Upper GI endoscopy
e. Laproscopy

Key : a ... corrected : b


Clincher : severe colicky pain, flank pain, traces blood cells in urinalysis
Although xray KUB does not show uric acid stones. Around 80% of renal
stones are calcium and are seen on xrays. Plus, in this patient, pain is
radiating from loin to groin as in uretric stones which are not visualised by
US. Colicky pain, flank pain and blood cells in urine point towards renal
pathology. When a renal pathology is suspected, US KUB is done, not US
abdomen.
Colonoscopy, upper GI endoscopy and laproscopy have different
indications.
888. A 45yo man has been admitted for an elective hernia surgery. 3d
later he presents with agitation, sweating, aggressiveness, and complains of
seeing snakes on the hosp wall. Chlordiazepoxide has been started for this
pt. What is the most appropriate next step?
a. Diazepam
b. Acamprosate
c. Disulfiram
d.
Thiamine
e. Methadone
Key : d
Clincher : postop patient, agitation, aggressiveness, mental confusion
Diazepam ... indicated in anxiety, acute alcohol withdrawl, muscular spasm,
convulsive disorders
Acamprosate ... for treatment of alcohol dependence
Disulfiram ... an alcohol abuse deterrent
Methadone ... used for severe refractory pain
Thiamine deficiency has been observed in an appreciable portion of post op
patients receiving parenteral alimentation with glucose but without
supplementary thiamine. Thiamine deficiency is of 2 types : wet beriberi and
dry beriberi
The symptoms of dry beriberi are:decreased muscle function, particularly in
the lower legs tingling or loss of feeling in the feet and hands pain mental
confusion difficulty speaking vomiting involuntary eye movement paralysis.

889. A woman with a prv hx of pain at the left wrist following a fall 4m
ago for which she didnt seek any tx now presented with pain in the same
wrist below the thumb and the pain is aggravated whenever she holds her
baby. What is the cause?
a. Fx radial head
b.
Scaphoid fx
c. Carpal tunnel syndrome
d. Colles fx
e. Ulnar fx
Key : b
Clincher : pain below thumb, aggravated by movement.
Fx radial head ... pain and swelling in elbow joint
Carpal tunnel syndrome ... pain, numbness or tingling in wrist joint, forearm
Colles fracture ... is another name of dinner fork deformity.
Ulnar fracture ... a broken ulna will cause instant pain, tenderness, swelling,
painful elbow movement.
SCAPHOID FRACTURE (source : http://www.assh.org/handcare/handarm-injuries/scaphoid-fracture)
The scaphoid is one of eight small bones that make up the carpal bones of
the wrist. It connects two rows of these bones - the proximal row and the
distal row. This connection puts it at extra risk for injury.
A fracture of the scaphoid bone usually occurs from a fall onto the
outstretched hand.
Pain (with or without swelling or bruising at the thumb side of the wrist)
can be noticed within days following a fall.
Scaphoid fractures are usually diagnosed by an x-ray of the wrist; however,
x-rays do not always show scaphoid fractures.
If the fracture is non-displaced (bone has not moved out of place at the
fracture), it usually can be successfully treated with a cast. Although the
fracture may heal in as little as six weeks, it may take longer for some
patients. If the fracture is in a certain part of the bone or if the fracture is at
all displaced (bone ends have shifted), surgery might be the best option.
890. A 29yo man was involved in an RTA. He presents with distended
neck veins, clear breath sounds and a trachea which is in the midline. His
RR=34bpm, BP=60/0mmHg. What is the most likely dx?

a.
b.
c.
d.

Simple pneumothorax
Tension pneumothorax
Cardiac tamponade
Pericarditis

Key : c
Clincher : RTA, clear breath sounds, trachea in midline
Simple pneumothorax ... respiratory distress, distant or absent breath sounds
Tension pneumothorax ... absent breath sounds, trachea shifts away from
affected side
Pericarditis ... pain and high grade fever
CARDIAC TEMPONADE (source : patient.co.uk)
Cardiac tamponade is caused by the accumulation of blood, fluid, pus, clots,
or gas in the pericardial space, resulting in reduced ventricular filling and
subsequent haemodynamic compromise. Cardiac tamponade is a medical
emergency.
Cardiac tamponade related to trauma or HIV is more common in young
adults. Tamponade due to malignancy and/or chronic kidney injury occurs
more frequently in elderly individuals.
Signs
Distended neck veins, hypotension, tachycardia, tachypnoea and
hepatomegaly.Muffled heart sounds. Increased jugular venous pressure
(JVP). Cyanosis and pulmonary oedema may occur.
Transthoracic echocardiogram is the investigation of choice.
Patients should be monitored in an intensive care unit. Pericardiocentesis
(echocardiography-guided being the procedure of choice) is the definitive
treatment but may be hazardous and not relieve symptoms in cases of small
effusions associated with constrictive pericarditis - eg, malignancy,
autoimmune conditions and viral infection.Oxygen.Volume expansion to
maintain adequate intravascular volume - small boluses work best.[7]
Improve venous return: bed rest with leg elevation.Positive inotropic drugs:
eg, dobutamine.

891. An elderly woman is found anemia. As part of her exam, she had a
barium enema which reveals a mass lesion in the ascending colon. What is
the single most appropriate dx?
a. Sigmoid volvulus
b. Anal fissure
c. Sigmoid carcinoma
d.
Cecal carcinoma
e. Diverticular disease
Key : d
Clincher :
Sigmoid volvulus ... more common in men, presents with sudden colicky
lower abdominal pain with distension
Anal fissure ... An anal fissure is a small tear of the skin around the anus.
Often an anal fissure will bleed a little. You may notice blood after you pass
faeces. The blood is usually bright red, and a small amount may be seen on
the toilet paper or coating your faeces.
Sigmoid carcinoma ... mass lesion would have been on left side.
Diverticular disease ... lower abdominal pain, usually left-sided.
COLON CANCER
Cancer of colon and rectum can exhibit itself in several ways. If you have
any of these symptoms, seek immediate medical help. You may notice
bleeding from your rectum or blood mixed with your stool.People
commonly attribute all rectal bleeding to hemorrhoids, thus preventing early
diagnosis owing to lack of concern over "bleeding hemorrhoids." New onset
of bright red blood in the stool always deserves an evaluation. Blood in the
stool may be less evident, and is sometimes invisible, or causes a black or
tarry stool.Rectal bleeding may be hidden and chronic and may only show
up as an iron deficiency anemia.It may be associated with fatigue and pale
skin due to the anemia.
If the tumor gets large enough, it may completely or partially block your
colon. You may notice the following symptoms of bowel obstruction:
Abdominal distension
Abdominal pain is rare in colon cancer.
persistent nausea or vomiting
Unexplained weight loss

Change in frequency or character of stool. Small-caliber (narrow) or ribbonlike stools


Sensation of incomplete evacuation after a bowel movement
Rectal pain: Pain rarely occurs with colon cancer and usually indicates a
bulky tumor in the rectum that may invade surrounding tissue.
892. A 55yo male after gastrectomy developed anemia. His MCV=106fl.
Exam: loss of proprioception and vibration sense. What is the most likely
dx?
a. IDA
b. Folate def
c.
Vit B12 def
d. Anemia of chronic disease
Key : c
Clincher : gastrectomy, anemia, mcv, loss of proprioception and vibration
sense
IDA ... mcv is decreased
Anemia of ch. Disease ... mcv is decreased
Folate def. Anemia ... mcv is increased but it doesn't affect proprioception
and vibration sense
VITAMIN B12 DEFICIENCY ANEMIA (source : patient.co.uk)
Vitamin B12 is present in meat and animal protein foods. Absorption occurs
in the terminal ileum and requires intrinsic factor (IF), a secretion of gastric
mucosal (parietal) cells, for transport across the intestinal mucosa. In
pernicious anaemia, IF production is deficient. It is believed to be an
autoimmune disease.Helicobacter pylori infection has been mooted to be an
initiating factor, with subsequent autoimmune changes affecting the gastric
mucosa. Genetic susceptibility to this process has been suspected.[1]
Causes
Pernicious anaemia accounts for 80% of cases of megaloblastic anaemia due
to impaired absorption of vitamin B12.
Other causes of vitamin B12 deficiency include:
Gastric causes: gastrectomy, gastric resection, atrophic gastritis, H. pylori
infection or congenital IF deficiency or abnormality.I
Inadequate dietary intake of vitamin B12 - eg, a vegan diet.

Intestinal causes - eg, malabsorption, ileal resection, Crohn's disease


affecting the ileum, chronic tropical sprue, HIV and any radiotherapy
causing irradiation of the ileum.
Drugs - eg, colchicine, neomycin, metformin, anticonvulsants.
Long-term use of drugs that affect gastric acid production (eg, H2 receptor
antagonists, proton pump inhibitors) can worsen deficiency because gastric
acid is needed to release vitamin B12 bound to proteins in food.
Presentation
Symptoms of anaemia may include fatigue and lethargy, dyspnoea,
faintness, palpitations and headache. Vitamin B12 deficiency may present
with unexplained neurological symptoms - eg, paraesthesia, numbness,
cognitive changes or visual disturbance.
Peripheral loss of vibratory sense and position are early indications of
central nervous system (CNS) involvement, accompanied by reflex loss and
mild-to-moderate weakness. Later stages may be characterised by spasticity,
Babinski's responses and ataxia.
Management
For patients with neurological involvement, referral to a haematologist is
recommended. Initial treatment is with hydroxocobalamin 1 mg on alternate
days until there is no further improvement, after which 1 mg should be given
every two months for life.
893. A 26yo male has been operated for abdominal trauma and
splenectomy was done. On the 3 post-op day the pt developed acute
abdominal pain and distention in the upper abdominal area with
hypotension. On insertion of ryle's tubes, 2L of coffee ground fluid was
aspirated. What is the most probable dx?
a.
Acute gastric dilatation
b. Reactionary hemorrhage
c. Subphrenic abscess
d. DVT
e. Left lower lobe atelectasis
rd

Key : a
Clincher : splenectomy, 3rd postop day, upper abdominal distension,
Reactionary hemorrhage ... occurs during first 48hrs

Subphrenic abscess ... takes longer to develop


DVT ... causes pain in leg normally
Left lower lobe atelectasis ... can occur secondary to any surgery but does
not cause upper abdominal distention
ACUTE GASTRIC DILATATION is one of the post splenectomy
complications which if not treated promptly leads to death.
894. A 50yo man presented with increased breathlessness at rest. He is
currently on furosemide, digoxin and isosorbide mononitrate. What drug is
going to help him?
a. Ramipril
b.
Bendroflumethiazide
c. Atenolol
d. Amlodipine
e. Diltiazem
Key : b
Clincher : increased breathlessness at rest, on furosemide already
Ramipril ... ACE inhibitor to treat high blood pressure decreasing the risk of
heart attack and stroke
Atenolol ... beta blocker to treat angina, hypertension and prevent heart
attack
Amlodipine ... calcium channel blocker used to treat hypertension, angina
and other conditions caused by coronary artery disease
Diltiazem ... calcium channel blocker used to treat hypertension, angina and
certain heart rhythm disorders.
895. A 31yo man underwent an operation where his hand was hanging
outside the table. After the operation he had wrist drop and sensory loss over
the dorsum of his hand. Which nerve was injured?
a.
Radial
b. Ulnar
c. Median
d. Axillary
e. Brachial
Key : a
Clincher : wrist drop, sensory loss over dorsum of hand

The radial nerve and its branches provide motor innervation to the dorsal
arm muscles (the triceps brachii and the anconeus) and the extrinsic
extensors of the wrists and hands; it also provides cutaneous sensory
innervation to most of the back of the hand.
896. What is the mode of spread of chickenpox?
a.
Airborne
b. Close contact
c. Fecal-oral
d. Blood
e. Vector
897. A 64yo man presents with ipsilateral vertigo, tinnitus and left side
hearing loss. Exam: Renne test +ve and Webers lateralizes to the right ear.
What is the most appropriate inv?
a. CT
b.
MRI brain
c. XR
d. Audiometry
e. None
f.
Caloric testing
Key : b
Clincher : vertigo, tinnitus, hearing loss
MRI has largely superseded CT scanning as the investigation of choice for
suspected acoustic neuroma. (Patient.co.uk)
ACOUSTIC NEUROMA (source : nhs )
An acoustic neuroma is a benign (non-cancerous) growth, or tumour, in the
brain. It's also known as a vestibular schwannoma.An acoustic neuroma
grows on the vestibulocochlear nerve, which helps control hearing and
balance. This nerve runs alongside the facial nerve, which carries
information from the brain to the face muscles.
A small acoustic neuroma can lead to hearing loss or tinnitus, vertigo
A large acoustic neuroma can cause headaches with blurred vision,

numbness or pain on one side of the face, problems with limb co-ordination
on one side of the body.
The cause of most acoustic neuromas is unknown, but a small number of
cases (about 5%) are caused by a rare, inherited condition called
neurofibromatosis type 2.Acoustic neuromas grow from the cells that cover
the vestibulocochlear nerve, called Schwann cells.
The three types of test you may have are :
Hearing tests, MRI, CT
Acoustic neuromas tend to grow slowly and don't spread to other parts of the
brain.They can be so small and grow so slowly that they may not cause any
symptoms or problems. In such cases, the acoustic neuroma may just be
monitored to avoid risks associated with surgery.In rare cases, the tumour
can grow large enough to press on the brain. However, most acoustic
neuromas can be treated before they reach this stage, either with surgery to
remove the tumour or radiotherapy to destroy it.
898. A 67yo man presents to the ED with pain in his left groin. He
suddenly collapses and his is not able to move or lift his leg. He is on
alendronate. What is the dx?
a.
Fx of neck of femur
b. Post hip dislocation
c. Fx of shaft of femur
d. Pelvic base fx
e. Peripheral vascular disease
Key : a
Clincher : age, pain in groin, unable to move leg, on alendronate
Alendronate being a bisphosphonate is used in osteoporosis in which
fracture of neck of femur is very common.
899. A young male met with a RTA and is suspected to have a femur fx.
His BP is 90/60mmHg. What is the next immediate action?
a. XR
b. IV fluids
c.
Put leg splint
d. Send bloods for inv

e.

US

Key : c
Clincher : RTA, femur fracture, low BP
In case of traumatic hemorrhagic shock, primary goal is to stop bleeding.
Shaft of femur fractures (source : patient.co.uk)
These are caused by a high-energy injury, such as road traffic accidents,
unless pathological fracture in a patient with osteoporosis or metastatic
disease.There are often associated injuries to the hip, pelvis, knee and other
parts of the body.
Diagnosis
Deformity, shortening, external rotation and abduction at the hip on the
affected side.
Management
Initial management:
Assess vital functions and any associated injuries.
Resuscitate and treat life-threatening injuries as necessary.
Splint fractures (Thomas' splint or equivalent traction splint)
X-rays of the femur.
Blood tests, including blood for cross-matching.
Obtain intravenous access and start fluid replacement.
Peripheral sensation and pulses should be closely monitored.
Analgesia: adequate intravenous analgesia. Femoral nerve block is usually
effective.
900. A 70yo pt presents with cough and SOB. He stopped smoking
cigarettes 2yrs ago but has a 50yr smoking hx before quitting.
CXR=consolidation and bilateral hilar lymphadenopathy. What is the best
inv for this pt?
a.
LN biopsy
b. Pleural fluid cytology
c. CT
d. MRI

e.

US

Key : a
Clincher : age, long h/o smoking, cough n SOB, consolidation and bihilar
lymphadenopathy
Pleural fluid cytology ... only gives suspicion of malignancy
CT ... not diagnostic but helps in staging
MRI ... CT is superior
US ... US guided biopsy is done for definitive diagnosis
LN biopsy ... done for definitive diagnosis
If the CT scan shows the cancer is at an early stage and you are fit to be
treated you may be asked to have another type of scan. This is called a
positron emission tomography-CT (PET-CT) scan. This shows up areas of
active cancer and whether it has spread to the lymph glands in the chest. If
the cancer has spread to these glands, you will be offered a biopsy. (Source:
patient.co.uk
901. A 27yo pt met with a RTA. While the NGT is passing, bowel sounds
are heard in the chest. CXR shows NGT curled. What is the dx?
a.
Diaphragm rupture
b. Aortic rupture
c. Splenic rupture
d. Bowel rupture
e. Liver rupture
Key : a
Clincher : RTA, bowel sounds in chest, NGT curled
An acquired diaphragmatic hernia (ADH) is usually the result of a blunt or
penetrating injury. Traffic accidents and falls cause the majority of blunt
injuries.
Symptoms :
Difficulty Breathing
Blue Discoloration of the Skin
Tachypnea
Tachycardia

Diminished or Absent Breath Sounds


Bowel Sounds in the Chest Area
Less-Full Abdomen
Diagnosis :
X-ray
ultrasound scan
CT scan
arterial blood gases (ABG)
902. A 62yo man dx with T2DM with BMI=33. Lifestyle modifications
have failed to control blood sugar. Labs: urea=3.6mmol/l,
creatinine=89mmol/l. what is the next appropriate management?
a.
Biguanide
b. Sulfonylurea
c. Insulin
d. Glitazone
e. Sulfonylurea receptor binder
Key : a
Clincher : BMI = 33, failure of lifestlye modifications
If BMI is >25 and life style modifications have failed, start biguanides
If BMI is < 25, start sulfonylurea
903. A pt presents with progressive dyspnea. He complains of cough,
wheeze and a table spoonful of mucopurulent sputum for the last 18m.
Spirometry has been done. FEV1/FVC=2.3/3.6. After taking salbutamol, the
ratio=2.4/3.7. What is the most likely dx?
a.
Chronic bronchitis
b. Asthma
c. Bronchiectasis
d. Lung fibrosis
e. Sarcoidosis
Key : a
Clincher : table spoonful of mucopurulent sputum, FEV1/FVC ratio being
<75%
Asthma presents with dyspnea, wheeze and cough with sputum. FEV1/FVC

ratio is<75%. COPD is more likely to cause an ongoing cough with sputum
than asthma.
Bronchiectasis ... There is copious amount of sputum. FEV1/FVC is being
<75%
Lung fibrosis and sarcoidosis present with dry cough. FEV1/FVC ratio
being >75%
Chronic bronchitis presents with cough, sputum, dypnoea and wheeze.
Shows obstructive pattern in lung function tests with FEV1/FVC ratio being
<70%
904. A 62yo man presents with cough, breathlessness and wheeze. 24%
O2, salbutamol and hydrocortisone were given. The symptoms havent
improved and so nebulized bronchodilator was repeated and IV
aminophylline was given. ABG: pH=7.31, RR=32. What is the next
appropriate management?
a.
Nasal IPPV
b. Intubation and ventilation
c. LABA
d. Toxapram
e. Amoxicillin PO
Key : a
Clincher : failure after IV aminophylline, pH and RR
If there is no response after IV aminophylline, consider nasal IPPV if RR
>30 & pH <735. It is delivered by nasal mask and a flow generator.
Intubation and ventilation is done if nasal IVVP fails.
(Source : OHCM Pg # 823 8th edition)
905. A young girl returns from holidays in Spain. She complains of
discharge from her ear and complains of tragal tenderness. Exam: tympanic
membrane normal. Aural toilet has been done. What is the next appropriate
med?
a. Antibiotic PO
b. Antibiotic IV
c. Steroid PO
d. Steroid drop
e.
Antibiotic drop with steroid

Key : e
Clincher : holiday, tragal tenderness
This girl has recently been on vacations. Swimming can cause acute otitis
externa with normal tympanic membrane, tender tragus. It is treated with
topical antibiotics. Steroids can be added.
906. A 23yo man sprained his right ankle 6wks ago while playing
football. He was tx with a below knee walking cast. On removal of the cast,
the pt noted to have right foot drop. He has weakness of extensors of the
ankle and toes and diminished pin prick sensation over the dorsum of the
foot. The ankle jerk is present and plantar reflex is flexor. What is the most
likely cause of the foot drop?
a.
Compression of common peroneal nerve
b. Compression of the tibial nerve
c. Compression of the S1 nerve root
d. Rupture of Achilles tendom
e. Tx of the medial collateral lig of the ankle
Key : a
Clincher : right foot drop, wkness of extensorsof ankle and toes, diminished
pin prick sensationover the dorsum of foot
Causes of unilateral foot drop (source: OHCM Pg # 471 8th edition)
DM
common peroneal nerve palsy
Stroke
Prolapsed disc
MS
907. A young man was knocked down during a fight in the waiting room
of the ED. He is now unconscious and unresponsive. What is the 1 thing
you would do?
a. Turn pt and put in recovery position
b. Put airway
c. Endotracheal intubation
d. Assess GCS
e. Start CPR
st

Key : b
Clincher : brought to ED unconcious, unresponsive after a fight
ABC
Airway Breathing Circulation
908. A 52yo man underwent a hemicolectomy. After a few days he
complains of left ventricular pain and fever. ECHO has been done and
shows a systolic murmur. What is the next appropriate inv?
a. CT
b. US
c. CXR
d.
Blood culture
e. LFT
909. A 19yo man has exercised induced asthma and is using a salbutamol
inhaler as req and beclomethasone 400ug BD. He complains that he has to
wake up at night for his inhaler. What is the single most appropriate tx?
a. Beclo
b. Regular salbutamol and budesonide
c.
Sodium cromoglycate
d. Oral steroid
e. Inhaled steroid
Key : c
Clincher : night symptoms of asthma
Usually a long acting beta agonist is needed to cover night symptoms but its
not in options. Others dont make much sense. In essence we want
undisturbed sleep. Sodium chromoglycate is the next best option.
910. Pt with a long hx of smoking is now suffering from bronchial ca.
histology reveals there are sheets of large polygonal or giant MNC. What is
the most likely dx?
a. Squamous cell ca
b. Small cell ca
c. Adenocarcinoma
d. Large cell ca
e. Oat cell ca

Key : d
Clincher : hitology reveals sheets of large polygonal or giant MNC
Large cell carcinoma ( lung)
It is a poorly differentiated malignant epithelial tumor. Consists of sheets of
large polygonal or giant MNC. One histological variation is large call
neuroendocrine carcinoma
911. A 27yo man presents with chest pain and respiratory distress. Exam:
tachycardia, hypotension and neck vein distension. Trachea is deviated to
the left side, breathing sounds on right side are absent and diminished on left
side. What is the next appropriate management?
a. CXR
b.
Right side aspiration (16G)
c. Left side aspiration (16G)
d. Right side drain with a small tube (12F)
e. Left side drain with a small tube (12F)
Key : b
Clincher : chest pain, resp.distress, hypotension, neck vein distension,
deviated trachea, absent breath sounds
Its a case of tension pneumothorax
Indications of simple aspiration include : PSP ( any size) , Small SSP in pts.
<50yrs
Needle aspiration (14to16G) is as effective as large bore (>20F) chest drain
and is associated with less hospitalisation and less stay.
Following failed needle aspiration, a small bore (<14F) chest drain insertion
is recommended.
912. A 16wk pregnant pt who was exposed to a child with chickenpox
came to GP for help. She was tested ve for varicella antibody. What is the
next most imp step in management?
a. Reassurance
b.
Ig
c. Ig + vaccine
d. Vaccine only
e. Acyclovir

Key : b
Clincher : 16 wk pregnant, exposed to child with chicken pox, tested -ve for
varicella antibody
Ig + vaccine or only vaccine are contraindicated during pregnancy
Acyclovir ... there are no signs of infection
Ig is given because there is only history of exposure and there are no signs of
infection yet.
913. A 68yo woman dx with T2DM and BMI=33. Lab: GFR=29,
urea=13, creatinine=390mmol/L. what is the next appropriate management?
a. Biguanide
b. Sulfonylurea
c.
Insulin
d. Glitazone
e. Sulfonylurea receptor binder
Key : c
Clincher : BMI = 33, Creatinine = 390 mmol/L
Biguanides ... cant be given if creatinine is > 150mmol/L
Sulfonylurea / sulfonylurea receptor binder ... is given when BMI is < 25
Glitazone ... 2nd line drug which also causes wt.gainby water retention
Insulin indications :
HBA1C > 7.5
DKA
when OHA dont control
Pregnancy
Increased BUN
914. A 5yo boy was brought to GP with high temp and many vesicles on
his back. What is the most appropriate management?
a. Topic acyclovir
b. Oral acyclovir
c. Oral antibiotics
d. Topical steroids
e.
None

Key : e
Clincher : age
no treatment is required in children from 28days old to 12years old in
chicken pox.
915. A woman came with the complaint of pain in her right arm when she
abducts it. She has recently moved to a new house. There is no hx of trauma.
Wht is the likely cause of her pain?
a. Rupture of the long head of biceps
b. Sprain of the acromio-clavicular ligament
c. Tendinitis of the abductor sheat
d.
Supraspinatus tendinitis
e. Shoulder dislocation
Key : d
Clincher : pain in her arm when she abducts it
Supraspinatous muscle initiates abduction of arm upto 15 after which
deltoid continues the abduction.
916. An 83yo man with longstanding COPD has become progressively
breathless over the last 2yrs. He is on salbutamol, ipratropium, salmetarol,
beclomethasone and theophylline. His FEV1<30%. What is the next
appropriate management?
a. Lung transplant
b. Trial of CPAP
c. Trial of non-invasive ventilation
d.
Assessment for long term O2 therapy
e. Short course of O2 therapy
Key : d
Clincher : every option has been tried, still has SOB, FEV1 is less than 30%
Lung transplant has no role in old age
when treatment has failed and patient still has SOB ... long term O2 therapy.
Indications of LTO2T include :
FEV1 less than 30%

polycythemia
cyanosis
periphral edema
raised JVP
O2 saturation less than 92%
917. A 49yo man complains of fullness in his left ear, recurrent vomiting
and tinnitus. What is the most appropriate med?
a.
Buccal prochlorperazine
b. Oral chlorpheniramine
c. Oral flupenphenazine
d. Buccal midazolam
e. IV rantidine
Key : a
Clincher : age, fullness in one ear, vomitting, tinnitus
MENIERES DISEASE (source : nhs)
Its a case of menieres disease.
Mnire's disease is a rare disorder that affects the inner ear. It can cause
vertigo, tinnitus, hearing loss, and a feeling of pressure deep inside the ear.
Mnire's disease often progresses through different stages. In the early
stages, most people have sudden and unpredictable attacks of vertigo,
accompanied by nausea, vomiting and dizziness. During the later stages, the
episodes of vertigo tend to occur less frequently and sometimes stop
altogether over time. However, the tinnitus and hearing loss often become
worse and you may be left with permanent balance and hearing problems.
Mnire's disease most commonly affects people aged 20-60 and it's thought
to be slightly more common in women than men.
During an attack of Mnire's disease, you may be prescribed medication to
treat the symptoms of vertigo, nausea and vomiting.This is usually
prochlorperazine or an antihistamine. If these work, you may be given a
supply to keep, so you can take them quickly during an attack.
If you experience vomiting during your attacks, you can take a type of
prochlorperazine called Buccastem. This comes as a tablet that you place

between your gums and your cheek, on the inside of your mouth. The tablet
dissolves and is absorbed into your body.
918. A man had a soft mass on his mandible. Mass is freely mobile and
has started growing progressively over the past 6m. The mass still moves
freely. What is the best inv for this pt?
a.
FNAC
b. CT
c. XR
d. MRI
e. ESR
Key : a
Clincher : soft freely mobile mass on mandible
Source OHCM Pg 597 8thed.
salivary gland tumours
80% are in the parotid, 80% of these are pleomorphic adenomas, 80% of
these are in the superficial lobe. Deflectionof ear outwards is a classical
signs. Remove any salivary swelling for assessment ifresent for >1month.
7th nerve palsy means malignancy.
Investigations (source patient.co.uk)
Ultrasound is the usual initial means to assess superficial lesions. Ultrasound
is more limited at visualising the deep lobe of the parotid and some minor
salivary glands depending on location.Ultrasound-guided fine-needle
aspiration (FNA) cytology is used to obtain cytological confirmation. CTguided biopsy can also be used.If deep tissue extension is suspected or
malignancy confirmed on cytology, an MRI or CT scan is used to evaluate
tumour bulk, local invasion and perineural spread.
919. A 63yo man has been brought to the hosp after collapsing during a
wedding. His ECG is below. What is the most likely dx?
a. VT
b. A-fib
c.
VF
d. A-flutter

e.

SVT

920. A 75yo war veteran complains of loss of appetite and says he has
lost weight over the past few months. He says that he has passed some blood
in his urine, however, he had no pain. A recent report shows that PSA
>5.5ng/ml. how will you manage this pt?
a.
Radical prostatectomy
b. TURP
c. Cryosurgery
d. Brachytherapy
e. Irradiation
1st choice should be radical prostatectomy as it seems not to be beyond localy advanced disease fro its
presentation. TURP is used to relief pressure symptom which is not yet in this patient and it does not cure
the cancer. For cryosurgery (killing cancer cell by freezing them) it is not widely available and its long-term
effectiveness has not yet been conclusively proven. Irradiation- If you consider surgery don't go for
irradiation first as following irradiation surgical complications are much higher. Brachytherapy is also a kind
of radiotherapy which is given locally (by placing radioactive seeds or needles in prostate).

921. A 19yo boy comes to the ED with pain, swelling and tenderness
2cm distal to Listers tubercle of radius. Exam: proximal pressure on the
extended thumb and index finger is painful. XR: no fx. What is the next
appropriate management for the pt?
a.
Immobilization with cast
b. Repeat XR
c. MRI
d. Surgery
e. None
Key : a
Clincher : swelling and tenderness 2cm distal to lister's tubercle, xray no fx
Scaphoid fracture
Only a few cases ojf scaphoid fracture are seen on xray. Treatment is to
immobilize with a scaphoid cast for 8 to 10days then repeat xray if it still
does not show and symptoms persist, go for a CT scan.
922. A 71yo man with a hx of 50yrs of smoking presents with cough,
hemoptysis, dyspnea and chest pain. He also has anorexia and weight loss.
The dx of lung cancer has been stabilized. Which electrolyte abnormality
can be seen?
a. Hyperkalemia

b.
c.
d.
e.

Hypocalcemia
Hyponatremia
Hypernatremia
Hypomagnesemia

Key : c
Clincher : lung cancer
Syndrome of inappropriate ADH secretion (SIADH)[8][9]
Inappropriate ADH secretion from posterior pituitary or from ectopic source
despite low serum osmolality.
Major diagnostic features
Hyponatraemia.Plasma hypo-osmolality proportional to
hyponatraemia.Inappropriately elevated urine osmolality (>100 mOsmol/kg)
commonly > plasma osmolarity.Persistent urine [Na+] >40 mmol/L with
normal salt intake.Euvolaemia.Normal thyroid and adrenal function.Extra
features include an elevated ADH level and low blood uric acid level.
Causes (not exhaustive)
Neurological: tumour, trauma, infection, Guillain-Barr syndrome, multiple
sclerosis, systemic lupus erythematosus, intracranial haemorrhage, sinus
thrombosis, AIDS, porphyria.Pulmonary: lung small-cell cancer,
mesothelioma, pneumonia, abscess, cystic fibrosis, asthma, tuberculosis,
positive-pressure ventilation.
Other malignancy: oropharyngeal, stomach, pancreas, leukaemia,
lymphoma, thymoma, and genitourinary tract cancers.
Drugs: chlorpropramide, carbamazepine, selective serotonin reuptake
inhibitor (SSRI) antidepressants, tricyclic antidepressants, lithium,
MDMA/ecstasy, tramadol, haloperidol, vincristine, desmopressin,
fluphenazine.
Miscellaneous: idiopathic, hereditary, pain, postoperative, stress, endurance
exercise and marathon runners, dermatomal herpes zoster.
923. A 56yo man who is hypertensive recently underwent a change in
meds. 2days later he developed wheezing. Which drug can cause this?
a.
Atenolol

b.
c.
d.
e.

Ramipril
Bendroflumethiazide
Verapamil
Furosemide

Key : a
Clincher : hypertensive, new meds, wheeze
Beta blockers cause bronchoconstriction leading to wheeze and SOB. it is
contraindicated in asthma.
924. A 33yo man has a temp=38.5C, cough and chest pain on the right
side on inspiration. He also has purulent sputum. What is the most likely
organism to cause pneumonia in this pt?
a. Gram +ve diplococci
b. Coagulase +ve cocci
c. PCP cold agglutinins
d. AFB
e. Gram ve diplococci
Key : c ....... corrected : a
Clincher : no specific pointers here
The most common cause of CAP strept.pneumonia which is a gram +ve
diplococci
925. A young mans arm was caught in a machine. XR showed no fx but arm is very
swollen. What is
the best tx?
a. Plaster cast
b. Wide splint with upward position
c. Analgesics
d. Antibiotics
e. Tetanus prophylaxis
b. Wide splint with upward position
Swollen arm is held high for swelling to settle.
Splint

Noncircumferential
Acute and definitive treatment of select fractures
Soft tissue injuries (sprains, tendons)
Acute management of injuries awaiting orthopedic intervention
Allows for acute swelling

Decreased risk of complications


Faster and easier application
Commercial splints available and appropriate for select injuries
May be static (preventing motion) or dynamic (functional; assisting with controlled
motion)
Lack of compliance
Increased range of motion at injury site
Not useful for definitive care of unstable or potentially unstable fractures

Cast

Circumferential
Definitive management of simple, complex, unstable, or potentially unstable
fractures
Severe, non acute soft tissue injuries unable to be managed with splinting
More effective immobilization
Higher risk of complications
More technically difficult to apply

926. A child was brought in to ED by his parents for taking his grand-dads meds. There
is an extra
systole in the ECG. Which drug was taken?
a. Digoxin
b. Amitriptyline
c. Atenolol
d. Ramipril
e. Bendroflumethiazide
a. Digoxin
In digoxin toxicity, the finding of frequent premature ventricular beats (PVCs) is the most
common and the earliest dysrhythmia. Sinus bradycardia is also very common. In
addition, depressed conduction is a predominant feature of digoxin toxicity. Other ECG
changes that suggest digoxin toxicity include bigeminal and trigeminal rhythms,
ventricular bigeminy, and bidirectional ventricular tachycardia
Features
generally unwell, lethargy, nausea & vomiting, anorexia, confusion, yellow-green
vision
arrhythmias (e.g. AV block, bradycardia)
Management
Digibind
correct arrhythmias
monitor potassium
Amitriptyline
Sinus tachycardia is the most common ECG finding
Widening of the PR, QRS, and QTc intervals; nonspecific ST-segment and T-wave
changes
Atenolol
bradycardia (slow heartbeat), severe hypotension with shock, acute heart failure,
hypoglycemia and bronchospastic reactions.

Ramipril
hypotension, tachycardia, hyperkalaemia, and acute renal failure.

927. A 5yo child came from Ghana 6wks ago. 2d ago he developed fever, vomiting and
neck stiffness.
He had taken malaria prophylaxis and had no rash. What is the dx?
a. Cerebral abscess
b. Cerebral malaria
c. Meningococcal meningitis
d. SAH
e. Cerebral tumor
f. Pneumonia
b. Cerebral malaria
Malaria prophylaxis doesn't provide full protection against all subtypes of malarial
parasites. Secondly meningococcal meningitis is fever with a rash. Here pt has no rash.
abscess have typical swinging fever pattern.
Suspect malaria in any patient has travelled to endemic areas within one year (patient
com). Antimalarial prophylaxis don't give full immunity and infection can be happened
(BNF).
a-e: focal signs
c- rash
d- sudden
f-cough
Consider malaria in every febrile patient returning from a malaria-endemic area within
the last year, especially in the previous three months, regardless of whether they have
taken chemoprophylaxis, as prompt recognition and appropriate treatment will improve
prognosis and prevent deaths.
928. A HTN pt on bendroflumethiazide 2.5mg/d has come for his routine checkup. Exam:
BP=145/85mmHg. Lab: K+=5.9, Na+=137. What is the most appropriate management
for this pt?
a. Stop meds
b. Continue same dose
c. Increase the dose
d. Decrease the dose
e. Repeat the blood test
e. Repeat the blood test
Repeat the test as thiazides cause hypokalemia
Common adverse effects
dehydration
postural hypotension
hyponatraemia, hypokalaemia, hypercalcaemia
gout
impaired glucose tolerance
impotence

929. A 65yo man presents with significant weight loss and complains of cough, SOB and
chest pain. Exam: left pupil constricted, drooping of left eyelid. What is the most likely
dx?
a. Pancoast tumor
b. Thoracic outlet syndrome
c. Cervical rib
d. Pneumonia
e. Bronchogenic ca
a. Pancoast tumor
Classically caused by an apical (superior pulmonary sulcus) malignant neoplasm of the
lung. The neoplasm is usually bronchogenic in origin (most commonly squamous cell
carcinoma, sometimes adenocarcinoma and large-cell carcinoma).
An ipsilateral invasion of the cervical sympathetic plexus leading to Horner's
syndrome (miosis, enophthalmos, ptosis; in 14-50% of patients).
Ipsilateral reflex sympathetic dystrophy may occur.
Shoulder and arm pain (brachial plexus invasion C8-T2) leading to wasting of the
intrinsic hand muscles and paraesthesiae in the medial side of the arm.
Less commonly, unilateral recurrent laryngeal nerve palsy producing unilateral
vocal cord paralysis (hoarse voice bovine cough), and/or phrenic nerve
involvement.
There may be arm oedema secondary to the compression of blood vessels.
930. A 4yo boy presents with fever, sore throat and lymphadenopathy. The dx of
tonsillitis has been made. He had 3 episodes last yr. What is the most appropriate
management for this pt?
a. Tonsillectomy
b. Paracetamol/ibuprofen
c. Oral penicillin V
d. IV penicillin
e. None
b. Paracetamol/ibuprofen
Management
paracetamol or ibuprofen for pain relief
antibiotics are not routinely indicated
NICE indications for antibiotics
features of marked systemic upset secondary to the acute sore throat
unilateral peritonsillitis
a history of rheumatic fever
an increased risk from acute infection (such as a child with diabetes mellitus or
immunodeficiency)
patients with acute sore throat/acute pharyngitis/acute tonsillitis when 3 or more
Centor criteria are present
The Centor criteria* are as follows:
presence of tonsillar exudate
tender anterior cervical lymphadenopathy or lymphadenitis

history of fever
absence of cough

If antibiotics are indicated then either phenoxymethylpenicillin or erythromycin (if


the patient is penicillin allergic) should be given. Either a 7 or 10 day course
should be given

Complications of tonsillitis include:


otitis media
quinsy - peritonsillar abscess
rheumatic fever and glomerulonephritis very rarely
The indications for tonsillectomy are controversial. NICE recommend that surgery should
be considered only if the person meets all of the following criteria
sore throats are due to tonsillitis (i.e. not recurrent upper respiratory tract
infections)
the person has five or more episodes of sore throat per year
symptoms have been occurring for at least a year
the episodes of sore throat are disabling and prevent normal functioning
931. A pt had passed a 4mm stone in his urine. He has a 3mm stone in the renal pelvis
found on US.
What is the management?
a. ESWL
b. None
c. Dormier basket
d. Surgery
e. PCNL
b. None
If the stone is <5mm in lower ureter ,it will pass spontaneously..
if it is >5mm ---> medical therapy ( nifedipine ,alpha blocker ) if not passed , go for
ESWL ( If < 1cm ) or dormia basket...
PCNL ---> when stone is large ,multiple or complex..
Percutaneous nephrostomy ---> presence or infection or obstruction , to safe the kidney
from reflux damage and save the person from Sepsis ( if pus collected )
932. A 4yo boy presents with fever, severe ear ache, vomiting and anorexia. He also has
mod
tonsillitis. Exam: tympanic membrane bulging. He came to the GP a few days ago and
was dx
with URTI. What is the most appropriate dx?
a. OE
b. Acute OM
c. Serous otitis
d. Chronic suppurative OM
e. Mastoiditis
b. Acute OM
Complications of tonsillitis include:

otitis media
quinsy - peritonsillar abscess
rheumatic fever and glomerulonephritis very rarely
antibiotics are not routinely recommended. NICE recommends however that they should
be considered in the following situations:
children younger than 2 years with bilateral acute otitis media
children with otorrhoea who have acute otitis media

933. A 3yo girl presents with complaints of sudden right facial weakness and
numbness and pain
around her ear. There are no symptoms. What is the most appropriate dx?
a. SAH
b. Bells palsy
c. Stroke
d. TIA
e. Subdural hemorrhage
b. Bells palsy
Bell's palsy may be defined as an acute, unilateral, idiopathic, facial nerve paralysis. The
aetiology is unknown although the role of the herpes simplex virus has been investigated
previously. The peak incidence is 20-40 years and the condition is more common in
pregnant women.
Features
lower motor neuron facial nerve palsy - forehead affected*
patients may also notice post-auricular pain (may precede paralysis), altered
taste, dry eyes, hyperacusis
Management
in the past a variety of treatment options have been proposed including no
treatment, prednisolone only and a combination of aciclovir and prednisolone
following a National Institute for Health randomised controlled trial it is now
recommended that prednisolone 1mg/kg for 10 days should be prescribed for
patients within 72 hours of onset of Bell's palsy. Adding in aciclovir gives no
additional benefit
eye care is important - prescription of artificial tears and eye lubricants should be
considered
934. A 6yo boy fell in the playground and has been holding his forearm complaining of
pain. Exam: no sign of deformity or swelling. However, there is minimal tenderness on
exam. What is the dx?
a. Fx mid radius
b. Fx mid ulnar
c. Fx neck of humerus
d. Fx shaft of humerus

e. Green stick fx of distal radius


e. Green stick fx of distal radius
Greenstick fractures common in children

Paediatric both-bone forearm fractures


Fractures may be of greenstick type (incomplete) or complete. A greenstick fracture can
occur in one bone with a complete fracture in the other. Complete fractures may be
undisplaced, minimally displaced or overriding. Fractures of the proximal third are
relatively rare. Middle third fractures account for about 18% of both-bone fractures and
distal third fractures for about 75%.
Mechanism of injury: usually an indirect injury following a fall on to an
outstretched hand. Occasionally caused by a direct trauma.
Presentation: pain, swelling and deformity at the fracture site.
Investigation: X-rays of the wrist, elbow and whole forearm should be taken.
Management: unlike the management of these fractures in adults, conservative
management is still the first line of treatment for paediatric forearm fractures,
especially in children less than 10 years old.
[6]

935. A 62yo man has been smoking about 15 cigarettes/day for 45yrs, and has been
working as a
builder since he was 24yo. He presents with chest pain, SOB, weight loss. CXR shows
bilateral
fibrosis and left side pleural effusion. What is the best inv that will lead to dx?
a. CXR
b. Pleural fluid aspiration of cytology
c. MRI
d. Pleural biopsy
e. CT
d. Pleural biopsy
Only biopsy confirms carcinoma
Ct is the next step not the best step towards the diagnosis. The best work up leading to
diagnosis should b pleural biopsy,
Asbestosis predisposing to mesothelioma and therefore pleural biopsy
Pleural fluid: straw coloured or blood stained. Cytological analysis occasionally leads to
the diagnosis but a pleural biopsy is usually required.

936. During a basketball match, one of the players suddenly collapsed to the ground
with coughing and SOB. What is the inv of choice?
a. CXR
b. CT
c. MRI
d. V/Q scan
e. CTPA
a. CXR

case of spontaneous pneumothorax...xray chest ...first


Basketball player... tall height.... more chances of apical subpleural blebs... its
SPONTANEOUS PNEUMOTHORAX.. so CXR is best

937. A 57yo man having HTN on oral anti-HTN. However, he is finding it difficult to
mobilize as he
feels dizzy whenever he tries to get up. What is the most appropriate inv for him?
a. Ambulatory BP
b. ECG
c. MRI
d. CXR
e. CT
a. Ambulatory BP

Who should be referred for ambulatory blood


pressure monitoring?

The National Institute for Health and Care Excellence (NICE) recommends that if
a clinic blood pressure is 140/90 mmHg or higher, ABPM should be offered to
confirm the diagnosis of hypertension. If a person is unable to tolerate ABPM,
home blood pressure monitoring (HBPM) is a suitable alternative to confirm the
diagnosis of hypertension.
Poorly controlled hypertension - eg, suspected drug resistance.
Patients who have developed target organ damage despite control of blood
pressure.
Patients who develop hypertension during pregnancy.
High-risk patients - eg, those with diabetes mellitus, those with cerebrovascular
disease, and kidney transplant recipients.
Suspicion of white coat hypertension - high blood pressure readings in clinic
which are normal at home.
Suspicion of reversed white coat hypertension, ie blood pressure readings are
normal in clinic but raised in the patient's own environment.
Postural hypotension.
Elderly patients with systolic hypertension.
938. A 33yo female complains of diplopia on upright gaze. Exam: ptosis can be seen.
There are no other complains or any significant PMH. What is the most appropriate inv
for him?
a. Ophthalmoscopy
b. Visual field test
c. TFT
d. CT
e. Checking red reflex
[5]

d. CT
ptosis can be due to neurological causes of muscle weakness in this case , but there is
associated diplopia so its better to exclude any nerve lesion through ct.
3rd nerve palsy

939. A tall rugby player was hit in the chest by a player of the opponent team. He
developed
breathlessness and his face went blue and purple. You have been called to look at him,
how will
you manage him?
a. Insert a needle in the 2nd ICS in the mid-clavicular line
b. Insert a needle in the 5th ICS in the mid-axillary line
c. Intubate the pt
d. Start CPR
e. Give oxygen
a. Insert a needle in the 2nd ICS in the mid-clavicular line
Tension pneumothorax

940. A young woman fell and hit her knee. Exam: valgus test +ve. What ligament was
most probably
injured?
a. Ant cruciate
b. Medial collateral
c. Lateral collateral
d. Post cruciate
e. Meniscus
b. Medial collateral
Medial collateral: Valgus stress test

Lateral collateral: Varus stress test


Anterior Cruciate: Anterior drawer test, Lachmans test, pivot shift test.
Posterior cruciate: Posterior drawer test, posterior sag test.
941. A 75yo man comes in complaining of difficulty in passing urine, poor stream and
dribbling at the end of voiding and anorexia. US shows bilateral hydronephrosis. What is
the cause of these
findings?
a. BPH
b. Renal stones
c. Bladder stones
d. Prostatic ca
e. UTI
d. Prostatic ca
bph is common and BOO signs are present with it earlier but here the anorexia is given
which points to carcinoma

bladder outlet obstruction: hesitancy, urinary retention


haematuria, haematospermia
pain: back, perineal or testicular
digital rectal examination: asymmetrical, hard, nodular enlargement with loss of
median sulcus

942. 2h after an appendectomy, a pt complains of a rapid HR and fever. He says there is


also
abdominal pain and pain in the shoulder area. What is happening to this pt?
a. Intra-abdominal bleeding
b. Anastomotic leak
c. Sepsis
d. Intestinal obstruction
a. Intra-abdominal bleeding
rapid HR due to haeg, shoulder pain due to diaphragmatic irritation following
intraabdominal bleeding and fever for tissue reaction following surgery
following appendicitis, gut anastomosis not needed and gut anastomosis is needed
following large or small gut ischaemic necrosis or gangrene.
Sepsis in 2 hours very unlikely
post op h'age : primary- immediate or as a continuation of intraop bleed. reactionary :
within 24hrs. secondary: upto 10 days.
Mx: 1. fluid replacement - crystalloids upto 1000ml bolus and then maintain. 2.direct
compression to control superficial bleeding if any. 3. cross match blood. 4. emergency
surgery
943. A 50yo man presents with the complaints of recurrent UTI and occasional blood in
the urine.
Some unusual cells have been seen in urine on routine exam. Which os the following inv
would

you like to carry out now?


a. Cystoscopy
b. Urine C&S
c. XR KUB
d. US
e. CBC
d. US
the unusual cells may indicate malignancy , which in turn can be the cause of recurrent
UTI .US followed by cystoscopy seems to be the appropriate approach.
unusual cell points towards possible malignancy! may be bladder cancer or renal cell
carcinoma. So we have two options, for bladder cystoscopy and for renal cell ca US.
Before going to more invasive procedure we can think first noninvasive procedure. So
US is more logical i think. for uti we can do c/s but that is not the major issue.
944. A 28yo drug user presents to ED collapsed and anuria. His serum K+=7.5mmol/l.
CXR shows early
pulmonary edema. What is the next appropriate management for this pt?
a. Urgent hemodialysis
b. IV calcium gluconate
c. IV insulin + dextrose
d. Furosemide
e. IV 0.9% NS
b. IV calcium gluconate
cardioprotective
Stabilisation of the cardiac membrane
intravenous calcium gluconate
Short-term shift in potassium from extracellular to intracellular fluid compartment
combined insulin/dextrose infusion
nebulised salbutamol
Removal of potassium from the body
calcium resonium (orally or enema)
loop diuretics
dialysis

945. DM man feels hot, painful lump near the anal region. What is the most probable dx?
a. Anal fissure
b. Abscess
c. Hematoma
d. Wart
e. External hemorrhoids
b. Abscess
Perianal abscess: the most common (60%)

High-risk groups include those with diabetes, immunocompromised patients, people who
engage in receptive anal sex and patients with inflammatory bowel disease.
Symptoms include painful, hardened tissue in the perianal area, discharge of pus from
the rectum, a lump or nodule, tenderness at the edge of the anus, fever, constipation or
pain associated with bowel movements.
Prompt surgical drainage.
Medication for pain relief.
Antibiotics are usually not necessary unless there is associated diabetes or
immunosuppression.
946. A 65yo lady with T1DM for the last 20y comes with a tender lump near the anal
opening. She
says she also has a fever. What tx should she get?
a. I&D + antibiotics
b. IV antibiotics
c. C&S of aspirate from swelling
d. Painkillers
e. Cautery of swelling
a. I&D + antibiotics
see q 245

947. An 80yo DM lady presents with redness and swelling over her right foot. It is tender
to touch,
warm and glossy. What are the complications this pt might develop?
a. Meningitis
b. Sepsis
c. Ulcer
d. Gangrene
d. Gangrene
Cellulitis to wet gangrene
In clinical setting in diabetics it's usually cellulitis then gangrene then sepsis.

Complications of cellulitis
Complications are uncommon but may include:
Abscess formation.
Gangrene.
Thrombophlebitis/lymphangitis.
Chronic leg oedema (a late complication which may predispose to further
episodes of infection).
Less common complications (occurring in <1%) include:
Necrotising fasciitis.
Osteomyelitis.
Compartment syndrome.
Acute glomerulonephritis.
Endocarditis.

Septicaemia.
Streptococcal toxic shock syndrome.

Flucloxacillin 500 mg four times daily (in adults) is usually given as first-line
Severe cellulitis should be treated with intravenous benzylpenicillin + flucloxacillin.
948. After surgery a pts left leg has become swollen and tender. The diameter of the
calf has
increased and passive movements cause pain. What is the most probable dx?
a. DVT
b. Lymphedema
c. Peripheral vascular disease
d. Hematoma
e. Superficial thrombophlebitis
a. DVT
because the human body is under hypercoagulable state after any operation, and
because of the diameter of calf has increased
Its not specified the site of surgery. Surgery is a highly stressful state and also leads to
immobilization both risks for dvt. Clinically there is swelling and tenderness- moses sign.
Pain on passive movements- homan sign.
If a DVT is 'likely' (2 points or more)
a proximal leg vein ultrasound scan should be carried out within 4 hours and, if
the result is negative, a D-dimer test
if a proximal leg vein ultrasound scan cannot be carried out within 4 hours a Ddimer test should be performed and low-molecular weight heparin administered
whilst waiting for the proximal leg vein ultrasound scan (which should be
performed within 24 hours)
If a DVT is 'unlikely' (1 point or less)
perform a D-dimer test and if it is positive arrange:
a proximal leg vein ultrasound scan within 4 hours
if a proximal leg vein ultrasound scan cannot be carried out within 4 hours lowmolecular weight heparin should be administered whilst waiting for the proximal
leg vein ultrasound scan (which should be performed within 24 hours)
Management
Low molecular weight heparin (LMWH) or fondaparinux should be given initially after a
DVT is diagnosed.
a vitamin K antagonist (i.e. warfarin) should be given within 24 hours of the
diagnosis
the LMWH or fondaparinux should be continued for at least 5 days or until the
international normalised ratio (INR) is 2.0 or above for at least 24 hours,
whichever is longer, i.e. LMWH or fondaparinux is given at the same time as
warfarin until the INR is in the therapeutic range
warfarin should be continued for at least 3 months. At 3 months, NICE advise
that clinicians should 'assess the risks and benefits of extending treatment'

949. 2h after an appendectomy, a pt complains of a rapid HR and fever. He says there is


also
abdominal pain and pain in the shoulder area. What is the first step in the management?
a. Maintain IV access and give IV fluids
b. Start IV antibiotics
c. Insert NGT for intestinal decompression
d. Cross match blood
e. Emergency exploratory laparotomy
a. Maintain IV access and give IV fluids
Q942
950. A pregnant woman presents with knee pain on movements. The pain becomes
worse at the end of the day. Radiology shows decreased joint space. Labs: CRP=12.
What is the 1st line med?
a. Paracetamol
b. NSAIDs
c. Oral steroid
d. Intra articular steroid
e. DMARDs
a. Paracetamol
Osteoarthritis... First line is paracetamol
safe in pregnancy
second-line treatment is oral NSAIDs/COX-2 inhibitors, opioids, capsaicin cream and
intra-articular corticosteroids.

951. A 68yo man presents with muscle weakness. He is not able to climb
stairs. He also complains of mild breathlessness. He says that he sometimes
feels difficulty in swallowing food. Labs: ALP=216, AST=49, ALT=43,
CK=417, ESR=16. What is the most likely dx?
a. Polymyositis
b. Polymyalgia rheumatic
c. Muscular dystrophy
d. Esophageal carcinoma
e. Osteoarthritis
Ans : A
Reason : Patients with Polymyositis have affected proximal muscle groups,
which presents itself with difficulty standing up from a chair , inability to
climb stairs, lifting objects and combing hair. Fatigue, myalgia, and muscle

cramps. fine muscular movements are affected late in the disease, dysphagia,
no rash.
Dermatomyositis presents with similar features along with skin
manifestations, blue purple heliotrope rash , shawl appearance ( over
shoulders and back).fever, arthralgia, malaise and weight
loss,tachyarrhythmias and dilated cardiomyopathy,git ulcers and infections.
Epidemiology: Polymyositis 30 and 60 years smaller peak at about 15 years
of age.
Dermatomyositis Peak age of Onset Adults 50. Peak onset age children 5-10
INVESTIGATIONS : CK, SGPT, SGOT, LDH,anti-Jo-1 antibodies in
polymyositis ( Lung symptoms also present)
diagnosis is established by electromyography (EMG) and is confirmed by
muscle biopsy. In polymyositis it is the definitive test.
positive antinuclear antibody (ANA) finding is common in patients with
dermatomyositis. Anti-Mi-2 antibodies are specific for
dermatomyositis.EMG and muscle biopsy can be diagnostic.
D/D. SLE, Sojrens, RA, Scleroderma
Treatment
STEP1 Steroid, STEP2 Azathioprine , STEP3 Cyclophosphamide , IV
immunoglobulins for resistant dermatomyositis
FOR LUNG DISEASE combination regimen including ciclosporin A or
tacrolimus with cyclophosphamide + steroid.
952. A 67yo builder presents with a persistent nodular lesion on upper
part of pinna with some telangiectasia around the lesion. What is the dx?
a.
Basal cell
b. Squamous cell
c. Keratocanthoma
d. Actinic keratosis
e. Bowens disease
Ans: A
Reason: presentation of basal cell CA is over sun exposed areas, lesions
small pearly white and raised with telangiectasia.rodent ulcer has an
indurated edge and ulcerated centre.
types: Nodular=Solitary, shiny, red nodule with large telangiectatic
vessels.Commonly on the face.

Superficial=Often multiple, usually on the upper trunk and shoulders,

Erythematous well-demarcated scaly plaques.


Morphoeic=Also known as sclerosing or infiltrative BCC.Usually
found in mid-facial sites.More aggressive and have poorly defined borders
Pigmented=Brown, blue or greyish lesion.Nodular or superficial
histology.Seen more often in individuals with dark skin.
Basosquamous
Mixed BCC and squamous cell carcinoma (SCC)

Treatment : Surgery : mohs micrographic surgery


Curettage and cautery/electrodesiccation.Not recommended for recurrent,
large, morphoeic tumours or tumours on the face
Cryotherapy/cryosurgery .Cryotherapy is well established for treating
small low-risk lesions, including superficial BCCs.Histology is not available
unless an incisional biopsy is taken first.
Imiquimod 5% cream:Topical Imiquimod appears to be effective in the
treatment of primary small superficial BCCs
Radiotherapy.The best indications for radiotherapy are BCC with
incomplete excision, recurrent BCC, nodular BCC of the head and neck
under 2 cm and BCC with invasion of bone or cartilage
953. A 68yo pt wakes up with slurred speech and right sided weakness.
CT shows cerebral infarct. What is the most appropriate tx?
a. Aspirin
b. Alteplase
c. Warfarin
d. Clopidogrel
e. Dipyridamole
ANS: B
Reason : If a patient with ischemic stroke presents within 4.5 hours of
ischemic episode thrombolysis can be tried. Do a CT 24 hrs Post
thrombolysis.after that patients are prescribed Aspirin 300 mg for 2 weeks.(
ischemic stroke is a gradual process and episodes might take place during
sleep)
CI:
major infarct or hemorrhage
mild deficit
Recent Birth,Surgery,Trauma
Past CNS bleed
Seizures at presentation

platelets <100
BP >220/130
954. A 73yo man who is recovering from surgery on the left carotid
artery in his neck. He has slurred speech. On protrusion of his tongue, the tip
deviated to the left. What is the single most appropriate option?
a. Accessory nerve
b. Facial nerve
c. Glossopharyngeal nerve
d.
Hypoglossal nerve
e. Vagus nerve
Ans: D
Reason: Ask the patient to protrude his/her tongue and note any deviation. A
fluttering motion called fibrillation rather than fasciculation may be seen
with an LMN lesion.
If the tongue deviates to one side when protruded, this suggests a
hypoglossal nerve lesion. If it is an LMN lesion, the protruded tongue will
deviate towards the side of the lesion. With a UMN lesion, the tongue will
deviate away from the side of the lesion
Causes of a single XII lesion: rare. Polio,syringomyelia tuberculosis,
median branch thrombosis of the vertebral artery.
955. A 24yo woman known to be suffering from panic disorder presents
to the hospital with tingling and numbness in her fingers. ABG: pH=7.52,
PCO2=2.2kPa, PO2=11kPa, Bicarb=20. What is the most likely condition?
a. Acute metabolic alkalosis
b.
Acute resp alkalosis
c. Compensated resp alkalosis
d. Compensated metabolic acidosis
e. Acute metabolic acidosis
Ans:B
Reason: Acute respiratory alkalosis due to hyperventilation which causes
CO2 washout.a secondary hypocalcemia results with shifting of serum Ca to
unionized bound form.
S/S Dizziness.Perioral tingling.Weakness.Tinnitus.Palpitations.Feeling of
choking or suffocation.Wheezing.Sweating.Loss of consciousness
(uncommon)
Treatment: rebreathing into paper bag (not in physical disease)

A panic attack is defined as a discrete episode of intense subjective fear,


where at least four of the characteristic symptoms, listed below, arise
rapidly and peak within 10 minutes of the onset of the attack:
Attacks usually last at least 10 minutes but their duration is variable.
The symptoms must not arise as a result of alcohol or substance
misuse, medical conditions or other psychiatric disorders, in order to
satisfy the diagnostic criteria
S/S
Palpitations, pounding heart or accelerated heart
rate.Sweating.Trembling.Dry mouth.Feeling short of breath.Feeling of
choking.Chest pain or discomfort.Nausea or abdominal distress.Feeling
dizzy, unsteady, light-headed or faint. Derealisation .Fear of losing control
or 'going crazy'.Fear of dying.Numbness or tingling sensations.Chills or hot
flushes
Treatment (In order as per Nice guidelines)
CBT
Medication : first line SSRI ( Fluoxetine, Paroxetine)
Consider clomipramine or imipramine after 12 weeks of unsuccessful
treatment
Step 3 Consider Alternative treatments
Step 4. Referral to specialist Mental health institution.
956. A 65yo man on dexamethasone underwent surgery. During and after
the surgery, his blood glucose was around 17-19 mmol/l. What will you give
the pt?
a. Insulin
b. Oral hypoglycemic
c. Remove dexamethasone
d.
IV Saline
e. IX dextrose
957. A 61yo man who had stroke 2y ago is on aspirin. He has RA but
suffers from pain and cant tolerate it. He is taking senna for constipation.
What is the best med to relieve his pain?
a. DMARDs
b.
Ibuprofen
c. Co-codamol

d.

Paracetamol

ANS: B
Reason: DMARDS have no role in acute RA pain control. Paracetamol and
weak opiates are rarely effective , ( another contraindication is constipation).
NSAIDS are good for symptom relief , recommended are Ibuprofen,
naproxen,diclofenac.
Steroids are preferred for acute relief and flares of RA
958. A young child was brought by his mother to the OPD complaining
that he raised the vol of the TV and didnt respond to her when she called
him. Exam: tympanic membrane was dull greyish and no shadow of handle
of malleus. What is the most probable dx?
a. Chronic OM
b. Acute OM
c.
Secretory OM
d. Otitis externa
e. Cholesteatoma
ANS: C
Reason: Otitis media with effusion (OME) is defined as inflammation of the
middle ear, accompanied by the accumulation of fluid in the middle-ear
cleft, without the symptoms and signs of acute inflammation,It often results
in conductive hearing loss and is the most common cause of hearing loss and
elective surgery in childhood. In ears with documented fluid, the average
hearing loss is 20 decibels (dB),but may be as high as 50 dB
S/S: Earache (not always present).Hearing loss and/or behavioural
problems.Hearing loss, which may be significant (20-30 dB), particularly if
it occurs in both ears and has persisted for over one month.It can occur
without hearing loss.
Treatment: Medication not recommended , OBSERVE,
NICE recommends that children who most benefit from surgery are those
with persistent bilateral OME lasting three or more months with a hearing
level in the better ear of 25-30 dB HL or worse averaged at 0.5, 1, 2 and 4
kHz

959. A 48yo woman always socially withdrawn has stopped going out of
the house. She is afraid to socialize because she fears that people will
criticize her. What is the most probable dx?
a. Agoraphobia
b. PTSD
c.
Social anxiety
d. OCD
e. GAD
ANS: C
Reason : Social Anxiety AKA Social Phobia, persistent fear and anxiety
about one or more social or performance situations.Social anxiety is a fear of
being around people and having to interact with them. Sufferers fear being
watched and criticised. Normal activities such as working, shopping, or
speaking on the telephone are marked by persistent feelings of anxiety and
self-consciousness. They feel dread as a situation approaches and afterwards
they analyse or ruminate on how they could have done better.They often
experience chronic insecurity about their relationships with others, excessive
sensitivity to criticism, and profound fears of being judged negatively,
mocked, or rejected by others
Treatment: CBT ,
If patient Declines NICE recommends Consider SSRIs (SErtraline,
Escitalopram), Pts who decline both consider short-term psychodynamic
psychotherapy.
960. Post gastric ulcer got perforated leading to bleeding involving the
gastro-duodenal artery. Where would fluid accumulate in the cavity?
a. Left paracolic gutter
b. Pelvic cavity
c. First part of duodenum
d.
Under the diaphragm
e. Retroperitoneal
961. A 4yo boy presents with recurrent episodes of self limiting
spontaneous bleeding. Coag test: PT normal, bleeding time normal, APTT
prolonged, Factor VIII decreased. His father and uncle suffer from a similar
illness. What is the most likely dx?
a.
Hemophilia A
b. Hemophilia B

c. Von willebrands disease


d. ITP
e. TTP
ANS : A
Reason It is a bleeding disorder which presents with the deficiency of factor
VIII, there are three forms,
Severe which usually presents in infancy ( time of circumcision or as
intracranial hemorrhage )factor activity <1%
Moderate which presents with bleeding following venepuncture (age Before
2 years) Factor Activity 1-5 %.
Mild. Only bleed after major trauma or surgery.Age >2 years, Factor
Activity >5%
INvestigations : PT,BT,fibrinogen and Von willebrand Normal
APTT) - usually prolonged, Factor VIII:C - is reduced
Treatment:
Children with severe haemophilia (ONce weekly prophylaxis with FActor
VIII)
Recombinant factor VIII preferred. Fresh frozen plasma and cryoprecipitate
should only be used in an emergency when the concentrates are not available
Desmopressin boosts Factor VIII activity.
962. A 53yo lady presents with hot flash and night sweats. Her LMP was
last year. She had MI recently. What is the most appropriate management for
her?
a. Raloxifene
b. Estrogen
c.
COCP
d. Evening primrose
e. Clonidine
Ans : Correct Answer is Clonidine
.Clonidine has cardiac uses like antihypertensive and diagnosis of pheochromocytoma (by reducing nor
epinephrine)
And non cardiac uses like post menopausal vasomotor symptoms ,opioid withdrawal,diabetic
diarrhoea,smoking cessation, and analgesia

HRT contraindicated in pts with recent MIs or ischemic episodes.


Contraindications:

pregnancy
undiagnosed abnormal vaginal bleeding

active thromboembolic disorder or acute-phase myocardial infarction


suspected or active breast or endometrial cancer
active liver disease with abnormal liver function tests
porphyria cutanea tarda

As transdermal oestrogen is associated with fewer risks than oral HRT, a


transdermal route may be preferable for many women. This route is also
advantageous for women with diabetes, hypertension and other
cardiovascular risk factors, and also especially with advancing age
963. A 73yo man who was a smoker has quit smoking for the past 3yrs. He
now presents with hoarseness of voice and cough since past 3wks. XR: mass
is visible in the mediastinum. What is the best inv to confirm the dx?
a. Bronchoscopy
b. Thoracoscopy
c. US
d. CT thorax
e.
LN biopsy
Ans: E
Reason : the patient seems to be affecting from Bronchogenic Carcinoma,
The Key diagnosis as rccomended by NICE
Xray
CT
Bronchoscopy with Biopsy
Biopsy of a convenient metastatic site should be performed if this is easier
than biopsy of a primary site . (Biopsy remains the best investigation to
confirm the diagnosis)
NICE strongly recommends a new imaging test,18F-deoxyglucose positron
emission tomography (FDG-PET), to help stage tumours
Treatment
Small cell tumours are usually treated with 4-6 cycles of multi-drug
platinum-based chemotherapy with the possibility of added radiotherapy in
limited stage disease.
Most Rapidly Growing+Worst Prognosis
Staging of the disease and the patients overall fitness and exercise ability
will determine the type of treatment chosen
Surgery , Radiotherapy, Radical Radiotherapy , RADIO+ Chemo
964. A 52yo man known DM presents to ED with sudden onset of pain in
the left loin and hematuria. Inv: 8mm stone in left lower ureter. Nifedipine
with steroids was prescribed as initial tx with supportive therapy. He

returned complaining of worsening pain, vomiting with passing of 2 stones.


Renal function tests indicate impending ARF. How will you manage this pt?
a. Continue same tx
b. Start alpha blocker
c. ESWL
d. Percutaneous nephrolithotomy
e.
Percutaneous nephrostomy
f.
Open surgery
Ans: E
Reason : Emergency Treatment with percutaneous nephrostomy and or
ureteric stent insertion is necessary if either pain or obstruction is persistent.
95% of those 2-4 mm in diameter pass spontaneously but passage may take
as long as 40 days
(ESWL) - shock waves are directed over the stone to break it apart. The
stone particles will then pass spontaneously.(Acute urinary tract infection or
urosepsis Contraindication for ESWL)
Percutaneous nephrolithotomy (PCNL) - used for large stones (>2 cm),
staghorn calculi and also cystine stone
Open surgery reserved for multiple stone/complicated cases

965. A lady who is alcohol dependent wants to quit but wants someone to
encourage her. What would you do?
a. Medication
b. Refer to social services
c. Refer to psychology
d. CBT
ANS: B
She should be Advised to join a Support Group to Help her Quit.
966. A young girl presented to OBGYN assessment unit with lower
abdominal pain and per vaginal bleeding after a hx of hysterosalpingograph
as a part of her infertility tx. Observation: BP=90/50mmHg, pulse=120bpm,
exam revealed rigid abdomen. What is the most appropriate next inv?
a. CT
b. XR erect and supine
c.
US abdomen

d.
e.

Coag profile
CXR

ANS: C
Reason : USG abdomen will confirm the presence of blood in the abdominal
cavity, infections can occur after Hysterosalpingography along with vaginal
bleeding, fever and foul smelling discharge.
TEST RISKS
allergic reaction to contrast dye
endometrial (uterine lining) or fallopian tube infection
injury to the uterus, such as perforation
967. A 21yo woman who is on COCP had to take azithromycin. What
should be advised for her contraception?
a. Using 7d condoms after antibiotics and avoid pill free break
b. Using 14d condoms after antibiotics and avoid pill free break
c. Using 7d condoms after antibiotics
d.
No extra precaution
e. Using 14d condoms after antibiotics
ANS: D
REASON : NON enzyme inducing antibiotics no extra precaution necessary.
Type of medication

Advice given

Non-enzyme-inducing
antibacterial.

Women should be advised that no additional


contraception is required.

Short course of
enzyme-inducing
antibacterials
rifampicin or rifabutin.

Women are advised to continue taking the COCP


and use additional precautions. Monophasic 21day pills should be taken either as an extended
regimen (continue packets without a break until
3-4 days of BTB occurs, then have 4-day pill-free
interval) or a tricycling regimen (three packets
without a break then a 4-day pill-free interval).
Additional contraception should be continued for
28 days after stopping the rifampicin/rifabutin.

Long-term course of
Should be advised to use an alternative, nonenzyme-inducing
hormonal method where possible (very potent
antibacterials
enzyme inducers).
rifampicin or rifabutin.
Other enzymeinducing drugs,
including
anticonvulsants, St
John's wort, etc.

Short course: advice is as per that above for


rifampicin/rifabutin. Long course: women should
be encouraged to use alternative methods of
contraception. If, having considered alternatives,
they still choose the COCP, the patient should be
advised of the increased risk of pregnancy.
Should use a preparation containing at least 50
micrograms of oestrogen. Tricycling or extended
regimens as above should be used. If BTB occurs
on 50 micrograms, the dose should be increased
to a maximum of 70 micrograms.

Lamotrigine.

Women should be advised not to take lamotrigine


with the COCP and should seek another form of
contraception (unless also taking a non-enzymeinducing anticonvulsant such as sodium
valproate).

Antiretroviral
therapies.

Those women on ritonavir-boosted protease


inhibitors should be advised to use alternative
methods of contraception.

Ulipristal acetate.

Women should use additional contraceptive


precautions for 14 days after taking ulipristal
acetate as ellaOne for emergency contraception
(16 days for Qlaira). Those taking ulipristal in a
higher dose as Esmya for fibroids should not be
advised to use alternative contraception.

968. A 60yo woman presented with radial fx and had a colles fx and
supracondylar fx in the past. What inv is req to detect her possibility of
having the same prb later?
a.
Dexa scan
b. MRI
c. Nuclear bone scan
d. CT

e.

Bone biopsy

ANS: A
Reason: Increasing Age is associated with osteoporosis and increasing
incidence of fractures, DEXA scan measures the density of the bone and
chances of bone fracture.
969. A 43yo woman presents with low mood, loss of libido, sleep
disturbance, tiredness, palpitation, chest discomfort, irritability and recurrent
worries. What is the most likely dx?
a. Seasonal Affective Disorder
b.
Mod depression
c. Dysthymia
d. GAD
e. Bipolar disorder
And: moderate depression
Typical features of depression , season affective disorder mostly affects
during specific time of the year mostly in winters ,patient tend to be low
,loss of interest and increased somnolence.
dysthymia is a serious state of chronic depression, which persists for at least
two years
970. Which of the following is true for tamoxifen?
a.
Increased incidence of endometrial carcinoma
b. Increased risk of breast ca
c. Increased risk of osteoporosis
d. Increased risk of ovarian ca
Ans: A
Tamoxifen is associated with increased risk of endometrial cancer .
971. A 45yo male complains of tremors in hands. Exam: tremors are
absent at rest but present when arms are held outstretched and persist on
movement. What is the most probable dx?
a. Parkinsonism
b.
Benign essential tremor
c. Cerebellar disease
d. Liver failure

e.

Stroke

Ans: B
Reason . the patient is suffering from postural tremors , no tremors at rest
,present on persisted posture and movement . Causes : Benign Essential
Tremor
Rest tremor: abolished on voluntary movement. Cause: parkinsonism.
Intention tremor: irregular, large-amplitude, worse at the end of purposeful
acts, eg finger-pointing or using a remote control. Cause: cerebellar damage
(eg MS, stroke). Postural tremor: absent at rest, present on maintained
posture (arms out-stretched) and may persist (but is not worse) on
movement. Causes: Benign essential tremor (autosomal dominant; improves
with alcohol), thyrotoxicosis, anxiety, -agonists.
972. Pregnant lady had her antenatal screening for HIV and Hep B. what
more antenatal inf should she be screened for?
a.
Rubella and syphilis
b. Toxoplasma and rubella
c. Syphilis toxoplasma
d. Hep C & E
e. Hep A & C
ANs : A
Reason: Routine ANtenatal INfectious Screen .. Rubella, Syphilis, HIV,
HEP B
Hep C offered if Patient thinks she may be infected.
973. A young man has been found in the park, drunk and brought to the
ED by ambulance. He recently lost his job and got divorced. He thinks
nurses are plotting against him. What is the most likely dx?
a. Schizoid personality
b. Borderline personality
c.
Schizophrenia
d. Psychotic depression
e. Paranoid personality
Ans: C
Reason : delusional, history of unstable job and family life.

974. An elderly man who used to work in the shipyard industry presented
with cough and SOB few weeks to months. He was given salbutamol
nebulization and antibiotics and admitted to the ward. He died 3d later. CT:
patchy infiltrates, pleural thickening and pleural effusion. Why is this a
coroners case?
a. Pt got wrong dx or management
b. Pt died soon after admission
c.
Death could be due to occupational illness
Ans: C
Reason: history suggests this could be a case of asbestosis.
975. A 26yo lady came with abdominal pain, vaginal discharge and low
grade fever. What is the most likely dx?
a. HELLP syndrome
b.
Acute PID
c. Ectopic pregnancy
d. Appendicitis
Ans: B
Reason : typical cilinical picture of PID.
S/S lower abdominal pain, deep dyspareunia , abnormal vaginal or cervical
discharge often purulent .lower abdominal tenderness, fever.
Diagnosis : testing for gonorrhea and chlamydia in lower genital tract.
Elevated ESR ,C reactive protein
D/D .ectopic pregnancy , appendicitis
Management : broad spectrum antibiotics to cover gonorrhea and chlamydia.
Cefoxitin/ceftriaxone followed by doxycline showed good outcome .
Oral ofloxacin 400 mg bd + mteronodalzole 400 mg bd 14 days
976. A new screening test has been devised to detect early stages of
prostate ca. However, the test tends to dx a lot of ppl with no cancer,
although they do have cancer as dx by other standard tests. What is this flaw
called?
a. False +ve
b. True +ve
c.
False ve

d. True ve
e. Poor specificity
ans: C
Reason : diagnosing disease positive patients as no disease present known as
false negative .
977. A 26yo political refugee has sought asylum in the UK and
complains of poor conc. He keeps getting thoughts of his family whom he
saw killed in a political coup. He is unable to sleep and feels hopeless about
his survival. Because of this he is afraid to go out. What is the most likely
dx?
a. Acute stress disorder
b.
PTSD
c. Social phobia
d. OCD
e. GAD
Ans: B
Reason : post traumatic stress disorder is delayed or prolonged response to
stressful situation. E.g
Sexual abuse, War, Road traffic accident, human disaster .
978. A 2yo boy presented with gradual swelling of feet and poor feeding.
He has gained weight and has dark urine. What is the single most
appropriate inv?
a. Serum albumin
b.
24h urinary protein
c. Serum calcium
d. BUE
e. Serum glucose
Ans: the clinical features correlate with loss of protein , it could be due to
any underlying disease presenting itself as nephrotic syndrome .
979. A 26yo lady presents with high fever, lower abdominal pain and
purulent vaginal discharge. She looks very unwell. What is the most
appropriate management?
a. Tetracycline 250mg QD
b. Doxycycline 100mg BD and metronidazole 400mg BD
c.
IV Ceftriaxone 2g with doxycycline 100mg

d.
e.

IV ceftriaxone 2g with doxycycline 500mg


Ofloxacin 400mg BD and metronidazole 400mg BD

Ans: C
Reason : patient seems to be suffering from acute PID and the recommended
management is broad spectrum antibiotics therapy to cover chlamydia and
gonorrhea.
First line in inpatient patients severe infection is Iv Ceftriaxone 2g +
doxycline 100 mg bd.
980. A 39wk pregnant woman came to labor suite 3d after an obstructed
labour presents with pain and swelling of one leg. Exam: leg has blue
mottling and is cold. What is the dx?
a. DVT
b. Post phlebitis syndrome
c.
Embolus
d. Varicose vein
e. Herpes gladiatorum
Ans:C
Reason : clinical features of limb ischemia suggestive of thromboembolic
phenomena ,
S/S The affected part becomes pale, pulseless, painful, paralysed,
paraesthetic and 'perishing with cold' ('the 6 Ps').[1]
The onset of fixed mottling of the skin implies irreversible changes
INV: Hand held Doppler , Blood Tests , identify Source (
ECG,Echo,Aortic USG, Popliteal femoral artery USG)
Treatment :- emergency , urgent heparinization
If occlusion embolic : embolectomy with Fogarty catheter , bypass graft (
post op heparinization required)
If occlusion thrombotic : I) intra arterial thrombolysis II) Angioplasty III)
bypass surgery
After thrombolysis or amputation if ischemia progressed treat underlying
cause .

981. An 8yo boy has his tonsils and adenoids removed. On the 7 post-op
day, he comes back to the ED with hemoptysis and fever. What is the most
appropriate management?
a.
Admit for IV antibiotics
b. Prescribe oral antibiotics and discharge
c. Packing
d. Surgery
e. Reassurance
Ans A
Hemorrhage between 1 to 2 weeks post op is called secondary hemorrhage
and is due to infection , depending on the severity of the condition the
patient should be admitted and managed on iv antibiotics .
th

982. A 50yo female had swelling in her ankles. She is a known alcoholic.
Now she presented with breathlessness and palpitations. What is the most
likely cause of her condition?
a. VT
b. SVT
c. A-flutter
d.
A-fib
e. V-ectopics
Ans D
Holiday Heart Syndrome : Holiday heart syndrome is an irregular
heartbeat pattern presented in individuals who are otherwise healthy. Coined
in 1978 the term is defined as arrhythmias sometimes following excessive
alcohol consumption; usually temporary"
.

Holiday heart syndrome can be the result of stress, dehydration, and drinking
alcohol. It is sometimes associated with binge drinking common during the
holiday season
Most Common Arryhtmias : Afib , Aflutter sometimes SVts.
Increased alcohol promotes alcoholic cardiomyopathy, heart failure and
arrhythmias.
S/S Alcoholic Cardiomyopathy
shortness of breath
swelling of the legs
rapid and irregular heartbeat

rapid and irregular pulse


fatigue, weakness, dizziness, fainting
an enlarged liver
cough that produces a frothy, pink mucus

983. A young boy has acute scrotal pain for a few hours. Exam: one testis
is very painful to touch. He had this kind of pain before but it was mild and
resolved itself within 30mins. What would you do next?
a.
Urgent exploration
b.
US
c. Antibiotics
d. IV fluids
e. Doppler US
Ans: A
Reason Testicular Torsion , usually affects unilaterally.typically sudden,
severe pain in one testis.abdominal pain , often comes during sports or
physical injury, history of previous, brief episodes of similar pain
(presumably a torsion that corrected itself ) nausea vomiting,
Diagnosis:
most important investigation is ultrasound integrated with colour Doppler. A
very significant finding is the detection of presence/absence of intratesticular
blood flow for the early identification of testicular torsion.
An acute scrotum in a child requires surgical exploration for a definitive
diagnosis
Management : It may be possible to reduce torsion manually , But the
Prefered option is always exploration.
984. An 8wk pregnant woman presents with persistent vomiting and
weight loss. Exam: HR=110bpm. Dehydration was corrected with NS
infusion and K+. The condition didnt improve so IM cyclizine was given.
She is still vomiting. What is the next appropriate management?
a. IV fluids
b. IV antiemetics
c.
IV steroids
d. Terminate pregnancy
e. Thiamine

Ans: C
Hyperemesis Gravidarum
intractable vomiting associated with weight loss of more than 5% of prepregnancy weight, dehydration, electrolyte imbalances, ketosis, and the need
for admission to hospital. Usually Occurs before 12 weeks of gestation.
Treatment:
Fluid Resuscitation with potassium replacement should be done
Vitamins should be routinely given , thiamine , pyridoxine
Nutritional support
thromboprophylaxis
Antiemetics to control vomiting
Corticosteroids to control intractable cases
985. A 28yo lady presents with dyspareunia and dysmenorrhea. She is
very obese. She now wants reversible contraceptive method. Which of the
following will be most suitable for her?
a.
Minera
b. COCP
c. POP
d. Copper T
e. Barrier method
Ans: A
Mirena preferred in Obese,diabetes,epilepsy,migraine and in women with
contraindication to oestrogens.Reduced menstrual loss and Dysmenorrhea ,
rapid return of fertility with removal.
Other Users: Menorrhagia and HRT (prevention of endometrial hyperplasia
during oestrogen therapy)
986. A young lady who is 28wks pregnant presents with vaginal
bleeding. She has lost about 200 ml of blood. Exam: uterus is tender.
Resuscitation has been done. What is the most imp inv to establish the dx?
a.
US
b. CT
c. D-dimer
d. Clotting profile
e. None
Ans: A

USG abdomen can easily visualize underlying cause of bleeding. To see


fetal status and uterus.
987. A 14yo girl presents with primary amenorrhea and a short stature.
What is the most likely dx?
a. Downs syndrome
b. Klinefeltners syndrome
c.
Turners syndrome
d. Fragile X syndrome
e. Normal finding
ANs: C
most likely it is Turner, which is associated with impaired pubertal growth
spurt and ovarian dysgenesis.
D/D Constitutional Growth failure
988. A 32yo woman wants reversible form of contraception. She has one
child delivered by emergency C-section. She also suffers from migraine and
heavy periods. What is the most suitable form of contraception for this lady?
a. COCP
b. Mini pill
c.
IUCD
d. Barrier method
e. Abstinence
Ans: C
Reason : Mirena preferred in Obese,diabetes,epilepsy,migraine and in
women with contraindication to oestrogens.Reduced menstrual loss and
Dysmenorrhea , rapid return of fertility with removal.
Other Users: Menorrhagia and HRT (prevention of endometrial hyperplasia
during oestrogen therapy)
989. A 45yo known hypertensive man presents with hematuria,
proteinuria and edema. What is the definitive dx test for him?
a. Urine protein
b. Renal biopsy
c. Renal function test
d. Urine microscopy
e. Serum protein
Ans: B

Pt seems to be suffering from nephritic syndrome , the definitive diagnosis


can be made on renal biopsy.
990. A 47yo man presents with proteinuria, BP=160/95 mmHg, small
kidneys that have smooth renal pelvis. What is the most probable dx?
a.
GN
b. Chronic pyelonephritis
c. Unilateral renal artery stenosis
d. Multiple myeloma
e. ARF
ANS : A
Reason : GN is a common cause of CKD, presenting with proteinuria,
hypertension and small kidneys.
991. You are the HO in the hospital and the lab report of a pt shows
glucose=4mmol/l, K+=5.2mmol/l, Na+129mmol/l. what is the most
appropriate management?
a.
NS 0.9%
b. NS 0.45%
c. NS 0.9% and insulin
d. Insulin
e. Dextrose
ANS: A
Reason: The immediate concern is correction of hyponatremia.
992. A 27yo man presents with abdominal pain. He says his urine is dark.
Exam: BP=160/105mmHg. What is the most appropriate inv?
a.
US
b. Renal biopsy
c. CT
d. Urine protein
e. Urine microscopy
Ans:A
USG will confirm
Abdominal pain, hematuria and hypertension are classic feature of
autosomal dominant polycystic kidney disease. The disease process usually

begins before the age of 30 yrs and renal failure are evident at about 60 yrs
of age
993. A 12m child with AIDS is due for his MMR vaccination. What is
the single most appropriate action?
a. Defer immunization for 2wks
b.
Dont give vaccine
c. Give half dose of vaccine
d. Give paracetamol with future doses of the same vaccine
e. Proceed with standard immunization schedule
ANS: B
Reason: Measles, mumps and rubella (MMR) vaccine is a freeze-dried
preparation containing live attenuated measles, mumps and rubella viruses
HIV-positive individuals. Severely immunocompromised patients should not
be given the vaccine but it is indicated for patients with mild-to-moderate
immunosuppression. The degree of immunosuppression is estimated using
the patient's age and CD4 count
994. A young man presents with sudden, severe pain and swelling in the
scrotum. Exam: one testis seems higher than the other. What is the most
probable dx?
a. Varicocele
b. Hematocele
c. Testicular tumor
d. Epidiymo-orchitis
e.
Testicular torsion
ANS: E
Reason:
History: Testicular Torsion , usually affects unilaterally.typically sudden,
severe pain in one testis.abdominal pain , often comes during sports or
physical injury, history of previous, brief episodes of similar pain
(presumably a torsion that corrected itself ) nausea vomiting
EXAM:usually reddening of the scrotal skin.There is a swollen, tender testis
retracted upwards.Lifting the testis up over the symphysis increases pain,
whereas in epididymitis this .usually relieves pain.testes on both sides are
characteristically in the 'bell-clapper position' with a horizontal long axis.

995. A 24yo male involved in RTA with XR: fx neck of humerus. What is
the single most associated nerve injury?
a.
Axillary nerve
b. Radial nerve
c. Median nerve
d. Ulnar nerve
ANS:A
Most common nerve injury associated with Humerus neck fracture is
Axillary Nerve injury.
996. A 64yo man complains of increasing SOB and cough for the past
18m. He coughs up a Tbsp of mucopurulent sputum with occasional specks
of blood. What is the most likely underlying cause?
a. Acute bronchitis
b.
Bronchiectasis
c.
Chronic bronchitis
d. Lung cancer
e. Pneumonia
ANS: B

It is usually accompanied by a chronic cough, mucopurulent sputum


production and recurrent infections
presentation:daily expectoration of large volumes of purulent sputum.
symptoms including dyspnoea, chest pain and haemoptysis
suspect bronchiectasis ,presents with Persistent productive cough, especially
if any one of the following:
Young age at presentation.History of symptoms over many years.Absence of
smoking history.Daily expectoration of large volumes of very purulent
sputum.Haemoptysis.Sputum colonisation with P. aeruginosa
Diagnosis: CXR normal or show ring or tubular opacities, tramlines and
fluid levels.
HRCT gold standard for diagnosis
Sputum Exam
Management: Physiotherapy

Antibiotic therapy in Acute Exacerbation. Amoxicillin, Clarithromycin,


Ciprofloxacin in P.aeruginosa.
Long term Antibiotic : pts having three or more exacerbations per year.
NOT RECOMMENDED Corticosteroids - inhaled or oral - unless there is
co-existent asthma.
Mucolytics.Leukotriene receptor antagonists.
Surgery:Lung resection surgery may be considered in patients with localised
disease in whom symptoms are not controlled by medical treatment
997. A 55yo man who is hypertensive suddenly lost his vision. The retina
is pale and fovea appears as a bright cherry red spot. What is the single most
appropriate tx?
a. Pan retinal photocoagulation
b. Corticosteroids
c. Scleral buckling
d. Surgical extraction of lens
e.
Pressure over eyeball
ANSS: E
Reason: Presentaion in Central Retinal Artery occlusion is painless loss of
vision unilaterally over a few seconds.Can be a HX of Amaurosis Fugax.
Exam:afferent pupillary defect, a pale retina with attenuation of the vessels.
segmentation of the blood column in the arteries ('cattle-trucking') and the
centre of the macula (supplied by the intact underlying choroid) stands out
as a cherry-red spot
Management: Presentation within 90-100 min Occular massage can be tried.
Paracentesis and acetazolamide to reduce intraocular pressure
Sublingual isosorbide dinitrate.
Oral pentoxyphylline
998. A 32yo man with schizophrenia and a hx of violence and distressing
auditory hallucinations was admitted to the ward with aggressive behavior
and has already smashed his room. He is refusing any oral meds. What is the
single most appropriate injection?
a. Flupenthixol
b. Fluphenazine
c.
Haloperidol
d. Paraldehyde

e.

Risperidone

ANS: C
Haloperidol indicated acute psychosis and violent settings,hyperactivity
aggression,hyperactive delirium,
Haloperidol can be used to treat acute psychosis and has proven efficacy for
agitation.benzodiazepines can decrease agitation and have efficacy similar to
Haloperidol but cause more sedation , Benzodiazepines other than
lorazepam and midazolam should not be administered IM because of erratic
absorption.
Ziprasidone, 20 mg IM, is well tolerated and has been shown to be effective
in decreasing acute agitation symptoms in patients with psychotic disorders.
Olanzapine is as effective as haloperidol in decreasing agitation in patients
with schizophrenia, with lower rates of EPS.Both IM ziprasidone and
olanzapine have a relatively rapid onset of action (within 30 minutes), which
makes them reasonable choices in the acute setting. Olanzapine has a long
half-life (21 to 50 hours); therefore, patients comorbid medical conditions,
such as cardiac abnormalities or hypotension, must be considered.If
parenteral medication is required, IM olanzapine or IM ziprasidone is
recommended. IM haloperidol with a benzodiazepine also can be
considered.
999. A 65yo man complains of hematuria, frequency, hesistancy and
nocturia. He reports that on certain occasions he finds it difficult to control
the urge to pass urine. Urine microscopy confirms the presence of blood but
no other features. What is the most porbable dx?
a.
BPH
b. Bladder ca
c.
Prostatic ca
d. Pyelonephritis
e. Prostatitis
ANS: C
Reason: Prostatic CA, presents with symptoms of LUTS initially and in
locally invasive disease there can be hematuria,dysuria,incontinence.
1000. A 60yo man presents with mass in the groin. Exam: mass lies below
the midpoint of the inguinal ligament and is pulsatile. What is the most
probable dx?
a. Direct inguinal hernia

b.
c.
d.
e.

Saphenavarix
Femoral hernia
Irreducible hernia
Femoral aneurysm

ANS: E
Femoral aneurysms:
These are the second most common peripheral aneurysm.Patients present
with local pressure symptoms, thrombosis, or distal embolisation.A pulsatile
mass can be felt in the groin
1001. An 82yo man has woken up with incoherent speech and difficulty in
finding the right words. Exam: otherwise normal, good comprehension.
Which anatomical site is most likely to be affected?
a.
Brocas area
b. Wernickes area
c. Midbrain
d. Parietal cortex
e. Pons
ANS: A
Reason: Brocas area is associated with motor part of speech, incoherent
speech, though the patient is able to understand speech.
Wernicke area is associated with sensory part of speech ,it affects
understanding and as well as speech production
1002. A 25yo woman has a recent cough, hoarseness and swelling in the
neck. There are several non-tender swellings on both sides of her neck. She
has lost 13kgs. She takes recreational drugs. What is the most probable dx?
a. Thyrotoxicosis
b. Hyperthyroidism
c. Vocal cord nodules
d. Carcinoma bronchus
e.
TB
ANS: TB
History and clinical picture suggests pulomonary TB spread locally and
distantly into lymph nodes.( non tender matted rubbery lymph nodes).Wt

loss cough. Immunocompromised , HIV, drug abusers are at increased risk


of developing
active TB.
Treatment Active Respiratory TB: 2month initial phase
Isoniazid+Rifampicin+erhambutol+pyrizinamide
4month continuition with Isoniazid+rifamipicin
Meningeal TB treated for 12 months : 2months
Isoniazid+Rifampicin+erhambutol+pyrizinamide
10 months Isoniazid+rifamipicin
Glucocorticoid with gradual withdrawal.
LTBI: 3month treatment with rifampicin + isoniazid
1003. A 30yo woman presents with acute headache. She complains of seeing
halos especially at night. What is the single most likely defect?
a. Paracentral scotoma
b. Monocular field loss
c.
Tunnel vision
d. Central scotoma
e. Cortical blindness
This is glaucoma
paracentral scotoma first, f/b a SEIDEL'S scotoma, f/b an arcuate and a double arcuate scotoma and finally
a tunnel vision leading to blindness.

1004. A 35yo man presents with a headache that worsens on bending his
head forward. What is the most likely dx?
a.
Chronic sinusitis
b.
SAH
c. Migraine
d. Cluster headache
e. Tension headache
Ans : A
Reason : None of the other type of headaches have any effect on bending.
1005. A 20yo man presents with painful swallowing. Exam: trismus and
unilateral enlargement of his tonsils. The peritonsillar region is red, inflamed
and swollen. What is the most appropriate tx?
a. Oral antibiotics

b.
c.
d.
e.

IV antibiotics and analgesics


I&D with antibiotics
Analgesics with antipyretics
Tonsillectomy

Ans:C/E
( Both Answers considered Equally Acceptable , But tonsillectomy should offer better
treatment)

Reason: The Patient Seems to be suffering from peritonsillar abscess


IV fluids and IV antibiotics along with analgesics should be
prescribed.Medical treatment alone is no longer considered sufficient,Needle
aspiration, incision and drainage and quinsy tonsillectomy are all considered
acceptable for the surgical management of acute peritonsillar abscess.Some
surgeons advocate acute (immediate) tonsillectomy as a treatment for
peritonsillar abscess.
1006. A 40yo manual worker presents with a swelling in the groin. Exam:
mass is found to be just above and lateral to the pubic tubercle. It is
reducible. On applying pressure on the internal ring there is no cough
impulse seen. What is the most probable dx?
a. Direct inguinal hernia
b.
Indirect inguinal hernia
c. Femoral hernia
d. Strangulated hernia
e. Femoral aneurysm
Ans: B
Ring occlusion done to confirm it
1007. A 34yo male presents with headache and vomiting. Exam:
temp=38.5C, neck stiffness, discharge from left ear and right sided
hyperreflexia with an extensor plantar response. What is the most likely dx?
a. Cerebral tumor
b. Meningitis
c. Cerebellar tumor
d.
Cerebral abscess
e. Normal pressure hydrocephalus
Ans: D

Reason:
unilateral (Right Sided)hyperreflexia and extensor plantar response indicates
upper motor type palsy, due to space occupying lesion( tumour/abscess) on
the left side. Headache & vomiting indicates raised intracranial pressure but
raised temperature and neck stiffness favours abscess/ meningitis. in
meningitis the aforementioned unilateral focal signs (hyperreflexia) usually
absent. left sided ear discharge indicates origin of abscess from left middle
ear. so roughly all symptoms indicates Left sided brain abscess
S/S Fever, headache, and neurological problems, while classic, only occur in
20% of people.
The symptoms of brain abscess are caused by a combination of increased
intracranial pressure due to a space-occupying lesion (headache, vomiting,
confusion, coma), infection (fever, fatigue etc.) and focal neurologic brain
tissue damage (hemiparesis, aphasia etc.). The most frequent presenting
symptoms are headache, drowsiness, confusion,seizures, hemiparesis or
speech difficulties together with fever with a rapidly progressive course. The
symptoms and findings depend largely on the specific location of the abscess
in the brain
Diagnosis: Ct brain , shows space occupying lesion after a few days there is
ring enhancement .
Management: IV antibiotics , sensitivity should be done . hyperbaric oxygen
therapy .

1008. A 26yo male presents with speech difficulties. Exam: nystagmus.


Which anatomical site is most likely to be affected?
a. Midbrain
b. Pons
c.
Cerebellum
d. Cerebrum
e. Vestibule cochlear nerve
ANS: C
Scanning speech , nystagmus, ataxia, dysdiadochokinesia, tremors,past
pointing.loss of balance.

1009. A 75yo man presents with Bells palsy. His PMH is significant for late
onset asthma and heart failure. He also reports to have consulted his GP for
generalized rash prv. CXR: multiple soft shadows and CBC: eosinophilia.
What is the single most likely positive antibody?
a.
P ANCA
b. C ANCA
c. Anti Ro
d. Anti DS DNA
e. Anti centromere
Ans: A
Reason: Churg Straus Syndrome
A rare diffuse vasculitic disease affecting coronary, pulmonary, cerebral,
abdominal visceral and skin circulations. The vasculitis affects small- and
medium-sized arteries and veins and is associated with asthma.
Presentation: Allergic rhinitis, Pulmonary Symp(ASthmna, granulomatous
infiltrates). Cardiac S/S (cardiac failure, myocarditis). Skin Manfestations (
Purpura , nodules, livedo reticularis). Glomerulonephritis, Peripheral
neuropathy,myositis.Bowel Ischemia,bleeding,perforation.
Investigations:
Antineutrophil cytoplasmic antibodies (ANCA): 70% of patients are
perinuclear staining (p-ANCA) positive (anti myeloperoxidase antibodies).
eosinophilia and anaemia on the FBC; elevated ESR and CRP; elevated
serum creatinine; increased serum IgE levels, hypergammaglobulinemia;
proteinuria, microscopic haematuria and red blood cell casts in the
urine.Chest X-ray: pulmonary opacities, transient pulmonary infiltrates,
pleural effusions.
Treatment: High dose steroids usually enough. Cyclophosphamide and
azathioprine in severe cases.
IV immune globulins +Plasma exchange
Overall prognosis good
1010. A 50yo man complains of visual prbs and dull pain in the left eye.
Fundoscopy reveals papilloedema. He was dx with MS 2yrs ago. There is no
consensual light reflex of the right eye. What is the single most likely
defect?

a.
b.
c.
d.
e.

Paracentral scotoma
Mono-ocular field loss
Homonymous upper quadrantanopia
Central scotoma
Homonymous lower quadrantanopia

ANS: B
Reason : As the Pt wa diagnosed with M.S, M.S mostly affects vision
unilaterally, with optic neuritis,papilloedema ,painful eye , decreased vision,
blindness or hemianopia.thus loss of consensual light reflex in the opposite
eye.
1011. A 54yo pt wakes up with right sided weakness. His current medication
is bendroflumethiazide for HTN. Pulse=92bpm, BP=160/90mmHg. CT
shows left cerebral infarct. What is the most appropriate tx?
a.
Alteplase
b. Aspirin
c. Clopidogrel
d. Dipyridamole
e. Simvastatin
Ans: A
Reason:Reason : If a patient with ischemic stroke presents within 4.5 hours
of ischemic episode thrombolysis can be tried. Do a CT 24 hrs Post
thrombolysis.after that patients are prescribed Aspirin 300 mg for 2 weeks.(
ischemic stroke is a gradual process and episodes might take place during
sleep)
CI:
major infarct or hemorrhage
mild deficit
Recent Birth,Surgery,Trauma
Past CNS bleed
Seizures at presentation
platelets <100
BP >220/130

1012. A 33yo man presented to the GP with hx of headaches and


photophobia. The GP examines him and finds a rash and is now ringing you
at the hospital for advice. What would you advise the GP?

a.
b.
c.
d.

Send pt home
Start IV benzylpenicillin
Conduct LP
Start IV ceftriaxone

ANS: B
Reason: Headache,photophobia and skin rash points towards developing
Meningococcal meningitis due to N.Meningitidis, the patient should be
immediately started on Iv Benzylpenicillin.
1013. An 89yo pt has lung cancer. His Na+=122mmol/l. What is the tx for
this?
a.
Demeclocycline
b. Vasopressin
c.
Restrict fluids
d. Reassure
ANS C
Common paraneoplastic syndrome due to small cell lung cancer is SIADH,
Treatment: Always treat the underlying cause
Hyponatremia is generally defined as a serum sodium level of less than 135
mEq/L and is considered severe when the serum sodium level is below 125
mEq/L
Midl hyponatremia present with suble unrecognized symptoms (131 mmol
or above ) change of gait,posture .
Neurological S/S develop below 115 mmol/L , seizures ,coma.
The development of S/S also depends upon how fast or severe the sodium
change is , sometimes in chronic settings pts are even able to handle severe
diminished sodium levels due to adaptation .
In asymptomatic Chronic setting : Fluid Restriction is firstline, If it is not
sufficient move towards drugs, DEMECLOCYCLINE (potent inhibitor of
vasopressin)
In acute not so severe
Hypovolemic : 0.9%/saline replacement .
Normovolemic: Fluid restrict (500-1000 ml/day) if inadequate consider
NaCl tablets or 3% saline .
Furosemide,demeclocycline,vaptans

In Acute (Symptomatic Patients) emergency : 3% NS should be given, the


target should be to increase serum NA <4-6 mmol/L over 1-2 hours before
more gradual correction. 0.5-1 meq Na/hr and 10-12meq/24 hrs.
1014. A 25yo woman who is 11wks pregnant had central abdominal pain for
36h. The pain is now colicky. There is no vaginal bleeding. She has vomited
once and has had an episode of loose motion. She looks ill, temp=37.8C and
there is rebound tenderness in the RIF. What is the most probable dx?
a. Salpingitis
b. PID
c.
Appendicitis
d. Ovarian torsion
e. Uterine fibroid
Ans: C
increased temperature, vomiting nausea, anorexia,pt usually constipated but
may have diarrhea,central abdominal pain which moves after few hours to
RIF,and rebound tenderness in RIF point towards appendicitis.
Alvardo Scoring
Symptoms: migratory RIF pain (scores 1), nausea or vomiting (1),
anorexia (1).
Signs: tenderness in the RIF (2), rebound tenderness in the RIF
(1), elevated temperature (1).
Laboratory findings: leukocytosis (2), shift to the left of
neutrophils (1).
From a total possible score of 10, one study recommended further
investigation with CT scan for a score of 4-6, and consideration of
appendicectomy for scores of 7 or above

1015. A 42yo man presents with stroke. He is not able to walk straight and
his speech is slurred. What is the initial appropriate inv?
a. CT brain
b. PET brain
c.
MRI brain
d. Carotid angiography
e. Monitor for 24h

ANS: C
Reason : Imaging for Cerebellar stroke
When neuroimaging is indicated, diffusion-weighted magnetic resonance
imaging (MRI) with magnetic resonance angiography is currently
considered the optimal study. For hemorrhagic strokes, computed
tomography (CT) and MRI are both excellent studies. However, for
ischemic strokes, MRI is clearly superior with an 83% sensitivity compared
to 26% for CT. Physicians should therefore not rely on CT scanning to rule
out cerebellar infarction
1016. A 24yo woman has severe depression 3m after the birth of her first
child. She is breastfeeding but is otherwise unable to look after the baby and
is convinced that her family is likely to kill her. She has no interest in
anything and keeps crying. What is the most appropriate tx?
a. Fluoxetine
b. Citalopram
c. CBT
d.
ECT
e. Haloperidol
Ans: D
The Patient is suffering from Postpartum psychosis ,Postpartum psychosis is
a severe mental illness which develops acutely in the early postnatal period.
It is a psychiatric emergency,
Symptoms may be depressive in nature (withdrawal, confusion, loss of
competence, distraction, catatonia) or manic (elation, lability, agitation,
rambling). There may be delusions (paranoia, jealousy, persecution,
grandiosity). There may be hallucinations which may be auditory, visual,
olfactory or tactile. There may be odd beliefs about the baby.Health
professionals must take into account the needs of the family and new baby,
as well as the risks of medication whilst breast-feeding
Management is Primarily Pharmacological , Using same guidance as for
other psychosis. Antipsychotics and Mood stabilising drugs. ECT is also
considered in some cases
As this patient is breast feeding ECT should be considrerd as there is risk of
drug secretion in breast milk.

1017. A 20yo woman with no prv hx of ear complains, presents with 1d hx


of severe pain in the right ear which is extremely tender to examine. What is
the single most likely dx?
a. Chondromalasia
b. Furuncle
c. Myringitis
d.
OE
e. OM
Ans: D
Reason : Acute Otitis Externa
his produces a similar temperature and lymphadenopathy. Swelling is more
diffuse and pain is variable with possible pruritus. Moving the ear or jaw is
painful. The canal, external ear, or both, are red, swollen, or eczematous,
with shedding of the scaly skin. There may be little,but thick, discharge in
the acute stage but it can become bloody if chronic. Hearing is often
impaired
Management:
clean ear canal, keep it dry,
Acetic acid 2% ear drops can be very effective against both bacterial and
fungal infection
Antibiotic drops may be used if infection is present. These are often given in
combination with steroids
1018. A couple has just finished their detox regime and wants a drug with a
pharmacological action to serve as a deterrent when they take alcohol. What
drug is the appropriate choice?
a. Disulfiram
b. Acamprosate
c. Vitamin supplement
d. Naloxone
e. Naltrexone
Ans: A
Disulfiram is used alongside other treatments and counselling for
alcoholism. It is only suitable for people who have been through
detoxification ('detox') and have stopped drinking alcohol. It acts as a
deterrent to drinking further alcohol

Drinking even small amount of alcohol produces a very unpleasant reaction,


disulfiram achieves this by blocking the enzyme which breaks down
acetaldehyde (intermediary product of alcohol metabolism)
Acamprosate is used for maintaining abstinence.
1019. A 68yo woman presents to the ED with confusion. Temp=39.3C and
productive cough. Sputum is rusty colored after 2 days. CXR shows right
lower lobe consolidation. What is the most likely organism?
a.
Streptococcus pneumonia
b. Staphylococcus aureus
c. Coxiella burnetti
d. Mycoplasma pneumonia
Ans: A
Reason:most common pathogen causing pneumonia in elderly, Rusty
Coloured Sputum, lobar cosolidation on cxr.
treatment : Amoxixillin 5-8 days
benzylpenicillin, cephalosporin
1020. A 70yo man with prostatic cancer has had severe acute back pain
waking him up at night for 6wks. What is the most appropriate inv?
a. MRI spine
b. Radionuclide bone scan
c. DEXA scan
d. Serum ALP concentration
e. Serum calcium concentration
Ans: B
Reason: A Radionuclide bone scan can detect tumor metastasis in
bones,diagnosis of fractures that may not be visible on traditional x-rays.a
nuclear bone scan is a functional test: it measures an aspect of bone
metabolism or bone remodelling , which is higher in a tumor abiding bone
tissue.
DEXA scan measures bone density which has significance in detecting
osteoporosis.
1021. An asymptomatic 56yo man who has never consumed alcohol came
for a routine checkup. Exam: increased skin pigmentation, spider angioma,

cardiomegaly, S3 gallop, liver firm with 8cm span, no ascites. He is in the


risk of which condition?
a. Cerebellar degeneration
b. Wernieckes encephalopathy
c. Renal failure
d.
Hepatoma
e. Hepatic vein thrombosis
Ans: D
The patient seems to be suffering from Hereditary Haemochromatosis, there
is a defect in HFE gene which affects iron metabolism by affecting hepcidin
and results in increased absorption of iron from intestine and increased
deposition in the body tissues. HH presents itself as Skin bronzing,
pancreatic insufficiency, hepatomegaly and later cirrhosis (These patients
are also at a higher risk of developing hepatocellular carcinoma)
cardiomyopathy, neurological or psychiatric symptoms.
1022. A 39yo male presents with visual symptoms. Ophthalmoscopy shows
papilloedema. Which anatomical site is most likely to be affected?
a. Optic nerve
b.
Optic disc
c. Optic radiation
d. Occulomotor nerve
e. Optic chiasma
Ans:B
Optic disc swelling can be caused by a number of conditions; papilloedema
relates more specifically to optic disc swelling secondary to raised
intracranial pressure.
Disc swelling is distinct from disc atrophy which refers to a loss of nerve
fibres at the optic nerve head and which results in a pale disc. Atrophy may
be primary (where it occurs without prior disc swelling) or secondary (where
it is preceded by disc swelling)
1023. A 75yo man has been attending the clinic for lower urinary tract
symptoms. His mood is very low and he says he feels unhappy, anxious and
unable to sleep. He has been dx with moderate depression. What tx would be
most effective for this pt?
a. Amitriptyline

b.
c.
d.
e.

Citalopram
CBT
Dosulepin
Diazepam

ANS; C
amitriptyline and Dosulepin both cause urinary retention, Citalopram causes
insomnia in more than 15% patients, patient is already suffering from both
problems.
MILD DEPRESSION:initial recommended treatment for mild depression is
CBT.
MODERATE-SEVERE: Offer antidepressant medication combined with
high-intensity psychological treatment (CBT or interpersonal therapy (IPT)).
For an individual with a chronic health problem and moderate depression,
this should be high-intensity psychological treatment alone in the first
instance
1024. A 48yo pt after surgical removal of mandibular ca presents with
perioral paresthesia and severe pain which is not relieved by oral morphine.
What is the next step in treating this pt?
a.Oral amitriptyline
b.Oral oxycodone
c.PCA
d.IV morphine
e.Fentanyl patch
f.Gabapentine
And: A
This is a neuropathic pain,According to NICE , offer a choice of
amitriptyline,duloxetine,gabapentin or pregabalin as initial treatment for
neuropathic pain.often switching to another drug if one fails.
First line antidepressants Amitriptyline
2nd line antiepileptics Gabapentin
3rd line Duloxetine
4th line Pregabalin.

1025. A 34yo man was slapped over his right ear in a fight. There is blood
coming from his external auditory canal and he has pain, deafness and
ringing in his ears. What is the most appropriate initial inv?
a.
CT
b. MRI
c. Otoscopy
d. Skull XR
e. Facial XR
Ans: C
Reason : traumatic perforation of the tympanic membrane can causes
pain,bleeding,hearing loss,tinnitus and vertigo. Diagnosis is based on
otoscopy ,treatment often is unnecessary.
Traumatic causes of preparation include
Insertion of objects
Concussion caused by an explosion or open handed slap across the ear
Head trauma with or without Mozilla fracture
Sudden negative pressure
Barotrauma
Diagnosis: otoscopy , audiometry
Management: often no treatment required ,for dirty injuries prescribe
antibiotics amoxicillin 500 mg tid 8 days.most perforation close
spontaneously ,those persisting more than 2 months require surgery .
1026. A 45yo man has developed an annular rash with a scaly edge on his thigh. The
rash has been spreading over the last 3wks. He has some general aches and pains.
What is the single most useful investigation?
a. ANA
b. Biopsy lesion
c. Lyme antibodies
d. Skin scrap for mycology
e. Skin swab for bacteria
Annular spreading rash points towards erythema migrans, which is characterisitic of
Lyme disease. Also accociated with general aches and myalgia.
Lyme disease caused by bacteria borrelia burgdorferi, spirochaete.
Humans may have it if bitten by tics infected by bacteria.
Presententation depends on stage of disease.
Stage 1, early/localized: characteristic is erythema migrans. (spreading annular rash
over 3-36 days, central clearing, target like appearance, bulls eye lesion) associated flu
like symptoms.

Stage 2, disseminated: occurs days to months later. Flu like symptoms. Neurological
involvement. Unilateral, bilateral facial nerve palsy, meningitis, encephalitis, peripheral
mononeuritis. Lymphocytic meningo rediculitis. Cardio vascular problems:
myo/pericarditis, heart block. Lymphocytoma. Cutaneous lesion.
Stage 3/late: arthritis in knee. Acrodermatitis chronic. Late neurological disorders.
Post lyme syndrome: similar to chronic fatigue syndrome.
Investigations: clinical diagnosis. Serology: antibodies. PCR.
Rx: Doxycycline 100mg/day. Or amoxicillin.
1027. A 80yo man with prostatic cancer has confusion, thirst, abdominal pain and an
abnormal ECG. What is the most appropriate inv?
a. MRI spine
b. Radionuclide bone scan
c. DEXA scan
d. Serum ALP concentration
e. Serum calcium concentration
man presenting with symptoms of hypercalcemia. Indicated by prostatic cancer with
mets.
Normal ca levels: 2.25-2.5mmol/L
Hypercalcemia: less than 2.8: ployuria, polydipsia, dyspepsia, depression,
At levels less than 3.5: all previous and muscle weakness, constipation, anorexia,
fatigue.
At levels >3.5: all prev and abd pain, vomiting, dehydration, arrhythmia, pancreatitis,
coma. Nephrocalcinosis.
Causes: malignancy and prim hyper PTH, sarcoisosis, TB, Endocrine conditions,
thiazide, vit D, Familial FHH, prolonged immobilization.
Investigations: Corrected Ca levels.
Rx: rehydrate with Normal Saline, furosemide. IV Bisphosphonates after hydration
.
1028. A 27yo lady after C-section developed epigastric pain after 8h. What is the
appropriate inv?
a. ABG
b. Coag profile
c. Liver enzyme
d. Liver biopsy
symptom is pointing towards liver abnormality. So C is most likely. Could be because of
HELLP syndrome characterized by hemolysis, elevated liver enzymes, low platelets.
Usually occurs in pregnant females of pre eclampsia and eclampsia. May present in last
half of pregnancy or shortly after delivery. Initial symptoms are non specific. Like
malaise, fatigues, epigastric pain, nausea
Investigation: CBC with peripheral smear, raised serum LDH, Bilirubin, liver enzymes.
Rx: deliver as soon as possible
If post partum then give steroids and plasma exchange.
Complications: DIC, pulmonary edema, renal failure, liver hemorrhage and failure.
Retinal detachment
1029. A 35yo woman presents with visual problems. CT brain reveals pituitary tumor.
What is the single most likely defect?

a. Homonymous hemianopia
b. Homonymous upper quadrantopia
c. Bitemporal hemianopia
d. Cortical blindness
e. Homonymous lower quadrantopia
dx: pituitary adenoma. Which presses on optic chiasm leading to bitemporal hemianopia.
Optic chiasm contains nasal fibres from both eyes.
If optic nerve is damaged it will lead to complete blindness in eye of same side.
If lesion is at optic chiasm, then bitemporal hemianopia
If the lesion is at optic tract, then contralateral homonymous hemianopia
If the Lesion is at temporal fibres of optic radiation, then contralateral upper quadrant
hemianopia. If parietal fibres are involved, then lower quadrant hemianopia.
Lesions in primary visual cortex then contralateral homonymous hemianopia with
macular sparing.
Cortical blindness is blindness with intact pupillary reflexes.
1030. A 45yo heroin addict was involved in a car crash and is now paraplegic. During
the 1st week of hospital stay he cried everyday because he couldnt remember the
accident. What is the most likely dx?
a. PTSD
b. Severe depression
c. Organic brain damage
symptoms are pointing towards brain damage.
It is a brain injury resulting from a medical cause and not a psychiatric cause. For eg
trauma, hemorrhage, concussion, hypoxia, hypercapnia, stroke, heart infections,
Alzhiemers, degenerative disorders, metabolic causes, kidney liver disease, drug and
alcohol.
Inv and Rx depend on underlying disorder.
Symptoms: agitation, confusion, dementia, delirium.
1031. A pt with T1DM has a fundus showing micro-aneurysm and hard exudate. What is
the single most likely dx?
a. Macular degeneration
b. Hypertensive retinopathy
c. MS
d. Diabetic background
e. Proliferative DM retinopathy
different stages of diabetic retinopathy:
Background retinopathy: microaneurysm, hemorrhagic blots, hard exudates
Pre proliferative: cotton wool spots, hemorrhage, venous bleeding
Proliferative: neo vascularisation
Maculopathy: visual acuity decreased, new vessels on optic disc, retinal detachment
1032. A 62yo man has multiple liver mets due to adenocarcinoma with an unknown
primary. He is deeply jaundiced and has ascites with edema upto the buttocks. He is
now drowsy and his family are worried that he is not drinking enough. His meds include:
haloperidol 1.5mg, lactulose 10ml. Bloods taken 3d ago: electrolytes normal,
urea=6.5mmol/l, creatinine=89mmol/l, calcium=2.04mmol/l, albumin=17g/L, total
bilirubin=189mmol/l. What is the single most appropriate management of his fluid intake?
a. Albumin infusion
b. Crystalloids IV

c. Crystalloids SC
d. Fluids via NGT
e. Fluids PO
scenario of decompensated liver disease as indicated by ascites, jaundice and
drowsiness. Fluid restriction should be done because of edema and ascites. His albumin
levels are low as normal value of albumin is 35-50g/L. ascites is to be managed with
fluid restriction, low salt, diuretics and daily weighing.
1033. A 2yo with atrophy of the buttocks, distended abdomen with frequent offensive
smelly stool. Choose the single most likely inv?
a. Upper GI endoscopy
b. Endomyseal/alpha glidin antibody
c. Sweat test
d. Colonscopy
e. Stool culture
Dx: celiac disease:
Immune mediated inflammatory systemic disorder provoked by gluten and prolamines in
genetically susceptible people. Gluten in wheat rye and barley.
Associated with HLA DQ2 and DQ8.
May present at any age. Babies and yung present after weening. Symptoms:
malabsorption, weight loss, failure to thrive, vomiting, anorexia, abd distension.
Older child presents with anemia, abd pain, malabsorption, mouth ulcers.
Dermatitis herpetiformis classic manifestation of skin Involvement.
INV: Anti TTG, Anti endomysial, anti gliadin antibodies.
IgA anti TTG preferred.
Antibodies used to monitor disease.
Confirmation by duodenal biopsy.
Rx: gluten restriction
1034. A 78yo woman is brought to the hospital complaining of back pain and is referred
to the surgeon. She has been saying that her mother is due to visit her today and that
somebody must have broken her lower back as she is in agony. Labs: creatinine=295
mmol/l, calcium=3.03mmol/l. Which inv is most likely to lead to a dx?
a. US KUB
b. XR Spine
c. IVU
d. Bence-Jones Protein
e. Mental state exam
points in favour of myeloma: Age, low back pain, increased creatinine and
hypercalcemia. XRAY spine would not lead to diagnosis and MSE would have been
done if the labs were normal.
Myeloma is due to abnormal proliferation of a single clone of plasma cells leading to
secretion og Ig immunoglobulin ot Ig fragment.
Symptoms: bone lesions, anemia, neutropenia, thrombocytopenia, recurrent infection,
renal impairment.
Dx made on:
serum or urine electrophoresis.
Plasma cells increased on marrow biopsy
End organ damage or bone lesion
Complications: hypercalcemia
Spinal cord compression
Hyperviscosity
Acute renal injury

Rx: supportive and chemotherapy. Allogenic tx.


1035. A 40yo woman presents with dysphagia. Exam: febrile with erythema and middle
neck swelling. What is the best management strategy?
a. IV antibiotics and drainage
b. Antipyretics
c. XR neck
d. Endoscopic diverticulectomy
e. I&D
presentation is that of a neck abscess. IV abx and InD forms the basis of Rx for neck
abscess.
1036. A young lady presents with gradually worsening headaches, visual disturbance,
and lack of energy. MRI shows 15mm tumor in the pituitary fossa. What is the tx of
choice?
a. Radiotherapy
b. Octreotide
c. Reassurance and f/u after 6m
d. Surgery
e. Chemotherapy
pituitary tumor: almost always benign and incurable. May be associated with MEN 1
syndrome. Types of tumors include
adenoma
prolactinoma
GH secreting
ACTH secreting
TSH, FSH, LH secreting
Symptoms depend on the hormone tht is being produced.
Local effects of tumor includes:: retro orbital headache, worse on waking up.
Obstruction of CSF resulting in hydroceph
Ocular nerve palsy causing squint
Disorder of thirst, appetite, temperature regulation if extended to hypothalamus.
INV: MRI
Rx: depends on type of tumor. Surgery in most cases
Bromocriptine in Prolactin secreting tumors.
Somatostatin analogues in GH secreting tumors
Small non functioning adenomas in asymptomatic pts dont require any Rx.
1037. A man with dementia has an ulcerative lesion on his forehead. He wants it
removed so it can help improve his memory. Wife says he is not fit to give consent.
What will you do?
a. Get letter signed from the GP
b. Get letter signed from the wife
c. Get letter signed from the pt
d. Refer to psychiatrist to assess the mental capacity to give consent
According to OHCM, consent in incapacitated requires a formal assessment to be
documented in medical notes. No one is able to give consent on behalf of any adult even
if he is incapacitated. So he should be referred to a psychiatrist.
1038. A pt with flame shaped hemorrhage on long term tx with nifedipine. What is the
single most likely dx?
a. Macular degeneration
b. HTN retinopathy

c. MS
d. DM background
e. Proliferative DM retinopathy
f. SLE
pt is on Ca channel blocker indicating he is hypertensive.
Fundoscopic findings in hypertensive retinopathy:
Grade 1: tortuous arteries with thick shiny walls, silver copper wiring
Grade 2: AV nipping
Grade 3: flame hemorrhages, dot and blot, hard soft exudates
Grade 4: papilledema
Rx. Control BP
1039. A pt whose pain is not relieved by oral codeine. What is the best management?
a. Oral oxycodone
b. Co-codamol
c. PCA
d. IV morphine
e. Oral morphine
pain management steps:
1. Non opiods: paracetamol, NSAID,
2. if not controlled with above, use weak opioids ie codeine and tramadol
3. Still not controlled then strong opioids like morphine, dimorphine, oxycodone, fentanyl.
Points in favour of morphine:
First line for severe pain in palliative care
Beneficial effect such as euphoria and detachment
4. IV morphine, pethidine and fentanyl patch if all of the above fail
1040. A 6wk baby with vomiting, irritability and palpable mass in the abdomen on
feeding. Choose the single most likely inv?
a. Upper GI endoscopy
b. Barium meal
c. US
d. CT abdomen
e. Barium enema
scenario of infantile hypertrophic pyloric stenosis. Presents at 3-8 weeks with vomits esp
after feed, large volume and projectile. Differentiating point btw other causes of vomiting
is that vomitus doesnt contain bile, no diarrhea but constipation. Alert, anxious. Hungry
o/e: left to right LUQ peristalsis during a feed. Olive sized pyloric mass present in RUQ
labs: water and NACL deficit, hypochloremic, hypo kalemic metabolic alkalosis.
Dx. Clinical. US may be done
Rx ramsdeths pyloromyotomy ot endoscopic surgery
1041. A 79yo man who is being treated with GnRH antagonist for proven
adenocarcinoma of the prostate attends a follow up session. What is the most
appropriate inv?
a. Serum AFP
b. Serum PSA
c. Serum acid phosphates conc
d. Serum ALP isoenzyme conc
e. Trans rectal US
urologists rely on rising PSA results to signal that a radical intervention (usually either
chemotherapy or radiotherapy) is necessary. This is particularly appropriate for older
patients with comorbidities, on the basis that they are likely to die of some other cause
before a slow-growing prostate tumour has an effect on their lifespan. Such 'active

monitoring' is also appropriate for any patient who wishes to avoid the side-effects of
interventional management.
Most prostate cancers are adenocarcinomas arising in the peripheral zone of the
prostate gland
Risk factors: Age
Black-african
Family hx
Factors such as food consumption, pattern of sexual behaviour, alcohol consumption,
exposure to ultraviolet radiation, chronic inflammation and occupational exposure have
all been considered as possible risk factors
* Local disease:
o Raised PSA on screening.
o Weak stream, hesitancy, sensation of incomplete emptying, urinary frequency,
urgency, urge incontinence.
o Urinary tract infection.
* Locally invasive disease:
o Haematuria, dysuria, incontinence.
o Haematospermia.
o Perineal and suprapubic pain. o Obstruction of ureters, causing loin pain, anuria,
symptoms of acute kidney injury or chronic kidney disease.
o Impotence.
o Rectal symptoms - eg, tenesmus.
* Metastatic disease:
o Bone pain or sciatica.
o Paraplegia secondary to spinal cord compression.
o Lymph node enlargement.
o Loin pain or anuria due to ureteric obstruction by lymph nodes.
o Lethargy (anaemia, uraemia).
o Weight loss, cachexia
* Abdominal palpation may demonstrate a palpable bladder due to outflow obstruction.
* DRE may reveal a hard, irregular prostate gland. Indications of possible prostate
cancer are:
o Asymmetry of the gland.
o A nodule within one lobe.
o Induration of part or all of the prostate.
o Lack of mobility - adhesion to surrounding tissue.
o Palpable seminal vesicles.
Differential diagnosis * All other causes of haematuria (eg, urinary tract infection) and
urinary tract obstruction. * Benign prostatic hyperplasia. * Prostatitis. * Bladder tumours.
INV:
PSA
Transrectal needle biopsy
Urinalysis to exclude renal and bladder pathology. Urine sent for microscopy, culture
and sensitivities.
Renal function tests to help exclude renal disease.
MRI should be considered for men with a negative TRUS core biopsy to determine
whether another biopsy is needed
MRI for staging
Bone scan for mets
The National Institute for Health and Care Excellence (NICE) referral guidelines for
suspected cancer state:[11]

* Men presenting with symptoms suggesting prostate cancer should have a DRE and
PSA test after counselling. Symptoms will be related to the lower urinary tract and may
be inflammatory or obstructive. Prostate cancer is also a possibility in male patients with
any
of the following unexplained symptoms: erectile dysfunction, haematuria, lower back
pain, bone pain or weight loss, especially in the elderly.
* Urinary infection should be excluded before PSA testing, especially in men presenting
with lower tract symptoms. The PSA test should be postponed for at least one month
after treatment of a proven urinary infection.
* If a hard, irregular prostate typical of a prostate carcinoma is felt on DRE, then the
patient should be referred urgently. The PSA should be measured and the result should
accompany the referral.
* Patients do not need urgent referral if the prostate is simply enlarged and the PSA is in
the age-specific reference range.
* In a man with or without LUTS and in whom the prostate is normal on DRE but the
age-specific PSA is raised or rising, an urgent referral should be made. Symptomatic
patients with high PSA levels should be referred urgently.
* If there is doubt about whether to refer an asymptomatic man with a borderline level of
PSA, the PSA test should be repeated after an interval of one to three months. If the
second test indicates that the PSA level is rising, the patient should be referred urgently.
Rx: low risk localized tumor: active surveillance/surgery (personal preference)
Intermediate to high risk:
Men with intermediate and high-risk localised prostate cancer should be offered a
combination of radical radiotherapy and androgen deprivation therapy, rather than
radical radiotherapy or androgen deprivation therapy alone.
Men with intermediate and high-risk localised prostate cancer should be offered 6
months of androgen deprivation therapy before, during or after radical external beam
radiotherapy.
Continuing androgen deprivation therapy for up to 3 years should be considered for
men with high-risk localised prostate cancer.
High-dose rate brachytherapy in combination with external beam radiotherapy should
be considered for men with intermediate and high-risk localised prostate cancer.
Brachytherapy alone should not be offered to men with high-risk localised prostate
cancer.
Locally advanced: pelvic radiotherapy
Metastasis: bilateral orchidectomy
Urinary tract obstruction, acute kidney injury, chronic kidney disease.
Sexual dysfunction: erectile dysfunction, loss of libido.
Metastatic spread: bone pain, pathological fractures, spinal cord compression.
1042. A middle aged woman has some weakness of hand after an injury. Which vertebra
will be the lowest to be included on cervical XR to dx the injury?
a. C7/T1
b. C8/T1
c. C5/C6
d. C6/C7
weakness of hand shows brachial plexus routes involvement ie C5-T1, so in Cervical
Xray, lowest vertebra included should be C7/T1 which will include all the nerve roots of
brachial plexus.

1043. A 50yo man with a known hx of stroke. He cant remember anything about his life.
What is the single most likely defect?
a. Homonymous hemianopia
b. Homonymous upper quadrantanopia
c. Bitemporal hemianopia
d. Binasal hemianopia
e. Homonymous lower quadrantanopia
Memory storing site is in the temporal lobe. Lesion of the temporal lobe leads to
homonymous upper quadrantanopia.
1044. An 18yo girl has been dx with anorexia nervosa and has mild depressive
symptoms. She has cut down her food intake for the last 18m and exercises 2h
everyday. Her BMI=15.5, BP=90/60mmHg. What would be the single most appropriate
management?
a. Refer to eating disorder clinic
b. Refer to psychodynamic therapy
c. Refer to acute medical team
d. Prescribe antidepressant
Answer should be A, according to OHCM. As the pt has moderate anorexia. Hence
should be referred to EDU rather than a medical unit as there are no severe symptoms
at present.
mild anorexia BMI >17.5: focus on building a trusting relationship and encourage use of
self help books and food diary. If no response within 8 wks, then consider referral to sec
care
moderate anorexia (BMI 15-17.5) routine referral to mental health team or adolescent
unit or eating disorder unit
severe anorexia (BMI <15, rapid wt loss, evidence of system failure): urgent referral to
eating disorder unit, medical unit
Admission to hospital should be considered if:
* There is risk of suicide or severe self-harm (acute psychiatric ward).
* Severe deterioration (may require admission to an acute medical ward).
* There is very low body weight (BMI less than 15 kg/m2) or rapid weight loss (admission
to an eating disorders unit may be most appropriate).
* Medical complications, such as pronounced oedema, severe electrolyte disturbance,
bradycardia, hypoglycaemia, or severe intercurrent infection (may require admission to
an acute medical ward).
* Home environment is impeding recovery.

Anorexia:
The defining clinical features are:
* Refusal to maintain a normal body weight for age and height.
* Weight below 85% of predicted. This means in adults a body mass index (BMI) below
17.5 kg/m2.
* Having a dread of gaining weight.
* Disturbance in the way weight or shape is experienced, resulting in over-evaluation of
size.
Other features include:.
* Other physical: includes fatigue, hypothermia, hypotension, peripheral oedema, gaunt
face, lanugo hair, scanty pubic hair, acrocyanosis (hands or feet are red or purple), and
bradycardia. Delay in secondary sexual development if pre-puberty.

* Symptoms such as fatigue, fainting, dizziness, constipation and intolerance of cold.


* Denial of the problem.
* Lack of desire for intervention, or resistance to it.
* Social withdrawal; few interests.
* Enhanced weight loss by over-exercise, diuretics, laxatives and self-induced vomiting.
Diagnosis:
SCOFF questionnaire (below) is a useful screening tool. Two or more positive answers
should prompt a more detailed history:
* Do you ever make yourself Sick because you feel uncomfortably full?
* Do you worry you have lost Control over how much you eat?
* Have you recently lost more than One stone in a three-month period?
* Do you believe yourself to be Fat when others say you are too thin?
* Would you say that Food dominates your life?
* Hypokalaemia: common and may cause fatal arrhythmias. * Hypotension. * Other
cardiac problems including arrhythmias, mitral valve prolapse, peripheral oedema,
sudden death. * Anaemia and thrombocytopenia. * Hypoglycaemia. * Osteoporosis:
restoring the patient's weight is the best treatment. Bone loss may never recover
completely even once weight is restored. * Constipation.
* Lack of growth in teenagers, and lack of development of secondary sexual
characteristics. * Infertility.
* Infections. * Renal calculi * Acute kidney injury or chronic kidney disease. * Alcoholism
in some patients. * Anxiety and mood disorders.
* Social difficulties.
1045. A 36yo woman has an injury to the right external laryngeal nerve during a thyroid
surgery. What symptom would be expected in this pt?
a. Stridor
b. Hoarseness
c. Aphonia
d. Dysphonia
e. Aphasia
stridor occurs in upper airway obstruction.
Hoarsness and aphonia in recurrent laryngeal nerve palsy
Dysphonia occurs in external laryngeal nerve pasly
Aphasia is a feature of stroke and any cortical lesion.
1046. A 75yo woman has weakness of the left side of her face. She has had a painful
ear for 48h. There are pustules in the left ear canal and on the eardrum. What is the
single most likely dx?
a. Chronic serous OM
b. Herpes zoster infection
c. Impacted earwax
d. Perforation of eardrum
e. Presbycusis
given symptoms particularly pustules point towards herpes infection. when the varicella
zoster virus (chickenpox) becomes reactivated in the geniculate ganglion of the VIIth
cranial nerve (facial nerve), it is called ramsay hunt syndrome.
ramsay Hunt syndrome:
presenting features:
* Vertigo and ipsilateral hearing loss.
* Tinnitus.

* Facial weakness or face drop.


* The patient also complains of rash or blisters which may be on the skin of the ear
canal, auricle or both, and may become infected secondarily, causing cellulitis
Signs
* There is a rash or herpetic blisters in the distribution of the nervus intermedius.
* The distribution of the rash varies, as does the area innervated by the nervus
intermedius. It may include the following:
o The anterior two thirds of the tongue.
o The soft palate.
o The external auditory canal.
o The pinna.
* An ipsilateral face drop or weakness may be obvious or it may be elicited on testing.
* There may be hyperacusis on that side due to paralysis of the stapedius and tensor
tympani.
* The patient may have associated ipsilateral hearing loss and balance problems.
The unilateral facial weakness is very similar to Bell's palsy but the rash is the
characteristic diagnostic feature to differentiate the two.
There is not always a rash, especially in younger patients. In children aged 5 to 15,
acute facial palsy, like a Bell's palsy and without a rash, may be produced by the
varicella-zoster virus.[6]
Trigeminal neuralgia is paroxysmal and tends to be precipitated by a stimulus such as
a cold wind or washing the face.
Other conditions in the differential diagnosis include postherpetic neuralgia, persistent
idiopathic facial pain and temporomandibular disorders.[7]
You may also consider otitis (external, media),[8] referred pain (eg dental abscess) and
carcinoma of the nasopharynx
Diagnosis: usually clinical. Occasionally audiometry and NCV for facial nerve damage.
1047. An 8wk baby boy is noted to be jaundiced. He is breast-feeting well and has
gained 300g since birth. His stools are yellow and his urine is pale straw colored. What
is the most likely dx?
a. Galactosemia
b. Biliary atresia
c. G6PD deficiency
d. Breast milk jaundice
e. Congenital viral infection
question is quite confusing as only some features point toward galactosemia. Firstly it
usually presents in neonatal period whereas this baby is 8 wks. However its variants
may present later in life. Secondly they feed poorly, but this is not the case here. Points
in favour of galactosemia are jaundice and poor wt gain.
Biliary atresia is acause of obstructive jaundice hence will cause dark urine and pale
stools, whereas here it is yellow stool and pale straw colored urine which points towards
galactosemia.
Breast milk jaundice is not associated with poor wt gain and the child would be healthy
Galactosemia:
* There is often feeding difficulty, with vomiting and failure to gain weight, with poor
growth in the first few weeks of life.
* Lethargy and hypotonia occur.
* Jaundice and hepatomegaly develop.
* There are often associated coagulation defects.
* Haemolytic anemia

* Metabolic acidosis
* Sepsis (often with Escherichia coli) can be fatal.
* Cataracts may be apparent even in the early days of life.
* Ascites may even be apparent in early life.
* The fontanelle is full.
* Developmental delay may affect speech, language and general learning.
* Adults may have short stature, ataxia and/or tremor.
* Hypergonadotrophic hypogonadism is common and in women, premature ovarian
failure. Those who conceive often have variant disease.
Galactosaemia should be considered when a term infant gets an E. coli sepsis and when
a neonate develops cataracts. It should also be considered with neonatal jaundice and
haemorrhage.
INV
Beutler's test involves a fluorescent spot test for GALT activity. It is now widely used for
the diagnosis of galactosaemia but will give false positives with glucose-6-phosphate
dehydrogenase deficiency.
positive test with Fehling's or Benedict's reagent
Rx
Remove milk from diet
Antibiotics, intravenous fluids and vitamin K are often required.
1048. A lady developed breast abscess after delivery. What is the most likely organism?
a. Staph aureus
b. Staph albus
c. GBS
d. Strep pyogenes
e. Strep faecalis
Dx: puerperal mastitis leading to abscess
The usual infecting organism is Staphylococcus aureus, although it may also be
Staphylococcus albus and streptococci. Meticillin-resistant Staphylococcus aureus
(MRSA) infection is increasing, and may be more common in women who have had a
caesarean section.
Dx: clinical
Rx
Incision and drainage of abscess with cavity packed open with gauze is recommended
if the overlying skin is thin or necrotic.
Parenteral antibiotics should be administered at the same time, with added coverage
for anaerobic bacteria. Fluid from the abscess should be cultured, and results used to
determine ongoing antibiotic treatment.
Needle aspiration of the abscess, repeated every other day until the pus no longer
accumulates, has been suggested as an alternative to open drainage.
In some cases breast-feeding may have to cease until the abscess is successfully
treated, but can usually resume later.
1049. A 32yo man suffering from MS presents with blurring of vision. Ophthalmoscopy
shows pallor of the optic disc. Which anatomical site is most likely to be affected?
a. Optic nerve
b. Optic disc
c. Optic radiation
d. Trigeminal
e. Oculomotor nerve
Optic neuritis is an acute, sometimes painful, reduction or loss of vision in one eye, and
is a relatively common presenting symptom of MS. Optic neuritis (ON) is inflammation of

the optic nerve. Classically there is a triad of clinical features - reduced vision (of varying
severity), eye pain (particularly on movement) and impaired colour vision.
Double vision
Facial weakness
Deafness
Depression
Taste and smell alteration
Loss of sensation in legstr. Myelitis
Urgency and frequency in passing urine
Impotence
Loss of thermoregulation
INV
Electrophysiology: can detect demyelination in apparently unaffected pathways with
characteristic delays. Visual evoked potential studies should be the first choice.
MRI scan: 95% of patients have periventricular lesions and over 90% show discrete
white matter abnormalities. Areas of focal demyelination can also be seen as plaques in
the optic nerve, brainstem and spinal cord.
Cerebrospinal fluid: rise in total protein with increase in immunoglobulin concentration
with presence of oligoclonal cases.
Rx:
Decrease stress
Steroid
Interferon
Monoclonal antibody, alemtuzumab
AZT for relapsing and remitting MS
1050. A 23yo man presents with severe pain in the right flank radiating to his groin. He is
rolling about on the floor. An IVU confirms a stone in the ureter which is 8mm in size.
Which tx modality will be most effective?
a. Fluids and alpha blockers
b. ESWL
c. CCB
d. Dormier basket
e. PCNL
Rx options for stone include:
Extracorporeal shock wave lithotripsy (ESWL) - shock waves are directed over the
stone to break it apart. The stone particles will then pass spontaneously.
Percutaneous nephrolithotomy (PCNL) - used for large stones (>2 cm), staghorn calculi
and also cystine stones. Stones are removed at the time of the procedure using a
nephroscope.
1051. A 37yo woman believes that her neighbours have been using her shower while
she is away from home. Her 42yo partner is convinced about this and calls the police.
What term best describes this situ?
a. Capgras syndrome
b. Cotard syndrome
c. Delusion of persecution
d. Folie a deux
e. Munchausen syndrome
Folie a duex is defined as a delusion or mental condition shared by two people in close
association.
Munchausen syndrome is when a person feigns illness to gain attention and sympathy.

Capgras syndrome is the irrational belief that a familiar person or place has been
replaced with an exact duplicate.
Cotards syndrome is the delusion where the patient believes that he/she is literally or
figuratively dead.
Delusion of persecution is a delusion where a person falsely believes that they are being
persecuted by someone who intends to do them harm.
Thus since both partners believe in the neighbors using their shower, the answer is folie
a duex.
1052. A 45yo woman has dull pain in her right ear which has been present for several
weeks. There is no discharge. Chewing is uncomfortable and her husband has noticed
that she grinds her teeth during sleep. The eardrum appears normal. What is the single
most likely dx?
a. Dental caries
b. Mumps
c. OM
d. Temporomandibular joint pain
e. Trigeminal neuralgia
Dx. Temporomandibular joint pain
Earache, facial pain, and joint clicking/popping related to malocclusion, teeth-grinding
(bruxism)
or joint derangement.
Stress making this a biopsychosocial disorder which may become a chronic pain
syndrome
Signs:
Joint tenderness exacerbated by lateral movement of the open jaw, or trigger
points in the pterygoids.
Imaging: MRI.
Associations: Depression; Ehlers Danlos
Rx: NSAIDs (PO or topical, eg Diclofenac); Stabilizing orthodontic occlusal prostheses;
cognitive therapy; physiotherapy; biofeedback; Reconstructive Surgery; acupuncture.
1053. A 42yo lady had corrective surgery for cyanotic congenital heart disease at the
age of 3y, after a palliative operation during infancy. There is a parasternal impulse and
an early diastolic murmur. What is the most probable dx?
a. Aortic regurgitation
b. Ischemic mitral regurgitation
c. Aortic stenosis
d. Pulmonary stenosis
e. Pulmonary regurgitation
Dx is pulmonary regurgitation.
42 yo lady has parasternal impulse and early diastolic murmur. Received corrective
surgery for cyanotic congenital heart disease (pulmonary hypertension, Eisenmenger
syndrome)
Early diastolic murmur only in aortic regurgitation or pulmonary regurgitation.
Causes of pulmonary hypertension also cause pulmonary regurgitation.
{Graham steell murmur if mitral stenosis+ pulmonary hypertension}
1054. A 45yo lady presents with hx of double vision and facial numbness. Which
anatomical site is most likely to be affected?
a. Cerebral cortex
b. Trigeminal nerve
c. Oculomotor nerve
d. Brain stem

e. Basal ganglia
Dx is brain stem.
Vertibobasilar circulation. Supplies the cerebellum, brainstem, occipital lobes. Occlusion
causes signs relating to any or all 3: hemianopia; cortical blindness; diplopia; vertigo;
nystagmus; ataxia; dysarthria; dysphasia; hemi- or quadriplegia; unilateral or bilateral
sensory symptoms; hiccups or coma.
1055. A 30yo woman has experienced restlessness, muscle tension and sleep
disturbance on most days over the last 6m. She worries excessively about a number of
everyday events and activities and is unable to control these feelings which are
impairing her ability to hold down her job. What is the most likely dx?
a. Panic disorder
b. GAD
c. Pheochromocytoma
d. Acute stress disorder
e. Social phobia
Dx is GAD.
GAD is anxiety and +3 somatic symptoms present over a course of 6 months.
Panic disorder is the experience of intense anxiety along with 4 symptoms of autonomic
hyperactivity lasting less than 30 mins.
Acute stress disorder is the experience of symptoms by a person under a maximum
period of 1 month following exposure to a traumatic event.
Social phobia is the fear of a situation where something potentially embarrassing might
happen.
Causes of GAD are genetic predisposition, stress and events involving stress.
Treatment of GAD is through symptom control, exercise, meditation, behavioural
therapy, hypnosis and various drugs such as benzodiazepines, ssris, azapirones, beta
blockers and antihistamines.
Prognosis gets better by age 50 years.
1056. Which of the following is not a degenerative corneal disease?
a. Band keratopathy
b. Marginal dystropathy
c. Fatty/lipid degeneration
d. Moorens ulcer
e. Keratoconus
Dx is Moorens ulcer.
Band keratopathy is characterized by the appearance of a band across the central
cornea, formed by the precipitation of calcium salts on the corneal surface (directly
under the epithelium). This form of corneal degeneration can result from a variety of
causes, either systemic or local, with visual acuity decreasing in proportion to the density
of the deposition.
Pellucid marginal degeneration is a degenerative corneal condition, often confused with
keratoconus.
Keratoconus is a degenerative disorder of the eye in which structural changes within the
cornea cause it to thin and change to a more conical shape than the more normal
gradual curve.
Moorens ulcer is a chronic, painful peripheral corneal ulcer of unknown cause that easily
leads to loss of vision. Severe pain, red, tearing and photophobic.
Fatty/lipid degeneration is degenerative.
In moorens ulcer severe pain is common and eye(s) may be very red, photophobic, and
tearing. It is more common in southern and central Africa, China, and India. Treatments
tried: steroidal and nonsteroidal anti-infl ammatory drops, cytotoxics (topical and

systemic), conjunctivectomy, and cornea debridement (superficial keratectomy). None is


known to be superior
1057. A 30yo man presents to hosp complaining that his urine has been very dark
recently, resembling coffee at worst. He has been under the weather 2wks back and had
taken a few days off work with a sore throat and coryzal symptoms. Urine dipstick in
hosp returns highly positive for blood and protein. He is admitted for supportive
management and is scheduled for a renal biopsy, which shows mesangial proliferation
with a positive immune-flurescence pattern. What is the most probable dx?
a. Membranous glomerulonephropathy
b. SLE
c. Wegeners granulomatosis
d. Post strep GN
e. IgA nephropathy
Dx is Post strep GN
History of sore throat, mesangial proliferation and immune flouresence pattern point to
post strep GN.
Presentation is usually nephritic syndrome.
Renal biopsy isnt performed unless atypical presentation.
IF shows IgG and C3 deposits
Serology shows inc ASOT and inc C3
Supportive treatment with more than 95% function recovered
1058. A 65yo lady presents with a 6h hx of facial droop and weakness in the left side of
her body. What single agent will she be prescribed for her whole life?
a. Clopidogrel
b. Altepase
c. Aspirin
d. Labetalol
Rx is clopidogrel.
65 yo lady with facial droop and weakness on left side is suggestive of a stroke.
1059. A 10yo boy is brought to the hosp with a rash over his buttocks a/w abdominal
pain and vomiting. In the ED, he is accompanied by his mother and stepfather. His
mother had left him for the weekend with the stepfather and was called to come back
from holiday as he started to have some hematuria with the rash. Social services had
been notified on arrive to hospital. What is the most probably dx?
a. NAI
b. ITP
c. HSP
d. ALL
e. HUS
HSP is a Small vessel vasculitis with purpura (non blanching purple papules)-buttocks
and extensor surfaces. Young. Glomerulonephritis, arthritis, abd pain (+_
intussusception) may mimic an acute abdomen. Rx is supportive.

1060. A man with hx of fall had confusion and laceration mark on the head. Which is the
most appropriate vessel affected?
a. Basilar artery
b. Middle meningeal artery
c. Vertebral artery
d. Diploic vein
The history of fall and laceration mark suggests an extradural hemorrhage. Therefore,
artery affected would be middle meningeal.
Test : CT scan to confirm hemorrhage.
Stabilize and transfer urgently (with skilled medical and nursing escorts) to a
neurosurgical unit for clot evacuation ligation of the bleeding vessel. Care of the airway
in an unconscious patient and measures to ICP often require intubation and ventilation
(+ mannitol IVI
Prognosis Excellent if diagnosis and operation early. Poor if coma, pupil abnormalities,
or decerebrate rigidity are present pre-op
1061. A 72yo lady is drowsy and her relatives want to take her home. She has been
prescribed diazepam 2.5mg. What is the best delivery route?
a. Oral
b. IV
c. IM
d. Per rectal
e. SC
Delivery is Per rectal.
Diazepam is not absorbed properly via the oral or IM route.
The IV route presents difficulties for non-medical carers.
The SC route is contraindicated in old people.
1062. A nonsmoker who has worked in coal mines for 20yrs presents with gradually
increasing SOB, limited exercise tolerance and a dry cough. His CXR shows round
fibrotic tissue demonstrating a mixed restrictive and obstructive ventilator defect with
irreversible airflow limitation and reduced gas transfer. What is the single most
appropriate dx?
a. Churg-strauss syndrome
b. Cryptogenic organizing
c. Extrinsic allergic alveolitis
d. Good pastures syndrome
e. Progressive massive fibrosis
f. Molluscum
Key: E
Page 192, Industrial Dust Disease; Chest Medicine; OHCM 9TH Edition.
Coal Workers Pneumonia (CWP): Underground Coal Mines. Over 15-20 years. Fibrosisround opacities (1-10 mm) esp in upper zone.
Clinical features: asymptomatic (co existing chronic bronchitis common)
PMF: Due to progression of CWP, which causes progressive dyspnoea, brosis, and,
eventually, cor pulmonale.
CXR: upper-zone brotic masses (110cm).
Management: Avoid exposure to coal dust; claim compensation.
[In Extrinsic AA: farmer- mushroom worker, bird fancier, malt worker, sugar or
bagassosis worker
4-6h post exposure: Fever, rigors, myalgia, dry cough, dyspnoea, crackles
In IPF (CFA): Dry cough, exertional dyspnea, dec weight, arthralgia, cyanosis, clubbing]

1063. A pt was complaining of pain within 6h after his appendectomy for gangrenous
appendix. What med is the best option for his pain relief?
a. IV morphine
b. Diclofenac per rectal
c. PCA
d. Tramadol
Rx is Tramadol
Who Pain Ladder
Rung 1 Non-opioid Paracetamol; NSAIDS
Rung 2 Weak opioid Codeine; dihydrocodeine; tramadol
Rung 3 Strong opioid Morphine; diamorphine; hydromorphone; oxycodone; fentanyl;
buprenorphine ( adjuvant analgesics)
Tramadol as pain is of moderate severity.
Morphine is for severe pain.
Diclofenac is for mild pain.
PCA is not a drug.
1064. A 62yo farmer presents with a persistent firm irregular lesion on upper part of
pinna which grew over the last few months. What is the most appropriate dx?
a. Basal cell
b. Squamous cell
c. Keratocanthoma
Dx is basal cell
Basal cell is most common.
Squamous and keratocanthoma are less common and similar in presentation.
GOLJAN PATHOLOGY it says that such lesions above the upper lip is basal cell. And
below that is SCC- if on face.
1065. A 24yo schizophrenic has been under antipsychotic tx for the last 1 yr and now
complains of Erectile Dysfunction. Which drug is most likely to have caused this?
a. Fluoxetine
b. Citalopram
c. Clozapine
d. Haloperidol
e. Risperidone
The answer here should be E Risperidone. C,D and E are the antipsychotics from the
options. OHCS states that atypical antipsychotics cause erectile dysfunction so
haloperidol goes out of the race. Since there are great chances of agranulocytosis by
using clozapine and the first choice is risperidone for schizo so the patient would have
been advised Risperidone which has caused ED.
1066. What is the most likely dx based on this ECG?
a. Normal
b. VT
c. Sinus Tachycardia
d. WPW syndrome
e. A-fib
Key: C
No other apparent abnormality
1067. A 45yo woman has recently been dx with MS and has been started on oral
steroids. She is brought to the hosp after having ingested 100 paracetamol tablets 4h
ago. She is refusing all med tx. What is the next best step?

a. Observe
b. Refer to psychiatrist to assess pts ability to refuse tx
c. Gastric lavage
d. Activated charcoal
e. Refer to social worker
Key says B.
Not a very clear answer though something like this given on:
Page 403, Psychiatry; OHCS 9TH Edition.
1068. A 44yo obese pt with findings: FBS=6 mmol/l, OGTT=10 mmol/l. What is the most
likely dx?
a. Impaired glucose tolerance
b. Diabetes insipidus
c. T1DM
d. T2DM
e. MODY
Dx is IGT
IGT: fasting <7 and OGTT >7.8 but <11.1
WHO criteria (for DM):
* Symptoms (polydipsia, polyuria, weight loss, lethargy, blurring, genital thrush)+ fasting
>7 and random >11.1
* Raised venous fasting >7 and random >11.1- on 2 separate occasions or OGTT> 11.1
* HB 1Ac> 48mmol/L (6.5%)
1069. A child distressed with fever, stridor and unable to swallow saliva. His RR=40bpm.
What is the initial step that needs to be taken?
a. Examine throat
b. Secure airway
c. Keep him laid flat
d. IV penicillin
Key: B
Page 158, URTI; Paeds; OHCS 9TH Edition.
DD for Stridor is Epiglotitis, bacterial tracheitis and Viral Croup. Based on symptomology
its epiglotitis [croup has barking cough and hoarseness, Bac Tracheitis has
mucopurulent exudates not cleared by coughing]
Cause of risk of obstruction in both tracheitis and epiglotittis, we first call the anesthetist
to secure the airway. [after first laryngoscopy that shows cherry red swollen epiglottis ]
Then 3rd generation Cefotaxime. For tracheitis, give additional flucloxacillin.
1070. A pt presents with hemoptysis 7d after tonsillectomy and adenoidectomy. What is
the next step of management?
a. Explore again
b. Pack it
c. Oral antibiotics and discharge
d. Admit and IV antibiotics
e. Ice cream and cold fluid
Key: D
Page 565, Tonsillectomy; ENT; OHCS 9TH Edition.
Primary: within 24 hr needs return to theatre. Secondary: from after 24 hrs to 5-10 days
post surgery is due to infections. Needs admission and IV antibiotics [along with HO
gargles and vasoconstrictors]
When severe hemorrhage- Admit, O2, IVI, crossmatch, antibiotics (co-amoxiclav)
1071. A 55yo man presents with swelling at the angle of the mandible which is
progressively increasing in size and its mobile for 6m. What is the most probable dx?

a. Benign parotid
b. Mandible tumor
c. Tonsillar carcinoma
Key: A
Page 578, Salivary Gland Tumours; ENT; OHCS 9TH Edition.
80% - benign pleomorphic adenoma, parotid gland and in superficial lobe.
Middle aged, man, parotid gland, slow growth and mobile: all favoring a benign parotid
swelling like Pleomorphic Adenoma.
Rx: Removal by superficial parotidectomy or Enucleation.
1072. A 61yo man, known smoker, comes to the hospital with complaints of painless
hematuria, urgency and dysuria. He has been worried about his loss of weight and
reduced general activity. Which inv would be diagnostic of his condition?
a. Urine microscopy
b. IVU
c. CT
d. Cystoscopy
e. US abdomen
f. KUB
g. Cystoscopy with biopsy
h. Mid stream urine for culture
i. Transrectal US
Key: G
Page 648, Bladder Tumors; Surgery; OHCM 9TH Edition.
Transitional Cell Carcinoma: painless hematuria, frequency urgency and dysuria ; UTI;
UTO plus Smoking is considered one of the important causative factors to TCC.
Cystoscopy with Biopsy is confirmatory
1073. An 8wk pregnant lady is brought to the ED due to severe vomiting. She was
administered IV fluids and oral anti-emetics. She still cant tolerate anything orally. What
is the next best tx?
a. IV feeding
b. IV antiemetics
c. Termination of pregnancy
d. PPI
e. IV steroid
Tx is IV anti emetics.
Woman may be progressing towards Hyperemesis Gravidarum.
IV feeding, IV steroids and PPIs are options if IV antiemetics fail to work
Termination is only the last option.
1074. A 48 yo man presents with bone pain. Labs: ALP=high, phosphate=normal. What
is the most likely dx?
a. Osteoporosis
b. Osteomalacia
c. Pagets disease
d. Fx
e. Myeloma
Key: C
Page 699, Clinical Chemistry; OHCM 9TH Edition.
Also known as osteitis Deformans. There is Inc bone turnover ass with osteoclastic and
osteoblastic activity causing brittle bones and bone pain. Mostly pts over 40 and Ca and
PO4 are normal. ALP is raised

1075. A 54yo lady presents with sudden severe pain in the left half of her skull. She also
complains of pain around her jaw. What is the most likely next step?
a. CT
b. MRI
c. Fundoscopy
d. ESR
e. Temporal artery biopsy
Key: D
Page 558, Vasculitis; Rheumatology; OHCM 9TH Edition.
Giant cell arteritis (GCA)= cranial or temporal arteritis. Common in the elderlyconsider
Takayasus if under 55yrs.
It is associated with PMR in 50%
Symptoms: Headache, temporal artery and scalp tenderness (eg when combing hair),
jaw claudication, amaurosis fugax, or sudden blindness, typically in one eye.
Extracranial symptoms may include dyspnoea, morning stiffness,and unequal or weak
pulses.
If you suspect GCA, do ESR and start prednisolone 60mg/d PO immediately then go for
temporal artery biopsy which is definitive.
1076. A 7yo school boy has been dx with meningococcal meningitis. What is the advice
for schoolmates and staff?
a. Rifampicin for the whole class and family
b. Rifampicin for the whole school and family
c. Meningococcal vaccine for the family
d. Benzylpenicillin
e. IV cefotaxime
Key is A
Prophylaxis: household contacts in droplet range or Those who have kissed the patient's
mouth. Give rifampicin (600 mg 12 hrly PO for 2 days).
1077. A pt came with dyskaryosis to the OPD. She is a heavy smoker and alcoholic.
Cervical smear shows abnormal cells. What is the best advice for her?
a. Colposcopy
b. Biopsy
c. Endocervical sample
d. Repeat after 4m
e. None
f. Cone biopsy
If there is class 3 mild moderate or class 4 severe dyskaryosis on smear, the next step is
ro refer the lady for colposcopy, and if needed punch biopsy.
Class 1
Normal pap smear: repeat in 3 years
Class 2
Inflammatory pap smear: Take swab and treat infection. Repeat in 6 months.
Colposcopy after 3 abnormal smears
Mild atypia: repeat in 4 months. Colposcopy after 2 abnormal smears
Class 3
Mild dyskaryosis: HPV test +/- colposcopy
Moderate dyskaryosis: colposcopy
Class 4
Severe dyskaryosis: colposcopy

Class 5
Suspected invasion and abnormal glandular cells: urgent colposcopy
1078. Pt with pain and swelling in left leg and thigh up to the level of inguinal ligament.
Where is the level of occlusion?
a. Femoro-popliteal artery
b. Left common iliac artery
c. Aortoiliac artery
d. Femoral artery
e. Profound femoral artery
The location of the pain in patients with peripheral arterial occlusive disease (PAOD) is
determined by the anatomic location of the arterial lesions. PAOD is most common in the
distal superficial femoral artery (located just above the knee joint), a location that
corresponds to claudication in the calf muscle area (the muscle group just distal to the
arterial disease). When atherosclerosis is distributed throughout the aortoiliac area, thigh
and buttock muscle claudication predominates.
In this scenario, there is pain in left leg and thigh, so femoral artery is more likely to be
occluded in this patient. Risk factors:Smoking, Diabetes mellitus. Hypertension.
Hyperlipidaemia: high total cholesterol and low high-density lipoprotein (HDL) cholesterol
are independent risk factors. Physical inactivity. Obesity.
The most common symptom is muscle pain in the lower limbs on exercise (intermittent
claudication):
* Walking impairment - eg, fatigue, aching, cramping or pain in the buttock, thigh, calf or
foot, particularly when symptoms are quickly relieved at rest. Pain comes on more
rapidly when walking uphill than on the flat. Claudication can occur in both legs but is
often worse in one leg.
* Similar pain may occur in the buttocks and thighs, associated with absent femoral
pulses and male impotence (Leriche's syndrome; caused by aorto-iliac obstruction).
* The differential diagnosis of pain in the lower limb when walking includes sciatica and
spinal stenosis, deep vein thrombosis, entrapment syndromes and muscle/tendon injury.
* The main method to confirm the diagnosis is Doppler ultrasonography (duplex
scanning). The ratio of systolic blood pressure at the ankle and in the arm - anklebrachial pressure index (ABPI) - provides a measure of blood flow at the level of
the ankle (as a general guide, normal = 1, claudication 0.6-0.9, rest pain 0.3-0.6,
impending gangrene 0.3 or less). The ABPI is a strong marker of cardiovascular
disease and is predictive of cardiovascular events and mortality
* Treatment includes reduction and modification of risk factors, medical management
with anti platelets and peripheral vasodilators. Surgical procedures such as
endovascular revascularization and bypass surgery.
1079. A 65yo man presents with dyspnea 3d after an MI. On auscultation he has a
pansystolic murmur at the apex radiating to the axilla. What is the most likely dx?
a. Ruptured papillary muscle
b. Ventricular aneurysm
c. Pericarditis
d. Pericardial effusion
e. VSD
Complications post MI
Cardiac arrest, unstable angina, bradycardia or heart block, tachyarrhythmias, right
ventricular failure, pericarditis, DVT and PE, systemic embolism, cardiac tamponade,
mitral regurgitation, VSD, late malignant vent arrhythmia, dresslers syndrome, left
ventricular aneurysm.

In this scenario, patient has presented to us post MI with complains of dysnea


(pulmonary edema). Auscultation reveals a pansystolic murmur at apex radiating to
axilla characteristic of MR murmur. MR post MI occurs as a result of papillary muscle
dysfunction (mild MR) or papillary muscle or chordal rupture or ischemia (severe MR).
Vsd will also present with pansystolic murmur but at lower left sternal edge.
Pericarditis and pericardial effusion with muffled heart sounds and pericardial pain
relieved by sitting forward.
Ventricular aneurysm occurs late after 4-6 weeks. Presents with LVF, angina, recurrent
VT or systemic embolization. Persistent ST segment elevation. Treatment is excise and
coagulate.
1080. A 64yo man with multiple myeloma has been vomiting since the past 2days. Labs:
Ca2+=3.2mmol/l, K+=5mmol/l, Na+=149mmol/l and PCV=55%. What is the most
appropriate next step?
a. IV insulin
b. IV calcium gluconate
c. IV fluids
d. IV bisphosphonates
e. Oral bisphosphonates
Multiple myeloma, vomiting point toward diagnosis of hypercalcemia indicated by raised
serum calcium and dehydration indicated by raised sodium, potassium and increased
pcv. There has been inadequate water replacement as a result of vomiting.
Treatment is to give IV fluids initially
Iv bisphosphonates may be used later
Iv furosemide may also be used after rehydration. Acute hypercalcaemia[11]
Treatment should be initiated in hospital on the advice of a specialist and should include:
* Increasing the circulating volume with 0.9% saline, helping to increase the urinary
output of calcium.
* A loop diuretic such as furosemide. This is occasionally used where there is fluid
overload but it does not reduce serum calcium .
* After rehydration, bisphosphonates (which act by reducing bone turnover) should be
administered intravenously. Pamidronate and zoledronic acid are commonly used.
Salmon calcitonin may also be given. It has fewer side-effects than bisphosphonates but
is less effective in reducing hypercalcaemia.
* Glucocorticoids are useful for hypercalcaemia due to vitamin D toxicity, sarcoidosis and
lymphoma.
* Gallium was identified as a useful drug when it was found that patients with malignancy
having gallium scans did not develop hypercalcaemia. It may be given intravenously to
patients with malignant hypercalcaemia who do not respond to bisphosphonates.[14]
* Cinacalcet hydrochloride is a calcimimetic (= mimicking the action of calcium) agent
that effectively reduces parathyroid levels in patients with secondary
hyperparathyroidism.[12]
* A new experimental approach to malignancy-associated hypercalcaemia involves the
blockade of receptor activator of nuclear factor kappa-B ligand, usually abbreviated as
RANKL. RANKL is a key element in the differentiation, function and survival of
osteoclasts, which plays an essential role in removing calcium ions from the bone in
response to PTH stimulation.[15] Denosumab, a human monoclonal antibody that acts in
this manner, is licensed for the prevention of osteoporotic fractures but is also useful for
patients with persistent or relapsed hypercalcaemia of malignancy.[16]
* Haemodialysis or peritoneal dialysis may be relevant in patients with severe
hypercalcaemia secondary to renal failure.

Non-PTH-mediated hypercalcaemia
Treatment depends on the underlying condition. PTH-mediated hypercalcaemia[8]
* Asymptomatic patients may be treated conservatively with regular monitoring of bone
density, renal function and serum and urinary calcium levels.
* For symptomatic patients, dietary calcium should be reduced - eg, minimise the intake
of dairy products and leafy vegetables. This approach has been questioned for
asymptomatic patients, in whom 1000-1200 mg calcium daily has been
recommended.[17]
* Bed-bound patients should be mobilised if possible. Symptomatic patients will respond
well to having the affected part of the parathyroid gland removed.
* There is no consensus on the operative treatment of asymptomatic patients. In
general, it tends to be reserved for patients who have impaired renal function,
hypercalciuria, low bone mineral density or severe hypercalcaemia
1081. A 30yo man from Australia returned from a business trip to Indonesia 6d ago
presenting with complaints of fever, joint and muscle ache and headache, in particular
behind the eye for the past 2 days. What is the most probable dx?
a. Malaria
b. Chicken pox
c. TB
d. Lymes disease
e. Dengue
a. Dengue
Fever, joint and muscle ache and headache behind eye, hx of travel to Indonesia all
point towards diagnosis of Dengue fever.
Symptoms
* Haemorrhagic fever syndromes begin with abrupt onset of fever and myalgia.
* Fever is associated with frontal or retro-orbital headache accompanied by onset of a
generalised rash.
* Symptoms regress for a day or two but may recur, although fever is rarely as high as at
the onset.
* Dengue fever cases experience severe bony and myalgic pain in legs, joints and lower
back which may last for weeks (hence, breakbone fever).
* Nausea, vomiting, cutaneous hyperaesthesia, taste disturbance and anorexia are
common.
* Abdominal pain may occur and, if severe, suggests possible DHF.
1082. A lady came for OB GYN assessment unit with hx of 8wk pregnancy and bleeding
per vagina for last 2 days. On bimanual exam, uterus =8wks in size. On speculum exam,
cervical os is closed. How do you confirm the viability of the fetus?
a. Transvaginal US
b. Serum BHCG
c. Urinary BHCG
d. Abdominal US
e. Per speculum exam
Transabdominal ultrasound will provide a panoramic view of the abdomen and pelvis
and is noninvasive, whereas transvaginal ultrasound provides a more limited pelvic view
and requires insertion of a probe into the vagina.
Transabdominal ultrasound cannot reliably diagnose pregnancies that are less than 6
weeks gestation. Transvaginal ultrasound, by contrast, can detect pregnancies earlier, at

approximately 4 to 5 weeks gestation. Prompt diagnosis made possible by


transvaginal ultrasound can, therefore, result in earlier treatment.
Scenario is that of threatened miscarriage.
Serum and urine b hcg maybe raised a few days until after death of fetus, per speculum
exam is not done for miscarriage.
1083. A 24yo lady has been low after the death of her husband and had stopped
contacting her family. She was started on SSRI tx and starts feeling better after a few
months. On discontinuing the meds she starts feeling that she has developed cancer just
like her husband. What is the most appropriate next step?
a. Continue SSRI
b. Add TCA
c. Neuropsychiatric analysis
d. CBT
e. Antipsychotics
Delusion of hypochondriasis is the diagnosis. Lady requires a neuropsychiatric analysis.
All the other options are wrong.
Hypochondriasis
This is a disorder where people fear that minor symptoms may be due to a serious
disease. For example, that a minor headache may be caused by a brain tumour, or a
mild rash is the start of skin cancer. Even normal bodily sensations such as 'tummy
rumbling' may be thought of as a symptom of serious illness. People with this disorder
have many such fears and spend a lot of time thinking about their symptoms.
This disorder is similar to somatisation disorder. The difference is that people with
hypochondriasis may accept the symptoms are minor but believe or fear they are
caused by some serious disease. Reassurance by a doctor does not usually help, as
people with hypochondriasis fear that the doctor has just not found the serious disease.
1084. A 24yo male who is sexually active with other males with hx of discharge per
urethra. Dx of chlamydia has been made. What is the possible complication if left
untreated?
a. Orchitis
b. Balanitis
c. Epididymo-orchitis
d. Acute abdomen
Chlamydia genital tract infection in men can lead to epididymo-orchitis and infertility if left
untreated.
Other causes are e.coli, mumps, gonorrhea and tb
CHLAMYDIA is common in males 35 years and above. Doxycycline 100 mg bd for 10
days is treatment.
1085. A person doesnt go outside the home because he thinks that people will look at
him and talk about him. He finds it difficult to associate with his peers in a restaurant or
under social settings. What is the most likely dx?
a. Agoraphobia
b. GAD
c. Panic disorder
d. Adjustment disorder
e. Social phobia
Agoraphobia is fear of open spaces.
Panic disorder or panic attack consists of episodes in which patient feels as if they are
going to die. Last for 10 mins.
Phobic disorders: anxiety in special situations.

Social phobia: phobia of situations in which a person may be closely minutely observed.
Such as in this scenario.
* Social anxiety is a fear of being around people and having to interact with them.
Sufferers fear being watched and criticised. Normal activities such as working, shopping,
or speaking on the telephone are marked by persistent feelings of anxiety and selfconsciousness. They feel dread as a situation approaches and afterwards they analyse
or ruminate on how they could have done better. Hence, it may be seen as a
fundamentally normal response but exaggerated to the point of being pathological.
* Physical symptoms include trembling, blushing, sweating and palpitations.
* They often experience chronic insecurity about their relationships with others,
excessive sensitivity to criticism, and profound fears of being judged negatively, mocked,
or rejected by others.
* There are two forms of the condition:
* Generalised social anxiety which affects most, if not all areas of life. This is the more
common type and affects around 70% of sufferers.
* Performance social anxiety, where these feelings only occur in a few specific situations
such as public speaking, eating in public or dealing with figures of authority.
* Treatment is CBT
Generalized anxiety disorder:
* Excessive anxiety and worry (apprehensive expectation), occurring more days than not
for at least six months, about a wide range of events or activities (such as work or school
performance).
* The person finds it difficult to control the worry.
Adjustment disorder is a group of symptoms, such as stress, feeling sad or hopeless,
and physical symptoms that can occur after you go through a stressful life event. The
symptoms occur because you are having a hard time coping. Your reaction is stronger
than expected for the type of event that occurred.
1086. A 63yo man presented with sudden onset of severe dyspnea, orthopnea, raised
JVP and bilateral basal crackles 3d after an episode of MI. A dx of acute congestive
cardiac failure was made and IV furosemide was started for this pt. What electrolyte
abnormality is expected?
a. High Na+, Low K+
b. Low Na+, High K+
c. Low Na+, Low K+
d. High Na+, High K+
e. Low Na+, Normal K+
Furosemide is a loop diuretic which inhibits transport of na/k/2cl in the thick ascending
limb of loop of henle. Diuresis usually occurs 4 hr after a dose. Effects is a massive nacl
excretion. Calcium secrestion also increased. Used in treatment of edema in heart
failure, ascites and pulmonary edema. Also used in severe hypercalcemia. Hypokalemic
metabolic alkalosis, hypovolemia, ototoxicity and allergic reactions are side effects.
1087. A 70yo hypertensive white british man on thiazide diuretics needs a 2nd drug to
control his BP. Which one of the following is the best choice for him?
a. Amlodipine (CCB)
b. Enapril (ACEi)
c. Propranolol (BB)
d. Increase dose of diuretic
e. Prazocin (Alpha blocker)
Key is b.
Hypertension management

If age is greater than 55, or black patient of any age: ca channel blocker or thiazide
(C/D)
If age is less than 55: ace inhibitor(A)
B blocker to be considerd in young, pregnancy or if increased sympathetic drive
Combination therapy
Ace inhibitor plus ca channel blocker or thiazide(A+C/D)
3 drugs needed: ace inhibitor plus ca channel blocker plus diuretic (A+C+D)
4th drug needed. Add higher dose diuretic or b blocker or alpha blocker(A+C+D+B)
1088. A 74yo lady who has had a stroke in the past has an indwelling catheter for 10m.
She presents with bluish-purple discoloration of the catheter bag. What is the most likely
explanation for this?
a. Normal change
b. Catheter degradation
c. Acidic urine
d. Alkaline urine
e. Bacterial colonization of the urinary tract
Explained in Samson notes. Renal section.
1089. A 62yo man has slow palpitations and the following ECG. What is the most likely
dx?
a. Sinus bradycardia
b. 1st degree heart block
c. Mobitz type 1 block
d. Mobitz type 2 block
e. Complete heart block
Progressive prolongation of PR interval followed by missed beat is indicative of MOBITZ
type 1 block.
Bradycardia is to be treated with atropine. If not controlled, then temporary pacing may
be required.
1090. A 29yo woman presents with lid lag, lid retraction and diplopia. What is the most
appropriate next step?
a. TFT
b. Tensilon test
c. Fundoscopy
d. Autoantibodies
e. EMG
Scenario suggestive of Graves disease, hyperthyroidism. Graves' disease:[9]
* This is the most common cause of hyperthyroidism and has an autoimmune basis. It is
mediated by B and T lymphocytes, characterised also by the presence of thyroidstimulating immunoglobulins (TSIs). These are directed at four different thyroid antigens:
* Thyroglobulin.
* Thyroid peroxidase (or antimicrosomal antibodies).
* Sodium-iodide symporter.
* TSH receptor.
* The condition is characterised by a small to moderate diffuse, firm goitre with 50% of
these showing ophthalmopathy.
* There may be a personal or family history of autoimmune disease.
* <5% have pretibial myxoedema called thyroid dermopathy (as can occur anywhere,
particularly following trauma). This is usually associated with moderate to severe
ophthalmopathy. 10-20% have clubbing (thyroid acropathy). Thyroid dermopathy usually

appears as non-pitting plaques with pink/purple colour. There are also nodular and
generalised forms.
* There may also be lymphoid hyperplasia including splenomegaly and an enlarged
thymus. * Associated with other autoimmune conditions - eg, pernicious anaemia, type
1diabetes mellitus.
Investigations * Thyroid function tests (TFTs): serum TSH can exclude primary
thyrotoxicosis. Confirm the diagnosis with free T4 levels. If TSH is suppressed but free
T4 levels are normal, then if not previously supplied, free T3 level is needed (T3
toxicosis occurs in 5% of patients).
* Autoantibodies - these are most commonly seen in Graves' disease:
* Antimicrosomal antibodies - against thyroid peroxidase.
* Antithyroglobulin antibodies.
* TSH-receptor antibodies which are commonly present in Graves' disease but are not
routinely measured.
* TSI if elevated helps to establish a diagnosis of Graves' disease.
1091. A 41yo man presents with longstanding foul smelling ear discharge and
progressive hearing loss. Otoscopy showed perforation of the pars flaccida and a mass
in the upper part of the middle ear. What is the most likely dx?
a. ASOM
b. CSOM
c. Acquired cholesteatoma
d. Congenital cholesteatoma
e. Barotrauma
Long standing foul smelling discharge and progressive hearing loss indicates CSOM, but
otoscopic findings of perforation and mass point towards acquired choleastoma which
can also be a complication of asom.
Secondary acquired cholesteatoma This arises as a result of insult to the tympanic
membrane, such as perforation secondary to acute otitis media or trauma, or due to
surgical manipulation of the drum. Squamous epithelium may be inadvertently implanted
by the insult so triggering the process of cellular growth resulting in cholesteatoma
formation.
* Frequent or unremitting painless otorrhoea which may be foul-smelling.
* Recurrent otitis, poorly responsive to antibiotic treatment.
* Progressive, unilateral conductive hearing loss.
* Tympanic membrane perforation (~90% of cases) or retracted tympanum.
* The only finding may be a pus-filled canal with granulation tissue
CT imaging is the investigation of choice if there is a need for assessment of the extent
of the lesion and to assess subtle bony defects.
Treatment
Surgery:
Open technique: tympanomastoidectomy
Closed technique : tympanoplasty
1092. A 9yo child presented with a rash on his skin which didnt respond to antibacterial
ointment. What med should be added next?
a. Corticosteroid
b. Antifungal
c. Emollient
d. Permethrin
e. Coal tar
1093. A young boy has a hx of epistaxis. CBC=normal, except APTT=47s. What is the
most likely dx?

a. Hemophilia
b. ITP
c. Sickle cell
d. HUS
e. Thalassaemia
Epistaxis and prolonged APTT point toward intrinsic pathway defect , so hemophilia is
the likely key.
Hemophilia A IS A FACTOR 8 DEFiciency inherited in a x linked recessive pattern.
Presentation early in life or after trauma/surgery. Bleeding into joints leading to crippling
arthropathies and muscle causing hematomas with nerve compression and
compartment syndrome. Diagnosed by inc APTT and dec factor 8 assay. Treatment with
desmopressin which inc factor 8 levels and recombinant factor 8 in major or life
threatening bleeds.
Hemophilia B is a similar condition arising due to deficiency of factor 9. Also inherited in
x linked recessive pattern. Behaves clinically like hemophilia A.
1094. A 29yo young man presents with complaints of recurrent attacks of diarrhea. He
says his stools contain blood and mucus. Sometimes he has low grade fever. What is
the most appropriate inv for his condition?
a. Stool culture
b. Plain abdominal XR
c. Per rectal exam
d. Barium enema
SCENARIO suggestive of Ulcerative Colitis:
Per rectal exam will not provide us with any evidence. Plain abdominal x ray will be
useful in an acute exacerbation of disease. Barium enema is the key but is not an
investigation of choice but can be used in acute setting to diagnose.. Here stool culture
seems appropriate also so we can rule out an infectious cause to the condition.
The diagnosis should be made on the basis of clinical suspicion supported by
appropriate macroscopic findings on sigmoidoscopy or colonoscopy, typical histological
findings on biopsy and negative stool examinations for infectious agents.
* Initial investigations include:
* FBC, renal function and electrolytes, LFTs, ESR and CRP.
* Low magnesium and serum albumin levels are sometimes found in ulcerative colitis. *
Stool culture, including ova, cysts and parasites and also Clostridium difficile toxin.
* Serological markers have been developed to differentiate ulcerative colitis from Crohn's
disease. p-ANCA is more commonly associated with ulcerative colitis, whilst ASCA is
more commonly associated with Crohn's disease.
* Abdominal imaging: this is essential in the initial assessment of patients with suspected
ulcerative colitis, to exclude toxic dilatation and perforation. It may also help to assess
disease extent or identify proximal constipation. In milder forms, ultrasound, CT, MRI
and radionuclide scanning may all be contributory.
* Sigmoidoscopy and rectal biopsy: for all patients presenting with diarrhoea, rigid
sigmoidoscopy should be performed unless there are immediate plans to perform
flexible sigmoidoscopy.
* Colonoscopy:
* This is usually preferable to flexible sigmoidoscopy, because the extent of disease can
be assessed but, in moderate-to-severe disease there is a higher risk of bowel
perforation and flexible sigmoidoscopy is safer

1095. A 26yo young man presents with chx of passing loose stools for the past 2m. He
says his stools contain blood and mucus and are a/w abdominal pain. He undergoes a
colonscopy after which he was started on tx. What is the most appropriate tx for his
condition?
a. Mesalazine
b. Corticosteroids
c. Infliximab
d. Cyclosporine
The scenario is that of Ulcerative colitis.
MANAGEMENT
Aminosalicylates: Mesalazine - 5-aminosalicylic acid (5-ASA) - is now the treatment of
choice for induction and maintenance of remission of mild-to-moderate ulcerative colitis.
Corticosteroids are used to induce remission in relapses of ulcerative colitis. They have
no role in maintenance therapy.
Thiopurines: Azathioprine and its active metabolite 6-mercaptopurine may be used
when:
* Patients are intolerant to corticosteroids.
* Patients need two or more corticosteroid courses in a calendar year.
* Disease relapses when the dose of prednisolone is less than 15 mg a day.
* Disease relapses within six weeks of stopping steroids.
Ciclosporin: This is an effective salvage therapy for patients with severe refractory colitis
and it has a rapid onset of action.
Infliximab is recommended as an option for the treatment of acute exacerbations of
severely active ulcerative colitis only in patients in whom ciclosporin is contra-indicated
or clinically inappropriate Up to 30% of patients will ultimately require colectomy for
ulcerative colitis.[3]
* Colectomy is an option for patients who do not respond to, or are intolerant of, medical
treatment, or in those with complications such as colorectal neoplasia.[15] * Because
ulcerative colitis is confined to the colorectum, colectomy is curative.[15]
* The usual procedure is a restorative proctocolectomy with ileal pouch-anal
anastomosis.[15]
1096. A 52 yo male with poorly controlled DM has now presented to his GP with pain in
the ear. Exam: skin around the ear is black in color and there was foul smelling
discharge from the ear. Pt also had conductive hearing loss. What is the most probable
dx?
a. Carbuncle
b. Folliculitis
c. Malignant OE
d. Cholesteatoma
e. Furuncle
Poorly controlled diabetes in this man points towards the diagnosis of Malignant Otitis
Externa. It occurs in the age group 45 to 75. It is aggressive life threatening infection of
the external year that can lead to temporal and mastoid destruction and base of skull
osteomyelitis. Also facial
nerve palsy. Diabetes and immunosuppression are risk factors. Usual cause is
pseudomonas, klebsiella and proteus may be involved as well. There maybe hearing
loss in otitis externa. Treatment is by surgical debridement, systemic and topical
antibiotics.
1097. A 55yo male has been admitted for elective herniorrhaphy. Which among the
following can be the reason to delay his surgery?
a. Controlled asthma

b. Controlled A-fib
c. DVT 2yrs ago
d. DBP 90 mmHg
e. MI 2m ago
Controlled asthma and controlled afib and dvt 2 years back are not contraindications. MI
2 months ago seems appropriate but new guidelines suggest even a period of two
months after MI is enough. Otherwise atleast a period of 3 months is needed.
1098. A 21yo female in her first pregnancy at 38wks was brought to the ED with
generalized tonic clonic seizure. IV MgSO4 was given but fits was not controlled. She is
having fits again. What is the single most imp immediate management of this pt?
a. IV MgSO4
b. IV diazepam
c. Immediate C-section
d. IV phenytoin
e. MgSO4 bolus
f. IV lorezepam
Treatment of Eclampsia
Treat first seizure with 4g MGSO4 in 100 ml NS IVI over 5 min and mantainenece IVI of
1g per hr for 24 hr.
If recurrent, give 2g ivi magnesium sulphate over 5 mins.
Use diazepam once if fits continue.
According to this regimen, the correct answer
1099. A 24yo lady with BMI=30 complains of facial hair growth and hx of amenorrhea.
FSH=10.9, prolactin=400IU, estradiol=177.8mmol/l, progesterone=normal, LH=33.2.
What is the most probable dx?
a. PCOS
b. Pregnancy
c. Cushings disease
d. CAH
e. POF
Young lady, obese, with facial hair and history of amenorrhea. Looks like PCOS as
cause seems to be ovarian indicated by a raised LH and FSH, and normal prolactin.
It encompasses a syndrome of polycystic ovaries, in association with systemic
symptoms causing reproductive, metabolic and psychological disturbances. These most
commonly present with infertility, amenorrhoea, acne or hirsutism.
The patient often presents in the peripubertal period through to her mid 20's.
Symptoms:
Oligomenorrhoea (defined as <9 periods per year)
Infertility or subfertility, Acne, Hirsutism, Alopecia, Obesity or difficulty losing weight,
Psychological symptoms[5] - mood swings, depression, anxiety, poor self-esteem Sleep
apnoea
Differential diagnosis * Thyroid disorder (particularly
hypothyroidism),Hyperprolactinaemia,Cushing's syndrome,Late-onset congenital adrenal
hyperplasia (very rare),Androgen-secreting ovarian or adrenal tumours,Ovarian
hyperthecosis (signs of virilism and biochemical androgen excess are much more
prominent in the latter three)
Investigations
* This may show LH elevated, LH:FSH ratio increased (>2), with FSH normalFree
testosterone levels may be raised, but if total testosterone is >5 nmol/L, exclude
androgen-secreting tumours and congenital adrenal hyperplasia.

* Laparoscopy or ultrasound shows characteristic ovaries (the average volume is three


times that of normal ovaries .Other blood tests, where indicated from the clinical picture,
to exclude other potential causes - eg, TFT (thyroid dysfunction), 17hydroxyprogesterone levels (congenital adrenal hyperplasia ), prolactin
(hyperprolactinaemia), DHEA-S and free androgen index (androgen-secreting tumours),
and 24-hour urinary cortisol (Cushing's syndrome).
* Fasting glucose
* Fasting lipid levels should be checked.
Diagnosis Two of the three following criteria are diagnostic of the condition (Rotterdam
criteria):[6]
* Polycystic ovaries (either 12 or more peripheral follicles or increased ovarian volume
(greater than 10 cm3)
* Oligo-ovulation or anovulation
* Clinical and/or biochemical signs of hyperandrogenism
MANAGEMENT
They should be advised on weight control and exercise.] It has been shown to improve
ovulation, pregnancy rate
Pharmacological treatment
There is no treatment which reverses the hormonal disturbances of PCOS and treats all
clinical features, so medical management is targeted at individual symptoms, and only in
association with lifestyle changes.
For women not planning pregnancy
* Co-cyprindrol: is licensed for treating hirsutism and acne, although not specifically in
PCOS. Combined oral contraceptive pill (COCP): is also used to control menstrual
irregularity.
* Metformin: has been increasingly used off-licence for PCOS; however, a Cochrane
review showed it to be less effective than the COCP for menstrual irregularity, hirsutism
and acne,[13] and the National Institute for Health and Care Excellence (NICE) Evidence
Summary suggests its side-effects and cost outweigh its benefits and any, as yet
unproven, long-term health benefits.[14]
* Eflornithine: may be used for hirsutism, as can cosmetic treatments (electrolysis, laser,
waxing, bleaching).
* Orlistat: can help with weight loss in women with PCOS and may improve insulin
sensitivity[15]
For women wishing to conceive and presenting with infertility
2013 NICE guidelines advise (after weight loss where indicated, and a full fertility workup) women should be treated with clomifene, metformin or a combination of the two.[10]
(A Cochrane review in 2012, however, found no benefit in live birth rates from the use of
metformin or other insulin-sensitising drugs, although it did improve numbers of
pregnancies.[16])
Complications
* Oligomenorrhoea or amenorrhoea are known to predispose to endometrial hyperplasia
and endometrial cancer in untreated cases. It has been suggested that women with
PCOS may have a higher cardiovascular risk than weight-matched controls.
* Women presenting with PCOS, particularly if they are obese (BMI greater than 30),
have a strong family history of type 2 diabetes or are over the age of 40 years, are
atincreased risk of type 2 diabetes and should be offered screening. The risk may be as
high as 10-20%.
* Women diagnosed with PCOS (or their partners) should be asked about snoring and
daytime fatigue/somnolence and informed of the possible risk of sleep apnoea. They
should be offered investigation and treatment when necessary.[20]

* Complications in pregnancy: there is a higher risk of gestational diabetes in women


with PCOS, which may be more than double..
1100. A 17yo girl with a lump in her breast was seen in the clinic. Exam: the lump was
free and mobile and not attached to the skin. Her mother wants further tests done. What
should be the next step?
a. CT
b. US breast
c. Punch biopsy
d. Reassure and send home
e. Stereotactic biopsy
Diagnosis is that of a Fibroadenoma breast. It is a benign tumour common in young
women. Diagnosis is made and confirmed on ultrasound Fibroadenoma This is a benign
tumour that is common in young women, mostly aged under 40 years. It is composed of
stromal and epithelial elements and probably represents increased sensitivity to
oestrogens:
* Complex and multiple fibroadenomas are associated with a two-fold increase in the risk
of breast cancer.
* They represent a hyperplasia or proliferation of a single terminal duct unit.
* Most stop growing at about 2 or 3 cm, but they can enlarge rather further.
* About 10% disappear each year.
* They tend to regress after the menopause.
* They occur in about half of women who receive ciclosporin after renal transplant.
* They are the most common tumour of the breast in those under 30 years old, but
overall they are second to breast cancer.
* Juvenile fibroadenomas can occur in teenage girls.
Both mammography and ultrasound may be used to examine the lump:
* Ultrasound tends to be preferred in younger women with dense breasts, as
mammograms are more difficult to interpret in this group. Routine mammography, as a
population screening tool, is not performed below the age of 50 years.
* Imaging studies may fail to give a firm diagnosis and biopsy or excision may be
required for peace of mind of both the patient and doctor.
* If there is confidence in the diagnosis then inactivity may lead to spontaneous
regression, but the patient must be advised to check the lump regularly and to return if it
starts to enlarge.
* Assessment often includes examination, imaging studies and fine-needle aspiration.

1100. A 17yo girl with a lump in her breast was seen in the clinic. Exam: the lump was free and
mobile
and not attached to the skin. Her mother wants further tests done. What should be the next
step?
a. CT
b. US breast

c. Punch biopsy
d. Reassure and send home
e. Stereotactic biopsy
key: B
cause: young girl , lump mobile and free >>> fibroadenoma
Triple assesment for breast lump

clinical examination

imaging :mamography if older than 35 years or u/s for younger patients

FNAC

THIS is a case of fibroadenoma and examination was done so the next step is imaging (pt is 17 so
u/s)
Fibroadenoma
This is a benign tumour that is common in young women, mostly aged under 40 years.
They are the most common tumour of the breast in those under 30 years old, but overall they
are second to breast cancer.
Assessment often includes examination, imaging studies and fine-needle aspiration.
Ultrasound >>> in younger women with dense breasts, as mammograms are more difficult to
interpret in this group.
Routine mammography, as a population screening tool, is not performed below the age of 50
years.
- The initial examination depends upon the age of the patient. Ultrasound for <35 years old;
Mammography and then Ultrasound for >35 years old.
- Ultrasound is best for dense breast tissue, whereas Mammography is best for less dense breast
tissue; eg, after menopause.
- CT is not done for breast lesions.
- Stereotactic Biopsy is the investigation of choice only when there are no palpable masses.
- You cannot reassure and send home as it might be a Fibroadenoma since the patient is young.
1101. A lady comes with a missing IUCD thread. Her LMP was 2wks ago. What is the single most
appropriate next step in management?
a. Abdominal US

b. Prescribe contraceptives
c. CT
d. Serum BHCG
e. Vaginal exam
key: A
cause: in case of lost thread we advise the pt with extra contraception like condom then we
start managing the case by : pregnancy test but in this case her LMP was two weeks ago so no
need for than , so the next step is to locate the IUCD using imaging studies ( us first then xray)
lost thread : OCS pg 298
the IUCD may have been expelled, so advise extra contaception and exclude pregnancy
seek coil on u/s ; if missing arrange x-ray to exclude extra-uterine coils ( surgical retrieval is
advised)

- In case of a lost thread, a number of measures should be taken. They are:


- If threads are not visible, or if they are but the stem of the device is palpable, the woman
should be advised to use condoms or abstain from intercourse until the site of the device (if
present) can be determined.
- Perform a speculum examination to ensure the device is not in the posterior fornix.
- Determine whether the woman is already pregnant.
- With consent, explore the lower part of the endocervical canal with narrow artery forceps:
threads which have been drawn a little way up are usually found by this method.
- An experienced operator may, after appropriate analgesia (eg, mefenamic acid 500 mg)
explore the uterine cavity with a retriever hook.
- Hormonal emergency contraception may be indicated.
- Ultrasound should be arranged to locate the device.
- If ultrasound does not locate the device and there is no definite history of expulsion then
abdominal X-ray should be performed to look for an extrauterine device.
- Expulsion should not otherwise be assumed.
- Hysteroscopy can be helpful if ultrasound is equivocal.
- Surgical retrieval of an extrauterine device is advised.

- The question asks for the most appropriate next step in the management of this case. That
step would be to exclude pregnancy via an abdominal ultrasound. IUCDs increase the risk of
ectopic pregnancy, for which again, the next step would be an abdominal ultrasound.

1102. A 32yo woman presents with hx of lower abdominal pain and vaginal discharge. She had
her
menses 4wk ago. She has a temp of 38.6C. What is the most suitable dx?
a. Acute appendicitis >>> (nausia,vomiting,RIF pain)
b. Acute PID
c. Endometriosis>>> dysparaunia
d. Ectopic pregnancy>>> missed period,abnormal vaginal bleeding
e. UTI>>>urinary symptoms
key: B
cause: clinical signs and symptoms fit ( fever above 38 + bilatral lower abdominal pain + vaginal
discharge)
PID
Diagnosis of acute PID made only on clinical signs
signs & symptoms

Bilateral lower abdominal pain.

Deep dyspareunia.

Abnormal vaginal bleeding (postcoital, intermenstrual or menorrhagia).

Vaginal or cervical discharge that is purulent.

Fever above 38C

Investigations:

Pregnancy test (pregnant women with PID should be admitted; ectopic pregnancy
may be confused with PID).

Cervical swabs for chlamydia and gonorrhoea

Endocervical swabs for C. trachomatis and N. gonorrhoeae

Treatment:

Mild or moderate disease can be managed in primary care or outpatients, whereas


clinically severe disease requires hospital admission for intravenous (IV) antibiotics.

The current outpatient treatment recommendation is ceftriaxone 500 mg as a


single (IM) dose, followed by doxycycline 100 mg orally twice daily and
metronidazole 400 mg twice daily for 14 days

Initial treatment is with doxycycline, single-dose IV ceftriaxone and IV


metronidazole, then change to oral doxycycline and metronidazole to complete 14
days of treatment.

- Symptoms of PID
- Bilateral Lower/Pelvic abdominal pain that ranges from mild to severe.
- Pain during sex
- Abnormal vaginal discharge
- Abnormal vaginal bleeding (1 in 4 cases)
- Risk Factors
- A recent change of sexual partner. The risk goes up with the number of partners.
- A previous episode of PID or sexually transmitted disease.
- A recent abortion.
- A recent operation or procedure on the womb (uterus).
- A contraceptive coil inserted recently.
- Tests to be done
- Endocervical Swab is the investigation of choice. High vaginal swab can also be taken.
- If the above doesnt show anything, an U/S can be done to look for inflamed fallopian tubes.
- Complications
- Infertility
- Ectopic Pregnancy
- Miscarriage and Still Births
- Persistent pain (including pain during sex)
- Reiters Syndrome
- Abscess formation
- Treatment

- Antibiotics
- The partner must also be treated
1103. A 40yo female was on COCP which she stopped 6m ago. But she has not had her periods
since
then. Labs: FSH=22, LH=24, prolactin=700, estradiol=80. What is the most appropriate dx?
a. Hypothalamic amenorrhea
b. Post pill amenorrhea
c. Prolactinoma
d. Pregnancy
e. Premature ovarian failure
key: E
reason: this is a case of secondary amenorrhea with elevated gonadotropins indicating ovaian
failure and the patient is 40 years so it's premature ovarian failure
causes of secondary amenorrhea:

with no androgen excess:

1-pregnancy(the most common cause of secondary amenorrhea),lactation,menopause.


2-premature ovarian failure:
this is a poorly understood condition that may represent an autoimmune phenomenon. It can
also follow radiotherapy or chemotherapy. With all these causes, menstruation and fertility can
sometimes resume spontaneously. Ovarian failure will cause elevation of gonadotrophins and so
hot flushes are likely. Premature menopause is defined as occurring before the age of 40.
3-depot and implant contraception
4-cevical stenosis and intrauterine adhessins >>> Asherman's syndrome
5-wt loss >>> especially if rapid
6-pitutary disease and hyperprolactinaemia
7-sheehan syndrome >>> the pituitary may be damaged by tumours, trauma, cranial irradiation,
sarcoidosis or tuberculosis
8- post pill amenorrhea
with androgen excess :

PCOS
Premature Ovarian Failure Secondary Amenorrhea (E)
- Causes
- Hyperprolactinemia
- PCOS
- Premature Ovarian Failure
- Post-pill Amenorrhea
- Ashermans Syndrome
- Investigations
- Pregnancy test (if appropriate)
- Follicle-stimulating hormone (FSH) and luteinising hormone (LH)
- PRL
- Total testosterone and sex hormone-binding globulin
- TSH
- A pelvic ultrasound may be useful in patients with suspected PCOS
- Treatment
Treatment is related to cause. Premature Ovarian Failure is irreversible but hormone
replacement is necessary for controlling the symptoms of Estrogen deficiency and protection
against Osteoporosis. Pregnancy can be achieved by oocyte donation or in vitro fertilization
techniques.
- What to look for in the question
- Secondary amenorrhea before or at the age of 40 (The age is 40 in this question)
- A raised Gonadotropin level

1104. A 25yo woman presents with a single lump in the breast and axilla. The lump is mobile
and hard
in consistency. The US, mammogram and FNA turn out to be normal. What is the most
appropriate inv to confirm the dx?
a. FNAC

b. MRI
c. Punch biopsy
d. Genetic testing and counselling
e. Core biopsy
key: E
reason : in palpable mass we do triple assessment : clinical examination,imaging,FNAC >>>if not
conclusive we do core biopsy then excional biopsy if still non conclusive
Breast lump triple assessment

examination

imaging: mamogaraphy,u/s

biopsy

* non palpable lesion:


1-core biopsy(image -guided)
2- open biopsy(needle localisation >>> radio opaque needles are used to guide the boipsy0
*palpable lesion:
1-FNAC
2-core biopsy
3-excision biopsy (entire lesion is removed)
4-incision biopsy (part of the lesion)
All patients with breast lumps must undergo triple assessments.
1st Assessment: Clinical examination of the breast including axillary lymph nodes
2nd Assessment: Imaging
- U/S for <35 years old
Mammography and THEN U/S for >35 years old patient.
3rd Assessment: Cytology
- If it is a cyst, perform FNAC
-Clear Fluid? Reassure the patient
- Blood-stained aspirate? Send to lab for Cytology

- Clear fluid but residual mass? Perform core biopsy


In this particular question:
The patient is a young woman with a single lump in the breast and the axilla. The lump is hard
and mobile which points towards Fibroadenoma but a swollen lymph node in the axilla points
towards a carcinoma. To find out which one it is, we need a core biopsy since an ultrasound, a
Mammogram and an FNAC turned out to be normal. The usual typical order of investigations in
such cases is:
- Ultrasound/ Mammogram depending upon the age.
- Mammogram
- FNAC
- Core Biopsy

1105. A 37yo lady stopped taking COCP 18m ago and she had amenorrhea for 12m duration.
Labs:
FSH=8, LH=7, prolactin=400, estradiol=500. What is the cause?
a. Hypothalamic amenorrhea
b. PCOS
c. Prolactinoma
d. Post pill amenorrhea
e. POF
key:D
reason :this is a case of secondary amenorrhea following COCP use,,,
decreased gonadotropins >>> so it's not POF,,
and there are no symptoms of increased androgens>>> so it's not PCOS ,,
NO increased prolactin >>>so not prolactinoma,,,
the history of amenorrhea after COCP use + decreased gonadotropins fit with >>> post pill
amenorrhea
Post-pill amenorrhoea
this is when stopping oral contraceptives does not lead to a resumption of a normal menstrual
cycle. It usually settles spontaneously in around three months but, if not, it requires

investigation. The condition is probably not a true entity but the cause of amenorrhoea started
whilst taking the contraceptives that induced an artificial cycle until they were stopped.
Post pill amenorrhea is described as the loss of menstrual periods for at least 6 months after
stopping birth control pills. The incidence of post-pill amenorrhea ranges from 0.2% to 3%.
- Cause of Post-pill Amenorrhea
Post-pill amenorrhea is believed to be due to suppression of the pituitary gland by the birth
control pills.
- Investigations
- The diagnosis of post-pill amenorrhea is usually made when there is loss of periods after a
prolonged history of taking birth control pills. - Ultrasonography will reveal ovaries with no signs
of developing follicles and ovulation even after having stopped the pills for 6 months.
- Blood tests showing a low level of FSH, LH and estrogen is usually sufficient to confirm the
diagnosis.
- Treatment
- The first line of treatment in case of post-pill amenorrhea is waiting for a spontaneous
remission of the amenorrhea and a spontaneous occurrence of periods.
- The time limit is usually six months. But if the woman is anxious to get her periods, active
treatment may be started after waiting for only three months.
- The standard treatment of post-pill amenorrhea is by stimulating the pituitary to produce FSH
and LH. This is done by the drug clomiphene citrate.
1106. A lady with a firm smooth breast lump in outer quadrant had a FNAC done. Results
showed
borderline benign changes. She also has a fam hx of breast cancer. What is the your next?
a. Mammography
b. US
c. Core biopsy
d. Genetic testing and counselling
e. Punch biopsy
key : D
reason: assessment of the case is complete (borderline benign change),,,next step is genetic
testing and counselling.
breast lump >>>discussed earlier

In this question a lady underwent an FNAC and the results have showed benign breast changes.
She also has a risk for developing breast cancer because of her family history. All options but
one are investigations, which we dont need at this point because we got all we could get from
the investigations. What needs to be done now is to assess the risk of breast cancer in this
patient. Also, the patient needs to be counselled about the disease. Therefore, we should for
genetic testing and counselling.
1107. A pt presents with mild dyskaryosis. 1y ago smear was normal. What is the most
appropriate
next step?
a. Cauterization
b. Repeat smear
c. Swab and culture
d. Cone biopsy
e. Colposcopy
key: E
Cervical screening
Cervical cancer is the third or fourth most common female malignancy worldwide
The screening process is done using LBC (liquid based cytology) or older method (PAP)
Interpreting smear results:
Cells are analysed to look for abnormalities in the appearance of the nucleus and other aspects
of cell morphology (dyskaryosis)

Negative (normal) >>> treat incidental findings eg,infection & recall as appropriate

inadequate : insufficient or unsuitable material sampled >>>>Repeat sample


immediately after treating any infection,,Repeat sample as soon as convenient if
technically inadequate,,,if persistent (three inadequate samples), advise assessment
by colposcopy

Borderline changes and mild dyskaryosis>>>> HPV testing ,,if positive >>
colopscopy,,

if negative >>> normal recall

if unreliable >>> repeat HPV in six month and if +ve >> colopscopy

moderate and severe dyskaryosis >>>> refer to colopscopy

Cervical Cancer When to refer for Colposcopy (E)

- Any smear showing mild, moderate or severe Dyskaryosis


- Any suggestion of malignancy
- 3 consecutive inflammatory smears
- 2 consecutive atypical smears
- 3 consecutive borderline smears
- 3 consecutive inadequate smears
- Post-coital bleeding
1108. An African lady presents with heavy but regular periods. Her uterine size correlates to
14wks
pregnancy. What is the most appropriate dx?
a. Blood dyscrasia
b. Hematoma
c. Fibroids
d. Adenomyosis
e. Incomplete abortion
key: C
reason: fibroids are three times more common in African americans than white american+
enlarged uterus + heavy menses
Fibroids

-age :commonly 30-50 ys

- more common in : obese, early menarche , African Americans ,

-protective factors: exercise, increased parity , may be smoking

types>> submucosal,,intramural,,subserosal

*presentation:

.. excessive or prolonged bleeding,,pelvic pain

..enlarged uterus>>pressure symptoms>>heaviness,constipation,urinary symptoms

..submucosal>>infertility,miscarriage,intermenstrual bleeding

..during pregnancy>>red degeneration

Investigations:

pelvic u/s

TVUS >> more accurate

saline infusion u/s is superior to TVUS and hysteroscopy in detection of submucosal


fibroids

MRI>>if myomectomy is considered and u/s is not conclusive

Endometrial sampling>>>in abnormal uterine bleeding

combination LDH, LDH isoenzyme 3 and gadolinium-enhanced MRI is highly


accurate in diagnosing leiomyosarcoma pre-operatively, if sarcoma is suspected
clinically

Treatment :

only required if symptomatic

Medical:

NSAIDs (ibuprofen)

Antifibrinolytic (tranexamic acid)

Combined hormonal contraception ( CHC)

Merina,,danazol,,GnRH agonists,,Mifepristone,,ulipristal acetate,,aromatase


inhibitor>>letrozole

Surgical:

indications>>failure of medical treatment,,infertility,,pressure


symptoms,,excessively enlarged uterine size

*myomectomy>> for those who want to maintain fertility

abdominal myomectomy : safe alternative to hysterectomy + risk of blood loss

laparoscopic myomectomy: less pain, shorter hospital stay and reduced recovery
time,subserous fibroid

hysteroscopic myomectomy : established surgical procedure for women with


submucosal fibroids and excessive uterine bleeding, infertility or repeated
miscarriages.

laparoscopic laser myomectomy >> not recommended by NICE

* TOTAL hysterectomy

laparotomy

laparoscopic assisted hysterectomy >> urinary tract injury+severe bleeding

* uterine artery embolization UAE

*MRI-guided transcutaneous focused ultrasound

- Notice that the question mentions an African female with regular but heavy periods. This is
because fibroids are mostly common in Afro-Caribbean females.
- Fibroids are responsive to estrogen and therefore increase in size, which in turn increases the
size of the uterus. E.g., in this patient, her uterine size correlates to 14 weeks pregnancy.
- Other symptoms
- Pelvic pain (Compression on to adjacent structures)
- Infertility/Recurrent Miscarriages
- Pelvic Mass
- The investigation of choice is an U/S
- Management
- Mirena Coil is the first choice if the fibroids are not big enough to restrict its insertion.
- If < 3 cm
- Trial of pharmacologic treatment first (Tranexamic Acid) first
- If it fails and uterus is not bigger than 10-week pregnancy, do endometrial ablation
- If the above fails, perform a hysterectomy
- If > 3 cm and wishes to retain uterus and/or wants to avoid surgery
- Go for Uterine Artery Embolization
- If > 3 cm and wishes to retain uterus, go for a hysteroscopic myomectomy or a myomectomy

1109. A 29yo at 38wks GA presents with a 2h hx of constant abdominal pain. She then passes
100 ml of
blood per vagina. What is the next appropriate inv?
a. USS
b. CTG
c. Clotting screen
d. Hgb

e. Kleihauer Betke test


key: A
reason : in case of antenatal bleeding we do u/s to investigate the cause of bleeding
Antenatal bleeding

never perform PV >> might increase bleeding

dangerous causes: Abruption,placenta praevia,vasa praevia

other caused:circumvallate placenta,placental sinuses cervical polyps,erosions and


carcinoma,cervicitis,vaginitis,vulvar varicosities

placental abruption>>>> part of placenta becomes detached ,, the outcome


depends on the amount of detachment and blood loss,,bleeding might be concealed

associations>>> preeclampsia,smoking,cocaine,IUGR,PROM,abdominal
trauma,multiple pregnancy,polyhydramnios,increased maternal age,non vertex
presentation,assisted reproductive techniques

placenta praevia>>>placenta is in the lower uterine symptoms,,bleeding is always


revealed

Management:

in severe bleeding>>> admission,,IV lines , O2 mask,, blood transfusion if in


shock,,catheterize,,if severe bleeding >> CS

in mild cases>>> IVI,,Hb , cross match,check vitals >> establish dx by u/s >>if
placenta praevia >>>cs

Bleeding During Third Trimester APH (A)


- Bleeding during the third trimester is either painful or painless bleeding.
- Painful bleeding points to Placental Abruption while painless bleeding points to Placenta Previa
- In this case, the patient is experiencing painful vaginal bleeding in the third trimester, so this
appears to be the case of Placental Abruption
- The investigation of choice in this case is an Ultrasound
- Risk Factors for Abruption
- Hypertension
- Smoking
- Multiple Pregnancy
- Cocaine/Amphetamine Use

- Increased Maternal Age


- Trauma to the abdomen
- Polyhydramnios
- Investigation
- Diagnosis is clinical but U/S is done to exclude Placenta Praevia and to check the well-being of
the baby.
- Management
- Always admit the patient to hospital for assessment and management. Phone 999/112/911 if
there are any major concerns regarding maternal or fetal well-being.
- The mainstays of management are resuscitation and accurate diagnosis of the underlying
cause.
- Severe bleeding or fetal distress: urgent delivery of the baby, irrespective of gestational age.
- Admit to hospital, even if bleeding is only a very small amount. There may be a large amount of
concealed bleeding with only a small amount of revealed vaginal bleeding.
- No vaginal examination should be attempted, at least until a placenta praevia is excluded by
ultrasound. It may initiate torrential bleeding from a placenta praevia.
- Resuscitation can be inadequate because of underestimation of blood loss and misleading
maternal response. A young woman may maintain a normal blood pressure until sudden and
catastrophic decompensation occurs.
- Take blood for FBC and clotting studies. Crossmatch, as heavy loss may require transfusion.
- Gentle palpation of the abdomen to determine the gestational age of the fetus, presentation
and position.
- Fetal monitoring.
- Arrange urgent ultrasound.
- With every episode of bleeding, a rhesus-negative woman should have a Kleihauer test and be
given prophylactic anti-D immunoglobulin
- Complications
- Premature Labour
- DIC
- Renal Tubular Necrosis
- PPH

- Placenta Accreta
- Points to look for diagnosis of Abruption
- Shock is out of proportion from visible blood loss
- Constant pain
- Tender tense uterus
- Fetal heart sounds absent/distressed
- Coagulation problems like DIC

1110. A 26yo woman had amenorrhea for 10wks and is pregnant. She experiences hyperemesis.
Now
she presents with vaginal bleed. Exam: uterus=16wks, closed os. What is the most probable dx?
a. Thyrotoxicosis
b. Hyperemesis gravidarum
c. Twins
d. Wrong dates
e. Molar pregnancy
key : E
reason: increased uterine size for date + hyperemesis + vaginal bleeding
Molar pregnancy

complete mole >>> MRI-guided transcutaneous focused ultrasound,,the most


common

partial mile >>>the trophoblast cells have three sets of chromosomes (triploid)

risk factors>>> mother over 45ys,,previous molar pregnancy,multiple


pregnancy,menarche over 12 ys ,asian women , oral contraceptive pills

presentation >>>vaginal bleeding in the first trimester , hyperemesis, abnormal


uterine enlargement, hyperthyroidism, anaemia, respiratory distress and preeclampsia are now rare as a result of routine use of ultrasound in early pregnancy

Investigations

Urine and blood levels of hCG >>> for follow up after evacuation

histology>>>> Definitive diagnosis is made by histological examination of the


products of conception.

u/s>>>>Ultrasound in the first trimester may not be reliable. The typical


'snowstorm' appearance occurs mainly in the second trimester

treatment :

Suction curettage is the method of choice of evacuation for complete molar


pregnancies.

Suction curettage is the method of choice of evacuation for partial molar


pregnancies except when the size of the fetal parts deters the use of suction
curettage and then medical evacuation can be used.

follow up :

A urinary pregnancy test should be performed three weeks after medical


management of failed pregnancy if products of conception are not sent for
histological examination.

complications: choriocarcinoma >>> follows a molar pregnancy and should always


be considered when a patient has continued vaginal bleeding after the end of a
pregnancy. It has the ability to spread locally, as well as metastasise.

Molar Pregnancy
- Tumor consists of chorionic villi which have swollen and degenerated
- It makes a lot of HCG and therefore gives rise to exaggerated pregnancy symptoms, e.g.
Hyperemesis Gravidarum
- Signs
- Exaggerated pregnancy symptoms
- A larger for dates uterus
- Hyperthyroidism
- Most present with early pregnancy failure
- Investigation
- Snowstorm appearance on U/S
- Management
- Suction and curettage is the method of choice for evacuation
- Give anti-D prophylaxis
- Monitor HCG levels every two weeks until they are normal

- Monitor monthly for six months after they return to normal


- Pregnancy should be avoided for a year while HCG levels are being monitored
- Measure HCG 6-8 weeks after any future pregnancy regardless of the outcome

1111. A pregnant woman of G2, GA 11wks presents with heavy vomiting, headache and reduced
urine
output. Urine analysis shows ketonuria. Choose the next best step?
a. US
b. Oral fluid replacement
c. Serum BHCG
d. Parental anti-emetics
e. IV fluids
key: E
reason : the pt has reduced urine output,vomiting ,headache ,ketonuria >>which indicates
hypovolemia >>> IV fluids
Vomiting in pregnancy
Presentation

Symptoms usually start between 4 and 7 weeks of gestation and resolve by 16


weeks in about 90% of women. Check for signs of dehydration

Management

Most cases are mild and do not require treatment

Diet >>> Advise the patient to rest; eat small, frequent meals that are high in
carbohydrate and low in fat

Anti-emetic drug treatment>>> This should only be given when symptoms are
persistent, severe and preventing daily activities( cyclizine, metoclopramide,
prochlorperazine, promethazine, chlorpromazine, domperidone and ondansetron )

Proton pump inhibitors and histamine H2-receptor antagonists may be used in


women who also have dyspepsia

Hyperemesis gravidarum:

intractable vomiting associated with weight loss of more than 5% of pre-pregnancy


weight, dehydration, electrolyte imbalances( hyponatremia,hypokalemia), ketosis,
and the need for admission to hospital.

management:

dietary advice

Fluid and electrolyte replacement >>> intravenous fluid and electrolyte replacement

Nutritional support (enteral or parenteral)

Vitamin supplements >>> thiamine

Thromboprophylaxis >>> LMWH

Anti-emetic drugs

Corticosteroids: may be used for intractable (failure to respond to conventional


treatment) cases of severe hyperemesis gravidarum in secondary care.

In cases of hyperemesis gravidarum: renal function and electrolytes, LFTs,


midstream urine and ultrasound (exclude multiple or molar pregnancy).

Treating Dehydration due to Heavy Vomiting (E)


- This patient has heavy vomiting which causes dehydration.
- The immediate next step here would be fluid correction through IV route.
1112. A pt had inflammatory changes on cervical smear. There is no vaginal discharge, no pelvic
pain
and no fever. What is the next step?
a. Repeat smear in 6m
b. Take swab
c. Treat with antibiotics
d. Colposcopy
e. Cone biopsy
key : b
reason : if inaccurate repeat the test ( full topic discussed earlier )
Inflammatory Changes on Cervical Smear (A)
- OHCS on encountering inflammatory changes: Take swab. Treat infection. Repeat in 6 months.
Colposcopy after 3 abnormal.

1113. A 37yo infertile lady with 5 cm subserosal and 3 cm submucosal fibroid is trying to get
pregnant.
Which is the most suitable option?
a. Clomiphene therapy
b. IVF
c. Myomectomy
d. Hysterectomy
e. IU insemination
key : c
reason : submucosal fibroids cause infertility >> myomectomy ( full topic discussed earlier)
- Refer to 1108. It is explained in detail over there.

1114. A young tall man and his wife are trying for babies and present at the infertility clinic. On
inv the
man has primary infertility and azoospermia. What other inv should be done?
a. Testosterone
b. LSH
c. FSH
d. Estradiol
e. Karyotyping
key : e
reason : in primary azoospermia + tall man >>> karyotyping >>> klinefelter's syndrome
Klinefelter's syndrome
karyotype XXY is associated with hypogonadism and disorders of spermatogenesis.
The classic clinical description

Infertility and small firm testes are present in about 99% of individuals ,decreased
facial and
pubic hair; loss of libido; impotence.

Tall and slender, with long legs, narrow shoulders, and wide hips.

Gynaecomastia or history of gynaecomastia during puberty; decreased libido;


history of undescended testes.

Learning disability; delayed speech development; behavioural problems;


psychosocial disturbances.

tiredness, reduced muscle power and stamina, and truncal obesity

Investigations

XXY males may be diagnosed before birth, through amniocentesis or chorionic villus
sampling.

Later, serum testosterone is low or low normal. FSH and LH are elevated (FSH >LH).

Diagnosis is confirmed by chromosomal analysis. The most common indications for


karyotyping are hypogonadism and infertility.

Management

testosterone replacement

fertility treatment : ICSI

Surgical treatment : for gynaecomastia

- Young tall man with primary infertility and Azoospermia points in the direction of Klinefelters
Syndrome.
- The definitive diagnosis is chromosomal analysis also known as karyotyping.

1115. A woman who is on regular COCP presented to you for advice on what to do as she has
to now
start to take a course of 7d antibiotics. What would you advice?
a. Continue regular COC
b. Continue COCP and backup contraception using condoms for 2d
c. Continue COCP and backup contraception using condoms for 7d
d. Continue COCP and backup contraception using condoms for 2wks
key : D
reason : There are many commonly used medications which can affect the efficacy of the pill
INTERACTIONS :

*Non-enzyme-inducing antibacterial >>> Women should be advised that no


additional contraception is required.

*Enzyme-inducing antibacterials rifampicin or rifabutin , and anticonvulsants, St


John's wort, (Short course two months or less ) >>> Women are advised to continue
taking the COCP ,use additional precautions AND should be continued for 28 days
after stopping the rifampicin/rifabutin.

*Enzyme-inducing antibacterials rifampicin or rifabutin,and anticonvulsants, St


John's wort (Long-term course) >>> Should be advised to use an alternative, nonhormonal method

*Lamotrigine >>> Women should be advised not to take lamotrigine with the COCP
and should seek another form of contraception (unless also taking a non-enzymeinducing anticonvulsant such as sodium valproate)

*Antiretroviral therapies>>> women on ritonavir-boosted protease inhibitors should


be advised to use alternative methods of contraception.

*Ulipristal acetate>>>Women should use additional contraceptive precautions for


14 days after taking ulipristal acetate

- Antibiotics like Rifampicin and Griseofulvin increase the breakdown of estrogen and therefore
can cause unwanted pregnancy.
- In such cases, backup contraception using condoms for two weeks should be advised.

1116. A lady presents with hot flashes and other symptoms of menopause. What is the tx
option?
a. Raloxifene
b. HRT
c. Bisphosphonate
d. COCP
e. Topical estrogen
key :B
reason : HRT is the gold standard treatment for hot flushes.
MENOPAUSE
The menopause is a natural phenomenon occurs in all women when the number of ovarian
follicles are depleted.
oestrogen & progesterone fall, and LH &FSH increase in response. Menstruation becomes
erratic and eventually stops
PRESENTATION

Menstrual irregularity >>> which may last for up to four years,The cycle may
lengthen or shorten,A slight increase in the amount of menstrual blood loss,10% of
women have an abrupt cessation of periods.

Hot flushes and sweats

Urinary and vaginal symptoms >>> dyspareunia, vaginal discomfort and dryness,
recurrent lower urinary tract infection and urinary incontinence

Sleep disturbance , Loss of libido AND Mood changes

Management

Healthy lifestyle >>> Stopping smoking, losing weight and limiting alcohol

Hormone replacement therapy (HRT) >>> the most effective treatment to


completely relieve the symptoms particularly Vasomotor symptoms (hot
flushes/night sweats) ,Mood swings ,Vaginal and bladder symptoms caused by the
menopause. It also prevents and reverses bone loss.

Alternatives to HRT >>> Herbal or complementary treatments

Other drugs >>> GABA , SSRI

1116. Menopause (B)


- Permanent cessation of menstruation for a minimum of 12 months in the absence of other
causes of Amenorrhea.
- Usually occurs between 45-55 years; average age is 52.
- Symptoms
- Hot flushes
- Depression
- Anxiety
- Irritability
- Mood swings
- Vaginal dryness
- Atrophic Vaginitis leading to dyspareunia
- Treatment
- Hormone-Replacement Therapy is the gold standard.
- Estrogen-only HRT is suitable for candidates who have had a hysterectomy
- Combined HRT is suitable for patients with a uterus

- Vaginal dryness and atrophic vaginitis should be treated with topical estrogen
- Long-term Complications
- Osteoporosis
- Cardiovascular disease
- Urogenital Atrophy Atrophic Vaginitis
- HRT Contraindications
- Estrogen-dependent cancer
- Past pulmonary embolus
- Undiagnosed PV bleeding
- Increased LFTs
- Pregnancy
- Breastfeeding
- Phlebitis
1117. A 28yo woman at 34wks GA for her first pregnancy attends antenatal clinic. Her blood
results:
Hgb=10.6, MCV=95, MCHC=350. What do you do for her?
a. Folate
b. Dextran
c. Ferrous sulphate
d. None
e. IV FeSO4
f. Explain this physiologic
hemodynamic anemia
g. Blood transfusion
key : F
reason : normal MCV,MCHC + in the second and third trimester anemia is considered when Hb is
less than 10.5
Anemia in pregnancy

definition >>> Hb level <11.0 g/dL at booking. In the second and third trimesters the diagnostic
level for anaemia is an Hb level of <10.5 g/dL. Postpartum the diagnostic level is 10.0 g/dL.
Iron-deficiency anaemia accounts for 85% of all cases of anaemia that are identified and is
characterised by MCV, and it's caused by nutritional deficiency or low iron stores
Presentation >>> often asymptomatic,Fatigue,Dyspnoea m pallor
Investigations

Hb

MCV 76 >>> the probable cause is iron deficiency

Normal MCV (76-96 fl) with low Hb is typical of pregnancy

Serum ferritin 10-50 g/L needs monitoring and <10 g/L requires treatment.

Management:

Women with known haemoglobinopathy >>> check serum ferritin >>> if low give
oral supplements

Women with unknown haemoglobinopathy status with a normocytic or microcytic


anaemia>>> oral supplements and haemoglobinopathy screening

Non-anaemic women at increased risk of iron deficiency >>> check serum ferritin
>>> if low give oral supplements

Women with established iron deficiency anaemia >>> give 100-200 mg elemental
iron daily, continue for at least three months

* Referral to secondary care in case of >>> significant symptoms ,severe anaemia (Hb<70
g/dL) ,late gestation (>34 weeks) or failure to respond to a trial of oral iron.
- Hb < 11.5 g/dl
- 85% of the times the cause is Iron deficiency
- Other causes are less common
- Investigations
- Hb
- MCV: If less than 76, it is most probably due to Iron deficiency. If it is normal then it is the
typical dilutional anemia of pregnancy.
- Serum Ferritin: 10-50 ug/L needs monitoring; less than 10 ug/L requires treatment
- Management
- Routine Iron Replacement is not recommended in the UK.

- Treat with oral Ferrous Sulphate if Hb is less than 11.5 g/dl

1118. A 34yo woman who never had fits or high BP developed fits 6h after delivery of a term
healthy
child. What is the most likely dx?
a. Eclampsia
b. Preeclampsia
c. Epilepsy
d. Pulmonary embolism
e. Pregnancy induced HTN
key : C
reason: postpartum fits>> occurs in 45% of cases of eclampsia
Eclampsia
Eclampsia is defined as the occurrence of one or more convulsions superimposed on preeclampsia. 44% of seizures occur postnatally, the remainder being antepartum (38%) or
intrapartum (18%).
Management of eclampsia:

Resuscitation >> The patient should be placed in the left lateral position and the
airway secured + O2

Treatment and prophylaxis of seizures >> Magnesium sulfate is the anticonvulsant


drug of choice, intubation may become necessary in women with repeated seizures

Treatment of hypertension >> (blood pressure >160/110 mmHg or mean arterial


pressure >125 mmHg) ,,,Intravenous hydralazine or labetalol are the two most
commonly used drugs ( SE >> fetal distress,so continuous fetal heart rate
monitoring)

Fluid therapy

Delivery >>>The definitive treatment of eclampsia is delivery

Complications :

haemolysis, HELLP syndrome (3%), disseminated intravascular coagulation (3%),


renal failure (4%) and adult respiratory distress syndrome (3%).

Cerebrovascular haemorrhage is a complicating factor in 1-2%.

- Symptoms of pre-eclampsia+Seizure = Eclampsia


- NOTE: If a woman has a fit within a few days after delivery, it is always eclampsia until proven
otherwise.
- Management
- ABC
- MgSO4 IV bolus 4g, then 1g IV infusion for 24 hours, and if seizure recurs give IV bolus.
- Monitor BP, pulse, RR and Oxygen saturation every 15 minutes.
- If BP> 160/110, give antihypertensive (Hydralazine, Labetalol, Nifedipine)
- NOTE: If the patient has been given MgSO4 and experiences another fit, repeat MgSO4.

1119. A 30yo lady who already has one child through a prv C-section demands a reversible
contraception. She presently experiences heavy and painful periods. What is the most
appropriate contraceptive you will recommend for her?
a. COCP
b. POP
c. Implanon
d. Danazol
e. Mirena
f. IUCD
key : E
reason: Women with idiopathic menorrhagia, Consider the LNG-IUS(Merina) as the first-line
option
Contraception in special groups

Menorrhagia >>> Women with idiopathic menorrhagia>>> Consider the LNGIUS(Merina) as the first-line option, COCP as the second-line option, and POP and
progestogen-only injectables as third-line options.

Fibroid >>> With distortion of the uterine cavity: cu-IUCDs and the LNG-IUS should
NOT be used

obese >>> COCP shouldn NOT be used

Migraine :

-WITH AURA >>> don't use COCP ---- use POP, progestogen-only implants and injectables,
and the LNG-IUS BUT they are not recommended to continue if the woman developed migraine
with aura while using them
-WITHOUT AURA (<35 years of age) >>> NOT usually recommended >>> continuation of
COCP, the combined contraceptive patch, and combined contraceptive vaginal ring
-WITHOUT AURA ( 35 years of age) >>> don't use COCP

DM >>> Methods that should not usually be used: progestogen-only injectables.


COCP, the combined contraceptive patch and the combined contraceptive vaginal
ring

Hypertension >>> Methods that are not usually recommended: COCP, the
combined contraceptive patch and combined contraceptive vaginal ring.

Irregular menses >>> COCP

Smoking >>> Methods that should not be used: COCP, combined contraceptive
patch and combined contraceptive vaginal ring.

Multiple risk factors for cardiovascular disease>>> Methods that should not be
used: COCP, combined contraceptive patch and combined contraceptive vaginal ring
AND POP are not usually recommended

Venous thromboembolism or risk of thromboembolism>>> Methods that should not


be used: COCP, the combined contraceptive patch, and the combined contraceptive
vaginal ring.

Sickle cell disease >>> intrauterine devices are not recommended, as they may be
associated with uterine bleeding and infection , Depot contraceptive (DepoProvera) is safe and has been found to improve the blood picture and reduce pain
crises.

Women taking anticoagulants for VTE >>> Methods that should not be used: COCP,
combined contraceptive patch and combined contraceptive vaginal ring AND POP
are not usually recommended

Sexually transmitted infection or pelvic inflammatory disease >>> insertion of a


cu-IUCD or the LNG-IUS is not recommended.

Contraception for those with learning disabilities >>> use of injectable


contraceptives and IUCDs is high

Contraception for those also taking enzyme enhancers >>>COCP - all women
should be advised to switch to a contraceptive method unaffected by enzyme
inducers (eg progestogen-only injectable, copper IUCD (Cu-IUCDs) or LNG-IUS).or
to cover with another method eg. condoms

POP>>> Advise alternative contraceptive methods

Progestogen-only implants >>> May continue with progestogen-only implants


with
additional contraceptive protection, such as
condoms, when taking liver enzyme-inducers and for four weeks after they are stopped.
drugs which induce liver enzymes include:
Antifungals: griseofulvin.
Antibiotics: rifampicin and rifabutin.
Anti-epileptics: carbamazepine, eslicarbazepine, phenytoin, phenobarbital,
primidone, oxcarbazepine, topiramate.
Central nervous system stimulant: modafinil.
Antiretroviral drugs: nelfinavir, nevirapine, ritonavir.
St John's wort.
- Mirena, also known as Intrauterine System, is a Levonorgestril-containing coil which is inserted
into the uterus.
- Local effect: reversible endometrial atrophy, makes implantation less likely and periods lighter
(20% reversible amenorrhea)
- Less risk of ectopic pregnancy
- Risk of STDs is reduced
- May benefit women with endometriosis, adenomyosis, fibroids or endometrial hyperplasia
- NOTE: Avoid if breast cancer.

1120. A 32yo woman comes with intermenstrual bleeding. Her last cervical smear was 1y ago
and was
negative. What test would you recommend for her initially?
a. Colposcopy
b. Cervical smear
c. Endocervical swab
d. Transvaginal US
e. Pelvic CT
key : b
reason : to exclude cervical carcinoma

- In this patient there is no indication for colposcopy. Indications for colposcopy include:
- Smear showing mild, moderate or severe Dyskaryosis
- Any suggestion of malignancy
- 3 consecutive inflammatory smears
- Glandular abnormal cells
- 2 consecutive atypical smears
- 3 consecutive borderline or inadequate smears
- Post-coital bleeding
- Endocervical Swab is not advisable at this point due to absence of any signs of infection
- Initially, a cervical smear should be taken as the previous one was negative one year ago (25-50
years, take cervical smear every 3 years)

1121. A 20yo woman has had abdominal pain in the LIF for 6wks duration. Over the past 48h,
she has
severe abdominal pain and has a fever of 39.1C. Pelvic US shows a complex cystic 7 cm mass in
the LIF. What is the most likely dx?
a. Endometriosis
b. Dermoid cyst
c. Ovarian ca
d. Tubo-ovarian abscess
e. Ectopic pregnancy
key : D
reason : fever 39.1 +cystic pelvic mass + localized abdominal pain
Tubo-ovarian abscess
A tubo-ovarian abscess is one type of pelvic abscess which is found in women of reproductive
age, and may be a complication of pelvic inflammatory disease. In this case it is an inflammatory
mass which involves the ovary and Fallopian tube
Presentation

Systemic features of toxicity: fever, malaise, anorexia, nausea, vomiting, pyrexia.

Local effects: eg, pain, deep tenderness , diarrhoea, tenesmus, mucous discharge
per rectum, urinary frequency, dysuria, vaginal bleeding or discharge.

Investigations

FBC >>> raised white cell count often but not invariably

Ultrasound

CT/MRI scanning may be more effective at identifying the origin of the abscess

Management:

Hospital admission

Drainage of the abscess along with antibiotic treatment

Antibiotics used alone are occasionally effective for very early, small abscesses

Procedures used for drainage of the pelvic abscess >>> Ultrasound-guided aspiration
and drainage----CT-guided aspiration and drainage----Endoscopic ultrasound-guided
drainage ----Laparotomy or laparoscopy with drainage of abscess

Definitive surgery may be required after initial drainage for some causes of pelvic
abscess, such as salpingo-oophorectomy for tubo-ovarian abscess.

-General symptomatology of pelvic infections


- Systemic features of toxicity: fever, malaise, anorexia, nausea, vomiting, pyrexia.
- Local effects: eg, pain, deep tenderness in one or both lower quadrants, diarrhoea, tenesmus,
mucous discharge per rectum, urinary frequency, dysuria, vaginal bleeding or discharge.
- Rectal or vaginal examination: may reveal tenderness of the pelvic peritoneum and bulging of
the anterior rectal wall.
- Partial obstruction of the small intestine: this may sometimes occur.
-Investigations
- FBC showing increased WBC count
- U/S
- CT/MRI scanning may help in tracing the origin of the abscess
- Management
- Arrange urgent admission to hospital.
- Management is usually by drainage of the abscess along with antibiotic treatment. Antibiotics
used alone are occasionally effective for very early, small abscesses

- Antibiotic choice is guided by the likely cause and local resistance patterns and guidelines, but
usually needs to be broad-spectrum until the pathogens are determined.
- Points in favour of Tubo-Ovarian Abscess
- Fever
- Pain in the LIF
- Severe abdominal pain
- U/S showing a cystic mass in the LIF

1122. A woman is 16wk pregnant and she is worried about abnormal chromosomal anomaly in
her
child. What is the definitive inv at this stage?
a. Amniocentesis
b. CVS
c. Parents karyotyping
d. Coombs test
e. Preimplantation genetic dx
key : A
Prenatal diagnosis

Amniocentesis : This is normally carried out from 15 weeks of gestation. A needle is


inserted into the amniotic cavity and amniotic fluid is sampled, allowing culture and
assessment of fetal cells.the most invasive prenatal diagnostic procedure.

Chorionic villus sampling : carried out at 11-13 weeks. Risk of miscarriage may be
slightly higher than for amniocentesis. sampling of the developing placenta and the
same type of analysis of fetal cells to detect chromosomal, genetically inherited and
endocrine or metabolic conditions.

Fetoscopy : This allows visualisation of the fetus, using endoscopic techniques. at


18-20 weeks of gestation

Cordocentesis/percutaneous umbilical blood sampling : at 18 weeks.ultrasound


guidance to obtain fetal blood cells from the umbilical cord.It enables
karyotyping/chromosome analysis, Intrauterine blood transfusions, Fetal viral
infection confirmation.

Fetal radiology : in suspected skeletal dysplasia, ultrasound (2D and 3D) and MRI are
now the investigations of choice

Ultrasound-guided percutaneous skin and organ biopsy : This can also be carried out
to allow skin, muscle, liver and other fetal organ analysis

Maternal blood tests : eg. Maternal serum alpha-fetoprotein levels can be measured
to aid the diagnosis of neural tube defects between 15-22 weeks

- Definitive tests
- Pre-implantation Genetic Diagnosis (earliest possible diagnosis; available to couples at risk of
having a child with specific genetic or chromosomal disorder such as CF, Thalassemia and
Huntingtons etc)
- Chorionic Villous Sampling (done between 10 and 13 weeks; enables early detection and
provides the mother with a choice to either continue or terminate pregnancy)
- Amniocentesis (done between 14-18 weeks; carries a very low risk of miscarriage)
- The patient is 16 weeks pregnant, so the best definitive testing that can be carried out at this
stage would be Amniocentesis.

1123. A 28yo lady with a fam hx of CF comes for genetic counselling and wants the earliest
possible dx
test for CF for the baby she is planning. She is not in favor of termination. What would you
recommend for her?
a. CVS
b. Amniocentesis
c. Pre-implantation genetic dx
d. Chromosomal karyotyping
e. Maternal serum test
f. Reassure
key : C
reason :pre-implantation genetic dx is the earliest diagnostic test
Pre-implantation prenatal diagnosis
This is a technique that allows the analysis of oocytes or embryos conceived through in vitro
fertilisation (IVF). This information then informs the choice of optimal embryos to be transferred

back to the mother. This is an accepted technique for avoiding the birth of affected children
from parents with a known genetic abnormality
- There is a positive family history of CF, therefore the earliest possible diagnosis can be made
via Pre-implantation genetic testing.
- See above for further details
1124. A 39yo woman in her 36th week GA with acute abdominal pain is rushed for immediate
delivery.
Her report: BP=110/60mmHg, Hgb=low, bilirubin=22, AST=35, Plt=60, APTT=60, PT=30,
Fibrinogen=0.6. What is the cause?
a. Pregnancy induced hypertension
b. DIC
c. HELLP syndrome
d. Acute fatty live
e. Obstetric cholestasis
key : B
reason : low fibrinogen,increased bilirubin ,normal AST , prolonged PT,PTT
DIC
The diagnosis of DIC should include both clinical and laboratory information:

(PT) elevated.

(aPTT) elevated.

Platelet counts in DIC are typically low

Fibrinogen level low.

Confirmatory tests :

The D-dimer test gives strong evidence of DIC.

Fibrin degradation products (FDPs) are helpful but can occur in other conditions
such as deep vein thrombosis (DVT)

In acute DIC, PT and aPTT are prolonged, and the platelet count and fibrinogen
decrease. D-dimer, FDP, and fibrin monomer levels are elevated

Management :

The cornerstone of the the management of DIC is treatment of the underlying condition. Thus,
infection will need antibiotics, and obstetric complications may need intervention
plasma and /or platelet transfusion
Conditions that may be complicated by DIC include:
* Infections, especially septicaemia, Escherichia coliO157, typhoid fever, Rocky Mountain
spotted fever and parasites. The rash of meningococcal septicaemia is classical.
* Malignancy, especially leukaemias.
* Major trauma including crush syndrome and, occasionally, burns. * Some connective tissue
disorders including antiphospholipid syndrome. * Complications of pregnancy including the
placental problem of placental abruption, amniotic fluid embolism, severe hypertension of
pregnancy with fulminating pre-eclampsia and HELLP syndrome. A retained dead fetus tends to
produce a thrombotic rather than a haemorrhagic state.
* Incompatible blood transfusion.
* Heat stroke.
* Dissecting aortic aneurysm.
* Some snake bites
If DIC is suspected then clotting screen tests are followed by confirmation:
- Prothrombin time (PT) elevated.
- Activated partial thromboplastin time (aPTT) elevated.
- Platelet counts in DIC are typically low, especially in acute sepsis-associated DIC, but may be
increased in malignancy-associated chronic DIC.
- Fibrinogen level low. If two results are positive,DIAGNOSIS is possible; if three are positive, it is
likely; if all four are positive, it is extremely likely.
- Confirmatory tests look for evidence of the simultaneous formation of thrombin and plasmin.
- The D-dimer test gives strong evidence of DIC.
- Fibrin degradation products (FDPs) are helpful but can occur in other conditions such as deep
vein thrombosis (DVT) and, in severe disease, they may be negative.
- In acute DIC, PT and aPTT are prolonged, and the platelet count and fibrinogen decrease. Ddimer, FDP, and fibrin monomer levels are elevated.

1125. A 36wk pregnant woman presents with sudden onset of uterine pain and bleeding, uterus
is

tender, no prv LSCS. What is the most appropriate cause?


a. Preeclampsia
b. DIC
c. Placental abruption
d. Placental previa
e. Ectopic pregnancy
f. Missed abortion
g. Ectropion
key : C
reason : a case of antepartum haemorrhage in the third trimester >>> pain + bleeding >>>
abruptin ( pain is common with abruption) ( full topic discussed earlier)
- Bleeding during the third trimester is either painful or painless bleeding.
- Painful bleeding points to Placental Abruption while painless bleeding points to Placenta Previa
- In this case, the patient is experiencing painful vaginal bleeding in the third trimester, so this
appears to be the case of Placental Abruption
- The investigation of choice in this case is an Ultrasound
- Risk Factors for Abruption
- Hypertension
- Smoking
- Multiple Pregnancy
- Cocaine/Amphetamine Use
- Increased Maternal Age
- Trauma to the abdomen
- Polyhydramnios
- Investigation
- Diagnosis is clinical but U/S is done to exclude Placenta Praevia and to check the well-being of
the baby.
- Management

- Always admit the patient to hospital for assessment and management. Phone 999/112/911 if
there are any major concerns regarding maternal or fetal well-being.
- The mainstays of management are resuscitation and accurate diagnosis of the underlying
cause.
- Severe bleeding or fetal distress: urgent delivery of the baby, irrespective of gestational age.
- Admit to hospital, even if bleeding is only a very small amount. There may be a large amount of
concealed bleeding with only a small amount of revealed vaginal bleeding.
- No vaginal examination should be attempted, at least until a placenta praevia is excluded by
ultrasound. It may initiate torrential bleeding from a placenta praevia.
- Resuscitation can be inadequate because of underestimation of blood loss and misleading
maternal response. A young woman may maintain a normal blood pressure until sudden and
catastrophic decompensation occurs.
- Take blood for FBC and clotting studies. Crossmatch, as heavy loss may require transfusion.
- Gentle palpation of the abdomen to determine the gestational age of the fetus, presentation
and position.
- Fetal monitoring.
- Arrange urgent ultrasound.
- With every episode of bleeding, a rhesus-negative woman should have a Kleihauer test and be
given prophylactic anti-D immunoglobulin
- Complications
- Premature Labour
- DIC
- Renal Tubular Necrosis
- PPH
- Placenta Accreta
- Points to look for diagnosis of Abruption
- Shock is out of proportion from visible blood loss
- Constant pain
- Tender tense uterus
- Fetal heart sounds absent/distressed
- Coagulation problems like DIC

1126. A 28wk pregnant woman presents with uterine bleeding after sexual intercourse. What is
the
most appropriate cause?
a. Preeclampsia
b. DIC
c. Placental abruption
d. Placental previa
e. Ectopic pregnancy
f. Missed abortion
g. Ectropion
key : G
reason :postcoital bleeding >>> mostly ectropion
Causes of postcoital bleeding

Infection,Trauma.

Cervical ectropion - especially in those women taking the combined oral


contraceptive pill (COCP).

Cervical or endometrial polyps.

Vaginal cancer , Cervical cancer - usually apparent on speculum examination.


Ectropion

It is most commonly seen in teenagers, during pregnancy and in women on


combined hormonal contraception.

It is generally an asymptomatic condition but patients occasionally present with


bleeding or excessive discharge.

Once a normal cervical smear has been confirmed, it is actively managed only if
there are symptoms. Over time, vaginal acidity promotes metaplasia to squamous
epithelium when the symptoms will disappear.

After stopping any oestrogen-containing contraceptive, treatment options are


controversial but include diathermy, cryotherapy, surgery with laser treatment and
microwave therapy.

- The cervix enlarges under the influence of oestrogen and as a result the endocervical canal is
everted. It is seen on examination as a red ring around the os and is so common as to be
regarded as normal. - It is most commonly seen in teenagers, during pregnancy and in women
on combined hormonal contraception.
- This seems to be the most appropriate among the given options because the patient is 28
weeks pregnant and had sex at this point.
- There are no signs of pre-eclampsia, DIC, Placental abruption and Placenta Praevia. Ectopic
pregnancy and missed abortion do not present at this stage.
1127. A 6wk pregnant woman presents with abdominal pain. She has prv hx of PID. What is the
most
likely dx?
a. Preeclampsia
b. DIC
c. Placental abruption
d. Placental previa
e. Ectopic pregnancy
f. Missed abortion
g. Ectropion
key : E
reason : hx of PID + early pregnancy + abdominal pain
Ectopic pregnancy
The majority of ectopic pregnancies occur in the ampullary or isthmic portions of the Fallopian
tubes
Risk factors

IUCD

PID

in tubes that have been divided in a sterilisation operation and where they have
been reconstructed to reverse on

Presentation :

30% of ectopic pregnancies present before a period has been missed.


Abdominal pain.

Pelvic pain.

Amenorrhoea or missed period.

Vaginal bleeding (with or without clots)

Investigations:

The most accurate method to detect a tubal pregnancy is transvaginal ultrasound.

hCG levels are performed in women with pregnancy of unknown location who are
clinically stable

Management :

Medical management: systemic methotrexate is offered first-line to those women


who are able to return for follow-up and who have the following:

No significant pain,Unruptured ectopic pregnancy with an adnexal mass <35 mm


and no visible heartbeat, No intrauterine pregnancy seen on ultrasound scan
,Serum hCG <1500 IU/L.

Surgical management: offered to those women who can not return for follow-up
after methotrexate or to those who have any of the following:

Significant pain.

Adnexal mass 35 mm.

Fetal heartbeat visible on scan.

Serum hCG level 5000 IU/L.

A laparoscopic approach is preferable. A salpingectomy should be performed,


unless the woman has other risk factors for infertility, in which case a
salpingotomy should be undertaken.

- Symptoms may develop at any time between 4 and 10 weeks of pregnancy. * - Pain on one
side of the lower tummy (abdomen). It may develop sharply, or may slowly get worse over
several days. It can become severe. * Vaginal bleeding often occurs, but not always. It is
often different to the bleeding of a period. For example, the bleeding may be heavier or
lighter than a normal period. The blood may look darker. However, you may think the
bleeding is a late period.
* Shoulder tip pain due to irritation of the diaphragm.
- Risk factors
- Women over the age of 35
- Previous ectopic pregnancy
- Scarring, damage or other structural abnormality of the fallopian tube

- PID (This patient in question has a history of PID)


- IUCDs
- Investigations
- Urine pregnancy test
- Transvaginal U/S
- Beta-HCG
- Diagnostic Laproscopy
- Treatment options
- Ruptured ectopic pregnancy
Emergency surgery is needed if a Fallopian tube ruptures with heavy bleeding. The main
aim is to stop the bleeding. The ruptured Fallopian tube and remnant of the early pregnancy
are then removed. The operation is often life-saving.
- Early ectopic pregnancy - before rupture
Ectopic pregnancy is most often diagnosed before rupture. Your doctor will discuss the
treatment options with you and, in many cases, you are able to decide which treatment is
best for you. These may include the following:
* Surgery. Removal of the tube (either the whole tube or part of it) and the ectopic
pregnancy is most commonly performed by keyhole surgery (a laparoscopic operation).
Removal of the Fallopian tube containing the ectopic pregnancy (salpingectomy) is usually
performed if the other tube is healthy. Removal of only a section of the tube with the ectopic
pregnancy in it (salpingotomy) is usually performed if the other tube is unhealthy; for
example, scarred from a previous infection. However, many women with an ectopic
pregnancy do not need to have an operation.
* Medical treatment. Medical treatment of ectopic pregnancies is now more common and
avoids the need for surgery. A medicine called methotrexate is often given, usually as an
injection. It works by killing the cells of the pregnancy growing in the Fallopian tube. It is
normally only advised if the pregnancy is very early. The advantage is that you do not need
an operation. The disadvantage is that you will need close observation for several weeks
with repeated blood tests andSCANS to check it has worked. You will need to have a blood
test for hCG every 2-3 days until your levels are low. Scans are usually repeated weekly.
Methotrexate can cause side-effects which include nausea and vomiting in some women. It
can be common for some abdominal pains to develop 3-7 days after having methotrexate.

1128. A 33wk pregnant woman presents with vaginal bleeding, low Hgb, low plt, increased
bilirubin,
AST normal, APTT & PT increased. What is the most likely dx?

a. Preeclampsia
b. DIC
c. Placental abruption
d. Placental previa
e. Ectopic pregnancy
f. Missed abortion
g. Ectropion
key : B
reason : low Hb,plt,,, high PT,PTT ,bilirubin,,,normal AST (full topic discussed earlier)
Conditions that may be complicated by DIC include:
* Infections, especially septicaemia, Escherichia coliO157, typhoid fever, Rocky Mountain
spotted fever and parasites. The rash of meningococcal septicaemia is classical.
* Malignancy, especially leukaemias.
* Major trauma including crush syndrome and, occasionally, burns. * Some connective tissue
disorders including antiphospholipid syndrome.[2] * Complications of pregnancy including the
placental problem of placental abruption, amniotic fluid embolism, severe hypertension of
pregnancy with fulminating pre-eclampsia and HELLP syndrome. A retained dead fetus tends to
produce a thrombotic rather than a haemorrhagic state.
* Incompatible blood transfusion.
* Heat stroke.
* Dissecting aortic aneurysm.
* Some snake bites
If DIC is suspected then clotting screen tests are followed by confirmation:
- Prothrombin time (PT) elevated.
- Activated partial thromboplastin time (aPTT) elevated.
- Platelet counts in DIC are typically low, especially in acute sepsis-associated DIC, but may be
increased in malignancy-associated chronic DIC.[3]
- Fibrinogen level low. If two results are positive,DIAGNOSIS is possible; if three are positive, it is
likely; if all four are positive, it is extremely likely.
- Confirmatory tests look for evidence of the simultaneous formation of thrombin and plasmin.
- The D-dimer test gives strong evidence of DIC.

- Fibrin degradation products (FDPs) are helpful but can occur in other conditions such as deep
vein thrombosis (DVT) and, in severe disease, they may be negative.
- In acute DIC, PT and aPTT are prolonged, and the platelet count and fibrinogen decrease. Ddimer, FDP, and fibrin monomer levels are elevated.
1129. A 25yo lady at her 28th week GA came for check up. Her BP=160/95mmHg, protein in
urine=6g/d.
What is the most likely dx?
a. Essential HTN
b. Gestational HTN
c. Chronic HTN
d. Preeclampsia
key : D
reason : HTN + protienuria +/- edema >>> preeclampsia
Pre-eclampsia
Pre-eclampsia is pregnancy-induced hypertension in association with proteinuria (>0.3 g in 24
hours) with or without oedema.
Severe pre-eclampsia is defined as diastolic blood pressure of at least 110 mm Hg, or systolic
blood pressure of at least 160 mm Hg, and/or symptoms, and/or biochemical and/or
haematological impairment
Presentation:

New hypertension ,New and/or significant proteinuria.

Other clinical features of severe pre-eclampsia include:

Severe headache - usually frontal.


Platelet count falling to below 100 x 109/
Abnormal liver enzymes (ALT or AST rising to above 70 IU/L).
HELLP syndrome: H (haemolysis) EL (elevated liver enzymes) LP (low platelets).
Investigations :

Urinalysis

Frequent monitoring of FBC, LFTs, renal function, electrolytes and serum urate

Clotting studies if there is severe pre-eclampsia or thrombocytopenia

24-hour urine collections for protein quantification and creatinine clearance.

Assessment of fetus -ultrasound

Management:

Control Blood pressure

Prevention of seizures >>> Magnesium sulfate

Fluid balance

Delivery

- Pre-eclampsia: BP > 140/90 and 300 mg proteinuria in 24-hour urine collection


- Mild to moderate: BP <160/110 with significant proteinuria and no maternal complications
- Severe: BP >160/110 with significant proteinuria or if maternal complications occur
- Risk factors
- Previous history of pre-eclampsia
- Maternal age > 40 years
- Family History
- DM, HTN, Renal Disease
- This is a case of severe pre-eclampsia as the BP of the patient in question suggests. This should
be managed along the following lines.
- Anti-hypertensives to bring BP down to less than 160/110
- IV Hydralazine is the first choice
- Labetolol
- MgSO4 to prevent eclampsia
- CTG and U/S to monitor the baby
NOTE: If less than 34 weeks gestation, give steroids to help production of surfactant.
- Complications
- Eclampsia
- HELLP Syndrome
- DIC
- Renal Failure

- Placental Abruption

1130. A 32yo woman has a hx of spontaneous abortions at 6wks, 12wks, and 20wks. She is now
keen
to conceive again. Which of the following would you prescribe for the next pregnancy?
a. MgSO4
b. Aspirin
c. Warfarin
d. Mefenamic acid
e. Heparin
key : B
reason : antiphospholipid syndrome is the most important treatable cause of recurrent
miscarriage >>>> Rx >>> Asprin
recurrent miscarriage
def. >>> the loss of three or more consecutive pregnancies
Aetiology :

Antiphospholipid syndrome >> investigation : anticardiolipin antibodies , lupus


anticoagulant >>> Rx : Aspirin

structural >>> uterine anomalies , fibroids , cervical incompetence ( late miscarriage)


>>> investigation : pelvic u/s >>> Rx in cervical incompetence : cerclage
(complication : uterine rupture)

endocrine >>> PCOS , uncontrolled diabetes

immune

thrombophilia

genetic abnormality

infection >>> bacterial vaginosis in the first trimester is a risk of second trimester
miscarriage and preterm delivery

- Recurrent miscarriage is defined as the loss of three or more consecutive pregnancies. Miscarriage, the most common complication of pregnancy, is the spontaneous loss of a
pregnancy before the fetus has reached viability. The term therefore includes all pregnancy

losses from the time of conception until 24 weeks of gestation in the UK. - Antiphospholipid
syndrome (APS):
- This is the most important treatable cause of recurrent miscarriage.
- Investigations
- Antiphospholipid antibodies:
The presence of these is associated with early miscarriages and maternal morbidity and is
referred to as primary APS. There is requirement for two tests at least six weeks apart showing
either lupus anticoagulant or anticardiolipin antibodies at significant levels.
- Women with recurrent first-trimester miscarriage and all women with one or more secondtrimester miscarriages should be screened for antiphospholipid antibodies before pregnancy.
- Women with second-trimester miscarriage should be screened for inherited thrombophilias
including factor V Leiden, factor II (prothrombin) gene mutation and protein S.
- All women with recurrent first-trimester miscarriage and all women with one or more secondtrimester miscarriages should have pelvic ultrasound to assess uterine anatomy.
- If uterine anomalies are detected then further investigations, such as hysteroscopy and/or
laparoscopy, may be required.
- Management
- General advice
- Reassurance should be given about the high probability of a successful outcome. In a large trial
that included women with 4.2 consecutive miscarriages and an average age of 32.7 years, the
placebo group was shown to have a live birth rate of 65%.
Pharmacological treatment
- In primary APS patients, heparin combined with low-dose aspirin improves live birth rate to
70%.[5] There ARE only limited data supporting the use of heparin in women without APS
- There is some evidence suggesting that use of metformin during pregnancy is associated with a
reduction in the miscarriage rate in women with polycystic ovarian syndrome
- However, the RCOG DOES NOT recommend its use in pregnancy at present until further
randomised prospective study results are available to provide adequate evidence of safety and
efficacy of its use.
- A Cochrane review found evidence of benefit for progestogen therapy in women with a history
of recurrent miscarriage. There was no statistically significant difference in rates of adverse
effects.
- However, there is currently a large randomised, double-blind, placebo-controlled multicentre
trial underway - the Progesterone in recurrent miscarriage (PROMISE) study - which aims to
provide a definitive answer regarding progesterone use in women with recurrent miscarriages.

Surgical
- Cervical cerclage is used where cervical incompetence is suspected. However, it is
overdiagnosed as a cause of second-trimester miscarriage. The cerclage procedure also carries a
risk of stimulating uterine contractions.
- Cerclage benefit increases as the cervix shortens to less than 25 mm. It has also been shown to
be beneficial in those women with a shortened cervical length of less than 25 mm.

1131. A 6yo child presents with hx of recurrent jaundice. Between the episodes he is totally fine.
Mother gives hx of jaundice being brought about by ongoing infections. What is the most likely
dx?
a. Hereditary spherocytosis
b. G6PD deficiency
c. Thalassemia
d. Sickle cell disease
e. Congenital storage disorder
key : B
reason : hemolytic attack triggered by infection + normal in between attacks >> G6PD deficiency
haemolytic anemia
DDx
Genetic

Red cell membrane abnormalities: hereditary spherocytosis, elliptocytosis.


Haemoglobin abnormalities: sickle cell anaemia, thalassaemia.
Enzyme defects: glucose-6-phosphate dehydrogenase (G6PD), pyruvate kinase
deficiency.

Acquired

Immune:

- Isoimmune: haemolytic disease of newborn, blood transfusion reaction.

Autoimmune:

-Warm antibody : SLE

-Cold antibody type :


-Drug-related :

Non-immune: trauma ,infection , hypersplenism, membrane disorders, paroxysmal


nocturnal haemoglobinuria, liver disease.
G6PD

PRESENTATION :
Most are asymptomatic,, History of drug or infection induced haemolysis,,Gallstones are
common, history of neonatal jaundice ,During a crisis jaundice occurs,
investigations :
G6PD enzyme activity - is the definitive test
FBC : anemia ,macrocytosis , reticulocytosis during the attack
Blood film in acute attack : Heinz bodies
Haemolysis - reduced levels of haptoglobin and elevated levels of bilirubin; haemoglobinuria.
Ultrasound examination of the abdomen may reveal splenomegaly and gallstones.
Management of acute haemolysis :
blood transfusion may be needed,Dialysis may be required in acute kidney injury.
Infants - more susceptible to neonatal jaundice, especially if premature, and exchange
transfusion may be required.
Management of chronic haemolysis :
Splenectomy may help ,Supplementation with folic acid
Avoid : precipitating drugs, broad beans and naphthalene - found in mothballs.
- X-linked disease with about 300 variants reported
- Precipitating factors
- Certain drugs like Primaquine, Methylthioninium, Nitrofurantoin, Sulfonamides including coTrimoxazole, Dapsone etc.
- Certain foods like broad beans
- Severe infection
- DKA
- Acute Kidney Injury

- Presentation
- Pallor of anemia
- Jaundice during crisis
- Back or abdominal pain
- Splenomegaly may occur
- Investigations
* FBC - anaemia.
* Macrocytosis - due to reduced folic acid which is required for erythropoiesis.
* Reticulocyte count - raised; gives indication of the bone marrow activity (bone marrow
sampling thus not needed).
* Blood film - acute haemolysis from G6PD deficiency can produce Heinz bodies, which are
denatured haemoglobin and bite cells (cells with Heinz bodies that pass through the spleen have
part of the membrane removed).
* Haemolysis - reduced levels of haptoglobin and elevated levels of bilirubin; haemoglobinuria.
* Direct antiglobulin test - to look for other causes of haemolysis; should be negative in G6PD
deficiency.
* Renal function - to ensure no renal failure as a precipitant.
* LFTs - to exclude other causes of raised bilirubin.
* G6PD enzyme activity - is the definitive test (as opposed to the amount of G6PD protein).
* Performing assays for G6PD during haemolysis and reticulocytosis may affect levels and not
reflect baseline values.
* Ultrasound examination of the abdomen may reveal splenomegaly and gallstones.
- Management
Avoidance of substances that may precipitate hemolysis is essential. Usually no further
management is required, although if hemolysis is marked there may be benefit from folate
supplementation.
Management of acute haemolysis
* Seek specialised advice.
* Blood transfusions may be needed.
* Dialysis may be required in acute kidney injury.

* Infants - more susceptible to neonatal jaundice, especially if premature, and exchange


transfusion may be required.
Management of chronic haemolysis or stable disease
* Splenectomy may help.
* Supplementation with folic acid.
* Avoidance of precipitating drugs, and broad beans (usually favism occurs in the Mediterranean
variety of the disease).
* Avoid naphthalene - found in mothballs.

1132. A 42yo woman who smokes 20 cigarettes/d presents with complaints of heavy bleeding
and
prolonged menstrual period. What is the most appropriate tx for her?
a. Tranexamic acid
b. COCP
c. Mefenamic acid
d. IUCD
e. Norethisterone
key : D
reason : smoker >>> so no use of COCP ,,, heavy bleeding >>> IUCD ( FULL TOPIC DICUSSED
EARLIER)
The key says IUCD (D) but it is not the first choice in heavy/prolonged menstrual periods. Mirena
Coil is. The second line is Tranexamic Acid. Third line is COCPs. Fourth line is Endometrial
ablation or Hysterectomy (If there is no desire to conceive). IUCD, according to patient.info
actually causes heavy or painful periods. The logical choice in this case would be Tranexamic
Acid since Mirena is not mentioned in the given options.
1133. A 17yo senior schoolgirl with complain of prolonged irregular menstrual period and heavy
blood
losses. What is the most appropriate tx for her?
a. Mefenamic acid
b. COCP

c. POP
d. IUCD
e. Mirena
KEY : B
reason : irregular menses + heavy bleeding >>> COCP can treat both (Full topic discussed earlier)
- COCPs are widely used for irregular menstrual periods.
- They also carry the advantage of causing a decrease in bleeding and menstrual pain (can be
used for dysmenorrhea and menorrhagia)
1134. A 32yo presents with heavy blood loss, US: uterine thickness>14mm. What is the most
appropriate tx for her?
a. Mefenamic acid
b. COCP
c. POP
d. IUCD
e. IU system (mirena)
key :E
reason : heavy bleeding >>> IUS is the first line (full topic discussed earlier)
- Mirena, also known as Intrauterine System, is a Levonorgestrel-containing coil which is
inserted into the uterus.
- Local effect: reversible endometrial atrophy, makes implantation less likely and periods lighter
(20% reversible amenorrhea)
- Less risk of ectopic pregnancy
- Risk of STDs is reduced
- May benefit women with endometriosis, adenomyosis, fibroids or endometrial hyperplasia
- NOTE: Avoid if breast cancer.

1135. A 37yo woman presents with heavy bleeding. Inv show subserosal fibroid=4 cm and
intramural

fibroid=6cm. Which is the most appropriate tx?


a. UAE
b. Abdominal hysterectomy
c. Hysteroscopic Myomectomy
d. Vaginal Hysterectomy
e. Abdominal myomectomy
key : e
reason : abdominal myomectomy is the best alternative for hysterectomy in patients who want
to keep their fertility ,, pt is 37 ys and this procedure can treat both intramural and subserosal
fibroids (full topic discussed earlier)
- Fibroids are responsive to estrogen and therefore increase in size, which in turn increases the
size of the uterus.
- Other symptoms
- Pelvic pain (Compression on to adjacent structures)
- Infertility/Recurrent Miscarriages
- Pelvic Mass
- The investigation of choice is an U/S
- Management
- Mirena Coil is the first choice if the fibroids are not big enough to restrict its insertion.
- If < 3 cm
- Trial of pharmacologic treatment first (Tranexamic Acid) first
- If it fails and uterus is not bigger than 10-week pregnancy, do endometrial ablation
- If the above fails, perform a hysterectomy
- If > 3 cm and wishes to retain uterus and/or wants to avoid surgery
- Go for Uterine Artery Embolization
- If > 3 cm and wishes to retain uterus, go for a hysteroscopic myomectomy or a myomectomy
NOTE: This patient has a subserosal fibroid so an abdominal approach should be adopted.
1136. A woman with sickle cell disease complains of heavy menstrual blood loss. What is the
most

appropriate tx?
a. COCP
b. Mirena
c. Depot provera
d. Copper IUS
e. Transdermal patch
key : c
reason : sickle cell disease :
intrauterine devices are not recommended, as they may be associated with uterine bleeding
and infection.
Combined hormonal methods are not recommended , because of the risk of thromboembolism
in sickle cell patients
Depot contraceptive (Depo-Provera) is safe and has been found to improve the blood picture
and reduce pain crises.[15]
The choice of contraceptive method needs to be considered carefully. The coil (intrauterine
contraceptive device) may cause particularly heavy painful periods. The use of injectable
contraceptives (such as Depo-Provera) has been reported to provide some protection against
sickling episodes.
1137. A 70yo woman is admitted with diarrhea, vomiting and dehydration. Exam: yellow visual
halos in
her eyes, ECG=bradycardia. She has a hx of chronic A-fib. Which drug causes the above
mentioned side effects?
a. Nifedipine
b. Ramipril
c. Atenolol
d. Lithium
e. Digoxin
key : e
reason : symptoms are classic for digoxin toxicity

Features suggestive of toxicity include nausea, vomiting, diarrhoea, dyspnoea, confusion,


dizziness, headache, blurred vision and diplopia ( yellow halos),bradycardia, skin rash, renal
dysfunction and hypokalaemia
Plasma concentrations is helpful when initiating therapy, checking compliance or detecting
toxicity. Levels above 2 nmol/L suggest toxicity.
- Adverse Effects of Digoxin
- Diarrhea
- Nausea
- Vomitting
- Dizziness
- Headache
- Maculopapular rash
- Cardiac dysrhythmia
- Arrhythmia in children
- Visual disturbances (Blurred or yellow vision)
- Heart Block
- Asystole
1138. A 33yo lady who is a drug addict wants to quit. She says she is ready to stop the drug
abuse. She
is supported by her friends and family. What drug tx would you give her?
a. Benzodiazepines
b. Diazipoxide
c. Lithium
d. Methadone
e. Disulfiram
key : D
reason :methadone is used to treat opioid withdrawal symptoms
Methadone or buprenorphine can be used in opioid dependance treatment

NICE recommends that, if both drugs are equally suitable, methadone should be prescribed as
first choice
- Methadone is used as a pain reliever and as part of drug addiction detoxification and
maintenance programs.
- For detoxification during withdrawals Methadone is the first choice.
- Methadone is also used for maintenance.
- Benzodiazepines, Diazepoxide and Disulfiram are specifically used in alcohol withdrawal.
- Lithium is used in Mania and Bipolar Affective Disorder.

1139. A 50yo lady has been suffering from chronic RA and is on methotrexate and naproxen. Her
CBC
shoes microcytic anemia. What is the most likely cause?
a. Anemia of chronic disease
b. GI hemorrhage
c. Menorrhagia
key : B
reason: because of use of NSAIDs >>> naproxen
anemia of chronic illness is normocytic normochromic
microcytic anemia >>> iron deficiency due to chronic blood loss >>> GI hrg
- There is a history of chronic NSAID use, this leads to weakening of defense mechanism of the
mucosa of the stomach.
- This leads to exposure of the mucosa to gastric acid and causes ulcers.
- Bleeding from these ulcers can lead to anemia.

1140. A 15yo male noticed swelling on the left knee following a fall while playing. The swelling
has not
subsided in spite of rest and analgesia. Exam: full knee movement with slight tenderness. He has
painless palpable mass in left inguinal region. What is the most probable dx?
a. Osteosarcoma
b. Ewings sarcoma

c. Chondrosarcoma
d. Lymphangiosarcoma
e. Osteodosteoma
key : A
reason : age ( highest in 15-19 years) + site ( around knee) + LN metastasis
Osteosarcoma

The most common primary bone malignancy in children. The incidence is highest
in 15-19 ys

The male:female ratio is 1.4:1.

The most common sites are around the knee (75%), or proximal humerus.

Often presents as a relatively painless tumour.

Rapidly metastasises to the lung >>associated with a poorer prognosis

X-ray shows combination of bone destruction and formation. Soft tissue


calcification produces a 'sunburst' appearance.

Disease-free survival has increased to 55-75% with surgery and effective


chemotherapy ,Chemotherapy alone is not as effective.

Presentation
pain, swelling and localised tenderness,Rapid growth and erythema ,pathological fractures
Investigations : Plain X-ray ,MRI and CT scan ,Biopsy
Treatment : surgery + chemotherapy
Osteosarcoma
This is the most common type of primary bone cancer but even this is rare. It only affects
around 150 people a year in the UK. It arises from bone-forming cells. Most cases occur in young
people between the ages of 10 to 25. It can, however, occur at any age. It typically develops in
the growing ends of the bone in young people, most commonly in bones next to the knee and
the upper arms. However, any bone can be affected.
Ewing's sarcoma
The cells of this cancer look different to the more common osteosarcoma. It only affects around
100 people a year in the UK. Most cases occur in young people between the ages of 10 to 20,
but it can occur at any age. It most commonly affects the hips (pelvis) and long bones in the leg.
- The age and symptoms fit both the above conditions.

1141. A 45yo female looking pale has bluish discoloration of hands whenever she goes out in
the cold.
She has also noticed some reddish spots on her body. She has symmetrical peripheral
arthropathy for the last yr. What is the most probable dx?
a. RA
b. Osteosarcoma
c. Limited systemic sclerosis
d. Diffuse systemic sclerosis
e. Chondrosarcoma
key : C
reason : classic picture of limited SSc
it's a disease of connective tissue disease like as RA, SLE. so arthritis here and symmetrical arthritis. two
variety of Systemic sclerosis. LCSS- Involvement distal to the knee and elbows with CREST syndrome,
DCSS- involvement proximal to knee and elbows with renal involvement (scleroderma renal crisis ).
presence of telangiectasia and Raynaud pheno indicates LCSS.in RA, arthritis present but not raynauds
pheno. Raynaud's also present in SLE.

scleroderma
Common presenting symptoms are Raynaud's phenomenon, skin hardening in hands or face,
and oesophageal symptoms.
types of SSC
Limited scleroderma >>> Generally a milder disease, with less skin involvement, slow onset and
slow progression.
70% of SSc cases.
Affects only the face, forearms and lower legs up to the knee.
The older term for limited scleroderma is CREST syndrome (= Calcinosis, Raynaud's disease,
(O)Esophageal dysmotility, Sclerodactyly, Telangiectasia).
Diffuse scleroderma >>> Usually a more rapid onset, with skin thickening and Raynaud's
phenomenon occurring together or within a short interval
30% of SSc cases ,, Involves also the upper arms, thighs or trunk
- Formerly known as the CREST Syndrome
- C: Calcinosis
- R: Raynauds Phenomenon

- E: Esophageal and Gut dysmotility


- S: Sclerodactyly
- T: Telangiectasias (Reddish Spots)

1142. A 60yo female has pain and stiffness in her right hip joint. Pain is not severe in the
morning but
increases as the day progresses. She has noticed some nodules in her hands. Inv: Hgb=low. What
is the most probable dx?
a. RA
b. Osteoarthritis
c. Gout
d. Pseudogout
e. Multiple myeloma
key : B
reason : A diagnosis of OA can be made clinically without investigations if a person:

Is aged 45 years or over; and

Has activity-related joint pain; and

Has either no morning joint-related stiffness or morning stiffness that lasts no


longer than 30 minutes.
osteoarthritis

Symptoms
Joint pain that is exacerbated by exercise and relieved by rest. Rest and night pain can occur in
advanced disease. Knee pain due to OA is usually bilateral and felt in and around the knee. Hip
pain due to OA is felt in the groin and anterior or lateral thigh. Hip OA pain can also be referred
to the knee and, in males, to the testicle on the affected side.
Joint stiffness in the morning or after rest.
Signs
Pain and Reduced range of joint movement.
Joint swelling/synovitis (warmth, effusion, synovial thickening).

Bony swelling and deformity due to osteophytes


- Commonest joint condition
- Usually monoarthritis
- Usually affects females 50 years and above
- Commonly affects the weight-bearing joints e.g, Hip and knee joints. (Notice the joint involved)
- Pain on movement and worsening towards the end of the day
- Bouchards nodes and Heberdens nodes are usually present.
- Investigations
- X-ray
- Treatment
-Advise exercise and activity, and physiotherapy
- Paracetamol
- Topical NSAIDs
- Intra-articular steroids
- Low-dose of tricyclic antidepressants for pain at night
- Weight reduction
- Joint replacement in end-stage Osteoarthritis

1143. A 30yo female has chronic diarrhea, mouth ulcers and skin tags. She complains of visual
prbs,
low back pain and morning stiffness. Inv: ESR and CRP=raised, Hgb=10 mg/dl. What is the most
probable dx?
a. SLE
b. Reactive Arthritis
c. Gout
d. Pseudogout
e. Seronegative arthritis
key : B

reason : the symptoms fits ( GIT or urinary tract symptoms >> diarrhea + law back pain + visual
problems >> uveitis + lab >> anemia and inreased ESR & CRP )
Reactive arthritis or Reiter's syndrome >>> is a form of seronegative spondyloarthritis clinically
associated with inflammatory back pain, GIT symptoms . The presence of large joint
oligoarthritis, urogenital tract infection and uveitis characterises Reiter's syndrome as a clinical
subtype of reactive arthritis.
Presentation

develops 2-4 weeks after a genitourinary or gastrointestinal infection.

The onset is most often acute, with malaise, fatigue, and fever.

An asymmetrical, predominantly lower extremity, Low back pain often occurs.

The complete Reiter's triad of urethritis, conjunctivitis, and arthritis may occur.

Skin ,nails and mucous membranes (mouth ulcers) may all be affected.

Eyes: uveitis, episcleritis, keratitis, and corneal ulcerations.

Gastrointestinal: abdominal pain and diarrhoea

Investigations :
**Once arthritis is observed, microbial tests and blood or synovial fluid cultures are negative,
and only serum antibodies are detected.

ESR and CRP are usually very high.

FBC: normocytic normochromic anaemia

HLA-B27 is positive

Management

In the acute phase, rest affected joints, aspirate synovial effusions.

Physiotherapy.

Non-steroidal anti-inflammatory drugs (NSAIDs).

Corticosteroids

Antibiotics to treat an identified causative organism

1144. A 28yo woman has been on tx for RA for 3yrs. She has gradual loss of vision in both eyes.
Her
IOP is normal. Red reflex is absent in both eyes. What is the single most likely dx?

a. Cataract
b. DM retinopathy
c. Hypermetropia
d. Macular degeneration
e. HTN retinopathy
key : A
reason : absent red reflex + tx for RA for 3yrs+ gradual loss of vision in both eyes.
the treatment plan of rheumatoid arthritis includes : corticosteroids which induce cataract
formation with long term use
- Cataracts are cloudy (opaque) areas that develop in the lens of an eye and affects vision
- Vision becomes gradually worse over the years.
- Most affected people develop a cataract for no apparent reason. Factors that may increase the
chance of developing cataracts include:
- Having a poor diet.
- Smoking.
- Being exposed to a lot of ultraviolet light.
- Diabetes.
- Steroid medicines.
- Having a family history of cataracts
- There are no medicines, eye drops or lasers that can treat cataracts. The only way of treating
cataracts is with an operation. This is a very common operation.

1145. An elderly man with recently dx HF has been treated with diuretics. He now develops
severe
joint pain in his left ankle with swelling and redness. What is single most likely inv?
a. XR of bone
b. Plasma RF
c. Joint fluid uric acid crystals
d. ESR

key : c
reason : both thiazides and loop diuretics can precipitate or worsen pre-existing gout. If a
diuretic is unavoidable, consider prophylaxis with allopurinol.
Gout
Primary gout occurs mainly in men aged 30-60 years presenting with acute attacks.
secondary gout is due to chronic diuretic therapy. It occurs in older subjects, both men and
women, and is often associated with osteoarthritis.
Risk factors :
Male sex ,Meat ,Seafood ,Diuretics ,Obesity ,Hypertension ,Coronary heart disease ,Diabetes
mellitus ,Chronic renal failure, High triglycerides
Presentation:
50% of all attacks and 70% of first attacks affect the first metatarsophalangeal joint.
Other sites often affected are: Knee, Midtarsal joints ,Wrists, Ankles ,Small hand joints ,Elbows
Management :
colchicine and/or NSAIDs as the first-line option for acute gout.
- There is a clear history of diuretic use which causes Hyperuricemia in almost 40% of the
patients.
- Hyperuricemia can lead to development of gout (Notice the redness, severe pain, swelling and
involvement of a single joint).
- Gout is usually precipitated by Trauma, Tumor Lysis Syndrome, Surgery, Infection, Diuretics,
Polycythemia, Leukemia, Cytotoxic Drugs and Alcohol Abuse
- Investigation
- Joint aspiration for microscopy, C/S, which shows negatively birefringent crystals
- Treatment
- NSAIDs
- If contraindicated, give Colchicine
- If there is renal failure, then both NSAIDs and Colchicine are problematic, so use steroids.

1146. A 60yo lady with a hx of HTN and suffering from RA since the last 10y now presents with
hot,

swollen and tender knee joint. What inv would you do for her?
a. XR
b. C&S of joint aspirate
c. US
d. MRI
e. CT
key : B
reason : septic arthritis is the most important diagnosis to exclude as, if left untreated, the
sequelae include permanent joint damage, impairment of function and even death
DDx of hot swollen tender joint
Infection:

Septic arthritis >> acute onset , monoarthritis, mostly knee in adults and hip in
children , plus constitutional symptoms ( fever , malaise)

Neisseria gonorrhoeae

Lyme disease >> erythema migrans

Rheumatoid arthritis >>> insidious onset ,polyarthritis


Crystal arthropathies - gout and pseudogout >>> acute onset , monoarthritis mostly, 70% of
attacks first occur in the big toe in gout
Reactive arthritis (now considered synonymous with Reiter's syndrome). >>> polyarthritis , Hx
of gastrointestinal or genitourinary infection
Trauma >>> Hx of trauma
- History of Rubor, Erythema and swelling in a single joint is Septic Arthritis until proven
otherwise.
- Therefore the investigation of choice would be C/S of joint aspirate.
- Rest of the investigations do not help in making the diagnosis or exclusion of septic arthritis.
1147. A 34yo man after an RTA was brought to the ED. He has BP=50/0mmHg and chest wall
with
asymmetrical movement, RR=34bpm. What would be the initial action?
a. IV fluid infusion

b. Intubation and ventilation


c. CT chest
d. Transfer to ITU
key : B
reason : in trauma patient we follow : Airway ,Breathing ,Circulation ,Disability , Exposure>> and
that patient has asymmetrical chest movement and increased RR>>> we have to secure the air
way and ventilate the pt
Initial assessment
the 'ABCDE' principles :
Airway maintenance with cervical spine protection
Breathing and ventilation
Circulation with haemorrhage control
Disability: neurological status
Exposure/environmental control

1148. A 7yo presented with chronic cough and is also found to be jaundiced on exam. What is
the
most likely dx?
a. Congenital diaphragmatic hernia
b. Congenital cystic adenomatoid malformation
c. Bronchiolitis
d. RDS
e. Alpha 1 antitrypsin deficiency
key : E
reason : clinical picture fits >>> The organs most commonly involved are the lungs and the liver.
Alpha 1 antitrypsin deficiency

A1AT deficiency is an inherited condition.

In A1AT deficiency, the protein is still produced but the A1AT molecule configuration
is changed. As a result, it cannot pass out of the liver into the bloodstream and so
cannot pass to the lungs and the rest of the body

Some people with A1AT deficiency develop liver disease. This results from the
congestion of A1AT in the liver cells, leading to cell destruction.

If there is a deficiency of A1AT then elastase can break down elastin unchecked; in
the lungs this can lead to the destruction of alveolar walls and emphysematous
change >>> COPD

Neonates with A1AT deficiency may present with neonatal jaundice and hepatitis;
older children may develop hepatitis, cirrhosis and liver failure due to A1AT
deficiency.

Investigations

Serum levels of alpha-1 antitrypsin

Phenotyping

CXR and lung function testing ,CT scanning of the chest.

LFTs and possibly liver biopsy.

- It is an autosomal recessive inherited disorder


- It commonly affects lungs (Emphysema) and liver (Cirrhosis and Hepatocellular Carcinoma)
- Investigation
- Serum alpha 1 antitrypsin levels
- Management
- Supportive treatment for Emphysema and Liver disease may be sufficient for some.
- Among the given options no condition can explain the involvement of lungs and liver at the
same time.

1149. A 65yo man had a bowel resection 5d ago. He is anuric and breathless. His BP=150/110
mmHg.
He has crackles at both lung bases and sacral edema. Bloods: K+=6.8mmol/l, urea=58 mmol/l,
creatinine=600 umol/l. What is the single most appropriate immediate management?
a. Bolus of 20U insulin
b. Calcium resonium enema
c. Dextrose-saline infusion
d. 5% dextrose infusion

e. 10U insulin, 50ml of 50% dextrose infusion


key : e
reason : this is a case of acute kidney injury following major surgery ( increased urea and
creatinine + hyperkalemia ( normal range 3.5-5 ) >>> to treat hyperkalemia >>> Shift potassium
into cells using 10U insulin, 50ml of 50% dextrose infusion
Hyperkalaemia
normal k+ 3.5-5 mmol/L
Mild Hyperkalaemia - 5.5-5.9 mmol/L
Moderate Hyperkalaemia - 6.0-6.4 mmol/L
Severe Hyperkalaemia - >6.5 mmol/L
Causes:

Renal causes: eg. Acute kidney injury (AKI) ,Chronic kidney disease (CKD)
Increased circulation of potassium:
Exogenous - eg, potassium supplementation.
Endogenous - eg, tumour lysis syndrome, rhabdomyolysis, trauma, burns.

A shift from the intracellular to the extracellular space:


Acidosis - eg, diabetic ketoacidosis (DKA).
Medications - eg, digoxin toxicity, suxamethonium, beta-blockade, theophylline.

ECG changes in hyperkalemia : Peaked T waves ,prolonged PR interval ,Wide QRS,bradycardia.


Management :

Stop further potassium accumulation: Stop any potassium supplements stop


digoxin and beta-blockers,Decrease potassium in the diet

Protect cardiac membrane: Give 10 ml 10% calcium gluconate

Shift potassium into cells: Insulin-glucose IV >> usually 10 units of Actrapid are
added to 50 ml of glucose 50% and infused over 30 minutes.

Remove potassium from the body : Calcium polystyrene sulfonate resin (Calcium
Resonium) with regular lactulose will remove potassium via the gastrointestinal
tract.

** in resistant hyperalaemia Haemodialysis may be required but is invasive.

1150. A 25yo woman presents with a painful shallow ulcer on the vulva. What inv has to be
done?
a. HSV antibodies
b. Syphilis serology
c. Swab for haemophilus ducreyi
d. Urine culture
e. Blood culture
key : c
reason : shallow painful ulcer >> chancroid
chancroid:
Chancroid is a sexually transmitted disease (STD) caused by haemophilus ducreyi characterized
by painful shallow with soft ragged margins necrotizing genital ulcers that may be accompanied
by inguinal lymphadenopathy.
- The symptoms point towards the causative organism being Haemophilus Ducreyi as it causes
painful shallow ulcers.
- Mnemonic: YOU CRY WITH DUCREYI
- The other conditions cannot explain the symptoms above as they do not cause painful shallow
ulcers. HSV cause vesicles while Syphilis causes painless ulcers.
- Urine and Blood Culture are not required for the same reason as stated above.

1151. A child was admitted with fever, generalized skin lesion, some of them are weeping
lesions and
some of them are crusted. What is the most probable dx?
a. Varicella
b. Impetigo
c. Drug reaction
d. Contact dermatitis
e. Scabies
Impetigo (B)
-Infection due to Staph. Aureus.

- Usually on the face with honey-coloured fluid in an erythematous base.


- Common in children.
- Children should be kept off school or nursery until there is no more blistering or crusting or
until 48 hours after antibiotic treatment has been started.
-Treatment
- Flucloxacillin
1152. A pt comes with 6m hx of painless bilateral swelling of the face which has been
progressively
increasing in size. On routine CXR, he is found to have perihilar lymphadenopathy. What is the
most probable dx?
a. Chronic sialadenitis
b. Thyroid adenoma
c. Carcinoma of salivary gland
d. Adenoid cystic carcinoma
e. Mikuliczs disease
Mikuliczs Disease (E) - Bilateral parotid and lacrimal gland enlargement was characterized by
the term Mikulicz's disease if the enlargement appeared apart from other diseases.
- In 80% of cases, the parotid gland is affected. Lacrimal glands are also affected. - The gland
affected has a diffuse swelling. The swelling can be asymptomatic, but mild pain can also be
associated. - A biopsy is needed to distinguish benign lymphoepithelial lesions from sialadenosis
(sialosis).
- Treatment
- Treatment usually consists of observation unless the patient has concern, there is pain,
drainage, or other symptoms related to the lesion. Surgical removal of the affected gland would
be recommended in those cases.

1153. A woman has widespread metastasis from a carcinoma. She presented with severe back
pain.
Where do you expect the cancer to be?
a. Lungs
b. Cervix

c. Ovary
d. Uterus
e. Breast
E - Breast.
- Occasionally, breast cancer presents as metastatic diseasethat has spread beyond the
original organ. The symptoms caused by metastatic breast cancer will depend on the location of
metastasis. Common sites of metastasis include bone, liver, lung and brain.
- Bone is the commonest site of metastasis in Breast Cancer

1154. A 10yo child has got progressive bilateral hearing loss. He has started to increase the TV
volume.
All other examination is normal. What is the most likely dx?
a. Wax
b. Foreign body
c. Bilateral OM with effusion
d. SNHL
e. Meningitis due to meningococcus
Progressive Bilateral Hearing Loss in a Child (C)
- Otitis media is an inflammation in the middle ear (the area behind the eardrum) that is usually
associated with the buildup of fluid. The fluid may or may not be infected.
- Symptoms, severity, frequency, and length of the condition vary. At one extreme is a single
short period of thin, clear, non-infected fluid without any pain or fever but with a slight decrease
in hearing ability. At the other extreme are repeated bouts with infection, thick "glue-like" fluid
and possible complications such as permanent hearing loss.
- Fluctuating conductive hearing loss nearly always occurs with all types of otitis media. In fact it
is the most common cause of hearing loss in young children.
Management General advice
* Give written information about OME to the parents.
* Advise parents or carers not to expose the child to tobacco smoke.
NICE recommends hearing aids for children with bilateral OME and hearing loss where surgery is
not acceptable or is contra-indicated. Each case needs to be considered on its own merits: the

need to assist hearing during a period of active observation, for example, has to be weighed
against evidence that the use of aids in children can increase anxiety.

1155. A child had a patchy rash following tx for sore throat & cervical LN enlargement. Which is
the
most appropriate antibiotic?
a. Ampicillin
b. Erythromycin
c. Cefuroxime
d. Metronidazole
e. Tetracycline
EBV infection (A)
NOTE: The question here is not asking for appropriate treatment. It is actually trying to ask
about the drug that has caused a rash in a child with the given symptoms.
- A child with a sore throat and Cervical Lymphadenopathy has an active EBV infection until
proven otherwise. - Ampicillin and amoxicillin are contraindicated during acute EpsteinBarr
virus infection since the vast majority of patients treated with them develop a diffuse nonallergic rash.

1156. A child with a hx of asthma is brought to ED with a cut on knee and sprained on her left
wrist.
Which is the best analgesic for her?
a. Paracetamol
b. NSAIDs
c. Co Codamol
d. Ibuprofen
A - Paracetamol
Paracetamol. paracetamol should only be given in this patient to relieve pain. NSAIDS should not be given
as the child has h/o asthma as nsaids may increase the risk of acute bronchospasm and co codamol has
codeine which can cause respiratory depression.

1157. A 15m baby girl presented to the ED with difficulty in breathing. Exam: she has intercostal

recessions and a wheeze. Temp=normal. What is the most likely dx?


a. URTI
b. Pneumonia
c. Bronchiolitis
d. RDS
e. Alpha 1 antitrypsin deficiency
key : c
reason : age : (younger than 2 years (most common between 2 and 6 month) + wheezy chest +
intercostal recession + normal temp )
Bronchiolitis is an acute infectious disease of the lower respiratory tract that occurs primarily in
the very young, most commonly infants between 2 and 6 months old.
It is a clinical diagnosis based upon: Breathing difficulties , Cough , Decreased
feeding ,Irritability ,Apnoeas in the very young ,Wheeze or crepitations on auscultation
the causative organism :

Respiratory syncytial virus (RSV) : the most common cause

Human metapneumovirus (hMPV) - the second most common cause

Investigations:

Pulse oximetry.

Nasopharyngeal aspirate for: RSV rapid testing

Viral cultures for RSV, influenza A and B, parainfluenza and adenovirus

Management:
Most infants with acute bronchiolitis will have mild, self-limiting illness and can be managed at
home. Supportive measures , with attention to fluid input, nutrition and temperature control.
1158. An 8yo boy develops a seizure affecting his right arm, seizure lasts for several mins. He
doesnt
remember anything what happened. On his CT: lesion in left hemisphere. What is the most
probable dx?
a. Epilepsy
b. Space occupying lesion

c. Dementia
d. Huntingtons chorea
e. Intracranial HTN
key : b
reason : CT lesion >>> Space occupying lesion
brain tumors in children
the most common subtype >>> astrocytoma followed by
the second most common >>> embryonal tumours ( primitive neuroectodermal tumours and
medulloblastoma)
Presentation:

increased intracranial pressure : Headache, nausea and vomiting, abnormalities of


gait and coordination, and papilloedema.

frontal lobe tumours are associated with personality change and occipital lobe
tumours are associated with visual deficits

Brainstem tumours: Abnormal gait and coordination, cranial nerve palsies,


pyramidal signs, headache and squint.

Central brain tumours: Headache, abnormal eye movements, squint, and nausea
and vomiting

Supratentorial tumours: Unspecified symptoms and signs of raised intracranial


pressure, seizures and papilloedema.

Posterior fossa tumours: Nausea and vomiting, headache, abnormal gait and
coordination, and papilloedema

investigations :

MRI , CT : MRI is better , it provides better images and there is no radiation


involved.

excision biopsy

1159. A 28yo female presented with complains of difficulties in swallowing liquids only. She also
suffers from recurrent chest infection in the past few months. What is the most probable dx?
a. Foreign body
b. Plummer vinson syndrome
c. Achalasia cardia

d. Peptic stricture
e. Esophageal carcinoma
key : c
reason : liquid dysphagia + recurrent chest infection ( due to regurgitation)
Achalasia is a motility disorder of the lower oesophageal or cardiac sphincter. The smooth
muscle layer of the oesophagus has impaired peristalsis and failure of the sphincter to relax
causes a functional stenosis
presentation :

The most common presenting feature is dysphagia. This affects solids more than
soft food or liquids.

Regurgitation , Chest pain , Heartburn ,

Nocturnal cough and even inhalation of refluxed contents >> recurrent chest
infection

Investigations :
Manometry is the gold standard for diagnosis of achalasia.

CXR : The classical picture of a CXR in achalasia shows a vastly dilated oesophagus
behind the heart

Barium swallow : the bird beak appearance

Endoscopy : can detect approximately a third of achalasia

lowe oesophogeal PH monitoring >> to exclude GERD

MANAGEMENT :

the Heller myotomy is the best treatment for those who are fit

Pneumatic dilatation is the preferred option for older unfit patients

Calcium-channel blockers and nitrates may be used for those who are unable to
tolerate other forms of treatment

Endoscopic injection of botulinum toxin >>> recurrence

1160. Mother having 2 children with CF. What is the risk of getting another baby?
a. 1:2
b. 1:8
c. 1:4

d. 1:16
e. 1:1
key : c
reason : cystic fibrisis is autosomal recessive gene so :

Cc
CC

Cc
Cc

normal

Cc
carrier

cc
diseased

1161. A 14yo boy has been dx with nephrotic syndrome. 5d later he presents with flank pain,
hematuria and fluctuating urea levels. A dx of renal vein thrombosis is made. What is the most
likely cause for renal vein thrombosis?
a. Protein C deficiency
b. Vasculitis
c. Loss of antithrombin III
d. High estrogen levels
e. Stasis
key : c
Complications of nephrotic syndrome include:

Decreased resistance to infections, due to urinary immunoglobulin loss.

Increased risk of arterial and venous thrombosis, due to loss of antithrombin III and
plasminogen in the urine, combined with an increase in hepatic synthesis of clotting
factors. Adults with membranous nephropathy are at particular risk

Acute kidney injury

Chronic kidney disease may occur as a result of an underlying cause - eg,


amyloidosis or diabetes.

Increased risk of osteitis fibrosa cystica and osteomalacia due to loss of vitamin Dbinding protein

1162. A 36yo woman presented with massive bleeding from multiple sites. Lab: fibrin
degradation
products: +++, plt=30, bleeding time=prolonged, PT=prolonged, APTT=prolonged. What is the

most likely dx?


a. Hemophilia
b. DIC
c. ITP
d. Factor V leiden
e. Warfarin
key : B
cause : lab criteria ( decreased fibrinogen and platelets +increased PTT,PT )
full topic discussed earlier
1163. A study was done amongst 2 hosp for the equal number of cancer pts. It was noted that
hosp A
had the higher rate of mortality than hosp B for treated cancer pts. What is the study done here
classified as?
a. Retrospective
b. Observational
c. Cohort
d. Case study
key : c
Longitudinal or cohort studies:

A group of people is followed over many years to ascertain how variables such as
smoking habits, exercise, occupation and geography may affect outcome.

Prospective studies are more highly rated than retrospective ones, although the
former obviously take many years to perform. Retrospective studies are more likely
to produce bias.

1164. A 17yo girl comes to see her GP after having unprotected sex 2d ago. She asks if her GP
can
explain to her how this prescribed procedure would work by helping her not to get pregnant.
a. It helps to prevent implantation
b. It helps in preventing or delaying ovulation
c. It causes an early miscarriage

d. It releases progesterone and stops ovulation


e. It causes local enzymatic reaction
key:A
reason : CU IUD prevents implantation
emergency contraception:

Progestogen-only - levonorgestrel >>> early in the cycle inhibits ovulation. later in


the cycle, it is unclear how it has its effect. use within 72 hours of UPSI

ulipristal acetate >>> inhibits ovulation. effective up to 120 hours after UPSI.
Pregnancy or suspected pregnancy should be excluded before use

IUCD >>> inhibitory effect on both fertilisation and implantation ( direct toxicity
effects of the copper on both ovum and sperm). up to five days after UPSI

1165. A 2d babys mother is worried about the babys hearing. Mother has a hx of conductive
hearing
loss. What is the most appropriate test?
a. Brain stem evoked response
b. CT
c. Fork test
d. MRI
e. Reassure
KEY :A
Neonatal hearing screening tests

Automated otoacoustic emissions (AOAE) test >>> measures the integrity of the
inner ear

Automated auditory brainstem responses (AABR) test >>> measures not only the
integrity of the inner ear, but also the auditory pathway.

1166. A healthy 8yo boy had antibiotic tx for meningitis. Initially he wasnt resuscitated. What
will be
the outcome if he receives full tx?
a. He will recover fully to his prv health
b. He will have hearing impairment

c. He will have brain abscess


d. He will have encephalitis
key : a
1167. A pt presented with jaundice, fever and upper abdominal pain within 24h after removal of
gallstone by ERCP. The cholangiography was done and it was patent. What is the possible cause
of his complaints?
a. Biliary infection
b. Acute pancreatitis
c. Perforation
key : b
cause : post ERCP pancreatitis

ERCP Complications

Pancreatitis - 2-9% of patients will develop pancreatitis , perioperative


indomethacin or diclofenac reduce the incidence of pancreatitis.

Infection may occur - although rates are low.

Bleeding may occur - although severe haemorrhage is rare.

Perforation of the duodenum with development of an acute abdomen.

Failure of gallstone retrieval - may need to revert to open or more invasive


procedures.

Prolonged pancreatic stenting is associated with stent occlusion, pancreatic duct


obstruction and pseudocyst formation.

1168. A mother presents with her 14m child. He holds furniture and other things to help him
stand
and walk. He can say mama and papa. He makes eye contact and smiles. He can transfer
objects from one hand to another. He responds to his name. what do you interpret from his
development?
a. Delayed gross motor development
b. Delayed fine motor development
c. Dela

yed verbal development


d. Normal development
e. Delayed social development
KEY : D

1169. A young child, 3yo, has presented with vomiting for 3d. Exam: mild-mod dehydration.
What is
his ABG profile likely to show?
a. pH low, PCO2 low
b. pH low, PCO2 high
c. pH high, PCO2 low
d. pH high, PCO2 high
e. pH normal, PCO2 normal
key : d
reason : vomiting causes metabolic alkalosis ( high ph ), and it is compansated by increased
PCO2
1170. A 68yo woman has been admitted with poor appetite, weight loss, poor concentration
and self
neglect for 3wks. She has not been eating or drinking adequately and has rarely left her bed. She
is expressive suicidal ideas and is convinced that people are out to kill her. She has been on
antidepressant therapy for the past 3m with no improvement. What is the most appropriate tx?
a. Antidepressants
b. CBT
c. Interpersonal therapy
d. ECT
e. Antipsychotics
KEY : D
REASON : suicidal thought is an indication of ECT specially after treatment failure
Indications of ECT

Severe depressive illness or refractory depression.

Catatonia.

A prolonged or severe episode of mania.


It should only be used if other treatment options have failed or the condition is
potentially life-threatening (eg, personal distress, social impairment or high suicide
risk).

ECT is not useful in schizophrenia

1171. A 78yo retired teacher was admitted for a hernioplasty procedure. After the operation he
became agitated, aggressive and confused. What is the most appropriate management?
a. Diazepam
b. Chlordiazepoxide
c. Vit B
d. Clozapine
e. Thiamine
key : b
reason :delirium tremens >>> first line >>> chlordiazepoxide
second line >>> diazepam
Delirium tremens:
Delirium tremens usually begins 24-72 hours after alcohol consumption has been reduced or
stopped,,there are signs of altered mental status eg, Hallucinations ,Confusion, Delusions
,Severe agitation , Seizures can also occur.
1172. A 25yo girl saw a tragic RTA in which a young boy was killed. The night of the event she
couldnt
sleep and the day after she suddenly lost her vision. She was prv fine and there was no hx of
medical or psychological prbs. What is the dx?
a. Conversion
b. Somatization
c. PTSD
d. Dissociation
e. GAD
key : a

reason : sudden lost her vision after she saw the accident >>> physical symptom after
psychological trauma >>> conversion
somatization >>> This is a chronic condition in which there are numerous physical complaints.
These complaints can last for years and result in substantial impairment.
1173. A 25yo man has been suffering from breathlessness and wheeze for 3m. He has been
taking
salbutamol 2puffs as required. In the last 2 wks his symptoms have worsened and he has to take
salbutamol more frequently during the day time. He also complains of excessive dyspnea at
night. What drugs or regimen would you like to add?
a. Prednisolone
b. Fluticasone + salbutamol inhaled
c. Beclomethasone inhaled
d. Montelukast PO
e. Salmetrol PO
key : c
reason :
asthma management in adults :

Step 1: mild, intermittent asthma >>> inhaled short-acting beta2 agonist

Step 2: introduction of regular preventer therapy >>> Inhaled steroids (


beclomethasone )are the most effective preventer drugs

indications :
A recent exacerbation ,Nocturnal asthma , Daytime symptoms or use of an inhaled shortacting beta2 agonist more than three times per week.

Step 3: add-on therapy >>> inhaled long-acting beta2 agonists (LABAs) such as
salmeterol or formoterol.

Step 4: poor control on moderate dose of inhaled steroid plus add-on therapy >>>
Trial an additional fourth drug over six weeks (eg, leukotriene receptor antagonist,
sustained-release theophylline or beta2 agonist tablet) and increase the inhaled
steroid to high-dose ranges.

Step 5: continuous or frequent use of oral steroids >>> he use of daily steroid tablet
in the lowest dose providing adequate control is suggested

1174. A 64yo man who was exposed to asbestos for 40yrs presents with weight loss and chest
pain.
The dx of mesothelioma has been made. He develops SOB and XR=pleural effusion. What is the
most appropriate management?
a. Thoracocenthesis
b. Chest drain
c. Radiation therapy
d. Pneumonectomy
e. Chemotherapy
key : e
chemotherapy in mesothelioma:

promising results have been achieved with pemetrexed and raltitrexed in


combination with cisplatin and other combinations, including cisplatin and
gemcitabine.

Single-agent therapy with vinorelbine may provide useful palliation with low toxicity

(NICE) has recommended pemetrexed as a possible treatment for malignant pleural


mesothelioma in people:

With advanced disease.

Whose cancer is not suitable for surgical resection.

Who have (WHO) performance status of 0 (able to carry out all normal activity
without restriction) or 1 (restricted in strenuous activity but able to move around
and carry out light work).

1175. A 72yo presents with polyuria and polydipsia. The fasting blood sugar is 8 and 10mmol/l.
BP=130/80 mmHg and the level of cholesterol=5.7mmol. There is microalbuminuria. What is
the single most appropriate next management?
a. ACEi and sulfonylurea
b. Statin and biguanide
c. Statin and glitazone
d. Insulin and ACEi
e. Statin and ACEi

key : e

reason : ACEi for microalbuminuria + statin for hyperlipidemia + Medication to


control hyperglycaemia may be required at the time of diagnosis of type 2 diabetes
or soon after.

Initial treatment for newly diagnosed diabetes :

Advice on diet and exercise

Prevention of coronary heart disease: blood pressure control , cholesterol-lowering


drugs , low dose aspirin and stop smoking

All patients with microalbuminuria or proteinuria should start (ACE) inhibitor, if


there are no contra-indications

Medication to control hyperglycaemia may be required at the time of diagnosis of


type 2 diabetes or soon after.

insulin therapy should be started immediately in those who are ill at presentation or
who have a high level of ketones in their urine. Insulin should also be considered,
regardless of age, if one or more of the following are present:

Rapid onset of symptoms.


Substantial loss of weight.
Weakness.
A first-degree relative who has type 1 diabetes

1175. A 72yo presents with polyuria and polydipsia. The fasting blood sugar is 8 and
10mmol/l.
BP=130/80 mmHg and the level of cholesterol=5.7mmol/l. There is microalbuminuria.
What is
the single most appropriate next management?
a. ACEi and sulfonylurea
b. Statin and biguanide
c. Statin and glitazone
d. Insulin and ACEi
e. Statin and ACEi
e. Statin and ACEi
ACEI should be considered in diabetic patients , especially in those with renal
complications. A statin is considered in all diabetic patients above the age of 40 years (
BNF ) . An oral hypoglycemic drug does not seem to be needed since the sugar levels
aren't that high and may respond to lifestyle and dietary modifications alone.

when microalbuminuria is present , the target BP is 125/75. therefore , ACE-i should be


given. Cholesterol is also raised , so statins
Diabetic pt, target BP <=130/80.
Diabetic pt plus microalbuminaria , target BP <=125/75.
1st line in diabetes RX is lifestyle modification than biguanides (if not responding to 1st)
1176. A 49yo woman presents to the OPD. Her oral glucose test after 2h of glucose
intake vs plasma
level in 2 different tests are 6mmol/l and 10mmol/l. This situation can be categoraized as
a. Impaired glucose tolerance
b. Impaired fasting glucose
c. T1DM
d. T2DM
e. Metabolic syndrome
a. Impaired glucose tolerance
A fasting glucose greater than or equal to 6.1 but less than 7.0 mmol/l implies impaired
fasting glucose (IFG)
Impaired glucose tolerance (IGT) is defined as fasting plasma glucose less than 7.0
mmol/l and OGTT 2-hour value greater than or equal to 7.8 mmol/l but less than 11.1
mmol/l
Diabetes UK suggests:
'People with IFG should then be offered an oral glucose tolerance test to rule out
a diagnosis of diabetes. A result below 11.1 mmol/l but above 7.8 mmol/l
indicates that the person doesn't have diabetes but does have IGT.'
1177. A white Englishman with a past hx of MI is a known HTN and DM. He is currently
on aspirin, statin and metformin. What would you add to the tx?
a. ACEi
b. Diuretic
c. Insulin
d. Beta blocker
e. CCB
a. ACEi
ACEI should even be added in patients who are non hypertensive as it reduces
morbidity and mortality post MI.
ACE inhibitors are also used to treat diabetic nephropathy and have a role in secondary
prevention of ischaemic heart disease.
1178. A 57yo man who had MI a few months ago has been having a low mood. A dx of
moderate
depression has been established. Which medication is the best tx for him?
a. SSRI
b. TCA
c. MAOi
d. Benzodiazepam

e. Mood stabilizer
a. SSRI
Selective serotonin reuptake inhibitors (SSRIs) are considered first-line treatment for the
majority of patients with moderate depression.
citalopram (re: QT interval) and fluoxetine are currently the preferred SSRIs
sertraline is useful post myocardial infarction as there is more evidence for its
safe use in this situation than other antidepressants
SSRIs should be used with caution in children and adolescents. Fluoxetine is the
drug of choice when an antidepressant is indicated
1179. A 12yo presents with chest pain. Exam: tachycardia, hypotension, dilated neck
veins and the trachea is not centrally placed. What is the next appropriate management?
a. Portable XR
b. Needle thoracocentesis
c. Chest drainage
d. ABG
e. CTPA
b. Needle thoracocentesis
clincher -trachea deviated, chest pain ..it is tension pneumo for sure !!
tension pneumo !! don't wait to fr CXR to confirm as delay in treatment may cause
cardiorespiratory arrest
Pneumothoraces are never awaited for an imaging unless they are chronic enough that
the lung underlying is adapted to one. Considering vitals of the patient it is a primary
pneumothorax with tension type dynamics. treatment includes two stages. first convert it
into an open one to decompress the tension via a wide bore iv line in 2nd ICS along the
midclavicular line immediately. Secondly convert it into a closed one by putting a chest
drain attached to an underwater seal.
1180. A 7yo child is being inv for TB. His parents dont agree for taking a BAL. what
other sample will
show growth of the organism?
a. Blood test
b. Throat swab
c. Gastric washing
d. Mantoux test
e. CSF
c. Gastric washing
For children we do early morning gastric washing because they swallow their sputum
throat swabs and sputum cultures are less preferred way to obtain a sample from
children.. Most children swallow their sputum so gastric washings samples are taken in
children.
If spontaneous sputum samples are not possible then consider bronchoscopy and
lavage or, in children, gastric washings.
1st- sputum for AFB
2nd-BAL

1181. A 51yo man had a MI a few days ago. He developed breathlessness. Echo was
done and showed a pansystolic murmur. What can be the cause of this symptom?
a. Ruptured papillary muscle
b. Acute pericarditis
c. Dresslers syndrome
d. Malignant VT
e. Ventricular aneurysm
a. Ruptured papillary muscle
Acute mitral regurgitation
More common with infero-posterior infarction and may be due to ischaemia or rupture of
the papillary muscle. An early-to-mid systolic murmur is typically heard. Patients are
treated with vasodilator therapy but often require emergency surgical repair.
1182. A 61yo man was found with K+=7.5 and ECG with prolong QRS complex. What is
the best
possible tx option?
a. Dialysis
b. IV calcium gluconate
c. IV insulin and dextrose
d. Salbutamol nebulizer
e. Loop diuretics
b. IV calcium gluconate
calcium gluconate stabilize the membrane and prevent arrythmia. Give it first
In this question the potassium levels have increased and wide QRS means calcium
levels have decreased.
Increases Potassium levels cause Ventricular Fibrillation.
D.O.C is Calcium Gluconate.
Wide QRS means Calcium levels decreased and can cause Laryngospasm and Tetany.
D.O.C is calcium Gluconate.
Stabilisation of the cardiac membrane
intravenous calcium gluconate
Short-term shift in potassium from extracellular to intracellular fluid compartment
combined insulin/dextrose infusion
nebulised salbutamol
Removal of potassium from the body
calcium resonium (orally or enema)
loop diuretics
dialysis
1183. A 38yo man presents with acute infection of skin in the leg. Dx of sellutitis has
been made. What
meds should be prescribed?
a. Penicillin + Flucloxacillin
b. Metronidazole + erythromycin
c. Vancomycin + metronidazole
d. Ceftriaxone + terbinafine

e. Ceftriaxone + flucloxacillin
a. Penicillin + Flucloxacillin
Penicillin ( streptococcal)+ flucloxacillin ( anti staphylococcal )
Cellulitis is most commonly caused by staph and strep hence fluclox and penicillin
respectively. But sometimes with IV Drug users MRSA ( vanco and cefazolin)and in
burns or immunocompromised or neutropenic it might be pseudomonas( ceftazidime,
cefepime, or piperacillin tazobactam, carbapenems or aminoglycosides )
1184. A 72yo man presents to the ED with chest pain. The following ECG was taken.
What is the most likely dx?

a. Anterior MI
b. Inferior MI
c. Lateral MI
d. Posterior MI
e. NSTEMI
e. NSTEMI
1185. A 36yo woman has recently spent a lot of money on buying clothes. She goes out
almost every
night with her friends. She believes that she knows better than her friends, so she should
choose
the restaurant for eating out with her friends. She gave hx of having low mood at 12y.
What is
the dx?
a. Mania
b. Depression
c. Bipolar affective disorder
d. Borderline personality disorder
e. Dysthymia
c. Bipolar affective disorder
a mental condition marked by alternating periods of elation and depression.
1186. A homeless lady presents with cough and fever. She complains of night sweats
and weight loss.
CXR has been done and shows opacity. What is the next appropriate management?
a. AFB

b. Mantoux test
c. IFN gamma testing
d. Bronchoscopy
e. CT
a. AFB
Mantoux test can only tell you "the exposure which may be the past", can't confirm active
disease
INVESTIGATIONS:
1- CXR
2- Sputum for AFB/culture
3- BAL
1187. A 32yo woman presents with malaise fatigue and fever. She complains about
weight loss. Exam:
malar rash with sparing of nasolabial fold can be seen. What is the most appropriate
inv?
a. Anti ds DNA
b. Anti histone
c. Anti centromere
d. Anti Jo
e. Anti Scl70
a. Anti ds DNA
SLE . Butterfly Rash, middle-aged woman.
The American College of Rheumatology Classification system for SLE suggests that
a person may be classified as having lupus if four or more of the following 11
criteria are present (which do not have to occur at the same time but can be
cumulative over a number of years):
Malar rash.
Discoid lupus.
Photosensitivity.
Oral or nasopharyngeal ulcers.
Non-erosive arthritis involving two or more peripheral joints.
Pleuritis or pericarditis.
Renal involvement with persistent proteinuria or cellular casts.
Seizures or psychosis.
Haematological disorder: haemolytic anaemia or leukopenia or lymphopenia or
thrombocytopenia.
Immunological disorder: anti-DNA antibody or anti-Sm or antiphospholipid
antibodies.
A positive antinuclear antibody.

1188. A 75yo man presents with back pain. Inv: plasma cells are found. What is the
most probable dx?
a. Multiple myeloma
b. AS
c. Disc prolapse

d. Leukemia
e. Myelofibrosis
a. Multiple myeloma

Myeloma: features
Multiple myeloma is a neoplasm of the bone marrow plasma cells. The peak incidence is
patients aged 60-70 years.
Clinical features
bone disease: bone pain, osteoporosis + pathological fractures (typically
vertebral), osteolytic lesions
lethargy
infection
hypercalcaemia
renal failure
other features: amyloidosis e.g. Macroglossia, carpal tunnel syndrome;
neuropathy; hyperviscosity
Diagnosis is based on:
monoclonal proteins (usually IgG or IgA) in the serum and urine (Bence Jones
proteins)
increased plasma cells in the bone marrow
bone lesions on the skeletal survey
Hypercalcaemia in myeloma
1189. A 45yo woman presents with complaints of abdominal pain and blood in stool. She
brings the stool sample from home but has never been able to produce a sample at the
hospital. Her urine and blood tests are normal. Exam: multiple scars on the abdomen
consistent with laparoscopies
and appendectomy. She insists on getting further inv although no abnormalities are
found.
What is the most likely dx?
a. Malingering
b. Somatization
c. Hypochondriasis
d. Conversion disorder
e. Munchausen syndrome
Somatization disorder
multiple physical SYMPTOMS present for at least 2 years
patient refuses to accept reassurance or negative test results
Hypochondriac disorder
persistent belief in the presence of an underlying serious DISEASE, e.g. cancer
patient again refuses to accept reassurance or negative test results
Conversion disorder
typically involves loss of motor or sensory function

the patient doesn't consciously feign the symptoms (factitious disorder) or seek
material gain (malingering)
patients may be indifferent to their apparent disorder - la belle indifference although this has not been backed up by some studies

Dissociative disorder
dissociation is a process of 'separating off' certain memories from normal
consciousness
in contrast to conversion disorder involves psychiatric symptoms e.g. Amnesia,
fugue, stupor
dissociative identity disorder (DID) is the new term for multiple personality
disorder as is the most severe form of dissociative disorder
Munchausen's syndrome
also known as factitious disorder
the intentional production of physical or psychological symptoms
Malingering
fraudulent simulation or exaggeration of symptoms with the intention of financial
or other gain
1190. A 36yo woman contacts the police to notify them she was responsible for a recent
disastrous flood with loss of lives. What kind of delusions is she suffering from?
a. Persecutory
b. Poverty
c. Guilt
d. Nihilistic
e. Reference
c. Guilt
an emotion that occurs when a person feels that they have violated a moral standard.
Persecutory: This is one of the most common types of delusions, centering around a
person's fixed, false belief that others aim to obstruct, harm, or kill him/her.
Poverty: The person strongly believes that he is financially incapacitated
Nihilistic: the delusion that things (or everything, including the self) do not exist; a sense
that everything is unreal
Reference: A neutral event is believed to have a special and personal meaning. For
example, a person with schizophrenia might believe a billboard or a celebrity is sending
a message meant specifically for them.
1191. A 27yo man presents with symptoms characterized by alternating mood swings
a/w flight of
ideas, elation, over activity and disinhibition, or low mood with lack of energy and social
withdrawal. What is the most probable dx?
a. Bipolar affective disorder
b. Dysthymia
c. Mania
d. Hypomania
e. Cyclothymia

a. Bipolar affective disorder


Basic definition
1192. Healthy parents have 2 children, a child with CF and a healthy child. They want to
have another child. What are the chances of that child being a carrier?
a. 1:4
b. 1:2
c. 2:3
d. 1:8
e. 1:16
b. 1:2
cystic fibrosis is autosomal recessive, thus when you cross match the genetics
according to Mendelian inheritance, if they had 4 children 1 would be normal, two would
be carriers and 1 would have cystic fibrosis (since the parents have a child with cystic
fibrosis both parents are carriers). This however does not translate to immediate
application but only tells us the probability of having one of the 3 scenarios (normal,
carrier, and affected). So chance of having a normal child is 1:4, carrier is 2:4 (divided
translates to 1;2) and affected by disease 1:4)

1193. A 64yo man believes a female newscaster is communicating directly with him
when she turns a
page. What kind of delusions is he suffering from?
a. Persecutory
b. Control
c. Grandeur
d. Nihilistic
e. Reference
e. Reference
A neutral event is believed to have a special and personal meaning. For example, a
person with schizophrenia might believe a billboard or a celebrity is sending a message
meant specifically for them.
Delusion of control: This is a false belief that another person, group of people, or
external force controls one's general thoughts, feelings, impulses, or behavior.
Grandeur: fixed, false belief that one possesses superior qualities such as genius, fame,
omnipotence, or wealth.
1194. A 7yo girl with allergy became acutely unwell while visiting a friends house and
has been
brought immediately to the ED. She is fully conscious but has got stridor, wheeze and
erythematous rash. She is receiving oxygen. What is the single immediate
management?
a. Check airway patency and prepare intubation
b. Give 0.25ml in 1000U epinephrine IM
c. Give 10 mg chlorpheniramine IM
d. Give 50 ml hydrocortisone IM
e. Obtain secure IV access
b. Give 0.25ml in 1000U epinephrine IM
Adrenaline is by far the most important drug in anaphylaxis and should be given as soon
as possible. The recommended doses for adrenaline, hydrocortisone and
chlorphenamine are as follows:
Adrenaline

Hydrocortisone

Chlorphenamine

< 6 months

150 micrograms (0.15ml 1


in 1,000)

25 mg

250
micrograms/kg

6 months - 6 years

150 micrograms (0.15ml 1


in 1,000)

50 mg

2.5 mg

6-12 years

300 micrograms (0.3ml 1


in 1,000)

100 mg

5 mg

Adult and child >


12 years

500 micrograms (0.5ml 1


in 1,000)

200 mg

10 mg

Adrenaline can be repeated every 5 minutes if necessary. The best site for IM injection
is the anterolateral aspect of the middle third of the thigh.
1195. A terminally ill pt with metastatic carcinoma presents with dysphagia and difficulty
in
swallowing. What is the best possible tx?
a. Nystatin suspension
b. Amphotericin B IV
c. PO fluconazole
d. Cotrimazole
e. Analgesic
c. PO fluconazole
Oesophageal candidiasis is the most common cause of oesophagitis in patients with
HIV. It is generally seen in patients with a CD4 count of less than 100. Typical symptoms
include dysphagia and odynophagia. Fluconazole and itraconazole are first-line
treatments
1196. A couple attends their GP because of marital problems. The wife states that her
husband is
having affairs although she has no proof of this. The husband states that she even had
him
followed by a private detective and this is putting considerable strain on their marriage.
What is
the most likely dx?
a. Fregoli syndrome
b. Cotard syndrome
c. Mood disorder
d. Ekbom syndrome
e. Othello syndrome
e. Othello syndrome
Delusional jealousy (Othello's syndrome) - eg believing a partner is being unfaithful.
The Fregoli delusion, or the delusion of doubles, is a rare disorder in which a person
holds a delusional belief that different people are in fact a single person who changes
appearance or is in disguise.
The Cotard delusion (also Cotard's syndrome and walking corpse syndrome) is a rare
mental illness, in which the afflicted person holds the delusion that he or she is dead,
either figuratively or literally
Ekbom syndrome, also called delusional parasitosis, is a psychiatric disorder
characterized by the patient's conviction that he or she is infested with parasites.
1197. A 65yo lady who is on thiazide suffers from falls in the morning. What is the cause
for her
symptoms?
a. Orthostatic hypotension
b. TIA
c. Epilepsy

a. Orthostatic hypotension
Common adverse effects
dehydration
postural hypotension
hyponatraemia, hypokalaemia, hypercalcaemia
gout
impaired glucose tolerance
impotence
1198. A boy was admitted with partial thickness burn, what is your next step?
a. Escharectomy
b. Dressing
c. Burst blisters
d. Local antibiotics
e. Refer to burn unit
All complex injuries should be referred - particularly
Age under 5 years or over 60 years.
Site of injury: face, hands, perineum, any flexure (including neck or axilla) and
circumferential dermal burns or a full-thickness burn of the limb, torso or neck.
Inhalation injury.
Mechanism of injury:
o Chemical burns affecting over 5% total body surface area burned (over
1% for hydrofluoric acid burns).
o Exposure to ionising radiation.
o High-pressure steam injury.
o High-tension electrical injury.
Suspected non-accidental injury in a child.
Large affected area:
o Age under 16 years: over 5% total body surface area burned.
o Age 16 years or older: over 10% total body surface area burned.
Co-existing conditions - eg, serious medical conditions, pregnancy or associated
fractures, head injury or crush injuries.
1199. A 28yo man presents with a 2h hx of rapid palpitations. He feels a little light
headed but is
otherwise well. Exam: pulse=170bpm and regular, BP=100/68mmHg. He has had 2
similar
episodes in the past. What is the most likely rhythm disturbance?
a. SVT
b. VF
c. VT
d. V-ectopics
e. A-fib
a. SVT
Acute management
vagal manoeuvres: e.g. Valsalva manoeuvre
intravenous adenosine 6mg 12mg 12mg: contraindicated in asthmatics verapamil is a preferable option

electrical cardioversion

Prevention of episodes
beta-blockers
radio-frequency ablation

1200. A child has hypothyroidism. What feature is a/w it?


a. Microglossia
b. Prolonged neonatal jaundice
c. Undescended testis
d. Anal tag
e. Left soft palate
b. Prolonged neonatal jaundice

CONGENITAL HYPOTHYROIDISM
Symptoms

Feeding difficulties
Somnolence
Lethargy
Low frequency of crying
Constipation

Signs

Large fontanelles
Myxoedema - with coarse features and a large head and oedema of the genitalia
and extremities
Nasal obstruction
Macroglossia
Low temperature (often <35C) with cold and mottled skin on the extremities
Jaundice - prolongation of the physiological jaundice
Umbilical hernia
Hypotonia
Hoarse voice
Cardiomegaly
Bradycardia
Pericardial effusion - usually asymptomatic
Failure of fusion of distal femoral epiphyses
The growing child will have short stature, hypertelorism, depressed bridge of
nose, narrow palpebral fissures and swollen eyelids
Refractory anaemia

1201. A 2wk girl presents with E-coli which is confirmed by urine culture. What is the
most appropriate next inv?
a. US
b. IVU

c. CT kidney
d. BUE
e. MCUG
a) US
ultrasonography - even one case of UTI in children or males needs to be investigated, in
case of females recurrences require investigation.
Child <3 months - refer to paediatrician
3 months to 3 years - urgent microscopy and culture
>3 years - Urine sample, dipstick test and culture
First line investigation, best initial image showing renal size, hydronephrosis, perinephric
collection, bladder residual volume, prostate.
1202. A lady from Asia presented with lump in her neck. FNAC has been done and
revealed lesions with caseous material in the center surrounded by fibrosis. What is the
most probable dx?
a. Thyroid carcinoma
b. TB lymphadenitis
c. Lymphoma
d. Inf Mono
e. Mesothelioma
b. TB lymphadenitis
Submandibular triangle of neck
Palpable nodes initially tender, firm, discrete, later turn matted, suppurative with
discharging sinuses.
Caseous material confirms diagnosis
1203. A 32yo woman has undergone a biopsy for a breast lump. The report says: a well
circumscribed
lump with clear margins and separated from the surrounding fatty tissue. What is the
most
appropriate interpretation of this report?
a. Fibroadenosis
b. Ca Breast
c. Mammary abscess
d. Fibroadenoma
e. Fat necrosis
d. Fibroadenoma
Mobile (the breast mouse), regular margins smooth lump, multiple lumps

One third regress, one third stay the same. one third get bigger
conservative treatment, observe. If in doubt FNAC with/without excision
1204. A young boy presented with peri-oral blisters. Some of which are weeping and
others are
crusted. What is the single most appropriate dx?
a. Impetigo
b. Varicella zoster
c. Shingles
d. Scabies
e. Herpes simplex
a. Impetigo
Non-bullous - tiny pustules change into honey coloured crusted plaques <2cm
Mostly on exposed area like face extremities especially where bites, abrasions,
lacerations, scratches, burns or trauma have occurred.
Spreads rapidly
Some itching may be present
Bullous - thin roof, so rupture spontaneously
Occur on face, trunk, extremities, buttocks or perineal region
1205. A 39yo man comes with umbilicated papules on his face. His CD4 count is
measured to be 35. What is the single most appropriate option?
a. Mycobacterium avium
intercellular
b. CMV
c. Streptokinase
d. Toxoplasmosis
e. Pneumocystis jerovici
f. Moluscum contagiosum
f. Moluscum contagiosum
Viral skin infection (Opportunistic) in immunocompromised individuals
Firm, smooth, Umbilicated papules
Skin coloured, white, translucent or slightly yellow
Occurs in clusters, in children, found on trunk or extremities
In adults, found on lower abdomen, inner thighs or genital region
1206. A 45yo man is admitted to ED with excruciating pain in the right leg. Exam: limb is
pale and
dorsalis pedis and posterior tibial pulses are absent. Pulse=88 bpm, irregular and he has
a

pansystolic murmur at apex. What is the most probable dx?


a. Thromboangiitis Obliterans
b. Sciatica
c. DVT
d. Atherosclerosis
e. Embolus
e. Embolus
Acute limb ischemia is caused
6 Ps - pale, pulseless, painful, paralysed, paraesthetic, perishing with cold
Ischaemia, occurs from abrupt interruption of blood flow due to emboli. A cardiac origin
is the source of emboli in most of the cases associated with atrial fibrillation, as in this
case.
1207. An 18yo man has a smooth, tender swelling extending from the ear to the angle of
the jaw of sudden onset. Temp=38.5C. What is the single most likely dx?
a. Dental caries
b. Mumps
c. OE
d. OM
e. Temporomandibular joint pain
b. Mumps
Fever, myalgia , malaise painful parotid swelling which becomes bilateral in 70%
Spread - droplets/saliva
Incubation period - 14-21 days
Complications - rare but may lead to orchitis arthritis, meningitis
1208. A 6wk baby has a blue mark near coccyx since birth. His mother is worried. What
would you do?
a. Reassure
b. Coag profile
c. Karyotyping
d. Skeletal survey
e. CT
a. Reassure
The mark is a Mongolian spot which is benign, flat, congenital birthmark with wavy
borders, irregular shape, blue to blue/black in colour
Usually present on buttocks or back
Disappears by 3-5 years of age, almost always by puberty

It resembles a bruise and might cause concern, however, the determining factor is
presence since birth
1209. A man presents with inoperable carcinoma and back pain. His pain has been well
controlled with morphine but he develops vomiting. Morphine was stopped and he was
started on
metoclopramide and fentanyl patches. He then develops neck stiffness and fever. What
is the
cause of these symptoms?
a. Metoclopramide
b. Fentanyl
c. Morphine
d. Meningitis
e. Metastasis
e. Metastasis
Leptomeningeal meningitis is caused due to metastasis
Associated with spinal signs like neck pain, back pain, neck stiffness, weakness of limbs
1210. A 51yo man has become increasingly fatigued for the past 10m. PE: no abnormal
findings. Labs: Hgb=9.2, Hct=27.9%, MCV=132fl, plt=242, WBC=7.59. Which of the
following morphologic
findings is most likely to be present on examination of his peripheral blood smear?
a. Hypersegmented neutrophils
b. Nucleated RBC
c. Blasts
d. Hypochromic, microcytic RBC
e. Schistocytes
a. Hypersegmented neutrophils
Megaloblastic anaemia is caused by folate deficiency and vitamin B12 deficiency
causing decreased Hg and increased MCV. They have similar blood film and bone
marrow biopsy appearances.
Blood film - hypersegmented neutrophils and macrocytes
1211. A 9yo girl with weekly abdominal pain and occasional headaches but not a/w
vomiting or
diarrhea. She maintains a good appetite. Lab: normal. CBC, BUE, etc are normal. Exam:
no
abnormality as found and the abdomen was soft and non-tender. What would you do for
her
next?
a. US abdomen
b. CT thorax
c. LFT
d. Reassure
e. Analgesics

d. Reassure
Childhood periodic syndromes - cyclical vomiting, abdominal migraine
Age 5-10 years
Bilateral pain with abdominal cramps and vomiting, The child usually falls asleep within
an hour of pain onset. Sleeping relieves pain.
No medication required. Reassurance as disappears with age.
1212. A 54yo male pt DM with BMI=33 who has been treated using dietary control up till
now presents to his GP with a fasting blood sugar of 14mmol/l and creatinine=90mmol/l.
Urine shows
glycosuria. No other abnormalities are found. What is the best next step in
management?
a. Biguanide
b. Sulfonylurea
c. Insulin
d. Sugar free diet
e. ACEi
a. Biguanide
Biguanide (Metformin) are first line treatment for type 2 Diabetes Mellitus. They work by
increasing insulin sensitivity. Additionally, they help with weight loss. They do not cause
hypoglycaemia.
NICE guidelines
1.
2.
3.
4.
5.

Metformin
Metformin + Sulfonylureas
Add Thiazolidinedione or Insulin
Insulin + Metformin + Sulfonylureas
Increase Insulin dose as needed

1213. What are the side effects of thiazide diuretics?


a. Hypocalcemia
b. Hyponatremia
c. Hypernatremia
d. Hyperkalemia
b. Hyponatremia
Thiazide diuretics - Bendroflumethiazide, hydrochlorothiazide
Low dose to treat hypertension
Also used in combination with loop diuretics to treat heart failure (Metolazone)
In case of hypertension, if patient is intolerant to calcium channel blockers, thiazide
diuretics are prescribed.

Side effects include hyponatremia, hypokalemia, hypomagnesemia, hyperuricaemia,


metabolic alkalosis, hypotension, hypovolaemia
1214. A 46yo man who is a heavy drinker is brought to the ED in a drowsy state. He is
responding
vaguely to questions. Exam: nystagmus and hyperreflexia. MCV=103fl. What is the most
likely
cause for his cognitive impairment?
a. B1 deficiency
b. B12 deficiency
c. Folate deficiency
d. B6 deficiency
e. Alcohol withdrawal
a. B1 deficiency
Thiamine deficiency Wernickes Encephalopathy - ophthalmoplegia (Nystagmus, lateral rectus palsy)
Ataxia - wide based gait
Confusion
Caused by alcoholism, malnutrition, eating disorders, prolonged vomiting
Korsakoff Psychosis - decreased ability to acquire new memories, hallucinations
1215. A 23yo female presented with a swelling of her neck that moved upwards
on protrusion of tongue. What is the next appropriate inv?
a. FNAC
b. Punch biopsy
c. Core biopsy
d. MRI neck
e. Radioactive thyroid scan
e. Radioactive thyroid scan
Thyroglossal cyst- A fluctuant swelling in the midline of the neck that moves upwards on
protruding the tongue
Usually non- tender and mobile
May be tender if infected, and is associated with dysphagia, dysphonia, draining sinus, fever
or increasing neck mass
May lead to airway obstruction in some cases
Should be palpated on physical examination.
Thyroid Function Tests
If not palpable, USG and CT scan are first choices.
Radio active Thyroid scanning may be used to demonstrate any functioning ectopic thyroid
accompanying the cyst.

1216. A 34yo man from Asia presented with 5m hx of productive cough, night sweats
and weight loss. His CXR reveals some shadowing in the left upper zone. What is the
single most discriminating
inv?
a. AFB for sputum
b. CXR
c. CT
d. TFT
e. US abdomen
a. AFB for sputum
In case of pulmonary TB, chest x-ray is done first and if suggestive of TB, sputum
samples are taken. Three sputum samples are required, with at least one of them an
early morning sample, preferably before starting treatment or within the first week.
Send samples for microscopy with ZN stain and culture for acid fast bacilli.
If sputum can not be obtained, bronchoscopy and lavage should be used. Incase of
children, gastric washings can be used.
1217. A prv healthy 23yo presented a week hx of bloody diarrhea and abdominal pain
with cramps and fever. Exam: tenderness in lower abdomen. What is the most
appropriate dx?
a. Celiac disease
b. Colorectal polyps
c. UC
d. Laxative abuse
e. Gastroenteritis
e. Gastroenteritis
It is a combination of nausea, vomiting, diarrhoea and abdominal pain, usually of
infectious origin.
In the UK, the two most common causes of gastroenteritis in adults are the norovirus
and food poisoning (most often caused by salmonella or campylobacter bacteria).
Norovirus is a common cause of winter vomiting. Bloody diarrhoea is usually due to
bacterial infection.
Its important to assess the degree of dehydration Mild - lassitude, anorexia, nausea, postural, hypotension
Moderate - apathy, dizziness, muscle cramps, dry tongue, sunken eyes, reduced skin
elasticity, oliguria, SBP >90
Severe - profound apathy, confusion leading to coma, shock, SBP <90, Oliguria or ,
Anuria
1218. A 10yo boy presents with irritability, sudden onset of pain and discharge from the
right ear.

Which antibiotic would be the 1st line of tx?


a. Amoxicillin
b. Ciprofloxacin
c. Flucloxacillin
d. Ceftazidime
e. Benzylpenicillin
a. Amoxicillin
Acute Otitis Media is inflammation of middle ear presenting with pain, malaise, irritability,
fever, vomiting. Red and bulging ear drums may be present. Also, hearing loss may be
present.
Usually resolves spontaneously without specific treatment. Analgesia may be required
In cases with symptoms longer than 2-3 days, children <2 years with bilateral disease or
bulging drum, any child with Otorrhoea, antibiotics are required. In such cases, a five
day course of Amoxicillin is prescribed. In case the patient is allergic to penicillin,
erythromycin is given.
1219. A 26yo man strongly believes that every elderly man he meets is his father.
Although they look different, he is sure it is father wearing different disguises. What kind
of delusions is this man
suffering from?
a. Delusion of persecution
b. Erotomania
c. Delusion of grandeur
d. Delusion of doubles
e. Delusion of reference
d. Delusion of doubles
Alson known as Fregoli Delusion these are extremely rare delusions in which a person
believes different people are all just a single person that is capable of morphing his or
her appearance as a disguise. It has been speculated that these delusions are usually
associated with brain lesions or damage.
1220. A 26yo passed a 4mm stone in his urine. On US a 3mm stone is found in the renal
pelvis. What is the single most appropriate management?
a. ESWL
b. None
c. Open Surgery
d. Conservative
d. Conservative
Renal stones are usually associated with sudden severe pain in the loin, radiating to the
groin, often caused by a moving stone. However, most symptomatic renal stones are
small, ie <5mm in diameter and pass spontaneously. In some people, stones up to 1cm
pass spontaneously. Any stones >1cm usually require intervention. A stone that has not
passed within 1-2 months is unlikely to do spontaneously.

1221. A 35yo man has had acute pain and swelling below the mandible on the left side
for 2h. The
swelling occurred after eating a large meal. What is the single most likely dx?
a. Laryngocele
b. Ranula
c. Neck abscess
d. Parotid calculus
e. Submandibular calculus
e. Submandibular calculus
A stone formed in one of the salivary glands can block their opening causing saliva to
back up into the gland leading to pain and swelling. This pain is heightened just after
starting a meal. Eventually, infection of the gland may occur, even forming an abscess in
some cases. This is known as Sialolithiasis and most commonly affects the
submandibular gland.
1222. A 45yo man has had impaired vision and pain on eye movement in his left eye
over the last 5 days. He also notes loss of color vision in the same eye. In the left eye,
the visual acuity is up to
counting fingers. When the pupil is stimulated with light, it dilates. His fundus is normal.
What is
the single most appropriate clinical dx?
a. Acute dacryocystitis
b. Acute iritis
c. Papillitis
d. Retrobulbar neuritis
e. Scleritis
d. Retrobulbar neuritis
Optic neuritis or retrobulbar neuritis is inflammation of the optic nerve. It consists of the
classic triad 1. Reduced vision
2. Eye pain, particularly on movement
3. Impaired colour vision
Common causes are multiple sclerosis, giant cell arteritis.
Typical signs - decreased pupillary light reaction in affected eye, or Marcus Gunn Pupil
1223. A 56yo pt has been dx with MS. She presents with a positive Rombergs test. She
also has
weakness and loss of sensations in all her 4 limbs. Which site is most likely to be
affected?
a. Cerebral cortex
b. Cerebellum
c. Cervical spinal cord
d. Thoracic spinal cord
e. Brain stem

c. Cervical spinal cord


Rombergs Test is used to identify instability of either peripheral or central cause

The patient stands up straight with feet together (or at a distance for them to
be steady) with arms outstretched. Then ask them to shut their eyes.
If they are unable to maintain their balance with their eyes closed, the test is
positive (usually fall to the side of the lesion so stay close by to prevent them
falling).
A positive test suggests a problem with proprioception or vestibular function.
Romberg's test can also be positive in neuromuscular disorders and may not
be reliable in very elderly people.

Multiple sclerosis causes demyelination of neurons in brain and spinal cord leading to
neurological symptoms and loss of sensation, difficulty in movement, coordination and
balance. In this case, all limbs are affected, so the level of lesion will be higher, ie.
cervical spain.
1224. A 58yo man suddenly becomes shocked several days after suffering an acute ant
MI. His CXR shows a large globular-shaped heart and clear lung fields. What is the
single most likely
explanation for the abnormal inv?
a. Acute pericarditis
b. Cardiac tamponade
c. Atrial thrombus
d. Left ventricular aneurysm
e. Dressler syndrome
b. Cardiac tamponade
It is caused by accumulation of blood or any other fluid or gas in the pericardial, leading
to reduced ventricular filling and haemodynamic compromise.
It is an emergency
Often associated with pericarditis, can occur after acute myocardial infarction,
sometimes as Dresslers syndrome.
Dyspnoea, tachycardia and tachypnoea, cold, clammy extremities.
Distended neck veins, muffled heart sounds
Becks Triad - Jv distension, hypotension, diminished heart sounds
Pulsus paradoxus
1225. A 56yo alcoholic man who has increased the amount of alcohol he is using wants
to attend his daughters wedding that is in 2wks. He is now coming to you for help. How
would you help him?
a. Acamprosate
b. Refer to clinical psychologist

c. Refer to GP
d. Desipramine
e. Refer to community mental health support group
a. Acamprosate
It is used after successful withdrawal of alcohol to prevent relapse. Used in combination
with individual psychological intervention
Possible neuroprotective role in detoxification
It does not interact with alcohol
Instead, it works by reducing cravings
1226. An 80yo woman fell over at her nursing home. XR shows fx of radius with
<10degree of dorsal
angulation. What is the single most appropriate tx?
a. Below elbow full plaster of paris
b. Below elbow splint plaster of paris
c. Closed reduction of fx
d. Elasticated support bandage
e. Open reduction and internal fixation
Colles Fracture (C)
* The classical definition is a fracture through the distal metaphysis of the radius,
approximately 4 cm proximal to the articular surface. The term is now more loosely used
for any fracture of the distal radius, with or without involvement of the ulna, with dorsal
(backward) displacement of the fracture fragments.
* It is common in older people who fall and have osteoporosis. Osteoporosis should be
considered in anyone with a Colles' fracture (see 'Osteoporosis and wrist fractures',
below).
* It can also occur in younger people with normal bones.
- Investigation of choice is X-rays of the wrists.
- Management
- Uncomplicated: Closed reduction
- Complicated: Open reduction and internal fixation (ORIF)
1227. A 16yo girl who is normally fit and well attends her GP complaining of heavy and
painful periods.
She is requesting tx for these complaints. She denies being sexually active. Select the
most
appropriate management for her menorrhagia?
a. Antifibrinolytics (tranexamic acid)
b. COCP
c. Endometrial ablation
d. IUS progestrogens (mirena)
e. NSAIDS (mefenamic acid)
Heavy and Painful periods (A)
- First-line treatment for menorrhagia is Mirena coil, but the patient in question does not
need contraception and she is having painful periods. The second line is Mefenamic acid

or Tranexamic Acid. Third line is COCPs. Fourth line is Endometrial ablation or


Hysterectomy (If there is no desire to conceive). IUCD, according to patient.info actually
causes heavy or painful periods.
- Therefore the best choice in this case is Mefenamic acid (NSAIDs)
1228. A 67yo lady with an ulcer on the anal margin. Which is the single most appropriate
LN involved?
a. External iliac LN
b. Pre-aortic LN
c. Aortic LN
d. Inguinal LN
e. Iliac LN
Lymph Node Involvement (D)
- Superficial inguinal lymph nodes drains from Penis, Scrotum, Perineum, buttock, vulva
and abdominal wall below the umbilicus which then drains into the deep inguinal lymph
nodes.
- There inguinal nodes (D) is the correct choice.
1229. A branch of the dominant coronary artery that supplies the inferior portion of the
septum. What is the single most appropriate option?
a. Septal branches
b. Obtuse marginal branches
c. Circumflex artery
d. Left main stem, post descending artery
e. Diagonal branch
Blood supply of the heart (D)
The Posterior Descending Artery is an artery running in the posterior interventricular
sulcus to the apex of the heart where it meets with the anterior interventricular artery. It
supplies the posterior (inferior, see below) 1/3rd of the interventricular septum.
The anatomical position of the artery is not really posterior, but inferior. The terminology
posterior is based on the viewing of the heart from Valentine position, not by the hearts
actual position.
1230. A 55yo female presented with anemia and dysphagia. There is a feeling of
something stuck in
the throat. The esophagus cant be negotiated beyond the crico-pharynx. What is the
most
probable dx?
a. Foreign body
b. Plummer vinson syndrome
c. Pharyngeal carcinoma
d. Barrets esophagus
e. Esophageal carcinoma
Plummer-Vinson Syndrome (B)

This is a condition where iron deficiency is associated with a post-cricoid oesophageal


web.
The syndrome most often affects middle-aged women (this may be related to a
propensity for iron deficiency), although it can occasionally present in a child.[2] The web
(containing mucosa and submucosa) occurs at the anterior post-cricoid area of the
upper oesophagus.
The presentation is usually with painless, intermittent dysphagia. It tends to be with solid
foods but, if untreated, may progress to soft foods and even liquids
Examination: The features that appear on examination are those associated with iron
deficiency. There may be pallor and even tachycardia if anaemia is marked. There may
be koilonychia (spoon-shaped nails), angular cheilitis and glossitis.
- Investigation
* FBC will show a microcytic, hypochromic anaemia. Ferritin is low. * Barium swallow
may show the web. This may need to be enhanced with videofluoroscopy.
* Biopsy may be required if malignancy is suspected clinically.
* Treatment
- Iron Replacement
- Endoscopic Dialatation
1231. A pt is on cancer tx with dexamethasone. According to her biochemical results her
K+=normal and her Na+=low. What is the dx?
a. Addisons
b. Dexamethasone side effect
c. Dilutional hyponatremia
Dexamethasone Dilutional Hyponatremia (C)
Exogenous glucocorticoids exert a tonic suppression of ADH secretion in the body. This
causes retention of water which leads to a decrease in the serum osmolality. Therefore
the correct option is C Dilutional Hyponatremia.
1232. A diabetic has been prescribed a long acting hypoglycemic in the morning and
short acting in the evening. He takes a regular lunch, but has been having hypoglycemic
attacks at around 4pm each day. What is the most appropriate intervention?
a. Recommend a heavier lunch
b. Review morning drug
c. Review evening drug
d. Review both drug
e. Reassure
Diabetes Mellitus (B)
- Recommending a heavier lunch carries the risk of increasing the blood sugar.
Reviewing the evening drug has no use since the patient does not experience any
hypoglycemic attacks after the evening drug until after he takes the morning drug. This
automatically rules out the review of both drugs. This patient cannot be reassured as he
is having hypoglycemic attacks for which an intervention is necessary.
- Since, the patient is on a long-acting hypoglycemic agent, and experiencing
hypoglycemic attacks at around 4 pm, this means that the drug being used in the
morning must be reviewed for appropriate intervention and replacement.

1233. A male pt presented with blood and mucus in stool. He has also noticed weight
loss but has no
hx of altered bowel habits. What is the dx?
a. Carcinoma of cecum
b. Carcinoma of descending colon
c. Carcinoma of sigmoid colon
d. Carcinoma of rectum
Carcinoma of the Cecum (A)
- The symptoms of Cecum Cancer can be difficult to detect. The most common
symptoms of Colorectal Cancer include a feeling of fullness or pressure in the rectum,
rectal bleeding and a frequent urge to defecate (tenesmus). Inflammations in the cecum,
unlike the rectum or the sigmoid colon or the descending colon, will not make the patient
feel the urge to defecate or cause bowel habit changes/irregularities, because the stool
passing through the rectum is slushy and can easily bypass masses in this part of the
intestine.
1234. A 22yo man keeps having persistent and intrusive thoughts that he is a dirty thief.
No matter
what he tries these thoughts keep coming to him. Any attempt to avoid these thoughts
leads to
serious anxiety. What is the most likely dx?
a. Schizophrenia
b. OCD
c. PTSD
d. Mania
e. Psychotic depression
OCD (B)
Symptoms typically include recurring thoughts and repetitive actions in response to the
recurring thoughts. A common example is recurring thoughts about germs and dirt, with
a need to wash your hands repeatedly to "clean off the germs".
- Treatment
The usual treatment for OCD is:
- Cognitive behavioural therapy (CBT); or
- Medication, usually with an SSRI antidepressant medicine; or
- A combination of CBT plus an SSRI antidepressant medicine.
1235. A 45yo female comes to the ED while having a generalized tonic clonic seizure
and she has
having difficulty breathing and is cyanosed. What is the tx option for her?
a. Secure airways
b. IV diazepam
c. IV phenytoin
d. Oxygen mask
Management of Status Epilepticus (A)
- Step-wise approach

- Open and maintain the airway, lay in recovery position. Remove false teeth if poorly
fitting, insert oral nasal airway, intubate if necessary
- Oxygen, 100% + suction (as required)
- IV access and take blood for investigations
- Slow IV bolus phase: to stop seizures, e.g, Lorazepam, Diazepam
- Correct hypotension with fluids
- IV infusion phase: If seizures continue start phenytoin. Alternate: Diazepam infusion
- General anesthesia phase
1236. A 30yo man is becoming concerned about the safety of his family. He has been
checking the
locks of the door every hour during the night. He becomes very anxious if his wife tries to
stop
him. What is the most likely dx?
a. Paranoid delusion
b. PTSD
c. Social phobia
d. OCD
e. GAD
OCD (D)
Described in 1234
1237. A 6wk baby has been dx as HIV+ve. Which immunization plan will you opt for
him?
a. Dont give any vaccine
b. Give all vaccines except live attenuated vaccines
c. Give only BCG vaccine
d. Give all vaccines except BCG vaccine
Vaccination in Children with Immunodeficiency states (D)
- An asymptomatic child with HIV infection can be given any vaccine except BCG.
- BCG vaccine is contraindicated in symptomatic HIV-infected child as well along with a
few live attenuated vaccines.
- In this scenario the child has just been diagnosed with an HIV infection which most
probably means that he is asymptomatic. So the answer would be D, Give all vaccines
except BCG.
1238. A 36yo man has been dx with DI. What electrolyte picture is expected to be seen?
a. High serum Na, low serum osmolarity, high urine osmolarity
b. Low serum Na, low serum osmolarity, high urine osmolarity
c. Low serum Na, high serum osmolarity, high urine osmolarity
d. High serum Na, high serum osmolarity, low urine osmolarity
e. Normal Na, normal serum osmolarity, normal urine osmolarity
Diabetes Insipidus (D)
Diabetes insipidus is a condition in which your ability to control the balance of water
within your body does not work properly. The kidneys are not able to regulate the

amount of water that passes out in the urine. This means that the patient passes large
amounts of dilute urine polyuria.
Because of passing more urine, and therefore losing more fluid from the body, to try to
compensate for this, the patient becomes thirstier and wants to drink more polydipsia.
Patients with this condition become dehydrated easily. The levels of sodium and
potassium salts in the blood can also become unbalanced and too high.
- Points to look for:
- Dilute Urine -> Low urine osmolality
- Dehydration -> High serum sodium and hence high serum osmolality.
1239. The artery that supplies the ant right ventricular wall. What is the single most
appropriate
option?
a. Acute marginal branch
b. Left ant descending artery
c. Coronary sinus
d. Circumflex artery
e. Right coronary artery
Blood supply of the heart (A)
- Acute Marginal Branch of the Right coronary artery supplies the anterior right
ventricular wall.
1240. A 55yo male presents to the ED after an RTA with breathlessness, engorged neck
veins and a
dull percussion note on the right side of his chest. Exam: pulse=140 bpm, BP=80/50
mmHg. What
is the most likely dx?
a. Hemothorax
b. Hemopneumothorax
c. Tension pneumothorax
d. Simple pneumothorax
Hemopneumothorax (B)
Hemothorax is rarely a solitary finding in blunt trauma.
Dull percussion note, in particular, takes the diagnosis away from any kind of
pneumothorax.
Low BP in this scenario indicates shock secondary to heavy blood loss
The reason Hemopneumothorax is the correct option is the presence of engorged neck
veins. Looking at the stem of the question now, we can see that the patient has signs of
both Hemothorax and Pneumothorax. Therefore the correct choice is B.
1241. A 32yo woman presents with complaints of having low back pain. She is taking
analgesics for it. All inv are normal. What will you advise her?
a. Bed rest
b. Physiotherapy
c. Advice to be more active
d. Admit

Back Pain Management (C)


- For nonspecific back pain focus on education and self-management.
- Advise patients to continue with normal activities and be active.
- Manage pain with analgesics.
- Offer physiotherapy, acupuncture or an exercise programme.
- Address psychosocial issues which may predispose to developing chronic pain and
disability.
- Last option is surgery.
1242. A 32yo woman suffers an episode of severe occipital headache with vomiting and
LOC. She is
brought to the hosp where she is found to be conscious and completely alert. Exam:
normal
pulse and BP with no abnormal neurological sign. What is the next step in her
management?
a. Admission for observation
b. CT brain
c. MRI head
d. Reassurance and discharge home
e. XR skull
Severe headache with LOC (B)
- The next step in this case should be CT-Brain to look for or rule out any organic cause.
- Reassurance and discharging the patient is a risky option.
- X-ray Skull does not have any utility in this condition.
- MRI and Admission should follow CT-Brain if required.
1243. A 30yo woman is taking tx for asthma. She has a HR=130bpm and peak
expiratory flow rate=400.
What is the most appropriate management?
a. Atenolol
b. Digoxin
c. Review drugs
Asthma (C)
The patient has a heart rate that is more than a 110, which means that she is having a
severe attack of asthma. Her expiratory flow rate is also decreased. She is already on
medication which apparently is not adequate to alleviate her symptoms. Digoxin and
Atenolol have no role in the treatment of Asthma. The correct option is to review the
drugs she is taking.
- Treatment of severe or life-threatening asthma
- Salbutamol 5m nebulized with oxygen and give prednisolone 30 mg PO
- If PEF remains less than 75%, repeat Salbutamol
- Monitor O2 saturation, HR and RR.
1244. A pt presents with a mask face. He also has gait prbs. Which class of drug is
causing this?
a. Anti-depressant
b. Antipsychotic

c. Anti-HTN
Anti-Psychotics Side-effects (B)
- Mask face and gait problems suggest Parkinsonism which is caused by deficiency of
Dopamine and anti-psychotic medications are known to deplete Dopamine and cause
Parkinsonism.
- The other medications mentioned do not cause the symptoms such as in this patient.
1245. A 16yo boy came home from boarding school with a cough. His CXR showed
bilateral
consolidations. What is the most likely organism which would have caused his
symptoms?
a. Legionella pneumophila
b. Mycoplasma pneumonia
c. Mycobacterium TB
d. Pneumocystis jiroveci
e. Pseudomonas aeruginosa
Atypical Pneumonia (A)
- History of living in boarding
- Bilateral consolidations
- Dry Cough
- Crepitations on auscultation
- Fever, anorexia and lethargy
The above points point clearly towards Legionella (A)
- Mycoplasma does not cause bilateral consolidation
- Staphylococcus causes bilateral basal cavitation and presents with a productive cough.
- Streptococcus shows lobar consolidation and presents with a productive cough.
1246. After an MI, a man presents with pansystolic murmur which is radiating to the
axilla. What is the
dx?
a. Tricuspid regurgitation
b. Mitral regurgitation
c. Aortic stenosis
d. Mitral stenosis
Mitral Regurgitation (B)
- A Pansystolic Murmur at the apex that radiates to the axilla is characteristic of Mitral
Regurgitation.
- The murmur of tricuspid regurge is also Pansystolic (soft) but does not radiate to the
axilla.
- Aortic stenosis presents with an ejection systolic murmur, while Mitral Stenosis
presents with a mid-diastolic murmur at the apex.
- Therefore the correct option is B, Mitral Regurgitation.
1247. A 34yo laborer developed severe pain in his lower back after lifting a sack of sand.
He also

complains of shooting pain down his leg. The GP has prescribed him complete bed rest,
with
painkillers and also scheduled an MRI for him. What is the most likely dx?
a. Peripheral vascular disease
b. Intervertebral disc prolapse
c. Hairline fx of the spine
d. Sprain of the back muscles
e. Muscle injury
Sudden Severe Back Pain after Lifting Heavy Load (B)
- Sudden onset severe lower back pain accompanied by shooting pain down the leg is
characteristic of Intervertebral Disc Prolapse.
- There are no signs of other systems involved so this makes the diagnosis of peripheral
vascular disease very unlikely.
- Muscle injuries, sprain of back muscles, and hairline fracture of the spine cannot
explain the shooting pain down the leg.
1248. A young man returns to his hostel and gets headache and lethargy. Now presents
with fever. There are crepitations on the auscultation of lung. What is the most likely
organism which would have caused his symptoms?
a. Legionella pneumonia
b. Mycoplasma
c. Staphylococcus
d. Streptococcus
Atypical Pneumonia (A)
- History of living in boarding
- Bilateral consolidations
- Dry Cough
- Crepitations on auscultation
- Fever, anorexia and lethargy
The above points point clearly towards Legionella (A)
- Mycoplasma does not cause bilateral consolidation
- Staphylococcus causes bilateral basal cavitation and presents with a productive cough.
- Streptococcus shows lobar consolidation and presents with a productive cough.
1249. A pt is about to undergo surgery. Her Hgb=8.9g/dl and MCV=70. What is the best
option for her?
a. Inv and postpone the surgery
b. Transfuse and proceed with surgery
c. Transfuse and defer surgery
d. Continue with surgery
Pre-Operative Considerations (A)
- Patients must have a full blood count even if fully fit
- If the patient is about to undergo an emergency operation, proceed even if Hb is low.
- If Hb is less than 8 then stabilize before proceeding with the operation
- For elective operations, only proceed if Hb is greater than 10.
- If Hb is less than 10 then defer the operation and investigate (as in this case)

- If Hb is less than 8 then blood transfusion must be done


- Since this patient has an Hb of 8.9 (less than 10) we have to postpone the surgery and
investigate.
- Blood transfusion would be required if Hb had been less than 8.
- We cannot proceed with surgery since Hb is less than 10
1250. A 24yo male presents with discomfort in the groin area and scrotal swelling.
Exam: scrotal skin is normal. What would be the next best step?
a. Urgent US
b. Urgent surgery
c. OPD referral
d. Antibiotics
Groin Discomfort and Scrotal Swelling (C)
-Discomfort in the groin and scrotal swelling dont indicate an emergency. This rules out
urgent ultrasound and urgent surgery.
- Since the diagnosis is unclear at this point and there are no signs of infection,
antibiotics cannot be prescribed.
- The only logical approach would be an OPD referral (C).
1251. A 22yo girl unhappy about her weight with BMI=22. She likes to have her dinner in
an expensive
restaurant. She does excessive shopping. K+=3.3. What is the dx?
a. Anorexia nervosa
b. Bipolar
c. OCD
d. Bulimia
Answer: D. Bulimia. Patient likes to have dinner in expensive restaurant, normal BMI ( as
patient after eating go for self induced vomiting) and low potassium point towards the
diagnosis. Bulimia nervosa is a type of eating disorder characterised by episodes of binge
eating followed by intentional vomiting
EXCLUSION:
A. Anorexia Nervosa:
People with anorexia nervosa maintain a low body weight as a result of a preoccupation with
weight, construed as either a fear of fatness or a pursuit of thinness. In spite of this, they
believe they are fat and are terrified of becoming what is, in reality, a normal weight or
shape. A diagnosis of anorexia nervosa is based on low body weight, weight loss measures
(particularly extreme dieting), psychological features (usually including distorted body
image), along with physical and endocrine sequelae.
B. Bipolar Disorder:
Bipolar disorder is a chronic episodic illness associated with behavioural disturbances. It
used to be called manic depression. It is characterised by episodes of mania (or hypomania)
and depression. Either one can occur first and one may be more dominant than the other but
all cases of mania eventually develop depression.
Bulimia nervosa is an eating disorder characterised by repeated episodes of uncontrolled
overeating (binges) followed by compensatory weight loss behaviours.
Features include:

Excessive preoccupation with body weight and shape


Undue emphasis on weight in self-evaluation
Feeling of lack of control over eating
Compensatory weight control mechanisms which can be:
o Self-induced vomiting
o Fasting
o Intensive exercise
o Abuse of medication such as laxatives, diuretics, thyroxine or
amfetamines

Presentation
The history often dates back to adolescence. The core features include:

o
o
o
o

o
o
o

Regular binge eating. Loss of control of eating during binges.


Attempts to counteract the binges - eg, vomiting, using laxatives,
diuretics, dietary restriction and excessive exercise.
Preoccupation with weight, body shape, and body image.
Preoccupation with food and diet. This is often rigid or ritualistic, and
deviations from a planned eating programme cause distress. The
affected person therefore starts to avoid eating with others and
becomes isolated.
Mood disturbance and anxiety are common, as are low self-esteem,
and self-harm.
Severe comorbid conditions may be present - eg, depression and
substance abuse.
Periods may be irregular.

Investigations

These are usually normal apart from serum potassium, which is often low.
Renal function and electrolytes should be checked in view of frequent selfinduced vomiting.

Management:

referral for specialist care is appropriate in all cases

cognitive behaviour therapy (CBT) is currently considered first-line treatment


interpersonal psychotherapy is also used but takes much longer than CBT
pharmacological treatments have a limited role - a trial of high-dose fluoxetine is
currently licensed for bulimia but long-term data is lacking

1252. A 59yo pt has been dx with HTN. His BP has been >160/90 mmHg on 3 separate
occasions. His biochemical profile is as follows: Na+=145 mmol/l, K+=6.2mmol/l,

creatinine=112 umol/l, urea=5.7mmol/l. What is the most appropriate anti-HTN drug for
him?
a. Amlodipine
b. Bendroflumethiazide
c. Ramipril
d. Lorsartan
e. Propranolol
Answer: B.Bendroflumethiazide. the patient has hyperkalemia so as in order to correct
that thiazide should be given as thiazide cause hypokalemia. so it B bendroflumethiazide
although recent guidelines now recommend other thiazide like diuretics such as
Indapamide and chlortalidone but it's an old question so bendroflumethiazide is the
answer.
Thiazide Diuretics
Thiazide diuretics work by inhibiting sodium absorption at the beginning of the distal
convoluted tubule (DCT). Potassium is lost as a result of more sodium reaching the
collecting ducts. Thiazide diuretics have a role in the treatment of mild heart failure although
loop diuretics are better for reducing overload. The main use of bendroflumethiazide was in
the management of hypertension but recent NICE guidelines now recommend other thiazidelike diuretics such as indapamide and chlortalidone.
Common adverse effects
dehydration
postural hypotension
hyponatraemia, hypokalaemia, hypercalcaemia
gout
impaired glucose tolerance
impotence
Rare adverse effects
thrombocytopenia
agranulocytosis
photosensitivity rash
pancreatitis

1253. A 22yo girl had a fight with her boyfriend and then took 22 tabs of paracetamol.
She was
commenced on N-acetyl cysteine and she was medically fit to go home the following
day. Which
of the following does she require?
a. OPD referral to relationship counselor
b. OPD referral to psychiatrist
c. Inpatient referral to psychiatrist
d. Inpatient referral to psychologist
Answer: C. inpatient referral to psychiatrist. In this case the patient is suicidal so we can
not discharge the patient and patient should be admitted in psychiatry department.

1254. A 74yo man presents with sudden onset of with right sided weakness and slurred
speech. He also has loss of sensation over the right side of the body and visual field
defects. CT shows
ischemic stroke. What is the most appropriate management?
a. Alteplase
b. Streptokinase
c. Nimodipine
d. Aspirin
e. Labetalol
Answer: D. Aspirin. The CT scan shows ischemic stroke and time when patient
presented is not given if it would have less than 4.5hrs we would have given alteplase.
STROKE MANAGEMENT
The Royal College of Physicians (RCP) published guidelines on the diagnosis and
management of patients following a stroke in 2004. NICE also issued stroke guidelines in
2008, although they modified their guidance with respect to antiplatelet therapy in 2010.
Selected points relating to the management of acute stroke include:
blood glucose, hydration, oxygen saturation and temperature should be maintained
within normal limits
blood pressure should not be lowered in the acute phase unless there are
complications e.g. Hypertensive encephalopathy*
aspirin 300mg orally or rectally should be given as soon as possible if a
haemorrhagic stroke has been excluded
with regards to atrial fibrillation, the RCP state: 'anticoagulants should not be started
until brain imaging has excluded haemorrhage, and usually not until 14 days have
passed from the onset of an ischaemic stroke'
if the cholesterol is > 3.5 mmol/l patients should be commenced on a statin. Many
physicians will delay treatment until after at least 48 hours due to the risk of
haemorrhagic transformation
Thrombolysis
Thrombolysis should only be given if:

it is administered within 4.5 hours of onset of stroke symptoms (unless as part of a


clinical trial)
haemorrhage has been definitively excluded (i.e. Imaging has been performed)

Alteplase is currently recommended by NICE.


Secondary prevention
NICE also published a technology appraisal in 2010 on the use of clopidogrel and
dipyridamole
Recommendations from NICE include:

clopidogrel is now recommended by NICE ahead of combination use of aspirin plus


modified release (MR) dipyridamole in people who have had an ischaemic stroke
aspirin plus MR dipyridamole is now recommended after an ischaemic stroke only if
clopidogrel is contraindicated or not tolerated, but treatment is no longer limited to 2
years' duration

MR dipyridamole alone is recommended after an ischaemic stroke only if aspirin or


clopidogrel are contraindicated or not tolerated, again with no limit on duration of
treatment

With regards to carotid artery endarterectomy:

recommend if patient has suffered stroke or TIA in the carotid territory and are not
severely disabled
should only be considered if carotid stenosis > 70% according ECST** criteria or >
50% according to NASCET*** criteria

1255. The artery that runs along the left AV groove. What is the single most appropriate
option?
a. Left internal mammary artery
b. Left anterior descending artery
c. Circumflex artery
d. Left main stem (LMS) post descending artery
e. Diagonal branch
Answer: C. Circumflex Artery.
Left Anterior Descending artery descends into interventricular groove.
1256. A 26yo man presents with painless hematuria. He has no other complaints and on
examination no other abnormality is found. What is the most appropriate initial inv to get
to a dx?
a. Cystoscopy
b. Midstream urine for culture
c. Abdominal US
d. MRI spine
e. Coag screening
Answer:C.Abdominal US. As the patient is young so we can't go for cystoscopy. The
initial investigation in this patient should be abdominal US to exclude any pathology in
the genitourinary tract. PKD can also be suspected here so US should be done to
exclude that aswell first.
1257. A pt, 50yo smoker and heavy drinker, presents with complaints of racing heart. A
24h EKG
comes out normal. What is your next step in management?
a. Echo
b. Reassure
c. Stress test
Answer: B.Reassure. Everything is normal in this patient so reassure the patient. If there
would have been chest pain or past h/o chest pain then stress test could be a viable
option.
1258. A 36yo woman came with uterine bleeding. Vaginal US reveals uterine
thickness=12mm. what is
the most probable dx?
a. Cervical ca
b. Endometrial ca

c. Ovarian ca
d. Breast ca
e. Vaginal ca
Answer: B.Endometrial CA. The increased uterine thickness points towards the
diagnosis.
Endometrial cancer is classically seen in postmenopausal women but around 25% of cases
occur before the menopause. It usually carries a good prognosis due to early detection
The risk factors for endometrial cancer are as follows*:
obesity
nulliparity
early menarche
late menopause
unopposed oestrogen. The addition of a progesterone to oestrogen reduces this risk
(e.g. In HRT). The BNF states that the additional risk is eliminated if a progestogen is
given continuously
diabetes mellitus
tamoxifen
polycystic ovarian syndrome
Features
postmenopausal bleeding is the classic symptom
pre-menopausal women may have a change intermenstrual bleeding
pain and discharge are unusual features
Investigation
first-line investigation is trans-vaginal ultrasound - a normal endometrial thickness (<
4 mm) has a high negative predictive value
hysteroscopy with endometrial biopsy
Management
localised disease is treated with total abdominal hysterectomy with bilateral salpingooophorectomy. Patients with high-risk disease may have postoperative radiotherapy
progestogen therapy is sometimes used in frail elderly women not consider suitable
for surgery

1259. A 30yo woman has PID which was treated with metronidazole and cephalosporin.
It is getting worse. What is the next best inv?
a. Endocervical swab
b. US
c. Laparotomy
d. High vaginal swab
Answer: B. US. US is done to rule out tubo ovarian abscess.
Pelvic inflammatory disease (PID) is a term used to describe infection and inflammation of
the female pelvic organs including the uterus, fallopian tubes, ovaries and the surrounding
peritoneum. It is usually the result of ascending infection from the endocervix
Causative organisms

Chlamydia trachomatis - the most common cause


Neisseria gonorrhoeae
Mycoplasma genitalium
Mycoplasma hominis

Features
lower abdominal pain
fever
deep dyspareunia
dysuria and menstrual irregularities may occur
vaginal or cervical discharge
cervical excitation
Investigation
screen for Chlamydia and Gonorrhoea
Management
due to the difficulty in making an accurate diagnosis, and the potential complications
of untreated PID, consensus guidelines recommend having a low threshold for
treatment
oral ofloxacin + oral metronidazole or intramuscular ceftriaxone + oral doxycycline +
oral metronidazole
RCOG guidelines suggest that in mild cases of PID intrauterine contraceptive
devices may be left in. The more recent BASHH guidelines suggest that the evidence
is limited but that ' Removal of the IUD should be considered and may be associated
with better short term clinical outcomes'
Complications
infertility - the risk may be as high as 10-20% after a single episode
chronic pelvic pain
ectopic pregnancy
perihepatitis
tubo ovarian abscess
reiter's syndrome

1260. A pregnant woman had hit her chest 3wks ago. Now she is 24wks pregnant and
presents with left upper quadrant mass with dimpling. What is the most probable dx?
a. Breast ca
b. Carcinoma
c. Fibroadenoma
d. Fibroadenosis
e. Fatty necrosis of breast
Answer:E. Fatty necrosis of breast. H/o of trauma to the breast and lump with dimpling
point towards the diagnosis.
Fat necrosis
It tends to be large, fatty breasts in obese women that have this problem:

It usually follows trauma.


The lump is usually painless and the skin around it may look red, bruised or
dimpled.
Biopsy may be required, but if the diagnosis is confirmed, no further
management is indicated.

1261. A pregnant pt with Rh ve who hasnt been prv sensitized delivers her first baby
without any
prbs. What would be the latest time to administer anti-sensitization?
a. 6h PP
b. 24h PP
c. 48h PP
d. 72h PP
e. 5d PP
Answer: D.72h pp.
Anti-D immunoglobulin should be given as soon as possible (but always within 72 hours) in
the following situations:
delivery of a Rh +ve infant, whether live or stillborn
any termination of pregnancy
miscarriage if gestation is > 12 weeks
ectopic pregnancy
external cephalic version
antepartum haemorrhage
amniocentesis, chorionic villus sampling, fetal blood sampling
1262. A 30yo primigravida who is 30wks GA presents to the L&D with absent fetal
movements. She also complains of severe headache, heartburn and seeing floaters
before her eyes for the last
few days. Exam: BP=170/110 mmHg, urine protein=++++, rock hard uterus, no visible
signs of
fetal movements. Choose the single most likely dx?
a. Abruption of placenta 2nd pre-eclampsia
b. Antepartum hemorrhage
c. Placenta previa
d. Primary PPH
e. IUFD
f. Abruption of placenta due to trauma
Answer: A. Abruption of placenta secondary to preeclampsia. Maternal Hypertension is
the most important cause of placental abruption.

Rigid abdomen/ hard rock uterus here indicates peritoneal irritation due to
bleeding (concealed haemorrhage)
exclusion:
IUFD: We need a lot more to conclude IUFD and cannot be based on just rock
hard uterus and no visible signs of fetal movements (such as auscultation,
cardiotocography, real time ultrasonography etc)
ABRUPTION OF PLACENTA
Abruption is the premature separation of a normally placed placenta before delivery of the
fetus, with blood collecting between the placenta and the uterus. It is one of the two most
important causes of antepartum haemorrhage (the other being placenta praevia), accounting
for 30% of all cases of antepartum haemorrhage.

There are two main forms:

Concealed (20% of cases) - where haemorrhage is confined within the uterine


cavity and is the more severe form. The amount of blood lost is easily
underestimated.
Revealed (80%) - where blood drains through the cervix, usually with incomplete
placental detachment and fewer associated problems.

Risk factors
There are recognised factors that increase the risk - these include:

Previous abruption carries the highest risk of abruption in current pregnancy.


Multiple pregnancy: twice as common with a twin pregnancy than with a
singleton.
Trauma:
o Road traffic accident.
o Domestic violence.
o Iatrogenic - eg, external cephalic version.
Threatened miscarriage earlier in current pregnancy.
Pre-eclampsia and maternal hypertension (most imp risk factor accounting for
approx: 44% of cases)
Multiparity.
Previous caesarean section.
Non-vertex presentations.
Smoking.
Cocaine or amphetamine use during pregnancy.
Thrombophilia.
Intrauterine infections.
Polyhydramnios.

The clinical features of placental abruption depend on the size and site of the bleeding.
The grades of haemorrhage described are:

mild - in this case there is only a small area of placental separation and the blood loss
is usually less than 200 ml. There may be abdominal discomfort and the uterus may
be tender
moderate - up to a 1/3 of the placenta separates. There is more severe bleeding (200600 ml). The patient complains of abdominal pain. On examination the patient may
have tachycardia but does not have signs of hypovolemia. The uterus is tender. Fetal
heart sounds are present
severe - in this condition more than half of the placenta separates. The abdominal pain
is more severe. On examination the uterus is tender and rigid (hard) - it may be
impossible to feel the fetus. Fetal heart sounds are reduced or absent. The patient
may be in a state of hypovolaemic shock

Diagnosis
Abruption is a clinical diagnosis with no available sensitive or reliable diagnostic tests.

Management:
Guidance from the Royal College of Obstetricians and Gynaecologists for moderate or
severe placental abruption is to follow ABCD of resuscitation:

Assess Airway and Breathing: high-flow oxygen.


Evaluate Circulation:
Access fetus and Decide on Delivery

1263. A 38yo woman, 10d post partum, presents to her GP with a hx of passing blood
clots per vagina since yesterday. Exam: BP=90/40 mmHg, pulse=110 bpm, temp=38C,
uterus tender on palpation and fundus is 2 cm above umbilicus, blood clots +++. Choose
the single most likely dx?
a. Abruption of placenta 2nd preeclampsia
b. Concealed hemorrhage
c. Primary PPH
d. Secondary PPH
e. Retained placenta
f. Scabies
Answer:D. Secondary PPH. The 10day post partum, signs of shock and blood clots all
point towards the diagnosis of secondary pph.

Post-partum haemorrhage
Post-partum haemorrhage (PPH) is defined as blood loss of > 500mls and may be primary or
secondary
Primary PPH
occurs within 24 hours
affects around 5-7% of deliveries
most common cause of PPH is uterine atony (90% of cases). Other causes include
genital trauma and clotting factors
Risk factors for primary PPH include*:
previous PPH
prolonged labour
pre-eclampsia
increased maternal age
polyhydramnios
emergency Caesarean section
placenta praevia
macrosomia
ritodrine (a beta-2 adrenergic receptor agonist used for tocolysis)
Management
ABC
IV syntocinon (oxytocin) 10 units or IV ergometrine 500 micrograms
IM carboprost

other options include: B-Lynch suture, ligation of the uterine arteries or internal iliac
arteries
if severe, uncontrolled haemorrhage then a hysterectomy is sometimes performed as
a life-saving procedure

Secondary PPH
occurs between 24 hours - 12 weeks**
due to retained placental tissue or endometritis
*the effect of parity on the risk of PPH is complicated. It was previously thought multiparity
was a risk factor but more modern studies suggest nulliparity is actually a risk factor

1264. A 22yo lady who is in her last trimester of pregnancy comes with hx of exposure to
a child dx with chicken pox 1d ago. She was investigated and was +ve for varicella
antibody. What is the single most appropriate management?
a. Give varicella I/g
b. Quarantine
c. Give varicella vaccination
d. Oral acyclovir
e. Reassure,
Answer: E. Reassure. Lady is +ve for varicella antibody so no need to give varicella
zoster Igs, just reassure the patient.

Chickenpox exposure in pregnancy


Chickenpox is caused by primary infection with varicella zoster virus. Shingles is reactivation
of dormant virus in dorsal root ganglion. In pregnancy there is a risk to both the mother and
also the fetus, a syndrome now termed fetal varicella syndrome
Risks to the mother
5 times greater risk of pneumonitis
Fetal varicella syndrome (FVS)
risk of FVS following maternal varicella exposure is around 1% if occurs before 20
weeks gestation
studies have shown a very small number of cases occurring between 20-28 weeks
gestation and none following 28 weeks
features of FVS include skin scarring, eye defects (microphthalmia), limb hypoplasia,
microcephaly and learning disabilities
Other risks to the fetus
shingles in infancy: 1-2% risk if maternal exposure in the second or third trimester
severe neonatal varicella: if mother develops rash between 5 days before and 2 days
after birth there is a risk of neonatal varicella, which may be fatal to the newborn child
in around 20% of cases

Management of chickenpox exposure


if there is any doubt about the mother previously having chickenpox maternal blood
should be urgently checked for varicella antibodies
if the pregnant women is not immune to varicella she should be given varicella zoster
immunoglobulin (VZIG) as soon as possible. RCOG and Greenbook guidelines
suggest VZIG is effective up to 10 days post exposure
consensus guidelines suggest oral aciclovir should be given if pregnant women with
chickenpox present within 24 hours of onset of the rash

1265. A 22yo woman who is 20wk pregnant came with pain and bleeding per vagina.
Exam: os is not open. What is the single most likely dx?
a. Threatened abortion
b. Missed abortion
c. APH
d. Miscarriage
e. Inevitable abortion
Answer. A. Threatened Abortion. Pain and bleeding per vagina and os closed all point
towards the diagnosis.
MISCARRIAGE
Miscarriage is defined as the loss of a pregnancy before 24 weeks of gestation.
Classification of miscarriage is as follows:
Threatened miscarriage: mild symptoms of bleeding. Usually little or no pain.
The cervical os is closed.
Inevitable miscarriage: usually presents with heavy bleeding with clots and pain.
The cervical os is open. The pregnancy will not continue and will proceed to
incomplete or complete miscarriage.
Incomplete miscarriage: this occurs when the products of conception are
partially expelled. Many incomplete miscarriages can be unrecognised missed
miscarriages.
Missed miscarriage: the fetus is dead but retained. The uterus is small for dates.
A pregnancy test can remain positive for several days. It presents with a history
of threatened miscarriage and persistent, dirty brown discharge. Early
pregnancy symptoms may have decreased or gone.
Habitual or recurrent miscarriage: three or more consecutive miscarriages.

1266. A 32yo lady G1, 28wks GA came to her ANC with a concern about pain relief
during labour. She has no medical illnesses and her pregnancy so far has been
uncomplicated. She wishes to feel her baby being born but at the same time she wants
something to work throughout her labour. What method of pain relief best matches this
ladys request?
a. C-section
b. Pudendal block
c. Entonox
d. TENS
e. Pethidine
C. Entonox.

Pain Relief In Labor:


1. Transcutaneous electrical nerve stimulation (TENS)
Randomised controlled trials provide no compelling evidence for TENS having any
analgesic effect during labour. so it is not recommended by NICE.
2. Acupuncture and hypnosis may be beneficial for the management of pain during
labour;
3. Water/birthing pool: Immersion in water during labour is claimed to increase
maternal relaxation and reduce analgesic requirements. It is supported by the Royal
College of Obstetricians and Gynaecologists (RCOG) for healthy women with
uncomplicated pregnancies.
4. Nitrous oxide and oxygen (Entonox)
This is a 50:50 mixture inhaled during painful contractions during the first and second
stages of labour. It is often used as a supplement to pethidine.

The main advantages are that it is under the patient's control, it takes effect
within seconds and wears off quickly with no side-effects.
Inhaled analgesia appears to be effective in reducing pain intensity and in
giving pain relief in labour

5. Intramuscular opiate: Parenteral opioids provide some relief from pain in labour but
are associated with adverse effects - eg, maternal nausea, vomiting and drowsiness.
6. Epidural analgesia: Epidural analgesia is a central nerve block technique achieved
by injection of a local anaesthetic close to the nerves that transmit pain. It is widely used
as a form of pain relief in labour. Advantages: It is the most effective way of relieving
pain in labour - providing complete relief in 95% of cases. It also has the benefit of
avoiding need for greater analgesia/general anaesthetic if forceps, vacuum extraction or
caesarean section are required. It is not associated with increase in symptoms related to
perineal trauma and pelvic floor muscle weakness.

7. Ambulatory epidural: This is a low-dose epidural that relieves pain, but allows
women to walk about during labour.
8. Local analgesia: This is used for women who have not had an epidural but require
forceps or vacuum extraction delivery. It is also used for repair of episiotomy or perineal
tear.
Pudendal nerve block: using lidocaine behind each ischial spine of the pelvis via the
vagina.
1267. A primipara at full term in labor has passed show and the cervix is 3cm dilated.
What is the single most appropriate management for her labor?
a. Repeat vaginal examination in 4h
b. CTG
c. IV syntocin drip

d. Repeat vaginal examination in 2h


e. Induction of labour
It is the first stage of labor since the cervix is only 3cm dilated. First stage of labour ends
when the cervix is 10cm dilated.
The first stage of labour
Latent phase (not necessarily continuous):
there are painful contractions, the cervix initially effaces (becomes shorter and
softer) then dilates to 4cm.
Established phase:
contractions with dilatation from 4 cm. A satisfactory rate of dilatation from 4 cm is
0.5cm/h.
The 1st stage generally takes 818h in a primip, and 512h in a multip.
During the first stage check maternal BP, and T 4-hourly, pulse hourly;
assess the contractions every 30min, their strength and their frequency (ideally 34 per
10min, lasting up to 1 min).
Offer vaginal examination e.g every 4h to assess the degree of cervical dilatation, the
position and the station of the head.
Auscultate fetal heart rate (if not continuously monitored), by Pinard or Doppler every
15min, listening for 1min after a contraction.

1268. A 36yo pregnant woman comes for evaluation with her husband. Her husband has
been
complaining of morning sickness, easy fatigability and even intermittent abdominal pain.
What
is the husband suffering from?
a. Ganser syndrome
b. Couvade syndrome
c. Pseudo-psychosis
d. Stockholm syndrome
e. Paris syndrome

Ganser syndrome is a type of factitious disorder, a mental illness in which a


person deliberately and consciously acts as if he or she has a physical or mental
illness when he or she is not really sick.
Couvade syndrome, also called sympathetic pregnancy, is a proposed condition
in which a partner experiences some of the same symptoms and behavior of an
expectant mother. These most often include minor weight gain, altered hormone
levels, morning nausea, and disturbed sleep patterns.
Stockholm syndrome, or capture-bonding, is a psychological phenomenon in
which hostages express empathy and sympathy and have positive feelings
toward their captors, sometimes to the point of defending and identifying with the
captors.

Pseudo Psychosis: As the name itself says, pseudo (psychosis), which means false, is not
a form of psychosis, but instead, pseudo psychosis is when someone is convinced that they
are suffering from psychosis when they are not. This doesnt mean that the person is
pretending or faking the symptoms of psychosis such as hallucinations, hearing voices or
other forms of being completely detached from reality.
Paris Syndrome: is a transient psychological disorder exhibited by some individuals visiting
or vacationing in Paris or elsewhere in Western Europe. It is characterized by a number of
psychiatric symptoms such as acute delusional states, hallucinations, feelings of persecution
(perceptions of being a victim of prejudice, aggression, or hostility from others), derealization,
depersonalization, anxiety, and also psychosomatic manifestations such as dizziness,
tachycardia, sweating, and others

1269. A woman comes to the ED complaining of pain in the right side of the abdomen,
she has 7wks amenorrhea. Her pregnancy test is +ve and US scan shows an empty
uterus. What is the next step?
a. Laparoscopy
b. HCG measurements
c. US
d. Laparotomy
e. Culdo-centhesis
Answer is B.
This is a case of ectopic pregnancy.
Always think of an ectopic in a sexually active woman with abdominal pain; bleeding;
fainting; or diarrhoea and vomiting. There is generally ~8 weeks amenorrhoea but an
ectopic may present before a period is missed. An early sign is often dark blood loss
(prune juice, as the decidua is lost from the uterus) or fresh.
Diagnosis: Early diagnosis is vital. Dipstix testing for HCG (human chorionic
gonadotrophin)
is sensitive to values of 25IU/L. do ultrasound. If HCG >6000IU/L and an intrauterine
gestational sac is not seen, ectopic pregnancy is very likely, as is the case if HCG 1000
1500IU/L and no sac is seen on transvaginal ultrasound.
1270. A 23yo woman who has had several recent partners has experienced post-coital
bleeding on gentle contact. What is the single most likely cause of her vaginal
discharge?
a. Cervical ca
b. Cervical ectropion
c. CIN
d. Chlamydial cervicitis
e. Gonococcal cervicitis
Answer is D

Causes of postcoital bleeding

Infection.
Cervical ectropion - especially in those women taking the combined oral
contraceptive pill (COCP).

Cervical or endometrial polyps.


Vaginal cancer.
Cervical cancer - usually apparent on speculum examination.
Trauma.

In this case the history of several recent partners points towards a sexually transmitted
disease so it is chlamydial cervicitis as chlamydia is transmitted sexually.
Chlamydial cervicitis:

Risk factors
Age <25 (the highest prevalence in women occurs between ages 16-19 years
and in men between ages 20-24 years).
Two or more sexual partners in the preceding year.
A recent change in sexual partner.
Non-barrier contraception.
Infection with another STI.
Poor socio-economic status.
Genetic predisposition
Symptoms: Vaginal discharge.

Dysuria (always consider chlamydia as a cause of sterile pyuria).


Vague lower abdominal pain.
Fever.
Intermenstrual or postcoital bleeding.
Dyspareunia.

A friable, inflamed cervix, sometimes with a follicular or 'cobblestone'


appearance, with contact bleeding.
Mucopurulent endocervical discharge.
Abdominal tenderness.
Pelvic adnexal tenderness on bimanual palpation.
Cervical excitation.

Signs:

Treatment: Doxycycline for 7 days or azithromycin single dose.

1271. A 68yo woman presents with post-coital bleeding following her first episode of
sexual
intercourse in 10yrs. What is the single most likely cause that has led to post-coital
bleeding?
a. Endometrial ca
b. Atrophic vaginitis
c. Endometrial polyp
d. Cervical ca

e. Cervical ectropion
Answer is Atrophic vaginitis.
In a case of post menopausal bleeding always first rule out endometrial
Carcinoma. Here since most likely cause is asked it is atrophic vaginitis.

Aetiology

Vaginal atrophy. The most common cause of PMB.


Use of HRT.
Endometrial hyperplasia; simple, complex, and atypical.
Endometrial cancer. The probability of a woman presenting with PMB having
endometrial cancer is 10%. However, 90% of women with endometrial cancer
present with PMB.[2]
Endometrial polyps or cervical polyps.
Cervical cancer; remember to check if the cervical smear is up-to-date.
Uterine sarcoma (rare).
Ovarian cancer, especially oestrogen-secreting (theca cell) ovarian tumours.
Vaginal cancer (very uncommon).
Vulval cancer may bleed, but the lesion should be obvious.
Non-gynaecological causes including trauma or a bleeding disorder.

1272. A 28yo woman 8wks GA had PID treated prvly and now comes with vaginal
bleeding, rigid
abdomen, BP=80/50 mmHg, pulse=140 bpm. What is the most probable dx?
a. Threatened abortion
b. Miscarriage
c. Missed abortion
d. Tubal pregnancy
e. Inevitable abortion
MISCARRIAGE
Miscarriage is defined as the loss of a pregnancy before 24 weeks of gestation.
Classification of miscarriage is as follows:
Threatened miscarriage: mild symptoms of bleeding. Usually little or no pain.
The cervical os is closed.
Inevitable miscarriage: usually presents with heavy bleeding with clots and pain.
The cervical os is open. The pregnancy will not continue and will proceed to
incomplete or complete miscarriage.
Incomplete miscarriage: this occurs when the products of conception are
partially expelled. Many incomplete miscarriages can be unrecognised missed
miscarriages.
Missed miscarriage: the fetus is dead but retained. The uterus is small for dates.
A pregnancy test can remain positive for several days. It presents with a history
of threatened miscarriage and persistent, dirty brown discharge. Early
pregnancy symptoms may have decreased or gone.
Habitual or recurrent miscarriage: three or more consecutive miscarriages.

1273. A 34yo primigravida who is 16wk GA comes for routine antenatal check up. Her
BP=160/100mmHg. She has a hx of repeated childhood UTI. What is the most likely
cause of her
high BP?
a. Essential HTN
b. Chronic pyelonephritis
c. Acute pyelonephritis
d. Pre-eclampsia
e. Chronic UTI
Recurrent episodes of UTI point towards chronic pyelonephritis.
Pre eclampsia has HTN with proteinuria/oedema after 20 weeks of gestation.
Risk Factors:
any structural renal tract anomalies, obstruction or calculi
Children with vesicoureteral reflux
Intrarenal reflux in neonates
Genetic predisposition
Any factors predisposing to recurrent urinary infection - eg, neurogenic bladder
Presentation:
Fever
Malaise
Loin pain
Nausea
Vomiting
Dysuria
Hypertension
Failure to thrive

1274. A 24yo woman has had lower abdominal pain for 12h. She is otherwise well. She
is at 10wks GA in a planned pregnancy. What is the single most appropriate test to inv
the cause of acute abdomen in this lady?
a. Abdominal US
b. Anti-phospholipid screen
c. CBC
d. Transvaginal US
e. Laparoscopy
The 2 main differentials of lower abdominal pain in the first part of pregnancy are ectopic
pregnancy and miscarriage. Both can have vaginal bleeding to but in miscarriage vaginal
bleeding is more pronounced and mostly occurs before abdominal pain so we should
first go for Transvaginal US to rule out ectopic pregnancy first as The most accurate
method to detect a tubal pregnancy is transvaginal ultrasound.
1275. A pt is at term and in labor, the membranes have ruptured, the liquor contains
meconium but the CTG is normal. The cervix is 3cm dilated. What is the single most
appropriate action?
a. BP monitoring
b. CTG

c. C-section
d. Fetal scalp blood sample
e. Internal rotation
d. Fetal scalp blood sample
NICE suggests continuous CTG monitoring, if its in the options it would have been the
best choice. FBS recommended only when CTG is abnormal.
one statement: Since ctg is normal, we need to do fetal scalp blood sampling to rule in/out fetal
distress.
These recommendations are from the National Institute for Health and Care Excellence (NICE), 2014

Intrapartum

If significant meconium staining is noted in labour, there should be continuous


electronic foetal monitoring.
This is defined as dark green or black amniotic fluid that is thick or tenacious, or
any amniotic fluid that contains lumps of meconium.
Transfer mother to obstetric-led care, if it is safe to do so and delivery is not
imminent.
If there are signs of fetal distress, a fetal blood sample should be obtained. If pH is
<7.21, there should be emergency delivery.
Ensure that the advanced resuscitation unit and appropriately trained staff are
available.
There should be no suction prior to delivery.

1276. A pt is at term and labor. The head has been delivered and you suspect shoulder
dystocia. What is the single most appropriate action?
a. C-section
b. Episiotomy
c. External rotation
d. Fetal scalp blood sample
e. Instrumental delivery
b. Episiotomy
Head is already delivered none of the other options except episiotomy seems
appropriate
Royal College of Obstetricians and Gynaecologists (RCOG) guidelines for shoulder
dystocia.
Get help. In addition to a senior obstetrician, an anaesthetist and paediatrician
should be called.
Stop the mother pushing. This may make impaction of the shoulders worse.
McRoberts' manoeuvre - the patient hyperflexes her hips so they are against her
abdomen. Mothers in labour may not have enough energy to do this by
themselves and may need the assistance of others in the room - which is usually
the case. Posterolateral pressure is applied suprapubically with traction on the
fetal head. This is the most effective procedure and should be performed first
(success rates are up to 90%).
If this fails, an episiotomy may be needed to facilitate the obstetrician trying
second-line manoeuvres - but the need for a caesarean section should be
considered.
[1]

Rubin's manoeuvre - press on the posterior fetal shoulder, thereby creating more
space to allow the anterior shoulder to be delivered.
Woods' screw manoeuvre - turning the anterior shoulder to the posterior position.
If these fail then delivery of the posterior shoulder may help.
However, at all times the need for a caesarean section should be considered and
should not be delayed.
NB: fundal pressure should NOT be applied.
1277. A 29yo female at 28wks GA presents to you with complains of hard stools and
constipation for last 2wks. CTG shows fetal tachycardia. What is the single most
appropriate tx?
a. Oral laxatives
b. Fiber diet
c. Phosphate enema
d. Lactulose
e. Reassure
b. Fiber diet
fiber diet free of any side-effects and safe in pregnancy
1278. A 16yo girl presents with heavy bleeding. What is the most appropriate initial inv?
a. Endometrial sampling
b. Transvaginal US
c. Hysteroscopy
d. Pelvic US
e. Exam under anesthesia
d. Pelvic US
1st initial investigation is the less invasive one always.
to confirm /exclude a pregnancy /mass
1279. A woman who is 7wks pregnant presents with excessive and severe vomiting and
put on IV
fluids and anti-emetic (ondansteron). She is complaining of severe headache and cant
take oral
fluids. What is the most appropriate management?
a. Termination of pregnancy
b. TPN
c. Feeds via NGT
d. P6 acupressure
e. IV hydrocortisone
e. IV hydrocortisone
antiemetics .. if they do not work .. steroids
tpn very rarely used
B and C for long term is not viable. D has little benefit in terms of allopathy. A is too
extreme at this early stage and it is only done when all methods have failed and there is
risk to the mother. E will alleviate the symptoms and improve likelihood of a viable
pregnancy.
HYPEREMESIS GRAVIDARUM

Vomiting that begins after 12 weeks of gestation is unlikely to be caused by hyperemesis


gravidarum

Management

[3]

Advice, including dietary advice, and support


Fluid and electrolyte replacement
Nutritional support (enteral or parenteral) may be required.
Vitamin supplements
Thromboprophylaxis
Anti-emetic drugs
Drug treatment options include cyclizine, metoclopramide, prochlorperazine,
promethazine, chlorpromazine, domperidone and ondansetron (selective 5hydroxytryptamine receptor antagonist), or combinations of these agents.
There is no evidence that any one anti-emetic is better than another.
Corticosteroids: may be used for intractable (failure to respond to conventional
treatment) cases of severe hyperemesis gravidarum in secondary care.

1280. A young lady with primary amenorrhea has normal LH, FSH, estradiol and
prolactin. Choose the single most likely dx?
a. PCOS
b. POF
c. Absent uterus
d. Absent ovaries
e. Turners syndrome
c. Absent uterus
in others hormones will be deranged.
PCOS raised LH:FSH ratio, Prolactin may be normal or mildly elevated. Testosterone
may be normal or mildly elevated.
POF raised FSH, LH levels
Turners syndrome LH and FSH may be elevated
Mayer-Rokitansky-Kster-Hauser (MRKH) syndrome is a disorder that occurs in females
and mainly affects the reproductive system. This condition causes the vagina and uterus
to be underdeveloped or absent.
1281. An obese lady presents with primary amenorrhea. She has high LH, normal FSH
and slightly high
prolactin levels. Choose the single most likely dx?
a. PCOS
b. POF
c. Hypothyroidism
d. Pregnancy
e. Primary obesity
a. PCOS
Features
subfertility and infertility
menstrual disturbances: oligomenorrhea and amenorrhoea

hirsutism, acne (due to hyperandrogenism)


obesity
acanthosis nigricans (due to insulin resistance)

Investigations
pelvic ultrasound: multiple cysts on the ovaries
FSH, LH, prolactin, TSH, and testosterone are useful investigations: raised
LH:FSH ratio is a 'classical' feature but is no longer thought to be useful in
diagnosis. Prolactin may be normal or mildly elevated. Testosterone may be
normal or mildly elevated - however, if markedly raised consider other causes
check for impaired glucose tolerance
1282. A 38yo lady presents with amenorrhea has very high LH and FSH levels, normal
prolactin and low estradiol. Choose the single most likely dx?
a. PCOS
b. POF
c. Hypothyroidism
d. Pregnancy
e. Menopause
b. POF
Premature ovarian failure is defined as the onset of menopausal symptoms and elevated
gonadotrophin levels before the age of 40 years. It occurs in around 1 in 100 women.
Causes
idiopathic - the most common cause
chemotherapy
autoimmune
radiation
Features are similar to those of the normal climacteric but the actual presenting problem
may differ
climacteric symptoms: hot flushes, night sweats
infertility
secondary amenorrhoea
raised FSH, LH levels

1283. A 77yo publican was admitted for an appendectomy. Post-op he becomes


confused, agitated and starts to pick at things. He is then given an IV drug which settles
this confusion. Which of the following drugs was given for his confusion?
a. Diazepam
b. Chlordiazepoxide
c. Thiamine
d. Vit B

b. Chlordiazepoxide
For alcohol withdrawal, chlordiazepoxide is 1st line. Diazepam is 2nd line.
Benzodiazepines are the recommended drugs for detoxification. They have a slower
onset of action and therefore are less likely to lead to abuse. A reducing dose of
chlordiazepoxide over 5-7 days is commonly used.
Diazepam is an alternative.

Symptoms typically present about eight hours after a significant fall in blood
alcohol levels. They peak on day 2 and, by day 4 or 5, the symptoms have
usually improved significantly.
Minor withdrawal symptoms (can appear 6-12 hours after alcohol has stopped)
o Insomnia and fatigue.
o Tremor.
o Mild anxiety/feeling nervous.
o Mild restlessness/agitation.
o Nausea and vomiting.
o Headache.
o Excessive sweating.
o Palpitations.
o Anorexia.
o Depression.
o Craving for alcohol.
Alcoholic hallucinosis (can appear 12-24 hours after alcohol has stopped)
o Includes visual, auditory or tactile hallucinations.
Withdrawal seizures (can appear 24-48 hours after alcohol has stopped)
o These are generalised tonic-clonic seizures.
Alcohol withdrawal delirium or 'delirium tremens' (can appear 48-72 hours after
alcohol has stopped).

1284. A 65yo lady presents with dyspareunia. What will you give her for her condition?
a. HRT
b. COCP
c. Estrogen gel
d. Testosterone gel
c. Estrogen gel
In older women vaginal dryness due to hormonal deficiency mainly oestrogen
Atrophic vaginitis
Current treatment guidelines for vaginal atrophy recommend the use of minimally
absorbed local vaginal oestrogens, along with non-hormonal lubricants or moisturisers,
coupled with maintenance of sexual activity.
1285. A 35yo lady with subserosal fibroid=4cm and submural fibroid=6cm is planning for
a child.
Which way will you remove the fibroids?
a. Laproscopy
b. Vaginal myomectomy
c. Abdominal myomectomy
d. Drugs

e. Reassure
c. Abdominal myomectomy
Subserous...laparoscopic
Intramural...abdominal
Submucous...hysteroscopy
Mixed ...abdominal
1286. A 32yo presents with heavy blood loss, US: uterine thickness>14mm. What is the
best possible
management for her?
a. COCP
b. UAE
c. Hysteroscopy myomectomy
d. Abdominal myomectomy
e. Endometrial ablation
b. UAE
UAE can shrink the endometrial bulk and resolve menorrhagia. So preferred here.
for Menorrhagia in young patient , childbearing age, UAE is preferred choice.
If fertility is not needed, thickness is < 3mm then endometrial ablation.
If fertility is required and >3mm then UAE
If fertility is not needed and >3mm then hysterectomy...
Since here the pt is 32yo so we assume we need to preserve her fertility and size is 14
mm so UAE is the answer.
Uterine artery embolism UAE would be a very good option here as it would help reduce
the endometrial thickness, I assume she has endometrial hyperplasia here,
myomectomy is treatment of choice for uterine fibroids in women who are subfertile,
COCP contain estrogen which would further worsen the hyperplasia,
Uterine fibroids
Features
may be asymptomatic
menorrhagia
lower abdominal pain: cramping pains, often during menstruation
bloating
urinary symptoms, e.g. frequency, may occur with larger fibroids
subfertility
Diagnosis
transvaginal ultrasound
Management
medical: symptomatic management e.g. with combined oral contraceptive pill.
GnRH agonists may reduce the size of the fibroid but are typically useful for
short-term treatment
surgery is sometimes needed: myomectomy, hysterscopic endometrial ablation,
hysterectomy
uterine artery embolization

1287. A pt comes with sudden loss of vision. Exam: high BP. Fundoscopy: retina
appears swollen.
Which blood vessel occlusion is involved?
a. Branch RVO
b. Branch RAO
c. CRAO
d. CRVO
d. CRVO
In CRAO you'd see a pale white retina,and is usually secondary to a thromboembolus ie
in people with condition predisposing to a hypercoagulative state,and yes HTN is one
such state...But a swollen retina indicates blood stasis,ie congestion in the retina due to
outflow obstruction due to blockage of the main blood outflow channel from the retina ie
CRV
In CRAO retina would be pale and you'd see a cherry red macula
In Non-ischaemic - mild defect. There are widespread dot-blot and flame haemorrhages
throughout the fundus and some disc oedema. In Ischaemic - severe visual impairment,
the fundus looks similar to the non-ischaemic picture but disc oedema is more severe.
CRVO: tomato splash, swollen huge optic disc, congested fundus
CRAO: white retina, pale fundus, cherry red spot
1288. A 2yo girl has had a temp=39C, poor appetite, abdominal pain and urinary
frequency for 3d.
What is the single most appropriate inv?
a. Catheter specimen of urine for culture
b. Clean catch urine specimen for culture
c. CBC
d. KUB US
e. Supra-pubic aspirate of urine for culture
b. Clean catch urine specimen for culture
protocol...clean catch of urine ,if positive for nitrates do US if negative do msu
Urine collection method
clean catch is preferable
if not possible then urine collection pads should be used
cotton wool balls, gauze and sanitary towels are not suitable
invasive methods such as suprapubic aspiration should only be used if noninvasive methods are not possible
Management
infants less than 3 months old should be referred immediately to a paediatrician
children aged more than 3 months old with an upper UTI should be considered
for admission to hospital. If not admitted oral antibiotics such as cephalosporin or
co-amoxiclav should be given for 7-10 days
children aged more than 3 months old with a lower UTI should be treated with
oral antibiotics for 3 days according to local guidelines, usually trimethoprim,
nitrofurantoin, cephalosporin or amoxicillin. Parents should be asked to bring the
children back if they remain unwell after 24-48 hours
antibiotic prophylaxis is not given after the first UTI but should be considered with
recurrent UTIs

1289. A child with T1DM who is not compliant with meds and eats a lot. He thinks that
he is short in his class. He is not happy. What would you do next?
a. Refer to psychologist
b. Refer to pediatrician
c. Refer to GP
d. Refer to social services
e. Change type of insulin
a. Refer to psychologist
patient is non-compliant... he is insecure and unhappy...so A
1290. An 8yo boy with a BMI=28 was admitted to a surgical ward following a MVC. He
was found to have glycosuria. When he recovered from his injury the glycosuria
resolved. What is the single
most appropriate follow-up inv?
a. Fasting blood glucose conc
b. Glycosylated hemoglobin - HbA1c
c. OGTT
d. Random blood glucose conc
e. Serum cortisol conc
a. Fasting blood glucose conc
to diagnose Diabetes, primordial mode of investigation is fasting blood glucose on
different occasions.
FBS to differentiate btw Stress-Induced Hyperglycaemia and DM.
1291. At birth, a baby boy at 38wks GA weighs 1.8kgs. He has hepato-splenomegaly
and a rash. Blood test show raised level of bilirubin and liver enzymes. What is the most
likely dx?
a. Galactosemia
b. Biliary atresia
c. G6PD deficiency
d. Rh incompatibility
e. Congenital viral infection
e. Congenital viral infection
congenital rubella....blueberry muffin rash with extra medullary hematopoiesis leading to
hepatosplenomegaly, deranged liver enzymes with low birth weight

Congenital infections
The major congenital infections encountered in examinations are rubella,
toxoplasmosis and cytomegalovirus
Cytomegalovirus is the most common congenital infection in the UK. Maternal infection
is usually asymptomatic
Rubella

Toxoplasmosis

Cytomegalovirus

Characteristic
features

Sensorineural
deafness
Congenital cataracts
Congenital heart
disease (e.g. patent
ductus arteriosus)
Glaucoma

Cerebral calcification
Chorioretinitis
Hydrocephalus

Growth retardation
Purpuric skin lesions

Other
features

Growth retardation
Hepatosplenomegaly
Purpuric skin lesions
'Salt and pepper'
chorioretinitis
Microphthalmia
Cerebral palsy

Anaemia
Sensorineural
Hepatosplenomegaly deafness
Cerebral palsy
Encephalitis/seizures
Pneumonitis
Hepatosplenomegaly
Anaemia
Jaundice
Cerebral palsy

1292. A 12yo boy with T1DM has poor long-term control. He is unconscious,
hyperventilating and
dehydrated. His blood glucose is 28mmol/l. What is the single most imp initial tx?
a. Albumin IV
b. Bicarbonate IV
c. Insulin IV
d. Insulin SC
e. Saline 0.9% IV
e. Saline 0.9% IV
Management
fluid replacement: most patients with DKA are deplete around 5-8 litres. Isotonic
saline is used initially
insulin: an intravenous infusion should be started at 0.1 unit/kg/hour. Once blood
glucose is < 15 mmol/l an infusion of 5% dextrose should be started
correction of hypokalaemia
1293. A 30yo woman on OCP presents with dilated tortuous veins crossing her
abdomen to join the tributaries to SVC. What is the single most likely cause?
a. Intra-abdominal malignancy
b. Ovarian cyst
c. Fibroids
d. Ascites
e. DVT
a. Intra-abdominal malignancy

1294. An 84yo woman with drusen and yellow spots in the center of retina. What is the
single most likely dx?
a. Macular degeneration
b. HTN retinopathy
c. MS
d. DM background
e. Proliferative DM retinopathy
a. Macular degeneration
dry (geographic atrophy) macular degeneration: characterised by drusen - yellow
round spots in Bruch's membrane
wet (exudative, neovascular) macular degeneration: characterised by choroidal
neovascularisation. Leakage of serous fluid and blood can subsequently result in
a rapid loss of vision. Carries worst prognosis
Features
reduced visual acuity: 'blurred', 'distorted' vision, central vision is affected first
central scotomas
fundoscopy: drusen, pigmentary changes
1295. A pt presents with headache, blurring of vision and acuity loss. On fundoscopy,
dots and blots
were noted with huge red swollen optic disc. What is the most probable dx?
a. CRAO
b. Branch RAO
c. CRVO
d. Optic atrophy
c. CRVO
Branch RVO: unilateral, painless blurred vision, metamorphopsia (image distortion) a
field defect. Fundoscopy will reveal vascular dilatation and tortuosity of the affected
vessels, with associated haemorrhages in that area only (look for an arc of
haemorrhages, like a trail left behind a cartoon image of a shooting star).
Retinal vein occlusion is one of the most common causes of sudden painless unilateral
loss of vision. Loss of vision is usually secondary to macular oedema.
Central retinal vein occlusion
incidence increases with age, more common than arterial occlusion
causes: glaucoma, polycythaemia, hypertension
severe retinal haemorrhages are usually seen on fundoscopy

1296. A 64yo DM has come for a routine eye check up. Fundoscopy: new vessels all
over the retina. What is the most appropriate management?
a. Strict sugar control
b. Regular eye check ups
c. Non urgent referral to specialist
d. Laser photocoagulation
e. Insulin
d. Laser photocoagulation
This has been the mainstay of treatment for a period of 25 years: the aim is to induce
regression of new blood vessels and reduce central macular thickening.

Traditional classification
Background retinopathy
microaneurysms (dots)

New classification
Mild NPDR
1 or more microaneurysm

blot haemorrhages (<=3)


hard exudates

Pre-proliferative retinopathy
cotton wool spots (soft
exudates; ischaemic nerve
fibres)
> 3 blot haemorrhages
venous beading/looping
deep/dark cluster
haemorrhages
more common in Type I
DM, treat with laser
photocoagulation

Moderate NPDR
microaneurysms
blot haemorrhages
hard exudates
cotton wool spots, venous
beading/looping and intraretinal
microvascular abnormalities (IRMA) less
severe than in severe NPDR
Severe NPDR
blot haemorrhages and microaneurysms
in 4 quadrants
venous beading in at least 2 quadrants
IRMA in at least 1 quadrant

1297. A 25yo primigravida of 8wk GA presents with severe lower abdominal pain,
vaginal bleeding and passage of clots. The internal os is open. What is the most likely
dx?
a. Appendicitis
b. Placental abruption
c. Ectopic pregnancy
d. Abortion
d. Abortion
Threatened miscarriage: mild symptoms of bleeding. Usually little or no pain. The
cervical os is closed.
Inevitable miscarriage: usually presents with heavy bleeding with clots and pain.
The cervical os is open. The pregnancy will not continue and will proceed to
incomplete or complete miscarriage.
Incomplete miscarriage: this occurs when the products of conception are partially
expelled. Many incomplete miscarriages can be unrecognised missed
miscarriages.
Missed miscarriage: the fetus is dead but retained. The uterus is small for dates.
A pregnancy test can remain positive for several days. It presents with a history
of threatened miscarriage and persistent, dirty brown discharge. Early pregnancy
symptoms may have decreased or gone.

1298. A man developed intense pain after using the end of a pencil to scratch his inner
ear. He took out the pencil from his ear and realized the end of the pencil with the rubber
part is still stuck in his ear. What is the most appropriate management?
a. Remove with a hook
b. Instill olive oil
c. Remove GA
d. Remove with magnet instrument
e. Do syringing

a. Remove with a hook


Insects should be killed prior to removal, using 2% lidocaine.
Remove batteries or magnets as soon as possible to prevent corrosion or burns.
Do not crush a battery during removal.
Adhesives (eg, Super Glue) may be removed manually within 1-2 days once
desquamation has occurred.

Methods for removal

Forceps or hook: grasp the object with forceps, or place a hook behind the object
and pull it out.
Irrigation is often effective. Irrigation with water is contra-indicated for soft
objects, organic matter or seeds (which may swell and increase the level of pain
and difficulty to remove if exposed to water).
Suction with a small catheter held in contact with the object may be effective.

1299. A 16yo boy presents with acute pain in the right ear and little bleeding from the
same ear. He had been in a boxing match and had sustained a blow to the ear. There is
little amount of blood
in the auditory canal and a small perforation of the eardrum. What is the most
appropriate
management?
a. Admission for parental
antibiotics
b. Nasal decongestant
c. Oral amoxicillin
d. OPD review
e. Packing of ear
f. Surgical intervention
g. Syringing ENT
c. Oral amoxicillin
A torn (perforated) eardrum will usually heal by itself within 6-8 weeks. It is a skin-like
structure and, like skin that is cut, it will usually heal. In some cases, a doctor may
prescribe antibiotic medicines if there is an infection or risk of infection developing in the
middle ear whilst the eardrum is healing.
Management
no treatment is needed in the majority of cases as the tympanic membrane will
usually heal after 6-8 weeks. It is advisable to avoid getting water in the ear
during this time
it is common practice to prescribe antibiotics to perforations which occur
following an episode of acute otitis media. NICE support this approach in the
2008 Respiratory tract infection guidelines
myringoplasty may be performed if the tympanic membrane does not heal by
itself
1300. A 45yo man has noticed difficulty hearing on the telephone. He is concerned
because his father has been moderately hard of hearing since middle age. BC=normal.
An audiogram shows

moderate hearing loss in both ears across all frequencies. What is the single most likely
dx?
a. Acoustic neuroma
b. Menieres disease
c. Noise induced deafness
d. Otosclerosis
e. Presbyacusis
d. Otosclerosis
Presbycusis is after age of 60 due to degenerative changes..... It's otosclerosis cox it has
strong family history in about 50 to 60 percent of cases....secondly it's ...bilateral
....negative rinne's test.
autosomal dominant
Age - it usually presents between teen years and middle age (typically between the ages
of 15 and 35). However, the average age for having surgery is rising
Positive family history - there is approximately 1/4 risk if one parent is affected; 1/2 risk if
both parents are affected.
Patients may have low-volume speech (enhanced bone conduction leads to the
perception of their own speech as 'loud').
Schwartze's sign - reddish-blue discolouration over promontory and oval window
niche, due to vascular hyperaemia of immature abnormal bone.
Tuning fork tests (Rinne's and Weber's tests) reveal conductive pattern deafness
in the majority of cases.
Audiometry is the primary investigation of choice. Bone and air conduction must be
tested and typically reveal a purely conductive
PRESBYCUSIS
Problems are often first noted in noisy environments; there is usually a slow,
insidious onset of symptoms with gradual progression.
The ability to understand speech is often the earliest symptom as high-frequency
hearing loss predominates. It may be the patient's friends/relatives who note the
problem, rather than the patient.
When assessing elderly patients with depression or cognitive impairment,
consider hearing loss as a cause of the symptoms. Tinnitus may be a feature of
presbyacusis when the hearing impairment becomes marked.
ACOUSTIC NEUROMA
Unilateral hearing loss or tinnitus
Impaired facial sensation
Balance problems

1301. The biological parents of a child with CF come to you to know about the
chances of their future
children with the same disease. What would you say to them?
a. There is a 1:4 chance that your future child will have this disease
b. All their unaffected children will be carriers of CF
c. Nothing can be predicted
d. It can 100% dx antenatally
Cystic Fibrosis is an autosomal recessive disorder.
When two people who carry the cystic fibrosis gene have a child, there is a:
1 in 4 chance that the child will have cystic fibrosis (by inheriting the cystic fibrosis
gene from both parents).
2 in 4 chance that the child will not have cystic fibrosis but will be a carrier (by
inheriting a cystic fibrosis gene from one parent but the normal gene from the other
parent).
1 in 4 chance that the child will not have cystic fibrosis and will not be a carrier
(by inheriting the normal gene from both parents).

1302. A 14yo boy presents with recurrent abdominal pain, malaise and weight
loss over 6m. Exam:
vague mass is felt in RIF. Colonoscopy shows transmural inflammation and
granulomata. What is
the most appropriate management?
a. Sulfasalazine
b. Paracetamol
c. Metronidazole
d. Ibuprofen
Colonoscopy indicate colonic involvement in this case of Crohns ( clinchers
here are transmural inflammation and granulomata ). Clinical experience
suggests a modest benefit of metronidazole (10 or 20 mg/kg/day) or the
combination of metronidazole and ciprofloxacin for primary or adjunctive
therapy of colonic Crohn's disease, but not for isolated small intestinal
disease.it is also to warn that Metronidazole in particular can be associated
with permanent peripheral neuropathy when used for prolonged periods,
especially at higher doses.
Prednisolone is the drug of first choice for Crohns disease.
1303. A 62yo prv shipyard worker complains of breathlessness and chest
pain for 6m. He has now developed a large pleural effusion. Which is the
single best diagnostic inv?
a. ABG
b. Bronchoscopy
c. CXR
d. Pleural biopsy

e. Transfer factor
ans. Pleural biopsy
Dx Malignant mesothelioma
Malignant mesothelioma should be considered in patients with history of
asbestos exposure and presenting with pleural effusion+chest pain .

Mesothelioma is an uncommon type of cancer that occurs in the


tissues covering the lungs or the abdomen. Past exposure to
asbestos is a risk factor for mesothelioma. Initial symptoms are
variable but can include shortness of breath, chest pain or abdominal
swelling, cough, hoarse voice.
1304. A 67yo man presents with a hx of increasing confusion and drowsiness.
He had a fall 2wk ago. CT head reveals a chronic subdural hematoma. What
is the best management for this pt?
a. Craniotomy
b. Burr hole drainage
c. Conservative management
d. Excision and biopsy
B. Burr hole craniotomy.
1305. A 45yo male with epigastric discomfort has been given triple therapy.
He has now returned after 4wks of epigastric discomfort. What inv would you
do for him?
a. ECG
b. H.pylori breath test
c. Endoscopy and biopsy
d. US
As the pt is below 55 yrs old, Urea breath test should be done following a
4wks triple therapy.. If H.pylori has not been eradicated, we give triple regime
for one more month, and if it's normal (h pylori eradicated) then proceed with
endoscopy. Had his age been more than 55yrs, we would have directly gone
for endoscopy. OHCM 243
1306. A 13yo boy with umbilical pain for the last 12h presents with anorexia,
nausea and has not passed a bowel motion 24h. What is your dx?
a. Acute appendicitis
b. IBD
c. IBS
d. Meckels diverticulum
e. Muscle strain
f. Ovarian cysts
g. PID
h. Psoas hematoma
i. Pyelonephritis
j. Uretric calculus
k.

its MD because it also presents with symptoms like that of Appendicitis except
rebound tenderness. Moreover, had it been AA, there would have been hx of
rebound tenderness and perhaps pain in RIF.
MD presents in 10-43% of symptomatic patients. The frequency of
complications of Meckel's diverticulum varies widely in the literature. Studies
varyingly report intestinal obstruction or haemorrhage as the most common
complication in adults.
The presenting symptoms are usually abdominal pain, vomiting and
constipation. Various mechanisms produce the obstruction, including a fibrotic
band attaching the diverticula to the abdominal wall causing a volvulus of the
small bowel and intussusception in which the diverticulum is the lead point.
An intussusception may present with redcurrant jelly stools or a palpable lump
in the lower abdomen.Meckel's diverticulum should always be considered in
the differential diagnosis of patients presenting with rectal bleeding or
intestinal obstruction.( patient.co)
MECKELS' Presentation
Asymptomatic
Meckel's diverticulum is a common incidental finding at laparotomy. The vast
majority of those with Meckel's diverticulum are asymptomatic. Complications
are most likely to occur when the diverticulum contains heterotypic tissue.
This is most often gastric, but may also be pancreatic, jejunal or colonic
mucosa. The lifetime risk of developing a complication that requires surgery is
thought to be 4-6%.
Haemorrhage
This accounts for 25-50% of all complications. It is more common in children
younger than 2 years (in which age group it is the most common
complication) and in males. The patient usually reports bright red blood in the
stools. The amount may vary from minimal recurrent episodes to a large
shock-producing haemorrhage. Meckel's diverticulum should always be
excluded in a child presenting with massive painless rectal bleeding.
The blood may be bright red if the bleeding is brisk, or darker if it is milder and
transit time is slow. Melaena-like tarry stool may also be seen if gastric tissue
present in the diverticulum ulcerates, or if it produces acid which causes
damage to the adjacent ileal mucosa.
APPENDICITIS is most common between the ages of 10 and 20 years but
can occur at any age.
Classic symptoms often do not appear in young children, in pregnant women
and in the elderly and the diagnosis is particularly easy to miss in these age
groups. The classical presentation consists of:
Pain:
Early periumbilical pain moves, after hours or sometimes days, to the right
iliac fossa (RIF) as the peritoneum becomes involved. Pain which wakes the
patient or keeps a child awake is significant.
Movement and coughing aggravate the pain. The patient may lie still with
shallow breathing. Deep breathing and coughing hurt.

Nausea, vomiting, anorexia. The patient is usually constipated or simply does


not want to have the bowels open, but may have diarrhoea. Rapidly
progressive cases may have recurrent vomiting without fever and diarrhoea.
This may be marked in post-ileal appendix (which is rare).

1307. A 46yo man with tachycardia has the following ECG. What is the most
likely dx?
a. SVT
b. VT
c. Mobitz I heart block
d. Atrial fibrillation
e. WPW syndrome
1308. A 24yo male is admitted with acute seere asthma. Tx is initiated with
100% oxygen, nebulized
salbutamol and ipratropium bromide nebulizers and IV hydrocortisone.
Despite initial tx there is
no improvement. Which is the next step in management?
a. IV aminophylline
b. IV magnesium sulphate
c. IV salbutamol
d. IM adrenaline
e. IV adrenaline
British Thoracic Society guidelines:
magnesium sulphate recommended as next step for patients who are not
responding (e.g. 1.2 - 2g IV over 20 mins)
little evidence to support use of IV aminophylline (although still mentioned in
management plans)
if no response consider IV salbutamol
1309. A 49yo man first presented with increasing difficulty in swallowing.
Several months later he developed weakness in his right foot. Now he can no
longer feed himself, he chokes on food and has become confined to a
wheelchair. What is the most likely dx?
a. Cerebral tumor
b. Myasthenia gravis
c. Lambert-Eaton syndrome
d. Motor neuron disease
e. Cerebro-vascular disease
In MND, motor nerves become damaged and eventually stop working.
Therefore, the muscles that the damaged nerves supply gradually lose their
strength. There are various subtypes of MND. In each type, symptoms tend to
start in different ways. However, as the disease progresses, the symptoms of

each type of MND tend to overlap. This means that symptoms in the later
stages of each type of MND become similar. The main types of MND are:
Amyotrophic lateral sclerosis (ALS). This is the classical MND and the
most common type. About 8 in 10 people with MND have this type. Symptoms
tend to start in the hands and feet. The muscles tend to become stiff as well
as weak at first.
Progressive bulbar palsy (PBP). About 2 in 10 people with MND have this
type. The muscles first affected are those used for talking, chewing and
swallowing (the bulbar muscles).
Progressive muscular atrophy (PMA). This is an uncommon form of MND.
The small muscles of the hands and feet are usually first affected but the
muscles are not stiff.
Primary lateral sclerosis (PLS). This is a rare type of MND. It mainly causes
weakness in the leg muscles. Some people with this type may also develop
clumsiness in the hands or develop speech problems.
1310. A 10yo boy with lower abdominal pain for the last 10d presents with a
hx of passing 6-8 loose stools. Temp=38.8C. He is tender in the right lower
quadrant and has an anal fistula. Choose the single most likely cause of
abdominal pain.
a. IBD
b. IBS
c. Pyelonephritis
d. Uretric calculus
e. Gastroenteritis
Anal fistula is the distinctive feature here only seen in Crohns (IBD)
Symptoms of Crohns Disease
Diarrhoea
Weight loss
Ulcers.
Generally feeling unwell,
Anaemia
Mouth ulcers
Anal fissures
Tenesmus
Symptoms of IBS include :
Pain and discomfort
Bloating
Changes in stools:
:Some people have bouts of diarrhoea, and some have bouts of
constipation.
Some people have bouts of diarrhoea that alternate with bouts of
constipation.
Other symptoms are:
o Feeling sick (nausea).
o Headache.

Belching.
Poor appetite.
Tiredness.
Backache.
Muscle pains.
Feeling quickly full after eating.
Heartburn.
Bladder symptoms (an associated irritable bladder).
1311. A 28yo woman with hx of drug addiction wants to start a family and
have a baby. She would like to stop taking heroin and asked for something to
help her stay away from it. What drug tx would you give her?
a. Naloxone
b. Acamprosate
c. Methadone
d. Chlordiazepoxide
e. Naltrexone
C
Treatment is different if a pt comes with opioid intoxication (NALOXONE
should be given ), and if someone comes and wants to leave opioids i.e,
detoxification (Methadone or buprenorphene would be the choice then )
Chlordiazepoxide and Acamprosate are used for alcoholism. Vit B Complex
and Chlordiazepoxide for alcohol withdrawal and acamprosate for
dependence. Oxazepam is the drug of choice for alcohol detoxification in pts
with liver disease as its not metabolized by liver.
o
o
o
o
o
o
o
o

Treatment of opioid intoxication


If a patient has collapsed and is thought to be acutely intoxicated, call
999/112/911 and refer urgently to hospital.

Naloxone (a pure opioid antagonist used for reversing opioid


intoxication) has a rapid onset of action and can be given IM, IV or
subcutaneously.

Therapy is otherwise mostly supportive, eg maintain airway,


ventilation if necessary and IV fluids.

Tx of detoxification

Methadone, and buprenorphine are equally effective in detoxification


regimens. The place of lofexidine in detoxification programmes requires
further research.[4]
Opioid detoxification should be offered as part of a package including
preparation and post-detoxification support to prevent relapse.
Psychosocial interventions (eg talking therapies, cognitive behavioural
therapy, family therapy) and keyworking should be delivered alongside
pharmacological interventions.[2]
If detoxification is unsuccessful, patients should have access back into
maintenance and other treatment.

1312. A pt with vesicles in the maxillary division of trigeminal nerve. Which


area of mucus membrane will be involved?
a. Palate
b. Cheek
c. Cornea
d. Conjunctiva
The ophthalmic nerve (V1) carries sensory information from the scalp and
forehead, the upper eyelid, the conjunctiva and cornea of the eye, the nose
(including the tip of the nose, except alae nasi), the nasal mucosa, the frontal
sinuses and parts of the meninges (the dura and blood vessels). The
maxillary nerve (V2) carries sensory information from the lower eyelid and
cheek, the nares and upper lip, the upper teeth and gums, the nasal mucosa,
the palate and roof of the pharynx, the maxillary, ethmoid and sphenoid
sinuses and parts of the meninges. The mandibular nerve (V3) carries sensory
information from the lower lip, the lower teeth and gums, the chin and jaw
(except the angle of the jaw, which is supplied by C2-C3), parts of the
external ear and parts of the meninges. The mandibular nerve carries touchposition and pain-temperature sensations from the mouth. Although it does
not carry taste sensation (the chorda tympani is responsible for taste), one of
its branchesthe lingual nervecarries sensation from the tongue.
for mucous membrane its palate for Maxillary... if they had not mentioned
mucous then mid face externally is supplied by Maxillary. Mandibular is
division that gives buccal branch and supplies the buccal mucosa with
sensory fibers...
Ans should be palate as question is being asked about the mucous membrane.

1313. A 52yo man presents with visual hallucinations and features of


cognitive impairment. What is
the most likely dx?
a. Frontotemporal dementia
b. Lewy body dementia
c. Delirium tremens
d. Alzheimers disease
e. Huntingtons disease
Dementia is a syndrome caused by a number of brain disorders which cause
memory loss, decline in some other aspect of cognition, and difficulties with
activities of daily living.
The symptoms fall into three groups:
Cognitive impairment: causing difficulties with memory, language,
attention, thinking, orientation, calculation, and problem-solving.

Psychiatric or behavioural disturbances: changes in personality,


emotional control, and social behaviour; depression, agitation,
hallucinations, and delusions.
Difficulties with activities of daily living, such as driving, shopping,
eating, and dressing.
There are subtle differences in the presentation of different types of dementia.
Alzheimer's disease tends to have an insidious onset, whereas vascular
dementia typically has a series of stepwise increases in symptom severity.
DLB may present with fluctuating levels of consciousness, hallucinations,
sleep disorders, falls and Parkinsonian features. In Parkinson's disease

dementia, the Parkinsonian features predate the dementia by a significant


amount of time. In frontotemporal dementia, behavioural changes (such as
disinhibition or apathy) and language disturbances are often presenting
features. It may be important to determine the type of dementia - in DLB, for
example, making this diagnosis will have important implications for treatment
(use of neuroleptics is avoided, as motor and mental impairment is worsened
and mortality may be increased).
Ans should be Lewy Body dementia bcoz of presence of hallucinations . Rest
of the types are explained above and DT is a psychotic condition typical of
withdrawal in chronic alcoholics, involving tremors, hallucinations, anxiety,
and disorientation.
1314. A 40yo woman who has recently returned from working in the middle
east complains of thirst, episode of loin pain, urinary frequency, dysuria and
has passed a urinary stone. All inv are normal. She plans to return to the
Middle East in a months time. What is the single best advice
to prevent recurrent stone formation?
a. Drink less milk
b. High fibre diet
c. Increase fluid intake
d. Low calcium diet
e. Low protein diet
The cause in this scenario seems to be dehydration as the pt lives in middle
east. She should increase her fluid intake in order to maintain adequate
urinary flow by preventing urinary stasis.
Several risk factors are recognised to increase the potential of a susceptible
individual to develop stones. These include:
Anatomical anomalies in the kidneys and/or urinary tract - eg,
horseshoe kidney, ureteral stricture.
Family history of renal stones.
Hypertension.
Gout.
Hyperparathyroidism.
Immobilisation.
Relative dehydration.

Metabolic disorders which increase excretion of solutes - eg, chronic


metabolic acidosis, hypercalciuria, hyperuricosuria.
Deficiency of citrate in the urine.
Cystinuria (an autosomal-recessive aminoaciduria).
Drugs - eg, diuretics such as triamterene and calcium/vitamin D
supplements.
More common occurrence in hot climates.
Increased risk of stones in higher socio-economic groups.
Contamination - as demonstrated by a spate of melaminecontaminated infant milk formula

1315. A 32yo man presents with 3d of scrotal pain. Exam: thickening o the left
testis and it is hot to touch. What is the most appropriate management?
a. Analgesia
b. Reassurance
c. Antibiotics
d. Referral to surgeon
Epididymo-orchitis
Causes :
In men under 35 years old, infection is most often due to a sexually
transmitted pathogen - eg, Chlamydia trachomatis and Neisseria
gonorrhoeae.
In men over 35 years old, infection is most often due to a non-sexually
transmitted Gram-negative enteric organism causing urinary tract
infections - eg, Escherichia coli,Pseudomonas spp. Specific risk factors
include recent instrumentation or catheterisation.
Signs n symptoms :
It usually presents with unilateral scrotal pain and swelling of relatively
acute onset.
Acute epididymitis is usually unilateral but is bilateral in 5-10% of the
patients.
Tenderness to palpation on the affected side.
Palpable swelling of the epididymis, starting with the tail at the lower
pole of the testis and spreading towards the head at the upper pole of
the testis with or without involvement of the testicle.
There may also be urethral discharge, secondary hydrocele, erythema
and/or oedema of the scrotum on the affected side and pyrexia.
Clincher here for making the diagnosis is the unilateral tenderness and
the hot temperature.
Differential would be testicular torsion , but that normally occurs in men
who are younger than 20 years though it can occur at any age.
A painful swollen testicle in an adolescent boy or a young man should be
managed as torsion until proven otherwise.

Torsion is more likely if the onset of pain is acute (typically around four
hours at presentation) and the pain is severe.
Empirical therapy should be given to all patients with epididymo-orchitis
before culture/NAAT results are available. The antibiotic regimen chosen
should be determined in the light of the immediate tests (urethral or FPU
smear, urinalysis) as well as age, sexual history including insertive anal
intercourse, any recent instrumentation or catheterisation and any known
urinary tract abnormalities. So, answer should be ANTIBIOTICS .

1316. A 34yo woman presents with truncal obesity, easy bruising,


hyperglycemia, high BP and depression. Which of the following invs will be
most helpful in localizing the cause for Cushings syndrome?
a. Serum cortisol
b. 24h urinary cortisol
c. Low dose dexamethasone suppression test
d. High dose dexamethasone suppression test
e. Overnight dexamethasone suppression test
Ans : High dose Dexamethasone suppression test
Investigations to confirm the presence of Cushing's syndrome :
The recommended diagnostic tests for the presence of Cushing's syndrome are 24hour urinary free cortisol, 1 mg overnight dexamethasone suppression test and latenight salivary cortisol

Once we know the diagnosis , following tests help us locate the position of
abnormality

Investigations to identify the cause of Cushing's


syndrome :
Plasma ACTH
Secretion is pulsatile and shows a diurnal variation, with the plasma
concentrations highest at 8 am and lowest at midnight. The secretion of
ACTH is increased by stress.
An undetectable plasma ACTH with an elevated serum cortisol level is
diagnostic of ACTH-independent Cushing's syndrome, which is due to
a primary cortisol-producing adrenal adenoma or carcinoma, or
exogenous glucocorticoid use. This should then be followed by an
abdominal CT or MRI scan if exogenous glucocorticoids are excluded
as the cause.
An elevated ACTH level is consistent with ACTH-dependent Cushing's
syndrome.

If the plasma ACTH level is detectable then the following tests are
indicated:
o High-dose dexamethasone suppression test:
The 8 mg overnight dexamethasone suppression test
and the 48-hour high-dose dexamethasone test may
be useful when baseline ACTH levels are equivocal.
They also help in determining whether a patient has
pituitary or ectopic ACTH production.
Greater than 90% reduction in basal urinary free
cortisol levels supports the diagnosis of a pituitary
adenoma; ectopic ACTH causes lesser degrees of
suppression.
o Inferior petrosal sinus sampling (IPS):
Performed with CRH stimulation to aid in determining the
source of excess ACTH.
A baseline and stimulated ratio of IPS to peripheral ACTH
of less than 1.8 is suggestive of ectopic ACTH, while a
ratio of IPS to peripheral ACTH of greater than 2 is
consistent with a pituitary adenoma.
o MRI of the pituitary.
o Chest and abdominal CT scans: for patients with suspected
adrenal tumours or ectopic ACTH.
o Plasma CRH: ectopic CRH production is a very rare cause of
Cushing's syndrome.
1317. A 32yo man develops hematuria 2wks after a sore throat. What is the
dx?
a. Post infection nephritis
b. IgA nephropathy
c. Membranous nephritis
d. Glomerulonephritis
Given key is d, but that is a broad term. One should always go for a
specific answer. We are now left with Post infection Nephritis and igA
nephropathy as they both follow a throat infection. In, igA nephropathy,
symptoms appear 1-3 days after the infection, whereas in Post infection,
symptoms appear 1-12 weeks after the sore throat. So, our answer here
should be A.
1318. An elder man who has anorexia, prostate symptoms and HTN. There
are small kidneys on US. What is the dx?
a. Hypertensive renal disease
b. Prostate ca
c. BPH
Only Hypertensive nephropathy leads to shrunken kidneys. Prostate
symptoms are not explained by Hypertension nephropathy unless there
is an outflow obstruction and the other two options can be the cause.

Should go with answer A. Needs to be discussed though. NOTE :


Kidney size is increased in the case of diabetic nephropathy.
1319. A 55yo woman with breast ca which has spread to lung, liver and bone
now presents with increasing constipation, weakness, thirst and anorexia for
the past 3d. Her only medication is haloperidol for hiccoughs. Today she is
disorientated and has left sided weakness. What is the most likely dx?
a. Brain mets
b. Hypercalcemia
c. Liver failure
Pt has all the features of Hyepercalcemia except the left sided weakness. But,
Brain mets can only cause localized weakness . So., the ans would be A.
1320. A 22yo man presents with a red, hot, swollen, metatarsal phalangeal
joint, sarcoilitis and
onycholysis. What is the single most likely cause of his condition?
a. Gout
b. RA
c. Reiters syndrome
d. Psoriatic arthropathy
Sacroilitis and nail changes are the clnchers here for PA.
Psoriatic arthropathy correlates poorly with cutaneous psoriasis and often precedes the
development of skin lesions. Around 10-20% percent of patients with skin lesions develop an
arthropathy with males and females being equally affected
Types*

rheumatoid-like polyarthritis: (30-40%, most common type)


asymmetrical oligoarthritis: typically affects hands and feet (20-30%)
sacroilitis
DIP joint disease (10%)
arthritis mutilans (severe deformity fingers/hand, 'telescoping fingers')

Management

treat as rheumatoid arthritis


but better prognossis

1321. Which of the following conditions requires operative management?


a. Cellulitis
b. Dyshidrosis
c. Erysipelas
d. Fourniers gangrene
e. Lymphangitis
Fourniers gangrene is a medical emergency that is difficult to recognise in the early
stages
It can be classified according to the causative organism:

type 1 is caused by mixed anaerobes and aerobes (often occurs post-surgery in


diabetics)

type 2 is caused by Streptococcus pyogenes

Features

acute onset
painful, erythematous lesion develops
extremely tender over infected tissue

Management

urgent surgical referral debridement


intravenous antibiotics

1322. A 55yo pt presents with collapse and complains of abdominal pain that
radiates to the back. An expansile abdominal mass if felt on examination and
the pt is in shock. What is the single most likely dx?
a. Ruptured aortic aneurysm
b. Renal colic
c. Trauma
d. Endocarditis
e. Atheroma
Expansile mass and radiating towards the back are the clinchers here. Ans is
RAA. Has already been discussed in detail in mcq 76.
1323. A house-bound 78yo man with severe COPD has had a gradual
deterioration over recent months and is now breathless at rest. He is on
maximal inhaled medical therapy. Result: pH=7.36, PaCO2=5.9kPa,
PaO2=6.9kPa. What is the single most appropriate additional tx?
a. Aminophylline PO
b. ACEi PO
c. Antibiotic PO
d. Oxygen
e. Steroid PO
The 2010 NICE guidelines on COPD clearly define which patients should be
assessed for and offered long-term oxygen therapy (LTOT). Patients who receive
LTOT should breathe supplementary oxygen for at least 15 hours a day. Oxygen
concentrators are used to provide a fixed supply for LTOT.
Assess patients if any of the following:

very severe airflow obstruction (FEV1 < 30% predicted). Assessment should
be 'considered' for patients with severe airflow obstruction (FEV1 30-49%
predicted)
cyanosis
polycythaemia
peripheral oedema
raised jugular venous pressure
oxygen saturations less than or equal to 92% on room air

Assessment is done by measuring arterial blood gases on 2 occasions at least 3


weeks apart in patients with stable COPD on optimal management.
Offer LTOT to patients with a pO2 of < 7.3 kPa or to those with a pO2 of 7.3 - 8 kPa
and one of the following:

secondary polycythaemia
nocturnal hypoxaemia
peripheral oedema
pulmonary hypertension

1324. A 79yo man has a swelling of the right groin which was clinically dx to
be indirect inguinal hernia. What is the single feature of the hearnia sac that
would confirm the dx?
a. Comes through femoral ring
b. Doesnt pass through the deep inguinal ring
c. Lies below and lateral to the pubic tubercle
d. Only passes through the superficial inguinal ring
e. Passes through the deep inguinal ring
Ans E
There are two types of inguinal hernia, direct and indirect, which are defined by their
relationship to the inferior epigastric vessels. Direct inguinal hernias occur medial to
the inferior epigastric vessels when abdominal contents herniate through a weak
spot in the fascia of the posterior wall of the inguinal canal, which is formed by the
transversalis fascia. Indirect inguinal hernias occur when abdominal contents
protrude through the deep inguinal ring, lateral to the inferior epigastric vessels;
this may be caused by failure of embryonic closure of the processus vaginalis..

1325. A 56yo woman with hx of breast cancer 10y ago has undergone radical
mastectomy and axillary LN removal, now complains of swollen upper limb
3wks after an insect bite. The bite site is better but gross edema is still
present. What is the cause?
a. Lymphedema
b. Breast Ca
c. Allergy
d. Filariasis
Upper limb lymphatics are drained by the Axillary lymph nodes. So, their
removal results in lymphedema.
1326. A homeless person is found wandering on the street. He had ataxic
gait, nystagmus and
opthalmoplegia. He looked unkempt and his clothes had a sweaty odour. He
had a dry mucous membrane with a BP=118/70mmHg and PR=90bpm. Blood
sugar level=8. Alcohol breath test= -ve. What would the most imp initial inv?
a. IV insulin
b. Vit B complex
c. Bolus IV 0.9%NS
d. IV dextrose
e. Antibiotics

Ans B

Vit B complex deficiency leads to symptoms :

Extreme tiredness or fatigue.

A lack of energy or lethargy.

Being out of breath.

Feeling faint.

Headache.

Ringing in the ears (tinnitus)

Lack of appetite.

Peripheral neuropathy

confusion, anxiety, paranoia

1327. A 34yo man has supra-orbital pain and tenderness and developed
tenderness over the maxilla. He also has mild fever. What is the single likely
cause for these symptoms?
a. Acute sinusitis
b. GCA
c. Trigeminal neuralgia
d. Maxillary carcinoma
Ans is A
Sinusitis describes an inflammation of the mucous membranes of the paranasal
sinuses. The sinuses are usually sterile - the most common infectious agents seen in
acute sinusitis are Streptococcus pneumoniae, Haemophilus influenzae and
rhinoviruses.
Predisposing factors include:

nasal obstruction e.g. Septal deviation or nasal polyps


recent local infection e.g. Rhinitis or dental extraction
swimming/diving
smoking

Features

facial pain: typically frontal pressure pain which is worse on bending forward
nasal discharge: usually thick and purulent
nasal obstruction: e.g. 'mouth breathing'
post-nasal drip: may produce chronic cough

Management of acute sinusitis

analgesia
intranasal decongestants
oral antibiotics are not normally required but may be given for severe
presentations. Amoxicillin is currently first-line

1328. A 51yo woman presents with painful tongue and complains of


tiredness. She is pale and has angular stomatitis and a smooth red tongue.
There is no koilonychea. Choose the single cell type you will find on the blood
film.
a. Numerous blast cells
b. Oval macrocytes
c. Spherocytes
d. Microcytic hypochromic
e. Mexican hat cells
f. Erythrocytes
These are the features of vit. b12 and folic acid deficiency. Angular stomatitis is the
clincher here. Ans would be B
1329. A 24yo woman presents with tingling and twitching of her fingers
followed by throbbing
unilateral headache. What is the most likely dx?
a. Tension headache
b. Migraine
c. Cluster headache
d. TIA
e. SAH
Migraine is a common type of primary headache. It is characterised typically by:

a severe, unilateral, throbbing headache


associated with nausea, photophobia and phonophobia
attacks may last up to 72 hours
patients characteristically go to a darkened, quiet room during an attack
'classic' migraine attacks are precipitated by an aura. These occur in around
one-third of migraine patients
typical aura are visual, progressive, last 5-60 minutes and are characterised
by transient hemianopic disturbance or a spreading scintillating scotoma
formal diagnostic criteria are produced by the International Headache Society
(see below)

Epidemiology

3 times more common in women


prevalence in men is around 6%, in women 18%

Common triggers for a migraine attack

tiredness, stress
alcohol
combined oral contraceptive pill
lack of food or dehydration
cheese, chocolate, red wines, citrus fruits
menstruation
bright lights

Migraine diagnostic criteria


A

At least 5 attacks fulfilling criteria B-D

Headache attacks lasting 4-72 hours* (untreated or unsuccessfully treated)

Headache has at least two of the following characteristics:

1. unilateral location*
2. pulsating quality (i.e., varying with the heartbeat)
3. moderate or severe pain intensity
4. aggravation by or causing avoidance of routine physical activity
(e.g., walking or climbing stairs)

During headache at least one of the following:

1. nausea and/or vomiting*


2. photophobia and phonophobia

Not attributed to another disorder (history and examination do not suggest a


secondary headache disorder or, if they do, it is ruled out by appropriate
investigations or headache attacks do not occur for the first time in close
temporal relation to the other disorder)

1330. A young child dx with chicken pox. Usually goes to day care. What is
the most appropriate
advice?
a. Child should be admitted to hospital straight away
b. Isolate the child from parents and siblings at home
c. Advice that he can go back to nursery when the rash is crusted over
Ans is C
Chickenpox is caused by primary infection with varicella zoster virus. Shingles is
reactivation of dormant virus in dorsal root ganglion
Chickenpox is highly infectious

spread via the respiratory route


can be caught from someone with shingles
infectivity = 4 days before rash, until 5 days after the rash first appeared*
incubation period = 10-21 days

Clinical features (tend to be more severe in older children/adults)

fever initially

itchy, rash starting on head/trunk before spreading. Initially macular then


papular then vesicular
systemic upset is usually mild

Management is supportive

keep cool, trim nails


calamine lotion
school exclusion: current HPA advice is 5 days from start of skin
eruption. They also state 'Traditionally children have been excluded
until all lesions are crusted. However, transmission has never been
reported beyond the fifth day of the rash.'
immunocompromised patients and newborns with peripartum exposure
should receive varicella zoster immunoglobulin (VZIG). If chickenpox
develops then IV aciclovir should be considered

A common complication is secondary bacterial infection of the lesions. Rare


complications include

pneumonia
encephalitis (cerebellar involvement may be seen)
disseminated haemorrhagic chickenpox
arthritis, nephritis and pancreatitis may very rarely be seen

1331. A 7yo boy is brought by his mother. There are multiple perioral and
forehead vesicles. Some vesicles are crusted and some are not. The face is
hot. What is the most likely dx?
a. Varicella zoster
b. Herpes zoster
c. Fungal infection
d. Impetigo
e. Psoriasis
Pain and Blisters are in the area of nerve supply. Herpes Zoster seems to be
the answer.
Shingles is an acute, unilateral, painful blistering rash caused by reactivation of the
Varicella Zoster Virus (VZV).
The 'shingles vaccine'
In 2013 the NHS introduced a vaccine to boost the immunity of elderly people
against herpes zoster. Some important points about the vaccine:

will be offered to patients at the age of 70 years (a catch-up programme will


also be launched initially)
is live-attenuated and given sub-cutaneously

As it is a live-attenuated vaccine the main contraindications are immunosuppression.


Side-effects

injection site reactions


less than 1 in 10,000 individuals will develop chickenpox

Management of shingles
Oral aciclovir is first-line. One of the main benefits of treatment is a reduction in the
incidence of post-herpetic neuralgia.

1332. A 5yo boy is rescued from a burning building and is presented to the
ED. He has 5% partial
thickness burns over the arms and legs and had soot in the mouth and nose.
His breathing has
become noisy. What is the single most immediate management?
a. Nebulized adrenaline
b. Nebulized salmetarol and oxygen
c. Needle cricothyrodotomy
d. Oropharyngeal airway
e. Intubation of airway
Intubation of airway should be done immediately in order to prevent blockage of airways
secondary to laryngeal edema.
1333. A new born bay is borught with pansystolic murmur at sternal border
but the baby is not
cyanosed. What is the dx?
a. VSD
b. ASD
c. TOF
d. PDA
Pansystolic murmers are seen in the following :
Holosystolic (pansystolic)

mitral/tricuspid regurgitation (high-pitched and 'blowing' in character)


VSD ('harsh' in character)

VSD:
A ventricular septal defect (VSD) is a defect in the ventricular septum, the wall
dividing the left and right ventricles of the heart.
The ventricular septum consists of an inferior muscular and superior membranous
portion and is extensively innervated with conducting cardiomyocytes.
During ventricular contraction, or systole, some of the blood from the left ventricle leaks into
the right ventricle, passes through the lungs and reenters the left ventricle via the pulmonary
veins and left atrium. This has two net effects. First, the circuitous refluxing of blood causes
volume overload on the left ventricle. Second, because the left ventricle normally has a much
higher systolic pressure (~120 mmHg) than the right ventricle (~20 mmHg), the leakage of

blood into the right ventricle therefore elevates right ventricular pressure and volume,
causing pulmonary hypertension with its associated symptoms.
In serious cases, the pulmonary arterial pressure can reach levels that equal the systemic
pressure. This reverses the left to right shunt, so that blood then flows from the right ventricle
into the left ventricle, resulting in cyanosis, as blood is by-passing the lungs for
oxygenation.[7]
This effect is more noticeable in patients with larger defects, who may present with
breathlessness, poor feeding and failure to thrive in infancy. Patients with smaller defects
may be asymptomatic. Four different septal defects exist, with perimembranous most
common, outlet, atrioventricular, and muscular less commonly
Inv :
Auscultation
Echo

1334. A woman complaining of diarrhea, abdominal pain and fatigue. All the
tests are found to be
normal. What is the cause?
a. Somatization
b. Conversion
c. Hypochondriasis
Somatisation disorder

multiple physical SYMPTOMS present for at least 2 years


patient refuses to accept reassurance or negative test results

1335. A 26yo man has returned from NY to the UK and noticed weight loss,
night sweats, temp=37.5C
and cervical lymphadenopathy. He also has splenomegaly. What is the dx?
a. TB
b. Lymphoma
c. Bronchial carcinoma
d. Bronchitis
Night sweats and cervical lymohadenopathy is a clincher here.
Lymphoma
Hodgkin's lymphoma is a malignant proliferation of lymphocytes characterised by the
presence of the Reed-Sternberg cell. It has a bimodal age distributions being most
common in the third and seventh decades.

1336. A mother got infected with Hep B during pregnancy. Her child is born
and she is worried about
the risk of infection to the baby with Hep B. What would you give to the baby?
a. Hep B Ig only
b. Hep B full vaccine and Ig
c. Hep B vaccine only once
d. Nothing until immune status is checked
e. Hep B vaccine once and Ig

Ans is B

Hepatitis B and pregnancy


Basics

all pregnant women are offered screening for hepatitis B


babies born to mothers who are chronically infected with hepatitis B or to
mothers who've had acute hepatitis B during pregnancy should receive a
complete course of vaccination + hepatitis B immunoglobulin
studies are currently evaluating the role of oral antiviral treatment (e.g.
Lamivudine) in the latter part of pregnancy
there is little evidence to suggest caesarean section reduces vertical
transmission rates
hepatitis B cannot be transmitted via breastfeeding (in contrast to HIV)

1337. A man suffers from Herpes Zoster affecting his face. Which of the
following mucos membrane is
to be affected?
a. Cheek
b. Cornea
c. Conjunctiva
d. Oropharynx
e. Palate
This one is controversial. Tried to find out on Facebook. Most of the people
agreed on conjunctiva as cornea is not a mucous membrane.
1338. A 34yo man sustains a fx to shaft of femur after falling from the roof of
his house. Exam: distal
pulses are not palpable. Which vessel is damaged?
a. Femoral artery
b. Circumflex femoral artery
c. Profundafemoris artery
d. Popliteal artery
e. Obturator artery
f. Dorsalispedis artery
Femoral artery is the main artery and seems to be damaged as distal pulses
are absent.

Shaft of femur fractures

These are caused by a high-energy injury, such as road traffic accidents,

unless pathological fracture in a patient with osteoporosis or metastatic disease.

There are often associated injuries to the hip, pelvis, knee and other parts of

the body.

Diagnosis

Deformity, shortening, external rotation and abduction at the hip on the

affected side.

Management

Initial management:

Assess vital functions and any associated chest, head, abdominal or

spinal injuries. Resuscitate and treat life-threatening injuries as necessary.

Splint fractures (Thomas' splint or equivalent traction splint).

X-rays of the femur.

Blood tests, including blood for cross-matching.

Obtain intravenous access and start fluid replacement.

Peripheral sensation and pulses should be closely monitored.

Analgesia: adequate intravenous analgesia. Femoral nerve block is

usually effective.

Further management

Intramedullary nailing is used for treating fractures of the femoral

shaft. 10]
[

Early immobilisation and treatment reduce the risk of complications.

NICE recommends physiotherapy assessment and, unless medically or


surgically contra-indicated, mobilisation on the day after surgery. Patients
should be offered mobilisation at least once a day and regular
physiotherapy review. 1]
[

Complications

Closed fractures may be associated with a large volume of blood loss before

becoming obvious with swelling of the thigh.

Later complications include fat embolism, deep vein thrombosis, pulmonary

embolism, infection, shortening, angulation and nonunion.

1339. A 9yo child doesnt play with his peers and has collected 200 cars. He
doesnt respond to any criticism. What is the dx?
a. Autism
b. Personality disorder
c. Schizophrenia
d. Rett syndrome
e. Social anxiety
Epidemiology

75% of children are male


usually develops before 3 years of age

All 3 of the following features must be present for a diagnosis to be made

global impairment of language and communication


impairment of social relationships
ritualistic and compulsive phenomena

Other features

most children have a decreased IQ - the 'idiot savant' is rare

Associated conditions

Fragile X
Rett's syndrome

1340. A 63 yo man with vague but persistent pain. On endoscopy: columnar


epithelium was found to be pouched into muscularis. What is the dx?
a. Adenocarcinoma
b. Adenoma
c. Peptic ulcer
d. H. pylori infection
Adenocarcinoma arises from glandular cells present in the lower third of the
esophagus, often where they have already transformed to intestinal cell type (a
condition known as Barrett's esophagus).
Adenocarcinoma is now the most common type of oesophageal cancer and is more
likely to develop in patients with a history of gastro-oesophageal reflux disease
(GORD) or Barrett's.
The majority of tumours are in the middle third of the oesophagus.

Risk factors

smoking
alcohol
GORD
Barrett's oesophagus
achalasia
Plummer-Vinson syndrome
rare: coeliac disease, scleroderma

1341. A 24yo man after a head injury presents with difficulty dressing himself,
difficulty in writing and inability to differentiate the fingers of his hand. Which
part of the brain is most likely to be affected?
a. Frontal lobe
b. Parietal lobe
c. Temporal lobe
d. Occipital lobe
e. Brainstem

The parietal lobe is one of the four major divisions of the cerebral
cortex. This lobe receives and processes sensory information from the
body.
Functions of major lobes are :
1. Frontal lobeconscious thought; damage can result in mood changes,
social differences, etc. The frontal lobes are the most uniquely human of all
the brain structures.
2. Parietal lobeplays important roles in integrating sensory information
from various senses, and in the manipulation of objects; portions of the
parietal lobe are involved with visuospatial processing
3. Occipital lobesense of sight; lesions can produce hallucinations
4. Temporal lobesenses of smell and sound, as well as processing of
complex stimuli like faces and scenes.
5. Limbic lobeemotion, memory
6. Insular cortexpain, some other senses.

1342. A 16yo boy in boarding school feels unwell. He developed cough and
rash .His CXR showed bilateral consolidations. What is the cause of his
symptoms?

a. Staph aureus
b. Legionella
c. Mycoplasma
d. Streptococcus
Homeless shelters ,Young army recruits living in barracks ,young students in dormitories(in
boarding schools) ------->mycoplasma,
In legionella questions u ll mostly find some water related hints or outdoor activity near water or
air conditioning system mention and most of the times GI symptoms along with respiratory
symptoms

Mycoplasma pneumoniae is a cause of atypical pneumonia which often affects


younger patients. It is associated with a number of characteristic complications such
as erythema multiforme and cold autoimmune haemolytic anaemia. Epidemics
ofMycoplasma pneumoniae classically occur every 4 years. It is important to
recognise atypical pneumonias as they may not respond to penicillins or
cephalosporins due to it lacking a peptidoglycan cell wall.
Features

the disease typically has a prolonged and gradual onset


flu-like symptoms classically precede a dry cough
bilateral consolidation on x-ray
complications may occur as below

Complications

cold agglutins (IgM) may cause an haemolytic anaemia, thrombocytopenia


erythema multiforme, erythema nodosum
meningoencephalitis, Guillain-Barre syndrome
bullous myringitis: painful vesicles on the tympanic membrane
pericarditis/myocarditis
gastrointestinal: hepatitis, pancreatitis
renal: acute glomerulonephritis

Investigations

diagnosis is generally by Mycoplasma serology


positive cold agglutination test

1343. A 10yo boy is brought to the ED 10h after injury to the foot. It was
punctured with a metal spike that passed through his shoe. What is the next
best step?
a. Ig
b. Ig and vaccine
c. Vaccine only
d. Clean the wound
e. Antibiotics
Initial step is always to clean the wound in order to get rid of source.
1344. A 56yo male presents with persistent watery diarrhea. What is the most
likely dx?
a. Treponema pallidum

b. Nesseria meningitides
c. Cryptosporidium
d. Staph aureus
e. Pseudomonas aeruginosa
It should be c.
C coz
T pallidum...syphilis...not diarrhoea
N menigitidis....menigitis
S aureus...also not organisms for diarrhea
P aeruginosa....again not causing diarrhoea

1345. A 2yo girl has frequency, urgency and burning micturition. She has
some supra pubic tenderness.
Which one of the following is the most appropriate initial inv?
a. Supra pubic aspiration of urine for C&S
b. Clean catch of urine for C&S
c. USG
d. IVU
e. MCUG
Ans is clean catch
Urinary tract infections (UTI) are more common in boys until 3 months of age (due to
more congenital abnormalities) after which the incidence is substantially higher in
girls. At least 8% of girls and 2% of boys will have a UTI in childhood
Presentation in childhood depends on age:

infants: poor feeding, vomiting, irritability


younger children: abdominal pain, fever, dysuria
older children: dysuria, frequency, haematuria
features which may suggest an upper UTI include: temperature > 38C, loin
pain/tenderness

NICE guidelines for checking urine sample in a child

if there are any symptoms or signs suggestive or a UTI


with unexplained fever of 38C or higher (test urine after 24 hours at the
latest)
with an alternative site of infection but who remain unwell (consider urine test
after 24 hours at the latest)

Urine collection method

clean catch is preferable


if not possible then urine collection pads should be used
cotton wool balls, gauze and sanitary towels are not suitable
invasive methods such as suprapubic aspiration should only be used if noninvasive methods are not possible

Management

infants less than 3 months old should be referred immediately to a


paediatrician
children aged more than 3 months old with an upper UTI should be
considered for admission to hospital. If not admitted oral antibiotics such as
cephalosporin or co-amoxiclav should be given for 7-10 days
children aged more than 3 months old with a lower UTI should be treated with
oral antibiotics for 3 days according to local guidelines, usually trimethoprim,
nitrofurantoin, cephalosporin or amoxicillin. Parents should be asked to bring
the children back if they remain unwell after 24-48 hours
antibiotic prophylaxis is not given after the first UTI but should be
considered with recurrent UTIs

1346. An 89yo man presents with carcinoma of posterior oropharynx. Which


is the single most appropriate LN involved?
a. Pre-aortic LN
b. Aortic LN
c. Submental LN
d. Submandibular LN
e. Deep cervical LN
Oropharyngeal Lumphatics >>> Retropharyngeal Ln >> Deep cervical LN
1347. A young boy presented to the OPD 12wks after renal transplantation
with fever and pain in lower abdomen. Renal functions were deranged. Renal
biopsy showed immune cell infiltrate and tubular damage. What is the most
probable dx?
a. Pyelonephritis
b. Chronic graft rejection
c. Acute rejection
d. Drug toxicity
e. Graft vs host disease
Hyperacute rejection : Within minutes of transplant
Acute: After one week upto months
Chronic : After years due to fibrosis
1348. A 56yo lady presents with a pathological fx of T11 vertebra. There is
found to be an underlying metastatic lesion. What is her most common
primary ca?
a. Lung
b. Breast
c. Uterine
d. Brain
Breast CA is most notorious for bony mets.
1349. A 6m infant has breast milk jaundice. He is otherwise feeding well and
is not dehydrated. What would his LFTs look like?
a. Total bilirubin:40, conjugated bilirubin<5%
b. Total bilirubin:300, conjugated bilirubin 85%
c. Total bilirubin:500, conjugated bilirubin>85%
d. Total bilirubin:400, conjugated bilirubin<85%

Breast milk jaundice will not have such high levels of total bilirubin. Plus the majority

would be unconjugated hence A

Breast milk jaundice: the baby is well and the jaundice usually resolves by

six weeks but occasionally continues for up to months.

1350. A 29yo man took a tour of Japan and also travelled to other parts of
Asia, developed fever,
petecia and rash on his body. He didnt take malaria prophylaxis prior to
travel. What is the most
likely dx?
a. Malaria
b. HSP
c. HIV
d. Dengue fever
e. ITP
Petechae due to low platelets and travel Hx to asia are cinchers here. Dengue
fever is a viral infection which can progress to viral haemorrhagic fever (also yellow
fever, Lassa fever, Ebola)
Basics

transmitted by the Aedes aegyti mosquito


incubation period of 7 days
a form of disseminated intravascular coagulation (DIC) known as dengue
haemorrhagic fever (DHF) may develop. Around 20-30% of these patients go
on to develop dengue shock syndrome (DSS)

Features

causes headache (often retro-orbital)


fever
myalgia
pleuritic pain
facial flushing (dengue)
maculopapular rash

Treatment is entirely symptomatic e.g. fluid resuscitation, blood transfusion etc

1351. A 4yo boy ingested his grandmothers medicine and has developed
dilated pupil. What is the
cause?
a. Amitryptiline
b. Paracetamol
c. Iron
d. Digoxin

Amtriptyline has side effects common to anticholinergics which include


mydriasis.
1352. A 46yo male presents with confusion and drowsiness. What is the most
likely dx?
a. Cryptococcus neoformans
b. Toxoplasma gondii
c. HSV
d. CMV
e. Candida albicans
Headache, confusion, drowsiness can be caused by Cryptococcus
neoformans as well as HSV. But, neoformans is relatively rare and usually
seen in immunocompromised, so we should go with HSV
1353. A child has developed rash after the tx of penicillin. What will be the
cause of rash?
a. Drug reaction
b. Kawasaki
c. Inf Mono
Drug reaction

Mild to moderate allergic reactions to penicillin are common, and


symptoms may include any of the following:

Hives (raised, extremely itchy spots that come and go over a period
of hours)

Tissue swelling under the skin, typically around the face (also known
as angioedema)

Throat tightness.

Wheezing.

Coughing.
1354. A child comes with recurrent joint pain, multiple bruises, swollen ankle
and unable to move his legs. What is the inv of choice?
a. ESR
b. RF
c. Clotting factors
Clotting factor deficiency presents with hemarthromas.
1355. A 66yo man has renal colic. He has also presented with acute onset
pain in his knee in the past.
What is the single most likely cause for renal failure?
a. SLE associated GN
b. Hypercalcemia
c. HTN
d. Hyperuricemia
e. Hyperoxaluria
Gouty arthritis with renal stone

Whereas if hypercalcemia would have been a cause then renal stone .excessive thirst secondary
to dehydration with generalized bony pain .(SLE is excluded as it occurs in females 8 times more
than males & affects symmetrically small joints) So, ans would be hyperuricemia

1356. A boy with a hx of recurrent swollen tender joints on both knees and
elbows and not able to participate in sports. What is the inv of choice to dx?
a. RF/ASO titre
b. Clotting factor
c. ESR
Clotting factor deficiency presents with hemarthromas.
1357. A 26yo man is referred for gastroscopy because of a hx of several
months of dyspepsia. He has
routine bloods checked and is found to have a serum calcium level=3.2mmol/l
with a venous
bicarbonate level of 33mmol/l. Renal and LFT are both mornal. CXR is
normal. What is the most
likely cause of his hypercalcemia?
a. Melanoma
b. Metastatic malignancy
c. Milk alkali syndrome
d. Primary hyperparathyroidism
e. Sarcoidosis
Its milk or calcium alkali syndrome as there is hypercalcemia and metabolic
alkalosis.

Interpreting Laboratory Values in Hypercalcaemia[7]

Condition

Serum
Phosph
ate

Serum
Alkaline
Phosphata
se

Urine
Calciu
m

Urine
Phosph
ate

PTH

Hyperparathyroi
dism

Low

Normalhigh

High
(in
67%
of
patient
s)

High

High

Vitamin D
excess

Normalhigh

Low

High

High

Low

Malignancy

Often
low

High
(except in
haematolog
ical
malignancy
, when
normal)

Variab
le

High

Varia
ble

Granulomatous
disease

Normalhigh

Normalhigh

High

Normal

Low

Calcium alkali
syndrome

Normalhigh

Normal

Norma
l

Normal

Low

Familial
hypocalciuric
hypercalcemia

Normal
or low

Normal

Low
(<200
mg/da
y)

Normal

High

1358. A 3yo boy presents with difficulty in walking and skin lesions. What is
the most likely causative
agent?
a. Strep pyogenes
b. Rubella virus
c. Parvovirus
d. Papovirus
e. Paramyxovirus

Fifth disease (also called erythema infectiosum) is caused by


parvovirus B19. A human virus, parvovirus B19 is not the same

parvovirus that veterinarians may be concerned about in pets,


especially dogs, and it cannot be passed from humans to animals or
vice versa.
Fifth disease starts with a low-grade fever, headache, rash, and cold-like symptoms,
such as a runny or stuffy nose. These symptoms pass, then a few days later the
rash appears. The bright red rash most commonly appears in the face, particularly
the cheeks. This is a defining symptom of the infection in children (hence the name
"slapped cheek disease"). Occasionally the rash will extend over the bridge of the
nose or around the mouth. In addition to red cheeks, children often develop a red,
lacy rash on the rest of the body, with the upper arms, torso, and legs being the most
common locations. The rash typically lasts a couple of days and may itch; some
cases have been known to last for several weeks. Patients are usually no longer
infectious once the rash has appeared.[2][3]
Teenagers and adults may present with a self-limited arthritis. It manifests in painful
swelling of the joints that feels similar to arthritis. Older children and adults with fifth
disease may have difficulty in walking and in bending joints such as wrists, knees,
ankles, fingers, and shoulders.

1359. A pt after his house fire came with hematemesis with erosion/ulcer of
esophagus and on examination there is 55% burn and on endoscopy there is
a stomach/gastric erosion and soot in the mouth. What is the tx?
a. PO PPI
b. IV PPI
c. PPI and antibiotic
d. H. pylori test
e. Tracheal intubation

Tracheal intubation should be done to prevent airway blockage


secondary to laryngeal edema.

1360. A 40yo man complains of severe colicky loin pain that radiates to his
scrotum. He is noted to
have microscopic hematuria. No masses are palpated. What is the single
most likely cause?
a. Acute cystitis
b. Bladder ca
c. Renal vein thrombosis
d. Acute pyelonephritis
e. Ureteric calculus
Its E

Presentation :

Many stones are asymptomatic and discovered during investigations for

other conditions.

The classical features of renal colic are sudden severe pain. It is usually

caused by stones in the kidney, renal pelvis or ureter, causing dilatation,


stretching and spasm of the ureter. In most cases no cause is found:

Pain starts in the loin about the level of the costovertebral angle (but

sometimes lower) and moves to the groin, with tenderness of the loin or
renal angle, sometimes with haematuria.

If the stone is high and distends the renal capsule then pain will be in

the flank but as it moves down pain will move anteriorly and down towards
the groin.

A stone that is moving is often more painful than a stone that is static.

The pain radiates down to the testis, scrotum, labia or anterior thigh.

Whereas the pain of biliary or intestinal colic is intermittent, the pain of

renal colic is more constant but there are often periods of relief or just a dull
ache before it returns. The pain may change as the stone moves. The
patient is often able to point to the place of maximal pain and this has a
good correlation with the current site of the stone.

Investigations

Basic analysis should include:

Stick testing of urine for red cells (suggestive of urolithiasis), white cells

and nitrites (both suggestive of infection) and pH (pH above 7 suggests


urea-splitting organisms such as Proteusspp. whilst a pH below 5 suggests
uric acid stones).

Midstream specimen of urine for microscopy (pyuria suggests

infection), culture and sensitivities.

Blood for FBC, CRP, renal function, electrolytes, calcium, phosphate

and urate, creatinine.

Prothrombin time and international normalised ratio if intervention is

planned.

Non-enhanced CT scanning is now the imaging modality of choice and has

replaced intravenous pyelogram (IVP). Ultrasound scanning may be helpful to


[12]

differentiate radio-opaque from radiolucent stones and in detecting evidence of


obstruction.

Plain X-rays of the kidney, ureter and bladder (KUB) are useful in watching

the passage of radio-opaque stones (around 75% of stones are of calcium and
so will be radio-opaque).

The European Association of Urology's guidelines on urolithiasis

recommend stone analysis for:

All first-time stone formers.

All patients with recurrent stones who are on pharmacological

preventing therapy.

Patients who have had early recurrence after complete stone

clearance.

Late recurrence after a long stone-free period (stone composition may

change).
Encourage the patient to try to catch the stone for analysis. This may mean
urinating through a tea strainer, a filter paper such as a coffee filter or a gauze.

Management

[2]

Initial management can either be done as an inpatient or on an urgent outpatient


basis, usually depending on how easily the pain can be controlled.

Indications for hospital admission

Fever.

Solitary kidney.

Known non-functioning kidney.

Inadequate pain relief or persistent pain.

Inability to take adequate fluids due to nausea and vomiting.

Anuria.

Pregnancy.

Poor social support.

Inability to arrange urgent outpatient department follow-up.

People over the age of 60 years should be admitted if there are concerns on

clinical condition or diagnostic certainty (a leaking aortic aneurysm may present


with identical symptoms).

1361. A 55yo man is having slow growing ascites. When we tap the peritoneal
fluid the protein is <25 and it is clear and yellow. What could be the origin for
ascites?
a. Budd-Chiari
b. Gastrinoma
c. Hepatoma
d. TB
e. Pancreatitis
Budd Chiari = occlusion of the hepatic vein by, for eg hepatoma, causes transudative ascitis.
Such occlusion increases the hydrostatic pressure within the vessel, giving rise to the fluid shift
from the intra vascular compartment to the interstitium with consequent ASCITIS
If SAAG is more than 11g/L(exudative protein less than 25g/L) , fluid is termed as Transudate.
Causes of high SAAG ("transudate") are:
Cirrhosis - 81% (alcoholic in 65%, viral in 10%, cryptogenic in 6%)
Heart failure - 3%
Nephrotic syndrome
Hepatic venous occlusion: Budd-Chiari syndrome or veno-occlusive disease
Constrictive pericarditis
Kwashiorkor (childhood protein-energy malnutrition)

If SAAG is less than 11g/L(exudative protein more than 25g/L) , fluid is termed as exudate
Causes of low SAAG ("exudate") are:
Cancer (metastasis and primary peritoneal carcinomatosis) - 10%
Infection: Tuberculosis - 2% or Spontaneous bacterial peritonitis
Pancreatitis - 1%
Serositis
Hereditary angioedema
Other Rare causes:
Meigs syndrome
Vasculitis
Hypothyroidism
Renal dialysis
Peritoneum mesothelioma
Abdominal tuberculosis

1362. A 7yo boy presents with his mother to GP surgery. His mother
describes he had presented this since 3wks ago. He had not experienced any
trauma. No other symptoms a/w the condition. Exam: non tender swollen
ankles bilaterally. There is no rash or lesion. He is otherwise well. Which
single test would be the best as an initial assessment?
a. Plasma electrolytes
b. Albumin
c. Total serum protein
d. Anti-streptolysin
24 Hr urinary protein gold standard, if not given then opt for serum albumin..
Its nephrotic Albumin For nephrotic syndrome ..minimal change disease

1363. In lyme disease, which complication is most likely to lead to collapse?


a. Dilated CM
b. AV block
c. Mild encephalitis
d. Meningitis
e. Myocarditis
Ans is b
Lyme disease is caused by the spirochaete Borrelia burgdorferi and is spread by
ticks
Features

early: erythema chronicum migrans + systemic features (fever, arthralgia)


CVS: heart block, myocarditis
neuro: cranial nerve palsies, meningitis

Investigation

serology: antibodies to Borrelia burgdorferi

Management

doxycycline if early disease. Amoxicillin is an alternative if doxycycline is


contraindicated (e.g. pregnancy)
ceftriaxone if disseminated disease
Jarisch-Herxheimer reaction is sometimes seen after initiating therapy: fever,
rash, tachycardia after first dose of antibiotic (more commonly seen in
syphilis, another spirochaetal disease)

1364. A 30yo pt came to the OPD with complaint of breathlessness and dry
cough. He has lost 5kgs in 2m. He is an IV drug abuser. Inv: CXR=bilateral
interstitial shadowing. What is the single most likely causative organism?
a. Klebsiella
b. TB
c. Chlamydia pneumonia
d. PCP
e. Chlamydia psitacci
IV drug abuser is a clincher pointing towards HIV. Most common infections in
HIV pts are caused by PCP.

HIV: Pneumocystis jiroveci pneumonia


Whilst the organism Pneumocystis carinii is now referred to as Pneumocystis
jiroveci, the term Pneumocystis carinii pneumonia (PCP) is still in common use

Pneumocystis jiroveci is an unicellular eukaryote, generally classified as a


fungus but some authorities consider it a protozoa
PCP is the most common opportunistic infection in AIDS
all patients with a CD4 count < 200/mm should receive PCP prophylaxis

Features

dyspnoea
dry cough
fever
very few chest signs

Pneumothorax is a common complication of PCP.


Extrapulmonary manifestations are rare (1-2% of cases), may cause

hepatosplenomegaly
lymphadenopathy
choroid lesions

Investigation

CXR: typically shows bilateral interstitial pulmonary infiltrates but can present
with other x-ray findings e.g. lobar consolidation. May be normal
exercise-induced desaturation
sputum often fails to show PCP, bronchoalveolar lavage (BAL) often needed
to demonstrate PCP (silver stain shows characteristic cysts)

Management

co-trimoxazole
IV pentamidine in severe cases
steroids if hypoxic (if pO2 < 9.3kPa then steroids reduce risk of respiratory
failure by 50% and death by a third)

1365. A 27yo female who had a RTA 7m back now complaints of attacks of
sudden onset rotational
vertigo which comes on with sharp movements of the head and neck. Which
of the following
would be most helpful?
a. Caloric testing
b. Hallpikes maneuver
c. Gutenbergers test
d. Menieres test
e. Otoscopy

DixHallpike test. The DixHallpike test or NylenBarany test is a


diagnostic maneuver used to identify benign paroxysmal positional
vertigo (BPPV).
Benign paroxysmal positional vertigo (BPPV) is one of the most common causes of
vertigo encountered. It is characterised by the sudden onset of dizziness and vertigo
triggered by changes in head position. The average age of onset is 55 years and it is
less common in younger patients.
Features

vertigo triggered by change in head position (e.g. rolling over in bed or gazing
upwards)
may be associated with nausea
each episode typically lasts 10-20 seconds
positive Dix-Hallpike manoeuvre

BPPV has a good prognosis and usually resolves spontaneously after a few weeks
to months. Symptomatic relief may be gained by:

Epley manoeuvre (successful in around 80% of cases)


teaching the patient exercises they can do themselves at home, for example
Brandt-Daroff exercises

Medication is often prescribed (e.g. Betahistine) but it tends to be of limited value.

1366. A man rescued from a building on fire presented with unconsciousness


without any evidence of burns or external injury or soot. What would you do
next?
a. 100% oxyen inhalation
b. 24% oxygen by mask
c. Hyperbaric oxygen in a hyperbaric chamber
d. Intubation
e. Refer to specialist unit
I am confused about this one as we should always be following ABC first, but
rule of thumb for securing an airway is GCS less than 8 or a rapid falling GCS. And there is no
soot or burn, obstruction due to laryngeal edema is less likely. Unconcioussness does not mean a
low GCS too low that intubation is indicated. As it seems to be the case of CO poisoning, First
give him isobaric Oxygen via a facemask and calculate his GCS ( by giving painful stimuli etc )
and then try securing his airway via a LMA or Guedel's airway or an ETT if expertise is available

1367. A pt has had 1 ep of depression and 2 eps of mania over the last year
and now presents with depression. He is on anti-depressants. What additional
pharmacological tx would now act as a prophylaxis for his condition?
a. Antidepressants
b. Antipsychotics
c. Mood stabilizers
d. No additions req
Ans is C

Long-term treatment in secondary care to prevent


relapse or recurrence
After each acute episode of mania or bipolar depression, a discussion should be had
with the patient and/or carer about the nature and course of the disorder, treatment
options, the risk of relapse after stopping treatment and the risks and benefits of

pharmacological and psychological therapy. Risks may be particularly relevant in


women of child-bearing age. Factors to take into account include:

The severity and frequency of episodes.

Previous response to therapy.

Symptoms between episodes.

Relapse triggers, warning signs of relapse and coping strategies.

Potential length of treatment and review arrangements.

Provide clear written information about bipolar disorder, including NICE's information
for the public and ensure there is enough time to discuss options and concerns.:

[1]

Options available include:

Pharmacological.

Lithium should be considered first-line, with the addition of valproate if

ineffective.

Valproate or olanzapine should be considered for patients intolerant of

lithium or who are not prepared to undergo regular monitoring.

If symptoms still continue then the patient should be referred to a

mental health specialist. Medications that might be used in this situation


are lamotrigine (especially in bipolar II disorder) or carbamazepine.

Lithium will require monitoring of levels and monitoring of renal function

and thyroid function. Patients need to be advised of adequate rehydration


and the dangers of suddenly stopping treatment.

Long-term therapy usually continues for two years but may be needed

for as long as five years.

If medication is stopped, patients should be made aware of early

warning symptoms of recurrence. Medication should be tailed off gradually


(unless acute toxicity develops). Mood should be monitored for two years
after treatment is stopped.

Cognitive behavioural therapy, interpersonal therapy or behavioural couples

therapy may be appropriate.

NICE provides a link to an evidence-based manual to a psychological

intervention that has been developed specifically for bipolar disorder .

Psychosocial education is beneficial. Various methods are available,

including teaching coping strategies and managing communication difficulties.


Psychosocial interventions are particularly important for paediatric and
adolescent patients, to provide families with an understanding of symptoms,
course, and treatment.

1368. A man presented with a purplish swelling at the anal area. It is acutely
painful and he complains of constipation for the last 2m. What is the most
appropriate management?
a. I&D
b. I&D + antibiotics
c. Reassure
d. Analgesia
e. Sclerotherapy
Ans is B
Piles (haemorrhoids) and perianal haematoma
Anal pain can sometimes be caused by piles or a perianal haematoma (burst blood vessel under
the skin at the edge of the anus).
Piles can become painful when they become "strangulated" and bulge outside the anus,
developing a blood clot. They are usually treated with painkillers, ointments and sometimes ice
packs, although surgery is occasionally needed.

A perianal haematoma is usually relieved by a simple procedure to remove the clot using a local
anaesthetic, sometimes done by your GP and then antibiotics are given

1369. A pt came to the ED after he had banged his car quite a few times on
reversing. He was complaining of seeing double while he tried to look back
during the process of reversing the car, he also complains of double vision on
looking at an outward gaze. Which nerve is involved?
a. Abducent nerve
b. Trochlear nerve
c. Oculomotor nerve
d. Optic nerve
e. Trigeminal nerve

External Ocular Paralysis

Muscle

Direction of pull

Result of paralysis

Cranial nerve

Medial rectus

Medially

Lateral

III

Superior rectus

Upwards

Downwards

III

Lateral rectus

Laterally

Medial

VI

Inferior rectus

Downwards

Upwards

III

Superior oblique

Down and out

Up and in

IV

Inferior oblique

Up and out

Down and in

III

Ans is abducens nerve


1370.. A pt had a stroke Now, there is left sided weakness and right side
facial numbness. CT shows ischemic stroke. Which one would you prescribe?
a. Alteplase
b. Aspirin
c. Clopidogrel
d. Heparin
e. Warfarin
Ans is A
The Royal College of Physicians (RCP) published guidelines on the diagnosis and
management of patients following a stroke in 2004. NICE also issued stroke guidelines in
2008, although they modified their guidance with respect to antiplatelet therapy in 2010.
Selected points relating to the management of acute stroke include:

blood glucose, hydration, oxygen saturation and temperature should be maintained


within normal limits
blood pressure should not be lowered in the acute phase unless there are
complications e.g. Hypertensive encephalopathy*
aspirin 300mg orally or rectally should be given as soon as possible if a
haemorrhagic stroke has been excluded
with regards to atrial fibrillation, the RCP state: 'anticoagulants should not be started
until brain imaging has excluded haemorrhage, and usually not until 14 days have
passed from the onset of an ischaemic stroke'
if the cholesterol is > 3.5 mmol/l patients should be commenced on a statin. Many
physicians will delay treatment until after at least 48 hours due to the risk of
haemorrhagic transformation

Thrombolysis
Thrombolysis should only be given if:

it is administered within 4.5 hours of onset of stroke symptoms (unless as part of a


clinical trial)
haemorrhage has been definitively excluded (i.e. Imaging has been performed)

Alteplase is currently recommended by NICE.

1371. A young boy presents with fever and cough. His father was dx with TB
a week ago. The parents dont want him to have a BAL under anesthesia.
Which other samples can be taken for dx?
a. Urine
b. Blood
c. CSF
d. Gastric washing
e. Sweat

Ans D
Coz goblet cells secrete mucins n they r found in airways n stomach too.

1372. A 50yo man came to the hosp a few months after he had a MI. Exam:
everything normal, S1 and S2 were heard on auscultation, but there is a new
pan-sytolic murmur. What is the most appropriate inv of choice?
a. ECG
b. 24h ECG
c. Echo
d. CXR
e. CT
Holosystolic (pansystolic)

mitral/tricuspid regurgitation (high-pitched and 'blowing' in character)


VSD ('harsh' in character)
Inv of choice would be ECHO

1373. A 73yo stroke pt has been on aspirin for 2yrs. He now presents with
epigastric pain and is asking for a tx. What is the most appropriate
management?
a. Laparotomy
b. NSAIDs
c. Omeprazole
Morphine
d. Tramadol
PPIs should be introduced in cases of discomfort by the prophylactic use of
NSAIDS.
1374. A 2yo girl is brought to the ED by her mother. The child is screaming
that there is something in
her ear and she appears agitated. Exam: a plastic bead is seen inside the
ear. What is the best
method of removal?
a. Forceps
b. Hook
c. Under general anesthesia
d. Syringing
e. Magnet
Ans is C as the kid is irritable.

Presentation

Most older children and adults will know that there is something in their ear

but sometimes a foreign body may get into the external ear canal without the
patient realising.

The patient may present with pain, deafness or discharge. Live insects may

cause a buzzing in the ear.

The appearance will vary according to the object and length of time it has

been in the ear:

An inanimate object that has been in the ear a very short time presents

with no abnormal finding other than the object itself.

Pain or bleeding may occur with objects that abrade the ear canal, from

rupture of the tympanic membrane, or from the patient's attempts to


remove the object.

With delayed presentation, erythema and swelling of the canal and a

foul-smelling discharge may be present.

Management
A great deal of care is required in order not to push the object deeper into the ear
canal and not to damage the ear canal. There is a high failure rate in removal of
foreign bodies from the ear.

Insects should be killed prior to removal, using 2% lidocaine.

Remove batteries or magnets as soon as possible to prevent corrosion or

burns. Do not crush a battery during removal.

Adhesives (eg, Super Glue) may be removed manually within 1-2 days

once desquamation has occurred. Referral to an ear, nose and throat specialist
is required if an adhesive is in contact with the tympanic membrane.

Methods for removal

Forceps or hook: grasp the object with forceps, or place a hook behind the

object and pull it out.

Irrigation is often effective. Irrigation with water is contra-indicated for soft

objects, organic matter or seeds (which may swell and increase the level of pain
and difficulty to remove if exposed to water).

Suction with a small catheter held in contact with the object may be

effective.

1375. During antenatal visits, the following tests are routinely offered to all
pregnant mothers apart
from HIV and Hep B?
a. Rubella and syphilis
b. Syphilis and toxoplasmosis
c. Hep C & thalassemia
d. CMV and rubella
e. Sickle cell anemia and Hep
Gestation

Purpose of visit

8 - 12 weeks (ideally
< 10 weeks)

Booking visit

general information e.g. diet, alcohol, smoking,


folic acid, vitamin D, antenatal classes
BP, urine dipstick, check BMI
Booking bloods/urine

FBC, blood group, rhesus status, red cell


alloantibodies, haemoglobinopathies
hepatitis B, syphilis, rubella
HIV test is offered to all women
urine culture to detect asymptomatic bacteriuria

10 - 13+6 weeks

Early scan to confirm dates, exclude multiple pregnancy

11 - 13+6 weeks

Down's syndrome screening including nuchal scan

16 weeks

Information on the anomaly and the blood results. If Hb <


11 g/dl consider iron
Routine care: BP and urine dipstick

18 - 20+6 weeks

Anomaly scan

25 weeks (only if
primip)

Routine care: BP, urine dipstick, symphysis-fundal height


(SFH)

28 weeks

Routine care: BP, urine dipstick, SFH


Second screen for anaemia and atypical red cell
alloantibodies. If Hb < 10.5 g/dl consider iron
First dose of anti-D prophylaxis to rhesus negative women

31 weeks (only if
primip)

Routine care as above

34 weeks

Routine care as above


Second dose of anti-D prophylaxis to rhesus negative
women*
Information on labour and birth plan

36 weeks

Routine care as above


Check presentation - offer external cephalic version if
indicated
Information on breast feeding, vitamin K, 'baby-blues'

38 weeks

Routine care as above

40 weeks (only if
primip)

Routine care as above


Discussion about options for prolonged pregnancy

41 weeks

Routine care as above


Discuss labour plans and possibility of induction

1376. A 32yo male complains of tremors everytime he tends to use his


muscles and when he is pointing at objects. No complaints at rest. His father
complained of similar problems. What is
the most probable dx?
a. Parkinsonism
b. Lithium toxicity
c. Thyrotoxicosis
d. Benign essential tremor
Essential tremor (previously called benign essential tremor) is an autosomal
dominant condition which usually affects both upper limbs
Features

postural tremor: worse if arms outstretched


improved by alcohol and rest
most common cause of titubation (head tremor)

Management

propranolol is first-line
primidone is sometimes used

1377. A 40yo woman with breast cancer has back pain which keep her awake
at night. She blames it on a gym session she had 2wks ago. She now has
difficulty in climbing stairs. There is tenderness over the right thoracic spine.
She has diminished fine touch and temp sensation in her right foot.
What is the single most appropriate inv?
a. Bone density scan
b. CT head
c. MRI spine
d. Nuclear bone scan
e. XR thoracolumbar spine
This was a case of spine metastasis (vertebra) and stress during exercising in gym caused
collapsing fracture of vertebra which lead to spinal cord compression. X-ray will only show
vertebral fracture and not the compression. As there is neuro features so to deliniate the spinal
cord compression we have to do MRI.

1378. A pregnant lady at her 39wk GA present with eclampsia. Soon after her
arrival in the labour suit, IV MgSO4 and IV hydralazine has been prescribed.
The pt then develops another fit in the hosp and maintenance dose of MgSO4
has been started. What is your next step in management?
a. Mg SO4 bolus
b. Delivery of baby
c. MgSO4 loading dose
d. Diazepam
Definitive tx of Eclampsia is delivery. Pt had already been given mgso4 twice
so we should now go for delivery.

Management of eclampsia

Resuscitation:

The patient should be placed in the left lateral position and the airway

secured.

Oxygen should be administered.

Treatment and prophylaxis of seizures:

Magnesium sulfate is the anticonvulsant drug of choice.

Intubation may become necessary in women with repeated seizures in

order to protect the airway and ensure adequate oxygenation.

Treatment of hypertension:

Reduction of severe hypertension (blood pressure >160/110 mm Hg or

mean arterial pressure >125 mm Hg) is essential to reduce the risk of


cerebrovascular accident. Treatment may also reduce the risk of further
seizures.

Intravenous hydralazine or labetalol are the two most commonly used

drugs. Both may precipitate fetal distress and therefore continuous fetal
heart rate monitoring is necessary.

Fluid therapy:

Close monitoring of fluid intake and urine output is mandatory.

Pre-loading the circulation with 400-500 ml colloid prior to regional

anaesthesia or vasodilatation with hydralazine may reduce the risk of


hypotension and fetal distress.

Delivery:

The definitive treatment of eclampsia is delivery. Attempts to

prolong pregnancy in order to improve fetal maturity are unlikely to be of


value.

However, it is unsafe to deliver the baby of an unstable mother even if

there is fetal distress. Once seizures are controlled, severe hypertension


treated and hypoxia corrected, delivery can be expedited.

Vaginal delivery should be considered but Caesarean section is likely

to be required in primigravidae, well before term and with an unfavourable


cervix.

After delivery, high-dependency care should be continued for a

minimum of 24 hours.
All patients need careful follow-up and a formal postnatal review to establish if there
is chronic hypertension, proteinuria or liver damage

1379. A man suffering from Influenza A since 5d ago. CXR: pneumonia. What
organism is responsible for pneumonia in this pt?

a. Hemophilius influenze
b. Klebsiella
c. Staphylococcus aureus
d. Streptococcus pneumonia
e. Pseudomonas
Ans would be staphylococcus aureus
1380. A pt admitted due to repeated attacks of pancreatitis presents with
dementia and loss of proprioception in the legs. What is the most appropriate
tx?
a. Thiamine
b. Pyridoxine
c. Cobolamin
d. Lipase
e. Antibiotics
Ans is C
Pancreatic enzymes in the duodenum cleave off the R-binders(which come from salivary glads)
from Vit-B12, which then combines with IF and this complex is absorbed in terminal ileum.
Pancreactic enzyme deficiency therefore leads to impaired absorption of Vit B12 and sub-acute
combined degeneration of spinal cord
.
Alcoholic + pancreatitis more in favor of B12
Alcoholic only think of thiamine

1381. A man after MI presented with sudden breathlessness and dyspnea.


Exam: scattered pansystolic murmur all over the precordium. What is the next
inv that will lead to dx?
a. ECG
b. Echo
c. CT
d. Blood culture
e. CXR
We would go for echo !!!!

1382. During a laparoscopic procedure, a trochar is inserted halfway between


the umbilicus and the ant superior iliac spine. What are the structures most
likely to be pierced?
a. Rectus sheath
b. Linea alba
c. External oblique aponeurosis
d. Internal oblique and transverse abdominal
e. Both C and D
There is a confusion regarding the answer of this question. But, I think the
answer should be E considering the insertions of ext oblique.
1383. A pt, a small child presented with URTI and later developed fever,
earache and tympanic membrane is dull. What is the likely dx?
a. OM
b. OE
c. Glue ear
d. Perforation of the tympanic membrane
e. Referred ear ache
Ans is A
The answer is A. Acute OM without perforation presents as such. The pus collected in the middle
ear causes the TM to lose its pearly white colour and appear dull or sometimes yellow. And it is
usually associated with pain. Glue ear presents with all the above mentioned points except the
fever. Here fever developed afterwards whereas glue ear usually follows a viral urti.
The differentiating point here is the presentation ..
AOM .. Otalgia ..
OME .. Hearing impairment noticed by parents is the mode of presentation in 80%.

1384. A 72yo male who is a regular smoker has come to the ED with
complaints of loss of weight and loss of appetite. He also complains of
odynophagia. Exam: actively bleeding ulcer on right tonsil. What is the most
appropriate dx?
a. Tonsillar ca
b. Vincents angina
c. Irritant ingestion
d. Paracoccidiodmycosis
e. Herpes simplex infection
More than 70% of tonsillar cancers are SCC. Most of the others are lymphomas.
Metastases to the palatine tonsils are rare but there have been reports of
secondaries from breast, lung, renal, pancreatic and colorectal malignancies.

Patients with tonsillar carcinomas may present with a neck mass, usually in

the jugulodigastric region. Even if the neck mass is not evident on casual
inspection, careful palpation may reveal cervical lymphadenopathy.

Sore throat, ear pain, foreign body or mass sensation, and bleeding may

occur.

Trismus is an ominous sign because it probably indicates involvement of the

parapharyngeal space. Such tumours may be large enough to involve or encase


the carotid sheath. In addition, the tumour may extend to the skull or
mediastinum.

If the tumour has involved the tongue base, contralateral nodes may be

involved.

Primary tonsillar tumours may grow entirely beneath the surface. The

clinician may therefore see nothing suspicious or may see only a slight increase
in the size of the tonsil or the firmness of the area.

Alternatively, an exophytic fungating mass with central ulceration and

heaped-up edges may be present. It may be deep red to white.

Weight loss and fatigue are not uncommon.

Treatment may lead to pain, xerostomia, infections, poor wound healing,

dysphagia, fistula formation, trismus, potential disfigurement and fatigue.

1385. A pt with regular episodes of SNHL, vertigo and tinnitus lasting >30min.
Neurological
exam=normal. What is the likely dx?
a. Menieres disease
b. Acoustic neuroma
c. Otosclerosis
d. Benign positional vertigo
e. Labrynthitis
Meniere's disease is a disorder of the inner ear of unknown cause. It is characterised
by excessive pressure and progressive dilation of the endolymphatic system. It is
more common in middle-aged adults but may be seen at any age. Meniere's disease
has a similar prevalence in both men and women.
Features

recurrent episodes of vertigo, tinnitus and hearing loss (sensorineural).


Vertigo is usually the prominent symptom
a sensation of aural fullness or pressure is now recognised as being common
other features include nystagmus and a positive Romberg test
episodes last minutes to hours
typically symptoms are unilateral but bilateral symptoms may develop after a
number of years

Natural history

symptoms resolve in the majority of patients after 5-10 years


the majority of patients will be left with a degree of hearing loss
psychological distress is common

Management

ENT assessment is required to confirm the diagnosis


patients should inform the DVLA. The current advice is to cease driving until
satisfactory control of symptoms is achieved
acute attacks: buccal or intramuscular prochlorperazine. Admission is
sometimes required
prevention: betahistine may be of benefit

1386. A pt with celiac disease from birth, now as an adult presented with
some abdominal symptoms. The biopsy shows infiltration of the gastric
epithelium by lymphocytes. What is the most likely dx?
a. Lymphoma
b. Diverticular disease
c. Lynch syndrome
d. Gastric TB
e. Peritoneal tumor

Lympho because that's the main complication of Celiac disease and in biopsy v can see
lymphocytic infiltrates. Pts with celiac diasease are at a riskt of developing MALT
lymphoma (enteropathy-associated T-cell lymphoma of small intestine)

1387. A 55yo man presented with hot, raised, tender area of skin on his right
leg. He is febrile with rigors. He has been started on flucloxacillin. What other
meds will you add?
a. Ciprofloxacin
b. Gentamicin
c. Metronidazole
d. Benzylpenicillin
e. Ceftriaxone
Cellulitis is a term used to describe an inflammation of the skin and subcutaneous
tissues, typically due to infection by Streptococcus pyogenes or Staphylcoccus
aureus.
Features

commonly occurs on the shins

erythema, pain, swelling


there may be some associated systemic upset such as fever

Management
The BNF recommends flucloxacillin as first-line treatment for mild/moderate
cellulitis. Clarithromycin or clindamycin is recommend in patients allergic to penicillin.
Many local protocols now suggest the use of oral clindamycin in patients who have
failed to respond to flucloxacillin.
Severe cellulitis should be treated with
flucloxacillin.

intravenous benzylpenicillin +

1388. A 65yo man has incurable bronchial cancer. He is unable to cough up


his secretions. This is leading to a distressing cough. Which of the following
drugs is most likely to help him?
a. Scopolamine
b. Xanomeline
c. Aceclidine
d. Pilocarpine
e. Cevimiline
Ans A
Scopolamine : Antisecretory
Xanomeline : Trial in Alzheimers and Pscizophrenia
Aceclidine : tx of narrow angle glaucoma
Cevimiline : Dry mouth
1389. A pt presented after eating a seafood dish at a local restaurant. He
complains of difficulty in breathing. His speech is slurred and his
BP=85/55mmHg. What would be the most appropriate next step?
a. IV adrenaline
b. IM adrenaline
c. SC adrenaline
d. PO chlorpheniramine
e. IV chlorpheniramine
Anaphylaxis may be defined as a severe, life-threatening, generalised or systemic
hypersensitivity reaction.
Anaphylaxis is one of the few times when you would not have time to look up the
dose of a medication. The Resuscitation Council guidelines on anaphylaxis have
recently been updated. Adrenaline is by far the most important drug in anaphylaxis
and should be given as soon as possible. The recommended doses for adrenaline,
hydrocortisone and chlorphenamine are as follows:
Adrenaline

Hydrocortisone

Chlorphenamine

< 6 months

150 micrograms
(0.15ml 1 in 1,000)

25 mg

250
micrograms/kg

6 months - 6
years

150 micrograms
(0.15ml 1 in 1,000)

50 mg

2.5 mg

6-12 years

300 micrograms
(0.3ml 1 in 1,000)

100 mg

5 mg

Adult and child


> 12 years

500 micrograms
(0.5ml 1 in 1,000)

200 mg

10 mg

Adrenaline can be repeated every 5 minutes if necessary. The best site for IM
injection is the anterolateral aspect of the middle third of the thigh.
Common identified causes of anaphylaxis

food (e.g. Nuts) - the most common cause in children


drugs
venom (e.g. Wasp sting)

1390. A 7yo boy presents with proptosis and periorbital edema. What is the
immediate action that needs to be taken?
a. IV morphine and immediate ophthalmoscopy
b. IV morphine
c. Observation only
Seems to be an incomplete recall. Considering the age, and unilateral
problem, diagnosis may be unilateral orbital cellulitis. Ans seems to be A
Presentation :
Sudden onset of unilateral swelling of conjunctiva and lids.
Proptosis (bulging of the eye).
Pain with movement of the eye, restriction of eye movements.
Blurred vision, reduced visual acuity, diplopia.
Pupil reactions may be abnormal - relative afferent pupillary defect
(RAPD); see the separate article on Examination of the Eye.
Fever, severe malaise.
Management :
Hospital admission under the joint care of the ophthalmologists and the
ENT surgeons is mandatory.
Intravenous antibiotics are used (eg, cefotaxime and flucloxacillin) in
addition to metronidazole in patients over 10 years old with chronic
sinonasal disease.[3]

Clindamycin plus a quinolone such as ciprofloxacin are used where


there is penicillin sensitivity. Vancomycin is also an alternative.
Optic nerve function is monitored every four hours (pupillary reactions,
visual acuity, colour vision and light brightness appreciation).
Treatment may be modified according to microbiology results and lasts
for 7-10 days.
Surgery is indicated where there is CT evidence of an orbital collection,
where there is no response to antibiotic treatment, where visual acuity
decreases and where there is an atypical picture which may warrant a
diagnostic biopsy. Surgery often concurrently warrants drainage of
infected sinuses
1391. A schizophrenic man complains that he can hear voices talking about
him and telling him to end
his life by cutting his throat. He only hears them when he wakes up from sleep
and not at other
times. What type of hallucinations is he having?
a. Somatic
b. Kinesthetic
c. Hypnogogic
d. Hypnopompic
e. Lilliputian
Hypnagogic - occur on falling asleep and are harmless.
Hypnopompic - occur on waking up and are harmless.
Auditory - of one or more talking voices; seen commonly in
schizophrenia.

Charles Bonnet's syndrome - visual hallucinations that blind persons


experience

1392. A 28yo woman complains of hearing strange voices in her bedroom as


she is falling asleep in the night. She says there is no one in the room except
her. On evaluation she has no other problems. What is she suffering from?
a. Delusion of persecution
b. Cotard syndrome
c. Hypnogogic hallucinations
d. Lilliputian hallucinations
e. Schizophrenia
Types of Hallucinations :
Hypnagogic - occur on falling asleep and are harmless.
Hypnopompic - occur on waking up and are harmless.
Auditory - of one or more talking voices; seen commonly in
schizophrenia.
Charles Bonnet's syndrome - visual hallucinations that blind persons
experience

1393. A 32yo man on psychiatric meds presents with coarse tremors and
diarrhea. What is the most likely altnernate to the drug causing the prb?
a. Lithium
b. Diazepam
c. Haloperidol
d. Valproate
e. Citalopram
Seems that Lithium has caused the symptoms. Its adverse effects are :
G I upset.
Coarse tremors
Hypo/hyper thyroidism
Diabetes insipidus
Ebstein anomaly(in foetus)
Lithium is used for bipolar disorder, alternatively we can give valproate (2nd
line)
1394. A man is brought to the ED after he was stabbed in the best. Chest is
clear bilaterally with muffled heart sounds. BP=60/0mmHg, pulse=120bpm,
JVP is raised. What is the most probable dx?
a. Pulmonary embolism
b. Cardiac tamponade
c. Pericardial effusion
d. Hemothorax
e. Pneumothorax

Cardiac tamponade
Features
dyspnoea
raised JVP, with an absent Y descent - this is due to the limited right
ventricular filling
tachycardia
hypotension
muffled heart sounds
Kussmaul's sign (much debate about this)
ECG: electrical alternans
Pulsus Paradoxus
1395. A 64yo alcoholic who has been dx with liver cirrhosis presents with a
massive ascites. What is
the mechanism of fluid accumulation in a pt with liver disease?
a. Cirrhosis
b. Portal HTN
c. Hypoalbuminemia
d. Liver failure
e. Hepatic encephalopathy
This question asks about the cause of a broader term - fluid accumulation which is mainly mediated by hypoalbuminemia. Liver cirrhosis is the final

stage of liver disease and hypoalbuminemia can be noticed prior to that


stage.
1396. A 38yo man presented to ED with severe pain in upper abdomen. He
has already taken course of
triple therapy and now had elective endoscopy 2d ago. He is in shock. What
is the most
probable dx?
a. Ca esophagus
b. Barrets esophagus
c. Mediastinitis
d. Ca stomach
Always suspect mediastinitis in a pt with shock and fever after a hx of
- Recent cardiothoracic surgery or instrumentation.
- Upper GI endoscopy.
- Bronchoscopy.
- Recent dental or oropharyngeal infection.
- Upper respiratory tract infection
- Ingestion of a foreign body (particularly button batteries by young children,
which may cause oesophageal rupture).
other signs may include edema of the neck and face , and crunching sound
when auscultation of the heart
Surgical referral is urgent
Boerhaave's syndrome

The term Boerhaave's syndrome is reserved for the 10% of esophageal perforations
which occur due to vomiting

This can occur due to with Alcoholics who drink a lot and then vomit forcefully
, this may cause esophageal perforation thereafter mediastinitis followed by
death .....
MalloryWeiss , also known as , gastro-esophageal laceration syndrome
refers to bleeding from tears ... rather than perforation .....
1397. A 68yo man who is a known case of liver cirrhosis has developed
ascites. What is the mechanism for the development of ascites?
a. Portal HTN
b. Hypoalbuminemia
c. Congestive heart failure
d. Liver failure
Ans can be A or B. Trigger factor for ascities is splancnic vasodilation
due to PHT. Hypoalbuminemia plays a vital role too. According to ohcm pg
260, Portal HTN is mentioned as a cause of ascities. Mechanism of "fluid
accumulation" (BROADER TERM ) is hypoalbuminaemia and ascites is portal HTN

1398. A man feels mild discomfort in the anal region and purulent discharge in
underpants. What is the most likely dx?
a. Feacal incontinence
b. Anal abscess
c. Fistula in ano

d. Anal tags
e. Rectal Ca
Fistula in ano is commonly seen in otherwise fit, young males. Associations:
crohn's, diabetes, obesity Causes : perianal sepsis, crohns. TB, Rectal CA.
Immunocompromise
Inv : Endoanal USS
In anal abscess- there would be severe pain & fever. Abscess is confined in
the cavity and is very painful. Rectal ca would have been presented with
bleeding or feeling of imcomplete defecation. . Some fistulla are painless
specially if old.
1399. A 38yo female presents with difficulty in looking upward and on
examination she was found to have lid lag as well. She also complains of her
heart racing at times. Which test will help in dx?
a. Tensilon test
b. 24h ECG
c. TFT
d. Schimmer test
e. Young Helmholtz ophthalmoscopy
Hyperthyroidism
Causes
Graves' disease
toxic nodular goitre
subacute (de Quervain's) thyroiditis
post-partum thyroiditis
acute phase of Hashimoto's thyroiditis (later results in hypothyroidism)
toxic adenoma (Plummer's disease)
amiodarone therapy
Investigation
TFTS (TSH down, T4 and T3 up)
thyroid autoantibodies
other investigations are not routinely done but includes isotope
scanning

1400. A young anxious mother of a 10m boy comes to you and requests a
test for CF. What is the most
appropriate inv?
a. Sweat test
b. Heel prick test
c. Breath test
d. CXR
CF is an autosomal recessive disease caused by mutations in the CF
transmembrane conductance regulator (CFTR) gene, on chromosome 7. The only
risk factor is a family history of the condition.

Signs
These may include:

Finger clubbing.
Cough with purulent sputum.
Crackles.
Wheezes (mainly in the upper lobes).
Forced expiratory volume in one second (FEV1) showing obstruction.

Babies diagnosed with CF will usually have no signs or symptoms.

Investigations
Sweat testing confirms the diagnosis and is 98% sensitive.
Chloride concentration >60 mmol/L with sodium concentration lower than
that of chloride on two separate occasions.

Molecular genetic testing for CFTR gene.

Sinus X-ray or CT scan - opacification of the sinuses is present in


almost all patients with CF.

CXR or CT of thorax.

Lung function testing - spirometry is unreliable before 6 years.

Sputum microbiology - common pathogens include Haemophilus


influenzae,Staphylococcus aureus, Pseudomonas aeruginosa,
Burkholderia cepacia, Escherichia coli and Klebsiella pneumoniae.

Various blood tests including FBC, U&Es, fasting glucose, LFTs and
vitamin A, D and E levels are usually performed.

Management of cystic fibrosis involves a multidisciplinary approach


Key points

regular (at least twice daily) chest physiotherapy and postural drainage.
Parents are usually taught to do this. Deep breathing exercises are also
useful
high calorie diet, including high fat intake*
vitamin supplementation
pancreatic enzyme supplements taken with meals
heart and lung transplant

1400. A young anxious mother of a 10m boy comes to you and requests a test for CF.
What is the most appropriate inv?
a. Sweat test
b. Heel prick test
c. Breath test
d. CXR
Answer= A- sweat test (Sweat testing confirms the diagnosis of cystic fibrosis and is 98%
sensitive)
exclusion of other options:
Heel prick test= this test is usually done on the 5th to 6th day of life. A blood spot is taken
from babys heel it is done for screening diseases like sickle cell disease, cystic fibrosis,
congenital hypothyroidism, phenylketonuria, homocystinuria etc.In cystic fibrosis the heel
prick test detects a chemical called immunoreactive trypsinogen.
CXR=not diagnostic in CF
Cystic Fibrosis:
CF is an autosomal recessive disease caused by mutations in the CF transmembrane
conductance regulator (CFTR) gene, on chromosome 7.
CFTR is an ATP-responsive chloride channel that also affects other cellular activities, such
as sodium transport across the respiratory epithelium, composition of cell surface
glycoprotein and antibacterial defences.
Clinical features:
neonates= failure to thrive,meconium ileus,rectal prolapse
children and adults= respiratory: recurrent chest infections ( recurrent lower respiratory tract
infection (LRTI) with chronic sputum production is the most common presentation)
GIT: pancreatic insufficiency, gallstones, cirrhosis etc
others= male infertility,vasculitis,nasal polyps,arthritis,osteoporosis,hypertrophic pulmonary
osteoarthropathy
Signs= finger clubbing,cyanosis,bilateral coarse crackles
Investigations=
DIAGNOSIS by SWEAT TEST= . Chloride concentration >60 mmol/L with sodium
concentration lower than that of chloride on two separate occasions.
Molecular genetic testing for CFTR gene
Sputum microbiology - common pathogens include Haemophilus influenzae,Staphylococcus
aureus, Pseudomonas aeruginosa, Burkholderia cepacia, Escherichia coli and Klebsiella
pneumoniae.
Treatment=
Patient care is most effective when provided in specialist centres by multidisciplinary teams.
symptomatic treatment for instance, respiratory= chest physiotherapy
GIT= pancreatic enzyme replacement, fat sol vitamin supplementation etc

1401. A 22yo Greek man presents with rapid anemia and jaundice following tx of
malaria. He is noted to have Heinx bodies. Choose the single most likely cause from the
given options?
a. G6PD deficiency
b. Anemia of chronic disease
c. Pernicious anemia
d. IDA
e. Vit B12 deficiency

Answer= Glucose 6 phosphate dehydrogenase deficiency (G6PD). Acute haemolysis


from G6PD deficiency can produce HEINZ BODIES which are denatured haemoglobin and
bite cells. it is precipitated by drugs such as primaquine (antimalarial),sulfonamides,aspirin.
exclusion of other causes=
anemia of chronic disease: commonest anemia in hospital patients.common causes include
Chronic infection,Inflammation - including connective tissue disorders,Neoplasia.
pernicious Anemia=type of megaloblastic anemia due to impaired absorption of vitamin b12
deficiency.it is caused by autoimmune atrophic gastritis leading to acchlorhydria and lack of
intrinsic factor secretion.
Iron def Anemia=microcytic hypochromic anemia caused by iron deficiency d/t blood loss,
poor diet,malabsorption etc. blood film shows anisocytosis and poikilocytosis
vit b12 def= macrocytic anemia caused by vit b12 deficiency d/t dietary def,pernicious
anemia,ileal resection,gastrectomy.Blood film will show hypersegmented polymorphs
G6PD is x linked chief rbc enzyme defect disorder. it mainly affects males. it is percipitated
by drugs like primaquine,sulfonamides and aspirin.Usually asymptomatic.In attacks,there is
rapid anemia and jaundice.
tests: fbc= anemia
Blood film= heinz bodies
G6PD enzyme activity - is the definitive test (as opposed to the amount of G6PD protein).
Treatment= Avoidance of the substances that may precipitate hemolysis is essential. If
severe then transfuse. if severe hemolysis, folate supplementation may be beneficial.

1402. A 65yo has terminal cancer and his pain is relieved by a fentanyl patch but he now
complains of shooting pain in his arm. Which of the following will add to his pain relief?
a. Gabapentin
b. Radiotherapy
c. Amitriptyline
d. Morphine
answer=A. Gabapentin.
it is the neuropathic pain and the first line treatment for neuropathic pain according to
recent nice guidelines is amitriptyline, duloxetine, gabapentin,pregabalin.if the first line
drug treatment doesn't work try one of the other 3 drugs
Gabapentin is 1st line for neuropathic pain. Amitriptyline is 1st line for diabetic
neuropathy but it is off license so now the first line for diabetic neuropathy is duloxetine. .
Neuropathic pain is defined as pain arising as a direct consequence of a lesion or disease
affecting the somatosensory system. it is often difficult to treat and responds poorly to
standard analgesia.
The discomfort is usually of a chronic nature and may be described by the patient as a
burning sensation, a sharp, stabbing or shooting pain, or 'like an electric shock'.
examples include= diabetic neuropathy,trigeminal neuralgia,post herpetic neuralgia,
prolapsed intervertebral discs. There are various causes of it including infections like guillain
barr syndrome, hiv etc, drugs like isoniazid, vincristine, cisplatin,nitrofurantoine etc,
malignancy.

treatment= pharmacological
All neuropathic pain (except trigeminal neuralgia): Offer a choice of amitriptyline, duloxetine,
gabapentin or pregabalin as initial treatment for neuropathic pain (except trigeminal
neuralgia). If the initial treatment is not effective or is not tolerated, offer one of the remaining
three drugs, and consider switching again if the second and third drugs tried are also not
effective or not tolerated.

1403. A 45yo male alcoholic presents after a large hematemesis. He has some spider
naevi on his chest, BP=100/76mmHg, pulse=110bpm. He has a swollen abdomen with
shifting dullness.
a. Gastric ca
b. Mallory-weiss tear+
c. Esophageal ca
d. Esophageal varices
e. Esophagitis
f. Peptic ulceration
Answer= esophageal varices. The most common causes of upper GI bleeding are peptic
ulcer and oesophago-gastric varices.Factors that increase the risk of variceal bleeding are
the decompensation of liver disease (ascites,bleeding esophageal varices,oedema
etc),alcohol intake,aspirin,nsaids etc spider naevi is also one of the signs of chronic liver
disease.
exclusion of other causes:
peptic ulceration= although the most common causes of upper GI bleeding are peptic ulcer
and oesophago-gastric varices but Helicobacter pylori infection is associated with about 95%
of duodenal ulcers and 80% of gastric ulcers.other causes may include
nsaids,pepsin,smoking,alcohol etc.symptoms commonly include epigastric
pain,nausea,dyspepsia etc
mallory weiss tear= Mallory-Weiss syndrome (MWS) is characterised by upper
gastrointestinal bleeding (UGIB) from mucosal lacerations in the upper gastrointestinal tract,
usually at the gastroesophageal junction or gastric cardia. Mallory Weiss tears account for 48% of cases of UGIB.
Haematemesis due to a Mallory Weiss tear usually occurs after a prolonged or forceful bout
of retching, vomiting, coughing, straining or even hiccupping.
Acute treatment of variceal haemorrhage
ABC
correct clotting: FFP, vitamin K
vasoactive agents: terlipressin is currently the only licensed vasoactive agent and is
supported by NICE guidelines. Octreotide may also be used.
prophylactic antibiotics reduce mortality in patients with liver cirrhosis
endoscopy: endoscopic variceal band ligation is superior to endoscopic
sclerotherapy. NICE recommend band ligation
Sengstaken-Blakemore tube if uncontrolled haemorrhage
Transjugular Intrahepatic Portosystemic Shunt (TIPSS) if above measures fail

Prophylaxis of variceal haemorrhage


propranolol: reduced rebleeding and mortality compared to placebo
endoscopic variceal band ligation (EVL) is superior to endoscopic sclerotherapy. It
should be performed at two-weekly intervals until all varices have been eradicated.
Proton pump inhibitor cover is given to prevent EVL-induced ulceration

1404. A 23yo woman presents with a 1cm small smooth, firm, mobile mass in her left
breast. She is very anxious. What is the most appropriate inv?
a. Mammography
b. US breast
c. FNAC
d. Mammography and US
Answer= US breast. In triple assessment for breast lump, ultrasound is done for <35yrs
old and ultrasound and mammography are done for >35ys old
Diagnosis=
Fibroadenoma:
it usually presents <30yrs.It is due to benign overgrowth of the collagenous
mesenchyme of one breast lobule..it is firm,smooth,mobile and painless lump. may be
multiple.
regress, stay the same and get bigger.
investigations=
Ultrasound tends to be preferred in younger women with dense breasts, as mammograms
are more difficult to interpret in this group. Routine mammography, as a population screening
tool, is not performed below the age of 50 years. treatment= reassurance and observation.
surgical excision if large.
triple assessment of breast lump:
1. clinical examination
2.radiology=us for <35yrs and us and mammography for >35yrs old
3. histology/cytology (fnac or core biopsy: u/s guided core biopsy is best for new lumps)
fnac is for cystic lesions and core biopsy for solid lumps.
1405. A pt was admitted with abdominal pain, diarrhea, pigmented palmar creases and
buccal mucosa.
What is the most probable dx?
a. Addisons disease
b. Cushing syndrome
c. Pheochromocytoma
d. Hyperthyroidism
e. Hypoparathyroidism
answer= addisons disease. Unexplained abdominal pain and vomiting plus the
pigmented palmar creases and buccal mucosa are its main features pointing towards the
diagnosis.
exclusion of other causes:
cushing syndrome= Cushing's syndrome is caused by prolonged exposure to elevated
levels of either endogenous or exogenous glucocorticoids (mainly cortisol)

main features include: truncal obesity, weight gain,buffalo hump,moon facies,facial plethora,
gonadal dysfunction (hirsutism,irregular menses,erectile dysfunction) etc
pheochromocytoma=catecholamine producing tumors causing hypertension, episodic
headaches.
hyperthyroidism= increased levels of thyroid hormones. features of that.
hypoparathyroidism= features of hypocalcemia like cramps, paraesthesias, carpopedal
spasm etc
addisons disease
it is the primary adrenocortical insufficiency. the destruction of adrenal cortex leads to the
glucocorticoid (cortisol) and mineralocorticoid (aldosterone) deficiency.
80% are due to the autoimmunity in uk. other causes may include TB (commonest cause
worldwide),adrenal metastasis,lymphoma, opportunistic infections in HIV, adrenal
hemorrhage (waterhouse friderichsen syndrome)
features=unexplained abdominal pain or vomiting, diarrhea/constipation, pigmented palmar
creases and buccal mucosa ( acth increases; crossreacts with melanin receptors),postural
hypotension, vitiligo.
Tests:
NA decreased and potassium increased, calcium increased,low glucose, cortisol
reduced. plasma renin and aldosterone levels to assess mineralocorticoid status
short ACTH stimulation test= may be done to confirm the diagnosis
21 hydroxylase adrenal autoantibodies positive in autoimmune disease in >80%
treatment=
patient education
Hormone replacement: both mineralocorticoid and glucocorticoid should be replaced
glucocorticoid replacement= hydrocortisone is mainstay
mineralocorticoid replacement= fludrocortisone is used

1406. A 36yo pt came with diarrhea, bleeding, weight loss and fistula. What is the single
most likely dx?
a. Colorectal ca
b. Celiac disease
c. CD
d. UC
e. IBS
Answer= C- crohn's disease. Diarrhea with weight loss and abdominal pain are the
common symptom esp in young patients these symptoms raise the suspicion of crohn's
disease. extraintestinal manifestations include anal fistulas, fissures and perianal
abscess etc
exclusion of other causes:
colorectal ca=commonly occurs in people aged 65 or more.
celiac disease= peaks in infancy and 50-60yrs, diarrhea with weight loss or anemia,
steatorrhea, bloating, failure to thrive in children
Ulcerative colitis= chronic diarrhea with or without blood and mucus,crampy abdominal
discomfort. Extraintestinal features does not include fistula and fissures
IBS= no organic cause can be found in it. Only diagnose it if abdominal pain is either
relieved by defecation or associated with altered stool form and there are more than or
equal to 2 of : urgency, incomplete evacuation, abd bloating or distension, mucosal PR,

worsening of symptoms after food. There is no weight loss in it


CROHNS DISEASE
Crohn's disease is a form of inflammatory bowel disease. It commonly affects the terminal
ileum and colon but may be seen anywhere from the mouth to anus.
Pathology
cause is unknown but there is a strong genetic susceptibility
inflammation occurs in all layers, down to the serosa. This is why patients with
Crohn's are prone to strictures, fistulas and adhesions
Crohn's disease typically presents in late adolescence or early adulthood. Features include:
presentation may be nonspecific symptoms such as weight loss and lethargy
diarrhoea: the most prominent symptom in adults. Crohn's colitis may cause bloody
diarrhoea
abdominal pain: the most prominent symptom in children
perianal disease: e.g. Skin tags or ulcers
extra-intestinal features are more common in patients with colitis or perianal disease
'extraintestinal' features of inflammatory bowel diseases

Common to both Crohn's


disease (CD) and Ulcerative
colitis (UC)

Related to
disease activity

Arthritis: pauciarticular,
asymmetric

Notes

Arthritis is the most common


extra-intestinal feature in both
CD and UC

Erythema nodosum
Episcleritis

Episcleritis is more common in


CD

Osteoporosis

Unrelated to
disease activity

Arthritis: polyarticular, symmetric

Primary sclerosing cholangitis is


much more common in UC

Uveitis
Uveitis is more common in UC
Pyoderma gangrenosum
Clubbing

Primary sclerosing cholangitis

1407. A 45yo man has undergone detox and now wants a drug to stop him from craving
alcohol. What
med would be that drug of choice?
a. Disulfiram
b. Acamprosate
c. Thiamine
d. Naloxone
e. Diazepam
Answer= B. Acamprosate. After successful withdrawal, acamprosate or oral naltrexone can
be considered in combination with an individual psychological intervention in abstinence or
for prevention of relapse. Acamprosate may help intense anxiety, craving and insomnia.
exclusion of other options=
Disulfiram can be used to treat chronic alcohol dependence. it causes extremely unpleasant
effects to any alcohol ingestion e.g flushing, throbbing headache and palpitations.
Diazepam= it is used in the treatment of acute withdrawal of alcohol.

1408. A 68yo man awoke to find that he is unable to close his left eye and dribbling
saliva from the left angle of his mouth. What is the single most appropriate option?
a. Facial nerve
b. Glossopharyngeal nerve
c. Hypoglossal nerve
d. Optic nerve
e. Vagus nerve
answer= A. Facial nerve. features of 7th nerve palsy include drooling of saliva, failure to
close the eye, unilateral sagging of the mouth, absence of nasolabial fold, loss of taste in
anterior 2/3rds of the tongue, hyperacusis (hypersensitivity to sounds) (loss of
innervation to stapedius)
exclusion of other nerves=
glossopharyngeal nerve
Contains sensory, motor (stylopharyngeus only) and parasympathetic fibres (salivary
glands). it supplies tonsil, palate and posterior third of tongue. so in its lesion there will be
loss of taste on posterior third of tongue and deviation of palate to one side.
Hypoglossal nerve=it supplies motor fibres to the tongue and most of the infrahyoid
muscles. An LMN lesion produces wasting of the ipsilateral side of the tongue, with
fasciculation; and on attempted protrusion the tongue deviates towards the affected side, but
the tongue deviates away from the side of a central lesion.

optic nerve=.Visual field defects, pupillary abnormalities,


Optic neuritis (pain on moving the eye, loss of central vision, afferent pupillary defect,
papilloedema),
The VIIth cranial (facial) nerve is largely motor in function (some sensory fibres from external
acoustic meatus, fibres controlling salivation and taste fibres from the anterior tongue in the
chorda tympani branch). It also supplies the stapedius (so a complete nerve lesion will alter
auditory acuity on the affected side).
lower motor neuron facial nerve palsy - forehead affected so patient cant wrinkle the
forehead
*upper motor neuron lesion 'spares' upper face so patient can wrinkle the forehead

1409. A 19yo female dx with trichomonas vaginalis. LMP was 10d ago. What is the best
antibiotic tx?
a. Erythromycin
b. Vancomycin
c. Metronidazole
d. Penicillin
e. Clarithromycin
f. Doxycycline
g. Fluconazole
h. Clotrimazole
Answer= C. Metronidazole. treatment for trichomonas vaginalis is Metronidazole 2g po
stat or 400mg/12h po for 5 days (e.g if pregnant). it should be given to the partner too.
Trichomonas vaginalis is a highly motile, flagellated protozoan parasite
Features=
vaginal discharge: offensive and thin fishy smelly discharge, yellow/green, frothy
vulvovaginitis,strawberry cervix, pH > 4.5
in men is usually asymptomatic but may cause urethritis
investigations= microscopy of a wet mount shows motile trophozoites

1410. A 35yo man has been given a dx of allergic rhinitis and asthma. Exam: peripheral
neuropathy with tingling and numbness in a glove and stocking distribution. Skin lesions
are present in the
form of tender subcutaneous nodules. The pt is responding well to corticosteroids. What
is the
single most appropriate dx?
a. AS
b. Churg-strauss syndrome
c. Cryptogenic organizing
d. Extrinsic allergic alveolitis
e. Tropical pulmonary eosinophilia
Answer= B. churg- strauss syndrome. allergic rhinitis with asthma points towards the
diagnosis of churg strauss syndrome.
mnemonic : BEAN SAP
BE: Blood Eosinophilia
A : Asthma
N : Neuropathy (mononeuritis multiplex) - usually common peroneal nerve
S : Sinus abnormality
A : Allergies
P : Perivascular eosinophils / vasculitis
Churg Strauss Syndrome
A triad of adult-onset asthma, eosinophilia, and vasculitis
( vasospasm MI DVT), affecting lungs, nerves, heart, and skin.
A septic-shock picture/systemic inflammatory response syndrome may occur (with
glomerulo nephritis/
renal failure, esp. if ANCA +ve).
Presentation:
The physical findings are specific to organ system involvement. There are three phases:
Allergic rhinitis and asthma.
Eosinophilic infiltrative disease, such as eosinophilic pneumonia or
gastroenteritis.
Systemic medium and small vessel vasculitis with granulomatous inflammation.
Investigations: Antineutrophil cytoplasmic antibodies (ANCA): 70% of patients are
perinuclear staining (p-ANCA) positive (anti myeloperoxidase antibodies).Other likely
findings include eosinophilia and anaemia on the FBC; elevated ESR and CRP; elevated
serum creatinine; increased serum IgE levels

Treatment= Steroids; biological agents if refractory disease,


eg rituximab.
1411. A 28yo woman comes with sudden onset vomiting and pain per abdomen. Exam:
mobile
swelling in the right iliac fossa. What is the most probable dx?
a. Ectopic pregnancy
b. Tubo-ovarian abscess
c. Acute appendicitis
d. Ovarian torsion

e. Diverticulitis
Answer= D Ovarian Torsion. Mobile swelling in rif sudden pain vomiting in female
exclusion of other options:
ectopic pregnancy= history of amenorrhea and pv bleeding and it won't be palpable
tubo- ovarian abscess= history of fever present
acute appendicitis= mass wont be mobile in it rather will be fixed
diverticulitis= pain on the left side
OVARIAN TORSION

Ovarian torsion (adnexal torsion) is an infrequent but significant cause of acute lower abdominal pain
in women. This condition is usually associated with reduced venous return from the ovary as a result of
stromal edema, internal hemorrhage, hyperstimulation, or a mass. The ovary and fallopian tube are
typically involved.

ETIOLOGY
Pregnancy is associated with, and may be responsible for, torsion in approximately 20% of adnexal
torsion cases
Ovarian tumors, both benign and malignant, are implicated in 50-60% of cases of torsion.
Dermoid tumors are most common.
patients with a history of pelvic surgery (principally tubal ligation) are at increased risk for torsion.

Features;
Classically, patients present with the sudden onset (commonly during exercise or other agitating
movement) of severe, unilateral lower abdominal pain that worsens intermittently over many hours.
The pain usually is localized over the involved side, often radiating to the back, pelvis, or thigh
Nausea and vomiting
Fever may occur as a late finding as the ovary becomes necrotic.

Investigations:
Diagnostic ultrasonography should be the first examination performed; typically, the affected ovary
is enlarged, with multiple immature or small follicles along its periphery.
Ultrasonography with color Doppler analysis is the method of choice for the evaluation of
adnexal torsion

Treatment:
Outpatient care has no role in the treatment of ovarian torsion. Patients with either a suspected or
confirmed diagnosis of ovarian torsion should be admitted and either operated on or observed by a
gynecologist. Laparoscopy can be used for both confirmation of the diagnosis and treatment.

(source medscape)

1412. A 68yo man on tx for an irregular heart beat comes to the ED. He has palpitations
for the last 3h. Exam: pulse=regular, 154bpm. Carotid sinus massage settled his pulse
down to 80bpm. What is the most likely rhythm disturbance?
a. SVT
b. V-fib
c. VT
d. V-ectopics
e. A-fib

Answer= A.SVT Palpitations and dizziness, which are the most common symptoms
reported in svt.
During an attack, tachycardia may be the only finding if the patient is otherwise healthy and
there is no cardiac dysfunction. During an episode of SVT the pulse rate is 140-250 beats
per minute (bpm).In haemodynamically stable regular narrow QRS-complex tachycardia,
vagal manoeuvres - eg, Valsalva, carotid massage, facial immersion in cold water.
1413. A 43yo man with a hx of hospital admissions talk about various topics, moving
from one loosely connected topic to another. What is the most likely dx?
a. Psychosis
b. Mania
c. Schizophrenia
d. Pressured speech
e. Verbal diarrhea
Answer= B. Mania. rapid flit from one subject to another ('flight of ideas') and pressured
speech (speech is rapid and copious) both are features of mania..
The presence of psychotic symptoms differentiates mania from hypomania
Psychotic symptoms
delusions of grandeur
auditory hallucinations
The following symptoms are common to both hypomania and mania
Mood

predominately elevated
irritable

Speech and thought


pressured
flight of ideas
poor attention
Behaviour
insomnia
loss of inhibitions: sexual promiscuity, overspending, risk-taking
increased appetite

1414. An 18yo girl presents with rahs on her trunk, abdominal pain, arthritis, proteinuria
and
hematuria. What is the most probable dx?
a. TTP
b. ITP
c. HSP
d. HUS
e. Measles

Answer= C. HSP. Henoch-Schnlein purpura (HSP) is an IgA-mediated, autoimmune


hypersensitivity vasculitis of childhood. The main clinical features are skin purpura,
arthritis, abdominal pain, gastrointestinal bleeding, orchitis and nephritis. In classic cases
palpable purpuric rash (with localized oedema) over buttocks and other sites are
extensor surfaces of arms and legs and trunk. Features of IgA nephropathy may occur
e.g. haematuria, renal failure.
exclusion of other causes:
HUS=Haemolytic uraemic syndrome is generally seen in young children and produces a
triad of:acute renal failure, microangiopathic haemolytic anaemia and
thrombocytopenia. Typical (or infection-induced) HUS is most commonly associated with
Escherichia coli with somatic (O) antigen 157 and flagella (H) antigen 7 - hence the
designation O157:H7.The classical presenting feature is profuse diarrhoea that turns
bloody 1 to 3 days later.

Henoch-Schnlein purpura (HSP) is an IgA-mediated, autoimmune hypersensitivity


vasculitis of childhood.
The main clinical features are skin purpura, arthritis, abdominal pain, gastrointestinal
bleeding, orchitis and nephritis. In classic cases palpable purpuric rash (with localized
oedema) over buttocks and other sites are extensor surfaces of arms and legs and
trunk. Features of IgA nephropathy may occur e.g. haematuria, renal failure.
Treatment:
analgesia for arthralgia
treatment of nephropathy is generally supportive. There is inconsistent evidence for the use
of steroids and immunosuppressants
Prognosis:
usually excellent, HSP is a self-limiting condition, especially in children without renal
involvement
around 1/3rd of patients have a relapse
HSP = 10 P's
PALPABLE
PURPURA
PLATELETS OK -- DDx from ITP
PRURITUS -- URTICARIA
PAIN -- ABDOMEN and LEGS
POSITIVE +guaiac -- FECES
PRESSURE -- increased blood pressure
PROTEINURIA
PREDNISONE = Tx

1415. A pt is on loop diuretics. What effect do loop diuretics produce?


a. Low Na+, low K+

b. Low Na+, normal K+


c. Normal Na+, normal K+
d. High Na+, low K+
e. High Na+, high K+
Answer= A. low na+, Low K+
Loop Diuretics
loop diuretics act by inhibiting the Na-K-Cl cotransporter (NKCC) in the thick ascending limb
of the loop of Henle, reducing the absorption of NaCl.
There are two variants of NKCC; loop diuretics act on NKCC2, which is more prevalent in the
kidneys.
Indications= heart failure: both acute (usually intravenously) and chronic (usually orally),
resistant hypertension, particularly in patients with renal impairment.
Adverse effects= hypotension, hyponatremia, hypokalemia,hypocalcemia,hypochloremic
alkalosis, hyperglycemia (less common with thiazides), renal impairment( from
dehydration+direct toxic effect) and gout

1416. A 6yo girl is being investigated for renal failure. She is found to have a congenital
abnormality of the insertion of the ureters into the urinary bladder. What is the single
most likely cause for renal failure in this pt?
a. SLE
b. PKD
c. Wilms tumor
d. Acute tubular necrosis
e. Reflux nephropathy
Answer= Reflux Nephropathy
1417. A 76yo man is in the CCU 2d after an acute MI. He tells you that he had an
episode of rapid pounding in the chest lasting for about 2mins. He remains conscious
throughout. What is the most likely rhythm?
a. SVT
b. VF
c. VT
d. V-ectopics
e. A-fib
Answer= C. Ventricular tachycardia. Palpitations (pounding chest) or arrhythmias 48hrs
post MI is almost always Ventricular tachycardia until proven otherwise.
Ventricular tachycardia (VT) is broad-complex tachycardia originating from a ventricular
ectopic focus. It has the potential to precipitate ventricular fibrillation and hence requires
urgent treatment.
There are two main types of VT:
monomorphic VT: most commonly caused by myocardial infarction
polymorphic VT: A subtype of polymorphic VT is torsades de pointes which is
precipitated by prolongation of the QT interval.
Management:
If the patient has adverse signs (systolic BP < 90 mmHg, chest pain, heart failure or rate >

150 beats/min) then immediate cardioversion is indicated.


In the absence of such signs antiarrhythmics may be used.
If these fail, then electrical cardioversion may be needed with synchronised DC shocks.
Drug therapy:

amiodarone: ideally administered through a central line


lidocaine: use with caution in severe left ventricular impairment
procainamide

Verapamil should NOT be used in VT


If drug therapy fails

electrophysiological study (EPS)


implantable cardioverter-defibrillator (ICD) - this is particularly indicated in patients
with significantly impaired LV function

1418. A 49yo man comes with hx of cough and SOB. His CD4 count is measured as
350. CXR shows lobar consolidation. What is the single most appropriate option?
a. Mycobacterium avium intracellulare
b. CMV
c. Streptococcus
d. Toxoplasmosis
e. Pneumocystis jiroveci
Answer= C Streptococcus. The patient presents with h/o cough. SOB and cxr shows
lobar consolidation. these point towards the diagnosis of pneumonia and streptococcus
is the most common cause of community acquired pneumonia.
exclusion of other options:
pneumocystis Jiroveci (PCP): PCP is the most common opportunistic infection in AIDS
which usually occur when CD4 count <200/mm. all patients with a CD4 count < 200/mm
should receive PCP prophylaxis
Community acquired pneumonia (CAP) may be caused by the following infectious agents:
Streptococcus pneumoniae (accounts for around 80% of cases)
Haemophilus influenzae
Staphylococcus aureus: commonly after the 'flu
atypical pneumonias (e.g. Due to Mycoplasma pneumoniae)
viruses
Klebsiella pneumoniae is classically in alcoholics
Streptococcus pneumoniae (pneumococcus) is the most common cause of communityacquired pneumonia
Clinical features
Symptoms: Fever, rigors, malaise, anorexia, dyspnoea, cough, purulent sputum,
haemoptysis, and pleuritic pain.
Signs: Pyrexia, cyanosis, confusion (elderly usually), tachypnoea, tachycardia, hypotension,
signs of consolidation (diminished expansion, dull percussion note, tactile vocal
fremitus/vocal resonance, bronchial breathing), and a pleural rub.

Tests:
CXR: lobar or multilobar infiltrates, cavitation or pleural effusion.
Assess oxygenation: oxygen saturation (ABGs if SaO2 <92% or severe pneumonia) and
BP.
Blood tests: FBC, U&E, LFT, CRP, blood cultures. Sputum for microscopy and culture.
Pleural fluid may be aspirated for culture.
Consider bronchoscopy and bronchoalveolar lavage if patient is immunocompromised or on
ITU.
Management:
CURB-65 criteria of severe pneumonia

Confusion (abbreviated mental test score <= 8/10)

Urea > 7 mmol/L


Respiratory rate >= 30 / min
BP: systolic <= 90 or diastolic <= 60 mmHg
age >= 65 years

Patients with 3 or more (out of 5) of the above criteria are regarded as having a severe
pneumonia
The British Thoracic Society published guidelines in 2009:

low or moderate severity CAP: oral amoxicillin. A macrolide should be added for
patients admitted to hospital
high severity CAP: intravenous co-amoxiclav + clarithromycin OR cefuroxime +
clarithromycin OR cefotaxime + clarithromycin
the current BNF has slightly different recommendations for high severity CAP:
intravenous benzylpenicillin + clarithromycin OR benzylpenicillin + doxycycline. For
'life-threatening' infections the BNF recommends the same as the BTS guidelines for
high-severity CAP

1419. A 32yo woman with prv hx of PID now presents with severe abdominal pain. Her
LMP was 8wks ago. What is the most probable dx?
a. Ectopic pregnancy
b. Ovarian torsion
c. Hematometrium
d. Chronic PID
e. Cholecystitis
Answer= A. Ectopic Pregnancy. typical history of 8 weeks amenorrhea with lower
abdominal pain. PID is one of its risk factors.
Ectopic pregnancy:
An ectopic pregnancy is one that occurs anywhere outside the uterus. By far the most
common place for ectopic pregnancy is the Fallopian tubes.
Risk Factors: Fertility treatments and intrauterine contraceptive devices (IUCDs) are the
most important associated risk factors. Pelvic inflammatory disease may cause complete
tubal occlusion or delay the transport of the embryo so that implantation occurs in the tube.
Adhesions from infection and inflammation from endometriosis may play a part.

presentation:
Symptoms and signs of ectopic pregnancy can resemble those of other more common
conditions, including urinary tract infections and gastrointestinal conditions.The most
common symptoms are= Abdominal pain.Pelvic pain.Amenorrhoea or missed period
(typically 6 to 8 weeks) Vaginal bleeding (with or without clots).
Investigations: urine pregnancy test should be performed in all the women of childbearing
age presenting with lower abd pain. The most accurate method to detect a tubal pregnancy
is transvaginal ultrasound.This can identify the location of the pregnancy and also whether
there is a fetal pole and heart beat.
Human chorionic gonadotrophin (hCG) levels are performed in women with pregnancy of
unknown location who are clinically stable. hCG levels are taken 48 hours apart.
Treatment: Anti D prophylaxis to all rhesus negative women. Surgical: if presents with
shock then Urgent
Laparotomy.
otherwise laparo
scopy is preferable.
A
salpingectomy should be performed, unless the woman has other risk factors for infertility, in
which case a salpingotomy should be undertaken.

1420. A 25yo who is 38wks pregnant presents to the labour ward with a hx of fewer fetal
movements than usual during the evening. She also says that abdominal contractions
are coming very few minutes and she is having a blood stained show per vagina for the
last few minutes. Exam: cervix is fully effaced, 9cm dilated, cephalic presentation and
station is +1. Choose the single most likely dx?
a. APH
b. Concealed hemorrhage
c. Labour
d. IUFD
e. IUGR
Answer= C.Labour. Normal labour occurs after 37 weeks gestation. This is the first stage
of labour as there are contractions and cervix is fully effaced.
NORMAL LABOUR
Normal labour occurs after 37wks gestation and results in spontaneous
vaginal delivery of the baby within 24h of the onset of regular spontaneous contractions.
It is often heralded by a show, ie a plug of cervical mucus and a little
blood as the membranes strip from the os (membranes may then rupture).
The first stage of labour
Latent phase (not necessarily continuous):
there are painful contractions, the cervix initially effaces (becomes shorter and
softer) then dilates to 4cm.
Established phase:
contractions with dilatation from 4 cm. A satisfactory rate of dilatation from 4 cm is
0.5cm/h.
The 1st stage generally takes 818h in a primip, and 512h in a multip.
During the first stage check maternal BP, and T 4-hourly, pulse hourly;

assess the contractions every 30min, their strength and their frequency (ideally 34 per
10min, lasting up to 1 min).
Offer vaginal examination e.g every 4h to assess the degree of cervical dilatation, the
position and the station of the head.
Auscultate fetal heart rate (if not continuously monitored), by Pinard or Doppler every
15min, listening for 1min after a contraction.
The second stage:
Passive stage is complete cervical dilatation (but no desire to push).
In active stage, the baby can be seen, there is full dilatation with expulsive contractions
and maternal effort (using abdominal muscles and the Valsalva manoeuvre until the
baby is born)
The third stage is delivery of the placenta.
As the uterus contracts to a <24-week size after the baby is born, the placenta separates
from the uterus through the spongy layer of the decidua basalis. It then buckles and a
small
amount of retroplacental haemorrhage aids its removal.
1421. A 30yo woman has a painless lump in the outer aspect of her left breast. She has
had a prv breast lump. Her grandmother had breast cancer at 70yrs. She has a 1 cm
smooth, firm, discrete, mobile lump in the other quadrant region of the left breast. What
is the single most likely dx?
a. Breast abscess
b. Breast carcinoma
c. Breast cyst
d. Fibroadenoma
e. Sebaceous cyst
Answer= Fibroadenoma= age is 30yrs, painless,mobile,smooth and firm lump.
Fibroadenoma:
it usually presents <30yrs.It is due to benign overgrowth of the collagenous
mesenchyme of one breast lobule..it is firm,smooth,mobile and painless lump. may be
multiple.
regress, stay the same and get bigger.
investigations=
Ultrasound tends to be preferred in younger women with dense breasts, as mammograms
are more difficult to interpret in this group. Routine mammography, as a population screening
tool, is not performed below the age of 50 years. treatment= reassurance and observation.
surgical excision if large.
1422. A 38yo woman is in the ED following an OD of her meds. She doesnt need med
tx for the OD. She says she wishes to be discharged. What is the single most
appropriate management?
a. Community psychiatric nurse visit
b. Psychiatric OPD review the next day
c. Prescribe anti-depressants
d. Admission under the mental health act

e. Discharge and allow to go home


Answer: D. Admission under the mental health act. Patient has overdosed, she is
suicidal and can do the same act again so she should not be discharged and she should
be admitted for her mental health assessment.
1423. A 63yo male presents after having had a seizure. Exam: alert and oriented. Exam:
inattention on
the left side and hyperreflexia of the arm. What is the most probable dx?
a. Cerebral tumor
b. Pituitary adenoma
c. Cerebellar abscess
d. Huntingtons chorea
e. Parkinsonism
Answer: A. Cerebral tumor. H/o seizure and inattention on left side and hyperreflexia of
the arm all point towards the diagnosis.
Primary brain tumours represent about 2% of all tumours diagnosed in the UK.
The presentation will depend on location and rate of growth but includes features of a spaceoccupying lesion and raised intracranial pressure (ICP):
Headache, which is typically worse in the mornings.
Nausea and vomiting.
Seizures.
Progressive focal neurological deficits - eg, diplopia associated with a cranial
nerve defect, visual field defect, neurological deficits affecting the upper and/or
lower limb.
Cognitive or behavioural symptoms.
Symptoms relating to location of mass - eg, frontal lobe lesions associated
with personality changes, disinhibition and parietal lobe lesions might be
associated with dysarthria.
Papilloedema (absence of papilloedema does not exclude a brain tumour).
Diagnosis:
largely rests on brain imaging - eg, CT scan and/or MRI scan (both with or without contrast).
MRI is more sensitive.
Management:
Surgery= Tumours should be resected whenever possible but some may be inaccessible.
Surgery should also be considered to reduce mass effect and treat hydrocephalus in order to
provide symptomatic relief.
If surgery is not an option then radiotherapy should be considered.
Radiotherapy= prolongs survival.
Chemotherapy= Its role is not as marked as in other tumors. Chemo-radiotherapy is used
post-op for gliomas or metastases.
Complications:
Acute haemorrhage into a tumour.
Blockage of cerebrospinal fluid outflow, causing hydrocephalus.
Sudden increases in ICP may lead to life-threatening brain herniation through the foramen
magnum or transtentorial foramina.
Complications of radiotherapy.

1424. A 70yo lady on Raloxifene for osteoporosis has recently to the UK from Australia.
She now presents with severe chest pain, SOB and suddenly collapsed in the ED. What
is the single most appropriate dx?
a. MI
b. Aortic dissection
c. Pulmonary embolism
d. Costochondritis
e. Pneumothorax
Answer: C. Pulmonary Embolism. patient is on Raloxifene which is selective estrogen
receptor modulator and it increases the risk of thromboembolic events.
PULMONARY EMBOLISM
Always suspect pulmonary embolism (PE) in sudden collapse 12wks after surgery.
Mechanism= Venous thrombi, usually from DVT, pass into the pulmonary circulation
and block blood flow to lungs. The source is often occult.
Risk factors
Malignancy.
Surgeryespecially pelvic and lower limb (much lower if prophylaxis used).
Immobility.
Combined oral contraceptive pill (there is also a slight risk attached to HRT).
Previous thromboembolism and inherited thrombophilia.
Signs and symptoms
Acute dyspnoea, pleuritic chest pain, haemoptysis, and syncope.
Hypotension, tachycardia, gallop rhythm, JVP, loud P2, right ventricular heave,
pleural rub, tachypnoea, cyanosis, AF.
Investigation: (2012 NICE guidelines)
All patients with symptoms or signs suggestive of a PE should have a history taken,
examination performed and a chest x-ray to exclude other pathology.
If a PE is still suspected a two-level PE Wells score should be performed:
Clinical feature

Clinical signs and symptoms of DVT (minimum of leg swelling and pain with
palpation of the deep veins)

Points

An alternative diagnosis is less likely than PE

Heart rate > 100 beats per minute

1.5

Immobilisation for more than 3 days or surgery in the previous 4 weeks

1.5

Previous DVT/PE

1.5

Haemoptysis

Malignancy (on treatment, treated in the last 6 months, or palliative)

Clinical probability simplified scores


PE likely - more than 4 points
PE unlikely - 4 points or less
If a PE is 'likely' (more than 4 points) = immediate computed tomography pulmonary
angiogram (CTPA).
If there is a delay in getting the CTPA = give low-molecular weight heparin until the scan is
performed.
If a PE is 'unlikely' (4 points or less) = D-dimer test.
If this is positive= immediate computed tomography pulmonary angiogram (CTPA).
If there is a delay in getting the CTPA= give low-molecular weight heparin until the scan is
performed.
If the patient has an allergy to contrast media or renal impairment a V/Q scan should be used
instead of a CTPA.
Management: (NICE guidelines)
Low molecular weight heparin (LMWH) or fondaparinux should be given initially after a PE is
diagnosed (except in massive PE)
a vitamin K antagonist (i.e. warfarin) should be given within 24 hours of the diagnosis.
Low molecular weight heparin (LMWH)= upto 5 days or until INR >2.
warfarin should be continued for at least 3 months.
NICE advise extending warfarin beyond 3 months for patients with unprovoked PE. This
essentially means that if there was no obvious cause or provoking factor (surgery, trauma,
significant immobility) it may imply the patient has a tendency to thrombosis and should be
given treatment longer than the norm of 3 months
for patients with active cancer NICE recommend using LMWH for 6 months.
Thrombolysis is now recommended as the first-line treatment for massive PE where there is
circulatory failure (e.g. hypotension).

1425. A 35yo woman complains of hoarseness of voice 3h after partial thyroidectomy.

She had no hx of phonation probs before the surgery. What is the single most
appropriate inv?
a. Laryngoscopy
b. Bronchoscopy
c. CT neck
d. CXR
e. Barium swallow
Answer= A.Laryngoscopy. h/o thyroidectomy which may lead to recurrent laryngeal
nerve injury resulting in hoarseness of voice or breathlessness. so laryngoscopy is done
to look for vocal cords.
THYROIDECTOMY
Possible complications following thyroid surgery include:
Minor complications such as collections of serous fluid (they resolve spontaneously if small
and asymptomatic but may require single or repeated aspiration if large) and poor scar
formation.
Bleeding, which may cause tracheal compression.
Recurrent laryngeal nerve injury: Innervates all of the intrinsic muscles of the larynx,
except the cricothyroid
muscle.
Patients with
unilateral vocal fold paralysis present with postoperative
hoarseness.
Bilateral vocal fold paralysis
may occur following a total thyroidectomy and usually presents immediately after
extubation.
Hypoparathyroidism
Thyrotoxic
storm
Superior
laryngeal nerve injury:. Trauma to the nerve results in an inability to lengthen a vocal fold
and thus to create a higher-pitched sound.The external branch is probably the most
commonly injured nerve in thyroid surgery. Most patients do not notice any change but the
problem may be career-ending for a professional
singer.
Infection.
Hypothyroidism

1426. A 40yo pt came to OPD with complaint of fever, pleuritic chest pain, productive
cough and painful vesicles around the lips. Exam: temp=38C. He has a hx of
splenectomy last yr. What is the single most likely causative organism?
a. Pneumococcal pneumonia
b. Staphylococcus
c. Klebsiella
d. Streptococcus
e. Chlamydia psitacci
Answer= A Pneumococcal pneumonia. the patient has a history of splenectomy and pts
with splenectomy are more prone to develop pneumococcus pneumonia that is why they
are given vaccines prophylactically.
Community acquired pneumonia (CAP) may be caused by the following infectious agents:
Streptococcus pneumoniae (accounts for around 80% of cases)

Haemophilus influenzae
Staphylococcus aureus: commonly after the 'flu
atypical pneumonias (e.g. Due to Mycoplasma pneumoniae)
viruses

Klebsiella pneumoniae is classically in alcoholics


Streptococcus pneumoniae (pneumococcus) is the most common cause of communityacquired pneumonia
Clinical features
Symptoms: Fever, rigors, malaise, anorexia, dyspnoea, cough, purulent sputum,
haemoptysis, and pleuritic pain.
Signs: Pyrexia, cyanosis, confusion (elderly usually), tachypnoea, tachycardia, hypotension,
signs of consolidation (diminished expansion, dull percussion note, tactile vocal
fremitus/vocal resonance, bronchial breathing), and a pleural rub.
Tests:
CXR: lobar or multilobar infiltrates, cavitation or pleural effusion.
Assess oxygenation: oxygen saturation (ABGs if SaO2 <92% or severe pneumonia) and
BP.
Blood tests: FBC, U&E, LFT, CRP, blood cultures. Sputum for microscopy and culture.
Pleural fluid may be aspirated for culture.
Consider bronchoscopy and bronchoalveolar lavage if patient is immunocompromised or on
ITU.
Management:
CURB-65 criteria of severe pneumonia

Confusion (abbreviated mental test score <= 8/10)

Urea > 7 mmol/L


Respiratory rate >= 30 / min
BP: systolic <= 90 or diastolic <= 60 mmHg
age >= 65 years

Patients with 3 or more (out of 5) of the above criteria are regarded as having a severe
pneumonia
The British Thoracic Society published guidelines in 2009:

low or moderate severity CAP: oral amoxicillin. A macrolide should be added for
patients admitted to hospital
high severity CAP: intravenous co-amoxiclav + clarithromycin OR cefuroxime +
clarithromycin OR cefotaxime + clarithromycin
the current BNF has slightly different recommendations for high severity CAP:
intravenous benzylpenicillin + clarithromycin OR benzylpenicillin + doxycycline. For
'life-threatening' infections the BNF recommends the same as the BTS guidelines for
high-severity CAP

1427. A 37yo male pt who recently returned back to UK from UAE attends the OPD with
complaint of dry cough, breathlessness and anorexia. According to him he had flu like

symptoms a week ago. He is slightly confused. Inv: lymphopenia & decreased Na+.
CXR: bi-basal consolidation. What is the single most likely causative organism?
a. Legionella
b. Chlamydia pneumoniae
c. PCP
d. Viral pneumonia
e. Chlamydia psittaci
Answer= A. legionella. travel history,flue like symptoms with dry cough and dyspnea,
confusion. chest x ray showing bi-basal consolidation. blood tests showing lymphopenia
and hyponatremia are all features of pneumonia caused by Legionella pneumophila.
LEGIONELLA PNEUMOPHILA
it colonizes water tanks kept at <60C (hotel air conditioning and hot water systems)
causing outbreaks of legionnaires disease.
flu like symptoms (fever,malaise,myalgia) precede dry cough and dyspnea.
Extra pulmonary features include anorexia, hepatitis,renal failure,confusion and coma.
CXR shows bi- basal consolidation. blood tests show lymphopenia and hyponatremia
and deranged LFTs. urinalysis may show hematuria.
Diagnosis= Legionella urine antigen/culture
Treatment= fluoroquinolone for 2-3 weeks or clarithromycin.
1428. A 20yo student came to the OPD with complains of headache, malaise, dry cough,
joint pain and vomiting. Exam: temp=39C. CXR: patchy consolidation. What is the single
most likely causative organism?
a. Pneumococcal pneumonia
b. Mycoplasma
c. Klebsiella
d. Streptococcus
e. PCP
Answer= B. Mycoplasma. flu like symptoms followed by dry cough. chest xray shows
patchy consolidation often of one lower lobe or reticulo nodular shadowing.
MYCOPLASMA PNEUMONIA
occurs in epidemics about every 4yrs. it presents insidiously with flu like symptoms
(fever, malaise,arthralgia) followed by a dry cough.
CXR= reticulo nodular shadowing and patchy consolidation often of one lower lobe.
Diagnosis= PCR sputum or serology
Treatment= clarithromycin (500mg/12h) or doxycycline or a fluoroquinolone
Complications= skin rash (erythema multiforme), stevens johnson syndrome,
meningoencephalitis or myelitis, guillain- barre syndrome
1429. A 45yo man presented to his GP with vague symptoms of headache, proximal
muscle weakness and nocturia. Test results show him to be severely HTN
(230/130mmHg) and hypokalemic. What is the most probable dx?
a. Addisons disease
b. Conns disease
c. Familial hyperaldosteronism

d. Cushings disease
e. Cushings syndrome
Answer: B. Conns Disease. proximal muscle weakness, nocturia, HTN and hypokalemia
are the features of conns disease which is due to excessive levels of aldosterone.
PRIMARY HYPERALDOSTERONISM
Primary hyperaldosteronism is excess production of aldosterone, independent of
the reninangiotensin system, causing sodium and water retention, and renin release.
Consider if the following features: hypertension, hypokalaemia or alkalosis in
someone not on diuretics. Sodium tends to be mildly raised or normal.
Symptoms:
signs of hypokalemia, weakness (even quadriparesis), cramps, paresthesia, polyuria,
polydipsia. raised blood pressure.
Causes: ~ are due to a solitary aldosterone-producing adenoma (Conns syndrome).
~ are due to bilateral adrenocortical hyperplasia.
Rare causes: adrenal carcinoma or glucocorticoid-remediable aldosteronism (GRA).
Investigations:
high serum aldosterone
low serum renin
high-resolution CT abdomen
adrenal vein sampling
Management
adrenal adenoma: surgery (laparoscopic adrenalectomy)
bilateral adrenocortical hyperplasia: aldosterone antagonist e.g. spironolactone

1430. A man says his insides are rotting and nobody has buried him. Which term best
describes his condition?
a. Delusion of nihilism
b. Delusion of guilt
c. Delusion of persecution
d. Incongruent affect
e. Clang association
Answer: A. Delusion of Nihilism. these are the delusions in which people deny their
existance and believe that they are rotting or they demand burial.
Cotards syndrome (nihilistic delusions)
We deny our existence, or believe we are rotting, or we demand burial, thinking we are a
corpse.
Cause: Depression, alcohol, syphilis, parietal lobe lesion, or just being born.
DELUSIONS
A delusion is a false belief which is firmly sustained and based on incorrect inference about
reality. This belief is held despite evidence to the contrary and is not accounted for by the
person's culture or religion.
Types of Delusions:
Monothematic - delusions are only relating to one particular topic.
Polythematic - range of delusional topics (seen in schizophrenia).
They can also be classified as:
Primary - occur in the mind, fully formed with no preceding reasons; strongly suggestive of
schizophrenia.
Secondary - eg depressed person feeling worthless.
Examples:
Delusional jealousy (Othello's syndrome) - eg believing a partner is being unfaithful.

Capgras' delusion - belief that a close relative has been replaced by someone
else who looks the same.
Unilateral neglect - belief that one limb or side does not exist.
Thought insertion - belief that someone is putting thoughts into the brain.
Grandiose delusion - belief of exaggerated self-worth.

1431. A man with chronic cough presents with copious purulent sputum. What is the
single most dx?
a. Bronchitis
b. Bronchiectasis
c. COPD
d. Pneumonia
e. Emphysema
Answer= b. Bronchiectasis. Copious purulent sputum with persistent cough are classical
features of bronchiectasis.
exclusion of other causes:

COPD= it includes chronic bronchitis and emphysema. symptoms include chronic cough
but it is not copious,dyspnea.usually there is h/o smoking.
Bronchiectasis is described as permanent dilatation of the airways secondary to chronic
infection or inflammation.
Most common organisms isolated from patients with bronchiectasis:
Haemophilus influenzae (most common), Pseudomonas aeruginosa, Klebsiella spp,
Streptococcus pneumoniae
Causes: post-infective:( tuberculosis, measles, pertussis, pneumonia) cystic fibrosis,
bronchial obstruction (e.g. lung cancer/foreign body), immune deficiency: (selective IgA,
hypogammaglobulinemia), allergic bronchopulmonary aspergillosis (ABPA), ciliary dyskinetic
syndromes: (Kartagener's syndrome, Young's syndrome), yellow nail syndrome
features= persistent cough, purulent copious sputum,intermittent hemoptysis. finger
clubbing, coarse inspiratory crepitations, wheeze ( all found in other diseases)
Tests: sputum culture. chest xray shows cystic shadows, thickened bronchial walls (tramline
and ring shadows). other investigations according to the causes.
Management:
postural drainage,
chest physiotherapy
antibiotics usually for acute exacerbations
surgery in selected cases (e.g localized disease,to control severe hemoptysis)

1432. A 32yo man working in a shipyard comes with SOB. Exam: dullness on left side of
the chest, pain
in left side of chest, pleuritic rub and crackles been heard on the same side. What is the
single
most likely dx?
a. Pericarditis
b. Pleurisy
c. Pleural effusion
d. CCF
e. TB
Answer= C. Pleural effusion. shipyard worker therefore exposure to asbestos which
leads to mesothelioma causing pleural effusion. dyspnea, pleuritic chest pain, decreased
chest expansion, stony dull percussion note, diminished breath sounds on affected side
are its features.
Pleural Effusion is the fluid in the pleural space.
effusions can be divided by their protein concentration into transudates (<30g/l) and
exudates (>30g/l)
Causes=
Transudates: commonly heart failure, cirrhosis, hypoalbuminemia, peritoneal dialysis,
constrictive pericarditis, hypothyroidism, nephrotic syndrome etc
Exudative: pneumonia, malignancy (bronchogenic ca, mesothelioma, lymphoma etc),
TB, rheumatoid arthritis,pulmonary infarction, pancreatitis, SLE etc
Features= asymptomatic or dyspnea,pleuritic chest pain, decreased chest expansion,
stony dull percussion and diminished breath sounds on the affected side. features of
other diseases may be present.

Tests= Chest X-ray: this is the first investigation if a pleural effusion is suspected clinically.
small effusions blunt the costophrenic angles. large are seen as water dense shadows with
concave upper borders. Bilateral effusions with an enlarged heart shadow are commonly
caused by congestive cardiac failure.
Ultrasound= done to identify the presence of fluid and in guiding therapeutic or diagnostic
aspiration.
Diagnostic aspiration and pleural biopsy are also done.
Management:
Small effusions that are not causing respiratory embarrassment may be managed by
observation.
symptomatic effusions= drainage. fluid is best removed slowly (0.5 -1.5L/hr)
Pleurodesis with tetracycline, bleomycin or talc may be helpful for recurrent effusions.
surgery= persistent collections and increasing pleural thickness (on U/S) require surgery

1433. A 67yo female presents with balance prbs. Exam: nystagmus on left lateral gaze,
a loss of the let corneal reflex and reduced hearing in the left ear. What is the most likely
dx?
a. Menieres disease
b. Acoustic neuroma
c. Cerebral abscess
d. Pituitary tumor
e. Gentamicin
Answer: B. Acoustic Neuroma. Balance problems, nystagmus, loss of corneal reflex and
reduced unilateral hearing points towards the diagnosis.
Acoustic neuromas (more correctly called vestibular schwannomas) account for
approximately five percent of intracranial tumours and 90 percent of cerebellopontine angle.
These are typically indolent, histologically benign subarachnoid tumours
that cause problems by local pressure, and then behave as space-occupying
lesions
Features can be predicted by the affected cranial nerves
cranial nerve VIII: hearing loss, vertigo, tinnitus
Progressive ipsilateral tinnitus sensorineural deafness (cochlear nerve
compression).
cranial nerve V: absent corneal reflex
cranial nerve VII: facial palsy
Tests:
MRI of the cerebellopontine angle is the investigation of choice
Treatment:
Surgery: (difficult, and often not needed, eg if elderly)
Hearing and the facial nerve can be preserved
(eg continuous intraoperative monitoring of evoked electromyograms).
Gamma knife surgery is one good way to neuroma volume.

1434. A 22yo man reports a 2d hx of hoarseness of voice. He denies any weight loss but
he has been smoking for 4yrs. What is the single most appropriate inv?

a. None
b. Laryngoscopy
c. Bronchoscopy
d. BAL
e. CXR
Answer: A. None. Young patient, only presents with a 2 day h/o hoarseness of voice and
denies weight loss so CA is less likely and there is no need to investigate.
HOARSENESS OF VOICE
Hoarseness entails difficulty producing sound with change in voice pitch or quality
(breathy,
scratchy, husky).
Investigate hoarseness (esp in smokers) lasting >3wks, as it is the chief (and
often the only) presentation of laryngeal carcinoma.
Ask about:
Gastrooesophageal reflux (GORD), dysphagia, smoking, stress, singing & shouting.
Voice overuse is a common cause (prevalence >50% in eg in teachers).
Tests:
Laryngoscopy (to assess cord mobility, inspect the mucosa, exclude local causes);
slowmotion videolaryngostroboscopy/acoustic analysis
(causes have characteristic vibration patterns).
1435. A 34yo IVDA presents with a 4m hx of productive cough. He has lost 10kgs. What
is the single
most appropriate inv?
a. Sputum for AFB
b. Laryngoscopy
c. Bronchoscopy
d. CT neck
e. CXR
Answer= A. sputum for AFB. chronic productive cough and h/o weight loss points
towards the diagnosis of tuberculosis so sputum for AFB should be done.
Tuberculosis (TB) is an infection caused by Mycobacterium tuberculosis that most commonly
affects the lungs.

primary tuberculosis=
A nonimmune host who is exposed to M. tuberculosis may develop primary infection of the
lungs. A small lung lesion known as a Ghon focus develops. The Ghon focus is composed of
tubercle-laden macrophages. The combination of a Ghon focus and hilar lymph nodes is
known as a Ghon complex.
In immunocompetent people the initially lesion usually heals by fibrosis. Those who are
immunocompromised may develop disseminated disease (miliary tuberculosis).
secondary tuberculosis=
If the host becomes immunocompromised the initial infection may become reactivated.
Reactivation generally occurs in the apex of the lungs and may spread locally or to more
distant sites. Possible causes of immuno compromise include: immunosuppressive drugs
like steroids, Hiv, malignancy.

The lungs remain the most common site for secondary tuberculosis. Extrapulmonary
infection may occur in the following areas: central nervous system (tuberculous meningitis the most serious complication), vertebral bodies (Pott's disease), cervical lymph nodes
(scrofuloderma), renal, gastrointestinal tract.
risk factors= close contacts of TB patient, alcoholics and other drug abusers, hiv positive and
other immunocompromised patients.etc
presentation=
general symptoms: fever, malaise, night sweats, fatigue, weight loss etc
pulmonary Symptoms= include chronic, productive cough with purulent bloodstained
sputum. May result in lobar collapse, bronchiectasis, pleural effusion and pneumonia.

1436. A 25yo pt came to the OPD with complaint of fever, malaise, breathlessness,
cough and anorexia. His gf has got similar symptoms. He had hx of sore throat and ear
discharge a month ago. What
is the single most likely causative organism?
a. Legionella
b. Mycoplasma
c. Chlamydia pneumoniae
d. PCP
e. Chlamydia psittaci
Answer: C.chlamydia pneumoniae. person to person contact, h/o sore throat and ear
discharge are diagnostic points.
Chlamydophila pneumoniae is the commonest chlamydial infection. person to person
spread occurs. features include pharyngitis, hoarseness, otitis followed by pneumonia.
Diagnosis= chlamydophila complement fixation test,pcr invasive samples
treatment= doxycycline or clarithromycin.
1437. A 72yo male presents with acute confusion. He has been in the hosp for 2wks
having been
treated for a DVT. The nurses have noticed that he became increasingly drowsy. Exam:
small
scalp laceration, a GCS of 8 and bilateral up-going plantar response.
a. Infection toxicity
b. Delirium tremens
c. Extradural hematoma
d. Subdural hematoma
e. Electrolyte imbalance
Answer= D. subdural Hematoma. elderly patient, having been treated for DVT so on
anticoagulants,increasing drowsiness, scalp lacerations (head trauma) all points towards
the diagnosis.
exclude other causes=
extradural hematoma= almost always there his h/o of head injury, both extradural and
sub dural have lucid interval. But extra dural has shorter time duration presenting in
hours to days while subdural may take weeks to months to present.

SUBDURAL HEMATOMA
Consider in all whose conscious level fluctuates, and elderly and also in those having an
evolving stroke, especially if on anticoagulants.
Bleeding is from bridging veins between cortex and venous sinuses (vulnerable to
deceleration
injury), resulting in accumulating haematoma between dura and arachnoid.
This gradually raises ICP, shifting midline structures away from the side of the clot
and, if untreated, eventual tentorial herniation and coning.
Most subdurals are from trauma but the trauma is often forgotten as it was so minor or
so long ago (up to 9 months).
It can also occur without trauma.
The elderly are most susceptible, as brain atrophy makes bridging veins
vulnerable. Other risk factors: falls (epileptics, alcoholics); anticoagulation.
Symptoms:
Fluctuating level of consciousness (seen in 35%) insidious physical or intellectual
slowing, sleepiness, headache, personality change, and unsteadiness.
Signs:
Raised ICP, seizures
Imaging:
CT/MRI shows clot midline shift
Look for crescent-shaped collection of blood over 1 hemisphere. The sickle shape
differentiates subdural blood from extradural haemorrhage.
Treatment:
1st line= Irrigation/evacuation, eg via burr twist drill and burr hole craniostomy.
2nd-line= craniotomy.
1438. A 50yo DM pt came to the OPD with complaint of of fever, muscle ache, dry cough
and anorexia. Inv: CXR=upper lobe cavitation. What is the single most likely causative
organism?
a. Legionella
b. Mycoplasma
c. Staphylococcus
d. Klebsiella
e. Streptococcus
Answer= D. Klebsiella. elderly, diabetic,upper lobe cavitation all points towards the
diagnosis.
klebsiella pneumonia occurs in elderly, diabetics and alcoholics. flu like symptoms.It
causes a cavitating pneumonia ,particularly of upper lobes,which is often drug resistant.
treatment= cefotaxime or imipenem

1439. A 20yo man complains that all his movements are being watched. Sometimes he
feels as though his actions are being controlled by his radio. At other times he is aware
of voices describing what he is doing. What is the most probable dx?
a. Mania
b. Drug induced psychosis
c. Delusion of control
d. Schizophrenia
e. Korsakoff psychosis
Answer= D. Schizophrenia. Auditory hallucinations, especially the echoing of thoughts, or a
third person 'commentary' on one's actions.Thought insertion, removal or interruption delusions about external control of thought.
Thought broadcasting - the delusion that others can hear one's thoughts are the features of
schizophrenia pointing towards the diagnosis.
Schizophrenia is the most common form of psychosis. It is a lifelong, condition, which can
take on either a chronic form or a form with relapsing and remitting episodes of acute illness.
Multiple factors are involved in schizophrenia - eg, genetic, environmental and social.
risk factors include= family history, intrauterine and perinatal complications,Intrauterine
infection, particularly viral, Abnormal early cognitive/neuromuscular development etc
features=
The hallmark symptoms of a psychotic illness are: Delusions,Hallucinations (auditory
hallucination like two or more voices discussing the patient in the third person,thought
echo,voices commenting on the patient's behaviour), Thought disorder (thought
insertion,thought withdrawal,thought broadcasting), Lack of insight.
These first Rank or positive symptoms of schizophrenia are absent in other psychotic
disorders.
NICE published guidelines on the management of schizophrenia in 2009. Key points:

oral atypical antipsychotics are first-line


cognitive behavioural therapy should be offered to all patients
close attention should be paid to cardiovascular risk-factor modification due to the
high rates of cardiovascular disease in schizophrenic patients (linked to antipsychotic
medication and high smoking rates)

1440. A 35yo is agitated and euphoric. He claims to be helping the prime minister with
economic
policy, although this is not true when checked. What is the most likely dx?
a. Mania
b. Schizophrenia
c. Hypomania
d. Drug induced personality disorder
e. Delusion of grandeur
Answer= E. Delusions of grandeur. it's characterized by exaggerated belief of
knowledge, power, self worth, relation to a famous person.

DELUSIONS
A delusion is a false belief which is firmly sustained and based on incorrect inference about
reality. This belief is held despite evidence to the contrary and is not accounted for by the
person's culture or religion.
Types of Delusions:
Monothematic - delusions are only relating to one particular topic.
Polythematic - range of delusional topics (seen in schizophrenia).
They can also be classified as:
Primary - occur in the mind, fully formed with no preceding reasons; strongly suggestive of
schizophrenia.
Secondary - eg depressed person feeling worthless.
Examples:
Delusional jealousy (Othello's syndrome) - eg believing a partner is being unfaithful.

Capgras' delusion - belief that a close relative has been replaced by someone
else who looks the same.
Unilateral neglect - belief that one limb or side does not exist.
Thought insertion - belief that someone is putting thoughts into the brain.
Grandiose delusion - belief of exaggerated self-worth.

1441. A 20yo student who recently visited Asia came to the OPD with complains of low
grade fever,
night sweats, anorexia and productive cough. Inv: CXR=cavitatory lesions in upper
lobes. What is
the single most likely causative organism?
a. Mycoplasma
b. Klebsiella
c. TB
d. PCP
e. Viral pneumonia
Answer= C. Tuberculosis. travel history, low grade fever with night sweats, anorexia and
productive cough, cavitation on chest x ray all are features of tuberculosis.
investigations for pulmonary TB:
Chest xray= cavitation, consolidation, fibrosis, calcification.
Microbiological samples: firm diagnosis rests on isolating the infecting organism, and
subsequent sensitivity testing can be used to guide antibiotic therapy.
for respiratory Tb = Send at least three spontaneous sputum samples for culture and
microscopy (including one early morning sample). Samples are analysed by Staining with
Ziehl-Neelsen (ZN) stain and rapid direct microscopy for acid/alcohol-fast bacilli.
Histologic = hallmark is caseating granulomas
Mantoux test for contact tracing.
Drug treatment for respiratory TB=
The standard therapy for treating active tuberculosis is:

Initial phase - first 2 months (RIPE)


Rifampicin, Isoniazid, Pyrazinamide, Ethambutol
(the 2006 NICE guidelines now recommend giving a 'fourth drug' such as ethambutol
routinely - previously this was only added if drug-resistant tuberculosis was suspected)
Continuation phase - next 4 months = Rifampicin,Isoniazid
The treatment for latent tuberculosis is isoniazid alone for 6 months
Patients with meningeal tuberculosis are treated for a prolonged period (at least 12 months)
with the addition of steroids.
Directly observed therapy with a three times a week dosing regimen may be
indicated in certain groups, including: homeless people with active tuberculosis, patients who
are likely to have poor concordance, all prisoners with active or latent tuberculosis.

1442. A 35yo man with T1DM is dehydrated with BP=90/50mmHg. What is the single
most
appropriate initial inv?
a. ABG
b. CBC
c. HbA1c
d. LFT
e. BUE
Answer= A. ABGS. type one diabetic, dehydrated, low BP are the features of Diabetic
ketoacidosis for which ABGs are done to confirm it and they will show Acidosis (low ph)
and low bicarbonate.
Diabetic Ketoacidosis(DKA) is a medical emergency that is characterized by
Hyperglycemia, Acidaemia (ph< 7.3) and ketonaemia.
American Diabetes Association diagnostic criteria are as follows:
blood glucose >13.8 mmol/l
pH < 7.30
serum bicarbonate <18 mmol/l
anion gap > 10
ketonaemia
The most common precipitating factors of diabetic ketoacidosis (DKA) are infection, missed
insulin doses and myocardial infarction.
the most common symptoms include polydipsia, polyuria, nausea, vomiting,
dehydration,gradual drowsiness,coma or deep breathing (kussmaul hyperventilation.
Arterial blood gases - metabolic acidosis with low pH and low HCO3; pCO2 should be normal
but can be depressed by respiratory compensation; low pO2 may indicate primary respiratory
problem as a precipitant.

Management: fluid replacement: most patients with DKA are deplete around 5-8 litres.
Isotonic saline is used initially
insulin: an intravenous infusion should be started at 0.1 unit/kg/hour. Once blood glucose is
< 15 mmol/l an infusion of 5% dextrose should be started

correction of hypokalaemia

1443. A 45yo woman presents with pruritis. Exam: skin pigmentation. Inv: raised ALP
and presence of anti-mitochondrial antibodies. What is the single most likely dx?
a. Psoriasis
b. Scabies
c. Atopic eczema
d. Dermatitis herpetiformis
e. Hyperthyroidism
f. Primary biliary cirrhosis
Answer= F. primary biliary cirrhosis. Middle aged women presenting with pruritis, skin
pigmentation and the ALP is raised along with the presence of anti microbial antibodies
all these features point towards the diagnosis.
Primary biliary cirrhosis is a chronic liver disorder typically seen in middle-aged females
(female:male ratio of 9:1).
The aetiology is not fully understood although it is thought to be an autoimmune condition.
Interlobular bile ducts become damaged by a chronic inflammatory process causing
progressive cholestasis which may eventually progress to cirrhosis. The classic
presentation is itching in a middle-aged woman
Antimitochondrial antibodies (AMA) are the hallmark of PBC.

Risk if:
+ve family history, many UTIS; smoking; past pregnancy; other autoimmune
diseases; use of nail polish/hair dye.
Features:
Lethargy, sleepiness, and pruritus may precede jaundice by years, Jaundice;
skin pigmentation; xanthelasma; xanthomata; hepatosplenomegaly
Associations:
Sjogren's syndrome (seen in up to 80% of patients)
rheumatoid arthritis
systemic sclerosis
thyroid disease
Tests:
Blood: Alk phos,gamma GT, and mildly AST & ALT; late disease: bilirubin, albumin,
prothrombin time
anti-mitochondrial antibodies (AMA) M2 subtype are present in 98% of patients and are
highly specific
smooth muscle antibodies in 30% of patients
raised serum IgM
Management:
pruritus: cholestyramine
fat-soluble vitamin supplementation
ursodeoxycholic acid
liver transplantation e.g. if bilirubin > 100 (PBC is a major indication) - recurrence in graft can
occur but is not usually a problem

1444. A 60yo man complains of tiredness, lethargy and itching that is severe after a hot
bath. He also has nocturia, polyuria and nausea and vomiting. Exam: pallor,
pigmentation and generalized edema. What is the single most likely dx?
a. Hyperthyroidism b. Lichen planus
c. Lymphoma
d. Eczema
e. Liver failure
f. CRF
Answer: F. CRF (Chronic Renal Failure), tiredness, lethargy, itching, nocturia,
polyuria,pigmentation, generalized edema all point towards the diagnosis.
CHRONIC RENAL FAILURE
Impaired renal function for >3 months based on abnormal structure or function, or GFR
<60mL/min/1.73m2 for >3 months with or without evidence of kidney damage.
Symptoms usually only occur once stage 4 is reached (GFR <30).
End-stage renal failure (ESRF) is defined as GFR <15 mL/min/1.73m2
or need for renal replacement therapy (RRTdialysis or transplant)
Common causes of chronic kidney disease
diabetic nephropathy
chronic glomerulonephritis
chronic pyelonephritis
hypertension
adult polycystic kidney disease
pyelonephritis and reflux nephropathy
Symptoms:It may be discovered by chance following a routine blood or urine test.

Specific symptoms usually develop only in severe CKD, and include anorexia,
nausea, vomiting, fatigue, weakness, pruritus, lethargy, peripheral oedema,
dyspnoea, insomnia, muscle cramps, pulmonary oedema, nocturia, polyuria
and headache.
Sexual dysfunction is common.
Hiccups, pericarditis, coma and seizures are only seen in very severe CKD.

CKD may be classified according to GFR:

CKD
stage

GFR range

Greater than 90 ml/min, with some sign of kidney damage on other tests (if all
the kidney tests* are normal, there is no CKD)

60-90 ml/min with some sign of kidney damage (if kidney tests* are normal,
there is no CKD)

3a

45-59 ml/min, a moderate reduction in kidney function

3b

30-44 ml/min, a moderate reduction in kidney function

15-29 ml/min, a severe reduction in kidney function

Less than 15 ml/min, established kidney failure - dialysis or a kidney


transplant may be needed

Tests:
Blood: Hb (normochromic, normocytic anaemia), ESR, U&E, glucose (DM),
Ca2+, PO4, alk phos (renal osteodystrophy).
PTH if CKD stage 3 or more.
Urine: Dipstick, MC&S, albumin: creatinine ratio or protein : creatinine ratio (p286).
Imaging: USS to check size, anatomy and corticomedullary differentiation.
In CKD kidneys are usually small (<9cm) but can be enlarged in infiltrative disorders
(amyloid,
myeloma), APKD and DM.
Histology: Consider renal biopsy if rapidly progressive disease or unclear cause and normal
sized kidneys.
Management:
Refer early to a nephrologist if the patient meets any of the following criteria:
Stage 4 and 5 CKD
Moderate proteinuria (ACR >70mg/mmol, see p286) unless due to DM and already
appropriately treated
Proteinuria with haematuria
Rapidly falling eGFR (>5mL/min/1.73m2 in 1yr, or >10mL/min/1.73m2 within 5yrs)
BP poorly controlled despite 4 antihypertensive drugs at therapeutic doses
Known or suspected rare or genetic causes of CKD
Suspected renal artery stenosis
Management of patients can be split into four main approaches:
investigations, limiting progression/complications, symptom control and preperation for renal
replacement therapy.

1445. A 30yo man complains of vague pain in the loin with BP=140/90mmHg. He is
found to have proteinuria and hematuria. What is the inv to confirm the dx?
a. Abdominal US

b. ANCA
c. ANA
d. Urine microscopy and culture
e. Stool culture
Answer= A. Abdominal US. Pain in the loin with hematuria and proteinuria and with high
blood pressure points towards the Polycystic Kidney Disease. so investigation of choice
in this case would be Abdominal US to confirm the diagnosis.
ADULT POLYCYSTIC KIDNEY DISEASE
It is an autosomal dominant condition. 85% of patients have mutations in PKD1
(chromosome 16) and Remainder have mutation in PKD2 (chromosome 4)
Features=
Loin pain is the most common symptom.
Hypertension is a common presenting feature
Bilateral kidney enlargement - abdominal examination may reveal enlarged and palpable
kidneys.
Gross haematuria following trauma (haemorrhage into a cyst) is a classic presenting feature
of ADPKD. It occurs in 30-50%.
Cyst infection, renal calculi, progressive renal
failure.
Extrarenal: liver cysts, intracranial aneurysm, SAH (subarachnoid haemorrhage), mitral valve
prolapse, ovarian cysts and diverticular disease
Investigations=
routine blood tests: fbc, urea,cr and electrolytes, GFR etc
urine analysis
Imaging is used to establish the diagnosis and to monitor disease progression
USS screening offers good sensitivity and specificity depending
on age.
Management=
Monitor U&E.
BP should be treated aggressively, with target levels of <130/80 mmHg (ACE-i are best
choices).
Treat infections, dialysis or transplantation for End stage renal failure, genetic
counselling.
Pain may be helped by laparoscopic cyst removal or nephrectomy.
Water intake, Na+ intake, and avoiding caffeine may also help.

1446. A 54yo man comes with sudden onset of palpitations and breathlessness. His
HR=164bpm. What is the single most appropriate tx in the acute phase?
a. Adenosine
b. Metaprolol
c. Verapamil
d. Amiodarone
Answer= A. Adenosine. sudden onset palpitations and breathlessness is SVT. For those
not hemodynamically compromised after Vagal Maneuvers first appropriate treatment in
acute phase is Adenosine.
SUPRAVENTRICULAR TACHYCARDIA

SVT is usually paroxysmal and episodes may occur regularly or very infrequently
(sometimes years apart). Episodes may only last for a few minutes or may last for up to
several months.
Episodes are characterised by the sudden onset of a narrow complex tachycardia, typically
an atrioventricular nodal reentry tachycardia (AVNRT). Other causes include atrioventricular
re-entry tachycardias (AVRT) and junctional tachycardias.
Risk Factors= SVT may occur at any age but often occurs in younger people in the absence
of heart disease.
Risk factors include: Previous Myocardial infarction, mitral valve prolapse, rheumatic
heart diseases, pericarditis, pneumonia, chronic lung disease, chronic alcohol
intoxication, digoxin toxicity
Presentation: Palpitations and dizziness are the most common symptoms reported.
Other symptoms including fatigue, light-headedness, chest discomfort, dyspnoea, polyuria
and syncope.
Investigations:
ECG= P wave absent or inverted after QRS.
Management=
Vagal manoeuvres (breath-holding, valsalva manoeuvre, carotid massage) are
1st line treatments if haemodynamically stable. IV adenosine is the drug of choice.
If adenosine fails, use verapamil 5 mg IV over 2min, or over 3min if elderly.
If haemodynamically unstable= DC (direct current) cardioversion.

1447. A 29yo woman has developed and itchy scaly rash particularly over her wrist with
fine white streaks overlying the lesion. Her nails have ridges and her buccal mucosa is
lined with a lacy white pattern. What is the single most likely dx?
a. Psoriasis
b. Scabies
c. Urticaria
d. Dermatitis herpetiformis
e. Hyperthyroidism
f. Lichen planus
Answer= F. Lichen Planus. itchy scaly rash particularly over wrist, white lace pattern all
point towards the diagnosis.
Exclusion of other options:
Psoriasis= well defined scaly red/ pink plaques with silvery scale on extensor aspects of
the elbows, knees, scalp and sacrum.
Scabies= widespread pruritus, linear burrows on the side of fingers, interdigital webs and
flexor aspects of the wrist
Dermatitis herpetiformis= itchy, vesicular skin lesions on the extensor surfaces (e.g.
elbows, knees, buttocks). it is associated with coeliac disease and is caused by deposition of
IgA in the dermis.
Lichen Planus
Lichen planus is a skin disorder of unknown aetiology, most probably being immune
mediated.
Features= lesions e.g on flexor aspects of wrist,forearms, ankles and legs
purple, pruritic, poly angular, planar (flat topped) papules, seen at any age + white lacy
markings ( known as Wickhams striae).

lesions elsewhere: scalp (scarring alopecia), nails ( longitudinal ridges), tongue, mouth (lacy
white areas on inner cheeks) and genital lesions.
Lesions often arise at the site of trauma.
usually persist for 6-18 months.
Lichenoid drug eruptions - causes: gold, quinine, thiazides
Treatment=
Topical Steroids ( with or without topical antifungals) are 1st line (esp in oral disease e.g
fluticasone spray).
extensive lichen planus may require oral steroids or immunosuppression

1448. The artery that runs in the ant inter-ventricular groove. What is the single most
appropriate option?
a. Acute marginal branch
b. Left ant descending artery
c. Coronary sinus
d. Circumflex artery
e. Right coronary artery
Answer= B. Left Anterior Descending Artery.
1449. Which virus is transmitted by the fecal-oral route?
a. Hep C
b. Coxsackie virus
c. Dengue
d. None of the above
Answer= B. Coxsackie virus. Spread is usually from the faeco-oral route or oral-oral route,
with an incubation period of 2-6 days.
The Coxsackieviruses are RNA viruses of the Picornaviridae family, Enterovirus genus
which includes echoviruses and polioviruses. Infections are often asymptomatic. They are
divided into groups A (Usually affects skin and mucous membranes ) and B (Usually affects
the heart, lungs, pancreas and liver).
Coxsackieviruses of both types are a leading cause of aseptic meningitis. They may also
cause nonspecific febrile and upper respiratory tract illness.
Usually diagnosis is clinical but some laboratory tests are available.
The virus can be isolated from throat, vesicle or rectal swabs (placed in viral transport
medium) or from faecal culture
IgM with enzyme-linked immunosorbent assay (ELISA) can aid diagnosis.
There is no known treatment for Coxsackievirus infections, so management is supportive

1450. A 40yo woman presented with generalized itching and soreness for few months.
She gave a hx of heavy menstrual periods. Exam: pallor. What is the single most likely
causative factor?
a. IDA
b. Lichen planus
c. Dermatitis herpetiformis
d. Eczema
e. Uremia

Answer= A. IDA (iron deficiency anemia). features of anemia like tiredness and pallor
due to the menorrhagia points towards the diagnosis
Iron Deficiency Anemia
it is microcytic hypochromic anemia. most common cause of anemia. (seen in up to 14%
of menstruating women).
Causes:
Blood loss, eg menorrhagia or GI bleeding
Poor diet may cause IDA in babies or children (but rarely in adults), those on special
diets, or wherever there is poverty.
Malabsorption (eg coeliac disease) is a cause of refractory IDA.
In the Tropics, hookworm (GI blood loss) is the most common cause.
Presentation:
koilonychia, atrophic glossitis, angular cheilosis and rarely, post-cricoid webs (PlummerVinson syndrome).
Tests:
Microcytic, hypochromic anaemia with anisocytosis and poikilocytosis
MCV, MCH and MCHC. Confi rmed by ferritin (also serum iron with
total iron-binding capacityTIBC, but these are less reliable).
Management:
Treat the cause.
Oral iron, eg ferrous sulfate 200mg/8h PO.( SE: nausea,
abdominal discomfort, diarrhoea or constipation, black stools).
Continue until Hb is normal and for at least 3 months, to replenish stores.
1451. A 7yo child presents with lesions on the trunk. Exam: some lesions are weeping
and others are crusted with a red base. What is the causative organism?
a. Herpes simplex
b. Varicella zoster
c. Rubella virus
d. Herpes zoster
Answer= B. Varicella zoster. lesions on the trunk that are weeping and crusting are all
the features of chickenpox
CHICKEN POX
Chickenpox is caused by primary infection with varicella zoster virus. Shingles is reactivation
of dormant virus in dorsal root ganglion.
Chickenpox is highly infectious
spread via the respiratory route
can be caught from someone with shingles
infectivity = 4 days before rash, until 5 days after the rash first appeared
incubation period = 10-21 days
Clinical features (tend to be more severe in older children/adults)

fever initially
itchy, rash starting on head/trunk before spreading. Initially macular then papular
then vesicular

systemic upset is usually mild

Management is supportive:
keep cool, trim nails, calamine lotion
school exclusion: current HPA advice is 5 days from start of skin eruption. They also state
'Traditionally children have been excluded until all lesions are crusted. However,
transmission has never been reported beyond the fifth day of the rash.' ( oxford says
until lesions scabbed)
immunocompromised patients and newborns with peripartum exposure should receive
varicella zoster immunoglobulin (VZIG). If chickenpox develops then IV aciclovir should be
considered
Complications: A common complication is secondary bacterial infection of the lesions.
Others: purpura fulminans/DIC, pneumonitis, and ataxia, are commoner in pregnancy and
adults than in children.
Shingles Treatment= acute zoster, eg with aciclovir 800mg 5 times/d PO for 7d if eGFR
>25;
if immunocompromised: 10 mg/kg/8h slowly IVI for 10d;
alternative: famciclovir or valaciclovir
If conjunctiva affected, use 3% acyclovir ointment Beware iritis; test acuity often.

1452. An 87yo woman with a hx of HTN has acute breathlessness. She has a
RR=32bpm, widespread lung crackles, pulse=120bpm, BP=160/90mmHg and elevated
venous pressure. Her peripheral O2 sat=85%. What is the single most appropriate initial
management?
a. IV antibiotics
b. IV furosemide
c. Nitrate infusion
d. Neb. Salbutamol
e. 100% oxygen
Answer= E. 100% oxygen. the pt is hypertensive with acute breathlessness and raised
JVP pointing towards the diagnosis of Acute Heart Failure.
Management of acute heart failure
Sit the patient upright

Oxygen
100% if no pre-existing lung disease

IV access and monitor ECG


Treat any arrhythmias, eg AF

Investigations whilst continuing treatment

Diamorphine 1.255mg IV slowly


Caution in liver failure and COPD

Furosemide 4080 mg IV slowly


Larger doses required in renal failure

GTN spray 2 puffs SL or 2 0.3mg tablets SL


Dont give if systolic BP <90 mmHg

Necessary investigations, examination, and history

If systolic BP 100 mmHg, start a nitrate infusion,


eg isosorbide dinitrate 210mg/h IVI; keep systolic BP 90 mmHg

If the patient is worsening:


Further dose of furosemide 4080mg
Consider CPAPimproves ventilation by recruiting more alveoli, driving fluid out
of alveolar spaces and into vasculature (get help before initiating!)
Increase nitrate infusion if able to do so without dropping systolic BP <100

If systolic BP <100 mmHg, treat as cardiogenic shock and refer to ICU


1453. A 25yo man presented with painless cervical lymphadenopathy with lethargy,
night sweats and itching. What is the single most likely causative factor?
a. Lymphoma
b. Polycythemia
c. IDA
d. Uremia
e. Drug induced
Answer= A. Lymphoma. painless cervical lymphadenopathy with lethargy, itching and
night sweats are the features of hodgkin's lymphoma
Hodgkins lymphoma: Lymphomas are disorders caused by malignant proliferations of
lymphocytes. These accumulate in the lymph nodes causing lymphadenopathy.In
Hodgkins lymphoma,characteristic cells with mirror-image nuclei are found, called
ReedSternberg cells
Cause: 2 peaks of incidence: young adults and elderly. EBV has a role in causing it.
Symptoms Often presents with enlarged, painless, non-tender, rubbery superficial
lymph nodes, typically cervical (6070%), also axillary or inguinal nodes. Nodes may
become matted. 25% have constitutional symptoms.
Mediastinal lymph node involvement can cause features due to mass effect, eg
bronchial or SVC obstruction or direct extension, eg causing pleural effusions.
Signs Lymph node enlargement. Also, cachexia, anaemia, spleno- or hepatomegaly.
Types:

Tests Tissue diagnosis: Lymph node excision biopsy if possible. Inc ESR or dec Hb
show worse prognosis.
Staging is by Ann-Arbor system.
Treatment: Depends on chemotherapy and radiotherapy.
1454.A 25yo male presents with fever and pain in the right lower thigh of 1m duration. Exam:
lower third of his thigh is red, hot and tender. The XR showed new bone formation. What
is the most probable dx?
a. Osteosarcomoa
b. Ewings sarcoma
c. Tuberculus arthritis
d. Exotosis
e. Fibrosarcoma
Answer= B. Ewings Sarcoma.
Ewings Sarcoma
it is the malignant round cell tumor of long bones (typically diaphysis) and limb girdles,
usually presents in adolescents.
Radiographs show bony destruction, concentric layers of new bone formation (onion
ring sign) and a soft tissue mass. MRI is helpful.
typically those with ewing sarcoma have a t 11:22 chromosomal translocation.
Treatment=
Chemotherapy, surgery and radiotherapy are required. The key adverse prognostic
factor is metastases at diagnosis

1455. A 76yo man presents with sore throat, local irritation by hot food, dysphagia and a
sensation of
a lump in his throat. He has a 20y hx of smoking. What is the single most likely dx?
a. Nasopharyngeal ca
b. Pharyngeal ca
c. Sinus squamous cell ca
d. Squamous cell laryngeal ca
e. Hypopharyngeal ca
Answer= B. Pharyngeal carcinoma. elderly, sore throat, dysphagia and sensation of
lump in the throat are the features of pharyngeal carcinoma.
Exclusion of other options:
Nasopharyngeal ca= most likely to cause a lump in the neck but may also cause nasal
obstruction, deafness and postnasal discharge.
Hypopharyngeal ca= problems with swallowing and ear pain are common symptoms and
hoarseness is not uncommon.
Pharyngeal carcinoma
Cancer of the pharynx is less common than other head and neck cancers. It occurs in three
locations: oropharynx, nasopharynx and hypopharynx.
Oropharyngeal cancer = typically affects people aged between 40 and 70 years
common symptoms: are a persistent sore throat, a lump in the mouth or throat, pain in the
ear.
Investigation: Biopsy is the only way to establish the diagnosis. A fine-needle aspiration
(FNA) or biopsy may be an alternative for a neck mass; lesions that are harder to reach may
require endoscopy.
Treatment:
surgery (using open or minimally invasive approaches for tumour resection and
reconstruction), radiotherapy, chemotherapy, or a combination of these methods.

1456. A 42yo female who is obese comes with severe upper abdominal pain and right
shoulder tip pain with a temp=37.8C. She has 5 children. What is the most probable dx?
a. ERCP
b. LFT
c. Serum amylase
d. MRCP
e. US abdomen
Answer= E. US Abdomen. Female, fat, fertile and forty with upper abdominal pain points
towards the diagnosis of gall stones for which investigation of choice is US abdomen.
Gallstones:
Pigment stones: (<10%)
Small, friable, and irregular. Causes: haemolysis.
Cholesterol stones: Large, often
solitary. Causes: age, obesity (Admirands triangle: risk of stone if lecithin, bile
salts, cholesterol).
Mixed stones: Faceted (calcium salts, pigment, and cholesterol).

Gallstone prevalence: 8% of those over 40yrs. 90% remain asymptomatic.


Risk factors for stones becoming symptomatic: smoking; parity.
Acute cholecystitis follows stone or sludge impaction in the neck of the gallbladder,
which may cause continuous epigastric or RUQ pain (referred to the right shoulder,
vomiting, fever, local peritonism, or a GB mass. If the stone moves to the common bile
duct (CBD), obstructive jaundice and cholangitis may occur
Tests: WCC,
Ultrasound Is the best way to demonstrate stones, being 90-95% sensitive.
it shows a thick-walled, shrunken GB (also seen in chronic disease)
Treatment:
NBM, pain relief, IVI, and eg cefuroxime 1.5g/8h IV.
Laparoscopic cholecystectomy is the treatment of choice for all patients fit for
GA.
Open surgery is required if there is GB perforation.
Cholecystostomy is also the preferred treatment for acalculous cholecystitis
1457. A 37yo laborer comes with hx of redness of left eye with foreign body sensation in
the same eye. What is the single most appropriate option?
a. Ciliary body
b. Sclera
c. Conjunctivitis
d. Cornea
e. Iris
Answer= D. Cornea. Injury is caused by a foreign body and foreign bodies most like
injure cornea leading to corneal ulcer if not removed.
Corneal injuries may be physical, chemical, environmental (eg, ultraviolet (UV) damage) or
infective.
Corneal abrasions are common. There is usually a history of minor trauma from a scratch,
grit or contact lens problem.
If there is no history of injury, consider infection - eg, herpes simplex infection, chemical
injury and corneal foreign bodies.
Symptoms of superficial corneal abrasion or corneal FB : Redness, pain, watering
(usually). FB sensation (usually), Blurred vision, Photophobia,Pain on eye movement,
Patients are fairly reliable at locating the FB.
Investigations: are not required if you can be sure that the injury is superficial.Plain X-rays of
of the orbit/face can be used to exclude known radiopaque FBs
Treatment= Removal of Foreign body. Use a topical anesthetic
Management of corneal abrasion= refer if large abrasion otherwise: Analgesia= Paracetamol
or ibuprofen are first line. topical antibiotics for 7 days= Chloramphenicol for 1st line.

1458. An 11yo boy came to the hospital with pain after falling off his bicycle. XR= fx at
distal radius with forward angulations. What is the single most probable dx?
a. Dinner fork deformity
b. Cubitus valgus

c. Gun stock deformity


d. Garden spade deformity
e. Genu valgus
Answer= D. Garden Spade Deformity. the fracture of distal radius with forward
angulations is smiths fracture the characteristic appearance of which is Garden Spade
Deformity.
Exclusion of other options:
Dinner Fork deformity= it is seen in Colles Fracture.Colles type (common in
osteoporotic postmenopausal women who fall on an outstretched hand). There is dorsal
angulation and displacement producing a dinner-fork wrist deformity.
SMITHs FRACTURE
Sometimes called a 'reverse Colles fracture',
In this injury, the distal radius fractures with the fragment angled and displaced
forward/palmarly.
It is usually caused by landing with the wrist in flexion - a backward fall on the palm of an
outstretched hand.
The characteristic appearance is called a 'garden spade deformity'.
The X-ray of a Smith's fracture is very similar to a Colles' fracture except with the
displacement anteriorly instead of posteriorly.
Fixation is needed in these fracture more commonly than in Colles fractures, as the
fracture fragment tends to migrate palmarly.

1459. A middle aged man with a lump in front of his neck which moves up while hes
swallowing. US shows a mass replacing the left lobe of thyroid. And spread to the
sternocleidomastoid and adjacent muscles. What is the most probable dx?
a. Thyroid ca
b. Pharyngeal pouch
c. Bronchus ca
d. Thyroid cyst
e. Larynx ca
Answer= A.Thyroid Carcinoma. lump in the neck, us showing mass replacing left lobe of
thyroid and metastasis points towards the diagnosis
Thyroid Cancers
1 Papillary: (60%). Often in younger patients. Spread: lymph nodes & lung
(jugulodigastric
node metastasis is the so-called lateral aberrant thyroid). : total thyroidectomy
to remove non-obvious tumour node excision radioiodine to ablate
residual cells. Give thyroxine to suppress TSH. Prognosis: better if young.
2 Follicular: (25%). Occur in middle-age & spreads early via blood (bone, lungs).
Well-differentiated. : total thyroidectomy + T4 suppression + radioiodine ablation.
3 Medullary: (5%). Sporadic (80%) or part of MEN syndrome (p215). May produce
calcitonin which can be used as a tumour marker. They do not concentrate iodine.
Perform a phaeochromocytoma screen pre-op. : thyroidectomy + node

clearance. External beam radio therapy should be considered to prevent regional


recurrence.
4 Lymphoma: (5%). May present with stridor or dysphagia. Do full staging
pre-treatment (chemoradiotherapy). Assess histology for mucosa-associated
lymphoid tissue (MALT) origin (associated with a good prognosis).
5 Anaplastic: Rare. :3:1. Elderly, poor response to any treatment. In the absence
of unresectable disease, excision + radiotherapy may be tried.
1460. A 28yo male complains of severe pain while trying to grasp any object. It started
since he
participated in skiing and had a fall and caught his thumb in the matting. Exam: rupture
of the
ulnar collateral ligament of MCP joint of the thumb. What is the single most probable
deformity?
a. Dinner fork deformity
b. Game keeper thumb
c. Mallet finger
d. Gun stock deformity
e. Garden spade deformity
Answer= B. GameKeepers Thumb. Rupture of ulnar collateral ligament of
metacarpophalangeal joint (MCP) during skiing points towards the diagnosis.
Gamekeepers thumb
This is so-called because of the laxity of the ulnar collateral ligament of the
metacarpophalangeal joint of the thumb during the forced thumb abduction that occurs
when wringing a pheasants neck.
The same injury is described in dry ski slope participants who fall and catch their
thumb in the matting (skier's thumb).
Diagnosis can be difficult as the thumb is so painful to examine.
Treatment:
complete tears = surgical. Radiographic
evaluation will detect a bony avulsion fragment.
Partial tears (clinically stable), or those associated with undisplaced avulsion
fractures of the proximal phalanx= simple
short-arm thumb spica casting.
1461. A 25yo male had an injury to the knee while playing football. XR=condylar fx of
tibia. What is the
single most probable deformity?
a. Dinner fork deformity
b. Gibbus
c. Cubitus valgus
d. Garden spade deformity
e. Genu valgus
Answer: E. Genu Valgus. (knock knee deformity seen in knee injuries) The typical gait
pattern is circumduction, requiring that the individual swing each leg outward while
walking in order to take a step without striking the planted limb with the moving limb.

Condylar Fracture of Tibia


The most common mechanism resulting in a tibial plateau fracture (condylar) is a valgus force with
axial loading. Of these fractures, 80% are motor vehiclerelated injuries, and the remainder are
sports-related injuries.
Trauma can be direct or can be related to a fall from a height, an industrial accident, or a sports injury.
Tibial plateau fractures may be either low-energy or high-energy. Low-energy fractures occur in
osteoporotic bone and typically are depressed fractures. High-energy fractures occur in low-energy
patients often as a result of motor vehiclerelated trauma, and the most common pattern of fracture in
this group is a splitting fracture.
Approximately 50% of the knees with closed tibial plateau fractures have injuries of the menisci and
cruciate ligaments that usually require surgical repair.
Because of the valgus stress at the moment of impact, the medial collateral ligament is at greater risk
than the lateral collateral ligament;
however, disruption of the lateral collateral ligament is of grave concern because of possible injuries to
the peroneal nerve and the popliteal vessels.
Management:
The following are absolute indications for surgery:

Open plateau fractures


Fractures with an associated compartment syndrome
Fractures associated with a vascular injury

1462. A 50yo man presents with itching after hot shower with dizziness, chest pain after
exercise. Exam: splenomegaly. What is the single most likely causative factor?
a. ALL
b. Lymphoma
c. Polycythemia
d. Scabies
e. Eczema
Answer= C. Polycythaemia. Itch after a hot bath, dizziness and splenomegaly are
characteristics of polycythaemia rubra vera
Polycythaemia rubra vera (PRV) is a myeloproliferative disorder caused by clonal
proliferation of a marrow stem cell leading to an increase in red cell volume, often
accompanied by overproduction of neutrophils and platelets.
A mutation in JAK2 is present in >90%.
Features:
hyperviscosity, headaches, dizziness, tinnitus, visual disturbance.
Itch after a hot bath, and erythromelalgia, a burning sensation in fingers and toes, are
characteristic.
Facial plethora and splenomegaly
Hypertension in a third of patients.
Investigations:
FBC: RCC, Hb, HCT, PCV, often also WBC and platelets B12 Marrow
shows hypercellularity with erythroid hyperplasia , serum erythropoietin
Raised red cell mass on
51Cr studies and splenomegaly, in the setting of a normal PaO2, is diagnostic
Treatment:

Aim to keep HCT <0.45 to risk of thrombosis. In younger patients at low risk, this is done by
venesection.
If higher risk (age >60yrs, previous thrombosis)= hydroxycarbamide (=hydroxyurea) is used.
-interferon is preferred in women
of childbearing age. Low-dose aspirin 75 mg daily PO is also given.

1463. A man presented with carcinoma of the bladder and has been working in factories.
He wants to know what dye has caused it. What is the single most likely cause?
a. Aniline
b. Asbestos
c. Latex
d. Silica
Answer= A. Aniline. Occupational exposure to aromatic amines (aniline) is the second
most important risk factor for bladder cancer first being smoking.
BLADDER CANCER:
In developed countries 90% of bladder cancers are transitional cell carcinomas.
Squamous cell carcinomas make up most of the remainder.
Grade 1differentiated;
Grade 2intermediate;
Grade 3poorly differentiated.
80% are confined to bladder mucosa, and only ~20% penetrate muscle.
Presentation:
Painless haematuria; recurrent UTIs; voiding irritability.
Risk Factors:
Smoking; aromatic amines (rubber industry); chronic cystitis; schistosomiasis
(risk of squamous cell carcinoma); pelvic irradiation.
Tests:
Cystoscopy with biopsy is diagnostic
Treatment:
Tis/Ta/T1: (80% of all patients) Diathermy via transurethral cystoscopy/transurethral
resection of bladder tumour (TURBT).
Consider intravesical chemotherapeutic agents for multiple small tumours or high-grade
tumours
T23: Radical cystectomy is the gold standard.
Post-op chemotherapy (eg M-VAC: methotrexate, vinblastine, adriamycin, and
cisplatin) is toxic but effective.
T4: Usually palliative chemo/radiotherapy
Tumour spread:
Local to pelvic structures;
lymphatic to iliac and para-aortic nodes;
haematogenous to liver and lungs.
TNM Staging:
Tis Carcinoma in situ Not felt at EUA

Ta Tumour confined to epithelium Not felt at EUA


T1 Tumour in lamina propria Not felt at EUA
T2 Superficial muscle involved Rubbery thickening at EUA
T3 Deep muscle involved EUA: mobile mass
T4 Invasion beyond bladder EUA: fixed mass
(EUA = examination under anaesthetic)

1464. A 62yo man presents with left sided hearing loss and tinnitus. He also complains
of vomiting and headache. Exam: papilledema and SNHL in the left ear. What is the
single most likely dx?
a. Meningioma
b. Nasopharyngeal ca
c. Acoustic neuroma
d. Pharyngeal ca
e. Menieres disease
Answer: C. Acoustic Neuroma. Tumor itself compresses 8th cranial nerve palsy causing
features like hearing loss, tinnitus which leads to the raised ICP causing headache,
vomiting, papilloedema all points towards the diagnosis.
Acoustic neuromas (more correctly called vestibular schwannomas) account for
approximately five percent of intracranial tumours and 90 percent of cerebellopontine angle.
These are typically indolent, histologically benign subarachnoid tumours
that cause problems by local pressure, and then behave as space-occupying
lesions
Features can be predicted by the affected cranial nerves
cranial nerve VIII: hearing loss, vertigo, tinnitus
Progressive ipsilateral tinnitus sensorineural deafness (cochlear nerve
compression).
cranial nerve V: absent corneal reflex
cranial nerve VII: facial palsy
Tests:
MRI of the cerebellopontine angle is the investigation of choice
Treatment:
Surgery: (difficult, and often not needed, eg if elderly)
Hearing and the facial nerve can be preserved
(eg continuous intraoperative monitoring of evoked electromyograms).
Gamma knife surgery is one good way to neuroma volume.

1465. A HIV +ve 55yo man presents with painless lymphadenopathy, fever, night sweats
and weight loss. What is the most probable dx?
a. Hodgkins lymphoma
b. NHL
c. ALL
d. AML

e. CML
Answer= B. NHL ( non Hodgkins Lymphoma) . Features like painless lymphadenopathy,
fever, night sweats and weight loss in HIV patient points towards the diagnosis of NHL.
This includes all lymphomas without ReedSternberg cells a diverse group.
Most are derived from B-cell lines; diffuse large B-cell lymphoma (DLBCL) is
commonest.
Causes:
Immunodeficiencydrugs; HIV, HTLV-1, H. pylori ; toxins; congenital
presentation:
Nodal disease (75% at presentation): superficial lymphadenopathy.
Extranodal disease (25%)Skin: T-cell lymphomas
Systemic symptomsfever, night sweats, weight loss (less common than in
Hodgkins lymphoma, and indicates disseminated disease).
Pancytopenia from marrow involvementanaemia, infection, bleeding (platelets).
Tests:
Blood: FBC, U&E, LFT, LDH worse prognosis, reflecting cell turnover.
Marrow and node biopsy for classification.
CT/MRI of chest, abdomen, pelvis for staging.
Send cytology of any effusion;
LP for CSF cytology if CNS signs.
Treatment:
Low grade: If symptomless, none may be needed. Radiotherapy may be curative in
localized disease.
Chlorambucil= diffuse disease.
Remissions =interferon or rituximab
High grade: (eg large B-cell lymphoma, DLBCL)=
R-CHOP regimen: Rituximab Cyclophosphamide, Hydroxydaunorubicin,
vincristine and Prednisolone.
1466. A 22yo man says that he can hear the voice of his deceased uncle telling him that
he is being spied on. The pt is distressed by this becoming low in mood and anxious and
has not left the house for 2wks. He is starting to drink increasing quantities of alcohol.
He is noticed to have thought-block and passivity phenomena. What is the single most
suitable med to treat his
symptom?
a. Diazepam
b. Disulfiram
c. Fluoxetine
d. Lithium
e. Olanzapine
Answer= E. Olanzapine. Auditory hallucinations, thought block and passivity points
towards the diagnosis of Schizophrenia for which atypical antipsychotics are 1st line.
Schizophrenia

NICE published guidelines on the management of schizophrenia in 2009.


Key points:
oral atypical antipsychotics are first-line
Examples of atypical antipsychotics
clozapine, olanzapine,risperidone,quetiapine,amisulpride
adverse effects:
weight gain
clozapine is associated with agranulocytosis

cognitive behavioural therapy should be offered to all patients


close attention should be paid to cardiovascular risk-factor modification due to the
high rates of cardiovascular disease in schizophrenic patients (linked to antipsychotic
medication and high smoking rates)

1467. A middle age Asian presents with episodes of fever with rigors and chills for last
1yr. Blood film: ring form of plasmodium with schuffner's dots in RBCs. What is the drug
to eradicate this infection?
a. Doxycycline
b. Mefloquine
c. Proguanil
d. Quinine
e. Artesunate
Answer= Mefloquine.Schffner's dots refers to a hematological finding that is associated
with malaria, exclusively found in Plasmodium ovale and Plasmodium vivax.
Plasmodium vivax induces morphologic alterations in infected host erythrocytes that are
visible by light microscopy as multiple brick-red dots. These morphologic changes, referred
to as Schffner's dots, are important in the identification of this species of malarial parasite
Non Falciparum Malaria
The most common cause of non-falciparum malaria is Plasmodium vivax, with Plasmodium
ovale and Plasmodium malariae accounting for the other cases.
Features
general features of malaria: fever, headache, splenomegaly
Plasmodium vivax/ovale: cyclical fever every 48 hours. Plasmodium malariae:
cyclical fever every 72 hours
Plasmodium malariae: is associated with nephrotic syndrome
Ovale and vivax malaria have a hypnozoite stage and may therefore relapse following
treatment.
Treatment
non-falciparum malarias are almost always chloroquine sensitive
patients with ovale or vivax malaria should be given primaquine following acute
treatment with chloroquine to destroy liver hypnozoites and prevent relapse

1468. A 50yo man presents with flight of ideas which are rambling and disinhibited. He is
distractible, confused and overactive. What is the most likely dx?
a. Dementia
b. Mania
c. Schizophrenia
d. Psychosis
e. Acute confusional state
Answer= B. Mania. Flight of ideas, distraction, confusion and overactive nature points
towards the diagnosis of mania.
Signs of mania
Mood:
Irritability (80%), euphoria (71%), lability (69%).
Cognition:
Grandiosity (78%); flight of ideas/racing thoughts (71%); distractibility/ poor
concentration (71%); confusion (25%), many conflicting lines of thought urgently racing
in contrary directions; lack of insight.
Behaviour:
Rapid speech (98%), hyperactivity (87%), sleep (81%), hypersexuality (57%),
extravagance (55%).
Psychotic symptoms: Delusions (48%), hallucinations (15%).
Less severe states are termed hypomania.
If depression alternates with mania, the term bipolar affective disorder is used .
Cyclical mood swings without the more florid features (as above) are termed
cyclothymia.
Causes=
Physical: Infections, hyperthyroidism; SLE; thrombotic thrombocytopenic purpura;
stroke; water dysregulation/Na+
Drugs: Amphetamines, cocaine, antidepressants (esp. venlafaxine), captopril, steroids,
procyclidine,L-dopa, baclofen.
Bipolar disorder: (Age at onset: <25.)
Tests= CT of the head, EEG, and screen for drugs/toxins.
Treating acute mania=
acute moderate/severe mania: olanzapine 10mg PO, adjust to
520mg/day (SE: weight; glucose), or valproate semisodium, eg 250mg/8h PO
Prophylaxis=
Those who have bipolar affective disorder after successful treatment of the manic or
depressive episode should have a mood stabilizer (lithium if good compliance) for
longer-term control.
1469. A pt presents with a lid lag, bulging eyes, ophthalmoplegia and thyroid bruit. What
inv will you do?
a. TFT
b. Eye sight

c. Tensilon test
d. US
e. FNAC
Answer= TFTS. lid lag, bulging eyes, ophthalmoplegia, thyroid bruit all points towards
the diagnosis of thyrotoxicosis (Graves disease) for which TFTS should be done.
Graves' disease is the most common cause of thyrotoxicosis. It is typically seen in women
aged 30-50 years.
Features
typical features of thyrotoxicosis
specific signs limited to Grave's (see below)
Features seen in Graves' but not in other causes of thyrotoxicosis
eye signs (30% of patients): exophthalmos, ophthalmoplegia
pretibial myxoedema
thyroid acropachy
Autoantibodies
anti-TSH receptor stimulating antibodies (90%)
anti-thyroid peroxidase antibodies (50%)
Treatment
1 Drugs: B-blockers (eg propranolol 40mg/6h) for rapid control of symptoms.
Anti-thyroid medication: 2 strategies (equally effective)
A) Titration, eg carbimazole 2040mg/24h PO for 4wks, reduce according to TFTs every 12
months.
B) Block-replace: Give carbimazole + thyroxine simultaneously (less risk of iatrogenic
hypothyroidism).
In Graves, maintain on either regimen for 1218 months then withdraw.
Carbimazole SE: agranulocytosis
2 Radioiodine: Most become hypothyroid post-treatment.
CI: pregnancy, lactation.
3 Thyroidectomy: Carries a risk of damage to recurrent laryngeal nerve (hoarse voice) and
hypoparathyroidism. Patients may become hypothyroid.

1470. A 30yo lady complaining of right ear deafness with decreased corneal reflex and
past pointing. Acoustic analysis shows SNHL. What is the next most appropriate inv to
do?
a. CT brain
b. CT acoustic canal
c. MRI brain
d. MRI acoustic canal
e. PET brain
Answer= C. MRI brain. unilateral ear deafness, SNHL (sensorineural hearing loss) and
decreased corneal reflex all points towards the diagnosis of Acoustic Neuroma for which
MRI is the investigation of choice.

Acoustic neuromas (more correctly called vestibular schwannomas) account for


approximately five percent of intracranial tumours and 90 percent of cerebellopontine angle.
These are typically indolent, histologically benign subarachnoid tumours
that cause problems by local pressure, and then behave as space-occupying
lesions
Features can be predicted by the affected cranial nerves
cranial nerve VIII: hearing loss, vertigo, tinnitus
Progressive ipsilateral tinnitus sensorineural deafness (cochlear nerve
compression).
cranial nerve V: absent corneal reflex
cranial nerve VII: facial palsy
Tests:
MRI of the cerebellopontine angle is the investigation of choice
Treatment:
Surgery: (difficult, and often not needed, eg if elderly)
Hearing and the facial nerve can be preserved
(eg continuous intraoperative monitoring of evoked electromyograms).
Gamma knife surgery is one good way to neuroma volume.

1471. A 29yo woman who returned from Egypt 2wks ago now presents with difficulty in
breathing, chest pain, cough and purulent sputum with an episode of blood staining. She
is on COCPs.
What is the most likely dx?
a. Pulmonary embolism
b. Pneumonia
c. Lung abscess
d. Pneumothorax
e. Pulmonary edema
Answer= Pneumonia. difficulty in breathing, chest pain, cough with purulent sputum all
points towards the diagnosis of pneumonia. cocps is a distracter here.
exclusion of other options:
Pulmonary Embolism= though cocps increase the risk of PE but no purulent sputum is
seen in it and it appears earlier.
Community acquired pneumonia (CAP) may be caused by the following infectious agents:
Streptococcus pneumoniae (accounts for around 80% of cases)
Haemophilus influenzae
Staphylococcus aureus: commonly after the 'flu
atypical pneumonias (e.g. Due to Mycoplasma pneumoniae)
viruses
Klebsiella pneumoniae is classically in alcoholics
Streptococcus pneumoniae (pneumococcus) is the most common cause of communityacquired pneumonia
Clinical features
Symptoms: Fever, rigors, malaise, anorexia, dyspnoea, cough, purulent sputum,
haemoptysis, and pleuritic pain.

Signs: Pyrexia, cyanosis, confusion (elderly usually), tachypnoea, tachycardia, hypotension,


signs of consolidation (diminished expansion, dull percussion note, tactile vocal
fremitus/vocal resonance, bronchial breathing), and a pleural rub.
Tests:
CXR: lobar or multilobar infiltrates, cavitation or pleural effusion.
Assess oxygenation: oxygen saturation (ABGs if SaO2 <92% or severe pneumonia) and
BP.
Blood tests: FBC, U&E, LFT, CRP, blood cultures. Sputum for microscopy and culture.
Pleural fluid may be aspirated for culture.
Consider bronchoscopy and bronchoalveolar lavage if patient is immunocompromised or on
ITU.
Management:
CURB-65 criteria of severe pneumonia

Confusion (abbreviated mental test score <= 8/10)


Urea > 7 mmol/L
Respiratory rate >= 30/min
BP: systolic <= 90 or diastolic <= 60 mmHg
age >= 65 years

Patients with 3 or more (out of 5) of the above criteria are regarded as having a severe
pneumonia
The British Thoracic Society published guidelines in 2009:

low or moderate severity CAP: oral amoxicillin. A macrolide should be added for
patients admitted to hospital
high severity CAP: intravenous co-amoxiclav + clarithromycin OR cefuroxime +
clarithromycin OR cefotaxime + clarithromycin
the current BNF has slightly different recommendations for high severity CAP:
intravenous benzylpenicillin + clarithromycin OR benzylpenicillin + doxycycline. For
'life-threatening' infections the BNF recommends the same as the BTS guidelines for
high-severity CAP

1472. A 60yo pt recovering from a surgery for toxic goiter is found to be hypotensive,
cyanosed in the recovery room. Exam: neck is tense. There is oozing of blood from the
drain. What is the most probable dx?
a. Thyroid storm
b. Reactionary hemorrhage
c. Secondary hemorrhage
d. Primary hemorrhage
e. Tracheomalacia
Answer= B. Reactionary Hemorrhage. It a post operative complications that occurs
within 1st 48hrs of surgery usually in the recovery room but usually hemorrhage followed
by thyroid surgery which can cause tracheal compression occurs within 1st 24hrs
immediately after surgery.
POSTOPERATIVE HEMORRHAGE
Primary Hemorrhage= it occurs at the time of injury or during the surgery.
Replace blood loss. If severe, return to theatre for adequate haemostasis. Treat shock
vigorously

Reactive haemorrhage: it occurs within 1st 24 to 48 hrs of the surgery usually in the
recovery room. It is due to the slippage of ligature or dislodgment of a clot
Replace blood and re-explore wound.
Secondary haemorrhage= (caused by infection) occurs 12 weeks (7 to 14 days) postop.

1473. A 40yo woman has had varicose vein surgery, planned as a day pt. After the op,
she is distressed by repeated retching and vomiting. Her pain is currently well controlled.
What is the best management strategy?
a. Tramadol
b. Co-codamol
c. IM morphine
d. IV ondansetron
e. PO ondansetron
Answer= D. IV Ondansetron
Since the pain is well settled so options A,B and C are irrelevant here as they are opiods
for pain relief. D & E are the two valid options. But in this case IV route is better because
she has repeated vomitting.
1474. A pt with renal failure has serum K+=7.5, raised creatinine and broad complex
tachycardia. What is the most appropriate management?
a. Calcium gluconate
b. Sodium bicarbonate
c. Dialysis
d. Furosemide
e. Sotalol
Answer= A. calcium gluconate. serum potassium is raised in this patient and it can be
life threatening so we need to manage that for that we give 10 ml of 10% of calcium
gluconate in order to stabilize cardiac membrane.
HYPERKALAEMIA MANAGEMENT
Untreated hyperkalaemia may cause life-threatening arrhythmias. Precipitating factors
should be addressed (e.g. acute renal failure) and aggravating drugs stopped (e.g. ACE
inhibitors). Management may be categorised by the aims of treatment
Stabilisation of the cardiac membrane
intravenous calcium gluconate (10 ml of 10% calcium gluconate)
Short-term shift in potassium from extracellular to intracellular fluid compartment
combined insulin/dextrose infusion ( usually 10 units of insulin (actrapid) are added to
50 ml of glucose 50% and infused over 30 minutes)
nebulised salbutamol
Removal of potassium from the body
calcium resonium (orally or enema)
loop diuretics
dialysis

1475. An 18yo lady in her 30th week of pregnancy is brought to the hosp in an altered
sensorium. She is taking slow, shallow breaths and her breath has a fruity smell. An
ABG: pH=7.20, urine ketones: +ve. What is the most probable dx?
a. HONK
b. DKA
c. HELLP syndrome
d. PIH
e. GDM
Answer= B. DKA. Slow shallow breaths and fruity smell in breath, ph< 7.30, ketonuria all
points towards the diagnosis of DKA ( diabetic keto acidosis).
Diabetic Ketoacidosis(DKA) is a medical emergency that is characterized by
Hyperglycemia, Acidaemia (ph< 7.3) and ketonaemia.
American Diabetes Association diagnostic criteria are as follows:
blood glucose >13.8 mmol/l
pH < 7.30
serum bicarbonate <18 mmol/l
anion gap > 10
ketonaemia
The most common precipitating factors of diabetic ketoacidosis (DKA) are infection, missed
insulin doses and myocardial infarction.
the most common symptoms include polydipsia, polyuria, nausea, vomiting,
dehydration,gradual drowsiness,coma or deep breathing (kussmaul hyperventilation.
Arterial blood gases - metabolic acidosis with low pH and low HCO3; pCO2 should be normal
but can be depressed by respiratory compensation; low pO2 may indicate primary respiratory
problem as a precipitant.

Management: fluid replacement: most patients with DKA are deplete around 5-8 litres.
Isotonic saline is used initially
Insulin: an intravenous infusion should be started at 0.1 unit/kg/hour. Once blood glucose is
< 15 mmol/l an infusion of 5% dextrose should be started
correction of hypokalaemia

1476. A 26yo man presented with abdominal distension and pain. His stools have been
mucoid and
sometimes blood stained. What is the most appropriate inv?
a. Stool C&S
b. Gastroscopy
c. IgG tissue transglutaminase
d. Barium meal
e. Jejunal biopsy
Answer= A. Stool C&S. this questions is an incomplete question. Mucoid and blood
stained stools with abdominal pain points towards the diagnosis of dysentery either by

shigella or campylobacter for which Stool C&S should be done. If we are suspecting
Inflammatory bowel disease then investigation should be barium Enema and no such
option is given.
Exclusion of other options:
Barium meal= it is done for upper GI
Jejunal Biopsy= It will not tell about the cause of the condition.
BACILLARY DYSENTERY
Shigella causes abdominal pain and bloody diarrhoea sudden fever, headache, and
occasionally neck stiffness. CSF is sterile.
Incubation: 17d.
Spread: Faecaloral.
Diagnosis: Stool culture.
Treatment: Fluids PO. Avoid antidiarrhoeal drugs. Drugs: ciprofloxacin 500mg/12h PO
for 35d.
shigellosis is often resistant to several antimicrobials: sensitivity testing is important
for all enteric fevers. There may be associated spondyloarthritis
CAMPYLOBACTER INFECTION
Incubation period= 2-5 days
There is a prodromal illness of fever, headache and myalgia lasting up to 24 hours. The fever
may be as high as 40C and, whether high or low, may persist for a week.
There are abdominal pains and cramps and profuse diarrhoea with up to 10 stools a day.
The stool is watery and often bloody.
Diagnosis: Stool culture.
Treatment: Fluids PO. Avoid antidiarrhoeal drugs. Drugs: ciprofloxacin 500mg/12h PO
for 35d.
1477. An 83yo elderly woman presented in the ED with cough, fever and sneezing. Tx
was given but she became confused and again presented with above said symptoms.
What is the cause of her condition?
a. Aspiration due to confusion
b. Alveolar damage due to drugs
c. Drug toxicity
d. Pneumothorax
Answer= A. Aspiration due to confusion.

1478. A 37yo man presents with some raised lesions on the shin. He came with cough
and also complains of arthralgia. Exam: bilateral hilar lymphadenopathy and erythema
nodosum is
present. What is the single most likely cause?
a. CD
b. UC
c. Sarcoidosis
d. Streptococcal infection
e. TB
Answer= C. Sarcoidosis. Bilateral hilar lymphadenopathy (BHL), erythema nodosum, fever

and polyarthralgia. these are the typical features of Lofgrens Syndrome which is acute form
of the disease sarcoidosis.
Sarcoidosis is a multisystem disorder of unknown aetiology characterised by non-caseating
granulomas. It is more common in young adults and in people of African descent
Features:
acute= erythema nodosum, bilateral hilar lymphadenopathy, swinging fever, polyarthralgia
insidious= dyspnoea, non-productive cough, malaise, weight loss
skin= lupus pernio
hypercalcaemia: macrophages inside the granulomas cause an increased conversion of
vitamin D to its active form (1,25-dihydroxycholecalciferol)
Syndromes associated with sarcoidosis
Lofgren's syndrome= is an acute form of the disease characterised by bilateral hilar
lymphadenopathy (BHL), erythema nodosum, fever and polyarthralgia. It usually carries an
excellent prognosis
In Mikulicz syndrome= there is enlargement of the parotid and lacrimal glands due to
sarcoidosis, tuberculosis or lymphoma
Heerfordt's syndrome (uveoparotid fever)= there is parotid enlargement, fever and uveitis
secondary to sarcoidosis

1479. A young lady with cervical ectropion bleeds on touch. What is the most
appropriate next inv?
a. Transvaginal US
b. Cervical smear
c. Punch biopsy
d. Serum estradiol
e. Colposcopy
Answer= B. Cervical Smear.
CERVICAL ECTROPION
On the ectocervix there is a transformation zone where the stratified squamous epithelium
meets the columnar epithelium of the cervical canal.
Elevated oestrogen levels (ovulatory phase, pregnancy, combined oral contraceptive pill use)
result in larger area of columnar epithelium being present on the ectocervix.
It is seen on examination as a red ring around the os and is so common as to be regarded as
normal.
This may result in the following features=
vaginal discharge, post-coital bleeding (usually asymptomatic)
Once a normal cervical smear has been confirmed, it is actively managed only if there are
symptoms
Treatment:
Ablative treatment (for example 'cold coagulation') is only used for troublesome symptoms.
otherwise no treatment is required.

1480. A 28yo man with recent onset of dyspepsia after eating spicy food and alcohol
consumption. H. pylori fecal antigen was negative. He returns after 1m with similar
symptoms despite being given omeprazole 40mg. What is the single best initial inv?
a. Hydrogen breath test

b. Gastroscopy
c. Barium meal
d. None
Answer= B. Gastroscopy. It is a stepwise approach . Patient has developed similar
symptoms after treatment with PPI when h.pylori fecal antigen test s negative so the
next step is to do Endoscopy (gastroscopy).
Alarm symptoms: Anaemia (iron deficiency); loss of weight; anorexia;
recent onset/progressive symptoms; melaena/haematemesis; swallowing difficulty.

>55yrs or ALARM signs


yes
NO
Do upper GI endoscopy
Stop drugs causing dyspepsia, eg NSAIDS
Lifestyle changes (p244)
Over-the-counter antacids, eg
magnesium trisilicate 10mL/8h PO
Review after 4wks
(no improvement)
If neg,PPIs or H2 blockers for 4wks
test for H.pylori
(eg omeprazole 20 mg/24h PO or

ranitidine 150mg/12 PO).


if positive treat to eradicate H.pylori

review after 4 wks


if improvement

( no improvement) no action required
improvement no improvement
urea breath test

(h.pylori eradicated?)
no further
Long-term,low dose
yes
action req
treatment
consider upper GI endoscopy
consider upper GI
endoscopy

1481. A 35yo woman who usually has 4 days mid-cycle bleeding, had her period 10d
ago. She has now presented with spots of blood. Her smear was normal 6m ago. Exam:
cervical ectropion which doesnt bleed on touch. What would you do?
a. Cervical smear
b. Endocervical swab
c. US guided biopsy
d. Laparotomy
e. Transvaginal US
f. Punch biopsy
g. Serum estradiol
h. Colposcopy
Answer= A Cervical Smear. examination shows cervical ectropion that doesn't bleed on
touch and most of the ectropion reverse to normal so we repeat cervical smear to see
that whether it persists or progresses or has regressed.
CERVICAL ECTROPION
On the ectocervix there is a transformation zone where the stratified squamous epithelium
meets the columnar epithelium of the cervical canal.
Elevated oestrogen levels (ovulatory phase, pregnancy, combined oral contraceptive pill use)
result in larger area of columnar epithelium being present on the ectocervix.

It is seen on examination as a red ring around the os and is so common as to be regarded as


normal.
This may result in the following features=
vaginal discharge, post-coital bleeding (usually asymptomatic)
Once a normal cervical smear has been confirmed, it is actively managed only if there are
symptoms
Treatment:
Ablative treatment (for example 'cold coagulation') is only used for troublesome symptoms.
otherwise no treatment is required.

1482. A 7yo boy presents with epistaxis of 2h duration. The bleeding has been
controlled. Inv: Plts=210, PT=13, APTT=42, bleeding time=normal. Which of the
following is the most likely dx?
a. Hemophilia
b. Von willebrand disease
c. ITP
d. Vit K deficiency
e. Liver disease
f. Anatomical defect
Answer= F.Anatomical defect. All parameters are normal so answer in this case would
be Anatomical defect.
von willebrand disease= prolonged bleeding time Haemphilia: prolonged APTT
1483. A pregnant woman returns from Sudan, now presenting with intermittent fever,
rigor and
seizures. What is the dx?
a. TB
b. Malaria
c. Meningitis
d. Lyme disease
Answer= B. Malaria. Travel history with intermittent fever, rigor and seizures all point
towards the diagnosis.
Falciparum malaria:
90% present within 1 month of the mosquito event, with prodromal headache, malaise,
myalgia
anorexia before the 1st fever paroxysm ( faints).
There may be no pattern to fever spikes (esp . initially);
Signs: Anaemia, jaundice, and hepatosplenomegaly. No rash or lymphadenopathy.
Complications:
Anaemia is common, Thrombocytopenia.
5 grim signs:
1 Consciousness/coma 2 Convulsions 3 Coexisting chronic illness 4 Acidosis
5 Renal failure (eg from acute tubular necrosis).
Pregnancy: Use chemoprophylaxis when pregnant in endemic areas.
Diagnosis: serial thin and thick films

Management:
Current guidelines suggest all patients with falciparum malaria should be admitted to hospital
initially - even semi-immune patients may worsen quickly.
uncomplicated falciparum malaria treatment:
Current UK guidelines suggest as possible alternative regimens for adults:
Oral quinine sulfate 600 mg/8 hours for 5-7 days plus doxycycline 200 mg daily
(or clindamycin 450 mg/8 hours for pregnant women) for 7 days.
Atovaquone-proguanil: 4 standard tablets daily for 3 days.
Artemether with lumefantrine: if weight >35 kg, 4 tablets stat and then a further 4
tablets at 8, 24, 36, 48 and 60 hours.
the WHO revised their treatment guidelines in 2010. These recommend that
artemisinin-based combination therapies should be used first-line in preference to
quinine.
complicated or severe Falciparum malaria treatment:
IV quinine dihydrochloride is the first-line antimalarial drug.
Oral quinine sulfate 600 mg tds should be substituted once the patient is well enough to
complete a 5- to 7-day course in total.
Artesunate regimen - for named adult patient use only, on expert advice
A second drug should always accompany these regimes.

1484. A pt is unresponsive and cyanosed. What is the most definitive 1st step in
management?
a. Chest compressions
b. Check airway
c. Call 999
d. Mouth to mouth
e. Recovery position
Answer= B. check airway

1485. A man was bitten by a drug addict and comes to the hosp with a wound. What inv
should be undertaken?
a. Hep C
b. Lyme disease
c. Hep B
d. Syphilis

e. Hep A
Answer= C.Hep B. Hepatitis B spreads via blood products, IV drug abusers, direct
contact.
Hepatitis B virus (HBV, a DNA virus.)
Spread: Blood products, IV drug abusers (IVDU),sexual, direct contact.
Risk groups: IV drug users and their sexual partners/carers; health workers;
haemophiliacs; job exposure to blood ; haemodialysis (and chronic renal failure);
close family members of a carrier or case; staff or residents of
institutions/prisons; babies of HBSAg +ve mothers
Incubation= 1- 6 months
Signs: Resemble hepatitis A but arthralgia and urticaria are commoner.
Tests: HBSAg (surface antigen) is present 16 months after exposure.
HBeAg (e antigen) is present for 13 months after acute illness and implies high
infectivity.
HBSAg persisting for >6 months defi nes carrier status and occurs in 510% of
infections;
Antibodies to HBCAg (anti-HBc) imply past infection;
Antibodies to HBSAg (anti-HBs) alone imply vaccination.
HBV PCR allows monitoring of response to therapy.
Vaccination:
Passive immunization (specific anti-HBV immunoglobulin) may be given
to non-immune contacts after high-risk exposure.
Treatment:
Avoid alcohol. Immunize sexual contacts.
Refer all with chronic liver inflammation (eg ALT 30iu/L) for antivirals, eg pegylated
(PEG) interferon alfa-2a, lamivudine, entecavir, adefovir.
The aim is to clear HBSAg and prevent cirrhosis and HCC (risk is if HBSAg and HBeAg
+ve).
Other complications: fulminant hepatic failure,
cholangiocarcinoma, cryoglobulinemia.
1486. An 18yo woman says that she cant walk around as she is very big for that room.
What is the most likely hallucination?
a. Extracampine visual hallucinations
b. Lilliputian visual hallucinations
c. Alice in wonderland syndrome
d. Hypnagogic hallucinations
Answer= B. lilliputian visual hallucinations.These are hallucinations seen in Alice in
wonderland syndrome. there is altered perception in size and shape of body parts or
objects an impaired sense of passing time.
Alice in Wonderland Syndrome (also known as Todd's syndrome, or lilliputian

hallucinations) is a disorienting neurological condition that affects human perception.


People experience micropsia, macropsia, pelopsia, teleopsia, or size distortion of other
sensory modalities. It is often associated with migraines, brain tumors, and the use of
psychoactive drugs. It can also be the initial symptom of the EpsteinBarr virus.
1487. A middle aged lady presented with fever, altered sensorium, bleeding gums and
jaundice. Labs:deranged renal function tests, normal PT/APTT, fragmented RBCs and
low plts. Whats the most
likely dx?
a. Cholesterol emboli
b. HUS
c. TTP
d. Hepatorenal syndrome
e. Sepsis
Answer= C. TTP. altered sensorium, jaundice, low plts all point towards the diagnosis.
exclusion of other causes:
Haemolytic uraemic syndrome (HUS) Though it has same features as TTP but it is
usually common in children. HUS is characterized by microangiopathic haemolytic
anaemia (MAHA): intravascular haemolysis + red cell fragmentation. Causes: 90% are
from E. coli strain O157
This typically affects young children in outbreaks (more common than sporadically)
after eating undercooked contaminated meat. Signs: Abdominal pain, bloody diarrhoea,
and AKI.
Tests: Haematuria/proteinuria. Blood film: fragmented RBC (schistocytes, platelets, Hb.
Clotting tests are normal.
treatment: Seek expert advice.
Dialysis for AKI may be needed. Plasma exchange is used in severe persistent disease.

TTP (THROMBOTIC THROMBOCYTOPENIC PURPURA)


There is an overlap between TTP and HUS, and many physicians consider them a
spectrum of disease.
All patients have MAHA (severe, often with jaundice) and low platelets.
Other features can include AKI, fluctuating CNS signs (eg seizures, hemiparesis,
consciousness, vision) and fever.
The classic description included the full pentad of features, but with the advent of
plasma exchange this is rarely seen.
Mortality is higher than childhood HUS and can be >90% if untreated, though reduced to
~20% with plasma exchange.
Pathophysiology: There is a genetic or acquired deficiency of
a protease (ADAMTS13) that normally cleaves multimers of von Willebrand factor
(VWF).
Causes: Idiopathic (40%), autoimmunity (eg SLE), cancer, pregnancy, drug associated
(eg quinine), bloody diarrhoea prodrome (as childhood HUS)
Tests: As HUS
treatment:
Urgent plasma exchange may be life-saving.
Steroids are the mainstay for non-responders.
.
1488. A child came to the ED with severe asthma and not responding to salbutamol
nebulizer and vomiting many times. What is the most appropriate management?
a. Salmeterol

b. Montelukast
c. Prednisolone
d. Budesonide inhaler
e. Oxygen
f. IV salbutamol
Answer= F. IV Salbutamol. it is a severe attack where optimum treatment is failed.

1489. A 73yo woman with skeletal and brain mets from breast ca has worsening low
back pain and blurring of vision. She has weakness of her legs, minimal knee and
absent ankle tendon reflexes, a palpable bladder, a power of 2/5 at the hip, 3/5 at the
knee and ankle, and tenderness over the 2nd lumbar vertebra. There is reduced
sensation in the perineum. She has been started on dexamethasone 16mg daily.What is
the single most likely cause of her weakness?
a. Paraneoplastic neuropathy
b. Progression of brain tumor
c. PID at L2/L3
d. Spinal cord compression
e. Steroid induced myopathy
Answer= D. spinal cord compression.
Spinal cord compression (as probably at L2/L3 region associated cauda equina
syndrome). Here blurring of vision can be explained by raised intracranial pressure
causing papilloedema (due to brain metastasis) which excludes blurring of vision from
"Paraneoplastic neuropathy". In progressive brain tumour you will get upper motor
neurone lesions which is not evident here. rather lower motor neurone type features and
bladder involvement and reduced perineal sensation justifies spinal cord compression
associated with cauda equina syndrome.
1490. A 78yo woman presents with unilateral headache and pain on chewing.
ESR=70mm/hr. She is on oral steroids. What is the appropriate additional therapy?
a. Bisphosphonates
b. HRT
c. ACEi
d. IFN
e. IV steroids
Answer= A. Bisphosphonates. unilateral headache and pain on chewing with raised ESR
all points towards the diagnosis of GIANT CELL ARTERITIS the management of which
is high dose steroids. so osteoporosis prophylaxis is given in these patients, thus

bisphosphonates are given.


Giant cell arteritis (GCA) = cranial or temporal arteritis
It is common in the elderlyconsider Takayasus if under 55yrs.
It is associated with PMR (polymyalgia Rheumatica) in 50%
Symptoms:
Headache, temporal artery and scalp tenderness (eg when combing hair), jaw
claudication, amaurosis fugax, or sudden blindness, typically in one eye.
Extracranial symptoms may include dyspnoea, morning stiffness, and unequal or weak
pulses.
If you suspect GCA, do ESR and start prednisolone 60mg/d PO immediately. The risk is
irreversible bilateral visual loss, which can occur suddenly if not treated.
Tests: ESR & CRP , platelets, alk phos, Hb.
Get a temporal artery biopsy within 7 days of starting steroids.
Prognosis: Typically a 2-year course, then complete remission.
Reduce prednisolone once symptoms have resolved and ESR; dose if symptoms
recur.
The main cause of death and morbidity in GCA is long-term steroid treatment so
consider
risks and benefits!
Give gastric and bone protection (PPI & bisphosphonate).

1491. A 48yo woman is admitted to the ED with a productive cough and mod fever. She
often has central chest pain and she regurgitates undigested food most of the time but
doesnt suffer
from acid reflux. These symptoms have been present for the last 3.5m which affects her
daily
food intake. CXR: air-fluid level behind a normal sized heart. What is the single most
likely dx?
a. Pharyngeal pouch
b. Hiatus hernia
c. Bulbar palsy
d. Achalasia
e. TB
Answer= D.Achalasia. Regurgitation of undigested foods, respiratory infection, absence
of acid reflux, air fluid level on chest x ray all point towards the diagnosis.
exclusion of other options:
Hiatal hernia=Hiatus hernia is a risk factor for GORD,in GORD patient will have
regurgitation of digested food particles and acid reflux.
ACHALASIA CARDIA
Failure of oesophageal peristalsis and of relaxation of lower oesophageal sphincter (LOS)
due to degenerative loss of ganglia from Auerbach's plexus i.e. LOS contracted, oesophagus

above dilated. Achalasia typically presents in middle-age and is equally common in men and
women.
Clinical features
dysphagia of BOTH liquids and solids
typically variation in severity of symptoms
heartburn
regurgitation of food - may lead to cough, aspiration pneumonia etc
malignant change in small number of patients
Investigations
manometry: excessive LOS tone which doesn't relax on swallowing - considered
most important diagnostic test
barium swallow shows grossly expanded oesophagus, fluid level, 'bird's beak'
appearance
CXR: wide mediastinum, fluid level
Treatment
intra-sphincteric injection of botulinum toxin
Heller cardiomyotomy
balloon dilation
drug therapy has a role but is limited by side-effects

1492. A retired ship worker has pleural effusion and pleural thickening on right side with
bilateral lung shadowing. What would you do to improve his symptoms?
a. Aspiration
b. Chest drain
c. Chemotherapy
d. Diuretic
Answer= C, Chemotherapy. ship worker developing pleural effusion and pleural
thickening
points towards the diagnosis of Mesothelioma for which Chemotherapy is definite
treatment.
Malignant mesothelioma is a tumour of mesothelial cells that usually occurs in
the pleura, and rarely in the peritoneum or other organs. It is associated with
occupational
exposure to asbestos,
Clinical features: Chest pain, dyspnoea, weight loss, finger clubbing, recurrent pleural
effusions.
Signs of metastases: lymphadenopathy, hepatomegaly, bone pain/tenderness,
abdominal pain/obstruction (peritoneal malignant mesothelioma).
Tests:
CXR/CT: pleural thickening/effusion. Bloody pleural fluid.
Treatment:

Patients are usually offered palliative chemotherapy (Pemetrexed + cisplatin) and there
is also a limited role for surgery and radiotherapy.

1493. An 88yo woman is a known smoker. She had an attack of MI 2y back and is
known to have peripheral vascular disease. She presents with an irreducible herniation
over the incision region of a surgery which she underwent in her childhood. What is the
most appropriate tx?
a. Truss
b. Elective herniorrhaphy
c. Urgent herniorrhaphy
d. Elective herniotomy
e. Reassure
Answer= B. Elective herniorrhaphy.
Incisional hernia is a risk of any abdominal surgery and is estimated to occur in 15% of
abdominal operations.

They are caused essentially by failure of the wound to heal but are probably the
result of multiple patient and technical factors.
Advances in technique and materials have not removed this problem.

Management
They require urgent repair with reinforcing mesh used in large hernias. This is
required particularly where the patient is obese.
Recurrence occurs in up to 50% of large hernias.

1494. A 72yo woman who is taking loop diuretics for left ventricular failure. She now is
suffering from
palpitations and muscle weakness. What is the electrolyte imbalance found?
a. Na+=130mmol/L, K+=2.5mmol/L
b. Na+=130mmol/L, K+=5.5mmol/L
c. Na+=140mmol/L, K+=4.5mmol/L
d. Na+=150mmol/L, K+=3.5mmol/L
e. None
Answer= A. Na+=130mmol/L, K+=2.5mmol/L
Loop Diuretics
loop diuretics act by inhibiting the Na-K-Cl cotransporter (NKCC) in the thick ascending limb
of the loop of Henle, reducing the absorption of NaCl.
There are two variants of NKCC; loop diuretics act on NKCC2, which is more prevalent in the
kidneys.
Indications= heart failure: both acute (usually intravenously) and chronic (usually orally),
resistant hypertension, particularly in patients with renal impairment.

Adverse effects= hypotension, hyponatremia, hypokalemia,hypocalcemia,hypochloremic


alkalosis, hyperglycemia (less common with thiazides), renal impairment( from
dehydration+direct toxic effect) and gout

1495. A young woman who is a marathon runner comes with secondary amenorrhea.
Inv: normal LH, FSH and estradiol, prolactin=600. What is the most likely dx?
a. Hypothalamic amenorrhea
b. Pregnancy
c. PCOS
d. Prolactinoma
e. Anorexia
Answer= A. Hypothalamic Amenorrhea. patient is a marathon runner and LH,FSH may
be normal if weight loss or excessive exercise are the cause. prolactins levels can also
be increased by stress.
Hypothalamic amenorrhea : excessive exercise can impair the pulsatile release of GnRH
from the hypothalamus. Although in most of such cases LH and FSH are low , a normal
level can be present depending on the timing of the test with regards to the menstrual
cycle.
In general , prolactin levels can be interpreted in the following way :
<500mU/L : normal.
500-1000 : drugs , stress ( including physical stress).
1000-5000 : drugs , microprolactinoma , disconnection hyperprolactinemia.
> 5000 : macroprolactinoma.
During pregnancy and lactation :750-8000 mU/L.
Hypothalamicpituitaryovarian causes are common (34% of cases) as control of the
menstrual cycle is easily upset, eg by stress (emotions, exams), exercise, weight loss.
Up to 44% of competitive athletes have amenorrhoea.
Tests HCG (eg urinary) to exclude pregnancy. FSH/LH (low if hypothalamic pituitary
cause but may be normal if weight loss or excessive exercise the cause. Raised eg
FSH>20 IU/L if premature menopause
Prolactin (by stress, hypothyroidism, prolactinomas and drugs, eg phenothiazines,
domperidone, metoclopramide).
Treatment is related to cause.
Hypothalamicpituitary axis malfunction:
If mild (eg stress, moderate weight loss) =there is sufficient activity to stimulate enough
ovarian oestrogen to produce an endometrium (which will be shed after a progesterone
challenge,
eg medroxyprogesterone acetate 10mg/24h for 10 days), but the timing is disordered so
cycles are not initiated.
If the disorder is more severe the axis shuts down (eg in severe weight loss)=
Reassurance and advice on diet or stress management, or psychiatric help if
appropriate.
she should be advised to use contraception as ovulation may occur at any time.
1496. A 4yo child comes with a sprain in his foot. Hx reveals that the child has had
recurrent
admissions to the hosp due to severe asthma. What is the most appropriate analgesic?

a. Diclofenac sodium
b. Ibuprofen.
c.Paracetamol
d. Codeine
Answer= C.Paracetamol. paracetamol should only be given in this patient to relieve pain.
NSAIDS should not be given as the child has h/o asthma as nsaids may increase the
risk of acute bronchospasm and codeine can cause respiratory depression.
1497. A 34yo pregnant woman, 38wk GA is in labor. She had a long 1st stage and
troublesome 2nd stage, has delivered a baby. After her placenta was delivered she had
a convulsion. What is the most probable management?
a. MgSO4 IV
b. Diazepam IV
c. IV fluid
d. Hydralazine IV
e. Anti-epileptic
Answer= MGSO4 IV. whenever a woman develops a fit few days after delivery, it is
always eclampsia until proven otherwise.
Treatment of hypertension:
If BP >160/110 mmHg or mean arterial pressure >125 mmHg, use labetalol
20mg IV increasing after 10min intervals to 40 mg then 80 mg until 200mg
total is given. Aim for BP 150/80100 mmHg.
Alternative is hydralazine.
Give prophylactic H2 blockers until normal postnatal care starts.
Restrict fluids to 80 mL/h. Hourly urine output. Renal failure is rare. Maintain
fluid restriction until postpartum diuresis. Fluid restriction is inappropriate
if there is haemorrhage.
Treatment of seizures (eclampsia):
Treat a first seizure with 4g magnesium sulfate in 100 mL 0.9% saline IVI
over 5min + maintenance IVI of 1g/h for 24h. Beware respiration.
If recurrent seizure give 2g IV magnesium sulfate over 5 min.
Check tendon reflexes and respiratory rate every 15min.
Stop magnesium sulfate IV if respiratory rate <14/min or tendon reflex
loss, or urine output <20 mL/h. Have IV calcium gluconate ready in case of
MgSO4 toxicity: 1g (10mL) over 10 min if respiratory depression.
Use diazepam once if fits continue (eg 510 mg slowly IV). If seizures continue,
ventilate and consider other causes (consider CT scan).

1498. A 23yo woman presents with offensive vaginal discharge. Vaginal pH=4.5. What is
the most likely organism?
a. Gardenella
b. Trichomonas

c. Candida
d. Mycoplasma
Answer= A. Gardnerella. offensive vaginal discharge with a vaginal ph of 4.5 points
towards the organism involved.
Bacterial vaginosis (BV) describes an overgrowth of predominantly anaerobic organisms
such as Gardnerella vaginalis. This leads to a consequent fall in lactic acid producing aerobic
lactobacilli resulting in a raised vaginal pH.
Whilst BV is not a sexually transmitted infection it is seen almost exclusively in sexually
active women.
Features
vaginal discharge: 'fishy', offensive
asymptomatic in 50%
Amsel's criteria for diagnosis of BV - 3 of the following 4 points should be present
thin, white homogenous discharge
clue cells on microscopy: stippled vaginal epithelial cells
vaginal pH > 4.5
positive whiff test (addition of potassium hydroxide results in fishy odour)
Management
oral metronidazole for 5-7 days
70-80% initial cure rate
relapse rate > 50% within 3 months
the BNF suggests topical metronidazole or topical clindamycin as alternatives

1499. A 62yo man has had anorectal pain aggravated by defecation for 3d. Rectal exam:
purple, tender lump at the anal verge. Flexible sigmoidoscopy: normal rectal mucosa
and hard feces. What is the best management strategy?
a. Anal hematoma
b. Anal fissure
c. Rectal ca
d. Diverticulitis
e. Angiodysplasia
Answer= A. Anal Hematoma. anorectal pain aggravated by defecation and tender lump
at the anal verge point towards the diagnosis.
Perianal haematoma (AKA thrombosed external pile)
Strictly, it is actually a clotted venous saccule.
It appears as a 24mm dark blueberry under the skin at the anal margin. It may be
evacuated under LA or left to resolve spontaneously.
Haemorrhoidal tissue is part of the normal anatomy which contributes to anal continence.
These mucosal vascular cushions are found in the left lateral, right posterior and right
anterior portions of the anal canal (3 o'clock, 7'o'clock and 11 o'clock respectively).
Haemorrhoids are said to exist when they become enlarged, congested and symptomatic
Clinical features

painless rectal bleeding is the most common symptom


pruritus
pain: usually not significant unless piles are thrombosed
soiling may occur with third or fourth degree piles

Types of haemorrhoids
External
originate below the dentate line
prone to thrombosis, may be painful
Internal

originate above the dentate line


do not generally cause pain

Grading of internal haemorrhoids


Grade I

Do not prolapse out of the anal canal

Grade II

Prolapse on defecation but reduce spontaneously

Grade III

Can be manually reduced

Grade IV

Cannot be reduced

Management
soften stools: increase dietary fibre and fluid intake
topical local anaesthetics and steroids may be used to help symptoms
outpatient treatments: rubber band ligation is superior to injection sclerotherapy
surgery is reserved for large symptomatic haemorrhoids which do not respond to
outpatient treatments
newer treatments: Doppler guided haemorrhoidal artery ligation, stapled
haemorrhoidopexy
Acutely thrombosed external haemorrhoids
typically present with significant pain
examination reveals a purplish, oedematous, tender subcutaneous perianal mass
if patient presents within 72 hours then referral should be considered for excision.
Otherwise patients can usually be managed with stool softeners, ice packs and
analgesia. Symptoms usually settle within 10 da

1500. A 43yo presents with severe vertigo on moving sidewards whilst sleeping. What
test would you do to confirm the dx?
a. Hallpikes manoeuvre
b. Rombergs test
c. Trendelenburg test
d. Heel-shin test
Answer= A. Hallpikes manoeuvre

. 1501. A 23yo man is having difficulty in speaking following a stab wound to the right of
his neck. On being asked to protrude his tongue, the tip deviated to the right. Which
anatomical site is most likely to be affected?
a. Facial nerve
b. Hypoglossal nerve
c. Vagus nerve
d. Trigeminal nerve
e. Glossopharyngeal nerve
Answer: B
The hypoglossal nerve is the twelfth cranial nerve XII, and innervates muscles of the
tongue.The nerve is involved in controlling tongue movements required for speech, food
manipulation (i.e. formation of bolus), and swallowing.Damage or lesions affecting
functions of the nerve may be supranuclear (as in cortical or rostrally to the hypoglossal
nucleus in the brainstem), nuclear or infranuclear (as in below or distal to the nucleus),
as well as unilateral or bilateral and will give symptoms which differ accordingly.[2]
Supranuclear injury of the brainstem affecting the hypoglossal nerve may give rise to
crossed symptoms due to a majority of the supranuclear innervation to the hypoglossal
nucleus being crossed. Symptoms often show deviation of the tongue towards the
paralyzed side when it is stuck out. This is because of the weaker genioglossal muscle.

1502. A girl presents with signs of hyperventilation. What is the most likely ABG
derangement?
a. pH increased, PCO2 increased
b. pH decreased, PCO2 increased
c. pH increased, PCO2 decreased
d. pH decreased, PCO2 decreased
Answer: I think C but given in key is A
Hyperventilation occurs when the rate and quantity of alveolar ventilation of carbon
dioxide exceeds the body's production of carbon dioxide. When alveolar ventilation is
excessive, more carbon dioxide will be removed from the blood stream than the body
can produce. This causes the concentration of carbon dioxide in the blood stream to fall
and produces a state known as hypocapnia. The body normally attempts to compensate
for this metabolically. If excess ventilation cannot be compensated metabolically, it will
lead to a rise in blood pH. This rise in blood pH is known as respiratory alkalosis.
Involuntary hyperventilation can occur in response to both physical and emotional
stimuli. These include reduced air pressure at high altitudes,[6] raised progesterone
levels in pregnancy, head injury, stroke, respiratory disorders such as asthma and
pneumonia, cardiovascular problems such as pulmonary embolisms, anemia, adverse
reactions to certain drugs, physical or emotional stress, fear, pain, and anxiety.
Hyperventilation can also be mechanically produced in people on respirators.[7]
1503. A pt presents with skin pigmentation, diarrhea, vomiting, abdominal pain and
postural
hypotension. What electrolyte abnormality is likely to occur?
a. Na+=130, K+=6.5
b. Na+=130, K+=2.5
c. Na+=13, K+=6.0
d. Na+=140, K+=8
e. Na+=130, K+=1.5
Answer: A
Addisons disease (also Addison disease, chronic adrenal insufficiency, hypocortisolism,
and hypoadrenalism) is a rare, chronic endocrine system disorder in which the adrenal
glands do not produce sufficient steroid hormones (glucocorticoids and
mineralocorticoids). Persistent nonspecific symptoms which should provoke
consideration of a diagnosis of adrenal insufficiency include:
Fatigue and weakness (common feature).
Anorexia.
Nausea.
Vomiting.
Weight loss.
Abdominal pain.
Diarrhoea.
Constipation.
Cravings for salt and salty foods such as soy sauce or liquorice (primary insufficiency).
Muscle cramps and joint pains.
Syncope or dizziness (due to hypotension).
Confusion.
Personality change.

Irritability.
Loss of pubic or axillary hair in women, delayed puberty in children.
Signs:
Hyperpigmentation - look at buccal mucosa, lips, palmar creases, new scars and in
areas subject to pressure such as elbows, knuckles and knees. (Not present in
secondary adrenal insufficiency.)
Hypotension.
Postural hypotension.
Laboratory abnormalities in adrenal insufficiency
Sodium - reduced in 90% of newly diagnosed cases of primary adrenal insufficiency.
Potassium - raised in 50% of newly diagnosed cases of primary adrenal insufficiency.
Calcium - raised in 10-20 % of newly diagnosed cases of primary adrenal insufficiency.
FBC - there may be anaemia, mild eosinophilia and lymphocytosis.
Glucose - often low in children.
LFTs - may be raised liver transaminases.
Cortisol - usually reduced:
Levels are highest between 8 am and 9 am when blood test should be taken.
Specialist advice should be sought in interpreting results for people on shift work (diurnal
variation may be altered), people taking oestrogen (can increase cortisol-binding globulin
production by the liver), pregnant women and people on long-term steroids.
Different assays are used so refer to local reference ranges. Generally levels of <100
nmol/L should prompt urgent investigation or admission, and levels of 100-150 nmol/L
require further investigation.
Salivary cortisol has been used for diagnosis, but not yet fully validated.
ACTH (also known as corticotropin) - when measured together with cortisol allows
differentiation of primary vs secondary insufficiency:
Levels are raised in primary insufficiency.
Levels are low or low normal in secondary insufficiency.
Plasma renin and aldosterone levels - will give an indication of mineralocorticoid activity.
(Renin is often high and aldosterone low in Addison's disease. Usually unaffected in
secondary insufficiency.)
Management:
Both glucocorticoid and mineralocorticoid replacement are required.
Glucocorticoid replacement - hydrocortisone is the mainstay of treatment; usually 15-30
mg in three divided doses with the highest dose in the morning (thus stimulating the
normal diurnal adrenal rhythm). Twice daily regimens are also used, although opinion on
their benefit varies. A modified-release once-daily preparation is also now available and
licensed for use, and is still being evaluated.
During minor illness or minor surgery, glucocorticoid doses may be increased up to three
times their normal dose to avoid adrenal crisis, and up to ten times for major illness or
major surgery.
If there is coexistent thyroid deficiency then thyroid hormones should not be replaced
before glucocorticoids, as a crisis may be precipitated.
Mineralocorticoid replacement - this is usually required in primary adrenal insufficiency.
Fludrocortisone is used and the usual adult dose is 50-300 micrograms per day,
depending on activity levels, weight and metabolism.
Assessing adequacy of therapy involves monitoring symptoms and signs, measuring
blood pressure and looking for postural hypotension and normalising of serum

electrolytes (Na and K).


Signs of over-replacement include raised blood pressure, thin skin, striae, easy bruising,
glucose intolerance, hyperglycaemia and electrolyte abnormalities.
Signs of under-replacement are the symptoms of Addison's disease persisting, ie
fatigue, postural hypotension, nausea, weight loss, and salt craving.

1504. A 10yo boy develops nasal bleeding. What is the best way to stop the bleeding
from the nose?
a. Pressure over base of the nose
b. Ice packs
c. Pressure over the soft tissues
d. Nasal packing
e. Surgery
Answer: C
The classification of nosebleeds is as anterior or posterior, depending upon the source
of bleeding. The blood supply to the nose is derived from branches of the internal
(anterior and posterior ethmoid arteries) and external carotid arteries (sphenopalatine
and branches of the internal maxillary arteries). Bleeding usually occurs when the
mucosa is eroded and vessels become exposed and subsequently break.
Epistaxis is usually benign, self-limiting and spontaneous. The majority are caused by
simple trauma. Although most incidents are not life-threatening, they can cause
significant parental concern when they occur in children.
Management of epistaxis:
Resuscitate the patient (if necessary) - remember the ABCD(E) of resuscitation.
Ask the patient to sit upright, leaning slightly forward, and to squeeze the bottom part of
the nose (NOT the bridge of the nose) for 10-20 minutes to try to stop the bleeding. The
patient should breathe through the mouth and spit out any blood/saliva into a bowl. An
ice pack on the bridge of the nose may help.
Monitor the patient's pulse and blood pressure.
If bleeding has stopped after this time (as it does in most cases) proceed to inspect the
nose, using a nasal speculum; consider cautery.
If the history is of severe and prolonged bleeding, get expert help - and watch carefully
for signs of hypovolemia.
1505. A pt came to the hosp with a complaint of severe chest pain lasting for >1h.
Following ECG test, pt revealed to have ST depression. He was already on aspirin.
What is the most specific tx for this pt?
a. GTN
b. Simvastatin
c. Clopidogrel
d. BB
e. LMWH
Answer: Key says B but i think A (should be E- most specific)
Non-ST-elevation ACS (NSTE-ACS): patients present with acute chest pain but without
persistent ST-segment elevation. The ECG shows persistent or transient ST-segment
depression or T-wave inversion, flat T waves, pseudo-normalisation of T waves, or no

ECG changes at presentation.


NSTE-ACS is further divided into:
Unstable angina: normal troponin levels.
Non-ST-elevation MI (NSTEMI): a rise in troponin levels.
The presentation of unstable angina and NSTEMI may be indistinguishable, and also
indistinguishable from acute STEMI.
NSTE-ACS can present in a variety of ways, including:[2]
Prolonged (longer than 20 minutes) anginal pain at rest.
New-onset angina with limitation of daily activities.
Recent destabilisation of previously stable angina, with moderate or severe limitation of
daily activities.
Post-myocardial infarction angina.
In unstable angina (and non-Q wave infarction), the ECG typically shows T-wave
inversion or ST-segment depression, but the ECG may be normal if some time has
elapsed since the last episode of pain.
Cardiac enzymes:
Within the first 6 hours, the sensitivity of troponins is superior to CK-MB for the detection
of myocardial infarction.
Troponin I and T become detectable in serum 3-6 hours after infarction, peak at 12-24
hours, and remain raised for up to 14 days.
Troponins are therefore usually tested 6 and 12 hours after the onset of pain.
In patients with unstable angina, minor troponin elevations may identify patients at risk
for subsequent cardiac events and death. Elevated troponin levels indicate an increased
risk of mortality in both the short term and long term. Patients with chest pain and
elevated troponin levels should remain in hospital for further assessment, including an
inpatient coronary angiogram.
The National Institute for Health and Care Excellence (NICE) recommends the Global
Registry of Acute Cardiac Events (GRACE) risk score.
Immediate management of a suspected ACS
Arrange urgent hospital admission (phone 999/112/911).
Resuscitation as required.
Pain relief: GTN and/or an intravenous opioid (use an antiemetic with opioids).
Single loading dose of 300 mg aspirin unless the person is allergic.
A resting 12-lead ECG - but don't delay transfer to hospital.
Assess oxygen saturation, using pulse oximetry before hospital admission if possible.
Give oxygen if oxygen saturation (SpO2) is less than 94% with no risk of hypercapnic
respiratory failure; aim for SpO2 of 94-98% (aim for 88-92% for people with chronic
obstructive pulmonary disease).
1506. A 69yo woman presents with a sudden onset of weakness of her right arm and
leg. She is known to be hypertensive. There has been no headache, LOC, visual,
speech or sensory symptoms. Exam: BP=180/90mmHg, pulse=100 and regular heart
sounds, no carotid bruit. Higher mental function tests are normal. No apraxia or neglect.
Speech, swallowing and sensation are normal. There are no visual field defects. There is
a mild facial weakness sparing the forehead. The right arm and leg are flaccid and weak.

Reflexes and tone are normal. There is a right extensor plantar response. What is the
most likely cause of this pts symptoms?
a. Cardioembolic stroke
b. Lacunar stroke
c. Right internal carotid artery atheroembolic stroke
d. Right internal carotid artery dissection
e. Right vertebral artery atheroembolic stroke
Answer: B
Lacunar infarcts (25%):
Small infarcts around the basal ganglia, internal capsule, thalamus and pons.
May cause pure motor, pure sensory, mixed motor and sensory signs, or ataxia.
Intact cognition/consciousness.
1507. A 34yo man has an intermittent epigastric pain for 3wks. It is worse by food but
helped by some tablets he obtained from the pharmacy. He had a similar episode 3yrs
ago and his doctor gave him a course of 3 types of tablets at the time. What is the most
appropriate next inv?
a. Abdomen US
b. Barium meal
c. Serum H.Pylori antibodies
d. C13 urea breath test
e. Upper GI endoscopy
Answer: D
Testing for H. pylori. Test using a carbon-13 urea breath test or a stool antigen test, or
laboratory-based serology where its performance has been locally validated. If re-testing
is required, a carbon-13 urea breath test is the chosen test. There is currently insufficient
evidence to recommend the stool antigen test as a test of eradication. Office-based
serological testing is not currently recommended because of its inadequate
performance.
1508. A girl with sickle cell anemia has painful bleeding and vaso-occlusive crisis during
her periods. What is the best possible management for this pt?
a. COCP
b. Tranexamic acid
c. Copper IUS
d. UAE
e. Depot provera
Answer: E
NICE recommends DMPA as a management option for heavy menstrual bleeding in
patients with sickle cell anemia. It also improves symptoms of dysmenorrhoea and
endometriosis.
No long-acting progestogen injection affects blood pressure.
Limited evidence suggests that the severity of the pain of sickle cell crises may be less
in women on DMPA. It is a safe option although there is a lack of evidence regarding the
risks of venous thrombosis in women with sickle cell disease.
1509. A 70yo pt comes with swelling in the parotid region for the last 10y. Exam: gland is
soft and cystic. Choose the most probable dx?

a. Pleomorphic adenoma
b. Carcinoma of the salivary glands
c. Mikuliczs disease
d. Adenoid cystic carcinoma
e. Parotid duct stones
Answer: D
Adenoid cystic carcinoma (ACC) is an uncommon and unusually indolent cancer arising
within glands and occurring mainly in the head and neck but also in the breast, trachea,
lacrimal glands, skin and vulva.It can present as a painless slow-growing mass in the
face or mouth.
1510. A 74yo man has been admitted unconscious with no hx. He has a GCS=6 and a
dilated left pupil which becomes insensitive to light. What is the single most likely dx?
a. Extradural hematoma
b. Meningitis
c. Opioid OD
d. Pontine hemorrhage
e. SAH
Answer: E
Ruptured posterior communicating artery aneurysm causes unilateral 3rd nerve palsy
and also SAH.(EDH also producing 3rd nerve palsy due to uncal herniation and raised
ICF and compressed brain but absent of relevant history regarding EDH such as head
trauma).
In elderly dura mater becomes more sticky and adherent to skull and it hardly allow any
blood to take place extradurally! So in elderly extradural hematoma is very unlikely.
Subarachnoid haemorrhage (SAH) is usually the result of bleeding from a berry
aneurysm in the Circle of Willis. These are called berry aneurysms because of their
shape. They were once thought to be mostly congenital but it is now thought that the
aetiology may involve susceptibility of the elastic lamina, in some patients, to stressors
such as hypertension and atherosclerosis.
There may be warning symptoms in the three weeks prior to SAH that represent small
leaks. These are called sentinel bleeds or expansion of the aneurysm. These are usually
headaches with the characteristics of SAH but which resolve by themselves without
further symptoms. They are estimated to occur in 10-15% of patients.
The most common symptoms are headache (48%), dizziness (10%), orbital pain (7%),
diplopia (4%) and visual loss (4%).
Signs may accompany these sentinel bleeds: sensory or motor disturbance (6%),
seizures (4%), ptosis (3%), bruits (3%) and dysphasia (2%).
If a sentinel bleed is suspected, patients should be admitted urgently for investigations
(treat as if an SAH has occurred).
Examination
Conscious level: on admission to hospital two thirds have a depressed level of
consciousness, of whom half are in coma. However, SAH patients can also walk into the
surgery, complaining of sudden onset of headache.
Neck stiffness may occur due to meningeal irritation by blood in the CSF, but it is not
invariable.
Ophthalmoscopy will show intraocular haemorrhages in around 15%, especially in those

with a depressed level of consciousness.


Isolated pupillary dilation with loss of light reflex may indicate brain herniation as a result
of rising intracranial pressure.
There may be focal neurological signs, suggestive of a stroke. Complete or partial palsy
of the oculomotor nerve is well recognised, especially with rupture of aneurysms of the
internal carotid artery at the origin of the posterior communicating artery.
Intraocular haemorrhage may occur in response to the raised pressure and is more
common in more severe SAH
Oculomotor nerve impairment may indicate bleeding from the posterior communicating
artery.
Hypertension is a risk factor for the condition but a marked rise in blood pressure may
also occur as a sympathetic reflex following intracerebral haemorrhage. This
sympathetic reflex can raise blood pressure to life-threatening levels, and surges of
adrenaline (epinephrine) may contribute to associated cardiac arrhythmias which may
both confuse the diagnosis and further threaten the patient.
In 3% of cases cardiac arrest follows SAH.
SAH in a person known to have seizures is suggestive of an arteriovenous malformation.
New-onset seizures are more indicative of ruptured berry aneurysm.
1511. A 27yo man presents to the ED with 2d hx of severe headache and pyrexia
(38.9C). CT: petechial hemorrhage in the temporal and inf frontal lobes. What is the
most likely dx?
a. Brain abscess
b. Meningococcal meningitis
c. Cerebral malaria
d. Herpes simplex encephalitis
e. New variant CID
Answer: D
Herpes simplex encephalitis (HSE) is recognised worldwide as the most frequent
infectious encephalitis.
In children older than 3 months and in adults: HSE is usually caused by herpes simplex
virus type 1 (HSV-1) and is localised to the temporal and frontal lobes.
In neonates: HSE is usually caused by herpes simplex virus type 2 (HSV-2) acquired at
the time of delivery, and brain involvement is generalised.
Other herpes viruses may cause encephalitis but much less frequently than HSV.
However, cytomegalovirus (CMV) encephalitis should be considered in those with
immunodeficiency.
1512. A 44yo woman with memory loss, poor concentration and inability to recognize
household
projects. She has right-handed involuntary writhing movement. There is strong fam hx of
similar
complain. What is the single most likely dx?
a. Pics dementia
b. Wilsons disease
c. Huntingtons disease
d. HIV associated dementia
e. Fronto-temporal dementia

Answer: C
Huntington's disease is associated with cell loss within the basal ganglia and cortex. It is
an autosomal-dominant, progressive neurodegenerative disorder with a distinct
phenotype, including chorea and dystonia, incoordination, cognitive decline, and
behavioural difficulties.[1] Huntington's disease was first described by George
Huntington in 1872. The disease is associated with increases in the length of a cysteineadenosine-guanine (CAG) triplet repeat present in a gene called 'huntingtin' located on
chromosome 4p16.3.Huntington's disease is the most common genetic cause of chorea.
Signs and Symptoms:
The mean age at onset of symptoms is 30-50 years.
In some cases symptoms start before the age of 20 years with behavioural disturbances
and learning difficulties at school (juvenile Huntington's disease)
Early signs may be personality change, self-neglect, apathy with clumsiness,
fidgeting with fleeting facial grimaces.
Huntington's disease then leads to progressive chorea, rigidity and dementia. It is
frequently associated with seizures.
Chorea is initially mild but may be severe and cause uncontrollable limb movements.
As the disease progresses, chorea is gradually replaced by dystonia and Parkinsonian
features.
Dysarthria, dysphagia and abnormal eye movements are common. There may also be
other movement disorders - eg, tics and myoclonus.
Huntington's disease is associated with increasing depression, bradykinesia, cognitive
impairment and aggression as the disease progresses.Behavioural difficulties include
apathy or lack of initiative, dysphoria, irritability, agitation or anxiety, poor self-care, poor
judgment and inflexibility.[1]Late features include spasticity, clonus, supranuclear gaze
palsy and extensor plantar responses. The rate of cognitive decline is very variable.
Investigations:
MRI and CT scans in moderate-to-severe Huntington's disease show a loss of striatal
volume and increased size of the frontal horns of the lateral ventricles, but scans are
usually unhelpful for diagnosis of early disorder.
If genetic testing is considered then extensive genetic counselling in a specialist unit is
required in view of the implications of an untreatable, familial, progressive,
neurodegenerative disease.
Treatment:
Current drug therapy has no effect on the progression of disability.
Hyperkinesias and psychiatric symptoms may respond well to pharmacotherapy, but
neuropsychological deficits and dementia remain untreatable.[8]
Patients, their families and carers require a great deal of physical and emotional support.
Chorea:
Benzodiazepines, valproic acid, dopamine-depleting agents (eg, tetrabenazine) and
neuroleptics may be useful.

1513. A 54yo man has collapsed suddenly following a headache. He has hypertension
and takes warfarin for prosthetic heart valve. GCS=4 and dilated left pupil. What is the
single most likely dx?
a. Ant circulation stroke
b. Post circulation stroke
c. Intracerebral hemorrhage
d. Intracerebellar hemorrhage

e. Pontine hemorrhage
Answer: C
Intracerebral hemorrhage occurs when a diseased blood vessel within the brain bursts,
allowing blood to leak inside the brain. (The name means within the cerebrum or brain).
The sudden increase in pressure within the brain can cause damage to the brain cells
surrounding the blood. If the amount of blood increases rapidly, the sudden buildup in
pressure can lead to unconsciousness or death. Intracerebral hemorrhage usually
occurs in selected parts of the brain, including the basal ganglia, cerebellum, brainstem,
or cortex.
What causes it?
The most common cause of intracerebral hemorrhage is high blood pressure
(hypertension). Since high blood pressure by itself often causes no symptoms, many
people with intracranial hemorrhage are not aware that they have high blood pressure,
or that it needs to be treated. Less common causes of intracerebral hemorrhage include
trauma, infections, tumors, blood clotting deficiencies, and abnormalities in blood
vessels (such as arteriovenous malformations). View an interactive tutorial on
arteriovenous malformations from the Toronto Brain Vascular Malformation Study
Group.
1514. A 5wk breast fed baby whose birth weight was 3.5kg and is now 4.5kg is thriving
well but is deeply jaundiced. What is the most likely dx?
a. Galactosemia
b. Breast milk jaundice
c. Thalassemia
d. Sickle cell disease
e. Congenital storage disorder
Answer: B (Breast milk jaundice: the baby is well and the jaundice usually resolves by
six weeks)
Jaundice is clinically detectable in the newborn when the serum bilirubin levels are
greater than 85 mol/L. This occurs in approximately 60% of term infants and 80% of
preterm infants. Hyperbilirubinemia is either unconjugated (which is potentially toxic but
may be physiological or pathological) or conjugated (not toxic but always pathological).
Without treatment, high levels of unconjugated bilirubin may lead to kernicterus.
Other causes of jaundice include:
Physiological jaundice:
This results from increased erythrocyte breakdown and immature liver function.
It presents at 2 or 3 days old, begins to disappear towards the end of the first week and
has resolved by day 10.
The bilirubin level does not usually rise above 200 mol/L and the baby remains well.
However, the bilirubin level may go much higher if the baby is premature or if there is
increased red cell breakdown - eg, extensive bruising, cephalohematoma.
Early neonatal jaundice (onset less than 24 hours):
Haemolytic disease: eg, haemolytic disease of the newborn (rhesus), ABO
incompatibility, glucose-6-phosphate dehydrogenase deficiency, spherocytosis.
Infection: congenital (eg, toxoplasmosis, rubella, cytomegalovirus (CMV), herpes
simplex, syphilis) or postnatal infection.
Increased haemolysis due to haematoma.

Maternal autoimmune haemolytic anaemia: eg, systemic lupus erythematosus.


Crigler-Najjar syndrome or Dubin-Johnson syndrome.
Gilbert's syndrome.
Prolonged jaundice (jaundice lasting for longer than 14 days in term infants and 21 days
in preterm infants):
Infection - eg, urinary tract infection.
Hypothyroidism, hypopituitarism.
Galactosaemia.
Breast milk jaundice: the baby is well and the jaundice usually resolves by six weeks
but occasionally continues for up to four months.
Gastrointestinal (GI): biliary atresia, choledochal cyst, neonatal hepatitis.
Conjugated hyperbilirubinemia:
Infection.
Parenteral nutrition.
Cystic fibrosis.
Metabolic: alpha-1-antitrypsin deficiency, galactosaemia, aminoacidurias,
organoacidaemias.
GI: biliary atresia, choledochal cyst, neonatal hepatitis.
Endocrine: hypothyroidism, hypopituitarism.
1515. A 71yo man with no prv immediate hx is brought to the ED by his wife who says
he has become progressively more forgetful, tends to lose his temper and is emotionally
labile. There is no hx of infectious disease or trauma. Whats the single most likely dx?
a. Picks dementia
b. Fronto-temporal dementia
c. Huntingtons disease
d. Alzheimers disease
e. Vascular dementia
Answer: D
Alzheimer's disease is the most common cause of dementia, and involves a progressive
degeneration of the cerebral cortex. There is widespread cortical atrophy. Neurons
affected develop surrounding amyloid plaques, neurofibrillary tangles, and produce less
acetylcholine. The cause is not yet known. Patients experience irreversible global,
progressive impairment of brain function, leading to reduced intellectual ability.
Diagnostic criteria
The National Institute for Health and Care Excellence (NICE) guidelines recommend the
National Institute of Neurological and Communicative Disorders and Stroke and the
Alzheimer's Disease and Related Disorders Association (NINCDS/ADRDA) diagnostic
criteria be used for the assessment of Alzheimer's disease. Alternatives are the ICD-10
or DSM-IV (now DSM-5 since the guidelines were written) criteria.[10][11]
The NINCDS/ADRDA criteria were proposed in 1984 and revised in 2011.[12] Core
features for diagnosis of Alzheimer's disease include:
Probable Alzheimer's disease
Dementia established by clinical examination and neuropsychological tests.
Deficits in two or more areas of cognition.
Insidious onset over months to years, and progressive worsening of memory and other
cognitive functions.
No disturbance of consciousness.
Onset between ages of 40 and 90. (Criterion removed in latest revision.)

Absence of systemic disorders or other brain diseases that could account for the
symptoms.
Possible Alzheimer's disease
Dementia with an atypical onset or course (ie sudden onset or insufficient documentation
of progressive decline); OR
Aetiologically mixed presentation (ie other criteria fit the diagnosis, but features of other
brain disorders or causes of dementia are present).
Mild cognitive impairment due to Alzheimer's disease
Newer diagnostic criteria have attempted to classify the symptomatic, pre-dementia
stage of Alzheimer's disease. The work group which revised the NINCDS/ADRDA
criteria in 2011 refers to this stage as "mild cognitive impairment" with clinical diagnostic
criteria as follows:[13]
Concern regarding a change in cognition (from patient, informer or clinician)
Impairment of one or more cognitive domains
Preservation of independence in functional abilities
Not having the features of dementia
1516. A 38yo woman with hemophilia who received several blood transfusions a few
years ago
presents with irritability and increasing memory deficit. She is unable to speak properly.
He is on
anti-TB tx. What is the single most likely dx?
a. Creutzfeldt Jacob disease
b. Drug toxicity
c. Vascular dementia
d. HIV associated dementia
e. Space occupying lesion
Answer: D
Repeated BT ---> patient got HIV inf ( immunocompromised)---> developed TB --> HIV
induced dementia
The use of plasma-derived factor VIII, before the availability of recombinant products, led
to infection with HIV, hepatitis B virus (HBV) and hepatitis C virus (HCV) in many
haemophiliacs. One case of likely transmission of variant Creutzfeldt-Jakob disease
(vCJD) by UK factor VIII concentrates has been reported in an elderly haemophilic
patient in the UK. The recent report of a blood test that may be used to detect vCJD has
raised the possibility of a new way to identify infected individuals, perhaps even before
the onset of clinical symptoms.
1517. An 18yo girl has menorrhagia and dysmenorrhea and requires contraception.
What drug will you
give her?
a. COCP
b. Mirena coil
c. Copper T
d. UAE
e. Depo provera
Answer: i think it should be B but in key it's A

Menorrhagia+dysmenorrhea------give COCP
Acc to nice guidelines the management steps of menorrhagia are:
When a first pharmaceutical treatment has proved ineffective then a second
pharmaceutical treatment should be considered rather than immediate referral to
surgery. If there is iron deficiency it should be corrected with oral iron.
First-line treatment
This is the levonorgestrel-releasing intrauterine system (IUS) - Mirena. This is longterm treatment and should be left in situ for at least 12 months.
One recent study has shown that women with menorrhagia reported more improvement
in bleeding and quality of life with the levonorgestrel-releasing IUS than with other
treatments available in primary care. In addition, they were more likely to continue with
this treatment.
However, the rate of discontinuation of Mirena treatment has been shown to be
relatively high - 16% at 12 months and 28% by 2 years.
Second-line treatment
This includes tranexamic acid, mefenamic acid or the combined oral contraceptive pill
(COCP):
Mefenamic acid works by inhibiting prostaglandin synthesis. It reduces menstrual loss by
around 25% in three quarters of women and is better tolerated than tranexamic acid.
Tranexamic acid is a plasminogen-activator inhibitor. It inhibits the dissolution of
thrombosis that leads to menstrual flow. It can reduce flow by up to 50%. It is most
effective at reducing menstrual loss associated with IUCDs, fibroids and bleeding
diathesis. Other non-steroidal anti-inflammatory drugs (NSAIDs) may also be used.
Side-effects include nausea, vomiting and diarrhoea. If there is disturbance in colour
vision then it should be discontinued.
The COCP suppresses production of gonadotrophins and reduces menstrual
blood loss by around 50%. It can improve dysmenorrhoea, lighten periods,
regulate the cycle, improve premenstrual symptoms, reduce the risk of PID and protect
the ovaries and endometrium against cancer.
Third-line treatment
This is with norethisterone.
The dose is 15 mg daily, from day 5 to 26 (or injected long-acting progestogens). This
can result in a significant reduction in menstrual blood loss, although women tend to find
the treatment less acceptable than intrauterine levonorgestrel. This regimen of
progestogen may have a role in the short-term treatment of menorrhagia.
However, there are very limited data regarding the use of progestogens and of
oestrogens and progestogens in combination in the treatment of irregular menstrual
bleeding associated with anovulation. There is still no consensus about which regimens
are the most effective.[9]
In secondary care 3-4 months of a gonadotrophin-releasing hormone (GnRH) analogue
may be offered before hysterectomy or myomectomy, where the uterus is enlarged or
distorted by fibroids. It is also a reasonable choice of therapy if other methods are
contra-indicated - but 'add-back' hormone therapy will be needed if continued for >6
months.
In the acute situation, a bleeding episode may be so disabling for the woman that
treatment with high-dose norethisterone (30 mg daily) needs to be used. This is

continued until bleeding is controlled, but is then tailed off.


Surgical options
The choice of treatment will depend on both the uterine size and the patient's desire to
retain her uterus.
1518. A pt of tuberculous abscess with the hx of prv abscess drainage presented with
fever and
tenderness between L2/L3 vertebra. Which is the best inv for this pt?
a. XR
b. CT
c. US
d. MRI
e. Blood culture
Answer: D
Investigations for pott's disease:
MRI scanning may demonstrate the extent of spinal compression and can show changes
at an early stage. Bone elements visible within the swelling, or abscesses, are strongly
suggestive of Pott's disease rather than malignancy.
CT scans and nuclear bone scans can also be used but MRI is best to assess risk to the
spinal cord.
1519. A 4yo child presents with repeated chest infections. He has yellow discoloration of
sclera and the mother gives a hx of diarrhea as well. What is the single inv most likely to
lead to a dx?
a. Sweat chloride test
b. Anti-endomysial antiboides
c. LFT
d. Jejunal biopsy
e. TFT
Answer: A
Features of cystic fibrosis:
High sodium sweat
Primary secretion of sweat duct is normal but CFTR does not absorb chloride ions,
which remain in the lumen and prevent sodium absorption.
Pancreatic insufficiency
Production of pancreatic enzymes is normal but defects in ion transport produce relative
dehydration of pancreatic secretions, causing their stagnation in the pancreatic ducts.
Biliary disease
Defective ion transfer across the bile duct causes reduced movement of water in the
lumen so that bile becomes concentrated, causing plugging and local damage.
Gastrointestinal disease
Low-volume secretions of increased viscosity, changes in fluid movement across both
the small and large intestine and dehydrated biliary and pancreatic secretions cause
intraluminal water deficiency.

Respiratory disease
Dehydration of the airway surfaces reduces mucociliary clearance and favours bacterial
colonisation, local bacterial defences are impaired by local salt concentrations and
bacterial adherence is increased by changes in cell surface glycoproteins.
Increased bacterial colonisation and reduced clearance produce inflammatory lung
damage due to an exuberant neutrophilic response involving mediators such as IL8 and
neutrophil elastase.
1520. An 82yo woman has been admitted from a nursing home with dense hemiplegia
and
homonymous hemianopia. She is dysphasic. What vessel is most likely to be involved?
a. Ant cerebral artery
b. Mid cerebral artery
c. Post cerebral artery
d. Internal carotid artery
e. Post inf cerebellar artery
Answer: B
Areas supplied by the middle cerebral artery include:
The bulk of the lateral surface of the hemisphere; except for the superior inch of the
frontal and parietal lobe (anterior cerebral artery), and the inferior part of the temporal
lobe.
Superior division supplies latero inferior frontal lobe (location of Broca's area i.e.
language expression)
Inferior division supplies lateral temporal lobe (location of Wernicke's area i.e. language
comprehension)
Deep branches supply the basal ganglia as well as the internal capsule.
Occlusion of the middle cerebral artery results in Middle cerebral artery syndrome,
potentially showing the following defects:
Paralysis (-plegia) or weakness (-paresis) of the contralateral face and arm
(faciobrachial)
Sensory loss of the contralateral face and arm.
Damage to the dominant hemisphere (usually the left hemisphere) results in aphasia i.e.
Broca's or Wernicke's
Damage to the non-dominant hemisphere (usually the right hemisphere) results in
contralateral neglect syndrome
Large MCA infarcts often have dviation conjugue, a gaze preference towards the side
of the lesion, especially during the acute period. Contralateral homonymous
hemianopsia is often present.
1521. A pt is dx with SIADH. Choose the appropriate biochemical change.
a. Plasma Na+ decrease and urine osmolarity increase
b. Plasma Na+ decrease and urine osmolarity decrease
c. Plasma Na+ increase and urine osmolarity decrease
d. Plasma Na+ increase and urine osmolarity increase
Answer: A
Syndrome of inappropriate ADH secretion (SIADH)[8][9]

Inappropriate ADH secretion from posterior pituitary or from ectopic source despite low
serum osmolality.
Major diagnostic features
Hyponatraemia.
Plasma hypo-osmolality proportional to hyponatraemia.
Inappropriately elevated urine osmolality (>100 mOsmol/kg) commonly > plasma
osmolarity.
Persistent urine [Na+] >40 mmol/L with normal salt intake.
Euvolemia.
Normal thyroid and adrenal function.
Extra features include an elevated ADH level and low blood uric acid level.
1522. A newborn that is electively intubated at birth and is due for surgery 48h after
birth. The
condition was suspected on antenatal US on CXR. What is the most likely dx?
a. CF
b. Congenital diaphragmatic hernia
c. Congenital cystic adenomatoid malformation
d. RDS
e. Alpha 1 antitrypsin deficiency
Answer: B (Many infants are now diagnosed in utero by ultrasound scan.
CXR or ultrasound scan will confirm the diagnosis in a neonate who has not previously
been diagnosed.)
Congenital diaphragmatic hernia is produced by the failure of the diaphragm to fuse
properly during fetal development, allowing the abdominal organs to migrate up into the
chest cavity. This results in the two primary problems underpinning congenital
diaphragmatic hernias: pulmonary hypertension and pulmonary hypoplasia. This is
compounded by dysfunction of the surfactant. Associated diseases, notably cardiac
abnormalities, are frequent.
Congenital diaphragmatic hernia occurs in 1 in 2,500 births. It accounts for 8% of all
major congenital defects. Males are more commonly affected than females with a ratio of
3:2.
Right-sided lesions are rare (10-15%) compared with left-sided (85%) as the liver plugs
the opening. Right congenital diaphragmatic hernia carries a disproportionately high
mortality and morbidity.[4][5]
Many cases are now diagnosed prenatally on routine ultrasound scans or scans
following the discovery of polyhydramnios in the mother.[7] This allows for detailed
planning of the delivery and immediate aftercare of the neonate. Previously undiagnosed
cases still occur and these will usually present at or very soon after birth, depending on
the severity of the hernia. Signs include:
Cyanosis soon after birth.
Tachypnoea.
Tachycardia.
Asymmetry of the chest wall.
Absent breath sounds on one side of the chest, usually the left with the heart shifted to
the right.
Bowel sounds audible over the chest wall.
The abdomen possibly feels 'less full' on palpation.

Management:
Children born without a prior diagnosis of congenital diaphragmatic hernia, present a
paediatric emergency and the initial management must be aimed at reducing the
pressure in the chest and increasing oxygenation. If bowel sounds are heard in the chest
of a neonate who has respiratory distress, the child should be resuscitated in a 'head
up', rather than the more usual 'head down', position.
Endotracheal intubation and mechanical ventilation are required for all infants with
severe disease who present in the first hours of life.
Avoid bag-and-mask ventilation in the delivery room because the stomach and intestines
become distended with air and further impair lung function.
Passage of an orogastric tube will facilitate location of the stomach on X-ray, as well as
permitting decompression of the stomach.
Use of surfactant at an early stage may be beneficial.
Blood gases should be monitored and an indwelling arterial catheter is advantageous.
An indwelling venous catheter will enable administration of drugs (eg, inotropic agents
and hypertonic solutions).
Surgery consists of replacing the abdominal organs within the abdominal cavity and
repairing the diaphragmatic defect. It used to be performed early, in the first 24 hours of
life. Some suggest that repair 24 hours after stabilisation is ideal but delays of up to 7 or
10 days are often well tolerated. Many surgeons now prefer to operate when
echocardiography has shown normal pulmonary arterial pressures maintained for at
least 24 to 48 hours. Therefore, delayed surgical repair is now usual, performed as an
elective procedure, and rarely as an out-of-hours procedure.

1523. A 63yo male undergoes abdominal surgery. On Monday morning, 3d post-op,


repeat samples
confirm serum K+=7.1mmol/l. His ECG shows broad QRS complexes. Which one of the
following
can be used as an effective tx for this pts hyperkalemia?
a. Calcium chloride IV
b. Calcium gluconate IV
c. Insulin subcutaneously
d. Furosemide IV
Answer: B
When arrhythmias occur, or when potassium levels exceed 6.5 mmol/l, emergency
lowering of potassium levels is needed. Several agents are used to transiently lower K+
levels. To treat myocardial excitability caused by hyperkalemia, Calcium (calcium
chloride or calcium gluconate) increases threshold potential through a mechanism that is
still unclear, thus restoring normal gradient between threshold potential and resting
membrane potential, which is elevated abnormally in hyperkalemia. Other agents used
to shift K in the cells are insulin or salbutamol. They control Hyperkalemia temporarily.
1524. A 25yo man attended in urological OPD has single testis. He was inv and other
testis was located in the abdomen. What is the best management plan for this pt?
a. Short trial of HCG
b. Orchidectomy

c. Orchidopexy
d. Reassurance
Answer: C
(should be B: Before 2 years of life, Orchidopexy may be done. But after that, tissue
usually atrophy, if it doesn't, there is a high probability of it developing into a tumour.
Some Urologists use 12-18 months to do Orchidopexy in Cryptorchidism, others have 912 months as reference age for pexy.)
An undescended testicle (testis) is more common in boys who are born prematurely.
Although in the majority of cases the testis descends by the age of 6 months, some boys
will need an operation. This is called an orchidopexy. This operation brings the testis
down from the tummy (abdomen) into the testes' sac (scrotum). There is an increased
risk of infertility and also cancer if the testis remains in the abdomen.
1525. A 56yo male who presented with epilepsy like symptoms has been dx with an
intracranial space occupying lesion. He now complains of thirst and mild dehydration.
His blood glucose is also increased. What is the single most appropriate immediate tx?
a. Insulin
b. IV fluids
c. Stop dexamethasone
d. Stop sodium valproate and change to another anti-epileptic
Answer:B
Diabetes insipidus is a condition in which your ability to control the balance of water
within your body is not working properly. Your kidneys are not able to retain water and
this causes you to pass large amounts of urine. Because of this, you become more
thirsty and want to drink more. There are two different types of diabetes insipidus: cranial
and nephrogenic.Treatment includes drinking plenty of fluids so that you do not become
dehydrated. Treatment with medicines may be also needed for both types of diabetes
insipidus.
1526. A mother brings her newborn to the hosp concerned about a blue patch on the
buttocks. The newborn is of mixed race and was delivered normally. What is the most
appropriate
management?
a. Reassurance
b. CBC
c. XR
d. Plt count
Answer: A
A Mongolian spot, also known as Mongolian blue spot, congenital dermal
melanocytosis,[1] and dermal melanocytosis is a benign, flat, congenital birthmark with
wavy borders and irregular shape. It normally disappears three to five years after birth
and almost always by puberty.[6] The most common color is blue, although they can be
blue-gray, blue-black or even deep brown.
1527. The ECG of a 65yo shows absent P waves, narrow QRS complex, ventricular rate
of 120bpm and irregular R-R interval. What is the most probable dx?
a. A-fib
b. A-flutter
c. SVT
d. Mobitz type 1 2nd degree heart block

e. Sinus tachycardia
Answer: A
Atrial fibrillation (AF) is the most common sustained cardiac arrhythmia, characterised by
irregularly irregular ventricular pulse and loss of association between the cardiac apex
beat and radial pulsation. Loss of active ventricular filling is associated with:
Stagnation of blood in the atria leading to thrombus formation and a risk of embolism,
increasing the risk of stroke.
Reduction in cardiac output (especially during exercise) which may lead to heart failure.
1528. The ECG of an 80yo pt of IHD shows sawtooth like waves, QRS complex of 80ms,
ventricular rate of 150bpm and regular R-R interval. What is the most probable dx?
a. A-fib
b. A-flutter
c. SVT
d. Mobitz type 1 2nd degree heart block
e. Sinus tachycardia
Answer: B
The common form of type I atrial flutter has saw-tooth flutter waves, best seen in leads
II, III, and aVF, with atrial rates of 240-340. A 12-lead ECG is gold standard for
diagnosis.[4]
ATRIAL FLUTTER
In Atrial flutter, Pulse may be irregular or regular, but is usually rapid. Arteriovenous
conduction is usually 2:1, making the ventricular rate approximately 150 bpm. 1:1
atrioventricular (AV) conduction may lead to haemodynamic collapse. Carotid massage
may decrease the ventricular rate
1529. A man brings his wife into the ED after finding her unconscious at home. He says
at breakfast
time she had complained of sudden severe headache. What is the most appropriate inv?
a. MRI
b. XR
c. CT brain
d. Carotid Doppler
Answer: C
The most characteristic presentation of subarachnoid hemorrhage is a sudden
explosive headache. This may last a few seconds or even a fraction of a second.
If SAH is suspected, CT scanning (without contrast) is the first line in investigation
because of the characteristically hyperdense appearance of blood in the basal cisterns.
Every patient in whom SAH is suspected should have a CT scan at the earliest
opportunity. This should be done immediately if the patient presents with sudden severe
headache and as soon as possible in all other cases.
Treatment of SAH:
Rebleeding is the most imminent danger; a first aim is therefore occlusion of the
aneurysm. Endovascular obliteration by means of platinum spirals (coiling) is now the
preferred mode of treatment, but some patients require a direct neurosurgical approach
(clipping).

1530. A 68yo lady with T2DM. Which drug should be prescribed?


a. Biguanides
b. Sulphonyl urea
c. Insulin
d. Lifestyle modifications
Answer: A
Pharmacologic Therapy
Early initiation of pharmacologic therapy is associated with improved glycemic control
and reduced long-term complications in type 2 diabetes. Drug classes used for the
treatment of type 2 diabetes include the following:
Biguanides
Sulfonylureas
Meglitinide derivatives
Alpha-glucosidase inhibitors
Thiazolidinediones (TZDs)
Glucagonlike peptide1 (GLP-1) agonists
Dipeptidyl peptidase IV (DPP-4) inhibitors
Selective sodium-glucose transporter-2 (SGLT-2) inhibitors
Insulins
Amylinomimetics
Bile acid sequestrants
Dopamine agonists
Metformin is the only biguanide in clinical use.Metformin lowers basal and postprandial
plasma glucose levels. Its mechanisms of action differ from those of other classes of oral
antidiabetic agents; metformin works by decreasing hepatic gluconeogenesis production.
It also decreases intestinal absorption of glucose and improves insulin sensitivity by
increasing peripheral glucose uptake and utilization. Unlike oral sulfonylureas, metformin
rarely causes hypoglycemia.
1531. In a laparoscopic mesh repair for hernia, when the trocar is inserted at midpoint
between
umbilicus and ischial spine. What structure will be pierced?
a. Linea alba
b. Rectus muscle
c. Conjoint tendon
d. External and internal oblique muscles
e. Inguinal ligament
Answer: D
Structures in Anterior Abdominal Wall
In human anatomy, the layers of the abdominal wall are (from superficial to deep):
Skin
Subcutaneous tissue
Fascia
Camper's fascia - fatty superficial layer.
Scarpa's fascia - deep fibrous layer.
Muscle
External oblique muscle
Internal oblique muscle

Rectus abdominis
Transverse abdominal muscle
Pyramidalis muscle
Fascia transversalis
Peritoneum
1532. A 48yo man has intermittent left sided lower abdominal pain and feels generally
unwell. He has lost his appetite and has lost weight. Temp=38.3C and he has
BP=190/100mmHg. What is the single inv most likely to lead to dx?
a. Colonoscopy
b. Endomysial antibodies
c. Fasting serum glucose conc
d. TFT
e. US abdomen
Answer: E
Investigations recommended for left iliac fossa pain on patient.co. These should be
tailored to the patient's symptoms and the examination findings. In the GP setting there
are a number of bedside tests that can be done to aid diagnosis:
Dip urine for pus cells, leukocytes and/or nitrites if urinary tract infection (UTI) is
suspected. Microscopic haematuria is usually present in ureteric colic. It can also occur
in abdominal aortic aneurysm.
Perform a pregnancy test if ectopic pregnancy or miscarriage is suspected.
If the pain is non-acute and can be managed in the GP setting, further investigations
may be requested:
Blood tests may include FBC, renal function, LFTs.
Vaginal swab tests can help to exclude pelvic infection.
Ultrasound scanning can show ovarian or other mass.
Referral for further bowel investigations may be necessary - eg, referral under the twoweek wait rule if bowel carcinoma is suspected.
1533. A man with DM comes to the ED after he collapsed at home. His GCS=10. What
should be the
next initial inv for this man?
a. Capillary blood sugar
b. MRI head
c. CT head
d. Serum electrolytes
Answer: A
Diabetic coma is a reversible form of coma found in people with diabetes mellitus. It is a
medical emergency.
Three different types of diabetic coma are identified:
1. Severe low blood sugar in a diabetic person
2. Diabetic ketoacidosis advanced enough to result in unconsciousness from a
combination of a severely increased blood sugar level, dehydration and shock, and
exhaustion
3. Hyperosmolar nonketotic coma in which an extremely high blood sugar level and

dehydration alone are sufficient to cause unconsciousness.


A quick look, and a glucose meter to determine the cause of unconsciousness in a
patient with diabetes. Laboratory confirmation can usually be obtained in half an hour or
less. Other conditions that can cause unconsciousness in a person with diabetes are
stroke, uremic encephalopathy, alcohol, drug overdose, head injury, or seizure.
1534. A 60yo DM pt presented with easy fatigability, weakness and numbness of hands
and swollen feet. Exam: pedal edema, sensory neuropathy and palpable liver and
spleen. Urine: proteinuria. US abdomen: enlarged kidney. Renal biopsy: amorphous
homogenous substance that stained red with congo-red. What is the dx?
a. DM retinopathy
b. Sarcoidosis
c. Wilms tumor
d. Amyloidosis
e. Glycogen storage disease
Answer: D (Aggregation of the congo-red dye and binding to amyloid fibrils tends to redshift the absorption spectrum, whereas binding to cellulose fibers has the opposite
effect.)
Apple-green birefringence of Congo red stained preparates under polarized light is
indicative for the presence of amyloid fibrils.
Amyloidosis is a rare disease that results from accumulation of inappropriately folded
proteins. These misfolded proteins are called amyloids. When proteins that are normally
soluble in water fold to become amyloids, they become insoluble and deposit in organs
or tissues, disrupting normal function. The type of protein that is misfolded and the organ
or tissue in which the misfolded proteins are deposited determines the clinical
manifestations of amyloidosis.
Amyloid deposition in the kidneys can cause nephrotic syndrome, which results
from a reduction in the kidney's ability to filter and hold on to proteins. In AA amyloidosis
the kidneys are involved in 91-96% of people,symptoms ranging from protein in the urine
to nephrotic syndrome and rarely renal insufficiency.
1535. A 75yo man has urinary symptoms of hesitancy, frequency and nocturia. Rectal
exam: large hard prostate. What is the most appropriate inv?
a. CA 125
b. CA 153
c. CA 199
d. CEA
e. PSA
Answer: E
Prostate-specific antigen (PSA)
Cancer type: Prostate cancer
Tissue analyzed: Blood
PSA is produced exclusively by epithelial prostatic cells, both benign and malignant. It is
also found in the serum. Serum PSA is currently the best method of detecting localised
prostatic cancer and monitoring response to treatment but it lacks specificity, as it is also
increased in most patients with benign prostatic hyperplasia.

1536. A child suffering from CF developed pneumonia. Which organism is responsible


for this
pneumonia?
a. H. influenza
b. Klebsiella
c. S. aureus
d. S. pneumonia
E. Pseudomonas
Answer: E
Chronic infections in cystic fibrosis,. Thick mucus in the lungs and sinuses provides an
ideal breeding ground for bacteria and fungi. People with cystic fibrosis may have
frequent bouts of bronchitis or pneumonia
Pseudomonas aeruginosa: Typical pneumonia aspiration or inhalation
green
sputum, abscess formation, Common cause of pneumonia in cystic fibrosis patients and
those with severely compromised respiratory defenses.
1537. An obese woman with hx of migraine presented with heavy bleeding during
menstruation which is painful and needs contraception too. What is the best possible
management for this pt?
a. COCP
b. Mirena coil
c. Copper T
d. UAE
e. Depo provera
Answer: B
Management of menorrhagia:
First-line treatment
This is the levonorgestrel-releasing intrauterine system (IUS) - Mirena. This is longterm treatment and should be left in situ for at least 12 months.[2]
Second-line treatment
This includes tranexamic acid, mefenamic acid or the combined oral contraceptive pill
(COCP):
Mefenamic acid works by inhibiting prostaglandin synthesis. It reduces menstrual loss by
around 25% in three quarters of women and is better tolerated than tranexamic acid.
Tranexamic acid is a plasminogen-activator inhibitor. It inhibits the dissolution of
thrombosis that leads to menstrual flow. It can reduce flow by up to 50%.[8] It is most
effective at reducing menstrual loss associated with IUCDs, fibroids and bleeding
diathesis. Other non-steroidal anti-inflammatory drugs (NSAIDs) may also be used.
Side-effects include nausea, vomiting and diarrhoea. If there is disturbance in colour
vision then it should be discontinued.
The COCP suppresses production of gonadotrophins and reduces menstrual blood loss
by around 50%. It can improve dysmenorrhoea, lighten periods, regulate the cycle,
improve premenstrual symptoms, reduce the risk of PID and protect the ovaries and
endometrium against cancer.
Third-line treatment
This is with norethisterone.

The dose is 15 mg daily, from day 5 to 26 (or injected long-acting progestogens). This
can result in a significant reduction in menstrual blood loss, although women tend to find
the treatment less acceptable than intrauterine levonorgestrel. This regimen of
progestogen may have a role in the short-term treatment of menorrhagia.
However, there are very limited data regarding the use of progestogens and of
oestrogens and progestogens in combination in the treatment of irregular menstrual
bleeding associated with anovulation. There is still no consensus about which regimens
are the most effective.
1538. A 2yo fell on outstretched hand on playground. He presents with pain on base of
the thumb. XR=no fx. What is the single most likely dx?
a. Colles fx
b. Head of radius
c. Mellet finger
d. Scaphoid fx
e. No fx
Answer: D
The scaphoid bone is one of the carpal bones in your hand around the area of your
wrist. It is the most common carpal bone to break (fracture). A scaphoid fracture is
usually caused by a fall on to an outstretched hand. Symptoms can include pain and
swelling around the wrist. Diagnosis of a scaphoid fracture can sometimes be difficult, as
not all show up on X-rays. Treatment is usually with a cast worn on your arm up to your
elbow for 8 to 12 weeks. Sometimes surgery is advised. Correct diagnosis and prompt
treatment of a scaphoid fracture can help to reduce complications.
1539. A pt was admitted with increased frequency of passing urine, increased thirst,
weakness and muscle cramps. What is the most probable dx?
a. Conns syndrome
b. Cushings syndrome
c. Pheochromocytoma
d. Hyperthyroidism
e. Hypoparathyroidism
Answer: A
Classic presentation of hyperaldosteronism include:
Hypertension.
Hypokalaemia (usually <3.5 mmol/L, although 70% of patients may be normokalaemic).
Metabolic alkalosis.
Sodium may be normal or at the high end of normal.
Patients may also have polyuria and subsequent polydipsia due to reduced ability of the
kidneys to concentrate urine.
Weakness may be present from hypokalaemia.
Headaches and lethargy may also be present.
Differential diagnosis of hyperaldosteronism with low renin
Primary hyperaldosteronism (adenoma, BAH, GRA, carcinoma).
Congenital adrenal hyperplasia.
Differential diagnosis of hyperaldosteronism with high renin

Renal artery stenosis (RAS).


Coarctation of the aorta.
Fibromuscular dysplasia.
Renin-secreting tumours.
Congestive cardiac failure.
Nephrotic syndrome.
Gitelman's syndrome.
Bartter's syndrome.
Diuretic use.
1540. A 69yo male presented with sudden onset of dysphagia. He is neither able to
swallow liquid nor
solid, he recently had a denture fitting. What is the most probable dx?
a. Foreign body
b. Plummer vinson syndrome
c. Achalasia cardia
d. Esophageal rupture
e. Esophageal ca
Answer: A
Causes of sudden onset of dysphagia could be:
Apthous ulcers
Carcinoma tongue
Oral cavity obstruction (fb/tumor)
Quinsy
Tonsillitis
Adenoiditis
Scleroderma
Dermatomyositis
Diphtheria
Esophageal spasm
Gastric carcinoma
Tetanus
Rabies
1541. A 62yo man with chronic schizophrenia presents with a mask like face and
involuntary pill rolling movement in both hands. He complains of chronic cough and
forgetfulness. He is on long term antipsychotic meds. What is the single most likely dx?
a. Shy drager syndrome
b. Parkinsonism
c. Huntingtons chorea
d. Tardive dyskinesia
e. Akathisia
Answer: B
Parkinsonism is a clinical syndrome characterized by tremor, bradykinesia, rigidity, and
postural instability which May later be associated with dementia or depression.
About 7% of people with parkinsonism have developed their symptoms following
treatment with particular medications. Side effect of medications, mainly neuroleptic
antipsychotics.

1542. A 34yo female presented with vomiting preceded by an occipital headache of


acute onset. Exam: conscious and alert with photophobia but no neck stiffness. CT:
normal. What is the most
appropriate further management?
a. CT brain with contrast
b. Repeat CT brain in 24h
c. CSF exam
d. Cerebral angio
e. MRI brain
Answer: C (to exclude increased intracranial pressure)
This test is done to measure pressures within the cerebrospinal fluid and to collect a
sample of the fluid for further testing. CSF analysis can be used to diagnose certain
neurologic disorders, particularly infections (such as meningitis) and brain or spinal cord
damage.
1543. A lady with post ileocolectomy closure of stoma has a small 4 cm swelling around
the stoma. What is the most appropriate management of the swelling?
a. Local exploration of swelling
b. Exploratory laparotomy
c. Open laparotomy and re-closure
d. Abdominal binder
e. Truss
f. Laparotomy with mesh repair
Answer: D ( Most hernias can be managed without surgery. Support belts and
appliances are the most successful aids. These are available on prescription in the UK)
Parastomal hernia
A parastomal hernia involves an ostomy in the area where the stoma exits the
abdominal cavity. The intestine or bowel extends beyond the abdominal cavity or
abdominal muscles; the area around the stoma appears as a swelling or protuberance.
Parastomal hernias are incisional hernias in the area of the abdominal musculature that
was incised to bring the intestine through the abdominal wall to form the stoma. They
may completely surround the stoma (called circumferential hernias) or may invade only
part of the stoma.
Parastomal hernias can occur any time after the surgical procedure but usually happen
within the first 2 years. Recurrences are common if the hernia needs to be repaired
surgically. Risk factors may be patient related or technical. Patient-related risk factors
include obesity, poor nutritional status at the time of surgery, presurgical steroid therapy,
wound sepsis, and chronic cough. Risk factors related to technical issues include size of
the surgical opening and whether surgery was done on an emergency or elective basis.
Parastomal hernias occur in four types. Initially, a parastomal hernia begins as an
unsightly distention in the area surrounding the stoma; the hernia enlarges, causing
pain, discomfort, and pouching problems resulting in peristomal skin complications that
require frequent assessment. Conservative therapy is the usual initial treatment.
Adjustments to the pouching system typically are required so changes in the shape of
the pouching surface can be accommodated. Also, a hernia support binder or pouch
support belt may be helpful. Avoid convex pouching systems; if this isnt possible, use
these systems with extreme caution. If the patient irrigates the colostomy, an ostomy
management specialist should advise the patient to discontinue irrigation until the

parastomal hernia resolves.


Surgery may be considered in extreme cases eg strangulation or obstruction or when
the hernia affects the seal between the skin and appliance causing leakage. Other
reasons may be if the hernia is causing a lot of pain or embarrassment to the ostomate,
1544. A 64 yo woman has been brought by her son for psychiatric evaluation. She says
that she has stopped living with her husband because she is convinced it is someone
else posing to be him. What kind of delusions is she suffering from?
a. Delusion of reference
b. Delusion of control
c. Delusion of guilt
d. Delusion of persecution
e. Delusion of doubles
Answer: B (most close option to capgras)
Capgras' delusion - belief that a close relative has been replaced by someone else who
looks the same.
A delusion is a belief held with strong conviction despite superior evidence to the
contrary.
Delusion of reference: The person falsely believes that insignificant remarks, events, or
objects in one's environment have personal meaning or significance.[
Delusion of control: This is a false belief that another person, group of people, or
external force controls one's general thoughts, feelings, impulses, or behavior.
Delusion of guilt or sin (or delusion of self-accusation): This is an ungrounded feeling of
remorse or guilt of delusional intensity
Delusion of persecution: Persecutory delusions are the most common type of delusions
and involve the theme of being followed, harassed, cheated, poisoned or drugged,
conspired against, spied on, attacked, or otherwise obstructed in the pursuit of goals.
Delusion of doubles: The Fregoli delusion, or the delusion of doubles, is a rare disorder
in which a person holds a delusional belief that different people are in fact a single
person who changes appearance or is in disguise.
1545. A 19yo man with known hx of OM presents with headache, lethargy, sweating and
shivering. What is the single most likely dx?
a. Furuncle
b. Meningitis
c. Myringitis
d. Nasopharyngeal tumor
e. OM

Answer: B
A very rare and serious complication of OM is meningitis. This can occur if the infection
spreads to the protective outer layer of the brain and spinal chord (the meninges).
Symptoms of meningitis can include:
severe headache
being sick
a high temperature (fever)
stiff neck
sensitivity to light
rapid breathing
AOM is seen frequently in children but is less common in adults. Smoking is a risk factor.
Om occurs more commonly in winters.
Complications of AOM:
Infra-temporal infections can include:
Tympanic membrane perforation.
Mastoiditis.
Facial nerve palsy.
Acute labyrinthitis.
Petrositis.
Acute necrotic otitis.
Chronic otitis media.
Intracranial infections can include:
Meningitis.
Encephalitis.
Brain abscess.
Otitic hydrocephalus (hydrocephalus associated with AOM, usually accompanied by
lateral sinus thrombosis but the exact pathophysiology is unclear).
Subarachnoid abscess.
Subdural abscess.
Sigmoid sinus thrombosis.
Rarely, systemic complications can occur, including bacteraemia, septic arthritis and
bacterial endocarditis.
1546. A 46yo woman has gained weight. She has sensitivity to cold. Her pulse = regular
at 50 bpm and heart=enlarged. What is the single most likely underlying mechanism for
this condition
a. Autoimmune
b. Degenerative
c. Congenital
d. Infective
e. Nutritional
a. Autoimmune

1547. A 70yo man presents with a punched out ulcer between his toes. He is a heavy
drinker and smoker. Exam: ulcer is yellow and the foot turns red when dangling off the
bed. What is the
single most likely dx?
a. Arterial ischemia ulcer
b. Malignancy
c. Neuropathic ulcer
d. Pressure ulcer
e. Venous stasis ulcer
Answer: A (These ulcers occur most commonly in areas of poor blood supply - eg,
the tip of the toes or over the tibia)
Arterial leg ulcers:
These are often more distal and on the dorsum of the foot or toes.Initially they have
irregular edges, but this may become more clearly defined. The ulcer base contains
greyish, granulation tissue. Handling, such as debriding these ulcers, produces little or
no blood.Nocturnal pain is typical. It is worse when supine and is relieved by dangling
the legs out of bed.There are often features of chronic ischaemia, such as hairlessness,
pale skin, absent pulses, nail dystrophy and wasting of calf muscles.
Risks for arterial ulcers
Coronary heart disease.
History of stroke or transient ischaemic attack.
Diabetes mellitus.
Peripheral arterial disease including intermittent claudication.
Obesity and immobility.
Treatment:
These will usually require referral for assessment and care. It is really the management
of peripheral vascular disease. See separate article Peripheral Arterial Disease.

1548. A 65yo woman complains of a painful discharging ulcer above her ankle on the
inner side of her left lower leg. Exam: the base of the ulcer is red and covered by a
yellow fibrous tissue. The border is irregular. The skin is tight. What is the single most
likely dx?
a. Arterial ischemia ulcer
b. Malignancy
c. Neuropathic ulcer
d. Pressure ulcer
e. Venous stasis ulcer
Answer: E (Venous ulceration is typically seen just above the medial malleolus.)
Exclusion of options
A; Arterial ulcer - look for reduced pulses in the foot, ankle and possibly femoral artery.
These ulcers occur most commonly in areas of poor blood supply - eg, the tip of the
toes or over the tibia - and are typically painful and deep. Other evidence of poor blood
supply may include peripheral cyanosis and claudication.
An arterial ulcer tends to occur on lateral side of distal leg and leg is pulseless and cool.

B: Malignancy - malignant ulcers in this area are rare but the possibility should not be
overlooked. Watch out for an ulcer with a rolled everted edge. If ulceration occurs in the
area of scar tissue, Marjolin's ulcer should be considered. Chronic venous ulcers can
develop into malignant ones, so any non-healing ulcer should be referred for biopsy.
C: Neuropathic ulcer - this is painless, deep, often with overlying hyperkeratosis and
occur at sites of loss of nerve supply and recurrent trauma, ie the heel, metatarsal
heads.
D: are localized injuries to the skin and/or underlying tissue that usually occur over a
bony prominence as a result of pressure, friction or rub.
Venous ulcers are caused by incompetent valves in the veins of the lower leg, especially
in the perforators. These incompetent valves cause blood to be squeezed out into the
superficial veins, when the calf muscles are contracted, instead of upwards towards the
heart. Dilation of superficial veins occurs (varicosities) and the subsequent raised
venous pressure results in oedema, venous eczema and ulceration. Valves may also
become damaged following the venous hypertension that occurs in pregnant women and
there may be congenital absence of valves.
80% of all leg ulcers are venous ulcers and a large shallow relatively painless ulcer with
an irregular granulating base in the 'gaiter' region of the leg (between the knee and
ankle) is likely to be venous in origin.
Investigations:
Measurement of ankle brachial pressure index (ABPI) using Doppler
Swabs for microbiology
Patch testing
Biopsy
Treatment:
Compression bandage
Debridement and cleaning
Dressing
Antibiotics
Pentoxifylline
Topical steroids
Aspirin
1549. A 55yo woman suffered from an acute MI 5d ago. While she was in the hosp the
pt developed features of pulmonary edema and heart failure. What is the most probable
cause of her present condition?
a. VSD
b. Ruptured papillary muscle
c. Pericarditis
d. A-fib
e. Re-infarction

Answer: B
All of the options listed are complications of MI but the pulmonary edema and heart
failure suggests rupture
Myocardial rupture is most common three to five days after myocardial infarction.
This may occur in the free walls of the ventricles, the septum between them, the
papillary muscles, or less commonly the atria.
Risk factors for myocardial rupture include completion of infarction (no revascularization
performed), female sex, advanced age, and a lack of a previous history of myocardial
infarction.[2] In addition, the risk of rupture is higher in individuals who are
revascularized with a thrombolytic agent than with PCI.Rupture of the papillary muscle
may lead to acute mitral regurgitation and subsequent pulmonary edema and possibly
even cardiogenic shock.
Rupture of papillary muscle or chordae tendinae:
Causes severe mitral regurgitation within the first week after infarction and is a lifethreatening complication. It is most often seen with inferior infarctions.
One study found a median time for papillary muscle rupture in patients treated with
fibrinolysis to be 13 hours after AMI.[2]
Papillary muscle rupture following AMI usually requires mitral replacement.
Ventricular septal rupture and free wall rupture
Risk factors: older age, female gender, non-smoker, anterior infarction, worse Killip class
on admission, increasing heart rate on admission, first myocardial infarction and
hypertension.[6]
Postinfarction VSD is relatively infrequent but life-threatening.[7] The incidence has
dramatically decreased with reperfusion therapy.
May develop as early as 24 hours after myocardial infarction but often presents 2-7 days
afterwards. Mortality rates are greater than 90%.
Ventricular septal rupture:[2]
Patients may initially have no clinically significant cardiopulmonary symptoms but rapid
recurrence of angina, hypotension, shock or pulmonary oedema develop.
Signs of ventricular septal rupture include a new harsh pansystolic murmur best heard at
the left lower sternal border, with worsening haemodynamic profile and biventricular
failure.
Diagnosis is by transoesophageal echocardiography or by showing a step-up in oxygen
saturation in the right ventricle on pulmonary artery catheterisation.
Postinfarction ventricular septal defects require urgent surgical closure.[8]
1550. A 76yo woman presents with deep stroke 6h ago. What would the immediate tx
be?
a. Aspirin 75mg
b. Aspirin 300mg
c. Streptokinase
d. IV heparin
e. Dipyridamole 200mg
Answer b. Aspirin 300mg
Aspirin (low-dose) is the most commonly used antiplatelet medicine when the stroke has
just happened.
Exclusion of other options:
C: If an ischaemic stroke is diagnosed and it has been less than four and a half hours

since symptoms started, you may be given a medicine directly into a vein, called
alteplase. (Here the time given is 6hr).
E: A combination of dipyridamole and aspirin (Acetylsalicylic acid/dipyridamole) is FDAapproved for the secondary prevention of stroke, However, it is not licensed as
monotherapy for stroke prophylaxis.
D: A review by the panel of the Stroke Council of the American Heart Association found
no strong evidence for effectiveness of anticoagulants in treating acute ischemic stroke.

There are two main types of stroke - ischaemic and haemorrhagic.


Stroke is the largest cause of disability in the UK and the third most common cause of
death (after heart disease and cancer). Most cases occur in people aged over 65. Each
year about 1 in 100 people over the age of 75 will have a stroke.
Symptoms of stroke;
Weakness of an arm, leg, or both. This may range from total paralysis of one side of the
body to mild clumsiness of one hand.
Weakness and twisting of one side of the face. This may cause you to drool saliva.
Problems with balance, coordination, vision, speech, communication or swallowing.
Dizziness or unsteadiness.
Numbness in a part of the body.
Headache.
Confusion.
Loss of consciousness (occurs in severe cases).
Investigations:
A brain scan (CT scan or MRI scan). This can determine the type of stroke (ischaemic or
haemorrhagic) and may detect rarer conditions which may have caused the stroke or
which may mimic a stroke.
Blood tests (to determine sugar and cholesterol levels)
CXR and Ecg( to rule out AF)
Immediate care
Ideally, you will be assessed quickly by a doctor. Commonly, a scan of the brain is
organised as soon as possible. The aim of the scan is to confirm the diagnosis and to tell
whether the stroke is an ischaemic or haemorrhagic stroke. This is very important to
know, as the initial treatment of the two is very different.
If an ischaemic stroke is diagnosed and it has been less than four and a half hours since
symptoms started, you may be given a medicine directly into a vein, called alteplase.
This is a clot-busting medicine which aims to dissolve the blood clot. The medical word
for this is thrombolysis. If the blood clot that caused the stroke can be dissolved shortly
after symptoms begin, it can improve the eventual outcome. This is because brain cells
that would have died are able to survive.
Further care
Antiplatelet medication. Platelets are tiny particles in the blood which help blood to clot.
Antiplatelet medication is usually advised if you have had an ischaemic stroke (due to a
blood clot). Antiplatelet medication reduces the stickiness of platelets. This helps to
prevent blood clots forming inside arteries, which helps to prevent a further stroke.
Aspirin (low-dose) is the most commonly used antiplatelet medicine when the stroke has
just happened. Another antiplatelet medicine, clopidogrel, is usually given long-term after

the initial treatment.


If you have atrial fibrillation, you have an increased risk of a blood clot forming in a heart
chamber and travelling to the brain to cause a stroke. If you have atrial fibrillation (or
certain other heart conditions), a medicine called warfarin may be prescribed. Warfarin
helps to prevent blood clots forming. Warfarin is an anticoagulant.
If you have a bleeding (haemorrhagic) stroke and are taking an anticoagulant medicine
such as warfarin, treatment to reverse the effect of the anticoagulation is given.
If a subarachnoid haemorrhage is the cause of the stroke, an operation to fix the leaking
blood vessel (artery) is sometimes an option.

1551. A 19yo man accuses his friend of making his right arm swing out at a stranger.
What is the best term to describe his condition?
a. Control
b. Persecution
c. Guilt
d. Reference
e. Grandeur
Key: Control (A)
Reason: Grandeur signifies lofty ideas about oneself, Persecution signifies a mistrust in
others and belief that they want to harm you, Reference means believing people are
talking about you, Guilt is the emotion of feeling bad about something you did or didnt
do.
Discussion: A delusion is a false belief which is firmly sustained and based on incorrect
inference about reality. This belief is held despite evidence to the contrary and is not
accounted for by the person's culture or religion. Karl Jaspers, a noted psychiatrist and
philosopher, described the three main criteria required for a delusion.
* Certainty - the patient believes the delusion absolutely.
* Incorrigibility - the belief cannot be shaken.
* Impossibility - the delusion is without doubt untrue.
Types of delusions:
* Monothematic - delusions are only relating to one particular topic. * Polythematic range of delusional topics (seen in schizophrenia).
1552. A 26yo man with history of hereditary haemorrhagic telangiectasia is planning to
start a family. What is the mode of inheritance?
a. AD with incomplete penetrance
b. Autosomal co-dominant
c. AR with incomplete penetrance
d. AD
e. AR
Key: Autosomal Dominant (D)

Reason: This disease is also known as Osler-Weber-Rendu disease and is


characterized by abnormal blood vessels in skin, GIT, lung, liver, brain.
Discussion: Hereditary haemorrhagic telangiectasia (HHT) is also known as OslerWeber-Rendu syndrome. The condition is characterised by vascular dysplasia leading to
telangiectasia. Epistaxis and gastrointestinal bleeding are frequent complications of
mucosal involvement. There are also often arteriovenous malformations (AVMs),
particularly of lungs, liver and brain. Most cases are due to mutations in the endoglin
(HHT1) or ACVRLK1 (HHT2) genes.
Presentation:
Age-related penetrance is seen in HHT. It does not present at birth but commonly
presents with recurrent epistaxis, usually in the teenage years. People with the condition
develop mucocutaneous lesions, usually involving the nasal mucosa, lips and tongue.
These lesions are sharply demarcated red-purple macules, papules or spider-like lesions
comprising a mat of
tortuous vessels. These can also occur in the conjunctiva, upper respiratory tract,
gastrointestinal (GI) tract, bladder, vagina, bronchi, brain and liver. Cutaneous
telangiectasias are often not evident until between 20 and 30 years of age.
DD:
CREST syndrome (= Calcinosis, Raynaud's disease, (o)Esophageal dysmotility,
Sclerodactyly, Telangiectasia).
Von Willebrand's disease
Investigations: CT, MRI, Angiography.
Management:
* Acute bleeds may need transfusion.
* Surgical/laser ablation.
* Oestrogen therapy seems to be beneficial.
Complications:
* Hemorrhage
* Cirrhosis
1553. A 50 y/o man with a known history of stroke is unable to get out of his house
because he cant find where the door is. He refuses help from his wife and says he is not
blind. What is the single most likely defect?
a. Paracentral scotoma
b. Tunnel vision
c. Total blindness
d. Central scotoma
e. Cortical blindness
Key: Cortical blindness (E)
Reason: The provided history of a CVA essentially rules out the rest of the options
because the only thing that applies here is a stroke in the Occipital cortex that causes
complete cortical blindness which renders the man helpless when it comes to finding
where the door is in his house.
* Tunnel Vision is a symptom of Retinitis Pigmentosa and comprises of a loss of
peripheral vision leading to a tunnel-like field defect.
* A scotoma is a partial loss of vision or blind spot in an otherwise normal visual field.
Discussion: A stroke means that the blood supply to a part of the brain is suddenly cut
off. The brain cells need a constant supply of oxygen from the blood. There are two main
types of stroke - ischaemic and haemorrhagic.
Symptoms:

* Weakness of an arm, leg, or both. This may range from total paralysis of one side of
the body to mild clumsiness of one hand.
* Weakness and twisting of one side of the face. This may cause you to drool saliva.
* Problems with balance, co-ordination, vision, speech, communication or swallowing.
* Dizziness or unsteadiness.
* Numbness in a part of the body.
* Headache.
* Confusion.
* Loss of consciousness (occurs in severe cases).
Action (1st hour):
* Protect the airway.
* Pulse, BP, ECG (rule out AF for embolus).
* Blood glucose (rule out decreased blood sugar).
* Urgent CT scan.
* Thrombolysis of ischemic stroke and within 4.5 hours by alteplase, etc.
* NPO by passing NG tube.
* IV fluids, but take care not to over-hydrate keeping cerebral edema in mind.
* Once hemorrhagic stroke is excluded via CT scan, give aspirin 300mg atleast.
* Once the patient has been managed acutely, investigate further to find out the cause of
the stroke while organizing appropriate rehabilitation of the patient at the same time.
1554. An elderly lady presents with confusion. She is apyrexial but complains of dysuria
for 2 days duration. What is the definitive dx investigation?
a. Blood culture
b. Urine nitrites
c. CT head
d. ECG
e. IVU
Key: Urine Nitrites (B)
Reason: According to the OHCM pg. 288, if symptoms are present, dipstick the urine
and treat empirically if nitrites or leucocytes are positive while awaiting culture of an
MSU.
ECG and CT Head are useless tests and IVU doesnt make sense because we arent
looking for an obstructive cause. You only go for imaging studies if it is recurrent and
there is failure with standard treatment. Most accurate test URINE CULTURE.
Discussion:
Urinary tract infection (UTI) - this implies the presence of characteristic symptoms and
significant bacteriuria from kidneys to bladder. Many laboratories regard 105 colonyforming units per millilitre (cfu/ml) as the threshold for diagnosing significant bacteriuria.
Organisms:
Several micro-organisms are known to cause UTI, but the majority of infections will be
produced by three organisms:
* Escherichia coli
* Staphylococcus saprophyticus
* Proteus mirabilis
Infection with less common organisms is more likely to occur in patients who have
underlying pathology and/or frequent infections, are immunosuppressed, or who are
catheterised. Organisms which may produce infection under these circumstances
include:
* Klebsiella spp.

* Proteus vulgaris
* Candida albicans
* Pseudomonas spp.
Signs: Fever, abdominal/ loin tenderness, foul smelling urine.
Symptoms: Frequency, dysuria, urgency, hematuria, suprapubic pain.
Tests:
* Dipstick urine nitrites or leukocytes positive mean UTI. Treat empirically.
* MSU sample for culture and sensitivity.
* CBC, Urea & Electrolytes, Blood culture if systemically unwell.
Imaging: usually done in males, children or persistent cases.
* USG and refer to urology for assessment.
* CT-KUB.
* Urodynamics.
Management:
* Drink plenty of fluids.
* Trimethoprim 200mg x 12 hourly PO OR Nitrofurantoin 50mg x 6 hourly PO for 3-6
days OR Amoxicillin 500mg x 8 hourly PO.
* Second-line: Co-Amoxiclav 7 day course PO.
1555. A 40yo woman on chemotherapy for metastatic breast carcinoma now presents
with painful swallowing. Exam: she has white plaques on top of friable mucosa in her
mouth and more seen on esophagoscopy. What is the most effective tx for this pt?
a. Antispasmodic
b. H2 blocker
c. Antibiotics
d. Antifungals
e. I&D
Key: Antifungals (D)
Reason: The immunocompromised state, location of infection, gross picture all point to
Esophageal Candidiasis which is treated with antifungals. The treatment of choice is
Fluconazole 400mg STAT, then 200mg per day PO.
Discussion:
* Candidal infection is the most common cause of invasive fungal infections in hospital
patients.
* Dysphagia, retrosternal discomfort.
* Fluconazole 50-100mg PO x 24hourly for 7-14 days.
* If invasive, fluconazole 400mg/day. Side effects nausea, raised LFTs,
thrombocytopenia.
* Severe systemic infection, amphotericin B.
1556. A 43yo woman has suffered with heavy periods for many years and has tried
many medical tx without success. She is constantly flooding and at times cant leave her
house due to heavy bleeding. She has completed her family of 5 children and her last
blood test showed
Hgb=8.9g/dl. She feels that she cant cope with the bleeding anymore and her husband
is asking for a tx that can guarantee success. What is the most appropriate management
to improve menorrhagia in this pt?
a. Endometrial ablation
b. Hysterectomy
c. Hysteroscopic/Laser resection of fibroids
d. Myomectomy

e. UAE
Key: Hysterectomy (B)
Reason: The woman is 43 years old, she has completed her family, her illness is of
severe distress to her and her family and her Hb is falling. Hysterectomy is the logical
choice because it makes the most sense. Endometrial ablation or laser resection have
adverse effects, myomectomy isnt at all curative in this patient's case and Uterine Artery
Embolization leads to recurrence which would be an unwanted effect in this patient.
Hysterectomy is the t/m of choice.
Discussion:
* May be associated with RCC.
* Oestrogen dependent.
* Present with menorrhagia, fertility problems, pain, mass.
* Management: Mirena for menorrhagia. Hysterectomy in women who have completed
their families. In younger women, a reversible menopausal state may be induced with
analogs of LHRH (goserelin). Side effects include menopausal symptoms and bone loss.
Fertility and fibroids used when drugs stopped, used pre-op to reduce bulk and in those
unfit for surgery.
* Myomectomy may be chosen in subfertility. Laparoscopic surgery with laser use is
possible. Complications severe haemorrhage requiring hysterectomy, post op
adhesions, embolising fibroids can shrink them (not widely available and very painful).
1557. A man on antipsychotic meds develops features of retinitis pigmentosa. Which
drug is most likely to cause these symptoms?
a. Thioridazine
b. Haloperidol
c. Chlorpromazine
d. Risperidone
Key: Thioridazine (A)
Reason: Simple factual statement. Thioridazine has been shown to cause symptoms
similar to retinitis pigmentosa including tunnel vision and night blindness.
Discussion:
* Mostly autosomal recessive. Also autosomal dominant or X-linked.
* Most common retinal hereditary disease.
Slit-lamp biomicroscopy is the key initial assessment. Further tests are to determine the
functional integrity of the retina and optic nerve:
* Visual acuity
* Visual field assessment
* Pupillary reflex response
* Colour defectiveness determination
* Refraction
Intraocular pressure will also need to be measured. To find out more about these tests,
see the separate article on Examination of the Eye. Imaging includes:
o Retinal photography
o Ultrasound of the eye
o Fluorescein angiography
o Optical computer tomography (OCT)
All of these can be performed in a general clinic. The most critical diagnostic test is the
electroretinogram (similar to the EEG of the brain or ECG of the heart).
Management:
o Vitamin A/beta-carotene

o Acetazolamide
o Lutein
o Bilberry
o Immunosuppressive agents (including steroids)
1558. Pt with low Hb, MCV=76, angular stomatitis, red tongue, and koilonychia. What is
the most probable dx?
a. Folate def
b. B12 def
c. Iron def
d. Vit E def
e. Hemolytic anemia
Key: Iron Deficiency Anemia (C)
Reason: Folate and B12 are automatically ruled out with the low MCV. Vitamin E
deficiency hasnt been shown to cause anemia. This is probably a fore-runner of
Plummer-Vinson syndrome which is related to Iron Deficiency Anemia in this case. The
treatment is oral Iron (Ferrous Sulphate 200mg PO x 8 hourly). Continue until Hb normal
and for atleast 3 months to replenish stores.
1559. A pt with sudden severe eye pain, red eye, visual blurring, acuity of only finger
counting, nausea, vomiting with a shallow ant chamber that is hazy on shining a torch.
What is the dx?
a. CRVO
b. Acute closed angle glaucoma
c. Uveitis
d. Iritis
e. Open angle glaucoma
Key: Acute Closed Angle Glaucoma (B)
Reason: The painful presentation rules out CRVO and Open Angle Glaucoma. The
characteristic nausea, vomiting and shallow anterior chamber signifies an acute event,
which in this case would be Acute Closed Angle Glaucoma. Uveitis and Iritis will not
have nausea or vomiting which signify markedly raised intraocular pressure.
Treatment
* Pilocarpine 2 drops x 2 hourly
* Acetazolamide 500mg IV x STAT OR Mannitol IV
* Good analgesics
* Anti-emetics
* Admit to monitor IOP
1560. A patient who works in a pet shop has temp = 37.5C, dyspnoea, chest pain and
cough. CXR: patchy consolidation. What is the most suitable t/m?
a. Amoxicillin
b. Tetracycline
c. Erythromycin
d. Clarithromycin
e. Penicillin
Key: Amoxicillin (A)

Reason: This is a mild case of Community Acquired Pneumonia (CAP) and the patient
has presented to you in a GP setting. Now, while the history would point towards
Chlamydia pneumonia or Chlamydia psittaci being the commoner cause of this
pneumonia, unless that is proved, you will treat this as Streptococcus pneumonia
acquired pneumonia which is the commonest form of pneumonia in this scenario.
Treatment of choice will be Oral Amoxicillin 500mg 1g x 8 hourly.
1561. A 50yo man complains of dysphagia after eating bread. Barium swallow reveals a
lower
Oesophageal ring. What is the most appropriate t/m?
a. Reassurance
b. Antispasmodics
c. Dilatation of the LES
d. Endoscopic diverticulectomy
e. I&D
Key: Dilatation of the LES (C)
Reason: There is a ring at the lower oesophagus. It is likely a stricture and the treatment
is endoscopic balloon dilatation. Reassurance will not help the patient, antispasmodics
are given in diffuse esophageal spasm and would not be helpful here and
diverticulectomy is mentioned for Zenkers diverticulum. Incision and drainage is stupid.
Discussion:
* Esophageal rings are concentric, smooth, thin extension of normal oesophageal tissue,
usually 3 to 5 mm thick. They consist of mucosa, submucosa and muscle. They may be
an incidental finding at barium studies or endoscopy. Incidence is unknown as most are
asymptomatic. There is no sex difference in the incidence of rings except that multiple
rings are usually found in young men. Rings are classified as A, B and C:
o A is uncommon and is a muscular ring several centimetres proximal to the
squamocolumnar junction. It may be an inconstant finding on barium swallow and there
is some debate as to whether it really is an anatomical entity.
o B ring or Schatzki's ring is really a web, as it involves only mucosa and submucosa. It
tends to mark the proximal part of a hiatus hernia and usually presents in a patient aged
over 50 whose main complaint is intermittent dysphagia to solid food, spanning months
or years, and it is non-progressive.
o C ring is a rare X-ray finding of indentation caused by the diaphragmatic crura. It rarely
causes symptoms.
1562. A 48yo nulliparous woman feels tired all the time. Her periods are regular but have
always
lasted for at least 10d. Choose the single most appropriate initial investigation?
a. High vaginal swab
b. Serum Hgb conc
c. TFT
d. None
e. Abdominal US
Key: Serum Hb Conc. (B)
Reason: The single most appropriate investigation for an elderly woman presenting with
heavy but regular bleeding and history of fatigue will be Serum Haemoglobin to rule out
anaemia. There should also be a low threshold for doing TFTs in this patient as
Hypothyroidism is a likely cause of menorrhagia and could also be considered the

answer. But the delay in result for TFTs would take it away from the most appropriate
investigation.
1563. A man got his hand caught in machinery at work. The fingers are swollen but the
XR shows no fracture. What is the most appropriate management?
a. Splint
b. Put in plaster
c. Broad arm sling for 1wk
d. Elevate in high sling for 2d
e. Neighbour strapping
Key: Elevate in high sling for 2 days (D)
Reason: The injury is not a severe one and shows swollen fingers indicating soft tissue
injury with no fracture. This essentially rules out all immobilizing treatment forms like
splinting and neighbour strapping or POP plastering. The best way to treat this would be
with elevation for 2 days and painkillers for the pain.
1564. A 39yo woman presents with symptoms recurring annually characterized by
depressed mood, being socially withdrawn, hypersomnia, lack of enjoyment in life which
last for several months. What is the most likely dx?
a. Seasonal Affective Disorder
b. Moderate depression
c. Dysthymia
d. GAD
e. Bipolar disorder
Key: Seasonal Affective Disorder (A)
Reason & Discussion: The annual nature of her symptoms coupled with the duration
would most likely make this a case of seasonal affective disorder, which is a
constellation of symptoms occurring mostly in the winter months characterized by the
aforementioned habits and chronic low mood. Depression has a time-frame that this
scenario doesnt meet, Dysthymia is depression with less severe but longer lasting
symptoms, GAD is unlikely as there arent any panic symptoms, Bipolar would require
periods of mania. Treatment for SAD is CBT and light therapy. The diagnosis of SAD is
based on:
* Depression cycles on a regular basis during autumn/winter.
* Full remission of symptoms in spring/summer.
* Seasonal symptoms for at least two consecutive years.
* Atypical features, which may or may not be present.
During the psychological examination, it is also important to assess:
* Suicidal ideation.
* Abnormal mechanisms of coping - eg, social isolation, alcohol use.
Treatment options also include Bupropion, recently shown to have benefit in SAD and
Agomelatine, a melatonin agonist.
1565. A 75yo man presents with ARF. He has been troubled by recurrent epistaxis but
over the last 3wks he reports to have coughed up blood too. What is the single most
likely positive antibody?
a. P ANCA
b. C ANCA
c. Anti Ro
d. Anti DS DNA

e. Anti centromere
Key: C ANCA (B)
Reason: This patient has Wegeners Granulomatosis which affects the upper and lower
respiratory tract and presents with renal insufficiency. It is C ANCA positive and is
treated by steroids or cyclophosphamide.
Discussion: Also called granulomatosis with polyangiitis. Affects the upper respiratory
tract, lungs and kidneys.
* Nasal obstruction, ulcers, epistaxis. Destruction of nasal septum causing a
characteristic saddle nose deformity.
* Rapidly progressive GN with crescent formation, hematuria and proteinuria can occur.
* Cough, hemoptysis, pleuritis.
Investigations:
* C-ANCA, Raised ESR and CRP.
* Urine R/E to check for proteinuria or hematuria. If present, renal biopsy to confirm.
* Chest X-ray Nodules and fluffy infiltrates of pulmonary haemorrhage.
* CT Scan Diffuse alveolar haemorrhage.
Treatment:
* Severe disease should be treated with steroids and cyclophosphamide or rituximab to
induce remission.
* Methotrexate or azathioprine for maintenance.
* Patients with severe renal disease may benefit for plasma exchange.
* Co-Trimoxazole should be given as prophylaxis against P. jiroveci infection.
1566. A woman is admitted to the hospital for elective abdominal hysterectomy. 2
months ago she was dx with DVT and pulmonary embolism and was started on warfarin.
What is the most appropriate pre-op measure you will take on this occasion?
a. Continue warfarin
b. Stop warfarin
c. Stop warfarin and start heparin
d. Increase warfarin dose
e. Add heparin
Key: Stop warfarin and start heparin (C)
Reason: Warfarin depletes Vit. K dependent clotting factors. It should be stopped for
them to increase in amount and heparin can provide short term cover in the meantime.
Discussion: A deep vein thrombosis (DVT) is a blood clot in a vein. Blood clots in veins
most often occur in the legs but can occur elsewhere in the body, including the arms.
Warfarin works by depleting Vitamin K dependent factors in the body and is used to
maintain an INR according to the specific condition of the patient. Now, heparin is a
short term fix and is only available in the form of injectables. In the case of this woman,
we need anti-coagulant cover as well as making sure she doesnt bleed excessively
during her surgery. The only way to do that is to stop
Warfarin because of its delayed reversibility regarding its effect and to administer short
acting heparin injections in the gap created by halting oral Warfarin.
1567. This condition affects middle aged women more than men and is characterized by
low mood, early morning waking, loss of libido, tiredness and suicidal intention last for at
least 2wks. What is the most probable dx?
a. Bipolar affective disorder
b. Dysthymia
c. Major depressive disorder

d. Schizoaffective disorder
e. Recurrent brief depression
Key: Major depressive disorder (C)
Reason: This is C because it fulfils the five criteria of Major Depression which are
(according to NICE guidelines):
* Loss of interest or pleasure [HAS TO BE THERE]
Any four out of the following:
* Poor appetite or weight loss
* Early waking
* Psychomotor retardation
* Decreased libido
* Decreased ability to concentrate
* Ideas of worthlessness of guilt
* Suicidal thought or action
Severity is based on the extent of symptoms and their functional impact:
* Subthreshold depressive symptoms - <5 symptoms.
* Mild depression - few, if any, symptoms in excess of the 5 required to make the
diagnosis, with symptoms resulting only in minor functional impairment.
* Moderate depression - symptoms or functional impairment are between 'mild' and
'severe'.
* Severe depression - most symptoms present and the symptoms markedly interfere
with normal function. It can occur with or without psychotic symptoms.
Discussion: Antidepressants are not recommended for the initial treatment of mild
depression, because the risk: benefit ratio is poor. However, their use may be
considered:
* If mild depression persists after other interventions, or is associated with psychosocial
and medical problems.
* In mild depression complicating the care of physical health problems.
* When a patient with a history of moderate or severe depression presents with mild
depression.
* With subthreshold depressive symptoms present for at least two years or persisting
after other interventions.
1568. A 10yo boy has fallen from a tree and injured his right chest. He has pain and
difficulty breathing. He is tachypneic and tender with an area of paradoxical chest wall
movement on the right side. What is the single most likely dx?
a. Diaphragmatic rupture
b. Flail chest
c. Fractured ribs
d. Hemothorax
e. Tension pneumothorax
Key: Flail Chest (B)
Reason: H/O Fall, Pain and difficulty breathing, Tender with paradoxical chest wall
movement ---> FLAIL CHEST.
Discussion: A case of flail chest should be managed according to the following
principles:
* High flow O2.
* CPAP.
* Chest physiotherapy.
* Adequate analgesia.

* Early intubation and ventilation in patients going towards or currently in respiratory


failure.
* Open fixation for patient who are unable to be weaned from ventilator or when
thoracotomy is performed for other injuries.
1569. A 37yo woman had an elective LSCS 1d ago. You are called to see her as she
becomes SOB with left sided chest pain and a cough. She has had 3 children, 2 born by
LSCS. Exam: she has reduced air entry at left lung base. Her observations include
sat=92% on air, BP=105/84mmHg, pulse=120bpm, temp=37.2C. Choose among the
options which C-section complications has she developed?
a. Aspiration pneumonia
b. Aspiration pneumonitis
c. Spontaneous pneumothorax
d. Pulmonary embolism
e. DVT
Key: Pulmonary embolism (D)
Reason: Decreasing saturation, Tachycardia, Presentation 1 day after major surgery
after Pregnancy and bring post-partum. Reduced air entry cannot be positive in any
other option and DVT would actually lead to Pulmonary Embolism. Pulmonary Embolism
is especially common in abdominal, pelvic surgeries ad hip/knee replacements.
Pregnancy and post-partum state is also a risk factor.
Discussion: Usually arises from a venous thrombosis in the pelvis or legs. Rare causes
include right ventricular thrombus, post MI, septic emboli, right sided endocarditis,
fat/air/amniotic fluid embolism, neoplastic cells, parasites.
Risk factors: Recent surgery especially abdominal, pelvic or hip/knee replacement.
Thrombophilia (antiphospholipid syndrome), leg fracture, prolonged bed rest,
malignancy, pregnancy, post-partum state, COCPs, HRT, previous pulmonary
embolism.
Symptoms: Acute dyspnoea, pleuritic chest pain, hemoptysis, dizziness, syncope.
Signs: Pyrexia, cyanosis, tachypnoea, tachycardia, hypertension, raised JVP, pleural
rub, pleural effusion. Look for signs of a cause like DVT.
Tests: CBC, Urea & Electrolytes, Clotting profile, D- Dimers. ABGs may show decreased
PaO2 and decreased PaCO2.
Imaging: Chest xray usually normal, ECG may be normal or show right ventricular strain
pattern (inverted T in V1 V4). Check OHCM Pg. 183 for table showing diagnosis and
scoring.
Management:
* Establish IV access.
* Start LMWH tinzaparin 175 units/kg/24h subcutaneous.
* Start colloid if systolic BP less than 90, manage BP accordingly, prevent hypovolemia.
* If BP is more than 90, start Warfarin. Loading regime 5-10mg PO.
* If obvious remedial cause, 6 weeks of warfarin is enough.
* If recurrent embolus or underlying malignancy, give Warfarin for more than 3-6 months.
* Investigate underlying cause.
* Stop heparin when INR is >2 and start warfarin.
* Thrombolysis for a massive pulmonary embolism, give alteplase 10mg IV over 1
minute, then 90mg IV over 2 hours. Max. 1.5mg/kg if less than 65 kg. Consider
placement of a vena caval filter in patients who develop emboli despite adequate anticoagulation.
1570. A pt. presents with increasing retrosternal pain and dysphagia for both solids and
liquids over 18m but denies weight loss. Chest is clear. What is the most likely dx?

a. Achalasia
b. Pharyngeal carcinoma
c. Oesophageal spasm
d. Oesophageal stricture
Key: Oesophageal Spasm (C)
Reason: Pain and no h/o wt. loss rules out CA Pharynx, Achalasia has non-progressive
dysphagia for both solids and liquids. Solids first, progressing to liquids would indicate
stricture benign or malignant. Spasm has intermittent dysphagia plus chest pain.
1571. A 70yo man presents with a fluctuant swelling of the scrotum which feels like
worms when he is standing but regresses when he lies down. What is the most probable
dx?
a. Varicocele
b. Hematocele
c. Testicular CA
d. Epididymal cyst
e. Saphena varix
Key: Varicocele (A)
Reason: BAG OF WORMS. Hello.
Hematocele would be a collection of blood in the tunica vaginalis with h/o trauma. CA
would have history of painless lump with weight loss and hormonal effects. Epididymal
cyst usually develops in adulthood and contains clear or milky fluid, lies above and
behind the testes. Saphena Varix would lie below and lateral to the scrotum. A
varicocele is a collection of enlarged (dilated) veins (blood vessels) in the scrotum. It
occurs next to and above one testicle (testis) or both testes (testicles). Treatment is
usually conservative with the best option being to leave the varicocele alone if it isnt
symptomatic. If it is, consider surgery or injection of a substance to block the veins. Both
methods work well usually.
1572. A 52yo woman has had a swelling in the neck, hoarseness and stridor-both
inspiratory and expiratory for 2 months. What is the most probable dx?
a. CA larynx
b. CA thyroid
c. Vocal cord nodules
d. CA bronchus
e. Thyrotoxicosis
Key: CA thyroid (B)
Reason: Vocal cord nodules will not have swelling, CA Larynx will not have swelling
either. CA Bronchus would present with hemoptysis and dyspnoea. Thyrotoxicosis
would have other systemic signs. This is CA Thyroid unless proven otherwise.
Discussion:
1. Papillary (60%): Younger patients, spread to lymph nodes and lung. Treatment total
thyroidectomy to remove non obvious tumour as well +/- node excision +/- radioiodine to
ablate residual cells. Give thyroxine to suppress TSH. Prognosis better if young and
female.
2. Follicular (25%): Middle age, spreads early via blood to bone and lungs, welldifferentiated. Treatment total thyroidectomy + T4 suppression + radioiodine ablation.
3. Medullary (5%): Sporadic or part of MEN syndrome, may produce calcitonin which
can be used as a tumour marker, they do not concentrate iodine. Perform a
pheochromocytoma screen post-op. Treatment is thyroidectomy + node excision.
External beam radiotherapy to prevent regional recurrence.

4. Lymphoma (5%): Female: male is 3:1. May present with stridor or dysphagia, do full
staging pre-treatment. Treatment chemoradiotherapy. Assess histologically for MALT
origin.
5. Anaplastic (rare): Female to male ratio is 3:1. Elderly, poor response to treatment. In
the absence of unresectable disease, excise plus radiotherapy.
1573. A woman became acutely SOB in the recovery bay and is coughing after GA.
Auscultation: reduced air entry at the right lung base and diffuse wheeze. Observation:
HR=88bpm, BP=112/76mmHg, temp=37.8C and sat=91% in air. Choose among the
options which C-section complication has she developed?
a. Aspiration pneumonitis
b. Spontaneous pneumothorax
c. Endometritis
d. Pulmonary embolism
e. Tension pneumothorax
Key: Aspiration pneumonitis (A)
Reason: Everything aside, look at the presentation of this illness. The woman presented
acutely after GA and with different vitals to the great mimicker in this period of illness
which would be Pulmonary Embolism. Vitally stable with mildly reduced SpO2 and
coughs after GA. The wheeze also points to aspiration. Pneumothorax wouldnt present
this way and neither would a tension pneumothorax. Endometritis is just stupid here.
1574. A 23yo female presents with paraesthesia and loss of distal pulses in her arms.
She is noted to be hypertensive. She describes feeling unwell a month prior with fever
and night sweats. What is the most probable dx?
a. Kawasaki disease
b. Takayasu arteritis
c. Buergers disease
d. Embolism
e. Raynauds phenomenon
Key: Takayasu Arteritis (B)
Reason: Takayasu arteritis because it presents in age < 50 years and loss of distal
pulses (pulseless disease). It is a systemic vasculitis which often affects women 20-40
years. Systemic features are common like fever, weight loss and night sweats.
Hypertension is also likely due to renal artery stenosis. Treatment is Prednisolone
1mg/kg/day.
Buergers would require a long smoking history. Kawasaki would present with
mucocutaneous symptoms. Raynauds would present with cyanosis of distal digits.
Discussion: Also called aortic-arch syndrome or pulseless disease. Rare outside of
Japan, systemic vasculitis. Affects the aorta and its major branches.
* Granulomatous inflammation, causes stenosis, thrombosis and aneurysms.
* Women aged 20-40 year.
* Symptoms depend on the artery involved with cerebral, eye and upper limb symptoms
dizziness, visual changes, weak arm pulses.
* Systemic features fever, weight loss, malaise, increased BP often due to renal artery
stenosis.
* Complications aortic valve regurgitation, aortic aneurysm and dissection, ischemic
stroke (increased BP and thrombus) and IHD.
* Diagnosis increased ESR and CRP, MRI & PET allow earlier diagnosis than
angiography.

* Treatment Initially prednisolone, methotrexate and cyclophosphamide in resistant


cases. BP control. Angioplasty, stenting, bypass for critical stenosis.
1575. A 35yo woman presents with mass in the groin. Exam: mass found just below and
lateral to the pubic tubercle. There is no cough impulse and it is irreducible. What is the
most probable dx?
a. Direct inguinal hernia
b. Strangulated hernia
c. Femoral hernia
d. Saphena varix
e. Femoral aneurysm
Key: Femoral Hernia (C)
Reason: Femoral Hernias are common in women and they present below and lateral to
the pubic tubercle. They are likely to be irreducible and have no cough impulse. It isnt
strangulated because strangulation presents with systemic signs like vomiting, fever,
pain and patient becomes toxic and requires surgery. Inguinal hernias would be above
and medial to the pubic tubercle. Saphena Varix is a dilatation of the termination of the
great saphenous vein into the sapheno-femoral junction but it has a positive cough
impulse and would also reduce spontaneously on lying down. The aneurysm would be
pulsatile and have a bruit as well.
1576. A 30yo woman has injured her left lower chest in a RTA. She has BP=80/50
mmHg,
Pulse= 120 bpm. Auscultation of chest= bowel sounds present. What is the single most
likely dx?
a. Diaphragmatic rupture
b. Flail chest
c. Fractured ribs
d. Ruptured oesophagus
e. Tension pneumothorax
Key: Diaphragmatic Rupture (A)
Reason: She is vitally unstable and bowel sounds are auscultate-able in her chest. Clear
answer of diaphragmatic rupture secondary to RTA. A flail segment would have a
paradoxically moving rib segment and rib fractures would show up on CXR.
Oesophageal rupture would have subcutaneous emphysema and Tension
pneumothorax would present with deviated trachea and absent breath sounds on one
side.
1577. A lady presents with a swelling below the groin crease that can be reduced. There
is no med hx of note. What is the most probable dx?
a. Inguinal hernia
b. Strangulated hernia
c. Testicular tumor
d. Epididymal cyst
e. Femoral hernia
Key: Femoral Hernia (E)
Reason: Obvious. Only option that fits. None of the other swellings are below the groin
crease, rather they are located in the scrotum.
1578. A 32yo woman of 38wks GA attends the antenatal day unit with pain in the
suprapubic area that radiates to the upper thighs and perineum. It is worse on walking.

Her urine dipstick showed a trace of protein but no white cells, nitrates or blood. Whats
the most likely dx?
a. Braxton Hicks contractions
b. Round ligament stretching
c. Symphysis pubis dysfunction
d. Labour
e. Complicated femoral hernia
Key: Symphysis Pubic Dysfunction (C)
Reason: It is a condition that causes excessive movement of the pubic Symphysis and
pain or discomfort in the pelvic region. The pain in the supra-pubic area radiating to the
legs and perineum combined with worse symptoms on walking would point to only one
thing Symphysis Pubis Dysfunction. The Urine exam is a detractor. Braxton-Hicks
contractions or Labour would not present this way, neither in intensity nor radiation.
Round ligament stretching doesnt make sense here and femoral hernia is not
associated with these symptoms.
1579. A 45yo mechanic presents with a reducible swelling in the groin, impulse on
coughing is present. He has mild dragging pain in the abdomen, otherwise hes normal.
What is the management strategy?
a. Truss
b. Elective herniorrhaphy
c. Urgent herniorrhaphy
d. Elective herniotomy
e. Reassure
Key: Elective Herniorrhaphy (B)
Reason: Herniotomy is done in children, Truss not suitable here, Urgent herniorrhaphy is
done only in strangulated cases. Reassurance isnt suitable here.
1580. A 25yo man present with a mass in the groin after heavy lifting. Exam: mass is
found just above and medial to the pubic tubercle. It is reducible. On applying pressure
on the internal ring,
cough impulse is still present. What is the most likely dx?
a. Direct inguinal hernia
b. Indirect inguinal hernia
c. Femoral hernia
d. Strangulated hernia
e. Femoral aneurysm
Key: Direct inguinal hernia (A)
Reason: Above and medial to the pubic tubercle, reducible inguinoscrotal swelling, ring
occlusion test ve indicates a direct inguinal hernia rather than an indirect inguinal
hernia. Discussion: Hernias always contain a portion of peritoneal sac and may contain
viscera, usually small bowel and omentum.
Presentation:
* Swelling in the groin that may appear with lifting and be accompanied by sudden pain.
* Indirect hernias are more prone to cause pain in the scrotum and cause a 'dragging
sensation'.
* An impulse (increase in swelling) may be palpable on coughing.
* It may not be possible to see the hernia if it is reduced.
* If a lump is present, it may be reducible

There are two types of inguinal hernia:


* Indirect: a protrusion through the internal inguinal ring passes along the inguinal canal
through the abdominal wall, running laterally to the inferior epigastric vessels. This is the
more common form accounting for 80% of inguinal hernias, especially in children. It is
associated with failure of the inguinal canal to close properly after passage of the testis
in utero or during the neonatal period.
* Direct: the hernia protrudes directly through a weakness in the posterior wall of the
inguinal canal, running medially to the inferior epigastric vessels. It is more common in
the elderly and rare in children.
Differential diagnosis: * Femoral hernia: this is seen in various forms, at simplest as a
small swelling in the top of the inside of the thigh. Alternatively, it may be deflected to
appear higher as an inguinal hernia. It is either irreducible or reduces only slowly with
pressure. * Hydrocele (when differentiating from an inguinoscrotal hernia, note that it is
possible to get above a hydrocele on examination).
* Spermatic cord hydrocele.
* Lymph node swelling. * Abscess.
* Saphena varix.
* Varicocele.
* Bleeding. * Undescended testis.
Management:
* Conventional surgery was based on Bassini's operation; this consisted of apposition of
the transversus abdominis and transversalis fascia and the lateral rectus sheath to the
inguinal ligament. The Shouldice technique uses two layers of running suture in a similar
fashion.
* However, the Lichtenstein technique is widely used, where a piece of open-weave
polypropylene mesh is used to repair and reinforce the abdominal wall. This operation is
easier to learn, gives earlier mobility and has a very low recurrence rate. The standard
repair now uses prostheses, usually polypropylene mesh. It is, however, associated with
a slightly increased risk of infection.
* Some of the traditional meshes are heavy and associated with postoperative stiffness
and pain. This has led to the development of lighter meshes. A systematic review has
failed to find any differences in long-term and short-term complications between the two.
* Laparoscopic repair is usually reserved for recurrences and bilateral hernias. There is
less postoperative pain, full recovery is better, and return to work is faster. However, the
price is increased compared with the conventional approach, and there appears to be a
higher number of serious complications of visceral (especially bladder) and vascular
injuries.
* There are two approaches: either the transabdominal preperitoneal (TAPP) or the
totally extraperitoneal (TEP) procedure. In TAPP, the surgeon goes into the peritoneal
cavity and places a mesh through a peritoneal incision over possible hernia sites. TEP is
different, as the peritoneal cavity is not entered and mesh is used to seal the hernia from
outside the peritoneum. The mesh, where used, becomes incorporated by fibrous tissue.
* Surgery can be performed on a day-case basis, and for seven days afterwards the
patient should avoid driving and lifting. The patient should be able to resume normal
activities over the subsequent 2-3 weeks, but, with a heavy job, it can take up to six
weeks to return to work.
* A truss may be required where surgery is inadvisable or refused; however, it can be
difficult for patients to manage and cannot be recommended as a definitive form of
treatment.
Complications:
These include:

* Recurrence: 1.0% - most happening within five years of operation. Recurrence rate
increases:
o In children aged younger than 1 year
o In elderly patients
o After incarcerations
o In those with ongoing increased intra-abdominal pressure
o Where there is growth failure
o With prematurity
o Where there are chronic respiratory problems
o In girls with sliding hernias
* Infarcted testis or ovary with atrophy.
* Wound infection. * Bladder injury.
* Intestinal injury.
* A hydrocele from fluid accumulation in the distal sac usually resolves spontaneously
but sometimes requires aspiration.

1581. A 35yo woman presents with a swelling in the neck. The swelling has increased in
size gradually over the last two years and the patient feels she has difficulty with
breathing. Exam: mass measures 8cm by 10 cm, soft and not warm to touch. It moves
with deglutition. Which is the most appropriate management of this mass?
a. Partial thyroidectomy
b. Oral thyroxine
c. Oral propylthiouracil
d. Excision biopsy
Key: Partial thyroidectomy (A)
Reason: The mass is large and obviously arising from the thyroid. The fact that it is
causing difficulty breathing would indicate a thyroidectomy. Thyroxine would be given in
hypothyroidism, PTU is given as an antithyroid drug and excision biopsy is undertaken if
CA is suspected which isnt the case here as the swelling is not hot. The mass is also
soft which doesnt go towards CA.
Discussion: Surgery is indicated in simple goitre if:
* There is clinical or radiological evidence of compression of surrounding structures,
especially the trachea.
* There are substernal goitres, which are best removed surgically, as biopsy is difficult
and clinical observation without frequent CT or MRI scans is impossible.
* The goitre continues to grow.
* There are cosmetic reasons - for example, large or unsightly.
Types of thyroid operations:
* Thyroid lobectomy to remove a nodule (solitary hot or cold nodules) and goitres that
occur in one lobe.
* Partial thyroid lobectomy to remove a solitary nodule in one specific part of the thyroid.
* Thyroid lobectomy with isthmectomy for benign Hrthle cell tumours and for nonaggressive thyroid cancers.
* Subtotal thyroidectomy (leaving enough of the gland to produce some hormones) is
now little used and has been replaced by total thyroidectomy or thyroid lobectomy alone.
* Total thyroidectomy for thyroid cancers, Hrthle cell tumours and also increasingly for
multinodular goitres and patients with Graves' disease.

* Robotic surgery: advantages include three-dimensional imaging and tremor


elimination. Robotic thyroid surgeries include thyroid lobectomy, total thyroidectomy,
central compartment neck dissection, and radical neck dissection for benign and
malignant thyroid diseases.
Thyroid surgery is safe in the elderly, assuming careful preoperative evaluation and risk
stratification.
Complications:
Possible complications following thyroid surgery include:
* Minor complications such as collections of serous fluid (they resolve spontaneously if
small and asymptomatic but may require single or repeated aspiration if large) and poor
scar formation.
* Bleeding, which may cause tracheal compression.
* Recurrent laryngeal nerve injury:
o Innervates all of the intrinsic muscles of the larynx, except the cricothyroid muscle.
o Patients with unilateral vocal fold paralysis present with postoperative hoarseness.
o Presentation is often subacute and voice changes may not present for days or weeks.
o Unilateral paralysis may resolve spontaneously.
o Bilateral vocal fold paralysis may occur following a total thyroidectomy and usually
presents immediately after extubation.
o Both vocal folds remain in the paramedian position, causing partial airway obstruction.
* Hypoparathyroidism: the resulting hypocalcaemia may be permanent but is usually
transient. The cause of transient hypocalcaemia postoperatively is not clearly
understood. * Thyrotoxic storm: is an unusual complication of surgery but is potentially
lethal.
* Superior laryngeal nerve injury:
o The external branch provides motor function to the cricothyroid muscle.
o Trauma to the nerve results in an inability to lengthen a vocal fold and thus to create a
higher-pitched sound.
o The external branch is probably the most commonly injured nerve in thyroid surgery.
o Most patients do not notice any change but the problem may be career-ending for a
professional singer.
* Infection: occurs in 1-2% of all cases. Peri-operative antibiotics are not recommended
for thyroid surgery. * Hypothyroidism.
* Damage to the sympathetic trunk may occur but is rare.
1582. A 46yo labourer reports swelling in the right groin. The non-painful swelling is
observable in both the erect and the recumbent positions. Exam: non-tender irreducible
4 cm mass in the right groin below and on the medial side of the inguinal ligament.
Which is the most likely dx in this pt.?
a. Indirect inguinal hernia
b. Femoral hernia
c. Saphenous Vein Varicocele
d. Hydrocele
Key: Femoral Hernia
Reason: The swelling is below and medial to the inguinal ligament. This rules out an
inguinal hernia and a hydrocele since they are present in the scrotum, away from the
region described in the scenario. The non-painful swelling doesnt disappear on lying
down and is irreducible which rules out a saphenous vein varicocele. Femoral hernia is
the answer and in this case, is an irreducible hernia. This hernia can also get
incarcerated, which means it may or may not have tenderness. An incarcerated hernia
can be strangulated when it develops systemic symptoms and severe pain and
tenderness.

1583. A camel rider sustained a kick to the lateral side of his right leg just below the
knee caused by the camel stick. The site is slightly bruised and tender to touch. During
physical
examination, he is unable to either dorsiflex or evert the foot. There is loss of sensation
over the front and outer half of the leg and dorsum of the foot. If these observations are
the result of damage to a nerve bundle, which is the most likely nerve affected?
a. Lateral popliteal
b. Peroneal
c. Tibia
d. Sural
Key: Peroneal (B)
Reason: The fibular nerve or the peroneal nerve supplies the muscles and skin of the
area stated. Only possible answer.
1584. A 46yo woman presents with sudden episode of abdominal pain which started
about 2h ago. The pain is located in the epigastrium and radiates to her back. She has
vomited twice since the onset of attack. The pain is made worse by lying flat on her back
and she is more comfortable sitting up and bending forwards. She was informed of the
presence of gallstones in her gall bladder four weeks earlier when she reported pain in
the right hypochondrium. The oral
temp=39C, BP=120/80mmHg and the radial pulse=118/min. There is no jaundice but
there is
marked tenderness in the epigastrium both on deep and superficial palpation. Which is
the
most appropriate inv for the cause of the patients pain?
a. Plain abdominal X-ray
b. Serum Amylase
c. Serum bilirubin
d. Barium Swallow
Key: Serum Amylase (B)
Reason: Sudden onset abdominal pain, radiating to back, accompanied by vomiting,
made worse by lying flat, gallstones +ve, epigastric tenderness, tachycardia, fever all
point to Acute Pancreatitis. Serum Amylase is the most appropriate test, Lipase is more
sensitive and specific but takes time to rise. Serum bilirubin will not help us in any way,
neither will a needless Barium Swallow. An XRAY will waste useful time which can be
used to the patients benefit.
Discussion:
This is a case of acute pancreatitis, usually managed in the surgical ward. This is selfperpetuating pancreatic inflammation by enzyme mediated autodigestion. Edema and
fluid shifts causing hypovolemia as extracellular fluid is trapped in the gut, peritoneum
and retroperitoneum.
Causes: (GET SMASHED)
* Gallstones
* Ethanol
* Trauma
* Steroids
* Mumps
* Autoimmune (PAN)

* Scorpion venom
* Hypercalcemia, Hypothermia, Hyperlipidemia
* ERCP and emboli
* Drugs
* Pregnancy and Neoplasia also possible causes.
Symptoms:
* Gradual or sudden epigastric or central abdominal pain (severe, radiates to back,
sitting forward may relieve)
* Vomiting
Signs:
* May be mild in serious disease - tachycardia, fever, jaundice, shock, ileus, rigid
abdomen +/- local/general tenderness.
* Periumbilical bruising (Cullens sign), Flank bruising (Grey Turners sign) from blood
vessel autodigestion and retroperitoneal haemorrhage.
Tests:
* Raised serum amylase >1000 units/ml. Starts to fall after 24-48 hours.
* Serum lipase raised more sensitive and specific for pancreatitis.
* ABGs to monitor acid base status and oxygenation.
* Abdominal X-ray no psoas shadow, retroperitoneal fluid increased.
* Erect chest X-ray helps exclude perforation (gas under diaphragm).
* CT is a standard choice of imaging, MRI may be even better. USG if gallstones +
raised AST.
* ERCP if LFTs worsen. CRP >150 mg/L at 36 hours after admission is a predictor of
severe pancreatitis.
Management:
* Severity assessment is essential, modified Glasgow criteria pg. 639.
* NPO, pass NG tube decreased pancreatic stimulation.
* IV Line, lots of 0.9% saline to counter third place sequestration until vitals are stable
and urine flow is >30mL/hour.
* Catheterize with Foleys.
* Analgesia with Pethidine 75-100mg/4 hours IM OR Morphine (may cause Oddi
sphincter to contract more but it is a better analgesic and not contraindicated.
* Vital monitoring.
* Daily CBC, urea & electrolytes, calcium, glucose, amylase, ABGs.
* If worsening, ITU. Give O2 if decreased O2-sats.
* In suspected abcess formation or pancreatic necrosis on CT, consider TPN +/laparotomy and debridement/necrosectomy.
* Antibiotics may help in severe disease Imipenem in more than 30% necrosis. There
is no consensus on prophylactic use if necrosis is present.
* ERCP + gallstone removal may be needed if there is progressive jaundice. Repeat
imaging, CT is performed in order to monitor progress.
Complications:
EARLY
* Shock
* ARDS
* Renal failure
* DIC
* Sepsis
* Hypocalcemia
* Hyperglycemia (transient 5% may need insulin)
LATE

* Pancreatic necrosis and pseudocyst


* Abcesses
* Bleeding from elastase eroding a major vessel like Splenic artery
* Thrombosis leading to bowel necrosis
* Fistula
* Recurrent edematous pancreatitis
1585. A 75yo Japanese woman reports repeated episodes of vomiting of undigested
food mixed with blood. She has lost 5 kgs in weight over the last one month. Clinical
exam: shows a frail woman with mild conjunctival pallor. Exam: non-tender slightly
mobile mass in the epigastric region. Which is the most likely dx?
a. Colon cancer
b. Gastric cancer
c. Gall bladder cancer
d. Oesophageal cancer
Key: Gastric CA (B)
Reason: Undigested food vomiting with blood rules out Colon CA, Gallbladder CA.
Oesophageal CA would have dysphagia as a symptom. Gastric CA is the most likely
diagnosis.
Discussion: Associated with pernicious anaemia, blood group A, H. pylori, atrophic
gastritis, adenomatous polyps, smoking, nitrosamine exposure, diet (high nitrates, high
salt, pickling, low Vit. C).
Borrmanns classification:
i. Polypoid.
ii. Excavating.
iii. Ulcerating and raised.
iv. Diffusely infiltrative.
Presents with non specific symptoms like dyspepsia (>1 month and age>50 warrants
investigation), weight loss, vomiting, dysphagia, anaemia.
Signs suggesting incurable disease = epigastric mass, hepatomegaly, jaundice, ascites,
Virchows node/Troisiers sign, Acanthosis nigricans.
Spread is local, lymphatic, blood-borne, transcoelomic eg. Krukenberg tumour.
Tests: Gastroscopy + multiple ulcer edge biopsies. Endoscopic ultrasound to evaluate
depth of invasion. CT/MRI to stage. Staging laparoscopy for locally advanced tumours.
Cytology of peritoneal washing can identify peritoneal mets.
Treat by Surgery Bilroth I & II, Roux-en-Y loop gastrectomy + combination
chemotherapy.
1586. A 45yo man, known to be chronically addicted to alcohol, presents in the ED and
reports two episodes of vomiting fresh bright red blood in the previous 6h. He estimated
the volume blood vomited at each bout to be more than 500mls. Clinical exam: the radial
pulse=120/min,
BP=90/60mmHg. There is no mass or tenderness in the epigastrium. The liver is
palpable for 3
cm below the costal margin and not tender. The patient is not jaundiced. The physician
resuscitates the patient with oxygen by face mask, rapid infusion of intravenous normal
saline
while he requests for haemoglobin level and whole blood for transfusion. Which is next
appropriate step in management?

a. Barium Swallow
b. Exploratory laparotomy
c. CT scan of the abdomen
d. Upper gastrointestinal endoscopy
Key: Upper GI Endoscopy (D)
Reason: The patient presented with emergency Upper GI Bleed, he is an alcoholic who
vomited fresh red blood with a palpable liver. Probable variceal bleed secondary to CLD
brought on by Chronic Alcoholism. Endoscopy should be performed on an urgent basis
after resuscitation. Should be performed within 4 hours of resus or when bleeding is
ongoing within 24h according to OHCM pg. 254.
1587. A 42yo woman reports to the surgeon that she is worried about a lump that she
feels the right breast. The surgeon observes a 2 cm by 3 cm mass in the right lower
quadrant of the breast. There are no associated skin changes and the mass has limited
mobility. There is no discharge from the nipple. There is no axillary lymph node
enlargement. Examination of the left breast and axilla was completely normal. A
mammogram report suggests the presence of
microcalcifications. Which is the most appropriate next step in the management of this
pt?
a. Observation for one year and repeat the mammography
b. A needle-guided biopsy of the breast
c. Excision biopsy of the breast
d. Partial mastectomy
Key: Needle Guided Biopsy (B)
Reason: Biopsy all breast lumps!!! Looks harmless, doesnt have any danger signs in
the history or examination. Observation is advisable but only with a biopsy, not without it.
Excision and mastectomy are radical choices for an undiagnosed lump.
1588. A 45yo man presents with a mass on the right side of the face. The mass was first
observed three months ago but has recently become visibly larger. He feels pain over
the mass and is unable to blow a whistle. Clinical examination shows that the mass is
likely to be the parotid gland. An oral examination shows a foul smelling discharge from
the duct of the gland and gentle probing shows that it is stenosed at the meatus. Which
of the following features suggests that the mass might be malignant?
a. Presence of pain
b. Recent enlargement
c. Facial nerve palsy
d. Stenosed duct meatus
Key: Facial nerve palsy (C)
Reason: Out of the options given, only facial nerve palsy is a sign of malignant invasion
showing invasion. The rest can be explained by benign ailments.
1589. A 6yo boy presents with jaundice following treatment with sulfathiazole.
Investigations
suggest that the jaundice is due to haemolysis caused by G6DP deficiency. Which is
true
regarding etiology of G6DP deficiency?
a. Inherited as autosomal dominant condition
b. Inherited as sex-linked dominant condition
c. Inherited as sex-linked recessive condition

d. Results from auto-antibodies to red cell antigens


Key: X-Linked recessive (C)
Reason: G6PD is inherited as an X-Linked recessive condition, which means it can
occur only in homozygotes. Usually happens in males.
Discussion: The enzyme glucose-6-phosphate dehydrogenase (G6PD) is one of the
enzymes of the pentose phosphate pathway. This pathway is involved in keeping an
adequate amount of the coenzyme nicotinamide adenine dinucleotide phosphate
(NADPH) in cells.
Classes of G6PD deficiency enzyme variants:
* Severe (I) - chronic non-spherocytic haemolytic anaemia.
* Severe (II) - less than 10% of normal enzyme activity.
* Moderate (III) - 10-60% of normal enzyme activity.
* Mild to none (IV) - 60-150% of normal enzyme activity.
* None (V) - greater than 150% of normal enzyme activity.
History:
* Depends upon the severity of the enzyme deficiency.
* Most are asymptomatic.
* May be a history of neonatal jaundice, severe enough to require exchange transfusion.
* History of drug-induced haemolysis.
* Gallstones are common.
Examination:
* Most often, examination is unremarkable.
* Pallor of anaemia.
* During a crisis jaundice occurs.
* Back or abdominal pain (usually occurs when >50% haemolysis occurs).
* Splenomegaly may occur.
Investigations:
* FBC - anaemia.
* Macrocytosis - due to reduced folic acid which is required for erythropoiesis.
* Reticulocyte count - raised; gives indication of the bone marrow activity (bone marrow
sampling thus not needed).
* Blood film - acute haemolysis from G6PD deficiency can produce Heinz bodies, which
are denatured haemoglobin and bite cells (cells with Heinz bodies that pass through the
spleen have part of the membrane removed).
* Haemolysis - reduced levels of haptoglobin and elevated levels of bilirubin;
haemoglobinuria.
* Direct antiglobulin test - to look for other causes of haemolysis; should be negative in
G6PD deficiency.
* Renal function - to ensure no renal failure as a precipitant.
* LFTs - to exclude other causes of raised bilirubin.
* G6PD enzyme activity - is the definitive test (as opposed to the amount of G6PD
protein).
* Performing assays for G6PD during haemolysis and reticulocytosis may affect levels
and not reflect baseline values.
* Ultrasound examination of the abdomen may reveal splenomegaly and gallstones.
Management:
* Avoidance of substances that may precipitate haemolysis is essential. Usually no
further management is required, although if haemolysis is marked there may be benefit
from folate supplementation.
MANAGEMENT OF ACUTE HAEMOLYSIS
* Seek specialised advice.

* Blood transfusions may be needed.


* Dialysis may be required in acute kidney injury.
* Infants - more susceptible to neonatal jaundice, especially if premature, and exchange
transfusion may be required.
MANAGEMENT OF CHRONIC HAEMOLYSIS OR STABLE DISEASE
* Splenectomy may help.
* Supplementation with folic acid.
* Avoidance of precipitating drugs, and broad beans (usually favism occurs in the
Mediterranean variety of the disease).
* Avoid naphthalene - found in mothballs.
1590. A 5yo previously healthy child has a 1-day history of severe pain in the throat,
breathing
difficulties and fever. On examination you find an anxious, septic-looking child with
drooling of
saliva and stridor. Which is the most appropriate initial management?
a. Intubation under general anaesthesia
b. Insertion of nasogastric tube
c. Fluid resuscitation and antibiotics IV
d. Anteroposterior & lateral neck x-ray
Key: Intubation under GA (A)
Reason: This is Acute Epiglottitis, a respiratory emergency caused by H. influenza
causing severe edema of the throat and a huge inflamed cherry red epiglottis. Throat
exam is severely contraindicated in this case and the intervention of choice is intubation
with an ETT under GA. NG tube makes no sense, fluid resuscitation and antibiotics have
a definite role along with
radiographs, but all those things come after the initial management. Antibiotic
management is Penicillin and Ceftriaxone 2g IV x 12 hourly.
Discussion:
Presentation:
Many of the signs and symptoms associated with acute epiglottitis are common, and
may occur in many less serious disorders. Patients with epiglottitis may present with any
of the following, and the symptoms may evolve very quickly over a period of a few hours.
The most common symptoms:
* Sore throat.
* Odynophagia (painful swallowing).
* Drooling (inability to swallow secretions).
* Fever.
* Anterior neck tenderness over the hyoid bone.
Other features:
* High temperature.
* Tachycardia.
* Ear pain.
* Cervical lymphadenopathy.
* The 'tripod sign' - the patient leans forward on outstretched arms to move inflamed
structures forward, thereby easing the upper airway obstruction.
With more severe epiglottitis:
* Dyspnoea.
* Dysphagia.
* Dysphonia.

* Stridor (late finding - indicates airway obstruction).


* Respiratory distress.
Differential diagnosis:
The differential diagnosis will depend on the presenting symptoms and age of the
patient; however, generally included are: * Pharyngitis * Laryngitis * Inhaled foreign body
* Croup * Retropharyngeal abscess
Investigations:
Patients who are suspected of having acute epiglottitis should not have their throat
examined with the aid of a tongue depressor, due to the risk of laryngeal obstruction;
rather, they should be urgently referred for laryngoscopy.
* Fibre-optic laryngoscopy remains the 'gold standard' for diagnosing epiglottitis,
as the epiglottis can be seen directly. Laryngoscopy in these patients should only be
performed in areas such as operating theatres which are prepared for intubation or
tracheostomy in the event of upper airway obstruction.
* Lateral neck X-ray may be useful if laryngoscopy is not possible. Soft-tissue radiograph
of the neck may show the 'thumbprint sign'.
* Throat swabs may be taken when the airway is secure, or when
intubation/tracheostomy facilities are at hand. Streptococci are becoming the major
pathogens in acute epiglottitis now.
* Blood cultures may be taken if the patient is systemically unwell.
* CT or MRI scans may be performed if abscess formation is suspected.
Management:
* The incidence of acute epiglottitis is falling in children but rising in adults. Initial
presentation may resemble a viral sore throat, so a high index of suspicion is needed.
Emergency referral is required if signs of airway obstruction are present (stridor).
Deterioration in symptoms may be rapid. Management is usually conservative but
intubation is occasionally needed (if >50% airway obstruction occurs).
* Surgical tracheostomy may be required in patients with severe airway obstruction in
whom intubation has not been possible.
* Abscess formation is being increasingly seen as epiglottitis cases in adults increase.
Drainage may be required in some patients.
Complications:
* Epiglottitis, if not adequately treated, may occasionally result in: * Abscess formation *
Meningitis * Septicaemia * Pneumothorax * Pneumo-mediastinitis (very rare)
1591. A 6yo boy has been noticed to have problems with co-ordinating his voluntary
movements over the last two years. He has a waddling gait and needs to support himself
on his hands when rising from the floor. He has larger calves than other boys but he
runs more slowly. Which is the most likely dx?
a. Myotonia
b. Myasthenia gravis
c. Duchenne muscular dystrophy
d. Muscular atrophy
Key: Duchenne Muscular Dystrophy (C)
Reason: Presentation by 6 years, difficulty co-ordinating movements, waddling gait,
Positive Gowers sign, pseudohypertrophy of calf muscles all point to Duchenne
Muscular Dystrophy as the cause in this boy. Diagnose by muscle biopsy and DNA
testing. Myasthenia Gravis would have descending paralysis, Myotonia (Myo from
Greek; muscle, and Tonus from Latin; tension) is a symptom of a small handful of certain
neuromuscular disorders characterized by delayed relaxation (prolonged contraction) of

the skeletal muscles after voluntary contraction or electrical stimulation and Muscular
atrophy can never have hypertrophied calves.
1592. A previously healthy, 10 month female child presents to your clinic with a 1-day
history of high fever, runny nose and conjunctivitis. The child looks unwell and is irritable.
Exam: child's
oropharynx shows that it is inflamed and there are small white spots on the oral mucosa.
Which is the most likely dx?
a. Kawasaki disease
b. Parvovirus infection
c. Herpes zoster
d. Measles
Key: Measles (D)
Reason: These symptoms along with the pathognomonic Koplik spots on the oral
mucosa point only to measles. It cannot be herpes zoster because there isnt a lot of
pain and dermatomal distribution, parvovirus would cause red cell crisis or aplastic crisis
and would not present this way. Kawasaki disease would have rash, a strawberry tongue
and limb involvement like arthralgias and swelling of hands or feet. Also predisposes to
coronary arteritis, check for aneurysms by ECHO!
Discussion:
MEASLES:
Rash for at least three days.
Fever for at least one day and at least one of the following:Cough
Coryza
Conjunctivitis
Prodrome: This lasts 2-4 days with fever, cough, runny nose, mild conjunctivitis and
diarrhoea.
Koplik's spots are pathognomonic and appear on the buccal mucosa - opposite the
second molar teeth - as small, red spots, each with a bluish-white speck (sometimes
compared with a grain of rice) in the centre. They occur in 60-70% of patients during the
prodrome and for up to 2-3 days after the rash disappears.
Rash (morbilliform = measles-like):This is first seen on the forehead and neck and
spreads, involving the trunk and finally the limbs, over 3-4 days. It may become
confluent in some areas.
The rash then fades after 3-4 days in the order of its appearance.
It leaves behind a brownish discoloration, sometimes accompanied by fine
desquamation.
Often, there is high fever (may be >40C) and a non-productive cough, with the patient
being clearly ill.
Also, swelling around the eyes and photophobia may be present.
Diagnosed clinically!!
Management:
Uncomplicated measles is usually self-limiting and treatment is mainly symptomatic,
with paracetamol or ibuprofen and with plenty of fluids. Patients should remain at home
to limit disease spread.
Monitor patients carefully for signs of complications and consider hospitalisation if these
appear.
1593. A 3d term, breast-fed infant is brought by the mother who reports that the child
has not been active and not feeding well. She also notices jaundice, which was not

present at birth and is increasing. Exam: the temp=35.4C, and the liver is palpable 2 cm
below the costal margin.
Which is the most likely dx?
a. Rhesus isoimmunisation
b. Inadequate breast milk
c. Congenital biliary tract obstruction.
d. Sepsis
Key: Sepsis (D)
Reason: Biliary atresia would have conjugated bilirubin and pale stools clinically,
breastfeeding jaundice, which manifests in the first 3 days of life is caused by insufficient
production or intake of breast milk. Rhesus disease doesnt make sense, and the
hypothermia would point towards Sepsis as being the likely cause of jaundice and other
symptoms. Liver is also normally palpable in children of this age so take that symptom
with a pinch of salt.
1594. A 65yo woman with DM, HTN and normal kidney function underwent a total right
hip
replacement. She had massive haemorrhage during the operation and was given 8 units
of
packed RBC. The blood pressure dropped to 60/40 mmHg for about two hours before it
was
corrected with blood transfusion. Two days after the surgery the serum creatinine level
rose to
4.2 mg/dl (normal <1.5 mg/dl), BUN was 50 mg/dl (normal 10-20 mg/dl) and potassium
5.0
mmol/L (normal 3.5-5.0 mmol/l). There were brown granular casts in the urine sediment.
Which
is the most likely cause of this complication?
a. Diabetic nephropathy
b. Malignant hypertension
c. Acute tubular necrosis
d. Interstitial nephritis
Key: Acute Tubular Necrosis (C)
Reason: This woman recently underwent major surgery and even though the blood loss
was corrected with blood transfusion of 8 units of packed cells, the blood pressure
dropped to 60/40 mmHg before being corrected which could possibly lead to Acute
Kidney Injury or ATN in this case. The rising Creatinine, Potassium and BUN all point to
the kidney being unable to undertake its normal functions. Diabetic Nephropathy would
have a setting other than post-op patients, malignant hypertension doesnt make sense
because there is no HTN in this case, interstitial nephritis would mention history of
certain drugs that cause this.
1595. A 78yo pt is diagnosed with metastatic lung cancer; there is no cure for his
condition. His son tells the physician that in the case of a diagnosis of cancer, the
physician must not tell his father. He wishes that his father does not suffer any
psychological distress caused by the knowledge of a terminal diagnosis. Which one of
the following ethical principles supports the sons request?
a. Patient autonomy
b. Beneficence
c. Justice

d. Non-maleficence
Key: Non-Maleficence (D)
Reason: Justice and Autonomy would involve the patient knowing about the diagnosis
so they are automatically ruled out, Beneficence entails a paternalistic approach in which
the doctor does what is best for the patient, again regarding him as aware of the
diagnosis. Non-maleficence is the appropriate choice considering it basically means to
do no harm on the doctor's part, by listening to the wishes of the attendant.
1596. A 23yo single male was brought to Emergency exhausted and frightened. His
father tells you that his son, who was previously healthy, had, for no apparent reason, a
sudden attack of fear, dizziness, sweating, palpitations and the feeling that his heart is
going to stop beating. The
symptoms started to decrease gradually after about 10 minutes. Which is the most likely
dx?
a. Panic attack
b. Delirious state
c. Alcohol withdrawal phenomena
d. Social phobia
Key: Panic attack (A)
Reason: The short term nature of the symptoms point to panic attack. We have no
alcohol history to regard withdrawal, there is no public setting to regard social phobia
and a delirious state would present with symptoms of mania and would almost never
need intervention, wouldnt resolve on its own.
1597. A 30yo woman, G2P1, at 37 weeks gestation mentions that her 3-year-old son
has just
developed chickenpox. She is not certain whether she has had the disease herself.
Which is the
next step in management?
a. Administration of varicella-zoster immune globulin IM
b. Measurement of varicella IgM level
c. Acyclovir tablets orally
d. Measurement of varicella IgG level
Key: Measurement of Varicella IgG level (D)
Reason: If you don't know the immunity status you have to measure varicella IgG level. If
+ve nothing to do but reassure the patient. If negative VZIG to be given.
1598. A 24yo primigravida presents to the ED with a history of 8-week amenorrhoea
followed by
heavy vaginal bleeding and severe, crampy abdominal pain. Exam: HR=110/min and
BP=120/80mmHg. The uterus is bulky. The cervix is dilated and there is active bleeding
from the cervical os, but no tissue has been expelled. Which of the following is the most
likely dx?
a. Inevitable abortion
b. Threatened Abortion
c. Incomplete abortion
d. Missed Abortion
Key: Inevitable abortion (A)

Reason: The cervix is dilated which rules out missed abortion completely. Threatened
abortion would only have bleeding. Incomplete abortion will have some expulsion of
tissue, this is clearly a case of inevitable abortion.
Discussion:
Classification of miscarriage is as follows:
Threatened miscarriage: mild symptoms of bleeding. Usually little or no pain. The
cervical os is closed.
Inevitable miscarriage: usually presents with heavy bleeding with clots and pain. The
cervical os is open. The pregnancy will not continue and will proceed to incomplete or
complete miscarriage.
Incomplete miscarriage: this occurs when the products of conception are partially
expelled. Many incomplete miscarriages can be unrecognised missed miscarriages.
Missed miscarriage: the fetus is dead but retained. The uterus is small for dates. A
pregnancy test can remain positive for several days. It presents with a history of
threatened miscarriage and persistent, dirty brown discharge. Early pregnancy
symptoms may have decreased or gone.
Habitual or recurrent miscarriage: three or more consecutive miscarriages.
Risk factors:
Age: it is more frequent in women aged >30 years and even more common in those
aged >35 years (due to an increased risk of random chromosomal abnormalities).
Incidence increases with the number of births: 6% in the first and second pregnancies
and 16% in further pregnancies.
Cigarette smoking of >14 per day doubles the risk over non-smokers.
Excess alcohol. Even low amounts - four units a week of alcohol consumption during
early pregnancy - have been shown to increase the risk of spontaneous abortion
substantially.[2]
Illicit drug use.
Uterine surgery or abnormalities - eg, incompetent cervix.
Connective tissue disorders (systemic lupus erythematosus, antiphospholipid antibodies
- lupus anticoagulant/anticardiolipin antibody).
Uncontrolled diabetes mellitus
Management:
Anti-D rhesus prophylaxis (250 IU) should be offered to all rhesus-negative women who
have a surgical procedure to manage a miscarriage.
However, anti-D rhesus prophylaxis does not have to be given to those women who:
* Receive solely medical management for an ectopic pregnancy or miscarriage.
* Have a threatened miscarriage.
* Have a complete miscarriage.
* Have a pregnancy of unknown location
Conservative management:
If a scan at the EPAU confirms a first-trimester miscarriage, expectant management
(waiting to see if the miscarriage will resolve naturally without intervention) for 7-14 days
can be offered as the initial management strategy.
If bleeding and pain have not started or bleeding and pain are persisting and/or
increasing then these women should have a repeat ultrasound examination performed.
Alternative management may be offered to those whose miscarriage is incomplete or
has not started.
Those women who have resolution of bleeding and pain should perform a pregnancy
test after three weeks. If this is still positive, they need to be reviewed and considered for
either medical or surgical management.

Women should be counselled so they are fully aware of what to expect. In most cases,
resorption of fetal tissue occurs without much bleeding. However, loss of fetal tissue
vaginally can be associated with heavy bleeding and pain and the patient may prefer to
opt for medical or surgical management rather than cope with this.
Medical management:
Women may opt for medical management at the initial stage or following expectant
treatment.
Medical management can cause more pain and bleeding than surgical management but
patients who opt for this approach cite 'being in control' and avoiding general
anaesthesia as the main reasons for their choice.
Vaginal misoprostol should be offered for the medical treatment of missed or incomplete
miscarriage.
Oral misoprostol is an acceptable alternative if this is the woman's preference.
Mifepristone should no longer be given as a treatment for missed or incomplete
miscarriage.
Women should be advised that bleeding can continue for up to three weeks.
Women should perform a pregnancy test three weeks after receiving medical
management, unless they have worsening symptoms. If these occur, they should be
reviewed to ensure there is no molar or ectopic pregnancy.
Surgical management
Clinical indications for offering surgical evacuation include persistent excessive
bleeding, haemodynamic instability, evidence of infected retained tissue and suspected
gestational trophoblastic disease.
Where clinically appropriate, women should be offered a choice of:Manual vacuum
aspiration under local anaesthetic in an outpatient or clinic setting.
Surgical management in a theatre, under general anaesthetic.
Vacuum aspiration is safe, quick to perform and less painful than sharp curettage
Serious complications of surgery include perforation, cervical tears, intra-abdominal
trauma, intrauterine adhesions and haemorrhage.
Tissue obtained at the time of miscarriage should be examined histologically to confirm
pregnancy and to exclude ectopic pregnancy or gestational trophoblastic disease.
Complications
Expectant management has been shown to lead to a higher risk of incomplete
miscarriage, need for unplanned (or additional) surgical emptying of the uterus, bleeding
and need for transfusion
However, risk of infection and psychological outcomes are similar for expectant and
surgical management.
After complete miscarriage, bleeding normally ceases within 10 days. If part of the
placenta remains, bleeding may continue with cramps. If this occurs then a repeat
ultrasound should be undertaken and surgery is often required.
1599. A 46yo woman comes for a routine gynaecological visit. On pelvic examination, a
1-cm red, granular lesion is noted on the posterior cervical lip, which is firm and bleeds
on contact. Which is the next best step for establishing a dx?
a. Cervical cytological smear
b. Punch biopsy
c. Transvaginal ultrasound
d. Colposcopy
Key: Punch biopsy (B)

Reason: The most appropriate or next best step for establishing a diagnosis would only
be punch biopsy in this case. Smear is irrelevant because the lesion has already been
observed, most likely a polyp. TVU also not likely because it wouldnt help at all.
Colposcopy is unneeded because a biopsy can easily be obtained with a Cuscos
speculum.
Discussion:
Polyps are the most common benign neoplasms of the cervix (found in 4% of the
gynaecological population). They may be endocervical or cervical:
Endocervical polyps are most usually found in the fourth to sixth decade of life. They
are cherry red lesions which may be single or multiple and may appear as a
pedunculated lesion on a stalk of varying length.
Cervical polyps are equally benign and tend to occur as single, smooth grey-white
lesions that bleed easily if touched.
Polyps tend to be asymptomatic but may also present with abnormal bleeding (such as
during intercourse, heavy menstrual periods, between menstrual periods or after the
menopause). Occasionally, they may grow big enough to obstruct the external os and so
cause infertility. Malignancy is rare.
Polyps may be removed but should all be sent to histology. If the woman is
asymptomatic, this can simply be done by twisting them off. Haemorrhage may also
occur and require cautery for haemostasis. Surgical dilatation and curettage,
electrosurgical excision or hysteroscopic polypectomy may be performed for more
persistent lesions. Occasionally, the appearance of the healed cervix following a cone
biopsy can mimic a large polyp.
All patients with suspicious lesions or who are symptomatic should be referred to a
specialist clinic and the lesions sent to histology.
1600. A 31yo woman, G5P4, who has amenorrhoea for 12 weeks and a positive
pregnancy test
presents to the ED with vaginal bleeding. Symphyseal-fundal height measurement
corresponds
to 22 weeks gestation. Ultrasound examination reveals bilateral cystic masses. No fetal
parts are seen during the examination. The cervix is closed. Which is the most likely dx?
a. Tubal pregnancy
b. Endometriosis
c. Hydatidiform mole
d. Threatened abortion
Key: Hydatidiform Mole (C)
Reason: The inconcordant amenorrhea and SFH point towards point towards a molar
pregnancy as the likely cause with the bilateral cystic masses. No fetal parts are seen
which rule out threatened abortion. Endometriosis would not present with amenorrhea
and a tubal pregnancy wouldnt have the SFH seen in this patient.
Discussion:
Complete molar pregnancies, all the genetic material comes from the father. An empty
oocyte lacking maternal genes is fertilised. Most commonly (75-80%) this arises from a
single sperm duplicating within an empty ovum. Less often an empty ovum is fertilised
by two sperm. There is no fetal tissue.
Partial molar pregnancies, the trophoblast cells have three sets of chromosomes
(triploid). Two sperm are believed to fertilise the ovum at the same time, leading to one
set of maternal and two sets of paternal chromosomes. Around 10% of partial moles are

tetraploid or mosaic in nature. There is usually evidence of fetal tissue or fetal blood cells
in a partial molar pregnancy. An embryo may be present at the start.
Tests:
Urine and blood levels of hCG. A urine pregnancy test should be performed in all cases
of persistent or irregular vaginal bleeding after a pregnancy event. Levels of hCG may
be of value in diagnosing molar pregnancies but are far more important in disease
follow-up.
Histology. Definitive diagnosis is made by histological examination of the products of
conception.
Ultrasound: in the first trimester may not be reliable. The typical 'snowstorm'
appearance occurs mainly in the second trimester, showing a heterogeneous mass with
no fetal development, and theca-lutein ovarian cysts.
Because of the lack of diagnostic reliability of ultrasound, products of conception from
all non-viable pregnancies should undergo histological examination in order not to miss
the diagnosis, and the chance of monitoring to prevent complications.
Management of hydatidiform moles:
Suction curettage is the method of choice of evacuation for complete molar
pregnancies.
Suction curettage is the method of choice of evacuation for partial molar pregnancies
except when the size of the fetal parts deters the use of suction curettage and then
medical evacuation can be used.
A urinary pregnancy test should be performed three weeks after medical management
of failed pregnancy if products of conception are not sent for histological examination.
Anti-D prophylaxis is required following evacuation of a PHM.
Follow up:
Two-weekly serum and urine samples until hCG concentrations are normal.
After hCG levels return to normal, monthly urine hCG testing. This continues for six
months from evacuation if levels have normalised within eight weeks; if not, monitoring
continues for six months from when levels became normal
Future pregnancy:
Women being monitored after molar pregnancy should be advised not to conceive until
their hCG levels have been normal for six months.
Women with GTD should be advised to use barrier methods of contraception until hCG
levels revert to normal. Once hCG levels have normalised, the combined oral
contraceptive pill (COCP) may be used.
1601. A married 25yo woman presents with 6h hx of abdominal pain located in the LIF.
The pain is persistent, of increasing intensity and not radiating first experienced while
she was lying down. She feels giddy when she tries to stand erect. The last menstrual
period was 6 weeks ago. The radial pulse=130/min and BP=80/40 mmHg. Pelvic US
shows free intra-peritoneal fluid. What is the most appropriate next step in
management?
a. Immediate laparoscopy.
b. Immediate laparotomy.
c. Pregnancy test (urine or serum).
d. Observation for 24 hours in the ICU
Key: Pregnancy test (urine or serum) (C)
Reason: This is a classic case of ruptured Ectopic pregnancy and the best step would
pregnancy test to establish the diagnosis. The immediate next step if the patient is

hemodynamically unstable would be an immediate laparoscopy. Observation wouldnt


help anyone and a laparotomy would be considered in third-world countries.
Discussion:
The majority of ectopic pregnancies occur in the ampullary or isthmic portions of the
Fallopian tubes.
Risk factors:
Fertility treatments and intrauterine contraceptive devices (IUCDs) are the most
important associated risk factors.
Pelvic inflammatory disease may cause complete tubal occlusion or delay the transport
of the embryo so that implantation occurs in the tube. Adhesions from infection and
inflammation from endometriosis may play a part.
Ectopic pregnancy has been reported in tubes that have been divided in a sterilisation
operation and where they have been reconstructed to reverse one.
Ectopic pregnancy can occur in the treatment of infertility.
Right-sided tubal pregnancy is more common than left-sided. This is thought to be from
spread of infection from appendicitis.
Presentation:
Symptoms and signs of ectopic pregnancy can resemble those of other more common
conditions, including urinary tract infections and gastrointestinal conditions.
The most common symptoms are:
* Abdominal pain.
* Pelvic pain.
* Amenorrhoea or missed period.
* Vaginal bleeding (with or without clots).
* Other symptoms may include:
* Dizziness, fainting or syncope.
* Breast tenderness.
* Shoulder tip pain.
* Urinary symptoms.
* Passage of tissue.
* Rectal pain or pressure on defecation.
There may be a history of a previous ectopic pregnancy. After one ectopic pregnancy
the chance of another in the other tube is much increased.
If the ectopic pregnancy has ruptured, bleeding is profuse and there may be features of
hypovolaemic shock, including feeling dizzy on standing. Most bleeding will be into the
pelvis and so vaginal bleeding may be minimal and misleading.
Diarrhoea and vomiting are possible, atypical clinical features of ectopic pregnancy.
Tests:
The most accurate method to detect a tubal pregnancy is transvaginal ultrasound.
* This can identify the location of the pregnancy and also whether there is a fetal pole
and heartbeat.
* Human chorionic gonadotrophin (hCG) levels are performed in women with pregnancy
of unknown location who are clinically stable.
* hCG levels are taken 48 hours apart. If there is a change in concentration between
50% decline and 63% rise inclusive over 48 hours then the woman should be referred
for clinical review in an early pregnancy assessment service within 24 hours
Management:
* Admit as an emergency if the diagnosis of ectopic pregnancy is considered a
possibility. A bedside pregnancy test should be performed on all women of childbearing
age presenting with lower abdominal pain where pregnancy is even the remotest
possibility.

* Anti-D rhesus prophylaxis should be given (at a dose of 250 IU) to all rhesus negative
women who have a surgical procedure to manage an ectopic pregnancy.
* Conservative management may be appropriate if the levels of hCG are falling and the
patient is clinically well. Repeat hCG levels are performed in these cases.
1602. A 40yo man has fallen off a roof. He is shocked and has chest pain. There is a
delay between the radial and femoral pulse. His CXR=widening of the mediastinum.
What is the single most likely dx?
a. Cardiac tamponade
b. Diaphragmatic rupture
c. Fx ribs
d. Tension pneumothorax
e. Traumatic rupture of aorta
Key: Traumatic rupture of the aorta (E)
Reason: The wide mediastinum and the radio-femoral delay make Traumatic rupture of
the Arch of the Aorta the most likely diagnosis in this case. Tension Pneumothorax will
not present this way and a rib fracture would be less severe than the patient is currently.
Diaphragmatic rupture would have bowel sounds in the chest and cardiac tamponade
would have Becks triad Muffled heart sounds, distended neck veins and decreased
blood pressure.
1603. A 36yo woman presents with swelling in the groin. Exam: swelling is diffuse and
soft and lies below the inguinal ligament. It empties with minimal pressure and refills with
release. There is a cough impulse and it disappears on lying down. On the calf of the
same leg there are varicosities on the medial aspect. What is the most likely dx?
a. Varicose vein
b. Varicocele
c. Saphena varix
d. Femoral hernia
e. Inguinal hernia
Key: Saphena Varix (C)
Reason: The emptying on lying down and the coexistence of varicose veins make a
saphena varix the most likely answer to this question. It is a dilatation of the
saphenofemoral junction due to incompetent valves when the saphenous vein drains
into the femoral vein. Femoral and Inguinal hernia not possible because the swelling is
diffuse and empties with minimal pressure. Varicose veins are present but they are not
the choice because the saphena varix is immediately below the inguinal ligament.
1604. A man presents with a swelling above the groin crease in the abdomen. He has
not had any med problems of note. What is the most probable dx?
a. Inguinal hernia
b. Spigelian hernia
c. Testicular tumor
d. Epidydimal cyst
e. Irreducible hernia
Key: Spigelian Hernia (B) Reason: A Spigelian hernia (or lateral ventral hernia) is a
hernia through the spigelian fascia, which is the aponeurotic layer between the rectus
abdominis muscle medially, and the semilunar line laterally. There is a common

misconception that they protrude below the arcuate line owing to deficiency of the
posterior rectus sheath at that level, but in fact the defect is almost always above the
arcuate line. These are generally interparietal hernias, meaning that they do not lie
below the subcutaneous fat but penetrate between the muscles of the abdominal wall;
therefore, there is often no notable swelling. All the other options lie below the groin
crease i.e. the inguinal ligament.
Discussion:
Spigelian: this is a hernia through the linea semilunaris muscle. Initially this causes
localised pain exacerbated by straining and coughing, but the pain may become less
localised and more an ache with time. Bulge can often be seen in the lower abdomen
with the patient erect and straining. This can be reduced by pressure with a 'gurgling'
noise and then the hernia orifice can often be felt. However, the defect may not be
palpable or a bulge may be found distant from the site. This needs prompt repair.
1605. A 70yo man presents with acutely painful, pale paralysed and pulseless left leg.
He is noted to have a-fib. What is the most probable dx?
a. Intermittent claudication
b. Cardiovascular syphilis
c. Buergers disease
d. Chronic limb ischemia
e. Acute limb ischemia
Key: Acute limb ischemia (E)
Reason: This is acute presentation of limb ischemia because the atrial fibrillation has
thrown a clot which has blocked arteries in the leg leading to this acute picture. Chronic
ischemia would present with amputation or gangrene plus lipodermatosclerosis of limbs,
intermittent claudication would present with pain on walking and no pain on rest,
Buergers disease would need a smoking history and syphilis of the CVS would have
aneurysms of the large arteries rather than small vessel involvement leading to limb
symptoms.
Acute limb ischaemia is most often due to either acute thrombotic occlusion of a
previously partially occluded, thrombosed arterial segment, or to embolus from a distant
site. Without surgical revascularisation, complete acute ischaemia leads to extensive
tissue necrosis within six hours.
Presentation: The affected part becomes pale, pulseless, painful, paralysed,
paraesthetic and 'perishing with cold' ('the 6 Ps')
Investigations:
Hand-held Doppler ultrasound scan may help demonstrate any residual arterial flow.
Blood tests
FBC (ischaemia is aggravated by anaemia).
ESR (inflammatory disease - eg, giant cell arteritis, other connective tissue disorders).
Glucose (diabetes).
Lipids.
Thrombophilia screen.
If diagnosis is in doubt, perform urgent arteriography.
Investigations to identify the source of embolus:
ECG.
Echocardiogram.
Aortic ultrasound.
Popliteal and femoral artery ultrasound.
Management

Urgent admission - this is an emergency and often requires urgent open surgery or
angioplasty. Objective sensory loss requires urgent treatment. Heparinization is needed
immediately (this may double the limb salvage rate), and provide analgesia.
The limb must be checked for evidence of compartment syndrome and, if necessary, a
fasciotomy should be performed.
If the occlusion is embolic, the options are surgical embolectomy (Fogarty balloon
embolectomy catheter) or local intra-arterial thrombolysis:
If embolectomy with a Fogarty catheter fails, an on-table angiogram is performed and
bypass graft or intraoperative thrombolysis considered. Routine intraoperative
angiography for arterial thromboembolectomy has been shown to be beneficial.
After successful embolectomy, anticoagulation with heparin is needed to prevent
recurrence. Many surgeons postpone heparin for six hours after surgery to reduce the
risk of a haematoma.
If the occlusion is due to thrombotic disease the options are intra-arterial thrombolysis,
angioplasty or bypass surgery. If due to thrombosis of an arterial graft, then thrombolysis
is the first step.
For patients with acute arterial emboli or thrombosis, treatment with immediate systemic
anticoagulation with unfractionated heparin has been recommended. This should be
followed by long-term warfarin in patients with embolism.
An arteriogram is performed and the catheter advanced into the thrombus.
Streptokinase, urokinase or tissue plasminogen activator (tPA) should be combined with
heparin and thrombolysis continued for 48 hours or until clot lysis.
Fibrinolysis usually takes between 6 and 72 hours to achieve clot lysis and so patients
with limb-threatening ischaemia are not candidates for local fibrinolysis, and require
emergent embolectomy. Local thrombolytic therapy is therefore reserved for patients
with non-life-threatening limb ischaemia.
If a limb is irreversibly ischaemic, amputation will be required.
1606. A 50yo woman complains of several months hx of weakness and difficulty
climbing stairs. Exam: fissuring of the skin of her hands. CXR: pulmonary fibrosis. What
is the single most likely positive antibody?
a. Anti Jo1
b. Anti Scl 70
c. Anti Ro
d. Anti dsDNA
e. Anti centromere
Key: Anti Jo1 (A)
Reason: Anticentromere would be present in limited scleroderma and Anti-Scl 70 would
be present in diffuse scleroderma. Anti-DSDNA would be positive in SLE, along with
Anti-Ro. Anti-Ro is also positive in Sjogrens syndrome and scleroderma. This is a case
of Polymyositis because none of the others would have the muscular weakness that is
present in this case. The antibody of choice would be Anti Jo1 antibody.
1607. A 65yo woman complaining of symptoms suggestive of Raynauds phenomenon
and difficulty in swallowing. Exam: painful lesions on her finger tips and facial
telangiectasis. What is the single most likely positive antibody?
a. Anti Jo1
b. Anti Scl 70

c. Anti Ro
d. Anti ds DNA
e. Anti centromere
Key: Anti-Centromere (E)
Reason: Anti-centromere antibody would be present in CREST syndrome which is also
called Limited scleroderma. All the others are already accounted for and would be
present in other connective tissue diseases.
1608. A 6yo boy presented about 4h ago with acute severe pain on the testis with the
left half slightly higher than the right. Pain was not relieved by any strong analgesic.
What is the initial
management?
a. Give strong analgesic
b. IV NS and monitor vital signs
c. Reassure
d. Immediate surgical referral
e. Cover with antibiotics
Key: Immediate surgical referral (D)
Reason: This is a case of possibly testicular torsion, if in any doubt, refer immediately to
surgery/urology. Testicular torsion presents with inflammation of one testes which is
tender, hot and swollen. It may lie high and/or transversely. Analgesics, IV fluids and
antibiotics may have a role but none of those things would be initial management.
Immediate consent is sought for orchidectomy but first it is tried to unwind the testicle to
avoid surgery. Reassurance is an asinine choice.
1609. A 60yo man is brought to the ED in an agitated state. He is lashing out violently.
Which drug in low dosage due to its relative lack of autonomic side effects is a drug of
choice in the tx of agitation in this pt?
a. Haloperidol
b. Diazepam
c. Fluoxetine
d. Clozapine
e. Chlorpromazine
Key: Haloperidol (A)
Reason: Acute psychosiss drug of choice is always Haloperidol. Diazepam would
sedate the patient but would not take care of his psychotic symptoms, fluoxetine is an
SSRI and would take 2 weeks to act, clozapine and chlorpromazine are not drugs of
choice in psychotic episodes due to their adverse effects.
1610. A 32yo woman of 40wks gestation attends the antenatal day unit with sudden
onset epigastric pain with nausea and vomiting. She is clinically jaundiced. Her
biochemistry results show a raised bilirubin, abnormal liver enzymes, high uric acid and
hypoglycemia. Whats the most likely dx?
a. Acute fatty liver of pregnancy
b. Obstetric cholestasis
c. Cholecystitis
d. HELLP syndrome
e. Acute hepatitis

Key: Acute Fatty Liver of Pregnancy (AFLP) (A)


Reason: Pain, nausea, vomiting, jaundice, fever with elevated liver enzymes and
bilirubin is clinically indicative of AFLP. Also can have elevated INR, TLC and
hypoglycaemia. It isnt cholestasis because there is no pruritis, cholecystitis isnt the
answer because there is no history of evidence of gallstones, HELLP isnt the answer
because there isnt any hemolysis or thrombocytopenia and acute hepatitis would
present subclinically or with very less symptoms like diarrhoea and vomiting alongwith
clinical history of food poisoning.
Discussion:
Epidemiology:
* It is a rare condition with an incidence of 5 in 100,000 pregnancies.
* Acute fatty liver of pregnancy (AFLP) tends to occur in late pregnancy.
* Risk factors include first pregnancies, pre-eclampsia, twin pregnancies and male
fetuses.
* It may be associated with a mutant gene producing a defect in mitochondrial fatty acid
oxidation and infants born to mothers with AFLP should be screened for defects in this
system.
Presentation:
* This usually presents acutely with nausea, vomiting and abdominal pain, fevers,
headache and pruritus, beginning typically at about 35 weeks of gestation but can occur
much earlier. It may also appear immediately after delivery.
* Jaundice appears soon after onset of symptoms and can become intense in a large
proportion of patients. Fulminant liver failure may follow.
Investigations:
* The white cell count is often elevated. There may also be neutrophilia and
thrombocytopenia.
* Liver transaminases are moderately high.
* Raised serum bilirubin.
* Abnormal clotting with coagulopathy (prolongation of prothrombin and partial
thromboplastin times with depression of fibrinogen levels).
* Biopsy would be diagnostic but coagulation problems often preclude it. CT/MRI
scanning may show reduced attenuation in the liver.
Management:
Consider early delivery, as the condition usually resolves afterwards with complete
recovery. Supportive ITU care is frequently required.
Complications:
AFLP is a life-threatening condition with a reported 1.8% maternal and 23% fetal
mortality rate. Serious complications include: * Disseminated intravascular coagulation
(DIC) and gastrointestinal bleeding.
* Hepatic coma. * Acute kidney injury. * Pancreatitis. * Hypoglycaemia.
1611. A 24yo man believes his bowels are blocked and his life is in ruin. What kind of
delusion is he suffering from?
a. Persecutory
b. Factitious
c. Guilt
d. Nihilistic
e. Hypochondriacal
Key: Nihilistic (D)

Reason: The man believes his bowels are blocked and his life is ruined. This is a
presentation of nihilistic delusions whereupon the individual believes his life or his
organs are useless and leading to his death, destruction or ruin. Persecutory delusions
involve the belief that people are out to get him or her and factitious delusions are when
the person himself is lying, guilt is an emotion and requires a history of inciting factor
while hypochondriacal delusions are delusions that are disease oriented and targeted to
receiving medicines or care from a doctor.
1612. A 75yo man with declining vision, cornea and pupils are normal, fundus shows
obscured margins. What is the single most likely dx?
a. Macular degeneration
b. HTN retinopathy
c. MS
d. DM background
e. Proliferative DM retinopathy
Key: Macular degeneration (A)
Reason: The age of the patient, normal opthalmological examination and obscured
margins of the fundus all point towards age related macular degeneration. HTN would
have other findings on the ocular exam, MS presents with optic neuritis and RAPD +ve
with red colour blindness developing, DM and proliferative DM would present with other
fundus findings according to degree and stage of diabetic retinopathy.
1613. A man under psychiatric tx develops GI distress and tremors. Which drug is most
likely to cause these symptoms?
a. Lithium
b. Diazepam
c. Citalopram
d. Clozapine
e. Imipramine
Key: Lithium (A)
Reason: Lithium causes these symptoms along with Diabetes insipidus.
1614. A 24yo man presents with painless hematuria. No other complaint and no
abnormality is found on physical exam. What is the most appropriate initial inv which is
helpful to get a dx?
a. Coag screening
b. MSU
c. Cystoscopy
d. MRI spine
e. Abdominal US
Key: Abdominal USG (E)
Reason: Painless hematuria in a young male without any other findings on history or
examination is likely Polycystic Kidney Disease which frequently presents with painless
gross hematuria. Coagulation screening in a 24y old man is only helpful with a history of
blood dyscrasias, mid stream urine will not help us in establishing a diagnosis in this
case, Cystoscopy is used to visualize the urinary tract, better to perform in older age
group to rule out CA and MRI spine will not help us at all.

1615. A 29yo woman presents to her GP with troublesome heavy periods. The med tx
that she has tried have made little difference. She is known to have large uterine
intramural fibroids. You
confirm that she is currently trying for more children. Select the most appropriate
management
for menorrhagia in this pt?
a. Danazol
b. Endometrial ablation
c. Hysterectomy
d. Hysteroscopic resection of fibroids
e. Myomectomy
Key: Myomectomy (E)
Reason: She is currently trying for more children, hysterectomy will not be useful in this
case obviously. OHCS pg. 276 states that chance of subsequent pregnancies is better
after myomectomy and it is the best treatment in this case. Endometrial ablation will not
affect the fibroids and danazol causes a post-menopausal state which would not help
her in conceiving.
1616. A 30yo schizophrenic female attacks her mother believing that aliens have
replaced her with an exact double. What condition is she suffering from?
a. Capgras syndrome
b. Ganser syndrome
c. Todd syndrome
d. Fregoli syndrome
e. Cotard syndrome
Key: Capgras Syndrome (A)
Reason: Capgras syndrome is an irrational belief that a familiar person or place has
been replaced by a duplicate.
Ganser syndrome is a fictitious disorder in which a patient deliberately acts as if he has
a physical or mental illness when he doesnt have it.
Todd syndrome/Alice In Wonderland syndrome/Lilliputian syndrome is a disorienting
neurological condition affecting human perception of size, shape and time.
Fregoli syndrome is a delusion of doubles, a delusional belief that different ppl are infact
a single person in disguise or change appearance.
Cotards syndrome/Nihilistic delusions is walking corpse syndrome, the person think
they are dead or that one of their organs has stopped functioning.
1617. A 38yo man has just returned from a holiday where he went swimming everyday.
For the last few days he has had irritation in both ears. Now his right ear is hot, red,
swollen and acutely
painful. What is the single most likely dx?
a. Foreign body
b. Impacted earwax
c. OE
d. OM
e. Perforation of eardrum
Key: Otitis Externa (C)
Reason: The swimming history, irritation in both ears and ear being hot, red, swollen and
painful indicates inflammation of the external acoustic meatus called Otitis Externa. It

isnt otitis media because of the lack of Tympanic membrane signs, perforated eardrum
would present with just pain and deafness, impacted earwax would also present with
pain and conductive deafness. Foreign body would have history of something being
used near or inside the ear and would be seen on examination of the ear canal.
1618. A healthy 2yo boy is brought to the ED having cut his hand playing in the garden.
He has a 2cm clean laceration. He has not received any routine immunizations as his
parents are concerned about possible side effects. There are no contraindications to
immunizations. What is the single most appropriate follow up inv?
a. Courses of DPT vaccine
b. Courses of DT
c. Single inj of DPT vaccine
d. Single inj of DT
e. Single inj of tetanus Ig
Key: Courses of DPT Vaccine (A)
Reason: The child is unimmunized and has no contraindication to vaccination. Keeping
in mind his age, single injections would be useless and would not help the patient.
Courses of DPT vaccine would be the best choice in this case.
1619. A 6wk child has hx of frequent vomiting which became worse during the last
weeks. He has no fever, recently he has passed stool only once every 2-3d. What inv
will you do to confirm the dx?
a. Abdominal US
b. Barium meal
c. Erect XR abdomen
d. Feed test
e. Reassure
Key: Abdominal USG (A)
Reason: Barium studies in this case are always the wrong answer, Abdominal USG is
going to give you a definitive diagnosis as soon as is realistically possible. The age,
symptoms point towards pyloric stenosis as the likely cause, X-ray Abdomen will not
show pyloric stenosis at all, Feed test is a useless answer and reassuring the patients
parents will not work because this is something that needs intervention, possibly
surgically by Ramstedts pyloromyotomy.
1620. A 30yo woman had an IUCD inserted 8-9m ago. Now on routine follow up the
thread is missing. Uterine US showed no IUCD in the uterus. What is the best
management?
a. Laparoscopy
b. Pelvic CT
c. Laparotomy
d. Pelvic XR
Key: Pelvic XR (D)
Reason: Thread is missing and it isnt seen in the uterus, Xray is the logical choice.
Laparotomy and laparoscopy are not needed unless it perforates an organ which is
highly unlikely. USG is the first choice which has been performed, Xray after that to
check location and then advice surgical retrieval if needed.

1621. A pt comes with weight loss and sleep disturbance has mild depression. He has a
hx of MI. What is the single most appropriate tx?
a. Diazepam
b. ECT
c. Imipramine
d. Lithium
e. Antipsychotics
Key: Diazepam
Reason: Diazepam is the mildest option available here, ECT is very unnecessary,
Imipramine and other TCAs are severely contraindicated with a history of MI, Lithium
causes MI as an adverse effect and antipsychotics are used for psychosis.
1622. A pt. comes back from India and presents with night sweats and
lymphadenopathy. XR:
Cavitation. What investigation should be done next?
a. CT scan
b. AFB stain
c. Blood culture
d. Bronchoscopy
Key: AFB Stain (B)
Reason: The symptoms and arrival from an endemic area for Pulmonary TB suggests
the best course of action would be to go for AFB staining via ZN stain. CT Scan would
not help in initial diagnosis, blood culture isnt first line for TB, sputum culture clearly is
and bronchoscopy is unnecessarily invasive.
1623. A 45yo woman has been extensively investigation for a lump she believes to be
cancer. She doesnt think doctors take her seriously and demands another referral. What
term best describes her condition?
a. Munchausen syndrome
b. Munchausens by proxy
c. Hypochondriasis
d. Malingering
e. Phobia
Key: Hypochondriasis (C)
Reason: This patient, with her history of extensive investigations or her fat folder
syndrome warrant nothing other than the label of being a Hypochondriac who wants
treatment for imaginary illnesses she has even after being counselled about the severity
of her illness.
Muchausens syndrome describes a patient who lies vividly, is addicted to institutions
and goes from hospital to hospital feigning illnesses hoping for laparotomy or
mastectomies.
Munchausens by proxy defines injury to a dependent person by carer to gain medical
attention. Malingering is the creation of a fictitious illness without even the lump which is
present in this case. Phobia is a fear of something irrational.
1624. A 15yo man presents with bitemporal hemianopia and spade-like hands. What is
the definite test to confirm the dx?
a. Early morning growth hormone

b. Insulin tolerance test


c. OGTT with growth hormone measurements
d. Random insulin-like growth factor (IGF-1)
e. Short ACTH test
Key: OGTT with growth hormone measurements (C)
Reason: The bitemporal hemianopia and spade-like hands point towards this being
acromegaly. The best initial test is insulin like growth factors but the definitive test that
confirms the diagnosis is OGTT with serial growth hormone measurements. Early
morning growth hormone would be raised anyway, it is released in a pulsatile manner,
Short ACTH is used for Cushings disease and Insulin tolerance test is not used for
growth hormone or acromegaly assessment.
Discussion:
How is acromegaly diagnosed?
* A blood test can measure the level of growth hormone. However, a single test is not
reliable. This is because the levels of growth hormone in the body fluctuate a lot
throughout the day in all people. * The diagnosis of acromegaly is made by a glucose
tolerance test. In this test you drink a sugar drink containing 75 grams of glucose. You
then have a series of blood tests over two hours. The glucose should lower the blood
level of growth hormone. However, if you have acromegaly, the growth hormone level
remains high.
* A blood test to measure the level of IGF-1 (see above) may be measured if acromegaly
is suspected. This may also be used as an index of disease activity to assess how well
treatment is working. * A magnetic resonance imaging (MRI) scan can show the size of
any tumour.
* Eye and visual tests can assess if the tumour is pressing on the optic nerve.
* If you are confirmed as having acromegaly, other tests will be needed to see if the
tumour is causing a lack or excess of other hormones made by the pituitary. * Other
tests may include chest X-ray, electrocardiogram (ECG) and X-rays of some of your
joints.
What are the treatments for acromegaly?
The aim of treatment is to reduce the level of growth hormone in the blood to normal,
and to reduce the size of an enlarged tumour. Many of the symptoms and features of
acromegaly will reverse or improve with successful treatment (apart from any fixed extra
bone growth that had occurred).
Surgical treatment
The most common treatment is to remove the adenoma by surgery. This is done using
very fine instruments. There are two different ways to operate on the pituitary gland:
o The first method is known as endonasal trans-sphenoidal surgery. This involves the
surgeon reaching your pituitary gland through a small cut (incision) in wall of one of your
nostrils. In the other way, the surgeon approaches the pituitary gland through a small
incision behind your upper lip, just above your front teeth. The instruments are passed
through the base of your skull - the sphenoid bone. The aim is to remove the adenoma,
but to leave the rest of the pituitary gland intact.
o The operation is successful, with no further treatment needed, in around 9 out of 10
cases with smaller tumours. The operation is less successful in those with larger
tumours. However, sometimes it is not possible to remove all the cells of the tumour. If
not all is removed and your growth hormone level remains high following surgery, other
treatments listed below are likely to work.
o Your surgeon will advise on the possible complications which can sometimes occur.
For example, sometimes the operation may damage some other parts of the pituitary

gland. This may cause a reduced production of some other hormones. If this occurs, you
will need to take replacement hormone therapy.
Medication
Medication can be used if surgery is not possible, or not wanted. It is also used whilst
waiting for surgery or radiotherapy. It can also be used in cases where surgery fails to
remove the tumour totally and the level of growth hormone remains high.
* Somatostatin analogues (octreotide and lanreotide) reduce the level of growth
hormone to normal in over half of cases, and reduce the size of the tumour in about 8 in
10 cases. However, these medicines need to be given as an injection. They work in a
similar way to somatostatin (described above) which is a hormone that prevents growth
hormone from being released from pituitary cells. These medicines used to be injected
several times a day. However, longer-acting preparations are now available as monthly
or fortnightly injections. Side-effects are not common with these medicines. Some
people develop tummy (abdominal) pains and diarrhoea, but these usually wear off with
time. Gallstones can also occur but rarely cause problems. * Dopamine agonists (such
as cabergoline, bromocriptine and quinagolide) can be taken as tablets. They work by
preventing the release of growth hormone from tumour cells. However, they only work
well in about 1 in 5 cases. Side-effects such as feeling sick and dizzy are also quite
common.
* Pegvisomant (Somavert) is taken as a daily injection. However, unlike the other
medicines listed above, it does not act directly at the pituitary. Pegvisomant works by
blocking the action of growth hormone on your body's cells. Therefore, although many of
the symptoms of growth hormone excess will be eased, it does not reduce the size of
the tumour and headaches are not eased.
* Radiotherapy * Radiotherapy is an option to reduce the size of the tumour and hence
reduce the production of growth hormone. Radiotherapy focuses high-intensity radiation
at your pituitary tumour to destroy the abnormal cells. It may be used if you are not able
to have surgery, or if surgery was only partially successful. However, it can take months
or years after the radiotherapy is given for the level of growth hormone to reduce to
normal. You can take medication whilst waiting for the effects of radiotherapy to work.
A possible side-effect of pituitary radiotherapy is damage to other normal pituitary cells.
This can cause a reduced level of some other hormones. However, if this occurs you
can take replacement hormone therapy.
Acromegaly and bowel cancer screening
As mentioned earlier, people with acromegaly have an increased chance of developing
bowel (colonic) polyps and bowel cancer. Therefore, if you are diagnosed with
acromegaly and are aged 40 or more you will normally be offered a routine colonoscopy
every 3-5 years. A colonoscopy is a test where an operator (a doctor or nurse) looks into
your large bowel (colon) with a flexible telescope. It can diagnose bowel problems such
as polyps and bowel cancer. The aim is to detect those people who develop cancer as
early as possible (before symptoms develop) when the chance of a complete cure is
high.However, you should always tell you doctor if you develop any new symptoms from
your bowel such as persistent diarrhoea, passing mucus, passing blood, or tummy
(abdominal) pain.
1625. A 22yo man has had an acute, painful, red right eye with blurring of vision for one
day. He had a similar episode 1y ago and has had episodic back pain and stiffness
relieved by exercise and diclofenac for four years. What is the SINGLE most likely cause
of his red eye?
a. Chorioretinitis

b. Conjunctivitis
c. Episcleritis
d. Iritis
e. Keratitis
Ans)D
Key)The symptoms described are characteristic of ankylosing spondylitis.(lower back
pain and stiffness which gets better after moving around and taking NSAIDS) the extra
articular manifestations of AS are uveitis.
Symptoms:

Anterior uveitis:eye pain,photophobia,redness,visual loss,pupil shape


changes,symptoms develop in few hours or days,maybe single eye is affected
Intermediate uveitis:painless blurred vision,FLOATERS,both eyes normally
affected
Posterior uveitis:painless blurred vision,severe vision loss,FLOATERS and
scotomas, take longer to develop.
the stem of the question talks about the symptoms of anterior uvitis, hence the
answer is iritis. (chorioretinitis is excluded as it comes under the symptoms of
posterior uvitis)
in episcleritis, the sclera may look blue and visual acquity is NORMAL.
conjunctivitis will present with similar symptoms as well but there will be a
discharge and the question will talk about eyes sticking together.Keratitis will be
identified with a white patch on the cornea.
Treatment of uvietis
1. Steroid eye drops
2. cycloplegic eye drops to relieve pain
3. dark sun glasses
4. painkillers
Reason: These symptoms point towards Ankylosing Spondylitis with the episodic back
pain that is relieved by exercise and NSAIDs for four years. The presentation is that of
Uveitis which is associated with autoimmune diseases, specifically Iritis with acutely
painful red eye. Chorioretinitis would present in the same way, but 1/3 or AS patients
have Iritis. Conjunctivitis and Episcleritis would not have blurring of vision and Keratitis
wouldnt be associated with systemic symptoms.
1626. A 40yo divorced man with bipolar affective disorder attends hospital following an
OD of 30 TCA tablets. His new partner has left him and he has stopped taking his
medicine and begun drinking heavily. He appears depressed, feels hopeless and is
ambivalent about being alive. He is now fit for discharge from the medical ward and
acknowledges the benefits of previous tx. What is the SINGLE most appropriate next
management?
a. Admission to the psychiatry ward
b. Arrange psychiatric outpatient follow-up
c. Discharge to the care of the general practitioner
d. Referral to local alcohol treatment team
e. Referral to clinical psychologist
Ans)A

Key) the trigger (partner leaving) precipitated an acute attack of maniac depression in
the patient. there is no other option BUT for him to be admitted in a psychiatric unit with
his actions being constantly observed.the stem of the question says he stopped taking
his medication. abrupt cessation of Lithium precipitates mania in 50% of the patients as
well and people with mania arent likely to adhere to their medications in the first place
either.Discharging the patient with an SSRI/antidepressant isnt a very good move since
SSRIs take at least 2-4 weeks for their effect to build up fully. therefore, proving that the
best possible answer to the question is still A.
Reason: The patient is high risk on the suicide risk assessment scale and doesnt
warrant out-patient follow up or discharge to be seen by a GP. He should be admitted
and treated in the Psychiatry ward as needed. Referral to the local alcohol treatment
team or clinical psychologist is never the right answer.
1627. A healthy baby boy is born at term to a woman who was unwell with confirmed
acute hep B
during pregnancy. The mother is very concerned that she may have infected the baby
with hep
B. What SINGLE preventative intervention should be given to the baby?
a. Full course of hepatitis B vaccine
b. Hepatitis B immunoglobulin alone
c. Hepatitis B vaccine and hepatitis B immunoglobulin
d. Hepatitis B vaccine as single dose
e. None until hepatitis B status confirmed
Ans)C
Key)Babies born to mothers infected with hepatitis B have a high risk of acquiring infection,
which can be prevented by vaccination at birth.All babies with seropositive mothers should
have the full primary course of hepatitis B immunisation and most should also have HBIG
within 24 hours of birth.
Reason: If the mother has active confirmed acute Hep B, give Immunoglobulins and
vaccinate the baby at birth. Also perform serology of the baby at 12-15 months (1 year). If
Hep B Antibodies +ve and HbSAg -ve, do nothing. Full course of the vaccine is unnecessary
because the aforementioned course will provide sufficient coverage, Ig alone and Vaccine
alone will not be enough for the baby. Full course with immunoglobulin will be given at
BIRTH, 1 MONTH, 2
MONTHS. (E) isnt a good option because the baby needs immediate coverage in case the
infection has been transmitted at delivery.

1628. A previously well 15yo girl had an acute onset of fever, sweating, bruising and
petechiae. A
blood count showed: Hgb=63g/L, WBC=1.1mg/L, Neutrophils=0.1, plt=14. No abnormal
white
cells were seen on the blood film. She was transfused and given IV antibiotics and her
condition
improved. 3wks later her blood count has returned to a similar picture. What is the
SINGLE most
likely underlying dx?
a. ALL
b. AML
c. Aplastic anemia
d. CML

e. Pernicious anemia
Key: Aplastic Anaemia (C)
Reason: The age of the patient and pancytopenic picture give us a clinical diagnosis of
Aplastic anemia. Normal WBC morphology rules out ALL, AML and pernicious anaemia
while the age rules out CML as a diagnosis.
1629. An 83yo woman admitted with a chest infection becomes confused with impaired
attention and poor concentration. She is restless and frightened. She is verbally abusive
and has perceptual
abnormalities. There is no significant prv psychiatric hx. What is the SINGLE most likely
dx?
a. Delirium
b. Drug induced psychosis
c. Lewy body dementia
d. Multi-infarct dementia
e. Psychotic depression
Key: Delirium (A)
Reason: Drug induced psychosis would require a drug history, dementia and psychotic
depression would have a significant previous psychiatric history. Delirium or Acute
Confusional States happen in the elderly in response to stressors like acute infections
and this is most likely brought on by the chest infection that has developed.
1630. A town has a population of 500,000. In a five year period there are 1250 cases of
bladder cancer diagnosed at the only hospital. During the same period the occupational
health department
diagnosed a further 500 cases. What is the annual incidence per million of bladder
cancer in this
population?
a. 2100
b. 1750
c. 1400
d. 700
e. 350
Key: 700 (D)
Reason: There are 1750 cases a 5 year period in this scenario. The number of cases in
1 year in the 0.5 million people in this town are 350 cases/0.5 million people. To
calculate the incidence per million of bladder cancer, we would simply double the
number to get an estimate which would give us 350 x 2 = 700 cases/million.
1631. A 28yo woman who has had a prv pulmonary embolism in pregnancy wishes to
discuss
contraception. She has menorrhagia but is otherwise well. What is the SINGLE most
suitable
contraceptive method for this patient?
a. COCP
b. Copper IUCD

c. Levonorgestrel intra-uterine system


d. Progestogen implant
e. POP
Key: Levonorgestrel Intra-Uterine System (C)
Reason: The woman has a history of thromboembolic disease, which essentially rules
out COCPs. POPs, Copper IUCD (Copper T) and Progestogen implants would not
help the menorrhagia. LNG-IUS (Mirena) is the hormone releasing device that is most
suitable in this patient and would be the contraceptive of choice.
1632. An 8yo girl has had left earache for 2d. The earache subsided about 2h ago with
the onset of a purulent discharge which relieved the pain. Her temperature is 39.2C.
What is the SINGLE most appropriate antibiotic?
a. Amoxicillin
b. Ciprofloxacin
c. Clindamycin
d. Erythromycin
e. Flucloxacillin
Key: Amoxicillin (A)
Reason: This is the picture of Acute Otitis Media which has led to tympanic membrane
perforation. PO Amoxicillin for 7 days is the treatment of choice with appropriate
analgesics. Amoxicillin +/- Clavulanate is the first line drug because it fits the organisms
responsible which are Strep pneumonia, Moraxella catarrhalis and H. influenza.
1633. A 38yo man has disturbing thoughts about his house being infected by germs. He
is anxious
about safety and checks the locks of his doors repeatedly before going to bed. For the
last 8wks
he has been washing his hands every time he touches the lock, 20-30 times a day. What
is the
SINGLE most appropriate management?
a. Antidepressant
b. Antipsychotic
c. Anxiolytic
d. CBT
e. Psychodynamic psychotherapy
Key: CBT (D)
Reason: This scenario describes a case of OCD for which the best management is CBT
followed by SSRIs or TCAs. The first line treatment is always CBT, not pharmacological
therapy. Psychotherapy is indicated in depression, psychosomatic disorders,
dissociative or conversion disorders, personality disorders, relationship problems or
grief.
1634. A 65yo man had closure of colostomy performed 5d ago. He is not systemically
unwell. There is

a tender, localised fluctuant swelling 4 cm in diameter in the wound. What is the SINGLE
most
appropriate management?
a. Abdominal support
b. Antibiotics
c. Laparotomy and re-suture wound
d. Local exploration of wound
e. Observation
Key: Local exploration of wound (D)
Reason: The patient is not systemically unwell, he has had colostomy performed 5 days
back. The swelling is tender, localised and fluctuant. Local exploration with I&D if
needed is the single most appropriate management option. Abdominal support is
unnecessary, so are antibiotics if the patient is systemically well. Laparotomy is indicated
if there is a bleed or some surgically correctable issue going on, observation is not the
right answer here because intervention is warranted.
1635. A 32yo woman has had a febrile illness and swelling of the small joints of her
hands, feet, wrists and knees for two days. She has a maculopapular rash and a few
palpable, small cervical lymph nodes. She was previously well. There is no history of
relevant travel outside the UK. She has two young children. What is the SINGLE most
likely dx?
a. Psoriasis
b. Reactive arthritis
c. Rheumatoid arthritis
d. Sarcoidosis
e. SLE
key)B
Ans)
Reactive arthritis is caused when a joint reacts to an infection elsewhere in the body. The
infection which triggers reactive arthritis is not actually in the joint, but is usually in the gut
(gastroentritis) or urethra (uretheritis).it can also occur after viral infections causing sore
throat skin rash etc.
Fever is not usually a feature of the other diseases mentioned here.

1636. A 16yo girl has had an enlarging mass in the right side of her neck for the last
6wks. She has had
no other symptoms. She has a 2 x 2 cm enlarged LN in the anterior triangle of the neck
with
several smaller associated LN palpable. Oropharyngeal examination shows tonsillar
membranes.
What is the SINGLE most likely dx?
a. Infectious mononucleosis
b. Leukaemia
c. Lymphoma
d. Sarcoidosis
e. Tuberculosis
key)A

Ans)infectious mononucleosis is seen primarily in young teenagers and college going


students (16 year old girl). the rest of the options are excluded because there is no
mention of weight loss(all other 4 options), chest involvement (sarcoidosis,TB), skin
involvement (leukemia=bruises,sarcoidosis=skin rash), night sweats (TB,lymphoma)
symptoms of infectious mononucleosis:
Sore throat; tonsillar enlargement is common, classically exudative and may be
massive
Fine macular non-pruritic rash, which rapidly disappears.
Lymphadenopathy, especially neck glands.
Arthralgias and myalgias low grade fever
Later signs include:
o Mild hepatomegaly and splenomegaly
o Jaundice

investigation:detection of heterophile antibodies by the monospot test or the paul


bunnell test

EBV is also associated with:

Burkitt's lymphoma.
B-cell lymphomas in patients with immunosuppression.
Undifferentiated carcinomas - eg, cancer of the nasopharynx and cancer of the
salivary glands.

treatment is supportive.

1635. A 32yo woman has had a febrile illness and swelling of the small joints of her
hands, feet, wrists
and knees for two days. She has a maculopapular rash and a few palpable, small
cervical lymph
Nodes. She was previously well. There is no history of relevant travel outside the UK.
She has
two young children. What is the SINGLE most likely dx?
a. Psoriasis
b. Reactive arthritis
c. Rheumatoid arthritis
d. Sarcoidosis
e. SLE
Key is B: Reactive Arthritis

Patient is having reactive arthritis because of any underlying infection,


most commonly urethral infection.
Reactive arthritis is caused when a joint reacts to an infection elsewhere in
the body. The infection which triggers reactive arthritis is not actually in
the joint, but is usually in the gut or urethra.
This is because they are most at risk of urethral infection from sexually
transmitted diseases. However, it can occur at any age and in anyone.
Treatment:
Antibiotics for infection
NSAIDs for joint pain
1636. A 16yo girl has had an enlarging mass in the right side of her neck for the last
6wks. She has had
no other symptoms. She has a 2 x 2 cm enlarged LN in the anterior triangle of the neck
with
several smaller associated LN palpable. Oropharyngeal examination shows tonsillar
membranes.
What is the SINGLE most likely dx?
a. Infectious mononucleosis
b. Leukaemia
c. Lymphoma
d. Sarcoidosis
e. Tuberculosis
Key is A: infectious Mononucleosis
Infectious mononucleosis (IM) is usually a self-limiting infection, most
often caused by Epstein-Barr virus (EBV)
The incubation period is between 1 and 2 months
Clinical infection is most common in populations with many young adults
Presentation:
Low-grade fever, fatigue and prolonged malaise.
Sore throat; tonsillar enlargement is common
Fine macular non-pruritic rash, which rapidly disappears.
Transient bilateral upper lid oedema.
Lymphadenopathy, especially neck glands.
1637. A 60yo man has had increasing pain in both buttocks, thighs and calves on
walking for three
months. He has also recently developed impotence. Femoral and distal pulses are
absent in both
limbs. What is the SINGLE most likely site of arterial obstruction?
a. Aorto iliac
b. External iliac
c. Femoropopliteal
d. Internal iliac
e. Tibial
Key is A: Aorto iliac
Page 718 OHCM 8 edition.
Patient is suufering from leriches syndrome.
Classic triad of:
Pain and claudication of buttock and thighs
Erectile dysfunction from aorto iliac occlusive disease
Absent femoral and distal pulse.
th

1638. A 78yo man has collapsed. He has had a severe headache for 12 hours and had
an URTI 3d ago.
He has a temp=39.2C, pulse=122bpm, BP=84/60mmHg and RR=34bpm but his chest is
clear. He
has a GCS=10 and some neck stiffness. He has been started on high-flow oxygen. What
is the
SINGLE most appropriate immediate management?
a. IV antibiotic; CT brain scan
b. IV antibiotic; LP
c. IV fluids; CT brain scan
d. IV fluids; IV antibiotic
e. IV fluids; LP
Key is D: IV Fluids; IV antibiotics
As his BP is low and has presented with nech stiffness so, IV fluids and V
antibiotics should be given, after giving him oxygen.
1639. A 16yo boy was brought to hospital in a comatose state having taken methadone
belonging to
his sister. He was given naloxone and rapidly became alert. Some hours later, he
gradually
becomes semi-conscious again.What is the SINGLE most likely reason for this patient
becoming
semi-conscious again in hospital?
a. Methadone hepatotoxicity has caused acute liver failure
b. Methadone is eliminated from the body more slowly than naloxone
c. Naloxone is a partial agonist at the central nervous system opioid receptor
d. The pt has misused another substance that has caused an intracranial bleed
e. The pt has misused another substance that is absorbed more slowly than methadone
Key is B: Methadone is eliminated from the body more slowly than naloxone
All other options are not favoring the scenario
1640. A 27yo woman who takes the COCP has had painless vaginal spotting and
discharge for 3 days.
Her last menstrual period, which lasted four days, finished 10 days ago. Her last cervical
smear
two years ago was normal. Abdominal and vaginal examinations are normal apart from a
mild
ectropion with contact bleeding. What is the SINGLE most appropriate initial inv?
a. Cervical smear
b. Colposcopy
c. Endocervical swab
d. Endometrial biopsy
e. Pelvic US
Key is C: Endocervical swab
As her cervical smear and examination of abdomen and vagina are normal, next
would be to exclude a STD for which Endocervical swab is taken.
Chlamydia is an sexually transmitted infection caused by a germ
(bacterium) calledChlamydia trachomatis
About 1 in 20 sexually active women in the UK are infected with chlamydia.
It is most common in women aged under 25. (About 1 in 12 women aged 20
are infected with chlamydia.)
Presentation:

Vaginal discharge. This is due to the neck of the womb (cervix) becoming
inflamed.
Pain or burning when you pass urine.
Vaginal bleeding or spotting between periods. In particular, bleeding after
you have sex.
Pain or discomfort in the lower tummy (abdomen) area (the pelvic area)
Diagnosis
Chlamydia can be confirmed by a swab taken from the neck of the womb
(cervix) in womenhave sex
Treatment
A short course of an antibiotic medicine usually clears chlamydial infection
Note: antibiotics can interfere with the combined oral contraceptive pill
(COCP). If you take the COCP you should use alternative methods of
contraception until seven days after finishing a course of antibiotics
1641. A 72yo man being investigated for anaemia is booked for a colonoscopy in 24
hours. What is the
SINGLE most appropriate management the night before the procedure?
a. Bisacodyl tablets
b. Glycerine suppository
c. Lactulose syrup
d. Magnesium citrate (orally)
e. Senna tablets
Key is D: Magnesium Citrate Orally
Colonoscopy is a test to assess your colon (large intestine).
What preparation do I need to do?
The colon needs to be empty so that the operator can obtain a clear view.
You will be instructed on how to take a special diet for a few days before
the test. You will also be given some laxatives to take.(magnesium citrate is
the least harmful)
You will need somebody to accompany you home, as you will be drowsy
with the sedative
1642. A 19yo woman has had progressive bilateral iliac fossa pain and dyspareunia for
3days. She has
an offensive vaginal discharge and feels unwell and feverish. Her temp=39C. An initial
antimicrobial regimen is commenced. What SINGLE set of organisms are the most
appropriate
for the antimicrobial regimen to cover?
a. Neisseria gonorrhoeae and Candida albicans
b. Neisseria gonorrhoeae and Candida albicans and Gardnerellavaginalia
c. Neisseria gonorrhoeae and Chlamydia trachomatis
d. Neisseria gonorrhoeae and Chlamydia trachomatis and Candida albicans
e. Neisseria gonorrhoeae and Chlamydia trachomatis and Gardnerellavaginalis
Key is C: Neisseria gonorrhoeae and Chlamydia trachomatis
Most common antibiotic given for STDs is azithromycin and it covers gram
negative bacteria which are neisseria and chlamydia.
Candida albicans is a fungal infection
Gardnerella is a gram variable bacteria
1643. A 48yo man with renal cancer had radiotherapy for metastatic spinal cord
compression at the
11th thoracic vertebra 4wks ago. He has retained sensation but is unable to stand. He
has pain

in a band around his lower trunk controlled by regular oral morphine. He is distressed by
increasingly frequent episodes of painful muscle spasms in his right leg. What is the
SINGLE most
appropriate management of his symptoms?
a. Amitriptyline
b. Baclofen
c. Fentanyl patch
d. Gabapentin
e. Increase morphine dose
Key is B: Baclofen
1644. A 4yo girl has had a temp=38.5C for 2days and has not wanted to eat her food.
Yesterday she
developed a sore throat and small, painful ulcers inside her mouth. Today she has small
blisters
on the palms of her hands and soles of her feet which are painful but not itchy. What is
the
SINGLE most likely underlying cause?
a. Coxsackie virus
b. Herpes simplex virus
c. Staphylococcus aureus
d. Streptococcus pneumonia
e. Varicella zoster virus
Key is A: coxsakie virus
Patient is suffering from HFMD (Hand,Foot and Mouth Disease)
HFMD is due to an infection that usually causes a typical illness, including
a typical rash. It is most commonly caused by the Coxsackie A16 virus
HFMD most commonly affects children under 10 years of age
This might include a high temperature (fever). After this, a sore throat
commonly occurs, quickly followed by small spots that develop inside the
mouth. These soon progress into small mouth ulcers
In many cases, spots also develop on the skin. This is typically a day or so
after the mouth ulcers develop. The spots are small lumps that are a few
millimetres in diameter and usually appear on the hands and feet, they are
not usually itchy but sometimes they can be a little bit sore.
Treatment:
There is no treatment that will take away the virus
1645. A 32yo woman has had 3 episodes of slurred speech and 2 episodes of transient
weakness of
both legs in the past 5yrs. Each episode has resolved in 3m. What is the SINGLE most
likely dx?
a. Meningioma
b. Migraine
c. Multiple sclerosis
d. Stroke
e. Transient ischaemic attack
Key is C: Multiple sclerosis
Points in favor: symptoms have a relapsing-remitting form
MS is thought to be an autoimmune disease
About 1 in 600 people in the UK develop MS. It can affect anyone at any
age, although it is rare in young children. It most commonly first develops

around the age of 30. MS is the most common disabling illness of young
adults in the UK. It is twice as common in women as in men.
symptoms include:
Visual problems
The first symptom of MS for around one in four people with MS is a disturbance of
vision. Inflammation (swelling) of the optic nerve can occur. This is called optic
neuritis. This can cause pain behind your eye and also some loss of your vision.
This usually only affects one eye.
Muscle spasms and spasticity
Tremors or spasms of some of your muscles may occur. This is usually due to
damage to the nerves that supply these muscles. Some muscles may shorten
(contract) tightly and can then become stiff and harder to use. This is called
spasticity.
Pain
There are two main types of pain that may occur in people with MS:
Neuropathic pain
Musculoskeletal pain
Fatigue
Extreme tiredness (fatigue) is one of the most common symptoms of MS.
Other symptoms which may occur include:
Numbness or tingling in parts of the skin. This is the most common
symptom of a first relapse.
Weakness or paralysis of some muscles. Mobility may be affected.
Problems with passing urine.
Inability to have an erection in men.
Difficulty with speaking.
Investigation and Treatment:
A firm diagnosis of MS is often not made until two or more relapses have
occurred
A magnetic resonance imaging (MRI) scan of the brain is a useful test. This
type of scan can detect small areas of inflammation and scarring in the
brain which occur in MS.
At present, although there is no cure for MS.
1646. An 8yo girl is complying with her asthma treatment of low-dose inhaled
corticosteroid
prophylaxis and short-acting bronchodilators as required. Her inhaler technique is good.
She
now has a frequent night cough and mild exercise-induced wheeze. What would be the
SINGLE
most appropriate change in her treatment?
a. Add leukotriene antagonist
b. Add oral theophylline
c. Add regular long-acting bronchodilator
d. Increase dose of inhaled corticosteroid
e. Short course of oral corticosteroid
Key is D: Increase dose of inhaled corticosteroid
A common treatment plan for a typical person with moderate asthma is:
A preventer inhaler (usually a steroid inhaler), taken each morning and at
bedtime. This usually prevents symptoms throughout the day and night.

A reliever inhaler (short acting bronchodilators) may be needed now and


then if breakthrough symptoms occur. For example, if symptoms flare up
when you have a cough or cold.
If exercise or sport causes symptoms then a dose of a reliever inhaler just
before the exercise usually prevents symptoms.
The dose of the preventer inhaler may need to be increased for a while if
you have a cough or cold, or during the hay fever season
1647. A 38yo man with longstanding alcohol dependence has vertigo and a tremor every
morning.
What is the SINGLE most likely dx?
a. Anxiety
b. Benign positional vertigo
c. Cerebellar degeneration
d. Optic neuritis
e. Temporal lobe epilepsy
Key is C: Cerebellar degeneration
Cerebellar degeneration is a process in which neurons in the cerebellum the area of the brain that controls coordination and balance - deteriorate
and die
Associated diseases:
ischemic or hemorrhagic stroke, when there is lack of blood flow or
oxygen to the cerebellum
cerebellar cortical atrophy, multisystem atrophy, and olivopontocerebellar
degeneration, progressive degenerative disorders in which cerebellar
degeneration is a key feature
Friedreichs ataxia, and other spinocerebellar ataxias, which are caused by
inherited genetic mutations that result in ongoing loss of neurons in the
cerebellum, brain stem, and spinal cord
transmissible spongiform encephalopathies (such as Creutzfeldt-Jakob
disease) in which abnormal proteins cause inflammation in the brain,
including the cerebellum
multiple sclerosis, in which damage to the insulating membrane (myelin)
that wraps around and protects nerve cells can involve the cerebellum
Other diseases that can cause cerebellar degeneration include:
chronic alcohol abuse that leads to temporary or permanent cerebellar
damage
1648. An 84yo woman with Alzheimer's dementia has recently become incontinent and
more
confused than usual. What is the SINGLE most likely dx?
a. Detrusor overactivity
b. Neuropathic bladder
c. Nocturnal enuresis
d. UTI
e. Uterine prolapse
Key is D: UTI
1649. A 4yo boy complains of pain around his right eye. He is unwell, febrile and also
suffers from pain
on the right side of his face. What is the most probable dx?
a. Allergic reaction
b. Furuncle
c. Folliculitis

d. Foreign body
e. Periorbital cellulitis
Key is E: Periorbital cellulitis (also called preseptal cellulitis)
preseptal cellulitis occurs at younger ages (80% of patients are under 10
years of age and most are younger than 5 with a mean age of 21 months)
Preseptal cellulitis:
Acute onset of swelling, redness, warmth and tenderness of the eyelid.
Fever, malaise, irritability in children.
Ptosis
Diagnosis: is usually made based on the clinical findings
Management:
Most children are initially admitted to hospital (even for preseptal cellulitis)
unless there is good reason not to. This may be just for 24 hours. Children
should be considered to have orbital cellulitis until proven otherwise (ie
repeated examinations normal, good response to antibiotics in first 24
hours and normal CT scan).
Oral co-amoxiclav may be used for both adults and children as long as
there is no allergy to penicillin. Clinical improvement should occur over 2448 hours.
1650. A pt presents with irregularly irregular pulse of 162bpm. What drug is most useful
initially?
a. Amiodarone
b. Digoxin
c. Bisoprolol
d. Warfarin
e. Heparin
Key is C: Bisoprolol
Bisoprolol is Beta blocker used as antihypertensive and to control the heart rate
aswell.
Bisoprolol slows down the activity of your heart by stopping messages sent by
some nerves to your heart. It does this by blocking tiny areas (called betaadrenergic receptors) where the messages are received by your heart. As a result,
your heart beats more slowly and with less force.
1651. A 59yo man has shown a change in his mood and personality over a 9m period.
He has
subsequently developed difficulty with memory and conc, and then progressive fidgety
movements of his limbs and facial musculature. By the time of medical assessment he
has frank
choreiform movements and a mini-mental state exam of 21/30. Other exam is normal.
He was
adopted and therefore no information on his famhx is available. He has 3 adult children
(27, 30,
33) of whom the 2 youngest are asymptomatic. However, the oldest son has recently
been inv
by the neurology dept for slightly erratic behavior and fidgety restless movements of both
legs.
Based on the likely clinical dx, which one of the following genetic patterns is most likely?
a. AD inheritance with anticipation
b. AD with variable penetrance
c. AR
d. X-linked

e. Mitochondrial disorder
Key is A: AD inheritance with anticipation
Patient is suffering from Huntingtons disease and that is autosomal
dominant with anticipation which means a genetic disorder is passed on to
the next generation, the symptoms of the genetic disorder become
apparent at an earlier age with each generation.
Huntingtons Disease:
It is an inherited (genetic) condition that affects the brain and nervous
system. It can interfere with movements of your body, can affect your
reasoning, awareness, thinking and judgement (cognition) and can lead to
a change in your behavior
This faulty gene is carried on chromosome 4.
HD affects between 5-10 people per 100,000 in the UK.
Presentation
The symptoms of HD can be grouped into three main areas:
Problems with movement
Problems with cognition
Mood and behavioural problems
Treatment:
At present there is no cure for HD. Also, there is no treatment that has
been found to delay the onset of symptoms or to delay the progression of
symptoms
1652. A 35yo pt has been dx with schizophrenia. He mimics the doctors and attendants
doing the
same physical actions as them. What symptom does this pt have?
a. Echopraxia
b. Echolalia
c. Perseveration
d. Apraxia
e. Anosognosia
Key:A
Ans)Echopraxia is the involuntary repetition or imitation of another person's actions. Similar
to echolalia, which is the involuntary repetition of sounds and language. Echopraxia has
long been recognized as a core feature of Tourette syndrome, and is considered a complex
tic, but it also occurs in autism spectrum disorders, schizophrenia and catatonia.Apraxia is a
motor disorder caused by damage to the brain (specifically the posterior parietal cortex), in
which someone has difficulty with the motor planning to perform tasks or movements when
asked, provided that the request or command is understood and he/she is willing to perform
the task. Apraxia is an acquired disorder of motor planning, but is not caused by
incoordination, sensory loss, or failure to comprehend simple commands (which can be
tested by asking the person to recognize the correct movement from a series)Anosognosia
is a deficit of self-awareness, a condition in which a person who suffers a certain disability
seems unaware of the existence of his or her disability.
Key is A: Echopraxia
Echopraxia is involuntary imitation of the movements and is a feature for the
diagnosis of schizophrenia.
Schizophrenia:
Schizophrenia is a serious mental health condition that causes disordered
ideas, beliefs and experiences. In a sense, people with schizophrenia lose
touch with reality and do not know which thoughts and experiences are true
and real and which are not.

Schizophrenia develops in about 1 in 100 people. It can occur in men and


women. The most common ages for it first to develop are 15-25 in men and 2535 in women.
Presentation:
Delusions
Hallucinations.
Disordered thoughts.

1653. A pt has loss of sensation on the tip of her tongue and the inner aspect of the lip.
Which nerve is
most likely to be involved?
a. Vagus nerve
b. Glossopharyngeal nerve
c. Lingual nerve
d. Buccal nerve
e. Facial nerve
Key is C: Lingual Nerve
a. Vagus nerve: It leaves the skull through the jugular foramen, passes within the
carotid sheath in the neck, through the thorax supplying the lungs, and continues
on via the oesophageal opening to supply the abdominal organs.
b. Glossopharyngeal nerve: Passes across the posterior fossa, through the
jugular foramen and into the neck, supplying tonsil, palate and posterior third of
tongue.
c. Lingual nerve: lingual nerve is a branch of the mandibular division of the
trigeminal nerve (CN V ), which supplies sensory innervation to thetongue. It also
carries fibers from the facial nerve, which return taste information from the
anterior two thirds of the tongue, via thechorda tympani
3

d. Buccal nerve:brach of mandibular division of trigeminal nerve, supplies the


skin and mucous membrane of the cheek
e. Facial nerve: Mainly motor (some sensory fibres from external acoustic meatus,
fibres controlling salivation and taste fibres from the anterior tongue).
1654. A 51yo woman complains of difficulty swallowing and also reddish dots on her
skin. A pic of her
hand is seen. What is the most appropriate term for the condition you would expect to
see?
a. Sclerodactyly
b. RA
c. Swan neck deformity
d. Polydactyly
e. Ulnar deformity
Key is A: sclerodactyly
Patient is suffering from crest syndrome, sclerodactyly its one of the features.
(page 554 OHCM)
Limited systemic sclerosis(CREST syndrome is a part of which) Calcinosis(sub cu
tissue), Raynauds phenomenon, esophageal and gut dysmotility, Sclerodactyly
and telengectesia. Skin involvement is limited to the face, hand and feet. It is
associated with anti-centromere antibodies in 70-80%. Pulmonary hypertension is
often present subclinically and become rapid life threatening.

1655. A 37yo female working as a healthcare assistant in a nursing home comes to the
ED with
complaints of severe itching all over her body. On asking she replies that she had
applied cream
on the body of a resident in the nursing home who had similar itches. What is the
mechanism of
itching?
a. Allergic reaction
b. Inflammation of keratinocytes
c. Allergic reaction developed due to use of topical steroid creams
d. Subcutaneous bleeding
e. None
Key is A: allergic reaction
A severe allergic reaction (anaphylaxis) will affect the whole body; in
susceptible individuals it may develop within seconds or minutes of
contact with the trigger factor and is potentially fatal.
Possible triggers can include skin or airborne contact with particular
materials, the injection of a specific drug, the sting of a certain insect or the
ingestion of a food such as peanuts
1656. A 65yo pt who had MI 1yr ago now comes to the ED complaining that his neighbor
is conspiring
against him. When his son is asked, he denies it and also narrates that sometimes his
father says
that everybody in his office is always talking about him, which is not the case. What is
the most
appropriate med?
a. TCA
b. Clozapine
c. Olanzapine
d. Lorazepam
Key is C: Olanzapine
TCA is not given at this age
Clozapine needs strict monitoring and can only be given under the supervision of
a psychiatrist
Olanzapaine is antipsychotic which is most suitable in this case
1657. You suspect Cushing's disease in a 50yo woman who has attended clinic with
glycosuria, HTN
and a suggestive body habitus. Initial inv point you towards a dx of Cushing's disease.
Which of
the following findings would be against this dx?
a. A normal 8am cortisol
b. Failure to suppress morning cortisol with dexamethasone
c. HTN requiring >2 antihypertensive agents
d. Impaired growth hormone response to glucose loading
e. Unilateral adrenal enlargement
Key is E: Unilateral adrenal enlargement
Cushings disease
Bilateral adrenal hyperplasia from an ACTH secreting pituitary adenoma.
Peak age 30-50 years, male female ratio 1:1
A low dose dexamethasone test leads to no change in plasma cortisol but 8 mg
may be enough

1658. Which finding, on clinical examination of the pulse, suggests a diagnosis of


hypertrophic
obstructive cardiomyopathy (HOCM)?
a. Irregularly irregular pulse suggesting A-fib
b. Pulsusalternans
c. Pulsusbigeminus
d. Pulsusbisferiens
e. Pulsusparadoxus
Key is C Pulsusbisferiens
pulsusbisferiens, is a sign where, onpalpation of the pulse, a double peak per
cardiac cycle can be appreciated. Bisferious means striking twice. Classically, it is
detected when aortic insufficiency exists in association with aortic stenosis, but
may also be found hypertrophic obstructive cardiomyopathy.
Hypertrophic Cardiomyopathy:
Hypertrophic cardiomyopathy (HCM) is an autosomal dominant genetic
disorder characterised by left ventricular hypertrophy (LVH), impaired
diastolic filling, and abnormalities of the mitral valve. These features can
cause dynamic obstruction of the left ventricular outflow tract, diastolic
dysfunction, myocardial ischaemia, and an increased risk of
supraventricular and ventricular tachyarrhythmias.
HCM is the most common genetic cardiovascular disease.
Epidemiology:The prevalence of HCM is about one in 500 and it tends to
affect men and black people more often. The obstructive form is seen in
25% of cases.
Hypertrophy can occur in any part of the left ventricle, although it is most
common in the anterior ventricular septum.
The presentation: is variable and includes dyspnoea (the most common
presenting symptom), chest pain, palpitations and syncope.
Examination: Classic examination findings are a forceful apex beat, with
double impulse(pulsusbisferiens) if the left ventricular outflow tract is
obstructed and a late ejection systolic murmur, which can be augmented
by standing or Valsalvamanoeuvre and diminished by squatting.
Investigation: Electrocardiogram (ECG): most patients have an abnormal
ECG, although electrocardiographic features are nonspecific and include
LVH, ST segment changes and T-wave inversion.
1659. A 60yo male is admitted with a 2d hx of lower abdominal pain and marked
vomiting. On
examination he has abdominal swelling, guarding and numerous audible bowel sounds.
What is
the likely dx?
a. Gallstone ileus
b. Ischemic colitis
c. Large bowel obstruction
d. Sigmoid volvulus
e. Small bowel obstruction
Key is D: sigmoid volvulus
History is not suggestive of ischemic colitis.
There are no bowel sounds heard in gallstone ileus, large bowel obstruction and
small bowel obstruction, hence it is sigmoid volvulus.
Sigmoid Volvulus:
[1]

[2]

In sigmoid volvulus, a large sigmoid loop full of faeces and distended with gas
twists on its mesenteric pedicle to create a closed-loop obstruction.
Risk factors:
The elderly.
Chronic constipation.
Megacolon, large redundant sigmoid colon and excessively mobile colon.
It is more common in men.
Presentation:
lower abdominal pain associated with gross abdominal distension and a
failure to pass either flatus or stool.
Abdominal examination reveals a tympanitic, distended (but usually nontender) abdomen and a palpable mass may be present.
Investigation and Management:
Characterstic Abdominal X-Ray with an inverted U loop of bowel that looks
a bit like a coffee bean.
Often managed by sigmoidoscopy and insertion of a flatus tube. Sigmoid
colectomy is sometimes required
1660. A 17-year-old boy is diagnosed with scabies. Which of the following statements
regarding
scabies is correct?
a. Is best treated by salicylate emulsion
b. It can be spread by a droplet infection
c. It causes itchiness in the skin even where there is no obvious lesion to be seen
d. It is caused by Staphylococcus aureus
e. Typically affects the face
Key is C: It causes itchiness in the skin even where there is no obvious lesion to
be seen
Excluded Points:
a. The usual scabies treatment is with permethrin cream. Permethrin is an
insecticide that kills the mites
b. You need close skin-to-skin contact with an infected person to catch scabies.
This is because the scabies mite cannot jump or fly
d. Scabies is caused by a mite (like a tiny insect) called Sarcoptesscabiei. The mite is
a parasite, meaning it lives off the host (a human) with no benefit to the host
e. Itching is often severe and tends to be in one place at first (often the hands)
Scabies:
Scabies is caused by a mite (like a tiny insect) called Sarcoptesscabiei. The
mite is a parasite, meaning it lives off the host (a human) with no benefit to
the host.
Scabies is common. In the UK, about 1 in 1,000 people develop scabies
each month. Scabies is more common in town (urban) areas, in women and
children, in the winter, and in the North of the country.
The skin-to-skin contact needs to be for a reasonable time to catch the
mite. You usually need to be in skin contact for 15-20 minutes to catch
scabies.
Scabies symptoms usually take 2-6 weeks to occur after you are first
Treatment:
Scabies can stay in your skin for ever if not treated. Treatment is needed for:
Anybody who has scabies; AND
All household members, close contacts, and sleeping/sexual partners of
the affected person - even if they have no symptoms. This is because it can
[2]

take up to six weeks to develop symptoms after you become infected.


Close contacts may be infected, but have no symptoms, and may pass on
the mite.
Apply 5% permethrin over whole body including scalp, face (avoid eyes),
neck and ears. Do not forget the soles; wash off after 8-12 h, repeat after 7
days.

1661. An anemic young man is found to have a macrocytosis of 90%. The most likely
cause is?
a. Zieves syndrome
b. Thalassemia minor
c. Chronic renal disease
d. IDA
e. Folate def
f. Chronic liver disease
g. HUS
h. Cytotoxic chemotherapy
i. Phenytoin
Ans: Floate Def
Zieve's syndrome is an acute metabolic condition that can occur during withdrawal from
prolonged alcohol abuse. It is defined byhemolytic anemia (with spur cells and
acanthocytes), hyperlipoproteinaemia (excessive blood lipoprotein), jaundice, and abdominal
pain.[1] The underlying cause is liver delipidization

1662. An association with HPV is a most characteristic feature of?


a. Torus
b. Exotosis
c. Pleomorphic adenoma
d. Verruca vulgaris
e. Fibroma
f. Epulis fissuratum
g. Mucocele
h. Pyogenic granuloma
i. Parulis
j. Ranula
Ans.D, it is most commonly associated with warts or verruca vulgaris.
Exotosis:benign outgrowth cartilaginous tissue
Plemorphic Adenoma: A Locally invasive benign tumor
Fibroma: benign tumors composed of fibrous tissue
Epulis Fissuratum:benign hyperplasia of fibrous connective tissue which develops as a
reactive lesion to chronic mechanical irritation produced by the flange of a poorly fitting
denture
Mucocele: a benign swelling containing mucin.
pyogenic granuloma: Pyogenic granulomata are common benign vascular lesions of the
skin and mucosa.a reactive inflammatory mass of blood vessels and a few fibroblasts
within the dermis of the skin.
Parulis: an elevated nodule at the site of a fistula draining a chronic periapical ABSCESS

Ranula: A ranula is a type of mucocele found on the floor of the mouth


1663. For the following type of surgery what is the most likely agent that may cause
post-operative infection -- aorto-iliofemoral reconstruction with a Dacron vascular
prosthesis?
a. Proteus
b. E.coli
c. Bacteroides fragilis
d. Staphylococcus aureus
e. Staphylococcus epidermis
f. C.perfringens
g. Pseudomonas aeruginosa
h. Streptococcus fecalis
i. Streptococcus pneumonia
j. Brucella melitensis
Ans: Staph epidermidis is most common cause of infections in prosthesis.
1664. A primigravida in the 17th week of her symptomless gestation is found, on US, to
have evidence
of placental tissue covering the cervical os. By the end of her pregnancy she is likely to
develop?
a. Placental migration
b. Uterine myoma
c. Uterine rupture
d. Choriocarcinoma
e. Chorangioma
f. Vasa previa
g. Subplacental abruption
placenta
h. Subchorionic abruption
placenta
i. Placenta accrete
j. Placenta previa
Ans: A

In 90% of pregnancies, an initial low lying placenta will be pulled upwards by the
growing uterus and assume a normal position in the upper segment. This
phenomenon is referred to as Migration
1665. An elderly lady with COPD has chronic SOB. She is listed for cataract extraction.
What is the
anaesthetic of choice?
a. Facial nerve block
b. Bupivacaine infiltration of the
peri-orbital skin
c. IV midazolam
d. Peribulbar acupuncture

e. Peribulbar lignocaine
infiltration
f. Topical xylocaine
g. IV alfentanil
h. Epidural anesthesia
i. General anesthesia
j. Retrobulbar xylocaine Inj
Ans:E

The most used mode of anaesthesia in ophthalmic feild is peribulbar using


lingocaine
1666. A 55yo chronic alcoholic with known hepatic cirrhosis has been on a heavy bout of
alcohol the
night before and was brought home by friends after falling several times in the pub.
While being
taken up the stairs to his bedroom he falls down the flight of 5 steps but sustains no
obvious
injuiry. His wife calls the ED the next day because she could not rouse him in the
morning. He is
brought in in a comatose state and both pupils appear dilated. Skull vault XR appears
normal.
a. Hepatic encephalopathy
b. Intracerebral hematoma
c. Brain stem injury
d. Extradural hematoma
e. Chronic subdural hemorrhage
f. Despressed skull fx
g. Vertibrobasilar ischemia
h. Acute subdural hematoma
i. SAH
j. Severe migraine attack
ANS: H
Acute subdural hemotoma, typical history of alcholics, falls and usually debilliated or
elderly, hepatic cirrhosis increases coagulopathy and chances for bleed.
1667. A 58yo man complains of nose disfigurement. He has a hx of facial erythema
particularly of the
cheeks and nose. Papules and pustules have been erupting at intervals over the last
10yrs. He
admits to a moderate regular consumption of alcohol. Exam: noted to have rhinophyma.
The
most likely dx is?
a. Eczema
b. Herpes simplex
c. Epidermolysis bullosa
d. Dermatomyositis
e. Tinea versicolor
f. Pemphigus vulgaris

g. Acne rosacea
h. Malignant melanoma
i. Psoriasis
j. Atopic dermatitis
Ans: Acne Rosacea
Reason: Acne Rosacea is characterised by recurrent episodes of facial flushing with
persistent erythema, telangiectasia, papules and pustules.It is a chronic acneform disorder of
the facial pilosebaceous glands with an increased reactivity of capillaries to heat, causing
flushing and eventually telangiectasia.Rhinophyma is an enlarged nose associated with
rosacea which occurs almost exclusively in men.
Management: Reassurance, benign disease, sunscreens, facial massage reduces
oedema.avoid topical steroids, avoid astringents.
Topical .75% MTZ firstline.
Azaleic Acid 15% gel.
Oral oxytetracyline, doxycyline or erythromycin.
1% Ivermection cream (better than MTZ)
Topical Brimonidine for flushing

1668. A 60yo man who presented with metastatic adenocarcinoma of unknown source.
He developed
rapidly progressive weakness of his arms and was found to have a deposit of tumour in
his
cervical spine. This was emergently treated with radiation. He developed considerable
nausea
and vomiting during his therapy and at the end of the course began to have bloody
vomiting.
Following rescusitation with 6 units of blood, what is the next test of choice?
a. Apt test
b. Neck, chest, abdominal XR
c. 24h esophageal pH probe test
d. CT abdomen
e. US abdomen
f. MRI abdomen
g. Barium swallow
h. Angiography
i. Nuclear scan
j. Endoscopy
Ans: Endoscopy
Reason: whenever there hemetemesis endoscopy should be carried out immediately if
the patients condition allows or it should be delayed till resucitation ,Underlying cause for
hemetemesis needs to be sorted out.( High dose radiation is a cause Ulceration and any
active bleeders must be treated)
1669. A pt has fine nail pitting, small yellow-brown areas of discoloration in the nailbed
involving the nails on both hands. These findings a
re commonly associated with?
a. Yellow nail syndrome

b. Leukonychia
c. Onychomycosis
d. Lichen planus
e. Pellagra
f. Thallium toxicity
g. Contact dermatitis
h. Zinc deficiency
i. Hypoalbuminemia
j. Psoriasis
Ans: Psoriasis
Characteristic nail changes include pitting, discolouration,subungual hyperkeratosis,
crumbling of the nail plate, and onycholysis.
Oil drop or salmon patch: translucent yellow-red discoloration in the nail bed.
Leukonychia: areas of white nail plate due to foci of parakeratosis within the body of the nail
plate.
1670. A young man develops nonfluent, effortful speech with dysarthria. He is able to
undertsand
speech. He fails to repeat the sentence. What would you do next?
a. XR skull
b. Non-contrast CT brain
c. Contrast CT brain
d. Contrast MRI optic nerves
e. 4-vessel cerebral angiogram
f. Single vessel cerebral
angiogram
g. Cerebral angiography
h. MRI frontal lobe
i. MRI pituitary gland
j. MRI temporal lobe
Ans: MRI Frontal lobe (Brocas area)
Production (Broca's) dysphasia/aphasia - lesions are located in the left pre-central areas.
This is a non-fluent or expressive aphasia since there are deficits in speech production,
prosody and syntactic comprehension. Patients will typically exhibit slow and halting speech
but with good semantic content. Comprehension is usually good. Unlike Wernicke's aphasia,
Broca's patients are aware of their language difficulties. Prosody is the study of the meter of
verse. Here it means the rhythm of speech
Sensory (Wernicke's) dysphasia/aphasia - lesions are located in the left posterior
perisylvian region and primary symptoms are general comprehension deficits, word retrieval
deficits and semantic paraphasias. Lesions in this area damage the semantic content of
language while leaving the language production function intact. The consequence is a fluent
or receptive aphasia in which speech is fluent but lacking in content. Patients lack awareness
of their speech difficulties. Semantics is the meaning of words. Semantic paraphrasia is the
substitution of a semantically related but incorrect word.

1671. A pt being sedated with fentanyl develops severe respiratory depression. This is
best reversed
using?
a. Ethanol
b. Naloxone
c. Phyostigmine
d. Atropine
e. Methylene blue
f. Diphenhydramine
g. Calcium disodium ethylene
diamine tetra-acetic acid
h. Deferoxamine mesylate
i. Flumazenil
j. Folic acid
Ans: Naloxone
Opioid Antagonist, reverses the effects of fentanyl, though it has to be administeres for a
longer period of time due long half life of fentanyl.
1672. A pt presented with the following blood work, MCV: Decreased Serum ferritin:
Decreased Total
iron binding capacity: Increased Serum iron: Decreased Marrow iron: Absent. What is
your dx?
a. Thalassemia trait
b. Hypoparathyroidism
c. Hereditary sideroblastic anemia
d. Protein energy malnutrition
e. Chronic renal failure
f. Anemia of chronic disease
g. Acute blood loss
h. IDA
i. Oral contraceptives
j. Megaloblastic anemia
H: Iron deficiency Anemia
Reason: S/S pallor, koilonychia,angular cheilitis, atrophic glossitis, IN marked Anemia (
Cardiac enlargemnet,Flow Murmurs,ankle oedema and heart failure)
Inv: FBC : shows microcytic hypochromic anemia
Serum ferrtitin Level reduced, normal 12-15 mcg/L, ( serum ferritin is falsely raised
during infections)
Anisocytosis and poikilocytosis
Total iron binding capacity is increased
Treatment: Iron supplememtation with B12 and folic acid
1673. A 20yo prv healthy woman presents with general malaise, severe cough and
breathlessness
which has not improved with a seven day course of amoxycillin. There is nothing
significant to
find on examination. The x-ray shows patchy shadowing throughout the lung fields. The
blood

film shows clumping of red cells with suggestion of cold agglutinins.


a. Mycobacterium avium complex
b. Coxiella burnetii
c. Escherichia coli (Gram -ve)
d. Haemophilus influenza
e. Legionella pneumophila
f. Strep pneumococcus
g. TB
h. Mycoplasma pneumonia
i. PCP
j. Staph aureus
Ans: Mycoplasma pneumonia
Reason : inablity to respond to a seven day course of amoxicillin suggests atypical
pneumonia, patchy shadows throughout lung fields and cold agglutination points towards
mycoplasma.
M. pneumoniae:[9]
Vague and slow-onset history over a few days or weeks of constitutional upset, fever,
headache, dry cough with tracheitic pleuritic pain, myalgia, malaise and sore throat.
This is like many of the common viral illnesses but the persistence and progression of
symptoms is what helps to mark it out.
In otherwise healthy individuals, it usually resolves spontaneously over a few weeks.
The hacking, dry cough can be very persistent.
Extra-respiratory features include rashes such as erythema multiforme, erythema nodosum
and urticaria; neurological complications like Guillain-Barr syndrome, transverse myelitis,
cerebellar ataxia and aseptic meningitis; haematological complications such as cold
agglutinin disease and haemolytic anaemia; joint symptoms like arthralgia and arthritis;
cardiac complications such as pericarditis and myocarditis; rarely, may cause pancreatitis

TReatment: Macrolides, Fluroquinolones


1674. An 18yo male works in a company where lunches are often catered. One day, the
water at the
company facility is not working, but they manage to have the lunch anyway. 2wks later,
he
becomes sick. He develops anorexia, nausea, malaise and jaundice. During the course
of the
next 4wks, 7 people who shared in the lunch become ill with similar symptoms. After a
few wks,
each of the 7 people completely recovers and they replace their caterer. What is a likely
dx?
a. Pancreatic ca b. Hemochromatosis
c. Laennecs cirrhosis
d. Hep A

e. HCC
f. Rotors syndrome
g. Primary biliary cirrhosis
h. Gilberts syndrome
i. Hep B
j. Hemolysis
Ans: Hepatitis A
Symptoms of Hepatitis A range from mild nauseas to liver failure (very rare).Spread is
normally by the faecal-oral route although there are occasional outbreaks through food
sources.Hand washing and good hygiene around food and drink prevent spread of
infection.Increasing age is a direct determinant of disease severity.
S/S The incubation period is 2-6 weeks with a mean of 4 weeks.
There is a prodrome of mild flu-like symptoms (anorexia, nausea, fatigue, malaise and joint
pain) preceding the jaundice. Smokers often lose their taste for tobacco. Diarrhoea can
occur, particularly in children.
This can progress to the icteric phase with:Dark urine (appears first).Pale stools (not always).
Jaundice occurring in 70-85% of adults with acute HAV infection. Abdominal pain occurring
in 40% of patients. Itch or pruritus. Arthralgias and skin rash.Tender hepatomegaly,
splenomegaly, and lymphadenopathy
Inv:IgM antibody to HAV is positive with onset of symptoms (usually about 3 to 4 weeks after
exposure but up to 6 weeks). The test is sensitive and specific. It remains positive for
between 3 and 6 months (up to 12 months). It remains positive in relapsing hepatitis.
IgG antibody to HAV appears soon after IgM and persists for many years. In the absence of
IgM it indicates past infection or vaccination rather than acute infection. IgG remains
detectable for life
LFTs
Mangement: Supportive, avoid alcohol.

1675. A 35yo 1st time donor suddenly passes out as she is donating blood. Which of the
following
steps would be least useful in managing this adverse event?
a. Ensure donor is adequately hydrated and has not skipped a meal
b. Elevating the donor's legs as this is usually due to a vasovagal syncope
c. Haemoglobin of the donor meets the minimum requirement for donation
d. The donation is usually continued along with simultaneous normal saline infusion
e. The donor should be encouraged to mobilise after they have recovered
Ans: C
Option C has no role after such an event has occured.

1676. An infant is being examined as part of a routine examination. The child can hold
its head up and
lifts its chest off a table. He has a palmer and rooting reflex as well as a social smile. He
is not
afraid of strangers. What is the most likely age of this child?
a. neonate
b. 2 months
c. 6 months
d. one year
e. one and a half years
f. two years
g. four years
h. seven years
i. ten years
j. fourteen year
Ans:6 months
Reason : holding head and raising chest upto 90 degrees from a surface while prone
(4months)
social smile 6 weeks, becomes increasingly socially responsive.
palmer and rooting reflexes are primitive reflexes
1677. A mother is concerned because her 1m boy has a swelling in his scrotum. He was
born
prematurely. On examination the swelling is seen to transilluminate. The likely cause is?
a. Lymphogranuloma Venereum
b. Testicular Torsion
c. Hydrocele
d. Epididymitis
e. Seminoma
f. Mature teratoma
g. Varicocele
h. Lymphoma
i. Orchitis
j. Spermatocele
Ans: Hydrocele
Reason: 1-2% neonates present with congenital hydrocele which disappears by 1-2
years.
Spermatoceles:Smooth, extratesticular, spherical cysts in the head of the epididymis are not
uncommon in adult men. They are benign and do not usually require treatment.Epididymal
cysts usually develop in adults around the age of 40. Epididymal cysts are rare in children
and when they occur, usually present around puberty.
1678. A 2m girl has an ante-natal diagnosis of right hydronephrosis. Postnatal serial US
exams revealed
increasing dilatation of the right pelvicalyceal system. No reflux was demonstrated on a
MUCG.
Appropriate management should include?

a. Surgical repair
b. Intermittent catheterization
c. Diuresis renography
d. Anticholinergic agents
e. Phenylpropanolamine
f. Gellhorn pessary
g. Biofeedback-assisted behavioral
treatment
h. Oral Estrogen therapy
i. Vaginal Estrogen therapy
j. Ring pessary
1679. Jean is a 72yo woman with recurrent bowel cancer following a hemi-colectomy 2y
ago. She is
known to have both local recurrence and liver mets and her pain has been under control
on MST
90mg bd. She has had quite severe pain in the RUQ for the past hour despite having
taken her
normal dose of MST. You find that she has an enlarged liver which is hard and irregular.
There is
marked localised tenderness over the right lobe of her liver. Her abdomen is otherwise
soft and
non-tender and the bowel sounds are normal. She is apyrexial. The tx of choice would
be?
a. Oral NSAIDs
b. TENS
c. radio therapy to the liver
d. IM diamorphine
e. Paracetamol
f. Prednisolone
g. Physiotherapy
h. epidural anaesthetic
i. Pitocin
j. Aspirin
Ans: I/M diamorphine . Pain ladder
1680. Titubation is a feature of disease involving the?
a. Cerebellum b. Basal ganglia
c. Corpus callosum
d. Pons
e. Temporal lobe
f. Occipital lobe
g. Optic chiasma
h. 3rd ventricle
i. Hypothalamus
j. Pituitary gland
Ans: Cerebellum
titubations mostly occur due to cerebellar lesions.

1681. A 50yo farmer complains of pain in his left arm. Exam: he appears to have a
neuropathy
affecting isolated nerves in multiple, random areas of his left arm. He also has a
palpable
purpura and tender nodules on both of his upper and lower limbs. A likely diagnosis is?
a. Carpal tunnel syndrome
b. Polyarteritis nodosa
c. Angina Pectoris
d. Gout
e. Cellulitis
f. Rheumatoid arthritis
g. Erysipelas
h. Fascitis
i. Reiter's Syndrome
j. Polymyalgia Rheumatica
Ans: PAN
PAN is necrotising arteritis of medium or small arteries without glomerulonephritis or
vasculitis in arterioles, capillaries, or venules, and not associated with antineutrophil
cytoplasmic antibodies (ANCAs).It can affect any organ but, for unknown reasons, it spares
the pulmonary and glomerular arteries.

Presentaion:Peripheral nerves and skin are the most frequently affected tissues.
PURPURA,LIVEDOID,SUBCUTANEOUS NODULES and NECROTIC
ULCERS.
Neurologically, MONONEURITIS MULTIPLEX>...involvemnet of
CNS,Git,kidneys and heart means higher mortality.RENAL
INVOLVEMENT:hypertension,AKI, GIT:necrosis,perforation.Myalgia

Investigations: Hepatitis B surface antigen is positive in 30%.


The p-ANCA test is usually negative in PAN.
There is a prominent acute phase response but this is nonspecific.
FBC shows leukocytosis with raised neutrophils.
Hypergammaglobulinemia occurs in 30%.
Biopsy
Arteriography shows aneurysms
TREATMENT: Corticosteroids..RELAPSE add Cyclophosphamide.=>
Azathioprine useful in maintenance therapy.
(IV-Ig) and aspirin are effective in childhood PAN.

1682. A patient with chronic neutropenia develops a chronic cough. A CXR reveals a
cavitating
intrapulmonary lesion containing a movable rounded ball lesion. A likely dx is?
a. Tuberculosis
b. Bronchiectasis
c. Cystic fibrosis

d. Pulmonary hemosiderosis
e. Mitral stenosis
f. Aspergillosis
g. Wegeners granulomatosis
h. Goodpastures syndrome
i. Pulmonary embolism
j. Non-SCLC
Ans: Aspergillosis.
mostly affects people with reduced immunity, reduced neutrophil count is also
predilection for aspergillosis. five clinical types of Aspergillosis
APBA
Severe Asthma with fungal sensitization
Aspergilloma : (fungal ball in already caivitated space due to TB, Sarcoidosis)
Invasive aspergillosis
Chronic Necrotizing Pulmonary Aspergillosis.
Aspergilloma usually presents with massive hemoptysis, cough and fever are rare,
Discovered occasionally in asymptomatic patiets on xray showing a mass in upper lobe
surrounded by air border. CT demonstrated fungal structure more accurately.
Surgical removal
Long term itraconazole therapy
Instillation of amphotericin
hemoptysis needs to be treated with bronchial artery embolization.
1683. A mother brings her 1yo infant to her pediatrician. She describes that following a
common cold
her child's voice has become hoarse and has developed a cough that sounds harsh and
brassy
and was worse at night. Exam: the child was noted to have trouble drawing air into its
lungs
between coughs and had trouble drawing air into its lungs. There was visible stridor on
inhalation. The cause is most likely to be?
a. EBV
b. Rhinovirus
c. Parainfluenza
d. Flavivirus
e. HIV
f. Rotavirus
g. CMV
h. Kemerovo
i. Creutzfeld-Jacob
j. Rubella
Ans: Parainfuenza
Parainfluenza virus causes several respiratory problms, laryngitis, bronchitis, pneumonia
and bronchiolitis,
A rough barking cough with croup
hoarsness and wheezing

labored breathing
runny nose, fever, cough
decreased appetitie, diarrhea.
Investigation: Viral culture of secretions.
Treatment: Symptoatic supportive treatment
Antibiotics incase of secondary infection.
1684. INR:Normal, APTT:Elevated, Thrombin time:Elevated, Plt count:Normal, Bleeding
time: Normal.
A likely aetiology is?
a. Waldenstrm's
macroglobulinaemia
b. Heparin
c. Szary cell leukaemia
d. Pelger-Het anomaly
e. von Willebrand's disease
f. Haemophilia
g. HIV infection
h. DIC
i. Acanthocytosis
j. Vit K deficiency
Ans: Heparin
PT-test for extrinsic system
INR- 0.9-1.2 (PT Control), Increased INR in warfarin, Vitamin K and liver disease
APTT- intrinsic system, increased PTT (Heparin,Haemophillia (Factor 8 affected))
THrombin Time- 10-15 seconds, increased in heparin, increasedd in DIC
Bledding Time (normal 7 min) - commonest ccause Von willlbrand disease

1685. An infant has diarrhea for 3d with weight loss from 10 kg to 9 kg. Exam: he is
noted to have dry
mucous membranes, poor skin turgor, markedly decreased urine output, and
tachycardia. His
BP=normal and compression-release of the nail beds shows satisfactory refilling.
Appropriate
treatment would include?
a. Plasmapheresis and plasma
infusion
b. 0.5% Normal Saline
c. Lactated Ringer's injection
d. Packed cells
e. Whole blood
f. Platelets
g. FFP
h. double strength Normal Saline
i. 5% dextrose in 0.5N saline

solution
j. IV heparin
ANS: C
1-5% body weight loos Mild dehydration
skin turgor decreassed, mucous membrane dry, urine low, HR raissed, BP
normal,perfusion normal,skin colour pale,irritable
GIVE ORS
5-10% body weight loss Moderate Dehydration
skin turgor decreased,mucous menbrane very dry,urine oligouric,HR raised,BP
normal,Perfusion prolonged capilllary refill,skin colour grey, lethargic,
GIVE ORS 50-100 mL/kg body weight over 2-4 hours,again starting with 5 mL every 5 minutes. if
ors fails then give IV Bolus of 20 ml/kg Isotonic solution.
10-15% body weight loss Severe Dehydration
skin turgor poor,m ucous membrane parched,urine anuric,HR raised, Bp Decreased,
perfusion prolonged , skin colour mottledd, Comatose.
intravenous isotonic fluid boluses (20-60 mL/kg) until perfusion improves.

EXAMPLES OF ISOTONIC SOLUTIONS

0.9% Saline (aka Normal Saline)


Ringers Solution
Lactated Ringers
5% Dextrose in 0.225% Saline
5% Dextrose in Water (technically, Isotonic, but physiology
hypotonic

1686. A 4yo boy has the sudden onset of bone pain. He begins experiencing bleeding of
his gums and
frequent bloody noses. His mother takes him to his pediatrician. Exam: he is pale and
has
numerous petechiae over his body, with lymphadenopathy and hepatosplenomegaly. He
has
WBC=100,000mm and numerous circulating blast cells. He is admitted to the hospital. A
bone
marrow biopsy=35% blast cells. Which of the following is most likely?
a. Mantle cell lymphoma
b. Infectious lymphocytosis
c. Waldenstroms
macroglobulinemia
d. CML
e. CLL
f. Burkitt lymphoma
g. ALL
h. Mycosis fungoides
i. Hairy cell leukemia

j. AML
1687. A 63yo male has anal canal carcinoma with no evidence of spread to the pelvic
wall, pelvic
muscles or lymph nodes. This is typically managed by?
a. Resection of the sigmoid colon
b. Right hemicolectomy
c. Left hemicolectomy
d. Transverse colectomy
e. Internal sphincterotomy
f. CT guided drainage
g. Diverticulectomy
h. Transverse colostomy
i. Chemotherapy and
radiatherapy
j. Abdominal perineal resection
1688. A 2m baby develops a life-threatening anemia. Blood tests show a normal serum
iron, ferritin
and TIBC. Hemoglobin electrophoresis reveals a markedly decreased Hemoglobin A
content and
an increased hemoglobin F content. This baby's anemia is likely to be secondary to?
a. Failure of alpha chain
production
b. Failure of beta chain production
c. Deficiency of B12
d. Lead poisoning
e. IDA
f. Presence of hemoglobin S
g. Presence of hemoglobin M
h. Deficiency of folate
i. Bone marrow failure
j. Inability to manufacture heme
1689. A 30yo caucasian man presented with a 2wk hx of gradually worsening vision in
his left eye. The
patient had been seen once by a neurologist 2yrs prv for flashes. At that time a head CT
was
normal. The patient was lost to follow up with the neurologist, but the flashes had
continued for
the 2yr period. The patient did not experience visual changes with activity or movement.
The
patient reported continued decreasing vision. Goldmann visual fields were done and
showed a
central scotoma. A MRI was done at this time and showed inflammation of the left optic
nerve.
A likely diagnosis is?
a. Pseudotumor
b. Orbital teratoma
c. Optic neuritis
d. Sarcoidosis
e. Optic glioma
f. Lymphangioma
g. Rhabdomyosarcoma

h. Retinal vascular shunts


i. Retinoblastoma
j. Mucormycosis
1690. A pregnant woman in an early stage of labour expresses the wish to have pain
relief during
labour. The anesthetist describes that if the patient wishes he can use medication as a
local
anesthetic to block the pain sensations of labour. Into which space should the local
anaesthetic
be normally injected?
a. Anterior pararenal space
b. Aryepiglottic space
c. Vestibule space
d. Epidural space
e. Sub-arachnoid space
f. Space of Disse
g. Middle ear
h. Posterior pararenal space
i. Supraglottic space j. Lesser sac
1691. A 29yo Afro-Caribbean man presents with a non-productive cough mild aches in
the ankles. The
symptoms have been present for 2m. His ESR is elevated. Ca: 2.69 mmol/l; PO43-: 1.20
mmol/l;
ALP: 80 iu/L. Serum 25(OH) D: 180 nmol/l.
Normal values for Calcium: 2.12-2.65mmol/l; Phosphate: 0.8-1.45mmol/l; ALP 30300iu/L;
Serum 25(OH) D: 20-105nmol/l; Urea: 2.5-6.7mmol/l; Creatinine: 70-120mol/l
a. Osteoporosis
b. Thiazide diuretics
c. Skeletal metastases
d. Primary hyperparathyroidism
e. Hypoparathyroidism
f. Osteomalacia
g. Multiple myeloma
h. Paget's disease of bone
i. Sarcoidosis
j. Hyperthyroidism
1692. A 22yo has had recent chickenpox. He now presents with confusion. He is noted
to have low
urine output and large petechiae all over his body. CXR: a large patch of consolidation is
seen.
The management of choice should be :
a. Ventilatory support
b. Open surgical debridement
c. Resection of superficial
petechiae with wide margin
d. Booster vaccine
e. TENS
f. Lontophoresis
g. Nephrostomy
h. Oral Corticosteroids

i. Brivudin
j. IV acyclovir
1693. A young girl with a psychiatric hx on med tx is brought to the dermatologist by her
mother
because of recurrent patchy hair loss. Exam: the hair shafts revealed twisting and
fractures. This
suggests the following pathology:
a. Infection with Trichophyton
tonsurans
b. Infection with Microsporum
canis
c. Alopecia areata
d. Telogen Effluvium
e. Androgenetic Alopecia
f. Lichen planus
g. Traction Alopecia
h. Alopecia totalis
i. Trichorrhexis nodosa
j. Trichotillomania
1694. Syphilis typically causes
a. Lymphogranuloma Venereum
b. Testicular Torsion
c. Hydrocele
d. Epididymitis
e. Seminoma
f. Mature teratoma
g. Varicocele
h. Lymphoma
i. Orchitis
j. Spermatocele
1695. A middle aged woman has severe collapse of the right femoral head requiring
replacement. The
removed femoral head is sent for pathology and is found to contain enlarged fat cells.
The
pathologist explains that this is the likely cause of the patient's femoral head collapse. A
likely
aetiology is
a. Septic emboli
b. Impaired venous drainage
c. Hgb SS disease
d. Steroid use
e. Alcoholism
f. Gaucher's disease
g. missed fracture
h. Cushing's disease
i. Radiation
j. Vasculitis
1696. A 7yo boy with frequent episodic asthma is on tx with sodium cromoglycate. His
physician wants
to add a non-steroid preventer. The mother of the boy, a teacher, has just read about a
nonsteroidal

medication which acts on the mast cells, stopping them from releasing harmful
chemicals. Her physician agrees to add this medication to the boy's drug regimen. Which
medication is the physician most likely to add to the boy's treatment?
a. Inhaled short acting
bronchodilator
b. SC adrenaline
c. Nedocromil Sodium
d. Inhaled long acting
bronchodilator
e. Inhaled sodium cromoglycate
f. Inhaled steroids
g. Inhaled SABA
h. Oral steroids
i. Nebulised bronchodilators
j. Oral theophylline
key)C
Investigations for asthma
Spirometry:
FEV1:FVC ratio, A low value indicates that you have narrowed airways which are typical in
asthma Therefore, spirometry may be repeated after treatment. An improvement in the value
after treatment with a bronchodilator to open up the airways is typical of asthma.
PEFR:morning readings are usualy lower than evening readings in asthmatics

1697. A 3yo boy is playing with his brother when he falls. He cries immediately and
refuses to walk. His
mother carries him to hospital. He had a full term NVD with no neonatal complications.
His
immunisations are up to date. Exam: looks well and well-nourished, no dysmorphic
features. He
has slight swelling, warmth and discomfort on the lower 1/3 of the left tibia, and refuses
to
weight bear. AP and lateral x rays of the tibia are normal. What is the most likely dx?
a. Ankle fx
b. Ankle sprain
c. Fibular fx

d. Knee dislocation
e. Tibial fx

Key)E
Ans)Toddler's fracture
Undisplaced spiral fractures of the tibial shaft in children under 7 years
old often follow minimal trauma and may not be visible on initial Xray.[8]
Can be difficult to diagnose but should be suspected whenever a child
presents with a limp or fails to bear weight on the leg.
Treatment consists of immobilisation for a few weeks to protect the
limb and to relieve pain.
Subperiosteal bone formation is usually apparent on X-rays by two
weeks.
1698. Which one of the following electrocardiographic changes is found in
hypercalcaemia?
a. Increased QRS interval
b. Prolonged Q-T interval
c. Short P-R interval
d. Short Q-T interval
key)D
Ans)Fact
1699. An elderly male pt with prior hx of hematemesis is having hx of long term use of
aspirin and
other drugs, now presents with severe epigastric pain, dysphagia and vomiting. He was
connected to vital monitors which were not reassuring. What is the management?
a. Oral antacids
b. IV PPI
c. Oral PPI
d. Endoscopy
e. Analgesia
key)D
Ans) long term use of an nsaid predisposes to peptic ulcers. the symptoms described
are of a possibly perforated peptic ulcer or acute upper Gi bleed caused by the ulcer.
The guidelines suggest that
Endoscopy is the primary diagnostic investigation in patients with acute UGIB

Endoscopy should be undertaken immediately after resuscitation for unstable


patients with severe acute UGIB.
Endoscopy should be undertaken within 24 hours of admission for all other
patients with UGIB.

1700. A 68yo man presents with bruising and hx of falls. He is found to have a mask-like
face, pillrolling
tremor and shuffling gait. EEG=normal. Which of the following conditions is he most
likely

being treated for?


a. HTN
b. DM
c. Psychosis
d. TIA
e. Complex partial seizure
key) C
Ans) features are classic for parkinsons disease. and people with PD usualy go on to
develop parkinson related dementia or psychosis.
Symptoms of Parkinsons:
Slowness of movement (bradykinesia). For example, it may become more of
an effort to walk or to get up out of a chair. This is a 'shuffling' walk with some
difficulty in starting, stopping, and turning easily.
Stiffness of muscles (rigidity), and muscles may feel more tense. Also, your
arms do not tend to swing as much when you walk.
Shaking (tremor) is common, but does not always occur. It typically affects the
fingers, thumbs, hands, and arms, but can affect other parts of the body. It is
most noticeable when you are resting. It may become worse when you are
anxious or emotional. It tends to become less when you use your hand to do
something such as picking up an object.
it is diagnosed clinically and treatment includes
1.
2.
3.
4.

levodopa plus a dopa decarboxylase inhibitor


dopamine agonists
anticholinergics (tremor)
MAO inhibitors

Potrebbero piacerti anche